MEGA EMRCS Flashcards
Which of the following drugs increases the rate of gastric emptying in the vagotomised stomach?
A Ondansetron
B Metoclopramide
C Cyclizine
D Erythromycin
E Chloramphenicol
D Erythromycin
Vagotomy seriously compromises gastric emptying which is why either a pyloroplasty or gastro- enterostomy is routinely performed at the same time.
Chloramphenicol has no effect on gastric emptying. Ondansetron slows gastric emptying slightly. Metoclopramide increases the rate of gastric emptying but its effects are mediated via the vagus nerve. Erythromycin enhances gastric emptying by acting via the motilin receptor in the gut.
A 36-year-old man with end-stage renal disease who is undergoing haemodialysis has normocytic normochromic anaemia. Which of the following is the most appropriate therapy?
A. Erythropoietin
B. Ferrous sulphate
C. Folate
D. Vitamin B6
E. Vitamin B12
ANSWER IS A
Erythropoietin, or EPO, is a glycoprotein hormone that is a cytokine for erythrocyte
(red blood cells) precursors in the bone marrow. Also called haematopoietin or haemopoietin, it is produced by the kidney and is the hormone regulating red blood cell production.
Erythropoietin is available as a therapeutic agent produced by recombinant DNA technology in mammalian cell culture. It is used in treating anaemia resulting from chronic renal failure or from cancer chemotherapy. Its use is also believed to be common as a doping agent in endurance sports such as bicycle racing, triathlons and marathon running.
A blood sample taken from the umbilical artery of a newborn was subjected to electrophoresis to detect antibodies (immunoglobulins). Which of the following antibodies will have the highest percentage in a newborn?
A. IgA
B. IgD
C. IgE
D. IgG
E. IgM
ANSWER IS D
IgG is a monomeric immunoglobulin, built of two heavy chains γ and two light chains. Each molecule has two antigen-binding sites.
This is the most abundant immunoglobulin and is approximately equally distributed in blood and in tissue liquids, constituting 75% of serum immunoglobulins in humans. This is the only isotype that can pass through the placenta, thereby providing protection to the newborn in its first weeks of life before its own immune system has developed.
It can bind to many kinds of pathogens, for example viruses, bacteria and fungi, and protects the body against them by complement activation (classic pathway),opsonisation or phagocytosis and neutralization of their toxins. There are four subclasses:
→IgG1 (66%), IgG2 (23%), IgG3 (7%) and IgG4 (4%):
- IgG1, IgG3 and IgG4 cross the placenta easily
- IgG3 is the most effective complement activator, followed by IgG1 and then IgG2
- IgG4 does not activate complement
- IgG1 and IgG3 bind with high affinity to Fc receptors on phagocytic cells
- IgG4 has intermediate affinity and IgG2 affinity is extremely
A 58-year-old male patient needed a blood transfusion after repair of an abdominal aortic aneurysm. His blood was sent to the laboratory. The technician, while checking for this patient’s blood group, said that the patient’s blood agglutinates with antisera anti-A and anti-D, while the patient’s serum agglutinates cells of blood group B. What is the blood group of this patient?
A. A positive
B. B positive
C. A negative
D. B negative
E. O positive
ANSWER IS A
According to the ABO blood typing system there are four different kinds of blood types: A, B, AB or O.
❖Blood groups A - If you belong to the blood group A, you have A antigens on the surface of your red blood cells and B antibodies in your blood plasma.
❖Blood group B - If you belong to the blood group B, you have B antigens on the surface of your red blood cells and A antibodies in your blood plasma.
❖Blood group AB - If you belong to the blood group AB, you have both A and B antigens on the surface of your red blood cells and no A or B antibodies at all in your blood plasma.
❖Blood group O - If you belong to the blood group O, you have neither A nor B antigens on the surface of your red blood cells but you have both A and B antibodies in your blood plasma.
Many people also have a so-called Rh factor on the red blood cell’s surface. This is also an antigen and those who have it are called Rh+. Those who have not are called Rh–. A person with Rh– blood does not have Rh antibodies naturally in the blood plasma (as one can have A or B antibodies, for instance) but they can develop Rh antibodies in the blood plasma if they receive blood from a person with Rh+ blood, whose Rh antigens can trigger the production of Rh antibodies.
A person with Rh+ blood can receive blood from a person with Rh– blood without any problems. So, in this vignette the patient’s blood group is A positive as he has antigen A, antibody B and Rh antigens.
In humans there are five types of antibody: IgA, IgD, IgE, IgG and IgM. Which of the following statements regarding IgM is CORRECT?
A. It binds to allergens
B. It functions mainly as an antigen receptor on B cells
C. It is the largest immunoglobulin molecule
D. It is the most abundant immunoglobulin
E. It is a tetramer of four
ANSWER IS C
IgM forms polymers where multiple immunoglobulins are covalently linked together
with disulphide bonds, normally as a pentamer or occasionally as a hexamer. It has a large molecular mass of approximately 900 kDa (in its pentamer form).
The J chain is attached to most pentamers, while hexamers do not possess the J chain due to space constraints in the complex. Because each monomer has two antigen binding sites, an IgM has 10 of them; however, it cannot bind 10 antigens at the same time because they hinder each other.
Because it is a large molecule, it cannot diffuse well and is found in the interstitium only in very low quantities. IgM is primarily found in serum; however, because of the J chain, it is also important as a secretory immunoglobulin.
Due to its polymeric nature, IgM possesses high avidity and is particularly effective at complement activation. It is sometimes called a ‘natural antibody’, but it is likely that the antibodies arise due to sensitization in the very young to antigens that are naturally occurring in nature. For example anti-A and anti-B IgM antibodies can be formed in early life as a result of exposure to anti-A- and anti-B-like substances that are present on bacteria or perhaps also on plant materials.
In germ-line cells, the gene segment encoding the μ constant region of the heavy chain is positioned first among other constant-region gene segments. For this reason, IgM is the first immunoglobulin expressed by mature B cells.
IgM is also by far the physically largest antibody in the circulation. IgM antibodies are mainly responsible for the clumping (agglutination) of red blood cells if the recipient of a blood transfusion receives blood that is not compatible with his/her blood type. IgM antibodies appear early in the course of an infection and usually do not reappear after further exposure. IgM antibodies do not pass across the human placenta. These two biological properties of IgM make it useful in the diagnosis of infectious diseases. Demonstrating IgM antibodies in a patient’s serum indicates recent infection or, in serum from a neonate, indicates intrauterine infection such as congenital rubella.
The lack of normal factor VIII causes haemophilia A, an inherited bleeding disorder. Factor VIII is synthesized predominantly in:
A. Hepatocytes
B. Histiocytes
C. Kupffer cells
D. Platelets
E. Vascular endothelium
ANSWER IS E
Factor VIII (FVIII) is an essential clotting factor. The lack of normal FVIII causes haemophilia A, an inherited bleeding disorder. The gene for Factor VIII is located on the X chromosome (Xq28). FVIII is a glycoprotein pro-cofactor. Factor VIII is synthesized predominantly in the vascular endothelium and is not affected by liver disease. In fact, levels usually are elevated in such instances. It is also synthesized and released into the bloodstream by the liver.
In the circulating blood, it is mainly bound to von Willebrand factor (vWF, also known as factor VIII-related antigen) to form a stable complex. Upon activation by thrombin or factor Xa, it dissociates from the complex to interact with factor IXa in the coagulation cascade. It is a co-factor to factor IXa in the activation of factor X, which, in turn, with its co-factor factor Va, activates more thrombin. Thrombin cleaves fibrinogen into fibrin, which polymerises and cross links (using factor XIII) into a blood clot. No longer protected by vWF, activated FVIII is proteolytically inactivated in the process (most prominently by activated protein C and factor IXa) and quickly cleared from the bloodstream.
FVIII concentrated from donated blood plasma or alternatively recombinant FVIII can be given to haemophiliacs to restore haemostasis. So, FVIII is also known as antihaemophilic factor. The transfer of a plasma by-product into the bloodstream of a patient with haemophilia often led to the transmission of diseases such as HIV and hepatitis before purification methods were improved. In the early 1990s, pharmaceutical companies began to produce recombinant synthesized factor products, which now prevent nearly all forms of disease transmission during replacement
A 45-year-old woman, with a past history of easy bruising and heavy menstrual periods, was admitted for elective cholecystectomy and was diagnosed with von Willebrand’s disease on routine preoperative investigations. von Willebrand’s disease is:
A. Autosomal dominant
B. Characterized by decreased bleeding time
C. Characterized by decreased factor VII
D. Characterized by decreased platelets
E. X-linked
ANSWER IS A
Von Willebrand’s disease (vWD) is the most common hereditary coagulation abnormality described in humans, although it can also be acquired as a result of other medical conditions. It arises from a qualitative or quantitative deficiency of von Willebrand factor (vWF), a multimeric protein that is required for platelet adhesion. It is known to affect humans and, in veterinary medicine, dogs.
There are three types of hereditary vWD, but other factors such as ABO blood group may also play a part in the cause of the condition. The various types of vWD present with varying degrees of bleeding tendency. Severe internal or joint bleeding is rare (only in type 3 vWD); bruising, nosebleeds, heavy menstrual periods (in women) and blood loss during childbirth (rare) may occur. Death may occur The vWF gene is located on chromosome 12 (12p13.2). It has 52 exons spanning 178 kbp. Types 1 and 2 are inherited as autosomal dominant traits and type 3 is inherited as autosomal recessive.
Occasionally type 2 also inherits recessively. In humans, the incidence of vWD is roughly about 1 in 100 individuals. Because most forms are rather mild, they are detected more often in women, whose bleeding tendency shows during menstruation. The actual abnormality (which does not necessarily lead to disease) occurs in 0.9–3% of the population. It may be more severe or apparent in people with blood group O. Acquired vWD can occur in patients with autoantibodies. In this case the function of vWF is not inhibited but the vWF– antibody complex is rapidly cleared from the circulation. A form of vWD occurs in patients with aortic valve stenosis, leading to gastrointestinal bleeding (Heyde’s syndrome). This form of acquired vWD may be more prevalent than is presently thought.
Acquired vWF has also been described in the following disorders: Wilms’ tumour, hypothyroidism and mesenchymal dysplasias. Patients with vWD normally require no regular treatment, although they are always at increased risk for bleeding. Prophylactic treatment is sometimes given for patients with vWD who are scheduled for surgery. They can be treated with human-derived medium purity factor VIII concentrates. Mild cases of vWD can be trialed on desmopressin (1-desamino-8-D- arginine vasopressin, DDAVP) (antihaemophilic factor, more commonly known as humate-P), which works by raising the patient’s own plasma levels of vWF by inducing release of vWF stored in the Weibel–Palade bodies in the endothelial cells
A 68-year-old woman complaining of easy fatigability and shortness of breath after abdominal aortic aneurysm repair was diagnosed with iron-deficiency anaemia and prescribed an oral iron preparation. Which of the following statements about iron metabolism is CORRECT?
A. Ferritin is a plasma protein that transports iron in the blood
B. Haemosiderin is a product of haemoglobin degradation
C. Iron is more efficiently absorbed in the ferrous state (Fe2+) than in the ferric
state (Fe3+)
D. Most iron in the body is stored as haemosiderin
E. The gastrointestinal rate of iron absorption is extremely high
ANSWER IS C
The absorption of non-haem iron in any food is strongly affected by the composition of the meals. Iron is more efficiently absorbed in the ferrous state (Fe2+) than in the ferric state (Fe3+) and commercial iron preparations often contain vitamin C to prevent oxidation of Fe2+ to Fe3+. Still, only 3–6% of the ingested daily iron is actually absorbed in the upper gastrointestinal tract.
Seventy per cent of the total body iron is used for haemoglobin and myoglobin; the remainder is stored as readily exchangeable ferritin and some is stored in less easily mobilized haemosiderin. When old red blood cells are destroyed by the tissue macrophage system, haem is separated from globin and degraded to biliverdin. Iron in the plasma is bound to the iron transporting protein transferrin. Transferrin level (total iron-binding capacity) and saturation are clinically important indicators of iron deficiency anaemia.
A 45-year-old man on warfarin for a mechanical mitral valve was admitted in the Accident and Emergency Department with persistent bleeding following dental extraction. He was told that his coagulation was deranged. Which of the following statements about blood coagulation is CORRECT?
A. Absence of Ca2+ promotes blood coagulation
B. Disseminated intravascular coagulation (DIC) results in depletion of fibrin
split products
C. Patients with haemophilia A usually have a normal bleeding time
D. von Willebrand factor suppresses platelet adhesion
E. von Willebrand factor suppresses blood coagulation
ANSWER IS C
Prolonged bleeding time is characteristic of platelet disorders, e.g., thrombocytopaenia. Patients with haemophilia A or B (i.e. absence of factor VIII or IX, respectively) have a prolonged partial thromboplastin time (PTT), but do not have a prolonged bleeding time. Ca2+ is a necessary co-factor for blood coagulation, and chelation of Ca2+ ions by citrate inhibits coagulation.
Von Willebrand factor is part of the factor VIII complex and also promotes platelet adherence to the vascular subendothelium. Patients who lack this factor (von Willebrand’s disease) have both a prolonged PTT and a prolonged bleeding time. Disseminated intravascular coagulation results in depletion of coagulation factors and accumulation of fibrin split products.
Nerve gas (organophosphate) is a weapon of chemical warfare that kills by causing respiratory and cardiovascular failure. The expected effect of organophosphate poisoning on the heart would be:
A. Decrease the force of myocardial contractions by potentiating the vagal tone to the ventricular muscle
B. Decrease the rate of rhythmicity of the sinoatrial (SA) node by inducing hyperpolarisation
C. Depolarize cells of the SA node by closing potassium channels under the control of the muscarinic acetylcholine receptor
D. Increase the rate of rhythmicity of the SA node by increasing the upward drift in membrane potential caused by sodium leakage
E. Increase conductivity at the atrioventricular (AV) junction by inducing depolarization
ANNSWER IS B
The toxic effects of nerve gas derive from its ability to inhibit the enzyme cholinesterase. The inhibition of this naturally occurring degradative enzyme engenders a massive accumulation of acetylcholine evoking an overstimulation of the acetylcholine receptors throughout the body. In the heart, specifically, acetylcholine released by the vagal nerve stimulates muscarinic receptors in the cells of the sinoatrial (SA) node.
This results in the opening of potassium channels and hyperpolarisation of the SA node. It therefore takes longer for sodium leakage to cause the membrane potentials of these cells to reach the threshold required for an action potential. The rate of rhythmicity is so decreased. A similar hyperpolarisation of the fibers at the atrioventricular (AV) junction decreases conduction velocity of atria impulses to the ventricle. The force of ventricular contractions is not affected by the vagus nerve.
The resting membrane potential of a neuronal cell body is –60 mV. Opening chloride channels in the neuronal membrane will most likely cause:
A. Depolarization to about –30 mV
B. Depolarization to about +30 mV
C. Hyperpolarisation to about –70 mV
D. Initiation of an action potential
E. No change in membrane potential
ANSWER IS C
Increasing the membrane’s conductance to chloride will result in chloride influx and the membrane potential approaching the value dictated by the chloride equilibrium potential (calculated from the Nernst equation), which is about –70 mV for neurons.
A value of –30 mV is near the Nernst potential for Cl− ions in smooth muscle cells, but not in neurons; +30 mV is near the Nernst potential for Na+ ions. The membrane potential would remain unchanged only if the cell resting membrane potential is already at the Nearest potential of the ion channels that were opened. Action potentials occur if the cell membrane is depolarised above threshold.
Chloride ions are associated with changes in neuronal membrane potential. Which of the following statements most accurately describes the response of a neuron to a decrease in the conductance of the cell membrane to chloride ions?
A. The cell will depolarize if its membrane potential is positive with respect to the equilibrium potential for chloride ions
B. The cell will hyperpolarize if its membrane potential is positive with respect to the equilibrium potential for chloride ions
C. The cell will hyperpolarize if the external chloride concentration is greater than the internal chloride concentration
D. The cell will hyperpolarize if the external chloride concentration is less than the internal chloride concentration
E. No change in membrane potential will occur if the external and internal chloride ion concentrations are equal
ANSWER IS A
Although electrogenic pumps may contribute to the membrane potential of certain cells, the major determinants of membrane potential are the external and internal concentrations of permanent ions and their relative permeabilities in the membrane. Decreasing the conductance causes the membrane potential to move away from the equilibrium potential for that ion. So, a decrease in the conductance of a membrane to chloride ions causes the cells to depolarize – that is, become more positive – if the membrane potential is positive with respect to the chloride equilibrium potential.
Conversely, increasing the conductance for an ion causes the membrane potential to approach the equilibrium potential for that ion. External and internal ion chloride concentrations are needed to calculate the Nernst potential for this ion, but a simple comparison of these two values does not allow predictions about the change in membrane potential.
Which of the following is not secreted by the parietal cells?
A-Hydrochloric acid
B-Mucus
C-Magnesium
D-Intrinsic factor
E-Calcium
Answer is B. Parietal cells: secrete HCl, Ca, Na, Mg and intrinsic factor Chief cells: secrete pepsinogen
Surface mucosal cells: secrete mucus and bicarbonate.
Gastric secretions
A working knowledge of gastric secretions is important for surgery because peptic ulcers are common, surgeons frequently prescribe anti secretory drugs and because there are still patients around who will have undergone acid lowering procedures (Vagotomy) in the past.
Gastric acid
Is produced by the parietal cells in the stomach
pH of gastric acid is around 2 with acidity being maintained by the H /K ATP ase pump. As part of the process bicarbonate ions will be secreted into the surrounding vessels.
Sodium and chloride ions are actively secreted from the parietal cell into the canaliculus. This sets up a negative potential across the membrane and as a result sodium and potassium ions diffuse across into the canaliculus.
Carbonic anhydrase forms carbonic acid which dissociates and the hydrogen ions formed by dissociation leave the cell via the H /K antiporter pump. At the same time sodium ions are actively absorbed. This leaves hydrogen and chloride ions in the canaliculus these mix and are secreted into the lumen of the oxyntic gland.
A 65 year old man is admitted for a below knee amputation. He is taking digoxin. Clinically the patient has an irregularly irregular pulse. What would you expect to see when you examine the jugular venous pressure?
A-Absent y waves
B-Slow y descent
C-Cannon waves
D-Steep y descent
E-Absent a waves
Jugular venous pressure
Absent a waves = Atrial fibrillation
Large a waves = Any cause of right ventricular hypertrophy, tricuspid stenosis Cannon waves (extra large a waves) = Complete heart block
Prominent v waves = Tricuspid regurgitation
Slow y descent = Tricuspid stenosis, right atrial myxoma
Steep y descent = Right ventricular failure, constrictive pericarditis, tricuspid regurgitation
Which part of the ECG represents atrial depolarization?
A-P wave
B-Q wave
C-T wave
D-QRS complex
E-P-R interval
Theme from April 2013 exam
Theme from April 2014 exam
The P wave represents atrial depolarization. Note that atrial repolarization is obscured within the QRS complex.
P wave
Represents the wave of depolarization that spreads from the SA node throughout the atria
Lasts 0.08 to 0.1 seconds (80-100 ms)
The isoelectric period after the P wave represents the time in which the impulse is traveling within the AV node
P-R interval
Time from the onset of the P wave to the beginning of the QRS complex
Ranges from 0.12 to 0.20 seconds in duration
Represents the time between the onset of atrial depolarization and the onset of ventricular depolarization
QRS complex
Represents ventricular depolarization
Duration of the QRS complex is normally 0.06 to 0.1 seconds
ST segment
Isoelectric period following the QRS
Represents period which the entire ventricle is depolarized and roughly corresponds to the plateau phase of the ventricular action potential
T wave
Represents ventricular repolarization and is longer in duration than depolarization
A small positive U wave may follow the T wave which represents the last remnants of ventricular repolarization.
Q-T interval
Represents the time for both ventricular depolarization and repolarization to occur, and therefore roughly estimates the duration of an average ventricular action potential.
Interval ranges from 0.2 to 0.4 seconds depending upon heart rate.
At high heart rates, ventricular action potentials shorten in duration, which decreases the Q-T interval. Therefore the Q-T interval is expressed as a “corrected Q-T (QTc)” by taking the Q- T interval and dividing it by the square root of the R-R interval (interval between ventricular depolarizations). This allows an assessment of the Q-T interval that is independent of heart rate.
Normal corrected Q-Tc interval is less than 0.44 seconds.
A 53 year old man undergoes a reversal of a loop colostomy. He recovers well and is discharged home. He is readmitted 10 days later with symptoms of vomiting and colicky abdominal pain. On examination he has a swelling of the loop colostomy site and it is tender. What is the most likely underlying diagnosis?
A-Haematoma
B-Intra abdominal adhesions
C-Anastomotic leak
D-Anastomotic stricture
E-Obstructed incisional hernia
In this scenario the most likely diagnosis would be obstructed incisional hernia. The tender swelling coupled with symptoms of obstruction point to this diagnosis. Prompt surgical exploration is warranted. Loop colostomy reversals are at high risk of this complication as the operative site is at increased risk of the development of post operative wound infections.
Acute incisional hernia
Any surgical procedure involving entry into a cavity containing viscera may be complicated by post operative hernia
The abdomen is the commonest site
The deep layer of the wound has usually broken down, allowing internal viscera to protrude through
Management is dictated by the patients clinical status and the timing of the hernia in relation to recent surgery
Bowel obstruction or tenderness at the hernia site both mandate early surgical intervention to reduce the risk of bowel necrosis Mature incisional hernias with a wide neck, and no symptoms, may be either left or listed for elective repair
Risk factors for the development of post operative incisional hernias include; post operative wound infections, long term steroid use, obesity and chronic cough
Theme: Abdominal stomas
A. End ileostomy
B. End colostomy
C. Loop ileostomy
D. Loop colostomy E. End jejunostomy F. Loop jejunostomy G. Caecostomy
For each of the following scenarios, please select the most appropriate type of stoma to be constructed. Each option may be selected once, more than once or not at all.
A 56 year old man is undergoing a low anterior resection for carcinoma of the rectum. A primary anastomosis is planned.
Loop ileostomy
Theme from April 2014 Exam
Colonic resections with an anastomosis below the peritoneal reflection may have an anastomotic leak rate (both clinical and radiological) of up to 15%. Therefore most surgeons will defunction such an anastomosis to reduce the clinical severity of an anastomotic leak. A loop ileostomy will achieve this end point and is relatively easy to reverse.
Theme: Abdominal stomas
A. End ileostomy
B. End colostomy
C. Loop ileostomy
D. Loop colostomy E. End jejunostomy F. Loop jejunostomy G. Caecostomy
For each of the following scenarios, please select the most appropriate type of stoma to be constructed. Each option may be selected once, more than once or not at all.
A 23 year old man with uncontrolled ulcerative colitis is undergoing an emergency sub total colectomy.
The correct answer is End ileostomy
Following a sub total colectomy the immediate surgical options include an end ileostomy or ileorectal anastomosis. In the emergency setting an ileorectal anastomosis would be unsafe.
Theme: Abdominal stomas
A. End ileostomy
B. End colostomy
C. Loop ileostomy
D. Loop colostomy E. End jejunostomy F. Loop jejunostomy G. Caecostomy
For each of the following scenarios, please select the most appropriate type of stoma to be constructed. Each option may be selected once, more than once or not at all.
A 63 year old women presents with large bowel obstruction. On examination she has a carcinoma 10cm from the anal verge.
The correct answer is Loop colostomy
Large bowel obstruction resulting from carcinoma should be resected, stented or defunctioned. The first two options typically apply to tumours above the peritoneal reflection. Lower tumours should be defunctioned with a loop colostomy and then formal staging undertaken prior to definitive surgery. An emergency attempted rectal resection carries a high risk of involvement of the circumferential resection margin and is not recommended.
Abdominal stomas
Stomas may be sited during a range of abdominal procedures and involve bringing the lumen or visceral contents onto the skin. In most cases this applies to the bowel. However, other organs or their contents may be diverted in case of need.
With bowel stomas the type method of construction and to a lesser extent the site will be determined by the contents of the bowel. In practice, small bowel stomas should be spouted so that their irritant contents are not in contact with the skin. Colonic stomas do not need to be spouted as their contents are less irritant.
In the ideal situation the site of the stoma should be marked with the patient prior to surgery. Stoma siting is important as it will ultimately influence the ability of the patient to manage their stoma and also reduce the risk of leakage. Leakage of stoma contents and subsequent maceration of the surrounding skin can rapidly progress into a spiraling loss of control of stoma contents.
Types of stomas
Name of stoma Use Common sites
Gastrostomy Gastric decompression or fixation Epigastrium Feeding
Loop jejunostomy
Seldom used as very high output
May be used following emergency laparotomy with planned early closure
Any location according to need
Percutaneous Usually performed for feeding purposes and site in the Usually left upper quadrant jejunostomy proximal bowel
Loop ileostomy
Defunctioning of colon e.g. following rectal cancer surgery
Does not decompress colon (if ileocaecal valve competent)
Usually right iliac fossa
End ilestomy
Usually following complete excision of colon or where ileo- colic anastomosis is not planned
May be used to defunction colon, but reversal is more difficult
Usually right iliac fossa
A 48 year old woman presents to the A&E department with a sudden onset, severe headache peaking within seconds. On examination, she appears anxious but has no focal neurological deficits. A non-contrast CT scan of the head is performed urgently and is found to be normal. There is no family history of aneurysms or other vascular abnormalities. What is the most appropriate next step in the management of this patient?
A. Prescribe subcutaneous sumatriptan
B. Perform lumbar puncture
C. Initiate antiviral therapy
D. Arrange outpatient neurology follow-up without further investigation
E. Request a magnetic resonance imaging (MRI) of the brain
B: In a patient with a sudden onset, severe headache, and normal CT scan, SAH must still be ruled out, as the CT scan may not detect biood if performed very early in the course of SAH. Lumbar puncture is the next step to look for xanthochromia and red blood cells in the cerebrospinal fluid. This will confirm or exclude the diagnosis of SAH Even if you were thinking migraine, you still need to do an LP first because a CT scan was arranged (which means that the clinicians originally thought it was a SAH) So commit to your suspected diagnosis.
A 32 year old woman attends her General Practice due to frequent migraines. She has 1-2 migraines a month which prevent her from going to work. She has a background of asthma controlled with salbutamol and inhaled steroids. She is in a long term relationship and uses condoms for contraception. What is the SINGLE most appropriate medication to offer for migraine prophylaxis?
A. Propranolol
B. Amitriptyline
C. Topiramate
D. Aspirin
E. Sumatriptan
Given the patient’s background and the need for migraine prophylaxis in a woman of childbearing age with a history of asthma, the most appropriate medication to offer would be Amitriptyline.
Explanation:
• Amitriptyline: This tricyclic antidepressant is effective for migraine prophylaxis and is generally safe for women of childbearing age. It is not contraindicated in asthma and does not have the teratogenic risks associated with Topiramate. • Propranolol: Should be avoided due to the patient’s asthma. • Topiramate: Should be avoided in women of childbearing age due to potential teratogenic effects. • Aspirin: Used for acute treatment, not for prophylaxis. • Sumatriptan: Used for acute treatment of migraines, not for prophylaxis.
Correct Answer:
B. Amitriptyline
A 30 year old man presents to the clinic with a 6 day history of nasal congestion, headache, a feeling of pressure behind the eyes, and a fever. The symptoms have been progressively worsening, and the patient reports difficulty sleeping due to the congestion. There is no history of allergies or any other significant past medical history. On examination, there is tenderness over the frontal and maxillary sinuses. What is the most appropriate initial management for this patient?
A. Oral paracetamol
B. Oral amoxicilin
C. Nasal pseudoephedrine
D. Oral cetirizine
E. Nasal fluticasone
A: The patient presents with symptoms suggestive of acute sinusitis and has a fever, which suggests an inflammatory process. According to NICE CKS guidelines, the management of acute sinusitis is primarily focused on symptomatic relief Oral paracetamol can be used to relieve pain, reduce inflammation, and lower the fever. Nasal pseudoephedrine is a decongestant, but there is insufficient evidence to recommend its use, and the same applies to oral cetirizine, an antihistamine Nasal corticosteroids can reduce inlammation, but are not recommended as firstline treatment by NICE CKS Nasal steroids should be considered if the patient has been having symptoms for 10 days or more.
A 77 year old man presents to A&E with chest pain, An ECG demonstrates ST-depression and T-wave inversion in the lateral leads. Serum troponin is 88 mmoll (+5). He is offered Aspirin and Fondaparinux. A decision is made for an urgent angiogram with coronary intervention, where the left circumflex artery requires insertion of a drug-eluting stent. His background includes atrial fibrillation for which takes apixaban. He is told he will require lifelong aspirin treatment together with 12 months of another antiplatelet agent. What is the SINGLE most appropriate antithrombotic medication?
A. Clopidogrel
B. Prasugrel
C. Ticagrelor
D. Bisoprolol
E. Ramipril
A: Clopidogrel should be used as a second antiplatelet medication in the context of anticoagulated patients (warfanin/DOAC). Usually patients will be started on aspinin and ticagrelor post-MI.
A 35 year old man presents to the Emergency Department with a history of ingesting a large number of aspirin tablets about 4 hours ago. He appears restless, and on examination, his respiratory rate is 30 breaths per minute. His arterial blood gas (ABG) shows: pH 7.50 (7.35-7.45) PaO2: 12.7 kPa (10-14) PaCO2: 4 kPa (4.7-6.0) Bicarbonate 22 mmol/L (22-26) The salicylate level is found to be elevated. What is the most likely acid-base disturbance?
A. Metabolic Alkalosis
B. Metabolic Acidosis
C. Respiratory Acidosis
D. Respiratory Alkalosis
E. Mixed metabolic acidosis and respiratory alkalosis
D: The patient has salicylate poisoning, as indicated by the history of aspirin ingestion and the elevated salicylate level Salicylate poisoning initially causes a respiratory alkalosis due to direct stimulation of the respiratory centre in the medulla, leading to hyperventilation. This is reflected in the ABG with a high pH and low PaCO2 Though salicylate poisoning can also lead to a metabolic acidosis, in this case, the bicarbonate level is 22 mmolit, which does not support a concurrent metabolic acidosis Therefore, the primary acid-base disturbance is respiratory alkalosis
A 56 year old man with chronic obstructive pulmonary disease (COPD) is brought into A&E following sudden onset shortness of breath. A chest x-ray confirms a right sided pneumothorax. The chest wall-hilar measurement is 2.4cm. What is the SINGLE most appropriate management?
A. Needle aspiration
B. Cannula decompression
C. Chest drain insertion
D. Supplemental oxygen
E. Pleurodesis
C: Large secondary pneumothorax requires chest drain
A 35 year old man presents to the clinic with concerns about his weight. He mentions he has a sedentary job, working at a desk for the majority of the day. He admits to unhealthy eating habits, often consuming fast foods and sugary drinks. On examination, his BMI is calculated to be 28 kg/m2. Which is the most appropriate initial management step for this patient?
A. Commence anti-obesity medication
B. Refer to a bariatric surgeon
C. Refer to a dietician
D. Order a full blood count
E. Provide lifestyle advice
E- Initial management for someone who is overweight (BMI 25-29.9) is to provide detailed
advice on diet, physical activity, and behavioural changes. Referral to a dietician or
consideration of ather interventions might be appropriate if initial advice does not result in
weight loss or if there are other health issues involved.
A 43 year old man presents to his General Practitioner with episodes of haematuria. He describes the bleeding with associated pain in his left side, but denies any frequency or urgency. He is concerned because his father had kidney problems and died at the age of 55. His blood pressure is 156/94 and urine dip demonstrates blood and protein. Which of the following represents the inheritance pattern of this condition?
A. X-linked dominant
B. Autosomal recessive
C. X-linked recessive
D. Autosomal dominant
E. No genetic link
E: Adult polycystic kidney disease is an autosomal dominant condition.
A 26 year old woman presents to her GP with a 6-month history of effortless regurgitation of recently ingested food within minutes of eating. The regurgitation is not preceded by retching. She has maintained her weight and does not report engaging in any compensatory behaviors such as vomiting, over-exercising, or periods of restrictive eating. The patient denies feelings of guilt after eating or a fear of weight gain. Physical examination is unremarkable, and she has no electrolyte abnormalities. Her symptoms are causing her distress and impairment in social functioning. What is the most likely diagnosis?
A. Anorexia nervosa
B. Bulimia nervosa
C. Atypical eating disorder
D. Rumination syndrome
E. Binge eating disorder
D: This syndrome is characterized by the repeated, effortless regurgitation of recently
ingested food. it is not preceded by retching, which differentiates it from conditions like
bulimia nervosa where vomiting is induced, The food can be re-chewed, re swallowed, or
spit out. Compensatory behaviours are only seen in anorexia and bulimia.
A 24 year old homosexual man presents to the sexual health clinic with penile discharge. He describes the discharge as green and sticky. He also complains of pain in his penis and groin. On examination there is reddening of the penile opening. Microscopy of the penile swab demonstrates the presence of a Gram-negative diplococci. What is the SINGLE most likely causative organism?
A. Trichomoniasis vaginalis
B. Chlamydia trachomatis
C. Bacterial vaginosis
D. Neisseria gonorrhoeae
E. Mycoplasma genitalium
The description of the patient’s symptoms and the findings from the microscopy point towards a specific causative organism.
Key Points:
1. Penile Discharge: Described as green and sticky. 2. Pain: In the penis and groin. 3. Reddening of the Penile Opening: Indicative of inflammation or infection. 4. Microscopy Findings: Presence of Gram-negative diplococci.
Interpretation:
• Gram-negative diplococci: This is a characteristic finding for Neisseria gonorrhoeae.
Diagnosis:
• Neisseria gonorrhoeae is the most likely causative organism given the presentation and microscopy findings.
Correct Answer:
D. Neisseria gonorrhoeae
Explanation:
• Trichomoniasis vaginalis: Typically causes frothy, yellow-green discharge, but is a protozoan and not a Gram-negative diplococci. • Chlamydia trachomatis: Often causes clear or white discharge and is an intracellular bacterium, not seen as Gram-negative diplococci. • Bacterial vaginosis: Primarily affects women and is characterized by a fishy odor and clue cells, not Gram-negative diplococci. • Mycoplasma genitalium: Causes non-gonococcal urethritis, typically clear discharge, and does not appear as Gram-negative diplococci on microscopy.
Therefore, Neisseria gonorrhoeae is the most consistent with the clinical and microscopic findings in this case.
60-year-old woman was found to have one focal area of microcalcification
(approximately 20 mm in diameter) in the left breast. A stereotactic core biopsy
of this area was taken for histological assessment, which revealed low-grade ductal carcinoma in situ. In light of this, what would be the most appropriate treatment modality for this patient?
A Wide local excision + postoperative radiotherapy
B Mastectomy + axillary clearance + postoperative radiotherapy
C Wide local excision + axillary clearance + postoperative radiotherapy
D Mastectomy + postoperative radiotherapy
E Mastectomy
Wide local excision + postoperative radiotherapy
Although mastectomy and mastectomy + postoperative radiotherapy are accepted treatment options for DCIS, wide local excision + postoperative radiotherapy is now frequently offered to women with tumours less than 40 mm in diameter of not an aggressive grade. Wide local excision attempts to conserve the unaffected breast tissue, thus avoiding the need for complete mastectomy, which carries greater psychological trauma to the patient. Mastectomy + postoperative radiotherapy is offered when DCIS is found in multiple areas of the breast, the tumour grade is aggressive, the size of the tumour is greater than 40 mm and if the ostrogen receptor status is negative. Ductal carcinoma in situ implies that the tumour is still confined to the basement membrane and has not broken through the latter. Thus axillary lymph nodes are at a greater chance of being affected once the tumour has broken through the basement membrane. With some exceptions, axillary staging and clearance is performed when multiple foci of extensive and aggressive DCIS are found.
A 47-year-old woman is diagnosed with an unofficial 2.5 cm Grade 3 invasive
ductal carcinoma of the right breast. Following MDT discussion the consultant
sees the patient in clinic to convey management options. Which of the following
would be the most appropriate management plan for this patient?
A Mastectomy and sentinel node biopsy
B Wide local excision
C Mastectomy and axillary clearance
D Wide local excision and sentinel node biopsy
E Wide local excision and axillary clearance
Wide local excision and sentinel node biopsy
The patient has a unifocal 2.5 cm malignant tumour in the right breast. The most appropriate management would be to perform a wide local excision of the breast tumour and it is essential to establish whether axillary disease is present or not. To assess disease positivity/negativity in the axilla, sentinel node biopsy should be performed. If the axillary sentinel lymph node is involved with metastatic tumour deposits originating from the primary breast tumour (e.g. macrometastases; tumour deposits. 2 mm), depending on the level of involvement of the axilla, the patient will undergo axillary clearance. This involves removing level 1-3 axillary lymph nodes to remove residual malignancy.
It is not common practice for a patient diagnosed with breast cancer to undergo axillary clearance if the axilla has not been assessed. In some cases, an axillary lymph node FNA cytology assessment may be performed if there is palpable axillary disease. If the result confirms malignant disease, then the patient will undergo axillary clearance. Otherwise, sentinel node biopsy is usually performed to plan, whether the patient requires axillary clearance.
The main indications for mastectomy are patients who have large tumours (>5 cm) that are unresponsive to chemotherapy, multifocal breast tumours, diffuse DCIS, and a large tumour relative to the patient’s breast tissue
65-year-old nulliparous woman presents to your clinic with a lump in her left breast, which was discovered 7 months ago. On examination you find a hard, ill-defined, non-tender, 3.5 cm lump behind the left nipple. The patient has also had bloody, non-purulent discharge from a single duct on the left nipple for over 3 months. The most likely diagnosis here is
A Periductal mastitis
B Breast carcinoma
C Mammary duct ectasia
D Duct papilloma
E Acute mastitis
Breast carcinoma
The most likely diagnosis here is breast carcinoma. The patient is in her sixth decade with a nulliparous history.
Furthermore, blood-stained nipple discharge coupled with the non-tender breast lump place her at high risk of breast cancer. Milky to dirty-green nipple discharge is usually seen with mammary duct ectasia (common in postmenopausal women) and often, but not always, occurs bilaterally. Acute and periductal mastitis are unlikely here due to the absence of mastalgia. Although duct papillomas are relatively common causes of bloodstained discharge, they do not classically present with a breast lump and other symptoms are rarely present.
Owing to the similar presentation of breast carcinoma, it is difficult to distinguish it from a duct papilloma by clinical examination alone.
A 58-year-old postmenopausal woman has been seen in clinic following discovery of a 3 cm, non-tender, irregular, firm lump in the upper outer quadrant of the left breast. Mammography and ultrasound imaging respectively reveal that the lump has areas of calcification and is a solid mass. The most appropriate course of action is
A Repeat mammography and ultrasound scans in 6 months
B Core biopsy
C Fine needle aspiration to ensure that the lump is not really fluid filled
D Repeat mammography and ultrasound scans in 3 months
E Reassure and discharge
Given the clinical presentation of a 58-year-old postmenopausal woman with a suspicious breast lump, the most appropriate course of action is to perform a biopsy to obtain a tissue diagnosis. The characteristics described (non-tender, irregular, firm lump with calcification on mammography and a solid mass on ultrasound) are concerning for malignancy.
Most appropriate course of action:
B. Core biopsy
Explanation:
• Repeat mammography and ultrasound scans in 6 months (A): Delaying further investigation in a postmenopausal woman with a suspicious breast lump is not appropriate. Prompt evaluation is necessary to rule out malignancy. • Core biopsy (B): A core biopsy provides a tissue sample for histopathological examination, which is crucial for diagnosing the nature of the lump, especially given the concerning features. • Fine needle aspiration (FNA) to ensure that the lump is not really fluid filled (C): FNA can help distinguish between cystic and solid masses, but it does not provide as much tissue as a core biopsy. In this case, core biopsy is preferred for a more definitive diagnosis. • Repeat mammography and ultrasound scans in 3 months (D): Similar to option A, this would delay necessary diagnosis and treatment. Immediate biopsy is more appropriate. • Reassure and discharge (E): Given the suspicious features of the lump, reassurance and discharge without further investigation is not appropriate.
Conclusion:
A core biopsy is the most appropriate next step to definitively diagnose the nature of the breast lump and guide further management.
A 48-year-old perimenopausal woman presents with a 2-month history of a painful lump in her right breast. On examination you find a 2.5 cm tense, fluctuant, mobile lump in the outer lower quadrant of the right breast. The most
appropriate next course of action is
A Mammography and core biopsy
B Core biopsy
C Fine needle aspiration
D Computed tomography scan
E Mammography and ultrasound
Considering the patient’s age and the need for a thorough evaluation to rule out malignancy, the combination of imaging techniques is indeed crucial. In women over 40, breast lumps should be thoroughly evaluated to exclude breast cancer, even if the lump appears cystic.
Most appropriate next course of action:
E. Mammography and ultrasound
Explanation:
• Mammography and core biopsy (A): Core biopsy is invasive and typically follows imaging if needed. • Core biopsy (B): Not the initial step without prior imaging. • Fine needle aspiration (C): While FNA can confirm a cyst and provide symptom relief, it does not provide the comprehensive assessment needed to exclude malignancy. • Computed tomography scan (D): Not typically used for initial evaluation of breast lumps. • Mammography and ultrasound (E): This combination is the best initial approach, especially in women over 40, as it provides detailed information about the nature of the lump. Mammography can identify suspicious calcifications, and ultrasound can differentiate between solid and cystic lesions.
Conclusion:
For a 48-year-old perimenopausal woman with a breast lump, mammography and ultrasound is the most appropriate initial course of action. This approach ensures a comprehensive assessment to rule out malignancy while providing detailed information about the lump’s nature.
During a ward round, you are asked by your surgical registrar about the
management of a phaeochromocytoma. Select from the list below the most
appropriate management plan for a phaeochromocytoma.
A Lifelong Beta and alfa blockade
B Alfa blockade, followed by ß blockade followed by surgical resection
C Surgical resection, followed by B blockade, followed by alfa blockade
D B blockade, followed by alfa blockade, followed by surgical resection
E Surgical resection
Alfa-blockade, followed by B-blockade followed by surgical resection Definitive treatment of a phaeochromocytoma is surgical resection, but before this is performed, pharmacological therapy is required to antagonize the effects of serum circulating catecholamines. a blockade with phenoxybenzamine is usually commenced 7-10 days before surgery, which allows blood volume expansion. Once this is achieved, ß blockade therapy is started. ß blockade cannot be initiated too early as unopposed a receptor stimulation may precipitate a hypertensive crisis. Once the patient has been established on adequate a and ß blockade, surgical resection of the tumour is carried out.
A 36-year-old nulliparous woman attends your clinic with a 7-day history of left breast pain after being involved in a car accident. On examining her breast, you notice a hard, irregular 3 cm, immobile, tender lump. You also notice some skin tethering and overlying bruising in the region of the lump. Ultrasound features suggest a benign pathology. The most likely diagnosis at this point is
A Breast abscess
B Breast cyst
C Mammary duct ectasia
D Fat necrosis
E Fibroadenosis
Fat necrosis
The most likely answer here is fat necrosis. Although the irregularity and hardness of the breast lump with overlying skin tethering suggests breast carcinoma, the history of recent trauma coupled with overlying breast lump tenderness and bruising indicates fat necrosis rather than breast cyst, fibroadenosis and breast abscess. Breast cyst, fibroadenosis and breast abscess are all unlikely here because of the history.
A 38-year-old woman, and mother of two healthy children, is diagnosed with a fluid-filled simple cyst after triple assessment. On ultrasound the inner surface
of the cyst looks entirely smooth. The woman does not have any significant
family history of carcinoma and the cyst is located in the outer-lower quadrant
of the right breast. What would be the most appropriate course of action?
A Follow-up appointment in 3 months
B Wide local excision
C Core biopsy
D Annual follow-up
E Reassure and discharge
Annual follow-up
Once a breast cyst is diagnosed and imaging features are benign, there is usually a follow-up with the patient in 1 year. Wide local excision, follow-up appointment in 3 months, reassure and discharge and core biopsy are incorrect. Benign breast cysts usually produce a strawcoloured aspirate. Once drained, fluid-filled benign cysts usually collapse and the lump disappears. If the aspirate is blood stained or the lump fails to resolve, reevaluation with triple assessment is done within a few weeks.
A 67-year-old woman, with a 25-year smoking history, on hormone replacement therapy, presents to clinic expressing concerns regarding an increase in the size of her right breast over the last 4 months. On examination, you find a non-tender, mobile, lobulated 10 cm mass with relatively smooth edges in the right breast. The right breast is significantly larger than the left and has a ‘teardrop” appearance and the skin looks normal. The most likely diagnosis here is
A Inflammatory breast carcinoma
B Malignant phylloides tumour
C Paget’s disease of the nipple
D Breast abscess
E Fibroadenoma
Phyllodes tumour
The most likely answer here is a malignant phyllodes tumour which accounts for 0.5% of all breast tumours.
The recent increase in size of the right breast coupled with the discovery of a non-tender, mobile lump, giving the right breast a ‘tear drop’ appearance, is suggestive of a malignant phyllodes tumour. Phyllodes tumours have many of the clinical and histological features of fibroadenomas (which are common in women in their early to late twenties). Inflammatory breast carcinoma, as its name suggests, is associated with pain (unlike other breast cancers), breast erythema, peau d’orange (the skin overlying the breast resembles that of red-coloured orange peel) and skin ridging with or without a palpable mass. Paget’s disease of the nipple presents as a unilateral, non-itchy, irregular eczematoid eruption of the nipple. It can be easily confused with eczema of the nipple, which usually occurs bilaterally, with pruritus, and can be nipple sparing. Breast abscess is unlikely here, and for more information, please refer to the answer to Question 2.
A worried 59-year-old city worker arrives at your clinic with a 1-month history of having noticed a non-itchy, persistent, burning rash in the region of her right breast. On examination, you find the right nipple and the skin overlying the
areola to be red and eczematous. The most likely cause is
A Malignant phyllodes tumour
B Basal cell carcinomal
C Breast abscess
D Mastitis
E Paget’s disease of the nipple
next
Paget’s disease of the nipple
The most likely answer here is Paget’s disease of the nipple. The nonitchy, eczematoid changes in the overlying skin of the right nipple and accompanying ymphadenopathy suggests underlying breast malignancy rather than eczema. Paget’s disease of the nipple is almost always associated with an underlying intraductal or invasive carcinoma. Early BCC lesions consist of raised pearly pink papules with fine overlying areas of telangiectasia. Late BCC lesions gradually ulcerate and are often referred to as ‘rodent ulcers’. A malignant phyllodes tumour is unlikely here due to the absence of a lump and increase in size of the breast. Mastitis and breast abscess are also unlikely due to the presenting history and clinical findings (please refer to the answer to Question 2).
A 47-year-old perimenopausal woman presents with a 3-week history of green discharge from the right nipple. On examination, the right nipple is non-tender, has a ‘slit-like’ appearance and is retracted. The most likely diagnosis is
A Breast carcinoma
B Duct papilloma
C Galactorrhoeal
D Fibroadenoma
E Mammary duct ectasia
Mammary duct ectasia This presentation is typical of mammary duct ectasia, which is common in the decade around the menopause. The nipple discharge can vary, ranging from milky, brown to a dirty green colour. The nipple discharge can be bilateral and occasionally it is associated with cyclical mastalgia. The history of nipple retraction can cause confusion, leading the clinician to suspect breast carcinoma. However, the presence of the slit in the nipple, coupled with the colour of the nipple discharge and the perimenopausal age of the patient, make it more likely to be duct ectasia. Galactorrhoea is unlikely here as nipple discharge is usually bilateral and milky in appearance and this usually follows lactation.
Duct papillomas usually present with blood-stained nipple discharge. Fibroadenoma does not usually present with nipple discharge and is common in women under the age of 35.
A 25-year-old woman is diagnosed with a 1.5 cm palpable area of fat necrosis of
the left breast by core biopsy following a traumatic injury 14 days earlier. She
has slight bruising of the lower outer quadrant of the left breast with moderate
tenderness. What would be the most appropriate course of management?
A Left mastectomy
B Wide local excision
C Reassurance and discharge
D Follow-up appointment for ultrasound in 3 months
E
Follow-up appointment for ultrasound in 6 months
A Reassurance and discharge Traumatic fat necrosis of the breast, in most cases, does not warrant any treatment or follow-up and will usually resolve with time. In some cases, fat necrosis can leave a hard, irregular lump (with some overlying skin tethering) which can be mistaken for carcinoma. History of recent trauma to the breasts) makes carcinoma unlikely. Core biopsy can be performed to confirm the diagnosis.
Therefore, follow-up appointment in 3 months, wide local excision, left mastectomy, and follow-up appointment in 6 months are all incorrect.
After a triple assessment, including core biopsy, a 28-year-old woman is diagnosed with a fibroadenoma of the left breast. The patient has a significant
family history of breast carcinoma. The non-tender lump is situated in the inner
lower quadrant of the left breast. The lump is approximately 1.5 cm x 1.5 cm.
What is the most appropriate course of management?
A The patient should be given the choice of excision or not and if not she could be discharged
B Perform triple assessment again in 6 weeks
C Reassure and follow-up after 3 months
D Fine needle aspiration
Excision of the lump
Excision of the lump
After a diagnosis of fibroadenoma is made, small lesions (i.e. <2.5 cm) do not warrant excision. In most cases, patients are reassured and discharged from clinic, but due to the patient’s significant family history of breast carcinoma, it is essential, and reassuring to her, for the lump to be excised. Therefore, excision of the lump is the most appropriate answer in this clinical vignette.
A 45-year-old perimenopausal woman is diagnosed with mammary duct ectasia of the right breast after having had small and infrequent amounts of milky green discharge from multiple ducts of the right nipple for over 2 months. The patient
has no significant family history and mammography findings are normal. What is
the most appropriate course of management?
A Mastectomy of the right breast
B Reassure and discharge
C Perform mammography of the right breast in 3 months
D Commence antibiotic therapy
E Surgical resection of the duct system of the right breast (Hadfield’s operation)
Reassure and discharge Mammary duct ectasia does not warrant treatment providing investigations are normal.
Duct excision is only performed if the discharge is frequent and excessive. Antibiotic therapy is not usually a treatment option for this condition and will not provide a cure. Performing a mammography in 3 months is unlikely to be beneficial. Mastectomy of the right breast is not a recommended treatment modality.
A 30-year-old woman who is 12 days postpartum and breastfeeding is diagnosed with acute mastitis of the left breast. Four days earlier, she discovered a painful crack in the region of the left nipple and noticed that the surrounding skin was tender, warm and red in colour. The patient is not allergic to penicillin and you decide to prescribe a course of antibiotics. What would be the most appropriate antibiotic for treating this condition?
A Flucloxacillin
B Cephalexin
C Amoxicillin
D Erythromycin
E Ciprofloxacin
Flucloxacillin
Flucloxacillin is the antibiotic of choice for treating lactational acute mastitis. The crack in the left nipple allows invasion by skin commensal Staphylococcus aureus, the most common cause of breast infection.
Flucloxacillin is a penicillinase-resistant antibiotic that is very effective against S. aureus infections.
Breastfeeding or milk expression is still encouraged during treatment and has shown to quicken recovery.
Amoxicillin (from the penicillin family), and ciprofloxacin (from the quinolone family) are ineffective against S. aureus. Erythromycin (from the macrolide family) is usually administered to patients who are allergic to penicillin formulas. It is not as effective as penicillinase-resistant antibiotics against S. aureus. Cephalexin (from the cephalosporin family), a moderate-spectrum antibiotic, can be used for S. aureus infections.
However, cephalosporins are not considered a first line antibiotic therapy for the treatment of breast abscesses due to moderate-spectrum activity. In this case, the patient is not allergic to penicillin and thus flucloxacillin is the more appropriate choice of treatment.
In which one of the following scenarios is the complication of lymphoedema of
the arm more likely to occur after resection of a breast tumour and axillary clearance?
A Mastectomy + axillary clearance + systemic chemotherapy
B Mastectomy + axillary clearance + postoperative radiotherapy to the chest wall
C Mastectomy + axillary clearance
D Mastectomy + axillary clearance + postoperative radiotherapy to the axilla
E Mastectomy + postoperative radiotherapy
Mastectomy + axillary clearance + postoperative radiotherapy to the axilla
Lymphoedema of the arm usually occurs following the combination of surgical axillary clearance and postoperative radiotherapy to the axilla. This therapeutic combination is therefore not offered to patients as it is associated with an unacceptably high incidence of lymphoedema. Mastectomy + axillary clearance + postoperative radiotherapy to the chest wall, mastectomy + axillary clearance + systemic chemotherapy, mastectomy + axillary clearance and mastectomy + postoperative radiotherapy are incorrect options where the incidence of lymphoedema is relatively lower compared with mastectomy + axillary clearance + postoperative radiotherapy to the axilla.
A 34-year-old premenopausal woman presents to your clinic with a lump in her
right breast. On examination you find a 2.5 cm fluctuant, mobile, tender lump in
the inner lower quadrant of the right breast. The ultrasound report suggests a
benign fluid-filled cyst. The most appropriate course of action is
A Ultrasound guided core biopsy
B Ultrasound guided fine needle aspiration
C Mammography
D Reassure and discharge
E Breast magnetic resonance imaging
Ultrasound guided fine needle aspiration
Ultrasound assessment of this patient’s breast lump confirms that it is a fluid-filled cyst with benign features.
The next best course of action is therefore to offer the patient fine needle aspiration of the cyst. The extracted fluid is usually discarded. In some cases, if the Radiologist suspects that the cyst has indeterminate features, the aspirated cystic fluid may be sent off for cytological analysis (if but in some cases the fluid may be blood). Breast MRI is used in the assessment of multifocal/ bilateral disease and also in patients with cosmetic implants who are at a predisposed risk of breast cancer. Core biopsy, reassure and discharge and mammography are thus incorrect answers. For further clarification, please refer to the answer to Question 7.
A 25-year-old woman presents to your clinic after discovering, for the first time, two lumps in the inner lower quadrant of her left breast. On examination you find these lumps to be 2 cm in size, solid, discrete, mobile and non-tender. The right breast is normal and there is no lymphadenopathy. The most appropriate
course of management is
A Request a core biopsy
B Request a mammogram
C Reassure the patient and discharge her
D Request fine needle aspiration
E Request an ultrasound of the left breast
Request an ultrasound of the left breast All breast lumps are investigated using ‘triple assessment’ which involves clinical examination, ultrasound breast scans for women younger than 35 (due to relatively denser breast tissue) or mammograms for women older than 35, followed by fine needle aspiration cytology and/or core biopsy for cytological and histological assessment, respectively. Women over the age of 35 tend to have less dense breast tissue, which increases the sensitivity and specificity of mammography.
This patient is 25, which coupled with the discovery of two new lumps, indicates that the next line investigation, following clinical examination, would be an ultrasound of the left breast. Mammography would not be suitable as this patient is under the age of 35.
Reassurance and discharge is clearly wrong as all breast lumps should undergo triple assessment before the patient is discharged. Fine needle aspiration and core biopsy, although they will be carried out, are not considered until radiological assessment has been performed.
You are attending a breast multidisciplinary team (MDT) meeting where the core biopsy histology results of a suspicious breast lesion are being discussed in a 55-year-old woman presenting with a right sided breast lump. The histopathologist states that the breast lesion possesses ‘B5b’ histology features. What is the most likely diagnosis?
A No breast abnormality
B Fibroadenoma
C Invasive breast carcinoma
D Benign breast cyst
E Ductal carcinoma in-situ
Invasive breast carcinoma
Triple assessment is adopted when investigating all breast lumps. Clinical history and examination is followed by imaging and then by cytological or histological analysis.
On palpation of a breast lump, a grade of 1-5 is given based on its clinical characteristics. These are:
• P1 - Normal breast tissue
• P2 - Benign breast tissue
• P3 - Suspicious but probably benign
• P4 - Suspicious but probably malignant
• P5 - Malignant
The same 1-5 grading is used for imaging, but an ‘M’ is used for mammography and ‘U’ for ultrasound.
Cytological grades are given as follows:
• C1 - Normal cell architecture or inadequate sample
• C2 - Benign cytology
• C3 - Suspcious but probably benign
• C4 - Suspicious but probably malignant
• C5 - Malignant
Histology grades for needle core biopsy are
• B1 - Normal breast tissue or inadequate sample
• B2 - Benign breast tissue
• B3 - Suspious but probably benign
• B4 - Suspicious but probably malignant
• B5a - Carcinoma in-situ (e.g. ductal/lobular carcinoma in situ)
• B5b - Invasive carcinoma (e.g. invasive ductal/lobular carcinoma)
With regard to the question, the breast lesion was given a ‘B5b’ histology grade from the needle core biopsy sample. This histology result implies invasive malignancy
A 28-year-old woman, who was hospitalized 2 months ago following a head injury, attends the outpatient clinic with a 6-week history of polyuria and
polydipsia and no other symptoms. Her blood pressure is 117/83 mmHg and her heart rate is 68 beats/min. From the list below, select the most appropriate
management option.
A Desmopressin
B Spironolactone
C Thyroxine
D Octreotide
E Carbimazole
Desmopressin
Symptoms of polyuria and polydipsia are common in many conditions. From the patient’s history, she had suffered a head injury (the triggering factor for her symptoms) and after 2 weeks her symptoms started.
The fact that she has no other symptoms makes it likely that this patient is suffering from diabetes insipidus, where vast amounts of dilute urine are secreted. Two forms exist: cranial diabetes insipidus (causes include head injury, cranial surgery, sarcoidosis), where there is a lack of ADH secretion from the posterior pituitary gland; and nephrogenic diabetes insipidus (causes include hypokalaemia, hypercalcaemia, drugs such as lithium and demecycline, genetic defects and heavy metal poisoning), where there is a lack of response of the kidneys to ADH. Diagnosis can be confirmed using the water deprivation test.
This patient has cranial diabetes insipidus. From the list of options, the most likely answer is treatment with desmopressin (a synthetic analogue of ADH).
Carbimazole is given to patients with hyperthyroidism.
Spironolactone, in the context of endocrine medicine, is given to patients with Conn’s syndrome. Thyroxine is given to hypothyroid patients and octreotide (a somatostatin analogue) can be used in patients with excessive growth hormone production (e.g. acromegaly) and patients with carcinoid syndrome.
A 33-year-old, non-smoking, breastfeeding woman is 10 days postpartum. She has a 4-day history of a slight crack on the surface of her left nipple. She presents with a 2-day history of severe continuous pain in the left breast, spiking pyrexia up to 38.8 with rigours which has prevented her from sleeping.
On examination, you find the outer quadrants of the left breast to be red, warm and tender with a hard 3 cm lump at the edge of the left nipple. The most likely diagnosis is
A Fat necrosis
B Breast cyst
C Breast abscess
D Acute mastitis
E Periductal mastitis
Breast abscess
Lactational breast abscesses are caused by the skin’s commensal microorganisms, nearly always staphylococci, infiltrating cracks in the nipple during breastfeeding. Segmental breast inflammation occurs, leading to cellulitis which, if not treated promptly, results in breast tissue necrosis, pus build up and abscess formation within the breast segment.
Continuous pain and sleepless nights coupled with the hard 3 cm painful lump suggest an abscess rather than acute mastitis. Fat necrosis is unlikely here with no history of previous minor local trauma. Periductal mastitis is possible, but classically occurs in non-lactating women of reproductive age and is associated with smoking. From the nature of the history and physical examination findings, it is unlikely to be a breast cyst.
A 21-year-old nulliparous woman presents to your clinic with a 1-month history of bilateral breast pain. The pain, which is dull and achy in nature, is poorly localized and widespread across both breasts. The pain gradually increases in
severity and is worse just before her menses. The pain usually starts to get
better once her menses start. On examination, both breasts are tender. There are no lumps, skin changes or obvious swellings. The most likely diagnosis here is
A Tietze’s syndrome
B Non-cyclical mastalgia
C Traumatic fat necrosis
D Acute bacterial mastitis
E Cyclical mastalgia
Cyclical mastalgia
The history of bilateral cyclical breast pain, which gradually intensifies before the start of the menses, and which is relieved once the menses start, is highly suggestive of cyclical mastalgia. The absolute cause of this condition is not yet clear, although it is thought to arise due to the sensitivity and responsiveness of breast tissue to hormones. Noncyclical mastalgia does not fit the history as it is not usually associated with the menstrual cycle. Tietze’s syndrome is characterized by tenderness usually over the second, third or fourth costochondral junctions because of chondritis of the costal cartilages. Acute bacterial mastitis is possible, but usually does not present bilaterally and is not associated with the menstrual cycle. The affected breast is usually erythematous, swollen and extremely tender.
Traumatic fat necrosis is clearly wrong as there is no history of recent trauma to the breasts.
A 43-year-old woman presents to your clinic with a 2-month history of localized
dull pain in the right breast. The pain intensifies just before her period. On
examination, you find a discrete 2.5 cm mobile, tense, tender, fluctuant lump in the lower inner quadrant of the right breast. The most likely diagnosis here is
A Breast cyst
B Fibroadenosis
C Fibroadenoma
D Fat necrosis
E Periductal mastitis
Breast cyst
Breast cysts usually occur in women over the age of 40 years through to the menopause. The aetiology in unclear, but it is thought that they occur due to hormonal imbalances around the menopause. Not all breast cysts manifest as pre-menstrual tenderness. The lump is mobile and fluctuant, suggesting that it is a fluid-filled cyst. Fibroadenomas classically present in females below the age of 35 and are infrequent after the age of 35-40. Fibroadenosis presents at a similar age, however the lump is fluctuant, making it more likely to be a cyst. Fat necrosis is unlikely here in the absence of previous minor local trauma (which is not stated in the history). Periductal mastitis is commonly seen in non-lactating women in their thirties (with a history of smoking) with pain usually developing in the areolar area.
A 46-year-old man is diagnosed with an ostrogen receptor positive invasive
ductal carcinoma of the right breast after having discovered a lump 3 months
before. The patient is found to have multiple involved axillary lymph nodes and
the tumour is of an aggressive phenotype. The most appropriate treatment
option for this patient is
A Cytotoxic chemotherapy and Tamoxifen but no surgery
B Palliative care programme
C Mastectomy + postoperative radiotherapy only
D Wide local excision and Tamoxifen only
E Mastectomy + axillary clearance + systemic chemotherapy + radiotherapy and tamoxifen
Mastectomy + axillary clearance + systemic chemotherapy + radiotherapy and Tamoxifen The treatment options for male breast cancer are similar to those for female breast cancer. The most common form of breast cancer in males is invasive ductal carcinoma, which is also the most common type in women. There are four clinical stages of ductal carcinoma and determining the clinical stage allows for the selection of initial treatment. Patients who have stage 1 or 2 (early breast cancer) are more suitable for surgery whereas surgery should be avoided in patients who have stage 3 and 4 breast cancer, that is locally advanced cancer or metastatic spread at presentation, respectively, and they should be treated with chemotherapy. In this case, the patient has stage 2 breast cancer, implying that he is suitable for surgery. In addition, this tumour is ostrogen receptor positive; 90% of male breast cancers are oestrogen receptor positive.
Mastectomy + axillary clearance, followed by systemic chemotherapy, radiotherapy and Tamoxifen, is the most suitable answer out of the five options. Chest wall radiotherapy coupled with systemic chemotherapy carries a better prognosis for the patient. Mastectomy alone would ensure tumour removal, but would not ensure the clearance of micrometastases, and the risk of recurrence/metastases would be higher without chemo/radiotherapy. Wide local excision would be difficult due to the invasive nature of the tumour and ensuring tumour clear margins of tissue. Mastectomy + radiotherapy is possible, but the addition of chemotherapy and hormonal therapy with Tamoxifen carries a better prognosis because of the higher Likelihood of removal of micrometastases. Patients who have stage 4 breast cancer (metastases at presentation) are usually offered palliative care
A worried 23-year-old woman, who started taking the combined contraceptive pill 3 months ago, presents with a 1-day history of discovering a painless lump in the right breast. The patient states that the lump was not there a month ago.
On examination, a slightly mobile, discrete, well-defined, non-tender, firm 1 cm diameter lump is found. There is no lymphadenopathy. The most likely diagnosis here is
A Lipoma
B Breast cyst
C Carcinoma of the breast
D Fibroadenoma
E Sebaceous cyst
Fibroadenoma
Fibroadenomas classically present in females below the age of 35 and are infrequent after the age of 35-40.
They are described as painless, rubbery to firm, non-fluctuant, discrete mobile breast lumps commonly referred to as ‘breast mice’. Breast cysts occur more frequently after the age of 35 and may present with breast pain. They can be fluctuant if fluid filled and characteristically are tense, discrete, mobile lumps.
Carcinoma of the breast is rare under the age of 35, and usually presents as a solitary, painless, ill-defined lump of varying size which may show signs of skin tethering.
Lipomas, although very common, are usually soft, fluctuant, irregularly defined lumps. Sebaceous cysts are typically round, soft lumps attached to the skin and have a central punctum.
Which statement considering the relations of nerves to the humerus is the most accurate?
A Mid-shaft humeral fractures will usually result in complete paralysis of triceps.
B The ulnar nerve is related to the lateral epicondyle.
C Deltoid may atrophy following shoulder dislocation.
D The axillary nerve runs around the anatomical neck.
E The median nerve runs in the spiral groove.
C-Deltoid will atrophy as a result of severe damage to the axillary nerve, which may occur following dislocation of the shoulder. The axillary nerve also innervates teres minor and skin over the lateral side of the proximal part of the arm. Sensory loss in this area suggests nerve damage.
The axillary nerve (C5, C6) winds around the surgical neck of the humerus, accompanying the posterior circumflex humeral vessels. It is prone to damage in humeral neck fractures or dislocation of the shoulder.
The radial nerve runs in the spiral groove, where it is accompanied by the profunda brachii artery.
Mid-shaft humeral fractures may damage the radial nerve as it runs in the spiral groove. The branches to the lateral and long head of triceps arise before the nerve enters the groove, and thus only the medial head is affected. The characteristic sign of this injury is wrist drop due to paralysis of the posterior compartment of the forearm.
The ulnar nerve passes posterior to the medial epicondyle where it is superficial, easily palpable and prone to injury.
Which of the following is not a risk factor for gallstone formation?
A Smoking
B Pregnancy
C Diet high in fats
D Contraceptive pill
E Crohn’s disease
Smoking.
Factors predisposing to gallstone formation include sepsis within the biliary tree, anatomical variants which predispose to stasis and changes in the composition of bile which make it lithogenic. Amirand’s triangle is a diagrammatic representation of how changes in bile composition can predispose to stone formation. Put simply, if the percentage composition of cholesterol increases, or the percentage composition of bile salts or phospholipids (lecithin) decreases, then the bile is more likely to form gallstones.
When taking a focused history and examination, candidates should aim to elicit/exclude the following risk factors for gallstone formation:
• Race (higher incidence in black and Asian populations)
• High-fat diet/history of hypercholesterolaemia
• Obesity
• High ostrogen states (CCP, pregnancy, PCOS; hence the slight female predominance)
• Haemolytic states
The loss of the terminal ileum following surgery or as a consequence of Crohn’s disease also predisposes to gallstone formation as the terminal ileum is the site of re-absorption of bile salts. Smoking is not known to directly influence the risk of gallstone formation.
A 60-year-oid man is found to have a 2 cm diameter mass in the upper lobe of his left lung. The lesion is excised and it is found that the lesion includes connective tissue, mature cartilage and ciliated epithelium. What is the most likely diagnosis?
A Sarcoma
B Hamartoma
C Adenomal
D Chondroma
E Carcinoma
Hamartomas are fundamentally comprised of disordered replications of normal tissue cells. The underlying mechanisms of anomalous replications are not fully recognized. The particular property is a clearly demarcated mass mainly containing fat and cartilage, but other tissue cells may also be present depending on the anatomic location. The size of most hamartomas is between 1 to 3 cm. References
https://www.ncbi.nlm.nih.gov/books/NBK562298/
A 35-year-old woman undergoes gastric bypass surgery for morbid obesity. At a subsequent surgical clinic review she complains of dizziness, sweating, palpitation and collapsing episodes after big meals without any vomiting or pain.
She is otherwise well and has no medical problems. What is the most likely explanation for her symptoms?
A Vasovagal syncope
B Operative denervation of stomach
C Release of insulin
D Release of gastrin
E Release of glucagon
The patient’s symptoms of dizziness, sweating, palpitations, and collapsing episodes after meals, particularly large ones, suggest dumping syndrome, a common complication after gastric bypass surgery. This condition is typically caused by the rapid transit of hyperosmolar food from the stomach into the small intestine, leading to a rapid fluid shift and subsequent release of hormones.
Key Points:
• Symptoms: Dizziness, sweating, palpitations, collapsing episodes after meals. • Context: Post-gastric bypass surgery.
Likely Mechanism:
• Early Dumping Syndrome: Occurs 15-30 minutes after eating, caused by rapid gastric emptying. • Pathophysiology: Rapid entry of hyperosmolar contents into the small intestine leads to fluid shift, triggering autonomic responses and release of gastrointestinal hormones. • Release of Insulin: The rapid absorption of carbohydrates can lead to a quick spike in blood glucose followed by a compensatory release of insulin, which can cause hypoglycemia and associated symptoms like dizziness, sweating, and palpitations.
Explanation:
• C Release of insulin: The rapid transit of food into the small intestine and subsequent hyperglycemia results in a quick release of insulin, which then causes hypoglycemia and the described symptoms.
Answer:
C Release of insulin
A 22-year-old man arrives to the Emergency Department with sudden breathlessness due to a large pneumothorax. A chest drain is inserted into the fifth left intercostal space in 5th mid-axillary line. There is haemorrhage into the drainage bottle. Which of the following structures is the most likely cause of this acute haemorrhage?
A Intercostal artery
B Right ventricle of the heart
C Lingula of the lung
D Spleen
E Left pericardiophrenic artery
A chest drain can cause an intercostal artery bleed during insertion or movement, either by direct trauma or tissue injury to the artery. Improper technique or anatomical variations can increase this risk. Prompt recognition and management are crucial. References
https://www.ncbi.nlm.nih.gov/pmc/articles/PMC861090
4/#:-:text=Complications%20such%20as%20intercost| al% 20artery,procedure%2C%20unlike%20in%20cathet|
er%20removal.
A 2-year-old child is referred because of an impalpable testis in the left
scrotum. Which of the following positions describes an ectopic testis?
A in the upper part of the scrotum
B In the inguinal canal.
C At the superficial inguinal ring
D At the root of the penis
E At the deep inguinal ring
The most common region in descending order: superficial inguinal pouch which lies anterior to the external oblique aponeurosis, perineum / anterior to root of penis, femoral triangle / upper thigh,
contralateral scrotum. References
https://radiopaedia.org/articles/ectopic-testis
An 82-year-old man has complete occlusion of his inferior mesenteric artery on angiography but no symptoms or signs of colonic ischemia. Which of the following arteries is the most likely additional source of blood supply to the
territory of the inferior mesenteric artery?
A Left colic
B Left gastroepiploic
C Superior rectal
D Splenic
E Middle colic
The middle colic artery passes in the layers of the transverse mesocolon to the transverse colon and divides into right and left branches: Right branch supplies the right portion of the transverse colon and anastomoses at the hepatic flexure with the ascending branch of the right colic artery. Left branch supplies the left portion of the transverse colon and anastomoses at the splenic flexure with the left colic artery of the inferior mesenteric artery.
References
https://radiopaedia.org/articles/middle-colic-artery
A 52-year-old man is found to have multiple myeloma. What skull X-ray
appearances would be characteristic?
A Multiple osteolytic lesions
B Hair on end appearance.
C Diffuse thickening of the calvarium
D Multiple fractures
E Multiple osteosclerotic lesions
The raindrop skull appearance of calvarial multiple myeloma is the presence of well defined lytic lesions (punched out lesions) of various sizes scattered throughout the skull.
References
https://radiopaedia.org/cases/multiple-myeloma-skull-
A 30-years-old alcoholic man presents with acute severe upper abdominal pain and vomiting. He is admitted to the intensive therapy unit with a diagnosis of severe acute pancreatitis Forty-eight hours later he develops peripheral paraesthesiae and carpopedal spasms. The most likely underlying metabolic abnormality is:-
A hypercalcaemia
B hypernatraemia
C hyponatraemia
D hypocalcaemia
E hypokalaemia
Pancreatitis can be associated with tetany and hypocalcemia. Ranson Criteria:
Criteria and Prognosis
At Admission
• Age > 55 years
• Leukocyte count > 16 x 103/mcL
• Blood glucose > 200 mg/dL
• Serum LDH > 350 IU/L
• Serum AST > 250 IU/L
At 48 hours
• Decrease in hematocrit > 10%
• Increase in BUN of > 8 mg/dL
• Serum calcium less than 8 mg/dL
• Pa02 < 60 mm Hg
• Base deficit > 4 mEq/L
• Estimated fluid sequestration > 6,000 mL
Score < 3 = Mortality 0-3% • Score 2 3 = Mortality 11-15% • Score ≥ 6 = Mortality 40%
References
https://emedicine.medscape.com/article/241893-
overview#:-:text=Pancreatitis%20can%20be%20assoc iated%20with,secretion%20may%20play%20a%20role
A 26-year-old man presents to the Emergency Department with extensive bleeding from his arm after sustaining a glass injury. On examination there is a 7 cm transverse laceration across the anterior aspect of his elbow. On exploring the cubital fossa you would expect the brachial artery to be:
A medial to the median nerve
B anterior to the median nerve
C lateral to the median nerve
D superficial to the bicipital aponeurosis
E lateral to the biceps tendon
In the cubital fossa, the anatomical relationships of the structures are crucial for surgical exploration and management. The brachial artery has a specific relationship to the median nerve and other structures in this region.
The correct answer is C lateral to the median nerve.
Explanation:
The anatomical arrangement of structures in the cubital fossa from medial to lateral is often remembered by the mnemonic “My Brother Throws Rad Parties,” which stands for:
• Medial nerve • Brachial artery • Tendon of the biceps brachii • Radial nerve (superficial and deep branches)
So, in detail:
• The median nerve lies most medially. • The brachial artery lies just lateral to the median nerve. • The biceps tendon lies lateral to the brachial artery. • The radial nerve is the most lateral structure in the cubital fossa.
Given these anatomical relationships:
• A: Medial to the median nerve – Incorrect. The brachial artery is lateral to the median nerve. • B: Anterior to the median nerve – Incorrect. The brachial artery is not anterior to the median nerve. • C: Lateral to the median nerve – Correct. This matches the anatomical relationship in the cubital fossa. • D: Superficial to the bicipital aponeurosis – Incorrect. The brachial artery is deep to the bicipital aponeurosis. • E: Lateral to the biceps tendon – Incorrect. The brachial artery is medial to the biceps tendon.
A 12-year-old girl has a diastolic murmur. It is maximally audible in the second
left intercostal space. Pathology of which structure is the most likely cause?
A Ductus arteriosus
B Pulmonary valve
C Aortic valve
D Mitral valve
E Tricuspid valve
Second left intercostal space is where pulmonary value is located.
EXPLANATION OF THE AUSCULTATION AREAS
A(Aortic area): The second intercostal space to the right of the sternum.
B(Pulmonic area: The second intercostal space to the left of the sternum.
C(Tricuspid area: The fourth intercostal space to the left of the sternum.
D(Mitral area) : The fifth intercostal space in the left midclavicular line.
A nulliparous 30-year-old Woman presents with a recurrent painful red area in
the areola. She gives a history of smoking cigarettes. The lesion is excised and
the histology shows squamous metaplasia of lactiferous ducts. Microbiological
culture does not grow any organism. Which of the following is the most likely diagnosis?
A Carcinoma of the breast
B Galactorrhea
C Breast abscess
D Fibroadenoma
E Subareolar abscess
A subareolar abscess is a painful collection of pus that forms beneath the areola, usually as a result of an infected milk duct. The presentation often includes localized pain, redness, swelling, and warmth in the affected breast area. Squamous metaplasia of lactiferous ducts can occur in cases of subareolar abscess. Squamous metaplasia is a process where the normal glandular epithelium lining the lactiferous ducts undergoes transformation into squamous epithelium.
This transformation is often associated with chronic inflammation and infection in the breast, such as in the case of a subareolar abscess.
References
https://www.ncbi.nlm.nih.gov/pmc/articles/PMC773308
8/#:-:text=Squamous%20metaplasia%20of%20lactifer
ous%20ducts%20(SMOLD)%20was%20previously%20 known%20as,changes%20to%20the%20breast%20duc
ts.
A 55-year-old man presents with a deep venous thrombosis. His full blood count shows a haemoglobin of 18.3 g/dL (normal 13.5-17.5), a white cell count of 8 x 10^9/L (normal 3.9- 10.0 x 10^9) and a platelet count of 200 x 10^9/L (normal
150-400 x 10^9) This hematological picture is most likely to be associated with:
A bronchial carcinoma
B transitional cell carcinoma of the kidney
C renal cell carcinoma
D prostatic carcinoma
E pancreatic carcinoma
The hematological picture presented, which includes an elevated hemoglobin level (polycythemia), is most likely associated with renal cell carcinoma.
Correct Answer:
C renal cell carcinoma
Explanation:
Renal cell carcinoma (RCC) is known for its potential to produce erythropoietin (EPO), a hormone that stimulates red blood cell production. This can lead to secondary polycythemia, which is characterized by elevated hemoglobin levels as seen in this patient. The other blood parameters (white cell count and platelet count) are within normal ranges, which is consistent with the isolated effect on red blood cell production typically seen in RCC-induced polycythemia.
• A bronchial carcinoma: While bronchial carcinoma can present with various paraneoplastic syndromes, polycythemia is not commonly associated with it. • B transitional cell carcinoma of the kidney: This type of carcinoma does not typically produce EPO or lead to polycythemia. • D prostatic carcinoma: This carcinoma does not commonly cause polycythemia. • E pancreatic carcinoma: Pancreatic carcinoma can be associated with various paraneoplastic syndromes, but polycythemia is not typically one of them.
Given the patient’s elevated hemoglobin level and the known association of RCC with EPO production, renal cell carcinoma is the most likely diagnosis.
A 72-year-old man with a body mass index of 18.4 has not eaten for four days following the removal of an adenocarcinoma from his descending colon. His urea
is found to be 12 mmol/L (normal 3.2-7.5) and creatinine 346 umol/L (normal 35-110). A blood gas profile is ordered. The most likely set of results would be:
A pH 7.36, p02 11.5kPa, pCo2 5.3kPa, Bicarbonate 30.5mmol/L
В рН 7.23, p02 13.6kPa, pCo2 3.8kPa, Bicarbonate 13.5mmol/L
С рН 7.43, p02 16.4kPa, pCo2 3.7kPa, Bicarbonate 21.0mmol/L
D pH 7.49, p02 12.6kPa, pCo2 3.9KPa, Bicarbonate 24.0mmol/L
Е рН 7.30, p02 8.8kPa, pCo2 8.3kPa, Bicarbonate 30.5mmol/L
Starvation ketoacidosis (SKA) represents one of three metabolic acidoses caused by the accumulation of ketone bodies within the bloodstream. While easily treated, it is a diagnosis that can be easily missed in patients with an unexplained metabolic acidosis.
The clinical scenario suggests that the patient is in acute kidney injury (AKI), likely due to dehydration and reduced intake following surgery, leading to elevated urea and creatinine levels. The patient has also not eaten for four days, which can contribute to metabolic derangements.
Given the elevated creatinine, the patient likely has reduced renal function, which often leads to a metabolic acidosis, characterized by a low pH, low bicarbonate, and a compensatory low pCO2 due to hyperventilation.
Correct Answer:
B рН 7.23, pO2 13.6kPa, pCO2 3.8kPa, Bicarbonate 13.5mmol/L
Explanation:
• pH 7.23: Indicates acidosis (pH < 7.35). • pO2 13.6kPa: Oxygen levels can vary but are within a reasonable range. • pCO2 3.8kPa: Low pCO2 indicates respiratory compensation for metabolic acidosis (hyperventilation). • Bicarbonate 13.5mmol/L: Low bicarbonate is consistent with metabolic acidosis.
Other options are less likely because they do not fit the expected acid-base disorder in the context of acute kidney injury and metabolic stress from not eating:
• A: Normal pH with high bicarbonate, suggesting metabolic alkalosis, which does not fit the clinical picture. • C: Normal to slightly elevated pH with low bicarbonate and low pCO2, suggesting respiratory alkalosis, which does not fit the clinical picture. • D: Elevated pH indicating alkalosis, which does not fit with the expected acidosis. • E: Low pH and high bicarbonate with high pCO2, suggesting a mixed disorder but not fitting with the typical response in acute kidney injury and fasting.
An intravenous drug abuser is having an echocardiogram for suspected endocarditis. Closure of the tricuspid valve occurs at the onset of which phase of the cardiac cycle?
A Isovolumetric contraction
B Isovolumetric relaxation
C Rapid ejection
D Atrial systole
E Rapid ventricular filling
The closure of the tricuspid valve occurs at the onset of the isovolumetric contraction phase of the cardiac cycle.
References
https://www.sciencedirect.com/topics/agricultural-and-
biological-sciences/isovolumetric-contraction
The pressure within the pleural space is negative with respect to atmospheric pressure, except for which of the following?
A At end inspiration
B If the patient has a tracheostomy
C During a Valsalva maneuver
D When taking a deep breath
E At end expiration
Initially, during a Valsalva, intrathoracic (intrapleural) pressure becomes very positive because of compression of the thoracic organs by the contracting rib cage.
Manoeuvre:
• Valsalva manoeuvre refers to a forced expiration against a closed glottis.
• This causes marked decrease in the thoracic volume causing deflation of lungs. Under such circumstances the intrapleural pressure can become positive by 60-70 mmHg.
• The common everyday activities in which Valsalva manoeuvre effect
straining
defecation,
phase
coughing,
during during
parturition.
In some of the pathological conditions such as
Pneumothorax
Hydrothorax
Hemothorde
Dothorax
Tends to be more positive.
-In Valsalva Maneuver pleural pressure becomes more positive.
-In reverse Valsalva/Mueller’s Maneuver pleural pressure becomes less positive.
References
https://cvphysiology.com/hemodynamics/h014
A 70-year-old woman from a nursing home presents to the Emergency
Department with abdominal pain and vomiting. On examination, she is
dehydrated and her abdomen is distended. There is a 3 cm - 4 cm swelling in her right groin which is non-tender, and there is no cough impulse. At operation, a femoral Hernia is found. Which of the following lies medial to the neck of the hernia?
A Femoral artery
B Femoral vein
C Femoral nerve
D Pectineal ligament
E Lacunar ligament
The correct answer to the question is:
E. Lacunar ligament
Explanation: In the anatomy of the femoral region, the femoral canal, through which a femoral hernia protrudes, is bounded by the lacunar ligament medially. The lacunar ligament is a crescent-shaped fibrous band that forms the medial boundary of the femoral ring. It is located medial to the femoral vein and, therefore, medial to the neck of a femoral hernia. This anatomical relationship is crucial for understanding the path of femoral hernias and their surgical treatment.
The femoral canal is an anatomical structure located in the femoral region. It is a small compartment within the femoral sheath, and its boundaries are important to understand, especially in the context of femoral hernias. The boundaries of the femoral canal are as follows:
1. Anterior Boundary: Inguinal ligament. 2. Posterior Boundary: Pectineal ligament (also known as the ligament of Cooper), which lies over the pectineus muscle and fascia. 3. Medial Boundary: Lacunar ligament. 4. Lateral Boundary: Femoral vein.
These boundaries form a conical space that allows for the passage of lymphatics and a small amount of fat. The femoral ring, the entrance to the femoral canal, is a potential weak spot where femoral hernias can occur.
48-year-old woman presents with abdominal pain and five to six episodes of
bloody diarrhoea each day. She also complains of pain- in the knees and elbows and recent visual problems. What is the most likely diagnosis?
A intestinal tuberculosis
Ulcerative colitis
C Infective colitis
D Collagenous colitis
E Diverticulitis
The symptoms described — abdominal pain, multiple episodes of bloody diarrhea, joint pain (knees and elbows), and visual problems — are highly suggestive of an inflammatory bowel disease (IBD). The most likely diagnosis among the options provided is:
B. Ulcerative colitis
Explanation:
• Ulcerative colitis is a chronic inflammatory bowel disease that typically presents with bloody diarrhea, abdominal pain, and can also have extra-intestinal manifestations such as arthritis (pain in joints) and uveitis or episcleritis (visual problems). • Intestinal tuberculosis usually presents with chronic abdominal pain, weight loss, and sometimes diarrhea, but bloody diarrhea is less typical and extra-intestinal symptoms such as joint pain and visual problems are less common. • Infective colitis can cause bloody diarrhea, but the chronic joint and visual symptoms are less characteristic. • Collagenous colitis typically presents with chronic watery diarrhea and does not usually cause bloody diarrhea or extra-intestinal symptoms like joint pain and visual issues. • Diverticulitis typically presents with localized abdominal pain, often in the left lower quadrant, and does not usually cause bloody diarrhea or extra-intestinal symptoms.
Therefore, the combination of gastrointestinal and extra-intestinal symptoms in this patient most strongly suggests ulcerative colitis.
A 65-year old woman collapses after a total hip replacement. A pulmonary embolism is suspected. Which of the following electrocardiogram changes would support this diagnosis?
A Dominant R wave in V6
B T wave inversion in V1-V3
C Left axis deviation
D ST elevation in V1-V3
E Left bundle branch block
The electrocardiogram (ECG) change that would support a diagnosis of pulmonary embolism is:
B. T wave inversion in V1-V3
Explanation:
• Pulmonary embolism often causes right ventricular strain, which can be reflected on an ECG. One of the common ECG changes seen in pulmonary embolism is T wave inversion in the right precordial leads (V1-V3). • Dominant R wave in V6 is not typically associated with pulmonary embolism. • Left axis deviation is more commonly associated with left-sided heart conditions and is not a typical finding in pulmonary embolism. • ST elevation in V1-V3 could indicate an acute myocardial infarction (MI) but is not typical for pulmonary embolism. • Left bundle branch block (LBBB) indicates a delay in electrical conduction in the left bundle branch and is not specific to pulmonary embolism.
Therefore, T wave inversion in V1-V3 is the most relevant ECG finding that would support the suspicion of a pulmonary embolism.
An 80-year-old man dislocates his shoulder it is reduced in the Emergency Department. At review three weeks later, he is unable to actively initiate abduction of his arm. Which muscle most likely damaged?
A Teres major
B Clavicular head oi pectoralis major
C Infraspinatus
D Trapezius
E Supraspinatus
The muscle most likely damaged in this scenario is:
E. Supraspinatus
Explanation:
• The supraspinatus muscle is part of the rotator cuff and is primarily responsible for initiating the abduction of the arm. If the patient is unable to actively initiate abduction, it suggests damage to this muscle. • Teres major is involved in the internal rotation and adduction of the arm, not abduction. • The clavicular head of pectoralis major assists in flexion, adduction, and internal rotation of the arm. • Infraspinatus is another rotator cuff muscle, but it is primarily responsible for external rotation of the arm, not abduction. • The trapezius muscle is involved in moving, rotating, and stabilizing the scapula, and it assists in shoulder elevation but is not responsible for initiating abduction.
Given the specific inability to initiate abduction, damage to the supraspinatus muscle is the most likely cause.
Consider the dermatomes of the upper limb. Which of the following statements best completes this sentence? The skin over the:
A Posterior surface of the forearm derives its cutaneous innervation from T1.
B Clavicle derives its cutaneous innervation from C4.
C Thumb derives its cutaneous innervation from T1.
D Little finger derives its cutaneous innervation from C7.
E Lateral aspect of the upper limb derives its cutaneous innervation from C8 and T1.
The correct statement is:
B. Clavicle derives its cutaneous innervation from C4.
Explanation:
• A. Posterior surface of the forearm derives its cutaneous innervation from T1: This is incorrect. The posterior surface of the forearm is typically innervated by C6 and C7 dermatomes. • B. Clavicle derives its cutaneous innervation from C4: This is correct. The skin over the clavicle is typically innervated by the C4 dermatome. • C. Thumb derives its cutaneous innervation from T1: This is incorrect. The thumb is innervated by the C6 dermatome. • D. Little finger derives its cutaneous innervation from C7: This is incorrect. The little finger is innervated by the C8 dermatome. • E. Lateral aspect of the upper limb derives its cutaneous innervation from C8 and T1: This is incorrect. The lateral aspect of the upper limb is primarily innervated by C5 and C6 dermatomes.
Therefore, the correct answer is that the skin over the clavicle derives its cutaneous innervation from C4.
Select the most appropriate ending for the following sentence. The median nerve:
A Supplies all of flexor digitorum profundus.
B Arises from both the posterior and medial cords of the brachial plexus.
C Gives no muscular branches in the arm.
D Enters the forearm by passing between the two heads of flexor carpi ulnaris.
E Passes from the medial to the lateral side of the brachial artery in the arm.
The most appropriate ending for the sentence is:
C. Gives no muscular branches in the arm.
Explanation:
• A. Supplies all of flexor digitorum profundus: This is incorrect. The median nerve supplies only the lateral half (index and middle fingers) of the flexor digitorum profundus, while the ulnar nerve supplies the medial half (ring and little fingers). • B. Arises from both the posterior and medial cords of the brachial plexus: This is incorrect. The median nerve arises from both the lateral and medial cords of the brachial plexus. • C. Gives no muscular branches in the arm: This is correct. The median nerve does not supply any muscles in the arm; its motor branches are all in the forearm and hand. • D. Enters the forearm by passing between the two heads of flexor carpi ulnaris: This is incorrect. The median nerve enters the forearm by passing between the two heads of the pronator teres, not the flexor carpi ulnaris. • E. Passes from the medial to the lateral side of the brachial artery in the arm: This is incorrect. The median nerve passes from the lateral to the medial side of the brachial artery as it descends in the arm.
Thus, the most accurate statement is that the median nerve gives no muscular branches in the arm.
Which of the following statements about the axillary artery is accurate?
A The axillary vein lies medial throughout its course.
B It is divided into three parts by pectoralis major.
C It becomes the brachial artery at the inferior border of teres minor.
D The cords of the brachial plexus are named according to their positions relative to the first part.
E It gives the suprascapular artery as one of its branches.
The accurate statement about the axillary artery is:
A. The axillary vein lies medial throughout its course.
Explanation:
• A. The axillary vein lies medial throughout its course: This is correct. The axillary vein lies medial to the axillary artery throughout its course in the axilla. • B. It is divided into three parts by pectoralis major: This is incorrect. The axillary artery is divided into three parts by the pectoralis minor muscle, not pectoralis major. • C. It becomes the brachial artery at the inferior border of teres minor: This is incorrect. The axillary artery becomes the brachial artery at the inferior border of the teres major muscle, not teres minor. • D. The cords of the brachial plexus are named according to their positions relative to the first part: This is incorrect. The cords of the brachial plexus are named according to their positions relative to the second part of the axillary artery. • E. It gives the suprascapular artery as one of its branches: This is incorrect. The suprascapular artery is a branch of the thyrocervical trunk, which is a branch of the subclavian artery, not the axillary artery.
Therefore, the most accurate statement is that the axillary vein lies medial throughout its course.
Which of the following statements regarding the radius and ulna is correct?
A Both articulate with the humerus at the elbow joint.
B Both have a styloid process at the proximal end.
C Fracture is most commonly of the Smith’s type.
D Direct injury usually produces transverse fractures of both bones in the distal third.
E Both articulate with the carpal bones at the wrist joint.
To determine which statement regarding the radius and ulna is correct, we will analyze each option step by step.
A. Both articulate with the humerus at the elbow joint.
The radius and ulna are both forearm bones that do indeed articulate with the humerus at the elbow joint. The ulna primarily forms the hinge joint with the humerus, while the radius also plays a role in this articulation, especially in terms of rotation during pronation and supination of the forearm. Therefore, this statement is correct.
B. Both have a styloid process at the proximal end.
The radius has a styloid process at its distal end, while the ulna has a styloid process at its distal end as well but not at its proximal end. This means that this statement is incorrect because only one of them has a styloid process at the proximal end.
C. Fracture is most commonly of the Smith’s type.
A Smith’s fracture refers to a fracture of the distal radius with volar angulation (the palm side). While fractures of the radius are common, particularly in older adults due to falls, they are more often Colles’ fractures rather than Smith’s fractures. Therefore, this statement is incorrect.
D. Direct injury usually produces transverse fractures of both bones in the distal third.
Direct injuries can lead to various types of fractures depending on the mechanism involved; however, transverse fractures specifically affecting both bones in their distal third are not typically described as common outcomes from direct trauma. This statement is therefore considered incorrect.
E. Both articulate with the carpal bones at the wrist joint.
The radius articulates directly with several carpal bones (specifically scaphoid and lunate) at the wrist joint, while the ulna does not have a direct articulation with any carpal bones; instead, it is separated from them by an articular disc (the triangular fibrocartilage complex). Thus, this statement is incorrect.
After analyzing all statements:
The only correct statement regarding both radius and ulna is:
A Both articulate with the humerus at the elbow joint.
Which of the following best describes the forearm?
A Pronator quadratus lies deep to the long flexor tendons just proximal to the wrist.
B The median nerve is related anteriorly to flexor digitorum superficialis.
C Flexor digitorum profundus is partially supplied by the posterior interosseous nerve.
D The ulnar nerve lies superficial to flexor carpi ulnaris.
E The radial nerve carries motor fibres in the anterior compartment of the forearm.
Let’s analyze each option to determine which best describes the forearm:
A. Pronator quadratus lies deep to the long flexor tendons just proximal to the wrist.
• True. Pronator quadratus is located deep to the flexor digitorum superficialis and profundus tendons near the wrist.
B. The median nerve is related anteriorly to flexor digitorum superficialis.
• False. The median nerve runs between the flexor digitorum superficialis and the flexor digitorum profundus, not anteriorly to flexor digitorum superficialis.
C. Flexor digitorum profundus is partially supplied by the posterior interosseous nerve.
• False. Flexor digitorum profundus is innervated by the median nerve (anterior interosseous branch) for the lateral part and the ulnar nerve for the medial part.
D. The ulnar nerve lies superficial to flexor carpi ulnaris.
• False. The ulnar nerve actually lies deep to flexor carpi ulnaris.
E. The radial nerve carries motor fibres in the anterior compartment of the forearm.
• False. The radial nerve supplies the muscles in the posterior compartment of the forearm.
Based on the analysis, the correct answer is:
A. Pronator quadratus lies deep to the long flexor tendons just proximal to the wrist.
Explanation: Pronator quadratus is a deep muscle in the forearm located just proximal to the wrist, lying beneath the tendons of the flexor digitorum profundus and superficialis, aligning with the description in the correct option.
Which of the following statements ends this sentence correctly? The spinal nerve root mediating the:
A Pectoral reflex is C5.
B Deltoid reflex is C4.
C Biceps jerk is C5.
D Triceps jerk is C6.
E Supinator jerk is C7.
C
The biceps jerk principally tests the C5 reflex arc, with a contribution from C6.
The supinator jerk corresponds primarily to the C6 reflex arc, with a contribution from C5.
The triceps jerk is principally mediated by C7, with a contribution from C8.
This reflex may be of occasional use, and is elicited by placing the examiner’s index finger across the tip of the shoulder on the deltoid muscle belly, and tapping the finger. It solely tests the reflex arc of spinal cord segment C5.
The pectoral reflex is elicited by placing the examiner’s index and middle fingers on the lateral border of pectoralis major and tapping them with a tendon hammer. It tests the reflex arc of C7.
Which of the following accurately describes the anterior compartment of the forearm?
A Palmaris longus is present in approximately 15% of arms.
B The median nerve enters the forearm between the two heads of flexor carpi ulnaris.
C Flexor digitorum profundus originates from the common flexor origin.
D The superficial flexor muscles are all innervated by the median nerve
E Flexor pollicis longus is characteristically unipennate.
E
Flexor pollicis longus is one of the deep group of flexor muscles. Its principle origin is from the radius. It is characteristically unipennate, which aids its identification during forearm surgery.
The median nerve enters the forearm between the two heads of pronator teres. The ulnar nerve enters the forearm by passing between the two heads of flexor carpi ulnaris.
Palmaris longus is a vestigial structure, absent in 13% of arms.
Flexor digitorum profundus is one of the deep group of flexor muscles and principally takes origin from the ulna.
The superficial group of flexor muscles are pronator teres, flexor carpi radialis, palmaris longus, flexor digitorum superficialis and flexor carpi ulnaris. The median nerve innervates all these muscles except flexor carpi ulnaris, which is innervated by the ulnar nerve.
Which of these statements best describes the carpus?
A The pisiform is usually the first bone to begin ossification.
B The most commonly fractured bone is the lunate.
C Fracture of the hamate may result in damage to the median nerve.
D It is markedly concave from side to side anteriorly.
E The scaphoid articulates with the 1st metacarpal.
D
The 8 carpal bones, arranged in 2 rows of 4, form the carpus. The carpus is convex posteriorly and concave anteriorly from side to side. This shape is maintained by the shape of the bones and by the pull of the flexor retinaculum.
The most commonly fractured bone is the scaphoid.
This occurs most commonly as a result of a fall onto the palm, with the hand extended and abducted. The principal blood supply of the scaphoid enters from its distal end. A fracture may therefore interrupt the blood supply to the proximal fragment, leading to avascular necrosis and consequent degenerative change.
The scaphoid is one of the four proximal bones of the wrist and articulates with the radius proximally. The trapezium is the carpal bone which articulates with the 1st metacarpal.
The deep branch of the ulnar nerve is closely related to the hook of the hamate, and is threatened by a hamate fracture. Decreased grip strength usually ensues.
The pisiform bone is a small, pea-shaped, sesamoid bone in the tendon of the flexor carpi ulnaris. Its ossification centre usually does not appear until the age of 9-12 years. It is the last carpal bone to begin ossification.
The flexor tendons of the forearm run within sheaths in the hand. Which of the following statements best describes these structures?
A The flexor digitorum profundus tendon pierces the flexor digitorum superficialis tendon.
B The cruciform pullies overlie the phalanges.
C The fibrous flexor sheaths run from the metacarpal heads to the proximal phalanges.
D The annular pullies overlie the interphalangeal joints.
E The synovial sheath of the middle finger tendon is continuous with its sheath in the carpal tunnel.
A
The flexor digitorum superficialis tendon splits to enclose the profundus tendon as it inserts into the middle phalanx of the finger. The profundus tendon continues to insert into the base of the distal phalanx.
The fibrous flexor sheaths run from the metacarpal heads to the distal phalanges. They are occupied by the tendons of flexors digitorum superficialis and profundus in the fingers and the tendon of flexor pollicis longus in the thumb.
The annular pullies overlie the phalanges and consist of strong transverse sheath fibres.
In the index, middle and ring fingers there is a gap between the common flexor sheath in the carpal tunnel and the synovial sheaths of the fingers. The sheaths of the thumb and little finger are continuous with those of the carpal tunnel.
The cruciform pullies overlie the interphalangeal joints and consist of loose fibres which are arranged obliquely.
Which of the following types of lymphedema is congenital?
A-Meiges’ disease
B-Milroy’s disease
C-Lymphoedema tarda
D-Lymphoedema following surgery
E-Lymphoedema due to venous insufficiency
Milroy’s disease is congenital. Meiges’ develops after birth.
Lymphoedema
- Due to impaired lymphatic drainage in the presence of normal capillary function.
Lymphoedema causes the accumulation of protein rich fluid, subdermal fibrosis and dermal thickening.
Characteristically fluid is confined to the epifascial space (skin and subcutaneous tissues); muscle compartments are free of oedema. It involves the foot, unlike other forms of oedema. There may be a ‘buffalo hump’ on the dorsum of the foot and the skin cannot be pinched due to subcutaneous fibrosis.
Causes of lymphoedema
Primary
Congenital < 1 year: sporadic, Milroy’s disease
Onset 1-35 years: sporadic, Meige’s disease
> 35 years: Tarda
Secondary
Bacterial/fungal/parasitic infection (filariasis)
Lymphatic malignancy
Radiotherapy to lymph nodes
Surgical resection of lymph nodes
DVT
Thrombophlebitis
Indications for surgery
Marked disability or deformity from limb swelling
Lymphoedema caused by proximal lymphatic obstruction with patent distal lymphatics suitable for a lymphatic drainage procedure
Lymphocutaneous fistulae and megalymphatics
Procedures
Homans operation Reduction procedure with preservation of overlying skin (which must be in good condition). Skin flaps are raised and the underlying tissue excised. Limb circumference typically reduced by a third.
Charles operation All skin and subcutaneous tissue around the calf are excised down to the deep fascia. Split skin grafts are placed over the site. May be performed if overlying skin is not in good condition. Larger reduction in size than with Homans procedure.
Lymphovenous anastamosis Identifiable lymphatics are anastomosed to sub dermal venules. Usually indicated in 2% of patients with proximal lymphatic obstruction and normal distal lymphatics.
Which of the following statements relating to pre-operative fluid management is false?
A-5% dextrose should be given cautiously in the elderly
B-Patients undergoing elective colonic resections may continue to drink water up to 2 hours prior to surgery
C-Normal saline increases the risk of hyperchloraemic acidosis
D-A 70kg man will need approximately 100mmol of sodium daily
E-Carbohydrate rich beverages and loading drinks can cause ileus therefore should be avoided
Carbohydrate loading is one of the enhanced recovery principles.
Pre operative fluid management
Fluid management has been described in the British Consensus guidelines on IV fluid therapy for Adult Surgical patients (GIFTASUP) and by NICE (CG174 December 2013 updated May 2017)
The Recommendations include:
Use Ringer’s lactate or Hartmann’s when a crystalloid is needed for resuscitation or replacement of fluids. Avoid 0.9% N. Saline (due to risk of hyperchloraemic acidosis) unless patient vomiting or has gastric drainage.
Use 4%/0.18% dextrose saline or 5% dextrose in maintenance fluids. It should not be used in resuscitation or as replacement fluids.
Adult maintenance fluid requirements are: Na 50-100 mmol/day and K 40-80 mmol/day in 1.5-2.5L fluid per day.
Patients for elective surgery should NOT be nil by mouth for >2 hours (unless has disorder of gastric emptying).
Patients for elective surgery should be given carbohydrate rich drinks 2-3h before. Ideally this should form part of a normal pre op plan to facilitate recovery.
Avoid mechanical bowel preparation.
If bowel prep is used, simultaneous administration of Hartmann’s or Ringer’s lactate should be considered.
Excessive fluid losses from vomiting should be treated with a crystalloid with potassium replacement. 0.9% N. Saline should be given if there is hypochloraemia. Otherwise Hartmann’s or Ringer lactate should be given for diarrhoea/ileostomy/ileus/obstruction. Hartmann’s should also be given in sodium losses secondary to diuretics.
High risk patients should receive fluids and inotropes.
An attempt should be made to detect pre or operative hypovolaemia using flow based measurements. If this is not available, then clinical evaluation is needed i.e. JVP, pulse volume etc.
In Blood loss or infection causing hypovolaemia should be treated with a balanced crystalloid or colloid (or until blood available in blood loss). A critically ill patient is unable to excrete Na or H20 leading to a 5% risk of interstitial oedema. Therefore 5% dextrose as well as colloid should be given.
If patients need IV fluid resuscitation, use crystalloids that contain sodium in the range 130-154 mmol/l, with a bolus of 500 ml over less than 15 minutes (NICE Guidance CG 174).
Where does the spinal cord terminate in neonates?
A-L1
B-L2
C-L3
D-L4
E-L5
At the 3rd month the foetus’s spinal cord occupies the entire length of the vertebral canal. The vertebral column then grows longer exceeding the growth rate of the spinal cord. This results with the cord being at L3 at birth and L1-2 by adulthood.
Spinal cord
- Located in a canal within the vertebral column that affords it structural support.
Rostrally it continues to the medulla oblongata of the brain and caudally it tapers at a level corresponding to the L1-2 interspace (in the adult), a central structure, the filum terminale anchors the cord to the first coccygeal vertebra.
The spinal cord is characterised by cervico-lumbar enlargements and these, broadly speaking, are the sites which correspond to the brachial and lumbar plexuses respectively.
There are some key points to note when considering the surgical anatomy of the spinal cord:
- During foetal growth the spinal cord becomes shorter than the spinal canal, hence the adult site of cord termination at the L1-2 level.
- Due to growth of the vertebral column the spine segmental levels may not always correspond to bony landmarks as they do in the cervical spine.
- The spinal cord is incompletely divided into two symmetrical halves by a dorsal median sulcus and ventral median fissure. Grey matter surrounds a central canal that is continuous rostrally with the ventricular system of the CNS.
- The grey matter is sub divided cytoarchitecturally into Rexeds laminae.
- Afferent fibres entering through the dorsal roots usually terminate near their point of entry but may travel for varying distances in Lissauers tract. In this way they may establish synaptic connections over several levels
- At the tip of the dorsal horn are afferents associated with nociceptive stimuli. The ventral horn contains neurones that innervate skeletal muscle.
The key point to remember when revising CNS anatomy is to keep a clinical perspective in mind. So it is worth classifying the ways in which the spinal cord may become injured. These include:
Trauma either direct or as a result of disc protrusion
Neoplasia either by direct invasion (rare) or as a result of pathological vertebral fracture
Inflammatory diseases such as Rheumatoid disease, or OA (formation of osteophytes compressing nerve roots etc.
Vascular either as a result of stroke (rare in cord) or as complication of aortic dissection
Infection historically diseases such as TB, epidural abscesses.
The anatomy of the cord will, to an extent dictate the clinical presentation. Some points/ conditions to remember:
Brown- Sequard syndrome-Hemisection of the cord producing ipsilateral loss of proprioception and upper motor neurone signs, plus contralateral loss of pain and temperature sensation. The explanation of this is that the fibres decussate at different levels.
Lesions below L1 will tend to present with lower motor neurone signs
A 25 year old junior doctor has a chest x-ray performed as part of a routine insurance medical examination. The x-ray shows evidence of rib notching. Auscultation of his chest reveals a systolic murmur which is loudest at the posterior aspect of the fourth intercostal space. What is the most likely diagnosis?
A-Patent ductus arteriosus
B-Aortic coarctation
C-Aortic dissection
D-Cervical rib
E-Subclavian steal syndrome
Coarctation of the aorta may occur due to the remnant of the ductus arteriosus acting as a fibrous constrictive band of the aorta. Weak arm pulses may be seen, radiofemoral delay is the classical physical finding. Collateral flow through the intercostal vessels may produce notching of the ribs, if the disease is long standing.
Vascular disease
Aortic dissection
Chest pain (anterior chest pain- ascending aorta, back pain - descending aorta)
Widening of aorta on chest x-ray
Diagnosis made by CT scanning
Treatment is either medical (Type B disease) or surgical (Type A disease)
Cervical rib
Supernumery fibrous band arising from seventh cervical vertebra
Incidence of 1 in 500
May cause thoracic outlet syndrome
Treatment involves surgical division of rib
Subclavian steal syndrome
Due to proximal stenotic lesion of the subclavian artery
Results in retrograte flow through vertebral or internal thoracic arteries
The result is that decrease in cerebral blood flow may occur and produce syncopal symptoms
A duplex scan and/ or angiogram will delineate the lesion and allow treatment to be planned
Takayasu’s arteritis
Large vessel granulomatous vasculitis
Results in intimal narrowing
Most commonly affects young asian females
Patients present with features of mild systemic illness, followed by pulseless phase with symptoms of vascular insufficiency
Treatment is with systemic steroids
Patent ductus arteriosus
Ductus arteriosus is a normal foetal vessel that closes spontaneously after birth
Results in high pressure, oxygenated blood entering the pulmonary circuit
Untreated patients develop symptoms of congestive cardiac failure
Coarctation of the aorta
Aortic stenosis at the site of the ductus arteriosus insertion
More prevalent in boys or females with Turners syndrome
Patients may present with symptoms of arterial insufficiency, such as syncope and claudication
Blood pressure mismatch may be seen, as may mismatch of pulse pressure in the upper and lower limbs
Treatment is either with angioplasty or surgical resection (the former is the most common)
Which of the following structures does not pass anterior to the lateral malleolus?
A-Anterior tibial artery
B-Extensor digitorum longus
C-Lateral branch of the superficial peroneal nerve
D-Peroneus brevis
E-Peroneus tertius
Peroneus brevis passes posterior to the lateral malleolus. Peroneus tertius is closely related to extensor digitorum and like that muscle, its tendon passes anterior to the lateral malleolus
Lateral malleolus
Structures posterior to the lateral malleolus and superficial to superior peroneal retinaculum
Sural nerve
Short saphenous vein
Structures posterior to the lateral malleolus and deep to superior peroneal retinaculum
Peroneus longus tendon
Peroneus brevis tendon
The calcaneofibular ligament is attached at the lateral malleolus
Which of the following drugs is not positively inotropic?
A-Dopamine
B-Glucagon
C-Theophylline
D-Sodium thiopentone
E-Dobutamine
Correct Answer:
D. Sodium thiopentone
Explanation:
Positive inotropic drugs increase the strength of cardiac muscle contraction, which improves cardiac output. Here’s a brief explanation of each drug:
• Dopamine: A catecholamine that increases heart contractility and cardiac output by stimulating beta-1 adrenergic receptors. • Glucagon: Increases heart contractility by stimulating adenylate cyclase and increasing cyclic AMP, independent of the adrenergic receptors. • Theophylline: A methylxanthine that can increase heart contractility through its action as a phosphodiesterase inhibitor, leading to increased cyclic AMP levels. • Sodium thiopentone (Thiopental): A barbiturate used for induction of anesthesia. It has a depressive effect on the central nervous system and does not have positive inotropic effects. In fact, it can cause myocardial depression and reduce cardiac output. • Dobutamine: A synthetic catecholamine that acts on beta-1 adrenergic receptors, increasing heart contractility and cardiac output.
Therefore, Sodium thiopentone is the drug that is not positively inotropic.
Inotropes are a class of drugs that increase the force or cardiac contractility. This may improve cardiac output. Increased blood pressure may have direct beneficial effects for the heart in that it improves myocardial perfusion pressure. Dopamine and dobutamine are both commonly used inotropes, they should be administered via a central line and in a monitored setting. Glucagon and theophylline are also positive inotropes (although not commonly used for this purpose). In contrast sodium thiopentone causes marked myocardial depression.
Inotropes and cardiovascular receptors
Inotropes are a class of drugs which work primarily by increasing cardiac output. They should be distinguished from vasoconstrictor drugs which are used specifically when the primary problem is peripheral vasodilatation.
Catecholamine type agents are commonly used and work by increasing cAMP levels by adenylate cyclase stimulation. This in turn increases intracellular calcium ion mobilisation and thus the force of contraction. Adrenaline works as a beta adrenergic receptor agonist at lower doses and an alpha receptor agonist at higher doses. Dopamine causes dopamine receptor mediated renal and mesenteric vascular dilatation and beta 1 receptor agonism at higher doses. This results in increased cardiac output. Since both heart rate and blood pressure are raised, there is less overall myocardial ischaemia. Dobutamine is a predominantly beta 1 receptor agonist with weak beta 2 and alpha receptor agonist properties. Noradrenaline is a catecholamine type agent and predominantly acts as an alpha receptor agonist and serves as a peripheral vasoconstrictor.
Phosphodiesterase inhibitors such as milrinone act specifically on the cardiac phosphodiesterase and increase cardiac output.
Inotrope Cardiovascular receptor action
Adrenaline α-1, α-2, β-1, β-2
Noradrenaline α-1,( α-2), (β-1), (β-2)
Dobutamine β-1, (β 2)
Dopamine (α-1), (α-2), (β-1), D-1,D-2
Minor receptor effects in brackets
Effects of receptor binding
α-1, α-2 vasoconstriction
β-1 increased cardiac contractility and HR
β-2 vasodilatation
D-1 renal and spleen vasodilatation
D-2 inhibits release of noradrenaline
A 28 year old man falls onto an outstretched hand. On examination, there is tenderness of the anatomical snuffbox. However, forearm and hand x-rays are normal. What is the most appropriate course of action?
Discharge with reassurance
Place in arm sling and discharge
Place in futura splint and review in fracture clinic
Admit for surgical exploration
Apply an external fixation device
Correct Answer:
3. Place in futura splint and review in fracture clinic
Explanation:
Tenderness in the anatomical snuffbox following a fall onto an outstretched hand is highly suggestive of a scaphoid fracture, even if initial X-rays are normal. Scaphoid fractures may not always be visible on initial X-rays and can lead to complications such as nonunion or avascular necrosis if not properly managed.
Appropriate management includes:
• Immobilization: Placing the wrist in a splint, such as a futura splint, to immobilize the area and prevent further injury. • Follow-up: Arranging for a review in the fracture clinic where further imaging (e.g., repeat X-rays, MRI, or CT) can be performed if needed to confirm the diagnosis.
Options not chosen:
• Discharge with reassurance: Inappropriate due to the high suspicion of a scaphoid fracture. • Place in arm sling and discharge: Insufficient as it does not adequately immobilize the wrist. • Admit for surgical exploration: Not indicated at this stage without a confirmed fracture. • Apply an external fixation device: Unnecessary for a suspected scaphoid fracture without a confirmed diagnosis.
Therefore, the best course of action is to place the wrist in a futura splint and arrange for a review in the fracture clinic.
This could well be a scaphoid fracture and should be temporarily immobilised pending further review. A futura splint will immobilise better than an arm sling for this problem. Fracture management
Bony injury resulting in a fracture may arise from trauma (excessive forces applied to bone), stress related (repetitive low velocity injury) or pathological (abnormal bone which fractures during normal use of following minimal trauma)
Diagnosis involves not just evaluating the fracture ; such as site and type of injury but also other associated injuries and distal neurovascular deficits. This may entail not just clinical examination but radiographs of proximal and distal joints.
When assessing x-rays it is important to assess for changes in length of the bone, the angulation of the distal bone, rotational effects, presence of material such as glass.
Fracture types
Fracture type Description
Oblique fracture Fracture lies obliquely to long axis of bone
Comminuted fracture >2 fragments
Segmental fracture More than one fracture along a bone
Transverse fracture Perpendicular to long axis of bone
Spiral fracture Severe oblique fracture with rotation along long axis of bone
Open Vs Closed
It is also important to distinguish open from closed injuries. The most common classification system for open fractures is the Gustilo and Anderson classification system (given below):
Grade Injury
1 Low energy wound <1cm
2 Greater than 1cm wound with moderate soft tissue damage
3 High energy wound > 10cm with extensive soft tissue damage
3 A (sub group of 3) Adequate soft tissue coverage
3 B (sub group of 3) Inadequate soft tissue coverage
3 C (sub group of 3) Associated arterial injury
Key points in management of fractures
Immobilise the fracture including the proximal and distal joints
Carefully monitor and document neurovascular status, particularly following reduction and immobilisation
Manage infection including tetanus prophylaxis
IV broad spectrum antibiotics for open injuries
As a general principle all open fractures should be thoroughly debrided ( and internal fixation devices avoided or used with extreme caution)
Open fractures constitute an emergency and should be debrided and lavaged within 6 hours of injury
A 56 year old man presents with episodic facial pain and discomfort whilst eating. He has suffered from halitosis recently and he frequently complains of a dry mouth. He has a smooth swelling underneath his right mandible. What is the most likely underlying diagnosis?
Stone impacted in Whartons duct
Stone impacted in Stensens duct
Benign adenoma of the submandibular gland
Adenocarcinoma of the submandibular gland
Squamous cell carcinoma of the submandibular gland
The symptoms are typical for sialolithiasis. The stones most commonly form in the submandibular gland and therefore may occlude Whartons duct. Stensens duct drains the parotid gland. Submandibular glands- disease
Physiology
The submandibular glands secrete approximately 800- 1000ml saliva per day. They typically produce mixed seromucinous secretions. When parasympathetic activity is dominant; the secretions will be more serous. The parasympathetic fibres are derived from the chorda tympani nerves and the submandibular ganglion. Sensory fibres are conveyed by the lingual branch of the mandibular nerve.
Sialolithiasis
80% of all salivary gland calculi occur in the submandibular gland
70% of the these calculi are radio-opaque
Stones are usually composed of calcium phosphate or calcium carbonate
Patients typically develop colicky pain and post prandial swelling of the gland
Investigation involves sialography to demonstrate the site of obstruction and associated other stones
Stones impacted in the distal aspect of Whartons duct may be removed orally, other stones and chronic inflammation will usually require gland excision
Sialadenitis
Usually occurs as a result of Staphylococcus aureus infection
Pus may be seen leaking from the duct, erythema may also be noted
Development of a sub mandibular abscess is a serious complication as it may spread through the other deep fascial spaces and occlude the airway
Submandibular tumours
Only 8% of salivary gland tumours affect the sub mandibular gland
Of these 50% are malignant (usually adenoid cystic carcinoma)
Diagnosis usually involves fine needle aspiration cytology
Imaging is with CT and MRI
In view of the high prevalence of malignancy, all masses of the submandibular glands should generally be excised.
A 20 year old man is admitted with bloody diarrhoea. He has been passing 10 stools per day, Hb-8.0, albumin-20. Stool culture negative. Evidence of colitis on endoscopy. He has been on intravenous steroids for 5 days and has now developed megacolon. His haemoglobin is falling and inflammatory markers are static. What is the most appropriate course of action?
Double the steroid dose
Undertake a loop ileostomy
Undertake a sub total colectomy and end ileostomy
Undertake a sub total colectomy and ileo-rectal anastomosis
Undertake a pan proctocolectomy
Given the clinical scenario of a 20-year-old man with bloody diarrhea, a failing response to intravenous steroids, and development of megacolon, the most appropriate course of action is a surgical intervention. The options of further medical management, such as doubling the steroid dose, are unlikely to be effective at this stage due to the risk of perforation and other complications associated with toxic megacolon.
The most appropriate course of action is:
Undertake a sub total colectomy and end ileostomy
Explanation:
1. Development of Megacolon: The patient has developed a toxic megacolon, which is a severe complication of colitis characterized by extreme dilatation of the colon and potential for perforation. This condition requires urgent surgical intervention. 2. Failure of Medical Management: The patient has been on intravenous steroids for 5 days without improvement, indicating that medical management has failed. 3. Rising Complications: With falling hemoglobin levels and static inflammatory markers, the patient’s condition is worsening, necessitating immediate surgery.
A subtotal colectomy with an end ileostomy is typically recommended as it removes the diseased colon and diverts fecal stream, reducing the risk of perforation and sepsis, and stabilizing the patient.
Pan proctocolectomy should not be performed in acute unwell patients
Loop ileostomy is not conventional management of UC
This man requires a sub total colectomy. Conservative management has failed. Patients with ulcerative colitis should undergo colectomy if there is no significant improvement in 5-7 days after initiating medical therapy if they have a severe attack of the disease. To undertake a pan proctocolectomy in an acutely unwell patient is very high risk and most unwise. Management of the rectum is generally left until the patient is more stable.
IBD
Ulcerative colitis Vs Crohns
Crohn’s disease Ulcerative colitis
Distribution Mouth to anus Rectum and colon
Macroscopic changes Cobblestone appearance, apthoid ulceration Contact bleeding
Depth of disease Transmural inflammation Superficial inflammation
Distribution pattern Patchy Continuous
Histological features Granulomas (non caseating epithelioid cell aggregates with Langerhans’ giant cells) Crypt abscesses, Inflammatory cells in the lamina propria
Surgical treatment
Ulcerative colitis
In UC the main place for surgery is when medical treatment has failed, in the emergency setting this will be a sub total colectomy, end ileostomy and a mucous fistula. Electively it will be a pan proctocolectomy, an ileoanal pouch may be a selected option for some. Remember that longstanding UC increases colorectal cancer risk.
Crohn’s disease
Unlike UC Crohn’s patients need to avoid surgeons, minimal resections are the rule. They should not have ileoanal pouches as they will do poorly with them. Management of Crohn’s ano rectal sepsis is with a minimal approach, simply drain sepsis and use setons to facilitate drainage. Definitive fistula surgery should be avoided.
A 16 year old boy is hit by a car and sustains a blow to the right side of his head. He is initially conscious but on arrival in the emergency department is comatose. On examination his right pupil is fixed and dilated. The neurosurgeons plan immediate surgery. What type of initial approach should be made?
Left parieto-temporal craniotomy
Right parieto-temporal craniotomy
Posterior fossa craniotomy
Left parieto-temporal burr holes
None of the above
A unilateral dilated pupil is a classic sign of transtentorial herniation. The medial aspect of the temporal lobe (uncus) herniates across the tentorium and causes pressure on the ipsilateral oculomotor nerve, interrupting parasympathetic input to the eye and resulting in a dilated pupil. In addition the brainstem is compressed. As the ipsilateral oculomotor nerve is being compressed, craniotomy (rather than Burr Holes) should be made on the ipsilateral side.
Head injury
Patients who suffer head injuries should be managed according to ATLS principles and extra cranial injuries should be managed alongside cranial trauma. Inadequate cardiac output will compromise CNS perfusion irrespective of the nature of the cranial injury.
Types of traumatic brain injury
Extradural haematoma Bleeding into the space between the dura mater and the skull. Often results from acceleration-deceleration trauma or a blow to the side of the head. The majority of extradural haematomas occur in the temporal region where skull fractures cause a rupture of the middle meningeal artery.
Features
Raised intracranial pressure
Some patients may exhibit a lucid interval
Subdural haematoma Bleeding into the outermost meningeal layer. Most commonly occur around the frontal and parietal lobes. May be either acute or chronic.
Risk factors include old age and alcoholism.
Slower onset of symptoms than a extradural haematoma.
Subarachnoid haemorrhage Usually occurs spontaneously in the context of a ruptured cerebral aneurysm, but may be seen in association with other injuries when a patient has sustained a traumatic brain injury.
Pathophysiology
Primary brain injury may be focal (contusion/ haematoma) or diffuse (diffuse axonal injury)
Diffuse axonal injury occurs as a result of mechanical shearing following deceleration, causing disruption and tearing of axons
Intra-cranial haematomas can be extradural, subdural or intracerebral, while contusions may occur adjacent to (coup) or contralateral (contre-coup) to the side of impact
Secondary brain injury occurs when cerebral oedema, ischaemia, infection, tonsillar or tentorial herniation exacerbates the original injury. The normal cerebral auto regulatory processes are disrupted following trauma rendering the brain more susceptible to blood flow changes and hypoxia
The Cushings reflex (hypertension and bradycardia) often occurs late and is usually a pre terminal event
Management
Where there is life threatening rising ICP such as in extra dural haematoma and whilst theatre is prepared or transfer arranged use of IV mannitol/ frusemide may be required.
Diffuse cerebral oedema may require decompressive craniotomy
Exploratory Burr Holes have little management in modern practice except where scanning may be unavailable and to thus facilitate creation of formal craniotomy flap
Depressed skull fractures that are open require formal surgical reduction and debridement, closed injuries may be managed non operatively if there is minimal displacement.
ICP monitoring is appropriate in those who have GCS 3-8 and normal CT scan.
ICP monitoring is mandatory in those who have GCS 3-8 and abnormal CT scan.
Hyponatraemia is most likely to be due to syndrome of inappropriate ADH secretion.
Minimum of cerebral perfusion pressure of 70mmHg in adults.
Minimum cerebral perfusion pressure of between 40 and 70 mmHg in children.
Interpretation of pupillary findings in head injuries
Pupil size Light response Interpretation
Unilaterally dilated Sluggish or fixed 3rd nerve compression secondary to tentorial herniation
Bilaterally dilated Sluggish or fixed
Poor CNS perfusion
Bilateral 3rd nerve palsy
Unilaterally dilated or equal Cross reactive (Marcus - Gunn) Optic nerve injury
Bilaterally constricted May be difficult to assess
Opiates
Pontine lesions
Metabolic encephalopathy
Unilaterally constricted Preserved Sympathetic pathway disruption
Which of the following pathological explanations best describes the initial pathological processes occurring in an abdominal aortic aneurysm in an otherwise well 65 year old, hypertensive male?
Loss of elastic fibres from the adventitia
Loss of collagen from the adventitia
Loss of collagen from the media
Loss of elastic fibres from the media
Decreased matrix metalloproteinases in the adventitia
D In established aneurysmal disease there is dilation of all layers of the arterial wall and loss of both elastin and collagen. The primary event is loss of elastic fibres with subsequent degradation of collagen fibres. Pathology of abdominal aortic aneurysm
Abdominal aortic aneurysms occur primarily as a result of the failure of elastic proteins within the extracellular matrix. Aneurysms typically represent dilation of all layers of the arterial wall. Most aneurysms are caused by degenerative disease. After the age of 50 years the normal diameter of the infrarenal aorta is 1.5cm in females and 1.7cm in males. Diameters of 3cm and greater, are considered aneurysmal. The pathophysiology involved in the development of aneurysms is complex and the primary event is loss of the intima with loss of elastic fibres from the media. This process is associated with, and potentiated by, increased proteolytic activity and lymphocytic infiltration.
Major risk factors for the development of aneurysms include smoking and hypertension. Rare but important causes include syphilis and connective tissues diseases such as Ehlers Danlos type 1 and Marfans syndrome.
A 45 year old woman with breast cancer is started on a chemotherapy regime containing epirubicin. What is the primary mode of action of this drug?
Intercalation of DNA
Antimetabolite
Monoclonal antibody to epidermal growth factor
Inhibition of DNA gyrase
Inhibition of topoisomerase 1
Epirubicin is an anthracycline chemotherapeutic agent primarily used in the treatment of various cancers, including breast cancer.
The primary mode of action of epirubicin is:
Intercalation of DNA
Explanation:
1. Intercalation of DNA: Epirubicin works by intercalating into DNA, which means it inserts itself between DNA base pairs. This intercalation disrupts the structure of the DNA helix, inhibiting DNA replication and transcription. Additionally, it stabilizes the topoisomerase II-DNA complex, preventing the religation of the DNA strands and leading to apoptosis of the cancer cells. 2. Antimetabolite: Antimetabolites are drugs that interfere with the normal metabolism of cells, usually by mimicking the normal substrates of key enzymes, but this is not the primary action of epirubicin. 3. Monoclonal antibody to epidermal growth factor: Monoclonal antibodies target specific proteins, such as the epidermal growth factor receptor (EGFR), but epirubicin is not a monoclonal antibody. 4. Inhibition of DNA gyrase: DNA gyrase is an enzyme targeted by some antibiotics, particularly in bacteria, but it is not the target of epirubicin. 5. Inhibition of topoisomerase 1: Epirubicin primarily inhibits topoisomerase II, not topoisomerase I.
Therefore, epirubicin’s primary mode of action is intercalation of DNA.
A 53 year old man is undergoing a radical gastrectomy for carcinoma of the stomach. Which of these structures will need to be divided to gain access to the coeliac axis?
Lesser omentum
Greater omentum
Falciform ligament
Median arcuate ligament
Gastrosplenic ligament
The lesser omentum will need to be divided. During a radical gastrectomy this forms one of the nodal stations that will need to be taken. Coeliac axis
The coeliac axis has three main branches.
Left gastric
Hepatic: branches-Right Gastric, Gastroduodenal, Superior Pancreaticoduodenal, Cystic (occasionally).
Splenic: branches- Pancreatic, Short Gastric, Left Gastroepiploic
It occasionally gives off one of the inferior phrenic arteries.
Relations
Anteriorly Lesser omentum
Right Right coeliac ganglion and caudate process of liver
Left Left coeliac ganglion and gastric cardia
Inferiorly Upper border of pancreas and renal vein
A 22 year old man is involved in a fight and sustains a skull fracture with an injury to the middle meningeal artery. A craniotomy is performed, and with considerable difficulty the haemorrhage from the middle meningeal artery is controlled by ligating it close to its origin. What is the most likely sensory impairment that the patient may notice post operatively?
Parasthesia of the ipsilateral external ear
Loss of taste sensation from the anterior two thirds of the tongue
Parasthesia overlying the angle of the jaw
Loss of sensation from the ipsilateral side of the tongue
Loss of taste from the posterior two thirds of the tongue
The most likely sensory impairment that the patient may notice post-operatively is:
Parasthesia overlying the angle of the jaw
Explanation:
The middle meningeal artery is a branch of the maxillary artery and supplies the meninges of the brain. During a craniotomy to control hemorrhage from the middle meningeal artery, nearby structures, including the auriculotemporal nerve, can be affected. The auriculotemporal nerve is a branch of the mandibular nerve (V3) and provides sensory innervation to the skin over the temple and the anterior portion of the ear, as well as the temporomandibular joint (TMJ).
Given this anatomical relationship, damage or manipulation of the auriculotemporal nerve during the surgery could lead to parasthesia (abnormal sensation) overlying the angle of the jaw. This is because the auriculotemporal nerve innervates that region.
Other options do not align as closely with the anatomical course and innervation area of the middle meningeal artery and its surrounding structures:
• Parasthesia of the ipsilateral external ear: This would more likely involve the great auricular nerve or lesser occipital nerve. • Loss of taste sensation from the anterior two-thirds of the tongue: This involves the chorda tympani branch of the facial nerve (VII). • Loss of sensation from the ipsilateral side of the tongue: This involves the lingual nerve, a branch of the mandibular nerve (V3). • Loss of taste from the posterior two-thirds of the tongue: This involves the glossopharyngeal nerve (IX).
As regards the internal jugular vein, which of the following statements is untrue?
It lies within the carotid sheath
It is the continuation of the sigmoid sinus
The terminal part of the thoracic duct crosses anterior to it to insert into the right subclavian vein
The hypoglossal nerve is closely related to it as it passes near the atlas
The vagus nerve is closely related to it within the carotid sheath
The terminal part of the thoracic duct inserts into the left subclavian vein and it passes posterior to the jugular vein.
Internal jugular vein
Each jugular vein begins in the jugular foramen, where they are the continuation of the sigmoid sinus. They terminate at the medial end of the clavicle where they unite with the subclavian vein.
The vein lies within the carotid sheath throughout its course. Below the skull the internal carotid artery and last four cranial nerves are anteromedial to the vein. Thereafter it is in contact medially with the internal (then common) carotid artery. The vagus lies posteromedially.
At its superior aspect, the vein is overlapped by sternocleidomastoid and covered by it at the inferior aspect of the vein.
Below the transverse process of the atlas it is crossed on its lateral side by the accessory nerve. At its mid point it is crossed by the inferior root of the ansa cervicalis.
Posterior to the vein are the transverse processes of the cervical vertebrae, the phenic nerve as it descends on the scalenus anterior, and the first part of the subclavian artery.
On the left side its also related to the thoracic duct.
A 43 year old lady undergoes a live donor related renal transplant. Over the next few years it is noted that her renal function progressively deteriorates. What is the most likely underlying explanation?
Type I hypersensitivity reaction
Type III hypersensitivity reaction
Type II hypersensitivity reaction
Type IV hypersensitivity reaction
None of the above
Chronic rejection of renal transplants is mediated via T lymphocytes and is therefore a type IV hypersensitivity reaction. This process can be mitigated by immunosupression. Organ Transplant
A number of different organ and tissue transplants are now available. In many cases an allograft is performed, where an organ is transplanted from one individual to another. Allografts will elicit an immune response and this is one of the main reasons for organ rejection.
Graft rejection occurs because allografts have allelic differences at genes that code immunohistocompatability complex genes. The main antigens that give rise to rejection are:
ABO blood group
Human leucocyte antigens (HLA)
Minor histocompatability antigens
ABO Matching
ABO incompatibility will result in early organ rejection (hyperacute) because of pre existing antibodies to other groups. Group O donors can give organs to any type of ABO recipient whereas group AB donor can only donate to AB recipient.
HLA System
The four most important HLA alleles are:
HLA A
HLA B
HLA C
HLA DR
An ideal organ match would be one in which all 8 alleles are matched (remember 2 from each parent, four each = 8 alleles). Modern immunosuppressive regimes help to manage the potential rejection due to HLA mismatching. However, the greater the number of mismatches the worse the long term outcome will be. T lymphocytes will recognise antigens bound to HLA molecules and will then become activated. Clonal expansion then occurs with a response directed against that antigen.
Types of organ rejection
Hyperacute. This occurs immediately through presence of pre formed antibodies (such as ABO incompatibility).
Acute. Occurs during the first 6 months and is usually T cell mediated. Usually tissue infiltrates and vascular lesions.
Chronic. Occurs after the first 6 months. Vascular changes predominate.
Hyperacute
Renal transplants at greatest risk and liver transplants at least risk. Although ABO incompatibility and HLA Class I incompatible transplants will all fare worse in long term.
Acute
All organs may undergo acute rejection. Mononuclear cell infiltrates predominate. All types of transplanted organ are susceptible and it may occur in up to 50% cases.
Chronic
Again all transplants with HLA mismatch may suffer this fate. Previous acute rejections and other immunosensitising events all increase the risk. Vascular changes are most prominent with myointimal proliferation leading to organ ischaemia. Organ specific changes are also seen such as loss of acinar cells in pancreas transplants and rapidly progressive coronary artery disease in cardiac transplants.
Surgical overview-Renal transplantation
A brief overview of the steps involved in renal transplantation is given.
Patients with end stage renal failure who are dialysis dependent or likely to become so in the immediate future are considered for transplant. Exclusion criteria include; active malignancy, old age (due to limited organ availability). Patients are medically optimised.
Donor kidneys, these may be taken from live related donors and close family, members may have less HLA mismatch than members of the general population. Laparoscopic donor nephrectomy further minimises the operative morbidity for the donor. Other organs are typically taken from brain dead or dying patients who have a cardiac arrest and in whom resuscitation is futile. The key event is to minimise the warm ischaemic time in the donor phase.
The kidney once removed is usually prepared on the bench in theatre by the transplant surgeon immediately prior to implantation and factors such as accessory renal arteries and vessel length are assessed and managed.
For first time recipients the operation is performed under general anaesthesia. A Rutherford-Morison incision is made on the preferred side. This provides excellent extraperitoneal access to the iliac vessels. The external iliac artery and vein are dissected out and following systemic heparinisation are cross clamped. The vein and artery are anastamosed to the iliacs and the clamps removed. The ureter is then implanted into the bladder and a stent is usually placed to maintain patency. The wounds are then closed and the patient recovered from surgery.
In the immediate phase a common problem encountered in cadaveric kidneys is acute tubular necrosis and this tends to resolve.
Graft survival times from cadaveric donors are typically of the order of 9 years and monozygotic twin transplant (live donor) may survive as long as 25 years.
A 10 year old child is admitted to the emergency department after a fall. On examination, the blood pressure is 100/55mmHg, pulse rate 90, abdomen soft but tender on the left. Abdominal imaging demonstrates a grade III splenic laceration. What is the most appropriate course of action?
Undertake an immediate laparotomy and splenectomy
Undertake a laparoscopy and laparoscopic splenectomy
Admit the child to the high dependency unit for close monitoring
Arrange splenic artery embolisation
Undertake a laparotomy and splenic repair
Splenic trauma is nearly always managed conservatively. Hilar injuries (grade IV) are less amenable to this and will tend to come to surgery.
The most appropriate course of action for a 10-year-old child with a grade III splenic laceration and stable vital signs is:
Admit the child to the high dependency unit for close monitoring
Explanation:
In pediatric patients, the management of splenic injuries often favors non-operative treatment, especially in the context of stable vital signs. Grade III splenic lacerations, while significant, can often be managed conservatively if the patient remains hemodynamically stable. Close monitoring in a high dependency unit (HDU) is appropriate to ensure any changes in the child’s condition are promptly addressed. This approach minimizes the risks associated with surgery and preserves splenic function, which is important for immunological reasons.
• Immediate laparotomy and splenectomy: This is typically reserved for patients who are hemodynamically unstable or have ongoing significant bleeding that cannot be controlled non-operatively. • Laparoscopy and laparoscopic splenectomy: Similar to open splenectomy, this is more invasive and generally not the first choice in hemodynamically stable pediatric patients. • Splenic artery embolisation: This is usually considered for patients with ongoing bleeding but can often be avoided in stable children. • Laparotomy and splenic repair: This is an option if non-operative management fails or if there is significant concern for complications that cannot be managed non-operatively. However, in a stable child, initial non-operative management is preferred.
Therefore, admitting the child to the HDU for close monitoring is the most appropriate and conservative approach, given the current stability of the patient.
Trauma management
The cornerstone of trauma management is embodied in the principles of ATLS.
Following trauma there is a trimodal death distribution:
Immediately following injury. Typically as result of brain or high spinal injuries, cardiac or great vessel damage. Salvage rate is low.
In early hours following injury. In this group deaths are due to phenomena such as splenic rupture, sub dural haematomas and haemopneumothoraces
In the days following injury. Usually due to sepsis or multi organ failure.
Aspects of trauma management
ABCDE approach.
Tension pneumothoraces will deteriorate with vigorous ventilation attempts.
External haemorrhage is managed as part of the primary survey. As a rule tourniquets should not be used. Blind application of clamps will tend to damage surrounding structures and packing is the preferred method of haemorrhage control.
Urinary catheters and naso gastric tubes may need inserting. Be wary of basal skull fractures and urethral injuries.
Patients with head and neck trauma should be assumed to have a cervical spine injury until proven otherwise.
Thoracic injuries
Simple pneumothorax
Mediastinal traversing wounds
Tracheobronchial tree injury
Haemothorax
Blunt cardiac injury
Diaphragmatic injury
Aortic disruption
Pulmonary contusion
Management of thoracic trauma
Simple pneumothorax insert chest drain. Aspiration is risky in trauma as pneumothorax may be from lung laceration and convert to tension pneumothorax.
Mediastinal traversing wounds These result from situations like stabbings. Exit and entry wounds in separate hemithoraces. The presence of a mediastinal haematoma indicates the likelihood of a great vessel injury. All patients should undergo CT angiogram and oesophageal contrast swallow. Indications for thoracotomy are largely related to blood loss and will be addressed below.
Tracheobronchial tree injury Unusual injuries. In blunt trauma most injuries occur within 4cm of the carina. Features suggesting this injury include haemoptysis and surgical emphysema. These injuries have a very large air leak and may have tension pneumothorax.
Haemothorax Usually caused by laceration of lung vessel or internal mammary artery by rib fracture. Patients should all have a wide bore 36F chest drain. Indications for thoracotomy include loss of more than 1.5L blood initially or ongoing losses of >200ml per hour for >2 hours.
Cardiac contusions Usually cardiac arrhythmias, often overlying sternal fracture. Perform echocardiography to exclude pericardial effusions and tamponade. Risk of arrhythmias falls after 24 hours.
Diaphragmatic injury Usually left sided. Direct surgical repair is performed.
Traumatic aortic disruption Commonest cause of death after RTA or falls. Usually incomplete laceration near ligamentum arteriosum. All survivors will have contained haematoma. Only 1-2% of patients with this injury will have a normal chest x-ray.
Pulmonary contusion Common and lethal. Insidious onset. Early intubation and ventilation.
Abdominal trauma
Deceleration injuries are common.
In blunt trauma requiring laparotomy the spleen is most commonly injured (40%)
Stab wounds traverse structures most commonly liver (40%)
Gunshot wounds have variable effects depending upon bullet type. Small bowel is most commonly injured (50%)
Patients with stab wounds and no peritoneal signs up to 25% will not enter the peritoneal cavity
Blood at urethral meatus suggests a urethral tear
High riding prostate on PR = urethral disruption
Mechanical testing for pelvic stability should only be performed once
Investigations in abdominal trauma
Diagnostic Peritoneal Lavage Abdominal CT scan USS
Indication Document bleeding if hypotensive Document organ injury if normotensive Document fluid if hypotensive
Advantages Early diagnosis and sensitive; 98% accurate Most specific for localising injury; 92 to 98% accurate Early diagnosis, non invasive and repeatable; 86 to 95% accurate
Disadvantages Invasive and may miss retroperitoneal and diaphragmatic injury Location of scanner away from facilities, time taken for reporting, need for contrast Operator dependent and may miss retroperitoneal injury
Amylase may be normal following pancreatic trauma
Urethrography if suspected urethral injury
A 10 year old boy is shot in the abdomen with an airgun pellet. He is concerned that he will get into trouble and the injury remains concealed for 10 days. Imaging using CT scanning shows it to be lodged in the left lobe of the liver. On examination, his abdomen is soft and non tender and he seems well. What is the best course of action?
Operate and remove the pellet on the next emergency list
Operate and remove the pellet on the next elective operating list
Do not operate and review the patient several weeks later
Extract the pellet using interventional radiology techniques
Perform an MRI scan
Given that the child is well, the risks of removing the pellet are not outweighed by the benefits and it should be left in situ at this stage. Management of acute cases- Paediatric
Children will often insert objects into orifices such as the nose and external auditory meatus
Assessment includes assessment of airway and haemodynamic status
Where the airway is not immediately threatened decisions can be made as to whether to manage in the emergency department or transfer to theatre
In general children do not tolerate procedures well and it is usually safer to remove objects in theatre and under general anaesthesia with a secure airway
A chest x-ray is required to ensure that no object is present in the chest, not all objects are radioopaque. However, signs such as focal consolidation may indicate small airway obstruction
In the case of small bore missile injuries the decision relating to surgery depends on the size of the missile and its location. Airgun pellets are a common culprit, if there is a long time interval between the incident and presentation and the object has not caused any significant problems then it may be best left alone
Airgun pellets (and glass) lodged in the soft tissues are usually notoriously difficult to localise and extract, no matter how superficial. Removal in theatre is usually the best option. If the object is radioopaque then an image intensifier should be used
A 54-year-old man presents to the Emergency Department with a 2 day history of a swollen, painful left knee. You aspirate the joint to avoid admission to the orthopaedic wards. Aspirated joint fluid shows calcium pyrophosphate crystals. Which of the following blood tests is most useful in revealing an underlying cause?
Transferrin saturation
ACTH
ANA
Serum ferritin
LDH
This is a typical presentation of pseudogout. An elevated transferrin saturation may indicate haemochromatosis, a recognised cause of pseudogout.
A high ferritin level is also seen in haemochromatosis but can be raised in a variety of infective and inflammatory processes, including pseudogout, as part of an acute phase response.
Pseudogout
Pseudogout is a form of microcrystal synovitis caused by the deposition of calcium pyrophosphate dihydrate in the synovium
Risk factors
hyperparathyroidism
hypothyroidism
haemochromatosis
acromegaly
low magnesium, low phosphate
Wilson’s disease
Features
knee, wrist and shoulders most commonly affected
joint aspiration: weakly-positively birefringent rhomboid shaped crystals
x-ray: chondrocalcinosis
Management
aspiration of joint fluid, to exclude septic arthritis
NSAIDs or intra-articular, intra-muscular or oral steroids as for gout
A 73 year old lady sustains a distal radius fracture and this is manipulated using a Biers block with prilocaine as the local anaesthetic agent. During the procedure the occlusion cuff deflates and the patient becomes progressively cyanosed. What is the treatment of choice?
Intravenous calcium gluconate
Exchange transfusion
Intravenous methylene blue
Intravenous sodium thiosulphate
Intravenous gelofusine
Prilocaine is a recognised cause of methaemoglobinaemia, this is characterised by the development of cyanosis and dyspnoea. This disorder occurs because of the change haemoglobin to a ferric subtype rather than ferrous (Fe2+). This type of change shifts the oxygen dissociation curve to the left and tissue hypoxia occurs. Methylene blue will revert the haemoglobin to the ferrous type and reverse this effect.
Local anaesthetic toxicity
Toxicity results from either accidental intravascular injection (rapid onset of symptoms-usually correct dose), or from excessive dosage (slower onset). Local anaesthetic agents not only exert a membrane stabilising effect on peripheral nerves but will also act on excitable membranes within the CNS and Heart. The sensory neurones in the CNS are suppressed before the motor ones. As a result the early symptoms will typically be those of circumoral paraesthesia and tinnitus, followed by falling GCS and eventually coma.
Management of toxicity
Stop injecting the anaesthetic agent
High flow 100% oxygen via face mask
Cardiovascular monitoring
Administer lipid emulsion (Intralipid 20%) at 1.5ml/Kg over 1 minute as a bolus
Consider lipid emulsion infusion, at 0.25ml/ Kg/ minute
If toxicity due to prilocaine then administer methylene blue
Safe doses
10ml of lignocaine 1% contains 100mg of drug, this would constitute 70% of the maximum safe dose in a 50 kg patient. Up to 7mg / kg can be administered if adrenaline is added to the solution.
Doses of local anaesthetics
Agent Dose plain Dose with adrenaline
Lignocaine 3mg/Kg 7mg/Kg
Bupivicane 2mg/Kg 2mg/Kg
Prilocaine 6mg/Kg 9mg/Kg
These are a guide only as actual doses depend on site of administration, tissue vascularity and co-morbidities.
A 28 year old man undergoes a completion right hemicolectomy for treatment of a 5cm appendiceal carcinoid. As part of his follow up he is due to undergo 24 hour urine collection for 5-HIAA. Which of the following causes an elevated 5-HIAA in a 24-hour urine collection?
Naproxen
Oranges
Flucloxacillin
Amiodarone
Beef
t is important to be aware of what can falsely elevate 5-HIAA to avoid diagnosing carcinoid syndrome incorrectly. These include:
Food: spinach, cheese, wine, caffeine, tomatoes
Drugs: Naproxen, Monoamine oxidase inhibitors
Recent surgery
Carcinoid syndrome
Carcinoid tumours secrete serotonin
Originate in neuroendocrine cells mainly in the intestine (midgut-distal ileum/appendix)
Can occur in the rectum, bronchi
Hormonal symptoms mainly occur when disease spreads outside the bowel
Clinical features
Onset: insidious over many years
Flushing face
Palpitations
Pulmonary valve stenosis and tricuspid regurgitation causing dyspnoea
Asthma
Severe diarrhoea (secretory, persists despite fasting)
Investigation
5-HIAA in a 24-hour urine collection
Somatostatin receptor scintigraphy
CT scan
Blood testing for chromogranin A
Treatment
Octreotide
Surgical removal
A 5 year old boy presents with recurrent headaches. As part of his assessment he undergoes an MRI scan of his brain. This demonstrates enlargement of the lateral and third ventricles. Where is the most likely site of obstruction?
Foramen of Luschka
Foramen of Magendie
Foramen of Munro
Aqueduct of Sylvius
None of the above
The CSF flows from the 3rd to the 4th ventricle via the Aqueduct of Sylvius.
Cerebrospinal fluid
The CSF fills the space between the arachnoid mater and pia mater (covering surface of the brain). The total volume of CSF in the brain is approximately 150ml. Approximately 500 ml is produced by the ependymal cells in the choroid plexus (70%), or blood vessels (30%). It is reabsorbed via the arachnoid granulations which project into the venous sinuses.
Circulation
1. Lateral ventricles (via foramen of Munro)
2. 3rd ventricle
3. Cerebral aqueduct (aqueduct of Sylvius)
4. 4th ventricle
5. Subarachnoid space (via foramina of Magendie and Luschka)
6. Reabsorbed into the venous system via arachnoid granulations into superior sagittal sinus
Composition
Glucose: 50-80mg/dl
Protein: 15-40 mg/dl
Red blood cells: Nil
White blood cells: 0-3 cells/ mm3
A 70 year old lady who has never smoked is identified as having a peripherally sited mass in her left lung. What is the most likely underlying diagnosis?
Small cell lung cancer
Gastric cancer metastasis
Adenocarcinoma
Squamous cell carcinoma
Lymphoma
Adenocarcinomas are the most common tumour type present in never smokers. They are usually located at the periphery.
Lung cancer
Lung cancers may be classified according to histological subtypes. The main distinction is between small cell and non small cell lung cancer. Non small cell lung cancer is the most common variant and accounts for 80% of all lung cancers.
Non small cell lung cancer
These share common features of prognosis and management. They comprise the following tumours:
Squamous cell carcinoma (25% cases)
Adenocarcinoma (40% cases)
Large cell carcinoma (10% cases)
Paraneoplastic features and early disease dissemination are less likely than with small cell lung carcinoma. Adenocarcinoma is the most common lung cancer type encountered in never smokers.
Small cell lung carcinoma
Small cell lung carcinomas are comprised of cells with a neuro endocrine differentiation. The neuroendocrine hormones may be released from these cells with a wide range of paraneoplastic associations. These tumours are strongly associated with smoking and will typically arise in the larger airways. They disseminate early in the course of the disease and although they are usually chemosensitive this seldom results in long lasting remissions.
A 45 year old man presents with bilateral inguinal hernias. The surgical team decide to repair these laparoscopically through an extraperitoneal approach. Through an infraumbilical incision the surgeons displace the inferior aspect of the rectus abdominis muscle anteriorly and place a prosthetic mesh into the area to repair the hernias. Which structure will lie posterior to the mesh?
Peritoneum
Internal oblique aponeurosis
External oblique aponeurosis
Posterior aspect of the rectus sheath
Bucks fascia
During a TEP repair of inguinal hernia the only structure to lie posterior to the mesh is peritoneum. The question is really only asking which structure lies posterior to the rectus abdominis muscle. Since this region is below the arcuate line, the transversalis fascia and peritoneum lie posterior to it. Bucks fascia lies in the penis.
Rectus abdominis muscle
The rectus sheath is formed by the aponeuroses of the lateral abdominal wall muscles. The rectus sheath has a composition that varies according to anatomical level.
- Above the costal margin the anterior sheath is composed of external oblique aponeurosis, the costal cartilages are posterior to it.
- From the costal margin to the arcuate line, the anterior rectus sheath is composed of external oblique aponeurosis and the anterior part of the internal oblique aponeurosis. The posterior part of the internal oblique aponeurosis and transversus abdominis form the posterior rectus sheath.
- Below the arcuate line the aponeuroses of all the abdominal muscles lie in anterior aspect of the rectus sheath. Posteriorly lies the transversalis fascia and peritoneum.
The arcuate line is the point at which the inferior epigastric vessels enter the rectus sheath.
A 3-month-old boy presents to your outpatient clinic as the parents describe seeing clear fluid discharge from the umbilicus. This has been noted since soon after the umbilical cord separated. Which one of the following is the most likely underlying (embryological) explanation?
A patent urachus
A patent vitello-intestinal duct
A urachal cyst
An exomphalos
An umbilical granuloma
After separation of the cord there should not be any discharge from the umbilicus. The presence of clear fluid points towards this being urine rather than bowel content coming from the umbilicus. A patent urachus implies a persistent collection to the bladder, allowing small amounts of urine to leak but the patient often remains completely well.
A patent urachus - correct
A patent vitello-intestinal duct - you would expect bowel content, not clear fluid with a connection to the bowel
A urachal cyst - a urachal cyst typically will not drain as the urachal tract is obliterated on either side of the cyst
An exomphalos - this is an abdominal wall defect with the bowel covered by the cord structures and would be identified soon after birth
An umbilical granuloma - a granuloma may leak small amounts of fluid but typically is more serosanguinous
The correct answer is:
A patent urachus
A 40-year-old man presents with clinical and endocrinological features of acromegaly. A microadenoma of the pituitary gland is identified. Which one of the following is the most compelling reason for treating this tumour?
Avoidance of cardiovascular complications
Avoidance of colonic cancer
Avoidance of diabetes
Avoidance of optic pathway compression
Avoidance of radiological progression
The most compelling reason for treating a pituitary microadenoma causing acromegaly is:
Avoidance of cardiovascular complications
Explanation:
Acromegaly is associated with significant morbidity and mortality due to its systemic effects. Cardiovascular complications are the leading cause of death in patients with acromegaly, primarily due to hypertension, cardiomyopathy, and other related issues. Effective treatment of the pituitary adenoma can reduce the excess production of growth hormone and mitigate these severe cardiovascular risks.
• Avoidance of colonic cancer: While there is an increased risk of colonic polyps and cancer in acromegaly patients, it is not the most immediate or compelling reason for treatment compared to cardiovascular risks. • Avoidance of diabetes: Diabetes is a common complication of acromegaly due to growth hormone-induced insulin resistance, but cardiovascular complications are more critical. • Avoidance of optic pathway compression: This is a concern for larger pituitary adenomas (macroadenomas) that can compress adjacent structures. Microadenomas (less than 10 mm in size) typically do not cause optic pathway compression. • Avoidance of radiological progression: Preventing tumor growth is important, but the primary focus is on mitigating the clinical consequences of hormone excess, particularly cardiovascular risks.
Therefore, the most compelling reason for treating a pituitary microadenoma in a patient with acromegaly is to avoid cardiovascular complications, which have the most significant impact on morbidity and mortality.
Whilst on call for orthopaedics you diagnose a child with septic arthritis and you are to assist your consultant in theatre performing a washout of the hip joint.
The approach used is the anterior approach between the rectus femoris and gluteus medius muscles. Which one of the following inter-nervous planes is used in this approach?
Direct rami of the lumbar plexus and the inferior gluteal nerve
Femoral nerve and superior gluteal nerve
Lateral cutaneous nerve and inferior gluteal nerve
Lateral cutaneous nerve and super gluteal nerve
Super gluteal nerve and inferior gluteal nerve
The approach described for the washout of the hip joint in septic arthritis involves the anterior approach between the rectus femoris and the gluteus medius muscles. This approach utilizes an internervous plane to minimize the risk of nerve damage.
The correct internervous plane for this approach is:
Femoral nerve and superior gluteal nerve
Explanation:
• The femoral nerve innervates the rectus femoris muscle. • The superior gluteal nerve innervates the gluteus medius muscle.
Thus, the correct answer is:
Femoral nerve and superior gluteal nerve
Which of the following visceral anastomoses has the lowest risk of anastomotic leak? You may assume that all are constructed in ideal circumstances.
Stapled ileocolic anastomosis
Hand sewn anastomosis of the proximal ileum
Stapled colorectal anastomosis defunctioned with loop ileostomy
Stapled colorectal anastomosis defunctioned with loop colostomy
Hand sewn oesophagojejunal anastomosis
Among the visceral anastomoses listed, the one with the lowest risk of anastomotic leak, assuming all are constructed in ideal circumstances, is generally considered to be:
Stapled ileocolic anastomosis
Explanation:
• Stapled ileocolic anastomosis typically has a lower risk of anastomotic leak compared to other types of anastomoses because the ileum and colon are relatively well-vascularized and less prone to tension and technical difficulties. • Hand-sewn anastomoses, particularly in complex areas like the oesophagojejunal junction, carry higher risks due to the technical challenges and the poor vascularity of the tissues involved. • Stapled colorectal anastomoses, even when defunctioned with loop ileostomy or loop colostomy, have higher risks of leaks compared to ileocolic anastomoses because the rectal area is more prone to complications due to factors like tension, poor blood supply, and potential for contamination.
Therefore, the correct answer is:
Stapled ileocolic anastomosis
For a visceral anastomosis to heal, three factors need to be satisfied:
1. Mucosal- mucosal apposition
2. A good blood supply
3. No tension
The more these are compromised, the higher the leak rate.
Rectal and oesophageal surgery have some of the highest rates of anastomotic leakage. Following anterior resection leaks are quoted to occur in up to 10% of cases. Small bowel anastomoses are the most technically forgiving. Factors increasing the risk of anastamotic leakage include previous irradiation, sepsis, malnutrition, poor blood supply and poor technique.
The defunctioning of rectal anastomoses may reduce the clinical impact of anastomotic leak and make it amenable to percutaneous drainage, but does not necessarily reduce the incidence of leaks themselves.
An 18 year old man is undergoing an orchidectomy via a scrotal approach. The surgeons mobilise the spermatic cord. From which of the following is the outermost layer of this structure derived?
Internal oblique aponeurosis
External oblique aponeurosis
Transversalis fascia
Rectus sheath
Campers fascia
The outermost covering of the spermatic cord is derived from the external oblique aponeurosis.This layer is added as the cord passes through the superficial inguinal ring. Scrotal and testicular anatomy
Spermatic cord
Formed by the vas deferens and is covered by the following structures:
Layer Origin
Internal spermatic fascia Transversalis fascia
Cremasteric fascia From the fascial coverings of internal oblique
External spermatic fascia External oblique aponeurosis
Contents of the cord
Vas deferens Transmits sperm and accessory gland secretions
Testicular artery Branch of abdominal aorta supplies testis and epididymis
Artery of vas deferens Arises from inferior vesical artery
Cremasteric artery Arises from inferior epigastric artery
Pampiniform plexus Venous plexus, drains into right or left testicular vein
Sympathetic nerve fibres Lie on arteries, the parasympathetic fibres lie on the vas
Genital branch of the genitofemoral nerve Supplies cremaster
Lymphatic vessels Drain to lumbar and para-aortic nodes
Scrotum
Composed of skin and closely attached dartos fascia.
Arterial supply from the anterior and posterior scrotal arteries
Lymphatic drainage to the inguinal lymph nodes
Parietal layer of the tunica vaginalis is the innermost layer
Testes
The testes are surrounded by the tunica vaginalis (closed peritoneal sac). The parietal layer of the tunica vaginalis adjacent to the internal spermatic fascia.
The testicular arteries arise from the aorta immediately inferiorly to the renal arteries.
The pampiniform plexus drains into the testicular veins, the left drains into the left renal vein and the right into the inferior vena cava.
Lymphatic drainage is to the para-aortic nodes.
A term neonate is born by emergency cesarean section the final stages of pregnancy having been complicated by polyhydramnios. Feeding results in vomiting and a imaging is performed which demonstrates extrinsic compression of the second part of the duodenum. What is the most likely embryological origin of the structure that is causing this compression?
The dorsal pancreas
The hepatic diverticulum
The vitellointestinal duct
Ladds bands
The ventral pancreas
The obstruction is most likely to be the result of an annular pancreas. The dorsal and ventral pancreas structures initially lie opposite each other. However, the ventral pancreas rotates to eventually fuse with the dorsal. Sometimes this can result in an annular pancreas and some patients can develop obstruction as a result.
Pancreatic embryology
The pancreas develops from a ventral and dorsal entodermal outgrowth from the duodenum. The ventral arises close to, or in common with, the hepatic diverticulum, and the larger, dorsal outgrowth arises slightly cranial to the ventral and extends into the mesoduodenum and dorsal mesogastrium. As a result of the differential growth in the duodenal wall, the point of attachment of the ventral outgrowth with the undivided stem of the hepatic diverticulum and the ventral mesentery are displaced around the right hand side of the duodenum to a dorsal position. As a result, the two pancreatic remnants eventually lie together and then become fused. In this process the ventral rudiment forms the uncinate process and the adjacent region of the head, while the dorsal forms the anterior part of the head, the body and the tail of the pancreas.
At the time of fusion, the ducts become joined. This occurs in a manner that results in the dorsal remnant being drained by the duct of the ventral remnant, this composite duct being named the pancreatic duct.
A 54-year-old man who has end stage diabetic nephropathy is being assessed for a renal transplant. When assessing the HLA matching between donor and recipient what is the most important HLA antigen to match?
DP
B
DR
C
A
Renal transplant HLA matching - DR is the most important. Renal transplant:HLA typing and graft failure
The human leucocyte antigen (HLA) system is the name given to the major histocompatibility complex (MHC) in humans. It is coded for on chromosome 6.
Some basic points on the HLA system
Class 1 antigens include A, B and C. Class 2 antigens include DP,DQ and DR
When HLA matching for a renal transplant the relative importance of the HLA antigens are as follows DR > B > A
Graft survival
1 year = 90%, 10 years = 60% for cadaveric transplants
1 year = 95%, 10 years = 70% for living-donor transplants
Post-op problems
ATN of graft
Vascular thrombosis
Urine leakage
UTI
Hyperacute acute rejection
Due to antibodies against donor HLA type 1 antigens
Rarely seen due to HLA matching
Acute graft failure (< 6 months)
Usually due to mismatched HLA
Other causes include cytomegalovirus infection
Management: give steroids, if resistant use monoclonal antibodies
Causes of chronic graft failure (> 6 months)
Chronic allograft nephropathy
Ureteric obstruction
Recurrence of original renal disease (MCGN > IgA > FSGS)
A 45 year old man is referred to the breast clinic with gynaecomastia. He takes the drugs listed below. Which is least likely to be the cause of his symptoms?
Spironolactone
Carbimazole
Chlorpromazine
Cimetidine
Methyldopa
Mnemonic for drugs causing gynaecomastia: DISCO
D igitalis
I soniazid
S pironolactone
C imetidine
O estrogen
Mnemonic for causes of gynaecomastia: METOCLOPRAMIDE
M etoclopramide
E ctopic oestrogen
T rauma skull/tumour breast, testes
O rchitis
C imetidine, Cushings
L iver cirrhosis
O besity
P araplegia
R A
A cromegaly
M ethyldopa
I soniazid
D igoxin
E thionamide
Carbimazole is not associated with gynaecomastia. Note the question asks for the least likely cause.
Gynaecomastia
Gynaecomastia describes an abnormal amount of breast tissue in males and is usually caused by an increased oestrogen:androgen ratio. It is important to differentiate the causes of galactorrhoea (due to the actions of prolactin on breast tissue) from those of gynaecomastia
Causes of gynaecomastia
physiological: normal in puberty
syndromes with androgen deficiency: Kallman’s, Klinefelter’s
testicular failure: e.g. Mumps
liver disease
testicular cancer e.g. Seminoma secreting HCG
ectopic tumour secretion
hyperthyroidism
haemodialysis
drugs: see below
Drug causes of gynaecomastia
spironolactone (most common drug cause)
cimetidine
digoxin
cannabis
finasteride
oestrogens, anabolic steroids
Very rare drug causes of gynaecomastia
tricyclics
isoniazid
calcium channel blockers
heroin
busulfan
methyldopa
Treatment options
identify and manage any overt underlying cause
liposuction provides the best cosmetic outcome
A 40 year old man undergoes a complex appendicectomy and the wound fails to heal satisfactorily. The wound site itself is associated with multiple sinuses and fistulas. Pus is sent for microbiology and shows gram positive organisms and sulphur granules. What is the most likely underlying diagnosis?
Infection with Staphylococcus aureus
Infection with Bacteroides fragilis
Actinomycosis
Crohns disease
Ulcerative colitis
The presence of chronic sinuses together with gram positive organisms and sulphur granules is highly suggestive of Actinomycosis. Crohns disease is associated with multiple fistulae, but not gram positive organisms with sulphur granules. Actinomycosis
Chronic, progressive granulomatous disease caused by filamentous gram positive anaerobic bacteria from the Actinomycetaceae family.
Actinomyces are commensal bacteria that become pathogenic when a mucosal barrier is breached.
The disease most commonly occurs in the head and neck, although it may also occur in the abdominal cavity and in the thorax.
The mass will often enlarge across tissue planes with the formation of multiple sinus tracts.
Abdominopelvic actinomycosis occurs most frequently in individuals that have had appendicitis (65%) cases.
Pathology
On histological examination gram positive organisms and evidence of sulphur granules.
Sulphur granules are colonies of organisms that appear as round or oval basophilic masses.
They are also seen in other conditions such as nocardiosis.
Treatment
Long term antibiotic therapy usually with penicillin.
Surgical resection is indicated for extensive necrotic tissue, non healing sinus tracts, abscesses or where biopsy is needed to exclude malignancy.
References
Wong V, Turmezei T and Weston V. Actinomycosis. BMJ 2011;343d6099.
A 48 year old lady is undergoing a left sided adrenalectomy for an adrenal adenoma. The superior adrenal artery is injured and starts to bleed, from which of the following does this vessel arise?
Left renal artery
Inferior phrenic artery
Aorta
Splenic
None of the above
The superior adrenal artery is a branch of the inferior phrenic artery.
Adrenal gland anatomy
Location Superomedially to the upper pole of each kidney
Relationships of the right adrenal Diaphragm-Posteriorly, Kidney-Inferiorly, Vena Cava-Medially, Hepato-renal pouch and bare area of the liver-Anteriorly
Relationships of the left adrenal Crus of the diaphragm-Postero- medially, Pancreas and splenic vessels-Inferiorly, Lesser sac and stomach-Anteriorly
Arterial supply Superior adrenal arteries- from inferior phrenic artery, Middle adrenal arteries - from aorta, Inferior adrenal arteries -from renal arteries
Venous drainage of the right adrenal Via one central vein directly into the IVC
Venous drainage of the left adrenal Via one central vein into the left renal vein
A 73 year old lady is admitted with small bowel obstruction and following surgery is diagnosed as having diaphragm disease. Which of the agents listed below is the most likely precipitant?
Alcohol abuse
Oral non steroidal anti inflammatory drugs
Abdominal radiotherapy
Oral steroids
Chronic laxative misuse
Diaphragm disease, which is characterized by multiple thin, diaphragm-like strictures in the small intestine, is most commonly associated with:
Oral non-steroidal anti-inflammatory drugs (NSAIDs)
Explanation:
• Oral NSAIDs are well-documented causes of diaphragm disease in the small intestine. NSAIDs can lead to mucosal injury, resulting in the formation of strictures and characteristic diaphragm-like lesions. • Alcohol abuse, abdominal radiotherapy, oral steroids, and chronic laxative misuse are not typically associated with diaphragm disease.
Therefore, the correct answer is:
Oral non-steroidal anti-inflammatory drugs
A 65 year old man presents with a new onset left sided hemiparesis. A CT scan of the brain is performed and this demonstrates a 4cm lesion within the right frontal lobe, it traverses the midline and displays marked central necrosis. There is extensive oedema surrounding the lesion. What is the most likely diagnosis?
Meningioma
Central neurocytoma
Ependymoma
Oligodendroglioma
Glioblastoma
Glioblastomas typically display such marked appearances and an important differential on imaging is one of cerebral abscess. A meningioma would not display such infiltrative behavior. Significant necrosis is more commonly seen with glioblastomas than with other CNS tumours. CNS tumours
60% = Glioma and metastatic disease
20% = Meningioma
10% = Pituitary lesions
In paediatric practice medulloblastomas (neuroectodermal tumours) were the commonest lesions, astrocytomas now account for the majority.
Tumours arising in right temporal and frontal lobe may reach considerable size before becoming symptomatic. Whereas tumours in the speech and visual areas will typically produce early symptoms.
Most paediatric CNS tumours are infratentorial
Most adult CNS tumours are supratentorial
Diagnosis
MRI Scanning provides the best resolution.
Treatment
Usually surgery, even if tumour cannot be completely resected conditions such as rising ICP can be addressed with tumour debulking and survival and quality of life prolonged.
Curative surgery can usually be undertaken with lesions such as meningiomas. Gliomas have a marked propensity to invade normal brain and resection of these lesions is nearly always incomplete.
A patient is seen in clinic complaining of abdominal pain. Routine bloods show:
Na+ 142 mmol/l
K+ 4.0 mmol/l
Chloride 104 mmol/l
Bicarbonate 19 mmol/l
Urea 7.0 mmol/l
Creatinine 112 µmol/l
What is the anion gap?
4 mmol/L
14 mmol/L
20 mmol/L
21 mmol/L
23 mmol/L
The anion gap may be calculated by using (sodium + potassium) - (bicarbonate + chloride)
= (142 + 4.0) - (104 + 19) = 23 mmol/L
Anion gap
The anion gap is calculated by:
(sodium + potassium) - (bicarbonate + chloride)
A normal anion gap is 8-14 mmol/L
It is useful to consider in patients with a metabolic acidosis:
Causes of a normal anion gap or hyperchloraemic metabolic acidosis
gastrointestinal bicarbonate loss: diarrhoea, ureterosigmoidostomy, fistula
renal tubular acidosis
drugs: e.g. acetazolamide
ammonium chloride injection
Addison’s disease
Causes of a raised anion gap metabolic acidosis
lactate: shock, hypoxia
ketones: diabetic ketoacidosis, alcohol
urate: renal failure
acid poisoning: salicylates, methanol
A 52 year old lady presents to the surgical clinic with a goitre. She is taking medication for an underlying psychiatric disorder. Which of the drugs listed below is most likely to be responsible?
Haloperidol
Imipramine
Amytryptiline
Lithium
Venlafaxine
A significant proportion of patients who take lithium treatment will develop a goitre and a number will become clinically hypothyroid. For this reason, monitoring of thyroid function tests is necessary during lithium therapy. Thyroid disease
Patients may present with a number of different manifestations of thyroid disease. They can be broadly sub classified according to whether they are euthyroid or have clinical signs of thyroid dysfunction. In addition it needs to be established whether they have a mass or not.
Assessment
History
Examination including USS
If a nodule is identified then it should be sampled ideally via an image guided fine needle aspiration
Radionucleotide scanning is of limited use
Thyroid Tumours
Papillary carcinoma
Follicular carcinoma
Anaplastic carcinoma
Medullary carcinoma
Lymphoma’s
Multinodular goitre
One of the most common reasons for presentation
Provided the patient is euthyroid and asymptomatic and no discrete nodules are seen, they can be reassured.
In those with compressive symptoms surgery is required and the best operation is a total thyroidectomy.
Sub total resections were practised in the past and simply result in recurrent disease that requires a difficult revisional resection.
Endocrine dysfunction
In general these patients are managed by physicians initially.
Surgery may be offered alongside radio iodine for patients with Graves disease that fails with medical management or in patients who would prefer not to be irradiated (e.g. pregnant women).
Patients with hypothyroidism do not generally get offered a thyroidectomy. Sometimes people inadvertently get offered resections during the early phase of Hashimotos thyroiditis, however, with time the toxic phase passes and patients can simply be managed with thyroxine.
Complications following surgery
Anatomical such as recurrent laryngeal nerve damage.
Bleeding. Owing to the confined space haematoma’s may rapidly lead to respiratory compromise owing to laryngeal oedema.
Damage to the parathyroid glands resulting in hypocalcaemia.
Further sources of information
1. http://www.acb.org.uk/docs/TFTguidelinefinal.pdf- Association of Clinical Biochemistry guidelines for thyroid function tests.
- British association of endocrine surgeons website- http://www.baets.org.uk
Which of the following muscles inserts onto the lesser tuberosity of the humerus?
Subscapularis
Deltoid
Supraspinatus
Teres minor
Infraspinatus
With the exception of subscapularis which inserts into the lesser tuberosity, the muscles of the rotator cuff insert into the greater tuberosity.
Shoulder joint
Shallow synovial ball and socket type of joint.
It is an inherently unstable joint, but is capable to a wide range of movement.
Stability is provided by muscles of the rotator cuff that pass from the scapula to insert in the greater tuberosity (all except sub scapularis-lesser tuberosity).
Glenoid labrum
Fibrocartilaginous rim attached to the free edge of the glenoid cavity
Tendon of the long head of biceps arises from within the joint from the supraglenoid tubercle, and is fused at this point to the labrum.
The long head of triceps attaches to the infraglenoid tubercle
Fibrous capsule
Attaches to the scapula external to the glenoid labrum and to the labrum itself (postero-superiorly)
Attaches to the humerus at the level of the anatomical neck superiorly and the surgical neck inferiorly
Anteriorly the capsule is in contact with the tendon of subscapularis, superiorly with the supraspinatus tendon, and posteriorly with the tendons of infraspinatus and teres minor. All these blend with the capsule towards their insertion.
Two defects in the fibrous capsule; superiorly for the tendon of biceps. Anteriorly there is a defect beneath the subscapularis tendon.
The inferior extension of the capsule is closely related to the axillary nerve at the surgical neck and this nerve is at risk in anteroinferior dislocations. It also means that proximally sited osteomyelitis may progress to septic arthritis.
Movements and muscles
Flexion Anterior part of deltoid
Pectoralis major
Biceps
Coracobrachialis
Extension Posterior deltoid
Teres major
Latissimus dorsi
Adduction Pectoralis major
Latissimus dorsi
Teres major
Coracobrachialis
Abduction Mid deltoid
Supraspinatus
Medial rotation Subscapularis
Anterior deltoid
Teres major
Latissimus dorsi
Lateral rotation Posterior deltoid
Infraspinatus
Teres minor
Important anatomical relations
Anteriorly Brachial plexus
Axillary artery and vein
Posterior Suprascapular nerve
Suprascapular vessels
Inferior Axillary nerve
Circumflex humeral vessels
A 22 year old man is referred to the surgical clinic. He has been complaining of varicose veins for many years. On examination he has extensive varicosities of the right leg, there are areas of marked port wine staining. The saphenofemoral junction is competent on doppler assessment. The most likely underlying diagnosis is:
Deep vein thrombosis
Klippel-Trenaunay syndrome
Varicose veins due to sapheno-popliteal junction incompetence
Sturge - Weber syndrome
Angiosarcoma
Sturge - Weber syndrome is a an arteriovenous malformation affecting the face and CNS, the peripheral vessels are not affected. Simple varicose veins should not typically be associated with port wine staining, nor should a DVT or angiosarcoma.
Klippel-Trenaunay syndrome
Klippel-Trenaunay-Weber syndrome generally affects a single extremity, although cases of multiple affected limbs have been reported. The leg is the most common site followed by the arms, the trunk, and rarely the head and the neck
Signs and symptoms
The birth defect is diagnosed by the presence of a combination of these symptoms:
One or more distinctive port-wine stains with sharp borders
Varicose veins
Hypertrophy of bony and soft tissues, that may lead to local gigantism or shrinking.
An improperly developed lymphatic system
In some cases, port-wine stains (capillary port wine type) may be absent. Such cases are very rare and may be classified as ‘atypical Klippel-Trenaunay syndrome’.
KTS can either affect blood vessels, lymph vessels, or both. The condition most commonly presents with a mixture of the two. Those with venous involvement experience increased pain and complications.
What is the mechanism of action of macrolides?
Causes misreading of mRNA
Interferes with cell wall formation
Inhibits DNA synthesis
Inhibits RNA synthesis
Inhibits protein synthesis
Macrolides act by inhibiting bacterial protein synthesis. If pushed to give an answer they are bacteriostatic in nature, but in reality this depends on the dose and type of organism being treated. Erythromycin was the first macrolide used clinically. Newer examples include clarithromycin and azithromycin.
Adverse effects of erythromycin
gastrointestinal side-effects are common
cholestatic jaundice: risk may be reduced if erythromycin stearate is used
P450 inhibitor
Antibiotics: mechanism of action
The lists below summarise the site of action of the commonly used antibiotics
Inhibit cell wall formation
penicillins
cephalosporins
Inhibit protein synthesis
aminoglycosides (cause misreading of mRNA)
chloramphenicol
macrolides (e.g. erythromycin)
tetracyclines
fusidic acid
Inhibit DNA synthesis
quinolones (e.g. ciprofloxacin)
metronidazole
sulphonamides
trimethoprim
Inhibit RNA synthesis
rifampicin
You are performing a study of weight in patients attending pre operative assessment clinic for elective knee replacement. Assuming that the results are normally distributed, what percentage of values lie within two standard deviations of the mean weight?
95.4%
5.3%
98.3%
10%
97.5%
In statistics, the 68-95-99.7 rule or three-sigma rule, or empirical rule states that for a normal distribution, nearly all values lie within 3 standard deviations of the mean.
About 68.27% of the values lie within 1 standard deviation of the mean. Similarly, about 95.45% of the values lie within 2 standard deviations of the mean. Nearly all (99.73%) of the values lie within 3 standard deviations of the mean
95.4% of values lie within 2 SD of the mean.
What is the longest part of the male urethra?
Membranous urethra
Spongy urethra
Prostatic urethra
Urethra within the internal urethra orifice
Urethra within the urethral crest
The spongy urethra is around 15cm long and is the longest part of the male urethra. Urethral anatomy
Female urethra
The female urethra is shorter and more acutely angulated than the male urethra. It is an extra-peritoneal structure and embedded in the endopelvic fascia. The neck of the bladder is subjected to transmitted intra-abdominal pressure and therefore deficiency in this area may result in stress urinary incontinence. Between the layers of the urogenital diaphragm the female urethra is surrounded by the external urethral sphincter, this is innervated by the pudendal nerve. It ultimately lies anterior to the vaginal orifice.
Male urethra
In males the urethra is much longer and is divided into four parts.
Pre-prostatic urethra Extremely short and lies between the bladder and prostate gland.It has a stellate lumen and is between 1 and 1.5cm long.Innervated by sympathetic noradrenergic fibres, as this region is composed of striated muscles bundles they may contract and prevent retrograde ejaculation.
Prostatic urethra This segment is wider than the membranous urethra and contains several openings for the transmission of semen (at the midpoint of the urethral crest).
Membranous urethra Narrowest part of the urethra and surrounded by external sphincter. It traverses the perineal membrane 2.5cm postero-inferior to the symphysis pubis.
Penile urethra Travels through the corpus spongiosum on the underside of the penis. It is the longest urethral segment.It is dilated at its origin as the infrabulbar fossa and again in the glans penis as the navicular fossa. The bulbo-urethral glands open into the spongiose section of the urethra 2.5cm below the perineal membrane.
The urothelium is transitional in nature near to the bladder and becomes squamous more distally.
Which one of the following serum proteins is most likely to increase in a patient with severe sepsis?
Transferrin
Transthyretin
Ferritin
Albumin
Cortisol binding protein
Ferritin can be markedly increased during acute illness. The other parameters tend to decrease during an acute phase response. Acute phase proteins
Acute phase proteins
CRP
procalcitonin
ferritin
fibrinogen
alpha-1 antitrypsin
caeruloplasmin
serum amyloid A
haptoglobin
complement
During the acute phase response the liver decreases the production of other proteins (sometimes referred to as negative acute phase proteins). Examples include:
albumin
transthyretin (formerly known as prealbumin)
transferrin
retinol binding protein
cortisol binding protein
Levels of CRP are commonly measured in acutely unwell patients. CRP is a protein synthesised in the liver and binds to phosphocholine in bacterial cells and on those cells undergoing apoptosis. In binding to these cells it is then able to activate the complement system. CRP levels are known to rise in patients following surgery. However, levels of greater than 150 at 48 hours post operatively are suggestive of evolving complications.
A 73 year old man undergoes a right below knee amputation for end stage peripheral vascular disease. He is reviewed in the clinic 8 weeks post operatively and complains of a persistent, burning discomfort over his amputation site stump. On examination, his wound has healed and proximal pulses have a biphasic signal on doppler ultrasound. What is the most appropriate management?
Commence amitryptyline
Commence fentanyl patch
Arrange duplex scan
Arrange MRI scan of the stump
Commence carbamazepine
This patient has neuropathic pain. Amitryptyline is the treatment of choice. Carbamazepine is mainly used for trigeminal neuralgia.
Neuropathic pain
Neuropathic pain may be defined as pain which arises following damage or disruption of the nervous system. It is often difficult to treat and responds poorly to standard analgesia.
Examples include:
diabetic neuropathy
post-herpetic neuralgia
trigeminal neuralgia
prolapsed intervertebral disc
NICE issued guidance in 2010 on the management of neuropathic pain:
first-line treatment*: oral amitriptyline or pregabalin
if satisfactory pain reduction is obtained with amitriptyline but the person cannot tolerate the adverse effects, consider oral imipramine or nortriptyline as an alternative
second-line treatment: if first-line treatment was with amitriptyline, switch to or combine with pregabalin. If first-line treatment was with pregabalin, switch to or
combine with amitriptyline
other options: pain management clinic, tramadol (not other strong opioids), topical lidocaine for localised pain if patients unable to take oral medication
*please note that for some specific conditions the guidance may vary. For example carbamazepine is used first-line for trigeminal neuralgia, duloxetine for diabetic neuropathy
A 20 year old man is involved in a road traffic accident. Following the incident he is unable to extend his wrist. However, this improves over the following weeks. Which type of injury is he most likely to have sustained?
Radial nerve neurotmesis
Radial nerve neuropraxia
Axillary nerve axonotmesis
Ulnar nerve neuropraxia
Ulnar nerve axonotmesis
The injury described—initial inability to extend the wrist following a road traffic accident, which then improves over the following weeks—is most consistent with a radial nerve neuropraxia.
Neuropraxia is a type of nerve injury characterized by a temporary loss of motor and sensory function due to blockage of nerve conduction. This is often caused by compression or blunt trauma, and typically, there is no physical damage to the nerve itself. Recovery is usually complete and occurs within days to weeks.
Here’s why the other options are less likely:
• Radial nerve neurotmesis: This is a more severe form of nerve injury where the nerve is completely severed. Recovery typically requires surgical intervention and takes much longer, with less chance of full recovery. • Axillary nerve axonotmesis: This involves damage to the axillary nerve and the nerve fibers (axons), but the surrounding connective tissue remains intact. It would not typically result in wrist drop, as the axillary nerve primarily innervates the deltoid muscle and the teres minor muscle. • Ulnar nerve neuropraxia: This would affect the ulnar nerve, which primarily controls muscles in the forearm and hand. It would not cause wrist drop, which is specifically related to the radial nerve. • Ulnar nerve axonotmesis: Similar to the above, this involves damage to the ulnar nerve and would not result in wrist drop.
Therefore, the most likely injury is radial nerve neuropraxia.
Neuropraxia
Nerve intact but electrical conduction is affected
Myelin sheath integrity is preserved
Full recovery
Autonomic function preserved
Wallerian degeneration does not occur
A 50 year old male who weighs 70Kg is severely unwell with intestinal failure and cannot receive enteral nutrition. He has become malnourished and the decision is made to commence total parenteral nutrition. What is the most appropriate initial dose of calories which should be administered via this route over 24 hours?
700 Kcal
2000 Kcal
3000 Kcal
500 Kcal
2500 Kcal
The appropriate initial dose of calories to be administered via total parenteral nutrition (TPN) for a severely unwell, malnourished 50-year-old male weighing 70 kg would be:
2000 Kcal
This recommendation aligns with standard clinical practice for initial TPN administration, where the caloric intake typically ranges from 25 to 35 kcal/kg/day, starting at the lower end for critically ill or malnourished patients and gradually increasing as tolerated.
Explanation:
TPN is used to provide nutrition intravenously when enteral feeding is not possible. The initial caloric needs are calculated based on the patient’s weight and clinical condition. For a 70 kg male:
• Minimum requirement: 70 \text{ kg} \times 25 \text{ kcal/kg/day} = 1750 \text{ kcal/day} • Optimal requirement: 70 \text{ kg} \times 30 \text{ kcal/kg/day} = 2100 \text{ kcal/day}
Thus, starting at 2000 kcal/day is a reasonable and appropriate initial dose.
As this person is severely ill and malnourished, the initial calories administered is reduced by 50% in the first few days.
Given that the initial caloric intake should be reduced by 50% in the first few days for a patient starting on total parenteral nutrition (TPN), the most appropriate initial dose of calories for this patient would be:
1000 Kcal
Explanation:
If we consider that the optimal caloric requirement for a 70 kg male is approximately 2000 kcal/day (as calculated previously), then reducing this by 50% for the initial period would result in an initial dose of:
2000 \text{ kcal/day} \times 0.5 = 1000 \text{ kcal/day}
Therefore, 1000 kcal/day is a safer starting dose to prevent complications such as refeeding syndrome. However, since 1000 kcal/day was not one of the options provided, the closest and most appropriate option would be:
700 Kcal
This represents a conservative approach to initiate TPN and gradually increase the caloric intake as the patient stabilizes.
Nutrition prescriptions
National institute of clinical excellence (NICE) guidelines
For people not severely ill and not at risk of refeeding syndrome aim to give
25-35 kcal/kg/day (lower if BMI > 25)
0.8-1.5g protein /kg/day
30-35 ml fluid/kg/day
Adequate electrolytes, minerals, vitamins
Severely ill patients aim to give < 50% of the energy and protein levels over the first 24-48h.
For people at high risk of refeeding syndrome:
Start at up to 10 kcal/kg/day increasing to full needs over 4-7 days
Start immediately before and during feeding: oral thiamine 200-300mg/day, vitamin B co strong 1 tds and supplements
Give K+ (2-4 mmol/kg/day), phosphate (0.3-0.6 mmol/kg/day), magnesium (0.2-0.4 mmol/kg/day)
A 24 year old man presents with symptoms of malaise, weight loss and lymphadenopathy. A lymph node biopsy is performed and the subsequent histology report states that there is evidence of granuloma formation and central necrosis. What is the most likely underlying cause?
Non Hodgkins lymphoma
Churg Strauss syndrome
Epstein Barr Virus infection
Rheumatoid nodule
Infection with Mycobacterium tuberculosis
These histological features are typically seen in TB. Necrosis occurring in granulomas is usually indicative of an underlying infective cause. Churg Strauss syndrome is a form of vasculitis, which is the usual histological finding. Granulomas are reported in the condition, but it is rare for them to demonstrate necrosis. Tuberculosis pathology
Is a form of primary chronic inflammation, caused by the inability of macrophages to kill the Mycobacterium tuberculosis.
The macrophages often migrate to regional lymph nodes, the lung lesion plus affected lymph nodes is referred to as a Ghon complex.
This leads to the formation of a granuloma which is a collection of epithelioid histiocytes.
There is the presence of caseous necrosis in the centre.
The inflammatory response is mediated by a type 4 hypersensitivity reaction.
In healthy individuals the disease may be contained, in the immunocompromised disseminated (miliary TB) may occur.
Diagnosis
Waxy membrane of mycobacteria prevents binding with normal stains. Ziehl - Neelsen staining is typically used.
Culture based methods take far longer.
Image showing acid- alcohol fast mycobacteria stained using the Ziehl- Neelsen method
A 7 year old boy falls off a wall the distance is 7 feet. He lands on his left side and there is left flank bruising. There is no haematuria. He is otherwise stable and haemoglobin is within normal limits. What is the most appropriate initial course of action?
Undertake a CT scan of the abdomen
Undertake an abdominal USS
Undertake diagnostic peritoneal lavage
Undertake a splenectomy
Arrange an angiogram and possibly proceed to embolisation
This will demonstrate any overt splenic injury. A CT scan carries a significant dose of radiation. In the absence of haemodynamic instability or other major associated injuries the use of USS to exclude intraabdominal free fluid (blood) would seem safe when coupled with active observation. An USS will also show splenic haematomas. Splenic trauma
- The spleen is one of the more commonly injured intra abdominal organs
In most cases the spleen can be conserved. The management is dictated by the associated injuries, haemodynamic status and extent of direct splenic injury.
Management of splenic trauma
Conservative Small subcapsular haematoma
Minimal intra abdominal blood
No hilar disruption
Laparotomy with conservation Increased amounts of intraabdominal blood
Moderate haemodynamic compromise
Tears or lacerations affecting <50%
Resection Hilar injuries
Major haemorrhage
Major associated injuries
Splenectomy
Technique
Trauma
GA
Long midline incision
If time permits insert a self retaining retractor (e.g. Balfour/ omnitract)
Large amount of free blood is usually present. Pack all 4 quadrants of the abdomen. Allow the anaesthetist to ‘catch up’
Remove the packs and assess the viability of the spleen. Hilar injuries and extensive parenchymal lacerations will usually require splenectomy.
Divide the short gastric vessels and ligate them.
Clamp the splenic artery and vein. Two clamps on the patient side are better and allow for double ligation and serve as a safety net if your assistant does not release the clamp smoothly.
Be careful not to damage the tail of the pancreas, if you do then this will need to be formally removed and the pancreatic duct closed.
Wash out the abdomen and place a tube drain to the splenic bed.
Some surgeons implant a portion of spleen into the omentum, whether you decide to do this is a matter of personal choice.
Post operatively the patient will require prophylactic penicillin V and pneumococcal vaccine.
Elective
Elective splenectomy is a very different operation from that performed in the emergency setting. The spleen is often large (sometimes massive). Most cases can be performed laparoscopically. The spleen will often be macerated inside a specimen bag to facilitate extraction.
Complications
Haemorrhage (may be early and either from short gastrics or splenic hilar vessels
Pancreatic fistula (from iatrogenic damage to pancreatic tail)
Thrombocytosis: prophylactic aspirin
Encapsulated bacteria infection e.g. Strep. pneumoniae, Haemophilus influenzae and Neisseria meningitidis
A 28 year old man is shot in the right chest and develops a right haemothorax necessitating a thoracotomy. The surgeons decide to place a vascular clamp across the hilum of the right lung. Which of the following structures will lie most anteriorly at this point?
Thoracic duct
Phrenic nerve
Vagus nerve
Pulmonary artery
Pulmonary vein
The phrenic nerve lies anteriorly at the root of the right lung.
Lung anatomy
The right lung is composed of 3 lobes divided by the oblique and transverse fissures. The left lung has two lobes divided by the oblique fissure.The apex of both lungs is approximately 4cm superior to the sterno-costal joint of the first rib. Immediately below this is a sulcus created by the subclavian artery.
Peripheral contact points of the lung
Base: diaphragm
Costal surface: corresponds to the cavity of the chest
Mediastinal surface: Contacts the mediastinal pleura. Has the cardiac impression. Above and behind this concavity is a triangular depression named the hilum, where the structures which form the root of the lung enter and leave the viscus. These structures are invested by pleura, which, below the hilum and behind the pericardial impression, forms the pulmonary ligament
Right lung
Above the hilum is the azygos vein; Superior to this is the groove for the superior vena cava and right innominate vein; behind this, and nearer the apex, is a furrow for the innominate artery. Behind the hilum and the attachment of the pulmonary ligament is a vertical groove for the oesophagus; In front and to the right of the lower part of the oesophageal groove is a deep concavity for the extrapericardiac portion of the inferior vena cava.
The root of the right lung lies behind the superior vena cava and the right atrium, and below the azygos vein.
The right main bronchus is shorter, wider and more vertical than the left main bronchus and therefore the route taken by most foreign bodies.
Left lung
Above the hilum is the furrow produced by the aortic arch, and then superiorly the groove accommodating the left subclavian artery; Behind the hilum and pulmonary ligament is a vertical groove produced by the descending aorta, and in front of this, near the base of the lung, is the lower part of the oesophagus.
The root of the left lung passes under the aortic arch and in front of the descending aorta.
Inferior borders of both lungs
6th rib in mid clavicular line
8th rib in mid axillary line
10th rib posteriorly
The pleura runs two ribs lower than the corresponding lung level.
Bronchopulmonary segments
Segment number Right lung Left lung
1 Apical Apical
2 Posterior Posterior
3 Anterior Anterior
4 Lateral Superior lingular
5 Medial Inferior lingular
6 Superior (apical) Superior (apical)
7 Medial basal Medial basal
8 Anterior basal Anterior basal
9 Lateral basal Lateral basal
10 Posterior basal Posterior basal
What is the lymphatic drainage of the female urethra?
Superficial inguinal nodes
Deep inguinal nodes
Internal iliac nodes
External iliac nodes
Para-aortic nodes
The entire female urethra drains to the internal iliac nodes. Urethral anatomy
Female urethra
The female urethra is shorter and more acutely angulated than the male urethra. It is an extra-peritoneal structure and embedded in the endopelvic fascia. The neck of the bladder is subjected to transmitted intra-abdominal pressure and therefore deficiency in this area may result in stress urinary incontinence. Between the layers of the urogenital diaphragm the female urethra is surrounded by the external urethral sphincter, this is innervated by the pudendal nerve. It ultimately lies anterior to the vaginal orifice.
Male urethra
In males the urethra is much longer and is divided into four parts.
Pre-prostatic urethra Extremely short and lies between the bladder and prostate gland.It has a stellate lumen and is between 1 and 1.5cm long.Innervated by sympathetic noradrenergic fibres, as this region is composed of striated muscles bundles they may contract and prevent retrograde ejaculation.
Prostatic urethra This segment is wider than the membranous urethra and contains several openings for the transmission of semen (at the midpoint of the urethral crest).
Membranous urethra Narrowest part of the urethra and surrounded by external sphincter. It traverses the perineal membrane 2.5cm postero-inferior to the symphysis pubis.
Penile urethra Travels through the corpus spongiosum on the underside of the penis. It is the longest urethral segment.It is dilated at its origin as the infrabulbar fossa and again in the glans penis as the navicular fossa. The bulbo-urethral glands open into the spongiose section of the urethra 2.5cm below the perineal membrane.
The urothelium is transitional in nature near to the bladder and becomes squamous more distally.
Which of the following statements relating to omphalocele is false?
The herniated organs lie outside the peritoneal sac
Cardiac abnormalities co-exist in 25%
The caecum is usually right sided
The defect occurs through the umbilicus
Mortality may be as high as 15%
The false statement regarding omphalocele is:
The herniated organs lie outside the peritoneal sac.
Explanation:
Omphalocele is a congenital defect in the abdominal wall where the intestines, liver, and occasionally other organs remain outside the abdomen in a sac because of a failure of the intestines to return to the abdominal cavity during fetal development. The sac is covered by a membrane consisting of peritoneum and amnion.
1. The herniated organs lie outside the peritoneal sac: This is false. In omphalocele, the herniated organs are contained within a protective sac formed by the peritoneum and amnion. 2. Cardiac abnormalities co-exist in 25%: This is true. Omphalocele is often associated with other congenital anomalies, including cardiac defects. 3. The caecum is usually right sided: This is true. The caecum and other abdominal organs are typically in their normal positions within the sac. 4. The defect occurs through the umbilicus: This is true. Omphalocele occurs through the umbilical ring. 5. Mortality may be as high as 15%: This is true. Mortality rates can be high, especially if there are associated congenital anomalies and complications.
Therefore, the correct answer is that the statement “The herniated organs lie outside the peritoneal sac” is false.
Gastroschisis: Isolated abnormality, bowel lies outside abdominal wall through defect located to right of umbilicus.
Exomphalos: Liver and gut remain covered with membranous sac connected to umbilical cord. It is associated with other developmental defects.
They are contained within the peritoneal sac and therefore do not have the fluid losses seen in gastroschisis. True malrotation is unusual and minor variants may not result in a requirement for surgery.
Paediatric Gastrointestinal disorders
Pyloric stenosis
M>F
5-10% Family history in parents
Projectile non bile stained vomiting at 4-6 weeks of life
Diagnosis is made by test feed or USS
Treatment: Ramstedt pyloromyotomy (open or laparoscopic)
Acute appendicitis
Uncommon under 3 years
When occurs may present atypically
Mesenteric adenitis
Central abdominal pain and URTI
Conservative management
Intussusception
Telescoping bowel
Proximal to or at the level of, ileocaecal valve
6-9 months age
Colicky pain, diarrhoea and vomiting, sausage shaped mass, red jelly stool.
Treatment: reduction with air insufflation
Malrotation
High caecum at the midline
Feature in exomphalos, congenital diaphragmatic hernia, intrinsic duodenal atresia
May be complicated by development of volvulus, infant with volvulus may have bile stained vomiting
Diagnosis is made by upper GI contrast study and USS
Treatment is by laparotomy, if volvulus is present (or at high risk of occurring then a ladds procedure is performed
Hirschsprung’s disease
Absence of ganglion cells from myenteric and submucosal plexuses
Occurs in 1/5000 births
Full thickness rectal biopsy for diagnosis
Delayed passage of meconium and abdominal distension
Treatment is with rectal washouts initially, thereafter an anorectal pull through procedure
Oesophageal atresia
Associated with tracheo-oesophageal fistula and polyhydramnios
May present with choking and cyanotic spells following aspiration
VACTERL associations
Meconium ileus
Usually delayed passage of meconium and abdominal distension
Majority have cystic fibrosis
X-Rays may not show a fluid level as the meconium is viscid (depends upon feeding), PR contrast studies may dislodge meconium plugs and be therapeutic
Infants who do not respond to PR contrast and NG N-acetyl cysteine will require surgery to remove the plugs
Biliary atresia
Jaundice > 14 days
Increased conjugated bilirubin
Urgent Kasai procedure
Necrotising enterocolitis
Prematurity is the main risk factor
Early features include abdominal distension and passage of bloody stools
X-Rays may show pneumatosis intestinalis and evidence of free air
Increased risk when empirical antibiotics are given to infants beyond 5 days
Treatment is with total gut rest and TPN, babies with perforations will require laparotomy
A 28 year old man is involved in a road traffic accident and sustains a flail chest injury. On arrival in the emergency department he is hypotensive. On examination; he has an elevated jugular venous pulse and auscultation of the heart reveals quiet heart sounds. What is the most likely diagnosis?
Pneumothorax
Myocardial contusion
Cardiac tamponade
Haemothorax
Ventricular septal defect
The presence of a cardiac tamponade is suggested by Becks Triad:
Hypotension
Muffled heart sounds
Raised JVP
Thoracic trauma
Key points related to thoracic trauma
Less than 10% of blunt chest trauma and 15-30% of penetrating chest trauma requires operative intervention.
The physiologic consequences of thoracic trauma are hypoxia, hypercarbia, and acidosis. Contusion, hematoma, and alveolar collapse, or changes in intrathoracic pressure relationships (e.g., tension pneumothorax and open pneumothorax) cause hypoxia and lead to metabolic acidosis. Hypercarbia causes respiratory acidosis and most often follows inadequate ventilation caused by changes in intrathoracic pressure relationships and depressed level of consciousness.
Types of thoracic trauma
Tension pneumothorax
Often laceration to lung parenchyma with flap
Pressure develops in thorax
Most common cause is mechanical ventilation in patient with pleural injury
Symptoms overlap with cardiac tamponade, hyper-resonant percussion note is more likely in tension pnemothorax
Flail chest
Chest wall disconnects from thoracic cage
Multiple rib fractures (at least two fractures per rib in at least two ribs)
Associated with pulmonary contusion
Abnormal chest motion
Avoid over hydration and fluid overload
Pneumothorax
Most common cause is lung laceration with air leakage
Most traumatic pneumothoraces should have a chest drain
Patients with traumatic pneumothorax should never be mechanically ventilated until a chest drain is inserted
Haemothorax
Most commonly due to laceration of lung, intercostal vessel or internal mammary artery
Haemothoraces large enough to appear on CXR are treated with large bore chest drain
Surgical exploration is warranted if >1500ml blood drained immediately
Cardiac tamponade
Beck’s triad: elevated venous pressure, reduced arterial pressure, reduced heart sounds
Pulsus paradoxus
May occur with as little as 100ml blood
Pulmonary contusion
Most common potentially lethal chest injury
Arterial blood gases and pulse oximetry important
Early intubation within an hour if significant hypoxia
Blunt cardiac injury
Usually occurs secondary to chest wall injury
ECG may show features of myocardial infarction
Sequelae: hypotension, arrhythmias, cardiac wall motion abnormalities
Aorta disruption
Deceleration injuries
Contained haematoma
Widened mediastinum
Diaphragm disruption
Most due to motor vehicle accidents and blunt trauma causing large radial tears (laceration injuries result in small tears)
More common on left side
Insert gastric tube, may pass into intrathoracic stomach
Mediastinal traversing wounds
Entrance wound in one hemithorax and exit wound/foreign body in opposite hemithorax
Mediastinal haematoma or pleural cap suggests great vessel injury
Mortality is 20%
As a busy surgical trainee on the colorectal unit you are given the unenviable task of reviewing the unit’s histopathology results for colonic polyps. Which of the polyp types described below has the greatest risk of malignant transformation?
Hyperplastic polyp
Tubular adenoma
Villous adenoma
Hamartomatous polyp
Pseudopolyp
Villous adenomas carry the highest risk of malignant transformation. Hyperplastic polyps carry little in the way of increased risk. Although, patients with hamartomatous polyp syndromes may have a high risk of malignancy, the polyps themselves have little malignant potential.
Colonic polyps
Colonic Polyps
May occur in isolation, or greater numbers as part of the polyposis syndromes. In FAP greater than 100 polyps are typically present. The risk of malignancy in association with adenomas is related to size, and is the order of 10% in a 1cm adenoma. Isolated adenomas seldom give risk of symptoms (unless large and distal). Distally sited villous lesions may produce mucous and if very large, electrolyte disturbances may occur.
Follow up of colonic polyps
Group Action
Colorectal cancer Colonoscopy 1 year post resection
Large non pedunculated colorectal polyps (LNPCP), R0 resection One off scope at 3 years
Large non pedunculated colorectal polyps (LNPCP) R1 or non en bloc resection Site check at 2-6 months and then a further scope at 12 months
High risk findings at baseline colonoscopy One off surveillance at 3 years
No high risk findings at baseline colonoscopy No colonoscopic surveillance and invite participation in NHSBCSP programme when due
High risk findings
More than 2 premalignant polyps including 1 or more advanced colorectal polyps
OR
More than 5 pre malignant polyps
Exceptions to guidelines
If patient more than 10 years younger than lower screening age and has polyps but no high risk findings, consider colonoscopy at 5 or 10 years.
Segmental resection or complete colectomy should be considered when:
- Incomplete excision of malignant polyp
- Malignant sessile polyp
- Malignant pedunculated polyp with submucosal invasion
- Polyps with poorly differentiated carcinoma
- Familial polyposis coli
-Screening from teenager up to 40 years by 2 yearly sigmoidoscopy/colonoscopy
-Panproctocolectomy and Ileostomy or Restorative Panproctocolectomy.
Rectal polypoidal lesions may be amenable to trans anal endoscopic microsurgery.
A 28 year old man has sustained a non salvageable testicular injury to his left testicle. The surgeon decides to perform an orchidectomy and divides the left testicular artery. From which of the following does this vessel originate?
Abdominal aorta
Internal iliac artery
Inferior epigastric artery
Inferior vesical artery
External iliac artery
The testicular artery is a branch of the abdominal aorta. Scrotal and testicular anatomy
Spermatic cord
Formed by the vas deferens and is covered by the following structures:
Layer Origin
Internal spermatic fascia Transversalis fascia
Cremasteric fascia From the fascial coverings of internal oblique
External spermatic fascia External oblique aponeurosis
Contents of the cord
Vas deferens Transmits sperm and accessory gland secretions
Testicular artery Branch of abdominal aorta supplies testis and epididymis
Artery of vas deferens Arises from inferior vesical artery
Cremasteric artery Arises from inferior epigastric artery
Pampiniform plexus Venous plexus, drains into right or left testicular vein
Sympathetic nerve fibres Lie on arteries, the parasympathetic fibres lie on the vas
Genital branch of the genitofemoral nerve Supplies cremaster
Lymphatic vessels Drain to lumbar and para-aortic nodes
Scrotum
Composed of skin and closely attached dartos fascia.
Arterial supply from the anterior and posterior scrotal arteries
Lymphatic drainage to the inguinal lymph nodes
Parietal layer of the tunica vaginalis is the innermost layer
Testes
The testes are surrounded by the tunica vaginalis (closed peritoneal sac). The parietal layer of the tunica vaginalis adjacent to the internal spermatic fascia.
The testicular arteries arise from the aorta immediately inferiorly to the renal arteries.
The pampiniform plexus drains into the testicular veins, the left drains into the left renal vein and the right into the inferior vena cava.
Lymphatic drainage is to the para-aortic nodes.
A 62 year old lady presents with a non tender lump overlying the distal interphalangeal joint of the index finger. On examination, she has a hard, non tender lump overlying the joint and deviation of the tip of the finger. What is the nature of the lesion?
Oslers nodes
Bouchards nodes
Heberdens nodes
Osteosarcoma
Infective collection
Heberdens nodes may produce swelling of the distal interphalangeal joint with deviation of the finger tip.
Hand diseases
Dupuytrens contracture
Fixed flexion contracture of the hand where the fingers bend towards the palm and cannot be fully extended.
Caused by underlying contractures of the palmar aponeurosis . The ring finger and little finger are the fingers most commonly affected. The middle finger may be affected in advanced cases, but the index finger and the thumb are nearly always spared.
Progresses slowly and is usually painless. In patients with this condition, the tissues under the skin on the palm of the hand thicken and shorten so that the tendons connected to the fingers cannot move freely. The palmar aponeurosis becomes hyperplastic and undergoes contracture.
Commonest in males over 40 years of age.
Association with liver cirrhosis and alcoholism. However, many cases are idiopathic.
Treatment is surgical and involves fasciectomy. However, the condition may recur and many surgical therapies are associated with risk of neurovascular damage to the digital nerves and arteries.
Carpal tunnel syndrome
Idiopathic median neuropathy at the carpal tunnel.
Characterised by altered sensation of the lateral 3 fingers.
The condition is commoner in females and is associated with other connective tissue disorders such as rheumatoid disease. It may also occur following trauma to the distal radius.
Symptoms occur mainly at night in early stages of the condition.
Examination may demonstrate wasting of the muscles of the thenar eminence and symptoms may be reproduced by Tinels test (compression of the contents of the carpal tunnel).
Formal diagnosis is usually made by electrophysiological studies.
Treatment is by surgical decompression of the carpal tunnel, a procedure achieved by division of the flexor retinaculum. Non - surgical options include splinting and bracing.
Miscellaneous hand lumps
Osler’s nodes Osler’s nodes are painful, red, raised lesions found on the hands and feet. They are the result of the deposition of immune complexes.
Bouchard’s nodes Hard, bony outgrowths or gelatinous cysts on the proximal interphalangeal joints (the middle joints of fingers or toes.) They are a sign of osteoarthritis, and are caused by formation of calcific spurs of the articular cartilage.
Heberden’s nodes Typically develop in middle age, beginning either with a chronic swelling of the affected joints or the sudden painful onset of redness, numbness, and loss of manual dexterity. This initial inflammation and pain eventually subsides, and the patient is left with a permanent bony outgrowth that often skews the fingertip sideways. It typically affects the DIP joint.
Ganglion Swelling in association with a tendon sheath commonly near a joint. They are common lesions in the wrist and hand. Usually they are asymptomatic and cause little in the way of functional compromise. They are fluid filled although the fluid is similar to synovial fluid it is slightly more viscous. When the cysts are troublesome they may be excised.
A surgeon is considering using lignocaine as local anasthesia for a minor procedure. Which of the following best accounts for its actions?
Blockade of neuronal acetylcholine receptors
Blockade of neuronal nicotinic receptors
Blockade of neuronal sodium channels
Blockade of neuronal potassium channels
Blockade of neuronal calcium channels
Lignocaine blocks sodium channels. They will typically be activated first, hence the pain some patients experience on administration.
Local anaesthetic agents
Lidocaine
An amide
Local anaesthetic and a less commonly used antiarrhythmic (affects Na channels in the axon)
Hepatic metabolism, protein bound, renally excreted
Toxicity: due to IV or excess administration. Increased risk if liver dysfunction or low protein states. Note acidosis causes lidocaine to detach from protein binding.
Drug interactions: Beta blockers, ciprofloxacin, phenytoin
Features of toxicity: Initial CNS over activity then depression as lidocaine initially blocks inhibitory pathways then blocks both inhibitory and activating pathways. Cardiac arrhythmias.
Increased doses may be used when combined with adrenaline to limit systemic absorption.
Cocaine
Pure cocaine is a salt, usually cocaine hydrochloride. It is supplied for local anaesthetic purposes as a paste.
It is supplied for clinical use in concentrations of 4 and 10%. It may be applied topically to the nasal mucosa. It has a rapid onset of action and has the additional advantage of causing marked vasoconstriction.
It is lipophillic and will readily cross the blood brain barrier. Its systemic effects also include cardiac arrhythmias and tachycardia.
Apart from its limited use in ENT surgery it is otherwise used rarely in mainstream surgical practice.
Bupivacaine
Bupivacaine binds to the intracellular portion of sodium channels and blocks sodium influx into nerve cells, which prevents depolarization.
It has a much longer duration of action than lignocaine and this is of use in that it may be used for topical wound infiltration at the conclusion of surgical procedures with long duration analgesic effect.
It is cardiotoxic and is therefore contra indicated in regional blockage in case the tourniquet fails.
Levobupivacaine (Chirocaine) is less cardiotoxic and causes less vasodilation.
Prilocaine
Similar mechanism of action to other local anaesthetic agents. However, it is far less cardiotoxic and is therefore the agent of choice for intravenous regional anaesthesia e.g. Biers Block.
All local anaesthetic agents dissociate in tissues and this contributes to their therapeutic effect. The dissociation constant shifts in tissues that are acidic e.g. where an abscess is present, and this reduces the efficacy.
Doses of local anaesthetics
Agent Dose plain Dose with adrenaline
Lignocaine 3mg/Kg 7mg/Kg
Bupivacaine 2mg/Kg 2mg/Kg
Prilocaine 6mg/Kg 9mg/Kg
These are a guide only as actual doses depend on site of administration, tissue vascularity and co-morbidities.
Maximum total local anaesthetic doses
Lignocaine 1% plain - 3mg/ Kg - 200mg (20ml)
Lignocaine 1% with 1 in 200,000 adrenaline - 7mg/Kg - 500mg (50ml)
Bupivacaine 0.5% - 2mg/kg- 150mg (30ml)
Maximum doses are based on ideal body weight
Effects of adrenaline
Adrenaline may be added to local anaesthetic drugs. It prolongs the duration of action at the site of injection and permits usage of higher doses (see above). It is contra indicated in patients taking MAOI’s or tricyclic antidepressants. The toxicity of bupivacaine is related to protein binding and addition of adrenaline to this drug does not permit increases in the total dose of bupivacaine, in contrast to the situation with lignocaine.
Which of the following statements is false in relation to consent?
All adults by law are assumed to be competent
The Bolam test defines if a patient has capacity or not
Consent 2 is the form signed by parents on behalf of their children
Implied consent is a form of consent
Consent 4 is the form signed when a patient is unable to consent for a treatment or investigation
The Bolam test defines if a decision made by a doctor is in agreement with the professional standard of medical practise.
Consent
There are 3 types of consent:
- Informed
- Expressed
- Implied
Consent forms used in UK NHS
Consent Form 1 For competent adults who are able to consent for themselves where consciousness may be impaired (e.g. GA)
Consent Form 2 For an adult consenting on behalf of a child where consciousness is impaired
Consent Form 3 For an adult or child where consciousness is not impaired
Consent Form 4 For adults who lack capacity to provide informed consent
Capacity
Key points include:
1. Understand and retain information
2. Patient believes the information to be true
3. Patient is able to weigh the information to make a decision
All patients must be assumed to have capacity
Consent in minors
Young children and older children who are not Gillick competent cannot consent for themselves. In British law the patients biological mother can always provide consent. The child’s father can consent if the parents are married (and the father is the biological father), or if the father is named on the birth certificate (irrespective of marital status). If parents are not married and the father is not named on the birth certificate then the father cannot consent.
A well toned weight lifter attends clinic reporting weakness of his left arm. There is weakness of flexion and supination of the forearm. Which of the following nerves is likely to have been affected?
Axillary
Suprascapular
Median
Radial
Musculocutaneous
Musculocutaneous nerve compression due to entrapment of the nerve between biceps and brachialis. Elbow flexion and supination of the arm are affected. This is a rare isolated injury.
Musculocutaneous nerve
Branch of lateral cord of brachial plexus
Path
It penetrates the coracobrachialis muscle
Passes obliquely between the biceps brachii and the brachialis to the lateral side of the arm
Above the elbow it pierces the deep fascia lateral to the tendon of the biceps brachii
Continues into the forearm as the lateral cutaneous nerve of the forearm
Innervates
Coracobrachialis
Biceps brachii
Brachialis
A 32 year old rugby player is crushed in a scrum. He is briefly concussed but then regains consciousness. He then collapses and is brought to ED. His GCS on arrival is 6/15 and his left pupil is dilated. What is the best course of definitive management?
Burr Hole decompression
Parietotemporal craniotomy
Intravenous mannitol
Posterior fossa craniotomy
Insertion of intra cranial pressure bolt monitor
This man needs urgent decompression and extradural haematoma is the most likely event, from a lacerated middle meningeal artery. Most neurosurgeons would perform a craniotomy. However, rural units and those units without neurosurgical kit facing this emergency may resort to Burr Holes.
Head injury management- NICE Guidelines
Summary of guidelines
All patients should be assessed within 15 minutes on arrival to A&E
Document all 3 components of the GCS
If GCS <8 or = to 8, consider stabilising the airway
Full spine immobilisation until assessment if:
- GCS < 15
- neck pain/tenderness
- paraesthesia extremities
- focal neurological deficit
- suspected c-spine injury
If a c-spine injury is suspected a 3 view c-spine x-ray is indicated. CT c-spine is preferred if:
- Intubated
- GCS <13
- Normal x-ray but continued concerns regarding c-spine injury
- Any focal neurology
- A CT head scan is being performed
- Initial plain films are abnormal
Immediate CT head (within 1 hour) if:
GCS < 12 on admission
GCS < 15 2 hours after admission
Suspected open or depressed skull fracture
Suspected skull base fracture (panda eyes, Battle’s sign, CSF from nose/ear, bleeding ear)
Focal neurology
Vomiting > 1 episode
Post traumatic seizure
Contact neurosurgeon if:
Persistent GCS < 8 or = 8
Unexplained confusion > 4h
Reduced GCS after admission
Progressive neurological signs
Incomplete recovery post seizure
Penetrating injury
Cerebrospinal fluid leak
Observations
1/2 hourly GCS until 15
In the latest guidance, there is a move to considering CT in those on anticoagulants rather than mandating it.
An orthopaedic surgeon makes a modification to an operative approach for total knee arthroplasty. After he has completed 25 cases, he stops and reviews his patient outcomes. He publishes the data. What level of evidence is supplied by this type of data?
II
IV
III
V
I
Case series that are non randomised and lack concurrent controls at best supply level IV evidence only. To qualify for level I and II evidence a prospective randomised controlled trial with appropriate blinding, control matching and power calculations is needed.
Levels of evidence
The level of evidence refers to the study design used by investigators to minimise bias.
Level of evidence Source
I Evidence obtained from systematic review of all relevant randomised controlled trials
II Evidence derived from at least one properly designed randomised controlled trial
III Evidence derived from well designed pseudo-randomised controlled trials (e.g. alternate allocation) or historical controls
IV Evidence derived from case series or case reports
V Panel or expert opinion
Many of the categories contain sub groups, detailed knowledge of these are not required for MRCS Part A.
A 30 year old man is admitted overnight, following a road traffic accident. He has an open tibial fracture with a 20 cm wound and extensive periosteal stripping. He is neurovascularly intact; IV antibiotics and wound dressing have been administered in the emergency department. What is the most appropriate course of action?
Immediate skeletal stabilisation and application of negative pressure dressing
Skeletal fixation followed by vascular reconstruction
Immediate vascuIar shunting, followed by temporary skeletal stabilisation and vascular reconstruction
Combined skeletal and soft tissue reconstruction on a scheduled operating list
Fasciotomy with four compartment decompression
This patient has a Gustillo-Anderson Grade 3B open fracture. He will require definitive skeletal and soft tissue reconstruction, which should be performed on a combined ortho-plastic scheduled operating list, as per the BOA/BAPRAS guidelines. The surgery does not have to be performed out of scheduled hours unless there is marine/ sewage contamination, vascular compromise or it is a polytrauma.
Whilst it is reasonable to apply an external fixator prior to definitive skeletal and soft tissue reconstruction, this should be converted to internal fixation within 72 hours.
Open fractures
The term open fracture refers to a disruption of the bony cortex associated with a breach in the overlying skin. Any wound that is present in the same limb as a fracture should be suspected as being representative of an open fracture. One of the main problems with open fractures is the associated injuries to the surrounding soft tissues. Whilst the skin is usually relatively resistant to trauma, underlying muscle can be damaged or devitalised, nerves, blood vessels and periosteum may all be disrupted the degree to which this occurs correlates with the severity of the injury and the outcome. These can be graded using the Gustilo and Anderson system (see below).
Grade Injury
1 Low energy wound <1cm
2 Greater than 1cm wound with moderate soft tissue damage
3 High energy wound > 10cm with extensive soft tissue damage
3 A (sub group of 3) Adequate soft tissue coverage
3 B (sub group of 3) Inadequate soft tissue coverage
3 C (sub group of 3) Associated arterial injury
In Type IIIc injuries, the mangled extremity scoring system (MESS) can help to predict the need for primary amputation.
Key points for initial management
Remove obvious contaminants from very contaminated wounds in the ED
Wound lavage should usually be undertaken in the OR
Early wound photography should be performed
Consider transfer of complex cases to centres that provide orthoplastic care
All wounds should be managed within 24 hours and high velocity ones within 12 hours, those with vascular compromise should be managed immediately
CT angiography is useful in delineating the extent of concommitant vascular injury
A man undergoes a high anterior resection for carcinoma of the upper rectum. Which of the following vessels will require ligation?
Superior mesenteric artery
Inferior mesenteric artery
Coeliac axis
Perineal artery
Middle colic artery
The IMA is usually divided during anterior resection. Not only is this borne out of oncological necessity but it also permits sufficient colonic mobilisation for anastomosis.
Colon anatomy
The colon commences with the caecum. This represents the most dilated segment of the human colon and its base (which is intraperitoneal) is marked by the convergence of teniae coli. At this point is located the vermiform appendix. The colon continues as the ascending colon, the posterior aspect of which is retroperitoneal. The line of demarcation between the intra and retro peritoneal right colon is visible as a white line, in the living, and forms the line of incision for colonic resections.
The ascending colon becomes the transverse colon after passing the hepatic flexure. At this location the colon becomes wholly intra peritoneal once again. The superior aspect of the transverse colon is the point of attachment of the transverse colon to the greater omentum. This is an important anatomical site since division of these attachments permits entry into the lesser sac. Separation of the greater omentum from the transverse colon is a routine operative step in both gastric and colonic resections.
At the left side of the abdomen the transverse colon passes to the left upper quadrant and makes an oblique inferior turn at the splenic flexure. Following this, the posterior aspect becomes retroperitoneal once again.
At the level of approximately L4 the descending colon becomes wholly intraperitoneal and becomes the sigmoid colon. Whilst the sigmoid is wholly intraperitoneal there are usually attachments laterally between the sigmoid and the lateral pelvic sidewall. These small congenital adhesions are not formal anatomical attachments but frequently require division during surgical resections.
At its distal end the sigmoid passes to the midline and at the region around the sacral promontary it becomes the upper rectum. This transition is visible macroscopically as the point where the teniae fuse. More distally the rectum passes through the peritoneum at the region of the peritoneal reflection and becomes extraperitoneal.
Arterial supply
Superior mesenteric artery and inferior mesenteric artery: linked by the marginal artery.
Ascending colon: ileocolic and right colic arteries
Transverse colon: middle colic artery
Descending and sigmoid colon: inferior mesenteric artery
Venous drainage
From regional veins (that accompany arteries) to superior and inferior mesenteric vein
Lymphatic drainage
Initially along nodal chains that accompany supplying arteries, then para-aortic nodes.
Embryology
Midgut- Second part of duodenum to 2/3 transverse colon
Hindgut- Distal 1/3 transverse colon to anus
Peritoneal location
The right and left colon are part intraperitoneal and part extraperitoneal. The sigmoid and transverse colon are generally wholly intraperitoneal. This has implications for the sequelae of perforations, which will tend to result in generalised peritonitis in the wholly intra peritoneal segments.
Colonic relations
Region of colon Relation
Caecum/ right colon Right ureter, gonadal vessels
Hepatic flexure Gallbladder (medially)
Splenic flexure Spleen and tail of pancreas
Distal sigmoid/ upper rectum Left ureter
Rectum Ureters, autonomic nerves, seminal vesicles, prostate, urethra (distally)
A 78 year old lady is admitted with a 3 hour history of passage of dark red blood per rectum. Prior to this event, she was otherwise well with no major medical co-morbidities. On examination, she has a mild tachycardia but other vital signs are normal, abdomen is soft and non tender. Digital rectal exam reveals dark blood but no other findings. What is the most likely underlying cause?
Diverticular disease
Meckels diverticulum
Jejunal diverticulosis
Angiodysplasia of the colon
Colonic cancer
Diverticular disease is the commonest cause of lower GI bleeding in adults.
Diverticular associated bleeding is the commonest cause of lower GI bleeding in older adults and it is important to note that it is not associated with diverticulitis which seldom bleeds. Most cases will stop bleeding spontaneously.
Lower Gastrointestinal bleeding
Colonic bleeding
This typically presents as bright red or dark red blood per rectum. Colonic bleeding rarely presents as malaena type stool, this is because blood in the colon has a powerful laxative effect and is rarely retained long enough for transformation to occur and because the digestive enzymes present in the small bowel are not present in the colon. Up to 15% of patients presenting with haemochezia will have an upper gastrointestinal source of haemorrhage.
As a general rule right sided bleeds tend to present with darker coloured blood than left sided bleeds. Haemorrhoidal bleeding typically presents as bright red rectal bleeding that occurs post defecation either onto toilet paper or into the toilet pan. It is very unusual for haemorrhoids alone to cause any degree of haemodynamic compromise.
Causes
Cause Presenting features
Colitis Bleeding may be brisk in advanced cases, diarrhoea is commonly present. Abdominal x-ray may show featureless colon.
Diverticular disease Acute diverticulitis often is not complicated by major bleeding and diverticular bleeds often occur sporadically. 75% all will cease spontaneously within 24-48 hours. Bleeding is often dark and of large volume.
Cancer Colonic cancers often bleed and for many patients this may be the first sign of the disease. Major bleeding from early lesions is uncommon
Haemorrhoidal bleeding Typically bright red bleeding occurring post defecation. Although patients may give graphic descriptions bleeding of sufficient volume to cause haemodynamic compromise is rare.
Angiodysplasia Apart from bleeding, which may be massive, these arteriovenous lesions cause little in the way of symptoms. The right side of the colon is more commonly affected.
Management
Prompt correction of any haemodynamic compromise is required. Unlike upper gastrointestinal bleeding the first line management is usually supportive. This is because in the acute setting endoscopy is rarely helpful.
When haemorrhoidal bleeding is suspected a proctosigmoidoscopy is reasonable as attempts at full colonoscopy are usually time consuming and often futile.
In the unstable patient the usual procedure would be an angiogram (either CT or percutaneous), when these are performed during a period of haemodynamic instability they may show a bleeding point and may be the only way of identifying a patch of angiodysplasia.
In others who are more stable the standard procedure would be a colonoscopy in the elective setting. In patients undergoing angiography attempts can be made to address the lesion in question such as coiling. Otherwise surgery will be necessary.
In patients with ulcerative colitis who have significant haemorrhage the standard approach would be a sub total colectomy, particularly if medical management has already been tried and is not effective.
Indications for surgery
Patients > 60 years
Continued bleeding despite endoscopic intervention
Recurrent bleeding
Known cardiovascular disease with poor response to hypotension
Surgery
Selective mesenteric embolisation if life threatening bleeding. This is most helpful if conducted during a period of relative haemodynamic instability. If all haemodynamic parameters are normal then the bleeding is most likely to have stopped and any angiography normal in appearance. In many units a CT angiogram will replace selective angiography but the same caveats will apply.
If the source of colonic bleeding is unclear; perform a laparotomy, on table colonic lavage and following this attempt a resection. A blind sub total colectomy is most unwise, for example bleeding from an small bowel arterio-venous malformation will not be treated by this manoeuvre.
Summary of Acute Lower GI bleeding recommendations
Consider admission if:
* Over 60 years
* Haemodynamically unstable/profuse PR bleeding
* On aspirin or NSAID
* Significant co morbidity
Management
All patients should have a history and examination, PR and proctoscopy
Colonoscopic haemostasis aimed for in post polypectomy or diverticular bleeding
A 25-year-old female with a history of bilateral vitreous haemorrhage is referred with bilateral lesions in the cerebellar region. What is the likely diagnosis?
Neurofibromatosis type I
Neurofibromatosis type II
Tuberous sclerosis
Von Hippel-Lindau syndrome
Sarcoidosis
Retinal and cerebellar haemangiomas are key features of Von Hippel-Lindau syndrome. Retinal haemangiomas are bilateral in 25% of patients and may lead to vitreous haemorrhage
Von Hippel-Lindau syndrome
Von Hippel-Lindau (VHL) syndrome is an autosomal dominant condition predisposing to neoplasia. It is due to an abnormality in the VHL gene located on short arm of chromosome 3
Features
cerebellar haemangiomas
retinal haemangiomas: vitreous haemorrhage
renal cysts (premalignant)
phaeochromocytoma
extra-renal cysts: epididymal, pancreatic, hepatic
endolymphatic sac tumours
A 45 year old man undergoes a surgical excision of a carotid body tumour. Histological analysis is most likely to demonstrate which of the following?
Paraganglionoma
Fibromatosis colli
Schwannoma
Lipoma
Neuroma
Carotid body tumours are the commonest type of head and neck paraganglionoma.
Carotid body tumour
These are rare tumours. However, they typically account for around 60% of head and neck paraganglionomas. They are usually tumours of middle age. Around 5% are bilateral and around 5% are malignant. They are rarely found as part of the MEN II or neurofibromatosis type I disease complexes.
They typically present as an asymptomatic neck mass in the anterior triangle of the neck. They are typically slow growing lesions.
Types of carotid body tumour
Sporadic - Accounts for 85% of cases
Familial - Seen in around 10% of cases and usually in younger patients
Hyperplastic - Seen in those at high altitude or in those with COPD
Imaging
They are readily imaged using duplex ultrasonography. CT angiography is sometimes helpful.
Treatment
Typically this comprises surgical resection. This is preceded by embolization in selected cases.
A 43 year old man has a nasogastric tube inserted. The nurse takes a small aspirate of the fluid from the stomach and tests the pH of the aspirate. What is the normal intragastric pH?
0.5
2
4
5
6
The intragastric pH is usually 2. Administration of proton pump inhibitors can result in almost complete abolition of acidity
Gastric secretions
A working knowledge of gastric secretions is important for surgery because peptic ulcers are common, surgeons frequently prescribe anti secretory drugs and because there are still patients around who will have undergone acid lowering procedures (Vagotomy) in the past.
Gastric acid
Is produced by the parietal cells in the stomach
pH of gastric acid is around 2 with acidity being maintained by the H+/K+ ATP ase pump. As part of the process bicarbonate ions will be secreted into the surrounding vessels.
Sodium and chloride ions are actively secreted from the parietal cell into the canaliculus. This sets up a negative potential across the membrane and as a result sodium and potassium ions diffuse across into the canaliculus.
Carbonic anhydrase forms carbonic acid which dissociates and the hydrogen ions formed by dissociation leave the cell via the H+/K+ antiporter pump. At the same time sodium ions are actively absorbed. This leaves hydrogen and chloride ions in the canaliculus these mix and are secreted into the lumen of the oxyntic gland.
Phases of gastric acid secretion
There are 3 phases of gastric secretion:
- Cephalic phase (smell / taste of food)
30% acid produced
Vagal cholinergic stimulation causing secretion of HCL and gastrin release from G cells - Gastric phase (distension of stomach )
60% acid produced
Stomach distension/low H+/peptides causes Gastrin release - Intestinal phase (food in duodenum)
10% acid produced
High acidity/distension/hypertonic solutions in the duodenum inhibits gastric acid secretion via enterogastrones (CCK, secretin) and neural reflexes.
Regulation of gastric acid production
Factors increasing production include:
Vagal nerve stimulation
Gastrin release
Histamine release (indirectly following gastrin release) from enterchromaffin like cells
Factors decreasing production include:
Somatostatin (inhibits histamine release)
Cholecystokinin
Secretin
The diagram below illustrates some of the factors involved in regulating gastric acid secretion and the relevant associated pharmacology
Below is a brief summary of the major hormones involved in food digestion:
Source Stimulus Actions
Gastrin
G cells in antrum of the stomach
Distension of stomach, extrinsic nerves
Inhibited by: low antral pH, somatostatin
Increase HCL, pepsinogen and IF secretion, increases gastric motility, trophic effect on gastric mucosa
CCK
I cells in upper small intestine
Partially digested proteins and triglycerides
Increases secretion of enzyme-rich fluid from pancreas, contraction of gallbladder and relaxation of sphincter of Oddi, decreases gastric emptying, trophic effect on pancreatic acinar cells, induces satiety
Secretin
S cells in upper small intestine
Acidic chyme, fatty acids
Increases secretion of bicarbonate-rich fluid from pancreas and hepatic duct cells, decreases gastric acid secretion, trophic effect on pancreatic acinar cells
VIP
Small intestine, pancreas
Neural
Stimulates secretion by pancreas and intestines, inhibits acid and pepsinogen secretion
Somatostatin
D cells in the pancreas and stomach
Fat, bile salts and glucose in the intestinal lumen
Decreases acid and pepsin secretion, decreases gastrin secretion, decreases pancreatic enzyme secretion, decreases insulin and glucagon secretion
inhibits trophic effects of gastrin, stimulates gastric mucous production
Which of the following structures lie between the lateral and medial heads of the triceps muscle?
Radial nerve
Median nerve
Ulnar nerve
Axillary nerve
Medial cutaneous nerve of the forearm
The radial nerve runs in its groove on between the two heads. The ulnar nerve lies anterior to the medial head. The axillary nerve passes through the quadrangular space. This lies superior to lateral head of the triceps muscle and thus the lateral border of the quadrangular space is the humerus. Therefore the correct answer is the radial nerve.
Triceps
Origin
Long head- infraglenoid tubercle of the scapula.
Lateral head- dorsal surface of the humerus, lateral and proximal to the groove of the radial nerve
Medial head- posterior surface of the humerus on the inferomedial side of the radial groove and both of the intermuscular septae
Insertion
Olecranon process of the ulna. Here the olecranon bursa is between the triceps tendon and olecranon.
Some fibres insert to the deep fascia of the forearm, posterior capsule of the elbow (preventing the capsule from being trapped between olecranon and olecranon fossa during extension)
Innervation Radial nerve
Blood supply Profunda brachii artery
Action Elbow extension. The long head can adduct the humerus and extend it from a flexed position
Relations The radial nerve and profunda brachii vessels lie between the lateral and medial heads
Which of the following is not typically seen in patients with a femoral neck fracture?
Malunion
Non union
Avascular necrosis
Shortening
External rotation
Malunion would be unusual with a femoral neck fracture. Because it is a weight bearing joint, if the fracture is not united then it does not heal at all. It is for this reason that most femoral neck fractures are fixed. Avascular necrosis is a well recognised complication and a total hip replacement or hemiarthroplasty is usually considered in the elderly.
Hip fractures
Background
Neck of femur (NOF) fracture is a common orthopaedic presentation, with over 65000 fractures in the UK per year. Like many orthopaedic injuries, there is a bimodal age distribution. It is imperative to distinguish between the high energy injury in a young patient, and the low energy osteoporotic fracture in the elderly, as their management aims are very different:
Young patient - Usually high energy trauma (e.g road traffic accident, horse riding) and needs treating in accordance with Advanced Trauma Life Support (ATLS) principles. Will often have associated injuries. Aim is to retain the patients own anatomy, and optimise their function.
Elderly patient - Predominantly female, fall from standing height (fragility fracture). Often patients have multiple comorbidities that will ultimately dictate their prognosis. Aim of orthopaedic treatment is to immediately regain patient mobility so that morbidity (infection, thromboembolic events, pressure sores etc) and mortality associated with prolonged bed rest is avoided. Left untreated, a neck of femur fracture can be considered a terminal event. Historically, mortality associated with elderly hip fracture is 10% at one month, and 30% at one year. However, this has been improved in the UK with the introduction of multidisciplinary, orthogeriatric lead care and the National Hip Fracture Database and Best Practice Tariff.
Pertinent anatomy
Osteology - normal neck-shaft angle is 130 +/- 7 degrees, and 10 +/- 7 degrees of neck anteversion.
Vascular supply - The predominant blood supply to the femoral head and neck is from the medial and lateral femoral circumflex arteries (branches of profunda femoris). These anastomose and pierce the joint capsule at the base of the neck, mainly posteriorly. There is a small vascular contribution from the artery of the ligament teres. Understanding the blood supply is fundamental to the decision making process in treating NOF fractures.
Presentation and initial management
Typically, patients present with pain in the hip/groin, a shortened, abducted, externally rotated leg (due to the unopposed pull of the muscles that act across the hip joint) and the inability to straight-leg-raise. With undisplaced fractures, signs are more subtle.
High energy injuries should be treated in line with ATLS principles. All patients should be fluid resuscitated, have adequate pain relief (often with a fascio-iliiaca nerve block), and be optimised for surgery. In addition, elderly patients should be assessed by an orthogeriatrician.
Imaging
Anteroposterior and cross-table lateral plain radiographs are sufficient to diagnose the majority of NOF fractures. If the fracture extends below the level of the lesser trochanter, or there is any possibility of pathological fracture, full length femur views are essential to plan surgery.
Where there is a high index of suspicion of fracture, but plain radiographs are inconclusive, gold standard investigation is MRI. However, if unavailable within 24 hours, or if the patient will not tolerate MRI, CT is appropriate. The majority of fractures can be seen with modern CT techniques, and so this is becoming first line in many hospitals.
Classification
There has been a move away from named classification systems towards descriptive classification systems.
Two main types of NOF exist: Intra-capsular, and extra-capsular. Extra-capsular fractures are further divided into pertrochanteric or subtrochanteric (within 5cm distal to the lesser trochanter). All fractures are then described as undisplaced, minimally displaced, or displaced.
Femoral neck and head blood supply disruption is common with intracapsular NOF fractures, and rare with extracapsular fractures. This fundamental principle underpins the practise of arthroplasty for intracapsular fractures, and fixation for extracapsular fractures.
If you wish to use a named classification system, the most commonly used are below:
Elderly intracapsular - Garden Classification
Young intrasapsular - Pauvels Classification
Intertrochanteric - Evans
Subtrochanteric - Russell Taylor
Treatment
In general, NOF fractures are treated operatively except if the patient is deemed unlikely to survive an anaesthetic. Best Practice Tarif (BPT) dictates that surgery should happen within 36 hours, as delay of greater than 48 hours is associated with increased morbidity and mortality. Below are suggested algorithms for the treatment of NOF. There are some areas of debate/controversy which are detailed below.
- The priority with the young patient is to retain the femoral head if possible, even with a displaced intracapsular fracture. The risk of avascular necrosis and non-union (and therefore revision surgery) associated with internal fixation needs weighing up against the sequelae of total hip replacement in the young (wear, dislocation, revision). Discussion is necessary with the patient, on a case by case basis.
** Undisplaced fractures in the elderly can be treated with internal fixation, often with cannulated screws. This is appropriate for valgus impacted subcapital fractures which are inherently stable, to prevent secondary displacement. This does still carry the risk of AVN or non-union, and therefore a future revision. For this reason, many surgeons advocate arthroplasty as a single surgery.
*** NICE guidance - patients who fulfil these criteria should be offered total hip replacement which conveys better function and prosthetic survivorship, compared with hemiarthroplasty, but at an increased risk of dislocation.
- Intertrochanteric fractures vary greatly in their stability. If the trochanter (and therefore lateral wall), and medial calcar is in tact, then the fracture configuration bears stability. This can be treated with a DHS, as collapse of the fracture is predictable. Where either or both structures are involved in the fracture, stability becomes compromised and many surgeons will favour using an intramedullary device. This is an ongoing debate, and difficult to test in an exam setting.
Post operative management
Patients should be mobilised fully weight bearing where possible. Care is multidisciplinary in its delivery. Elderly patients should have orthogeriatrician assessment of comorbidity, and bone health with secondary prevention measures if appropriate. There should be early involvement of physiotherapy and occupational therapy services. For further guidance see sources listed below.
A 16 year old boy develops a painful swelling of his distal femur. An osteoblastic sarcoma is diagnosed. To which of the following sites is this lesion most likely to metastasise?
Inguinal lymph nodes
Common iliac lymph nodes
Liver
Brain
Lung
Sarcomas in which Lymphatic Metastasis is seen:
‘RACE For MS’
R: Rhabdomyosarcoma
A: Angiosarcoma
C: Clear cell sarcoma
E: Epithelial cell sarcoma
For: Fibrosarcoma
M: Malignant fibrous histiocytoma
S: Synovial cell sarcoma
Or
‘SCARE’
Synovial sarcoma
Clear cell sarcoma
Angiosarcoma
Rhabdomyosarcoma
Epithelioid sarcoma
Sarcomas often metastasise via the haematogenous route and the lung is a common site for sarcoma metastasis. The liver and brain are often spared (at least initially). A smaller number may develop lymphatic metastasis
Sarcomas
Malignant tumours of mesenchymal origin
Types
May be either bone or soft tissue in origin.
Bone sarcoma include:
Osteosarcoma
Ewings sarcoma (although non bony sites recognised)
Chondrosarcoma - originate from Chondrocytes
Soft tissue sarcoma are a far more heterogeneous group and include:
Liposarcoma-adipocytes
Rhabdomyosarcoma-striated muscle
Leiomyosarcoma-smooth muscle
Synovial sarcomas- close to joints (cell of origin not known but not synovium)
Malignant fibrous histiocytoma is now referred to as undifferentiated pleomorphic sarcoma. Careful histological assessment of lesions now allows more accurate categorisation of sarcoma subtypes than was previously possible.
Features
Certain features of a mass or swelling should raise suspicion for a sarcoma these include:
Large >5cm soft tissue mass
Deep tissue location or intra muscular location
Rapid growth
Painful lump
Assessment
Imaging of suspicious masses should utilise a combination of MRI, CT and USS. Blind biopsy should not be performed prior to imaging and where required should be done in such a way that the biopsy tract can be subsequently included in any resection.
Ewings sarcoma
Commoner in males
Incidence of 0.3 / 1, 000, 000
Onset typically between 10 and 20 years of age
Location by femoral diaphysis is commonest site
Histologically it is a small round tumour
Blood borne metastasis is common and chemotherapy is often combined with surgery
Osteosarcoma
Mesenchymal cells with osteoblastic differentiation
20% of all primary bone tumours
Incidence of 5 per 1,000,000
Peak age 15-30, commoner in males
Limb preserving surgery may be possible and many patients will receive chemotherapy
Liposarcoma
Malignancy of adipocytes
Rare, approximately 2.5 per 1,000,000. They are the most common soft tissue sarcoma
Typically located in deep locations such as retroperitoneum
Affect older age group usually >40 years of age
May be well differentiated and thus slow growing although may undergo de-differentiation and disease progression
Many tumours will have a pseudocapsule that can misleadingly allow surgeons to feel that they can ‘shell out’ these lesions. In reality, tumour may invade at the edge of the pseudocapsule and result in local recurrence if this strategy is adopted
Usually resistant to radiotherapy, although this is often used in a palliative setting
Malignant Fibrous Histiocytoma
Tumour with large number of histiocytes
Also described as undifferentiated pleomorphic sarcoma NOS (i.e. Cell of origin is not known)
Four major subtypes are recognised: storiform-pleomorphic (70% cases), myxoid (less aggressive), giant cell and inflammatory
Treatment is usually with surgical resection and adjuvant radiotherapy as this reduces the likelihood of local recurrence
A 63 year old man is recovering following an open extended right hemicolectomy for carcinoma of the colonic splenic flexure. Two days post operatively he develops a persistent pyrexia. What is the least likely cause?
Ileus
Aspiration pneumonia
Anastomotic leak
Wound infection
Urinary tract infection
An ileus in itself is seldom a cause of a pyrexia. It may serve as a proxy marker of other complications. In this scenario atelectasis would be the most likely underlying cause, as open extended right hemicolectomies will necessitate a long midline incision. Anastomotic leaks are less common after right sided colonic surgery and the timeframe for it is rather short (but are possible). Both wound infections and UTI’s ,may complicate major abdominal surgery at any stage. We remind you to check the wording of the question, it asks for the ‘least likely’ cause of pyrexia.
Pyrexia- post operative
Many surgical patients will develop a pyrexia post operatively. The cause and investigation depends upon the nature of the infection.
The following scenarios may account for post operative pyrexia:
Cause Features
Anastomotic leak
Swinging pyrexia
Ileus
Increasing abdominal pain
Raised inflammatory markers
Wound infection
Evidence of superficial erythema, discharge of pus or increasing pain
Usually mild pyrexia (unless major or deep seated wound infection)
May be accompanied by evidence of wound dehisence
Inflammatory markers raised
Atelectasis
Usually complicates abdominal surgery
Most common after midline laparotomies (pain impairs ventilation)
Pyrexia usually mild and non swinging
Most patients will have chest signs on examination
Inflammatory markers raised
Central line sepsis
Patients with complex venous access
May have marked pyrexia
Access site may show evidence of erythema
Diagnosis is by blood culture from line, line removal and subsequent tip culture
Groin lines and those for TPN have the highest risk
Inflammatory markers raised
Urinary tract infection
Common in surgical patients
Usually occur in patients with indwelling urinary catheters
Diagnosis is by dipstick and CSU and signs of raised inflammatory markers
Treatment is with antibiotics (to cover hospital acquired organisms)
An otherwise fit 74 year old man presents with pain in the right hip following minimal trauma. On examination, his leg is shortened and externally rotated. Plain films demonstrate a displaced intracapsular fracture of the femoral neck. What is the best course of action?
Uncemented hemi arthroplasty
Dynamic hip screw
Total hip replacement
Intra medullary nail
Percutaneous pins
In otherwise fit patients aged over 70, the best long term functional outcomes are obtained with total hip arthroplasty.
Hip fractures
Background
Neck of femur (NOF) fracture is a common orthopaedic presentation, with over 65000 fractures in the UK per year. Like many orthopaedic injuries, there is a bimodal age distribution. It is imperative to distinguish between the high energy injury in a young patient, and the low energy osteoporotic fracture in the elderly, as their management aims are very different:
Young patient - Usually high energy trauma (e.g road traffic accident, horse riding) and needs treating in accordance with Advanced Trauma Life Support (ATLS) principles. Will often have associated injuries. Aim is to retain the patients own anatomy, and optimise their function.
Elderly patient - Predominantly female, fall from standing height (fragility fracture). Often patients have multiple comorbidities that will ultimately dictate their prognosis. Aim of orthopaedic treatment is to immediately regain patient mobility so that morbidity (infection, thromboembolic events, pressure sores etc) and mortality associated with prolonged bed rest is avoided. Left untreated, a neck of femur fracture can be considered a terminal event. Historically, mortality associated with elderly hip fracture is 10% at one month, and 30% at one year. However, this has been improved in the UK with the introduction of multidisciplinary, orthogeriatric lead care and the National Hip Fracture Database and Best Practice Tariff.
Pertinent anatomy
Osteology - normal neck-shaft angle is 130 +/- 7 degrees, and 10 +/- 7 degrees of neck anteversion.
Vascular supply - The predominant blood supply to the femoral head and neck is from the medial and lateral femoral circumflex arteries (branches of profunda femoris). These anastomose and pierce the joint capsule at the base of the neck, mainly posteriorly. There is a small vascular contribution from the artery of the ligament teres. Understanding the blood supply is fundamental to the decision making process in treating NOF fractures.
Presentation and initial management
Typically, patients present with pain in the hip/groin, a shortened, abducted, externally rotated leg (due to the unopposed pull of the muscles that act across the hip joint) and the inability to straight-leg-raise. With undisplaced fractures, signs are more subtle.
High energy injuries should be treated in line with ATLS principles. All patients should be fluid resuscitated, have adequate pain relief (often with a fascio-iliiaca nerve block), and be optimised for surgery. In addition, elderly patients should be assessed by an orthogeriatrician.
Imaging
Anteroposterior and cross-table lateral plain radiographs are sufficient to diagnose the majority of NOF fractures. If the fracture extends below the level of the lesser trochanter, or there is any possibility of pathological fracture, full length femur views are essential to plan surgery.
Where there is a high index of suspicion of fracture, but plain radiographs are inconclusive, gold standard investigation is MRI. However, if unavailable within 24 hours, or if the patient will not tolerate MRI, CT is appropriate. The majority of fractures can be seen with modern CT techniques, and so this is becoming first line in many hospitals.
Classification
There has been a move away from named classification systems towards descriptive classification systems.
Two main types of NOF exist: Intra-capsular, and extra-capsular. Extra-capsular fractures are further divided into pertrochanteric or subtrochanteric (within 5cm distal to the lesser trochanter). All fractures are then described as undisplaced, minimally displaced, or displaced.
Femoral neck and head blood supply disruption is common with intracapsular NOF fractures, and rare with extracapsular fractures. This fundamental principle underpins the practise of arthroplasty for intracapsular fractures, and fixation for extracapsular fractures.
If you wish to use a named classification system, the most commonly used are below:
Elderly intracapsular - Garden Classification
Young intrasapsular - Pauvels Classification
Intertrochanteric - Evans
Subtrochanteric - Russell Taylor
Treatment
In general, NOF fractures are treated operatively except if the patient is deemed unlikely to survive an anaesthetic. Best Practice Tarif (BPT) dictates that surgery should happen within 36 hours, as delay of greater than 48 hours is associated with increased morbidity and mortality. Below are suggested algorithms for the treatment of NOF. There are some areas of debate/controversy which are detailed below.
- The priority with the young patient is to retain the femoral head if possible, even with a displaced intracapsular fracture. The risk of avascular necrosis and non-union (and therefore revision surgery) associated with internal fixation needs weighing up against the sequelae of total hip replacement in the young (wear, dislocation, revision). Discussion is necessary with the patient, on a case by case basis.
** Undisplaced fractures in the elderly can be treated with internal fixation, often with cannulated screws. This is appropriate for valgus impacted subcapital fractures which are inherently stable, to prevent secondary displacement. This does still carry the risk of AVN or non-union, and therefore a future revision. For this reason, many surgeons advocate arthroplasty as a single surgery.
*** NICE guidance - patients who fulfil these criteria should be offered total hip replacement which conveys better function and prosthetic survivorship, compared with hemiarthroplasty, but at an increased risk of dislocation.
* Intertrochanteric fractures vary greatly in their stability. If the trochanter (and therefore lateral wall), and medial calcar is in tact, then the fracture configuration bears stability. This can be treated with a DHS, as collapse of the fracture is predictable. Where either or both structures are involved in the fracture, stability becomes compromised and many surgeons will favour using an intramedullary device. This is an ongoing debate, and difficult to test in an exam setting.
Post operative management
Patients should be mobilised fully weight bearing where possible. Care is multidisciplinary in its delivery. Elderly patients should have orthogeriatrician assessment of comorbidity, and bone health with secondary prevention measures if appropriate. There should be early involvement of physiotherapy and occupational therapy services.
At which of the following vertebral body levels does the common carotid artery typically bifurcate into the external and internal carotid arteries?
C4
C2
C1
C6
C7
It terminates at the upper border of the thyroid cartilage, Which is usually located at C4. Common carotid artery
The right common carotid artery arises at the bifurcation of the brachiocephalic trunk, the left common carotid arises from the arch of the aorta. Both terminate at the level of the upper border of the thyroid cartilage (the lower border of the third cervical vertebra) by dividing into the internal and external carotid arteries.
Left common carotid artery
This vessel arises immediately to the left and slightly behind the origin of the brachiocephalic trunk. Its thoracic portion is 2.5- 3.5 cm in length and runs superolaterally to the sternoclavicular joint.
In the thorax
The vessel is in contact, from below upwards, with the trachea, left recurrent laryngeal nerve, left margin of the oesophagus. Anteriorly the left brachiocephalic vein runs across the artery, and the cardiac branches from the left vagus descend in front of it. These structures together with the thymus and the anterior margins of the left lung and pleura separate the artery from the manubrium.
In the neck
The artery runs superiorly deep to sternocleidomastoid and then enters the anterior triangle. At this point it lies within the carotid sheath with the vagus nerve and the internal jugular vein. Posteriorly the sympathetic trunk lies between the vessel and the prevertebral fascia. At the level of C7 the vertebral artery and thoracic duct lie behind it. The anterior tubercle of C6 transverse process is prominent and the artery can be compressed against this structure (it corresponds to the level of the cricoid).
Anteriorly at C6 the omohyoid muscle passes superficial to the artery.
Within the carotid sheath the jugular vein lies lateral to the artery.
Right common carotid artery
The right common carotid arises from the brachiocephalic artery. The right common carotid artery corresponds with the cervical portion of the left common carotid, except that there is no thoracic duct on the right. The oesophagus is less closely related to the right carotid than the left.
Summary points about the carotid anatomy
Path
Passes behind the sternoclavicular joint (12% patients above this level) to the upper border of the thyroid cartilage, to divide into the external (ECA) and internal carotid arteries (ICA).
Relations
Level of 6th cervical vertebra crossed by omohyoid
Then passes deep to the thyrohyoid, sternohyoid, sternomastoid muscles.
Passes anterior to the carotid tubercle (transverse process 6th cervical vertebra)-NB compression here stops haemorrhage.
The inferior thyroid artery passes posterior to the common carotid artery.
Then : Left common carotid artery crosses the thoracic duct, Right common carotid artery crossed by recurrent laryngeal nerve
A female infant is born by cesarean section at 38 weeks gestation for foetal distress. The attending paediatricians notice that she has a single palmar crease and prominent epicanthic folds of the eyes. Soon after the birth the mother tries to feed the child who has a projectile vomit about 10 minutes after feeding. On examination, she has a soft, non distended abdomen. What is the most likely cause?
Pyloric stenosis
Duodenal atresia
Milk allergy
Meconium ileus
Anal atresia
Proximally sited atresia will produce high volume vomits which may or may not be bile stained. Abdominal distension is characteristically absent. Whilst under resuscitated children may be a little dehydrated they are seldom severely ill. The presence of Trisomy 21 (palmar and eye signs) increases the likelihood of duodenal atresia.
Bilious vomiting in neonates
Causes of intestinal obstruction with bilious vomiting in neonates
Disorder Incidence and causation Age at presentation Diagnosis Treatment
Duodenal atresia 1 in 5000 (higher in Downs syndrome) Few hours after birth AXR shows ‘double bubble sign, contrast study may confirm Duodenoduodenostomy
Malrotation with volvulus Usually cause by incomplete rotation during embryogenesis Usually 3-7 days after birth, volvulus with compromised circulation may result in peritoneal signs and haemodynamic instability Upper GI contrast study may show DJ flexure is more medially placed, USS may show abnormal orientation of SMA and SMV Ladd’s procedure
Jejunal/ ileal atresia Usually caused by vascular insufficiency in utero, usually 1 in 3000 Usually within 24 hours of birth AXR will show air-fluid levels Laparotomy with primary resection and anastomosis
Meconium ileus Occurs in between 15 and20% of those babies with cystic fibrosis, otherwise 1 in 5000 Typically in first 24-48 hours of life with abdominal distension and bilious vomiting Air - fluid levels on AXR, sweat test to confirm cystic fibrosis Surgical decompression, serosal damage may require segmental resection
Necrotising enterocolitis Up to 2.4 per 1000 births, risks increased in prematurity and inter-current illness Usually second week of life Dilated bowel loops on AXR, pneumatosis and portal venous air Conservative and supportive for non perforated cases, laparotomy and resection in cases of perforation of ongoing clinical deteriorat
A 73 year old man develops a lesion affecting the lobule of his ear. Into which of the following lymph node groups will the lesion drain?
Deep cervical
Superficial cervical
Retroauricular nodes
Occipital nodes
Internal jugular nodes
The lobule of the ear drains to the superficial cervical nodes. Lymphatic drainage of the auricle
Lymphatic Drainage
The lateral surface of the upper half drains to the superficial parotid lymph nodes
The cranial surface of the superior half drains to the mastoid nodes and deep cervical lymph nodes
The lower half and lobule drain into the superficial cervical lymph nodes.
In Froment’s test which muscle function is tested?
Flexor pollicis longus
Abductor pollicis longus
Abductor pollicis brevis
Adductor pollicis
Opponens pollicis
Fromen’s test, also known as the Fromen (or Froment) sign, is a clinical test used to evaluate the function of the ulnar nerve, specifically assessing the strength of the adductor pollicis muscle. Here is how it is typically performed:
- The patient is asked to hold a piece of paper between their thumb and index finger.
- The examiner attempts to pull the paper away.
In a positive Fromen’s sign, the patient compensates for a weak adductor pollicis by flexing the thumb’s interphalangeal joint, which is facilitated by the flexor pollicis longus muscle, innervated by the median nerve. This compensation indicates weakness or paralysis of the adductor pollicis muscle due to ulnar nerve dysfunction.
Fromen’s test can be useful in diagnosing conditions that affect the ulnar nerve, such as cubital tunnel syndrome or ulnar neuropathy.
Assess for ulnar nerve palsy
Adductor pollicis muscle function tested
Hold a piece of paper between their thumb and index finger. The object is then pulled away. If ulnar nerve palsy, unable to hold the paper and will flex the flexor pollicis longus to compensate (flexion of thumb at interphalangeal joint).
Phalen’s test
Assess carpal tunnel syndrome
More sensitive than Tinel’s sign
Hold wrist in maximum flexion and the test is positive if there is numbness in the median nerve distribution.
Tinel’s sign
Assess for carpal tunnel syndrome
Tap the median nerve at the wrist and the test is positive if there is tingling/electric-like sensations over the distribution of the median nerve.
A 33 year old lady is 32 weeks pregnant and it is suspected that there may be a perforated abdominal viscus. What is the most appropriate course of action?
Arrange an abdominal CT scan
Arrange an abdominal MRI scan
Undertake a laparotomy
Undertake a laparoscopy
Undertake diagnostic peritoneal lavage
The keyword here is suspected. That being the case the investigation is CT. Although it is known that ionising radiation exposure can lead to cell death, mutation of germ cells, and carcinogenesis, there is no common modern radiographic procedure that results in radiation exposure to a level that threatens embryo or fetal well-being.Radiation exposure of <5 rads (a computed tomographic abdomen/pelvis study carries an exposure of 3.5 rads) has not been associated with fetal defects or loss. Careful shielding of the patient can also minimise exposure. Importantly, the use of ultrasound for diagnosis is clearly safe in pregnancy, although it should be used to evaluate and answer a defined clinical problem. A negative laparoscopy/ laparotomy poses signifcant foetal risks and at either procedure the enlarged uterus can result in missed pathology.
Abdominal imaging in pregnancy
Pregnant patients will often present with abdominal pain and the co-existing physiological leucocytosis can make laboratory assessment less reliable. In the non pregnant patient there is a low threshold for cross sectional imaging with CT scanning. The main foetal risks from cross sectional imaging in pregnancy are delayed rather than immediate. That being the case, where CT is likely to facilitate accurate diagnosis that improves short term foetal outcomes or saves foetal life, it is justified.
At all times, alternatives to CT need to be considered and utilised where possible. MRI is often used though for complex sequences movement artifact can limit the quality of views obtained. The use of shields can also limit radiation exposure to the foetus.
A 77-year-old man who is a smoker, collapses 12 days following an esophagectomy. Prior to his collapse he complained of a sudden onset of chest pain and dyspnea on examination is blood pressure is 90/60 mmHg. His ECG shows right heart strain. What is the most likely explanation?
Pulmonary embolism
Simple pneumothorax
Tension pneumothorax
Atelectesis
Myocardial infarction
The event occurring at day 12 with right sided ECG changes all favor PE. Surgery and cancer are risk factors. Pneumothorax would tend to occur much earlier in the post operative period.
Pulmonary Embolism: ECG changes
- No changes
S1, Q3, T3
Tall R waves: V1
P pulmonale (peaked P waves): inferior leads
Right axis deviation, Right bundle branch block
Atrial arrhythmias
T wave inversion: V1, V2, V3
Right ventricular strain: if identified is associated with adverse short-term outcome and adds prognostic value to echocardiographic evidence of right ventricular dysfunction in patients with acute pulmonary embolism and normal blood pressure.
An obese 12 year old boy is referred with pain in the left knee and hip. On examination, he has an antaglic gait and limitation of internal rotation. His knee has normal range of passive and active movement. What is the most likely diagnosis?
Septic arthritis
Developmental dysplasia of the hip
Perthes disease
Osteoarthritis of the hip
Slipped upper femoral epiphysis
Based on the information provided, the most likely diagnosis for this 12-year-old obese boy is:
Slipped Upper Femoral Epiphysis (SUFE)
Here’s why this is the most probable diagnosis:
- Age and body habitus: SUFE typically occurs in adolescents, often during growth spurts. It is more common in obese children, which matches the description of this patient.
- Symptoms:
- Pain in the hip and knee: SUFE often presents with hip pain that can be referred to the knee.
- Antalgic gait: This is a common finding in SUFE, where the patient tries to avoid putting weight on the affected side.
- Physical examination:
- Limitation of internal rotation: This is a classic sign of SUFE. The slippage of the femoral head causes restricted internal rotation of the hip.
- Normal knee examination: The knee pain is referred from the hip, so the knee itself shows no abnormalities on examination.
Let’s briefly consider why the other options are less likely:
- Septic arthritis: This would typically present with more acute symptoms, fever, and more severe limitation of movement.
- Developmental dysplasia of the hip: This is usually diagnosed in infancy or early childhood, not at 12 years old.
- Perthes disease: While possible, it’s less common in obese children and typically affects younger children (4-8 years old).
- Osteoarthritis of the hip: This is extremely rare in a 12-year-old and would not be a primary consideration.
Given the patient’s age, obesity, symptoms, and examination findings, Slipped Upper Femoral Epiphysis (SUFE) is the most likely diagnosis. This condition requires prompt recognition and treatment to prevent further slippage and potential complications.
Slipped upper femoral epiphysis is commonest in obese adolescent males. The x-ray will show displacement of the femoral epiphysis inferolaterally. Treatment is usually with rest and non weight bearing crutches.
Paediatric orthopaedics
Diagnosis Mode of presentation Treatment Radiology
Developmental dysplasia of the hip Usually diagnosed in infancy by screening tests. May be bilateral, when disease is unilateral there may be leg length inequality. As disease progresses child may limp and then early onset arthritis. More common in extended breech babies. Splints and harnesses or traction. In later years osteotomy and hip realignment procedures may be needed. In arthritis a joint replacement may be needed. However, this is best deferred if possible as it will almost certainly require revision Initially no obvious change on plain films and USS gives best resolution until 3 months of age. On plain films Shentons line should form a smooth arc
Perthes Disease Hip pain (may be referred to the knee) usually occurring between 5 and 12 years of age. Bilateral disease in 20%. Remove pressure from joint to allow normal development. Physiotherapy. Usually self-limiting if diagnosed and treated promptly. X-rays will show flattened femoral head. Eventually in untreated cases the femoral head will fragment.
Slipped upper femoral epiphysis Typically seen in obese male adolescents. Pain is often referred to the knee. Limitation to internal rotation is usually seen. Knee pain is usually present 2 months prior to hip slipping. Bilateral in 20%. Bed rest and non-weight bearing. Aim to avoid avascular necrosis. If severe slippage or risk of it occurring then percutaneous pinning of the hip may be required. X-rays will show the femoral head displaced and falling inferolaterally (like a melting ice cream cone) The Southwick angle gives indication of disease severity
In the distal third of the upper arm, where is the musculocutaneous nerve located?
Between the biceps brachii and brachialis muscles
Between the brachialis and brachioradialis muscles
Between the brachioradialis and triceps muscles
Between the brachialis and triceps muscles
Between the humerus and brachialis muscles
The musculocutaneous nerve lies between the biceps and brachialis muscles.
Musculocutaneous nerve
Branch of lateral cord of brachial plexus
Path
It penetrates the coracobrachialis muscle
Passes obliquely between the biceps brachii and the brachialis to the lateral side of the arm
Above the elbow it pierces the deep fascia lateral to the tendon of the biceps brachii
Continues into the forearm as the lateral cutaneous nerve of the forearm
Innervates
Coracobrachialis
Biceps brachii
Brachialis
Which of the following tumours are most likely to give rise to para-aortic nodal metastasis early?
Ovarian
Bladder
Rectal
Caecal
Cervical
Ovarian tumours are supplied by the ovarian vessels, these branch directly from the aorta. The cervix drains to the internal and external iliac nodes.
Para-aortic lymphadenopathy
Organ sites that may metastasise (early) to the para-aortic lymph nodes:
Testis
Ovary
Uterine fundus
Many other organs may result in para-aortic nodal disease. However, these deposits will represent a much later stage of the disease, since other nodal stations are involved earlier.
A 76 year old man complains of symptoms of claudication. The decision is made to measure his ankle brachial pressure index. The signal from the dorsalis pedis artery is auscultated with a hand held doppler device. This vessel is the continuation of which of the following?
Posterior tibial artery
Anterior tibial artery
Peroneal artery
Popliteal artery
None of the above
The dorsalis pedis is a continuation of the anterior tibial artery.
Foot- anatomy
Arches of the foot
The foot is conventionally considered to have two arches.
The longitudinal arch is higher on the medial than on the lateral side. The posterior part of the calcaneum forms a posterior pillar to support the arch. The lateral part of this structure passes via the cuboid bone and the lateral two metatarsal bones. The medial part of this structure is more important. The head of the talus marks the summit of this arch, located between the sustentaculum tali and the navicular bone. The anterior pillar of the medial arch is composed of the navicular bone, the three cuneiforms and the medial three metatarsal bones.
The transverse arch is situated on the anterior part of the tarsus and the posterior part of the metatarsus. The cuneiforms and metatarsal bases narrow inferiorly, which contributes to the shape of the arch.
Intertarsal joints
Sub talar joint Formed by the cylindrical facet on the lower surface of the body of the talus and the posterior facet on the upper surface of the calcaneus. The facet on the talus is concave anteroposteriorly, the other is convex. The synovial cavity of this joint does not communicate with any other joint.
Talocalcaneonavicular joint The anterior part of the socket is formed by the concave articular surface of the navicular bone, posteriorly by the upper surface of the sustentaculum tali. The talus sits within this socket
Calcaneocuboid joint Highest point in the lateral part of the longitudinal arch. The lower aspect of this joint is reinforced by the long plantar and plantar calcaneocuboid ligaments.
Transverse tarsal joint The talocalcaneonavicular joint and the calcaneocuboid joint extend across the tarsus in an irregular transverse plane, between the talus and calcaneus behind and the navicular and cuboid bones in front. This plane is termed the transverse tarsal joint.
Cuneonavicular joint Formed between the convex anterior surface of the navicular bone and the concave surface of the the posterior ends of the three cuneiforms.
Intercuneiform joints Between the three cuneiform bones.
Cuneocuboid joint Between the circular facets on the lateral cuneiform bone and the cuboid. This joint contributes to the tarsal part of the transverse arch.
A detailed knowledge of the joints is not required for MRCS Part A. However, the contribution they play to the overall structure of the foot should be appreciated
Muscles of the foot
Muscle Origin Insertion Nerve supply Action
Abductor hallucis Medial side of the calcaneus, flexor retinaculum, plantar aponeurosis Medial side of the base of the proximal phalanx Medial plantar nerve Abducts the great toe
Flexor digitorum brevis Medial process of the calcaneus, plantar eponeurosis. Via 4 tendons into the middle phalanges of the lateral 4 toes. Medial plantar nerve Flexes all the joints of the lateral 4 toes except for the interphalangeal joint.
Abductor digit minimi From the tubercle of the calcaneus and from the plantar aponeurosis Together with flexor digit minimi brevis into the lateral side of the base of the proximal phalanx of the little toe Lateral plantar nerve Abducts the little toe at the metatarsophalangeal joint
Flexor hallucis brevis From the medial side of the plantar surface of the cuboid bone, from the adjacent part of the lateral cuneiform bone and from the tendon of tibialis posterior. Into the proximal phalanx of the great toe, the tendon contains a sesamoid bone Medial plantar nerve Flexes the metatarsophalangeal joint of the great toe.
Adductor hallucis Arises from two heads. The oblique head arises from the sheath of the peroneus longus tendon, and from the plantar surfaces of the bases of the 2nd, 3rd and 4th metatarsal bones. The transverse head arises from the plantar surface of the lateral 4 metatarsophalangeal joints and from the deep transverse metatarsal ligament. Lateral side of the base of the proximal phalanx of the great toe. Lateral plantar nerve Adducts the great toe towards the second toe. Helps maintain the transverse arch of the foot.
Extensor digitorum brevis On the dorsal surface of the foot from the upper surface of the calcaneus and its associated fascia Via four thin tendons which run forward and medially to be inserted into the medial four toes. The lateral three tendons join with hoods of extensor digitorum longus. Deep peroneal Extend the metatarsophalangeal joint of the medial four toes. It is unable to extend the interphalangeal joint without the assistance of the lumbrical muscles.
Nerves in the foot
Lateral plantar nerve
Passes anterolaterally towards the base of the 5th metatarsal between flexor digitorum brevis and flexor accessorius. On the medial aspect of the lateral plantar artery. At the base of the 5th metatarsal it splits into superficial and deep branches.
Medial plantar nerve
Passes forwards with the medial plantar artery under the cover of the flexor retinaculum to the interval between abductor hallucis and flexor digitorum brevis on the sole of the foot.
Plantar arteries
Arise under the cover of the flexor retinaculum, midway between the tip of the medial malleolus and the most prominent part of the medial side of the heel.
Medial plantar artery. Passes forwards medial to medial plantar nerve in the space between abductor hallucis and flexor digitorum brevis.Ends by uniting with a branch of the 1st plantar metatarsal artery.
Lateral plantar artery. Runs obliquely across the sole of the foot. It lies lateral to the lateral plantar nerve. At the base of the 5th metatarsal bone it arches medially across the foot on the metatarsals
Dorsalis pedis artery
This vessel is a direct continuation of the anterior tibial artery. It commences on the front of the ankle joint and runs to the proximal end of the first metatarsal space. Here is gives off the arcuate artery and continues forwards as the first dorsal metatarsal artery. It is accompanied by two veins throughout its length. It is crossed by the extensor hallucis brevis
Loss of taste sensation from the posterior third of the tongue is most likely the result to an injury to which of the structures listed below?
Hypoglossal nerve
Chorda tympani nerve
Facial nerve
Mandibular branch of the trigeminal nerve
Glossopharyngeal nerve
The glossopharyngeal nerve supplies the taste and general sensation to the posterior third of the tongue.
Tongue
The tongue is a muscular organ concerned with both feeding and speech. It is shaped like an inverted boot with a root through which extrinsic muscles pass to attach it to the soft palate, the styloid process, the hyoid bone and the mandible. It has a body that comprises muscle intermingled with lingual salivary glands, connective tissue, nerves, vessels and lymphatic follicles.
The dorsum of the tongue is divided by an ill defined V shaped groove, the sulcus terminalis, into an anterior two thirds and a posterior third.
The arterial blood supply to the tongue is from the paired sets of lingual arteries that arise from the external carotid. The venous drainage is via dorsal lingual veins which then join the lingual veins and eventually into the sublingual vein.
The nerve supply is as follows:
The lingual branch of the mandibular division of the trigeminal nerve supplies common sensation to the anterior two thirds
The chorda tympani branch of the facial nerve supplies taste to the anterior two thirds
The lingual branch of the glossopharyngeal nerve supplies common sensation and taste to the posterior third of the tongue
All intrinsic muscles of the tongue are supplied by the hypoglossal nerve
A 33 year old female is admitted for varicose vein surgery. She is fit and well. After the procedure she is persistently bleeding. She is known to have menorrhagia. Investigations show a prolonged bleeding time and increased APTT. She has a normal PT and platelet count. What is the most likely cause?
Anti phospholipid syndrome
Haemophilia
Factor V Leiden deficiency
von Willebrands disease
Protein C and S deficiency
Bleeding post operatively, epistaxis and menorrhagia may indicate a diagnosis of vWD. Haemoarthroses are rare. The bleeding time is usually normal in haemophilia (X-linked) and vitamin K deficiency.
von Willebrands disease
- Most common inherited bleeding disorder
All vWD is caused by mutations in the gene for von Willebrand factor. von Willebrand factor is an adhesive glycoprotein that is secreted by endothelium and megakaryocytes
von Willebrand factor promotes platelet adhesion to damaged endothelium and other platelets. It is also involved in the transport and stabilization of factor VIII
There are 7 subtypes of von Willebrand disease. The commonest is type I (autosomal dominant) which accounts for 80% of cases, type 2vWD (autosomal dominant or recessive) accounts for 15% of cases
There is a significant spectrum of severity ranging from spontaneous bleeding and epistaxis through to troublesome excessive bleeding following minor procedures
The test that is most commonly used are von willebrand factor assays
Treatments include administration of tranexamic acid for minor cases undergoing minor procedures. More significant bleeding or more significant procedures respond well to DDAVP. This is most effective in type I, less effective in type 2 and contraindicated in type 2B. Patients with type 3 disease do not respond to DDAVP as they lack the ability to secrete vWF
Individuals who cannot have DDAVP or in whom it is contra indicated usually receive factor VIII concentrates containing vWF
Administration of neostigmine to a patient who has received a non depolarizing muscle relaxant is most likely to result in which of the following?
Prolongation of muscle relaxation
Tachycardia
Hypertension
Bradycardia
Decreased gut peristalsis
Neostigmine can cause bradycardia and atropine is often administered concomitantly to counter this effect.
Muscle relaxants
Suxamethonium
Depolarising neuromuscular blocker
Inhibits action of acetylcholine at the neuromuscular junction
Degraded by plasma cholinesterase and acetylcholinesterase (affected by lack of acetylcholinesterase)
Fastest onset and shortest duration of action of all muscle relaxants
Produces generalised muscular contraction prior to paralysis
Adverse effects include hyperkalaemia, malignant hyperthermia, delayed recovery
Atracurium
Non depolarising neuromuscular blocking drug
Duration of action usually 30-45 minutes
Generalised histamine release on administration may produce facial flushing, tachycardia and hypotension
Not excreted by liver or kidney, broken down in tissues by hydrolysis
Reversed by neostigmine
Vecuronium
Non depolarising neuromuscular blocking drug
Duration of action approximately 30 - 40 minutes
Degraded by liver and kidney and effects prolonged in organ dysfunction
Effects may be reversed by neostigmine
Pancuronium
Non depolarising neuromuscular blocker
Onset of action approximately 2-3 minutes
Duration of action up to 2 hours
Effects may be partially reversed with drugs such as neostigmine
A 22 year old is found to have bilateral acoustic neuromas. Which of the disorders listed below is most likely to be present?
Neurofibromatosis Type I
Neurofibromatosis Type II
MEN I
MEN II
Gardner’s syndrome
In NF2 bilateral acoustic neuromas are characteristic with a family history of Neurofibroma.
Genetics and surgical disease
Some of the more commonly occurring genetic conditions occurring in surgical patients are presented here.
Li-Fraumeni Syndrome
Autosomal dominant
Consists of germline mutations to p53 tumour suppressor gene
High incidence of malignancies particularly sarcomas and leukaemias
Diagnosed when:
*Individual develops sarcoma under 45 years
*First degree relative diagnosed with any cancer below age 45 years and another family member develops malignancy under 45 years or sarcoma at any age
BRCA 1 and 2
Carried on chromosome 17 (BRCA 1) and Chromosome 13 (BRCA 2)
Linked to developing breast cancer (60%) risk.
Associated risk of developing ovarian cancer (55% with BRCA 1 and 25% with BRCA 2).
Lynch Syndrome
Autosomal dominant
Develop colonic cancer and endometrial cancer at young age
80% of affected individuals will get colonic and/ or endometrial cancer
High risk individuals may be identified using the Amsterdam criteria
Amsterdam criteria
Three or more family members with a confirmed diagnosis of colorectal cancer, one of whom is a first degree (parent, child, sibling) relative of the other two.
Two successive affected generations.
One or more colon cancers diagnosed under age 50 years.
Familial adenomatous polyposis (FAP) has been excluded.
Gardners syndrome
Autosomal dominant familial colorectal polyposis
Multiple colonic polyps
Extra colonic diseases include: skull osteoma, thyroid cancer and epidermoid cysts
Desmoid tumours are seen in 15%
Mutation of APC gene located on chromosome 5
Due to colonic polyps most patients will undergo colectomy to reduce risk of colorectal cancer
Now considered a variant of familial adenomatous polyposis coli
During an operation for varicose veins the surgeons are mobilising the long saphenous vein. Near its point of entry to the femoral vein an artery is injured and bleeding is encountered. From where is the bleeding most likely to originate?
Femoral artery
Profunda femoris artery
Superficial circumflex iliac artery
Superficial epigastric artery
Deep external pudendal artery
The deep external pudendal artery is a branch of the SFA and it runs medially under the long saphenous vein near its point of union with the femoral vein. The superficial external pudendal artery lies superior to the SFJ. Neither vessel is functionally important and if injured they are best ligated.
Femoral triangle anatomy
Boundaries
Superiorly Inguinal ligament
Laterally Sartorius
Medially Adductor longus
Floor Iliopsoas, adductor longus and pectineus
Roof
Fascia lata and Superficial fascia
Superficial inguinal lymph nodes (palpable below the inguinal ligament)
Long saphenous vein
Which of the nerves listed below is responsible for the innervation of gluteus medius?
Inferior gluteal nerve
Sciatic nerve
Superior gluteal nerve
Perineal nerve
Inferior rectal nerve
Superior and inferior gluteal nerve
Superior gluteal nerve
Arises from dorsal surface of the sacral plexus (L4, L5, S1)
Passes into gluteal region together with superior gluteal vessels
Supplies gluteus medius and minimus
Inferior gluteal nerve
Arises from dorsal surface of sacral plexus (L5, S1 and S2)
Runs medial to the posterior femoral cutaneous nerve
Enters gluteal region at inferior border of piriformis
Supplies gluteus maximus
A 73 year old man is recovering from a stroke but is deemed to have an unsafe swallow. Apart from his CVA his past medical history includes rate controlled atrial fibrillation and a previous oesophagectomy. What is the best option for long term feeding?
Endoscopically inserted PEG tube
Surgically inserted PEG tube
Surgically inserted feeding jejunostomy tube
TPN via peripheral venous access system
TPN via a central line
Given the patient’s history of a previous esophagectomy and the current need for long-term feeding due to an unsafe swallow, the best option for long-term feeding would be:
Surgically inserted feeding jejunostomy tube
Here’s why:
- Previous Esophagectomy: This history complicates the placement of a percutaneous endoscopic gastrostomy (PEG) tube, whether endoscopic or surgical, as the anatomy may be altered significantly.
- Safety Concerns: A feeding jejunostomy tube can bypass the stomach and esophagus altogether, effectively preventing potential complications related to his previous esophagectomy.
- Long-term Feeding: A surgically inserted jejunostomy tube is suitable for long-term nutritional support, especially in patients where the upper gastrointestinal tract cannot be used safely.
- TPN (Total Parenteral Nutrition): While TPN can be used for nutritional support, it is generally considered as a last resort due to the high risk of complications such as infections, liver dysfunction, and metabolic disturbances, especially over the long term. Peripheral TPN is not suitable for long-term use, and central TPN carries significant risks.
Therefore, the most appropriate and safest option for long-term feeding in this patient is the surgically inserted feeding jejunostomy tube.
A surgical resection specimen is analysed histologically. The pathologist comments that at the periphery of the resected specimen, oxyphil cells are identified. In which of the structures listed below are these cells typically found?
Thymus
Thyroid gland
Parathyroid gland
Lymph node
Adrenal gland
Oxyphil cells are typically found in parathyroid glands. Parathyroid glands- anatomy
Four parathyroid glands
Located posterior to the thyroid gland
They lie within the pretracheal fascia
Embryology
The parathyroids develop from the extremities of the third and fourth pharyngeal pouches. The parathyroids derived from the fourth pharyngeal pouch are located more superiorly and are associated with the thyroid gland. Those derived from the third pharyngeal pouch lie more inferiorly and may become associated with the thymus.
Blood supply
The blood supply to the parathyroid glands is derived from the inferior and superior thyroid arteries[1]. There is a rich anastomosis between the two vessels. Venous drainage is into the thyroid veins.
Relations
Laterally Common carotid
Medially Recurrent laryngeal nerve, trachea
Anterior Thyroid
Posterior Pretracheal fascia
In which of the conditions described below is Rovsing’s sign most likely to be absent?
Locally advanced caecal cancer
Para ileal appendicitis
Right sided colonic diverticulitis
Retrocaecal appendicitis
Severe terminal ileal Crohns disease
Any advanced right iliac fossa pathology can result in a positive Rovsings sign. However, in retrocaecal appendicitis, it may be absent and this fact can contribute to a delayed diagnosis if undue weight is placed on the presence of the sign in making the diagnosis.
A 22 year old man presents with an aching pain and discomfort in his right testicle. He has felt systemically unwell for the preceding 48 hours. On examination, there is tenderness of the right testicle. He has an exaggerated cremasteric reflex. What is the correct course of action?
Scrotal exploration via a scrotal approach
Testicular inspection via an inguinal approach
Administration of antibiotics
Fine needle aspiration cytology
Reassure and discharge
This is likely to represent epididymo-orchitis, this is usually due to infection with gonorrhoea or chlamydia in this age group. In addition to treatment with antibiotics contact tracing and appropriate swabs should also be performed.
Testicular disorders
Testicular cancer
Testicular cancer is the most common malignancy in men aged 20-30 years. Around 95% of cases of testicular cancer are germ-cell tumours. Germ cell tumours may essentially be divided into:
Tumour type Key features Tumour markers Pathology
Seminoma
Commonest subtype (50%)
Average age at diagnosis = 40
Even advanced disease associated with 5 year survival of 73%
AFP usually normal
HCG elevated in 10% seminomas
Lactate dehydrogenase; elevated in 10-20% seminomas (but also in many other conditions)
Sheet like lobular patterns of cells with substantial fibrous component. Fibrous septa contain lymphocytic inclusions and granulomas may be seen.
Non seminomatous germ cell tumours (42%)
Teratoma
Yolk sac tumour
Choriocarcinoma
Mixed germ cell tumours (10%)
Younger age at presentation =20-30 years
Advanced disease carries worse prognosis (48% at 5 years)
Retroperitoneal lymph node dissection may be needed for residual disease after chemotherapy
Risk factors for testicular cancer
Cryptorchidism
Infertility
Family history
Klinefelter’s syndrome
Mumps orchitis
Features
A painless lump is the most common presenting symptom
Pain may also be present in a minority of men
Other possible features include hydrocele, gynaecomastia
Diagnosis
Ultrasound is first-line
CT scanning of the chest/ abdomen and pelvis is used for staging
Tumour markers (see above) should be measured
Management
Orchidectomy (Inguinal approach)
Chemotherapy and radiotherapy may be given depending on staging
Abdominal lesions >1cm following chemotherapy may require retroperitoneal lymph node dissection.
Prognosis is generally excellent
5 year survival for seminomas is around 95% if Stage I
5 year survival for teratomas is around 85% if Stage I
Benign disease
Epididymo-orchitis
Acute epididymitis is an acute inflammation of the epididymis, often involving the testis and usually caused by bacterial infection.
Infection spreads from the urethra or bladder. In men <35 years, gonorrhoea or chlamydia are the usual infections.
Amiodarone is a recognised non infective cause of epididymitis, which resolves on stopping the drug.
Tenderness is usually confined to the epididymis, which may facilitate differentiating it from torsion where pain usually affects the entire testis.
Testicular torsion
Twist of the spermatic cord resulting in testicular ischaemia and necrosis.
Most common in males aged between 10 and 30 (peak incidence 13-15 years)
Pain is usually severe and of sudden onset.
Cremasteric reflex is lost and elevation of the testis does not ease the pain.
Treatment is with surgical exploration. If a torted testis is identified then both testis should be fixed as the condition of bell clapper testis is often bilateral.
Hydrocele
Presents as a mass that transilluminates, usually possible to ‘get above’ it on examination.
In younger men it should be investigated with USS to exclude tumour.
In children it may occur as a result of a patent processus vaginalis.
Treatment in adults is with a Lords or Jabouley procedure.
Treatment in children is with trans inguinal ligation of PPV.
A 4 year old is admitted with right iliac fossa pain and is due to undergo an appendicectomy. The nursing staff would like to give the child an infusion of intravenous fluid whilst waiting for theatre. Assuming electrolytes are normal, which of the following is an appropriate fluid for infusion in this situation?
10% Dextrose solution
0.9% Saline solution
0.45% saline/ 5% glucose solution
Gelofusin
None of the above
0.45% saline/ 5% glucose solutions carry a risk of hyponatraemia and is contra indicated- see below.
Paediatric fluid management
Since 2000 there have been at least 4 reported deaths from fluid induced hyponatraemia in children. This led to the National Patient Safety Agency introducing revised guidelines in 2007. These have been reviewed and extensively updated by NICE in 2015 and further modified in 2020.
Indications for IV fluids include:
Resuscitation and circulatory support
Replacing on-going fluid losses
Maintenance fluids for children for whom oral fluids are not appropriate
Correction of electrolyte disturbances
Fluids to be avoided
Outside the neonatal period saline / glucose solutions should not be given. The greatest risk is with saline 0.18 / glucose 4% solutions. The report states that 0.45% saline / 5% glucose may be used. But preference should be given to isotonic solutions and few indications exist for this solution either.
The key point emphasised in the NICE guidelines in the avoidance of glucose containing solutions and instead of routinely giving glucose IV to children and neonates, the blood glucose levels should be monitored. In neonates in the first few days of life, sodium levels in the range of 131-154 mmol/l may be too high and a neonatologist consulted on a case by case basis.
Fluids to be used
If children and young people need IV fluids for routine maintenance, initially use isotonic crystalloids that contain sodium in the range 131 to 154 mmol/litre
Potassium should be added to maintenance fluids according patients plasma potassium levels (which should be monitored).
Blood glucose levels should be monitored in individuals at risk of hypoglycaemia
Intraoperative fluid management
If children and young people need IV fluids for routine maintenance, initially use isotonic crystalloids that contain sodium in the range 131 to 154 mmol/litre
Blood glucose levels should be monitored
Maintenance fluids
Weight Water requirement/kg/day Na mmol/kg/day K mmol/kg/day
First 10Kg body weight 100ml 2-4 1.5-2.5
Second 10Kg body weight 50ml 1-2 0.5-1.5
Subsequent Kg 20ml 0.5-1.0 0.2-0.7
Blood glucose will need to be monitored
A 23 year old man is reviewed on the ward 10 days following a laparotomy. The wound is inspected and is healing well. Which of the following processes is least likely to be occurring in the wound at this stage?
Angiogenesis
Synthesis of collagen
Necrosis of fibroblasts
Secretion of matrix metalloproteinases by fibroblasts
Proliferation of fibroblasts
Wound Healing Processes at 10 Days Post-Laparotomy
Wound healing is a complex biological process that occurs in several overlapping phases: hemostasis, inflammation, proliferation, and remodeling. By the tenth day following a laparotomy, the wound is typically in the proliferative phase of healing. During this phase, several key processes are actively occurring:
Angiogenesis: This is the formation of new blood vessels from pre-existing ones. It is crucial for supplying nutrients and oxygen to the healing tissue. At 10 days post-surgery, angiogenesis is likely to be well underway as the body attempts to restore blood supply to the area.
Synthesis of Collagen: Fibroblasts play a significant role in synthesizing collagen, which provides structural support to the wound. By this stage, collagen deposition is prominent as part of the body’s efforts to strengthen the newly formed tissue.
Necrosis of Fibroblasts: This refers to cell death due to injury or lack of blood supply. In a healing wound at this stage, necrosis of fibroblasts would be less likely because these cells are essential for wound repair and are actively proliferating and synthesizing extracellular matrix components.
Secretion of Matrix Metalloproteinases (MMPs) by Fibroblasts: MMPs are enzymes that degrade various components of the extracellular matrix and play a role in tissue remodeling. While their secretion can occur during wound healing, it is more prominent later in the remodeling phase rather than at 10 days post-surgery when fibroblast activity focuses on synthesis rather than degradation.
Proliferation of Fibroblasts: This process involves an increase in fibroblast numbers as they migrate into the wound site and begin producing collagen and other extracellular matrix components necessary for repair.
Given these considerations, we can analyze which process is least likely occurring at this stage:
Angiogenesis is expected.
Synthesis of collagen is expected.
Proliferation of fibroblasts is expected.
Secretion of MMPs may occur but not as prominently as other processes.
Necrosis of fibroblasts would be atypical since they are vital for healing.
Based on this analysis, necrosis of fibroblasts is least likely to be occurring in the wound at this stage because fibroblasts are actively involved in healing rather than undergoing necrosis.
Answer: Necrosis of fibroblasts
A 43 year old man is due to undergo an excision of the sub mandibular gland. Which of the following incisions is the most appropriate for this procedure?
A transversely orientated incision 3cm below the mandible
A transversely orientated incision immediately inferior to the mandible
A vertical incision 3 cm anterior to the angle of the mandible and extending inferiorly
A transversely orientated incision 2cm above the mandible
A transversely orientated incision 12cm below the mandible
To access the sub mandibular gland a transverse incision 3cm below the mandible should be made. Incisions located higher than this may damage the marginal mandibular branch of the facial nerve.
Anterior triangle of the neck
Boundaries
Anterior border of the Sternocleidomastoid
Lower border of mandible
Anterior midline
Sub triangles (divided by Digastric above and Omohyoid)
Muscular triangle: Neck strap muscles
Carotid triangle: Carotid sheath
Submandibular Triangle (digastric)
Contents of the anterior triangle
Digastric triangle Submandibular gland
Submandibular nodes
Facial vessels
Hypoglossal nerve
Muscular triangle Strap muscles
Jugular vein
Carotid triangle Carotid sheath (Common carotid, vagus and internal jugular vein)
Ansa cervicalis
Nerve supply to digastric muscle
Anterior: Mylohyoid nerve
Posterior: Facial nerve
Which of the muscles listed below is not innervated by the median nerve?
Flexor pollicis brevis
Lateral two lumbricals
Pronator teres
Opponens pollicis
Adductor pollicis
Adductor pollicis is innervated by the ulnar nerve.
Medial two lumbricals innervated by the ulnar nerve.
Median nerve
The median nerve is formed by the union of a lateral and medial root respectively from the lateral (C5,6,7) and medial (C8 and T1) cords of the brachial plexus; the medial root passes anterior to the third part of the axillary artery. The nerve descends lateral to the brachial artery, crosses to its medial side (usually passing anterior to the artery). It passes deep to the bicipital aponeurosis and the median cubital vein at the elbow.
It passes between the two heads of the pronator teres muscle, and runs on the deep surface of flexor digitorum superficialis (within its fascial sheath).
Near the wrist it becomes superficial between the tendons of flexor digitorum superficialis and flexor carpi radialis, deep to palmaris longus tendon. It passes deep to the flexor retinaculum to enter the palm, but lies anterior to the long flexor tendons within the carpal tunnel.
Branches
Region Branch
Upper arm No branches, although the nerve commonly communicates with the musculocutaneous nerve
Forearm Pronator teres
Pronator quadratus
Flexor carpi radialis
Palmaris longus
Flexor digitorum superficialis
Flexor pollicis longus
Flexor digitorum profundus (only the radial half)
Distal forearm Palmar cutaneous branch
Hand (Motor) Motor supply (LOAF)
Lateral 2 lumbricals
Opponens pollicis
Abductor pollicis brevis
Flexor pollicis brevis
Hand (Sensory)
Over thumb and lateral 2 ½ fingers
On the palmar aspect this projects proximally, on the dorsal aspect only the distal regions are innervated with the radial nerve providing the more proximal cutaneous innervation.
Patterns of damage
Damage at wrist
e.g. carpal tunnel syndrome
paralysis and wasting of thenar eminence muscles and opponens pollicis (ape hand deformity)
sensory loss to palmar aspect of lateral (radial) 2 ½ fingers
Damage at elbow, as above plus:
unable to pronate forearm
weak wrist flexion
ulnar deviation of wrist
Anterior interosseous nerve (branch of median nerve)
leaves just below the elbow
results in loss of pronation of forearm and weakness of long flexors of thumb and index finger
In which of the following operations are Skew flaps created?
Hindquarter amputation
Above knee amputation
Gritti- Stokes amputation
Below knee amputation
Symes amputation
This is one variant of a below knee amputation. The Burgess flap is the other commonly practised approach.
Amputations
Amputations are indicated when the affected limb is one of the following:
Dead non viable
Deadly where it is posing a major threat to life
Dead useless where it is viable but a prosthesis would be preferable
Orthopaedic surgery
Amputation is often undertaken as an option of last resort e.g. Limb salvage has failed and the limb is so non functional that mobility needs would be best met with prosthesis.
Chronic fracture non union or significant limb shortening following trauma would fit into this category. Occasionally following major trauma a primary amputation is preferable. This would be the case in an open fracture with major distal neurovascular compromise and other more life threatening injuries are present.
Vascular surgery
The first two categories are the most prevalent.
Diabetic foot sepsis is often a major cause of sepsis which can spread rapidly in the presence of established peripheral vascular disease.
As a general rule the main issue in vascular surgery is to optimise vascular inflow prior to surgery. The more distal the planned amputation is to be, the more important this rule becomes.
In other situations there has been something such as an embolic event that has not been revascularised in time. In this case the limb shows fixed mottling and an amputation will be needed.
Types of amputations
As the vast majority of commonly performed amputations affect the lower limbs these will be covered here.
The main categories of amputations are:
Pelvic disarticulation (hindquarter)
Above knee amputation
Gritti Stokes (through knee amputation)
Below knee amputation (using either Skew or Burgess flaps)
Syme’s amputation (through ankle)
Amputations of mid foot and digits
Choosing a level of amputation depends on:
The disease process being treated
Desired functional outcome
Co-morbidities of the patient
Above knee amputations
Quick to perform
Heal reliably
Patients regain their general health quickly
For this benefit, a functional price has to be paid and many patients over the age of 70 will never walk on an above knee prosthesis.
Above knee amputations use equal anterior-posterior flaps
Below knee amputations
Technically more challenging to perform
Heal less reliably than their above knee counterparts.
However, many more patients are able to walk using a below knee prosthesis.
In below knee amputations the two main flaps are Skew flaps or the Burgess long posterior flap. Skew flaps result in a less bulky limb that is easier to attach a prosthesis to.
It is worth remembering that whilst it may be technically feasible to offer a below knee amputation there may be circumstances where an above knee option is preferable. For example, in fixed flexion deformities of the lower limb, little functional benefit would be gained from below knee amputation surgery.
A Medical F1 phones you as he is concerned his patient has had a major internal bleed. The patient is 42 years old and is known to have sickle cell anaemia. His blood results are:
Hb 3.7 g /dl
Reticulocyte count 0.4%
His Hb is normally 7g/dl. What is the diagnosis?
Psoas haemorrhage
Acute sequestration
Parvovirus
Splenic haemorrhage
Acute haemolysis
A sudden anemia and a LOW reticulocyte count indicates parvovirus. Acute sequestration and haemolysis causes a high reticulocyte count. There is no clinical indication to suspect a bleed, therefore you can advise the F1 not to panic and to speak to the haematologists!
Sickle cell anaemia
Autosomal recessive
Single base mutation
Deoxygenated cells become sickle in shape
Causes: short red cell survival, obstruction of microvessels and infarction
Sickling is precipitated by: dehydration, infection, hypoxia
Manifest at 6 months age
Africans, Middle East, Indian
Diagnosis: Hb electrophoresis
Sickle crises
Bone pain
Pleuritic chest pain: acute sickle chest syndrome commonest cause of death
CVA, seizures
Papillary necrosis
Splenic infarcts
Priapism
Hepatic pain
Hb does not fall during a crisis, unless there is
Aplasia: parvovirus
Acute sequestration
Haemolysis
Long-term complications
Infections: Streptococcus pnemoniae
Chronic leg ulcers
Gallstones: haemolysis
Aseptic necrosis of bone
Chronic renal disease
Retinal detachment, proliferative retinopathy
Surgical complications
Bowel ischaemia
Cholecystitis
Avascular necrosis
Management
Supportive
Hydroxyurea
Repeated transfusions pre operatively
Exchange transfusion in emergencies
Sickle cell trait
Heterozygous state
Asymptomatic
Symptoms associated with extreme situations ie anaesthesia complications
Protective against Plasmodium falciparum
A 73 year old man develops disseminated intravascular coagulation following an abdominal aortic aneurysm repair. He receives an infusion of cryoprecipitate. What is the major constituent of this infusion?
Factor VIII
Factor IX
Protein C
Protein S
Factor V
A- Cryoprecipitate
- Blood product made from plasma
Usually transfused as 6 unit pool
Indications include massive haemorrhage and uncontrolled bleeding due to haemophilia
Composition
Agent Quantity
Factor VIII 100IU
Fibrinogen 250mg
von Willebrand factor Variable
Factor XIII Variable
Which of the following local anaesthetics is not an amino amide type?
Lignocaine
Xylocaine
Procaine
Bupivacaine
Prilocaine
All local anaesthetics have a chemical bond linking an amine to either an amide or an ester. Most local anaesthetics are of the amino- amide types, these have a more favorable side effect profile and are more stable in solution. Procaine and benzocaine have amino - ester groups, these are metabolised by pseudocholinesterases.
Local anaesthetic agents
Lidocaine
An amide
Local anaesthetic and a less commonly used antiarrhythmic (affects Na channels in the axon)
Hepatic metabolism, protein bound, renally excreted
Toxicity: due to IV or excess administration. Increased risk if liver dysfunction or low protein states. Note acidosis causes lidocaine to detach from protein binding.
Drug interactions: Beta blockers, ciprofloxacin, phenytoin
Features of toxicity: Initial CNS over activity then depression as lidocaine initially blocks inhibitory pathways then blocks both inhibitory and activating pathways. Cardiac arrhythmias.
Increased doses may be used when combined with adrenaline to limit systemic absorption.
Cocaine
Pure cocaine is a salt, usually cocaine hydrochloride. It is supplied for local anaesthetic purposes as a paste.
It is supplied for clinical use in concentrations of 4 and 10%. It may be applied topically to the nasal mucosa. It has a rapid onset of action and has the additional advantage of causing marked vasoconstriction.
It is lipophillic and will readily cross the blood brain barrier. Its systemic effects also include cardiac arrhythmias and tachycardia.
Apart from its limited use in ENT surgery it is otherwise used rarely in mainstream surgical practice.
Bupivacaine
Bupivacaine binds to the intracellular portion of sodium channels and blocks sodium influx into nerve cells, which prevents depolarization.
It has a much longer duration of action than lignocaine and this is of use in that it may be used for topical wound infiltration at the conclusion of surgical procedures with long duration analgesic effect.
It is cardiotoxic and is therefore contra indicated in regional blockage in case the tourniquet fails.
Levobupivacaine (Chirocaine) is less cardiotoxic and causes less vasodilation.
Prilocaine
Similar mechanism of action to other local anaesthetic agents. However, it is far less cardiotoxic and is therefore the agent of choice for intravenous regional anaesthesia e.g. Biers Block.
All local anaesthetic agents dissociate in tissues and this contributes to their therapeutic effect. The dissociation constant shifts in tissues that are acidic e.g. where an abscess is present, and this reduces the efficacy.
Doses of local anaesthetics
Agent Dose plain Dose with adrenaline
Lignocaine 3mg/Kg 7mg/Kg
Bupivacaine 2mg/Kg 2mg/Kg
Prilocaine 6mg/Kg 9mg/Kg
These are a guide only as actual doses depend on site of administration, tissue vascularity and co-morbidities.
Maximum total local anaesthetic doses
Lignocaine 1% plain - 3mg/ Kg - 200mg (20ml)
Lignocaine 1% with 1 in 200,000 adrenaline - 7mg/Kg - 500mg (50ml)
Bupivacaine 0.5% - 2mg/kg- 150mg (30ml)
Maximum doses are based on ideal body weight
Effects of adrenaline
Adrenaline may be added to local anaesthetic drugs. It prolongs the duration of action at the site of injection and permits usage of higher doses (see above). It is contra indicated in patients taking MAOI’s or tricyclic antidepressants. The toxicity of bupivacaine is related to protein binding and addition of adrenaline to this drug does not permit increases in the total dose of bupivacaine, in contrast to the situation with lignocaine.
References
An excellent review is provided by:
French J and Sharp L. Local Anaesthetics. Ann R Coll Surg Engl 2012; 94: 76-80.
A 22 year old man is involved in a fight and is stabbed in the posterior aspect of his right leg. The knife passes into the popliteal fossa. He sustains an injury to his tibial nerve. Which muscle is least likely to be compromised as a result?
Tibialis posterior
Flexor hallucis longus
Flexor digitorum brevis
Soleus
Peroneus tertius
Peroneus tertius is innervated by the deep peroneal nerve.
Tibial nerve
Begins at the upper border of the popliteal fossa and is a branch of the sciatic nerve.
Root values: L4, L5, S1, S2, S3
Muscles innervated
Popliteus
Gastrocnemius
Soleus
Plantaris
Tibialis posterior
Flexor hallucis longus
Flexor digitorum brevis and longus
Terminates by dividing into the medial and lateral plantar nerves.
You review a 42-year-old woman 8 months following a renal transplant for focal segmental glomerulosclerosis. She is on a combination of tacrolimus, mycophenolate, and prednisolone. She has now presented with a five day history of feeling generally unwell with jaundice, fatigue and arthralgia. On examination she has jaundice, widespread lymphadenopathy and hepatomegaly. What is the most likely diagnosis?
Hepatitis C
Epstein-Barr virus
HIV
Hepatitis B
Cytomegalovirus
Post transplant complications
CMV: 4 weeks to 6 months post transplant
EBV: post transplant lymphoproliferative disease. > 6 months post transplant
Post transplant lymphoproliferative disorder is most commonly associated with Epstein-Barr virus. It typically occurs 6 months post transplant and is associated with high dose immunosupressant therapy. Remember cytomegalovirus presents within the first 4 weeks to 6 months post transplant.
Renal transplant:HLA typing and graft failure
The human leucocyte antigen (HLA) system is the name given to the major histocompatibility complex (MHC) in humans. It is coded for on chromosome 6.
Some basic points on the HLA system
Class 1 antigens include A, B and C. Class 2 antigens include DP,DQ and DR
When HLA matching for a renal transplant the relative importance of the HLA antigens are as follows DR > B > A
Graft survival
1 year = 90%, 10 years = 60% for cadaveric transplants
1 year = 95%, 10 years = 70% for living-donor transplants
Post-op problems
ATN of graft
Vascular thrombosis
Urine leakage
UTI
Hyperacute acute rejection
Due to antibodies against donor HLA type 1 antigens
Rarely seen due to HLA matching
Acute graft failure (< 6 months)
Usually due to mismatched HLA
Other causes include cytomegalovirus infection
Management: give steroids, if resistant use monoclonal antibodies
Causes of chronic graft failure (> 6 months)
Chronic allograft nephropathy
Ureteric obstruction
Recurrence of original renal disease (MCGN > IgA > FSGS)
Which of the following is not a content of the anterior triangle of the neck?
Ansa cervicalis
Vagus nerve
Anterior jugular vein
Transverse cervical nerve
Hypoglossal nerve
The transverse cervical nerve lies within the posterior triangle (the transverse cervical nerve divides into superior and inferior branches of the anterior margin of SCM). The anterior jugular vein is formed in the submental region and descends in the superficial fascia near the median plane. It passes inferior to enter the suprasternal space, it is linked to the contralateral anterior jugular vein by the jugular venous arch.
Anterior triangle of the neck
Boundaries
Anterior border of the Sternocleidomastoid
Lower border of mandible
Anterior midline
Sub triangles (divided by Digastric above and Omohyoid)
Muscular triangle: Neck strap muscles
Carotid triangle: Carotid sheath
Submandibular Triangle (digastric)
Contents of the anterior triangle
Digastric triangle Submandibular gland
Submandibular nodes
Facial vessels
Hypoglossal nerve
Muscular triangle Strap muscles
Jugular vein
Carotid triangle Carotid sheath (Common carotid, vagus and internal jugular vein)
Ansa cervicalis
Nerve supply to digastric muscle
Anterior: Mylohyoid nerve
Posterior: Facial nerve
A 55 year old man with carcinoma of the larynx is undergoing a difficult laryngectomy. The surgeons divide the thyrocervical trunk, from which of the following vessels does this structure most commonly originate?
Subclavian artery
Common carotid artery
Vertebral artery
External carotid artery
Internal carotid artery
The thyrocervical trunk is a branch of the subclavian artery. It arises from the first part between the subclavian artery and the inner border of scalenus anterior. It branches off the subclavian distal to the vertebral artery.
Root of the neck
Thoracic Outlet
Where the subclavian artery and vein and the brachial plexus exit the thorax and enter the arm.
They pass over the 1st rib and under the clavicle.
The subclavian vein is the most anterior structure and is immediately anterior to scalenus anterior and its attachment to the first rib.
Then subclavian artery passes between the anterior and middle scalene muscles.
At the level of the first rib, the lower cervical nerve roots combine to form the 3 trunks of the brachial plexus. The lowest trunk is formed by the union of C8 and T1, and this trunk lies directly posterior to the artery and is in contact with the superior surface of the first rib.
Thoracic outlet obstruction causes neurovascular compromise.
Which type of fracture is seen when a 22 year old drunk man is involved in a fight and injures his thumb when he punches his opponent?
Barton’s
Bennett’s
Galeazzi
Colles’
Smith’s
When a person, such as a 22-year-old man, injures his thumb by punching an opponent, the most likely type of fracture to occur is a Bennett’s fracture. This is a fracture at the base of the thumb metacarpal involving the carpometacarpal (CMC) joint, resulting in a clean break with a significant piece of bone broken off. This type of fracture is common in scenarios involving direct trauma to the thumb, such as punching.
Eponymous fractures
Colles’ fracture (dinner fork deformity)
Fall onto extended outstretched hand
Classical Colles’ fractures have the following 3 features:
- Transverse fracture of the radius
- 1 inch proximal to the radio-carpal joint
- Dorsal displacement and angulation
Smith’s fracture (reverse Colles’ fracture)
Volar angulation of distal radius fragment (Garden spade deformity)
Caused by falling backwards onto the palm of an outstretched hand or falling with wrists flexed
Bennett’s fracture
Intra-articular fracture of the first carpometacarpal joint
Impact on flexed metacarpal, caused by fist fights
X-ray: triangular fragment at ulnar base of metacarpal
Monteggia’s fracture
Dislocation of the proximal radioulnar joint in association with an ulna fracture
Fall on outstretched hand with forced pronation
Needs prompt diagnosis to avoid disability
Galeazzi fracture
Radial shaft fracture with associated dislocation of the distal radioulnar joint
Direct blow
Pott’s fracture
Bimalleolar ankle fracture
Forced foot eversion
Barton’s fracture
Distal radius fracture (Colles’/Smith’s) with associated radiocarpal dislocation
Fall onto extended and pronated wrist
Involvement of the joint is a defining feature
Holstein Lewis Fracture
A HolsteinLewis fracture is a fracture of the distal third of the humerus resulting in entrapment of the radial nerve.
The radial nerve is one of the major nerves of the upper limb. It innervates all of the muscles in the extensor compartments of the arm.
Conservative treatment includes reduction and use of a functional brace
Vascular injury may require open surgery
A 72 year old man is undergoing a left pneumonectomy for carcinoma of the bronchus. As the surgeons approach the root of the lung, which structure will lie most anteriorly (in the anatomical plane)?
Vagus nerve
Phrenic nerve
Bronchus
Pulmonary vein
Pulmonary artery
The phrenic nerve is the most anteriorly located structure in the lung root. The vagus nerve lies most posteriorly.
Lung anatomy
The right lung is composed of 3 lobes divided by the oblique and transverse fissures. The left lung has two lobes divided by the oblique fissure.The apex of both lungs is approximately 4cm superior to the sterno-costal joint of the first rib. Immediately below this is a sulcus created by the subclavian artery.
Peripheral contact points of the lung
Base: diaphragm
Costal surface: corresponds to the cavity of the chest
Mediastinal surface: Contacts the mediastinal pleura. Has the cardiac impression. Above and behind this concavity is a triangular depression named the hilum, where the structures which form the root of the lung enter and leave the viscus. These structures are invested by pleura, which, below the hilum and behind the pericardial impression, forms the pulmonary ligament
Right lung
Above the hilum is the azygos vein; Superior to this is the groove for the superior vena cava and right innominate vein; behind this, and nearer the apex, is a furrow for the innominate artery. Behind the hilum and the attachment of the pulmonary ligament is a vertical groove for the oesophagus; In front and to the right of the lower part of the oesophageal groove is a deep concavity for the extrapericardiac portion of the inferior vena cava.
The root of the right lung lies behind the superior vena cava and the right atrium, and below the azygos vein.
The right main bronchus is shorter, wider and more vertical than the left main bronchus and therefore the route taken by most foreign bodies.
Left lung
Above the hilum is the furrow produced by the aortic arch, and then superiorly the groove accommodating the left subclavian artery; Behind the hilum and pulmonary ligament is a vertical groove produced by the descending aorta, and in front of this, near the base of the lung, is the lower part of the oesophagus.
The root of the left lung passes under the aortic arch and in front of the descending aorta.
Inferior borders of both lungs
6th rib in mid clavicular line
8th rib in mid axillary line
10th rib posteriorly
The pleura runs two ribs lower than the corresponding lung level.
Bronchopulmonary segments
Segment number Right lung Left lung
1 Apical Apical
2 Posterior Posterior
3 Anterior Anterior
4 Lateral Superior lingular
5 Medial Inferior lingular
6 Superior (apical) Superior (apical)
7 Medial basal Medial basal
8 Anterior basal Anterior basal
9 Lateral basal Lateral basal
10 Posterior basal Posterior basal
Which nerve lies medially to the lobes of the thyroid gland, in the groove between the oesophagus and trachea?
Vagus nerve
External laryngeal nerve
Recurrent laryngeal nerve
Ansa cervicalis
Phrenic nerve
The recurrent laryngeal nerve may be injured at this site during ligation of the inferior thyroid artery.
Thyroid gland
Right and left lobes connected by isthmus
Surrounded by sheath from pretracheal layer of deep fascia
Apex: Lamina of thyroid cartilage
Base: 4th-5th tracheal ring
Pyramidal lobe: from isthmus
May be attached to foramen caecum at the base of the tongue
Relations
Anteromedially
Sternothyroid
Superior belly of omohyoid
Sternohyoid
Anterior aspect of sternocleidomastoid
Posterolaterally Carotid sheath
Medially
Larynx
Trachea
Pharynx
Oesophagus
Cricothyroid muscle
External laryngeal nerve (near superior thyroid artery)
Recurrent laryngeal nerve (near inferior thyroid artery)
Posterior
Parathyroid glands
Anastomosis of superior and inferior thyroid arteries
Isthmus
Anteriorly: Sternothyroids, sternohyoids, anterior jugular veins
Posteriorly: 2nd, 3rd, 4th tracheal rings (attached via Ligament of Berry)
Blood Supply
Arterial
Superior thyroid artery (1st branch of external carotid)
Inferior thyroid artery (from thyrocervical trunk)
Thyroidea ima (in 10% of population -from brachiocephalic artery or aorta)
Venous
Superior and middle thyroid veins - into the IJV
Inferior thyroid vein - into the brachiocephalic veins
A 43 year old lady is receiving chemotherapy for the treatment of metastatic breast cancer. You are called because it has become apparent that her doxorubicin infusion has extravasated. What is the most appropriate course of action?
Stop the infusion and administer dexamethasone through the infusion device
Stop the infusion and administer hyaluronidase through the infusion device
Stop the infusion and apply a cold compress to the site
Stop the infusion and apply a warm compress to the site
Stop the infusion and administer sodium bicarbonate through the infusion device
The application of cold compresses is indicated in doxorubicin extravasation. Warm compresses increase the risk of doxorubicin ulceration. Hyaluronidase is indicated in the extravasation of contrast media, TPN and vinca alkaloids. However, if administered following doxorubicin extravasation it will dramatically worsen the situation and is contra indicated.
Up to 50% of those sustaining severe injuries will require delayed surgical reconstruction.
Extravasation injury
Chemotherapy may be complicated by extravasation reactions in up to 6% of cases. The following chemotherapy agents are recognised causes of extravasation reactions; doxorubicin, vincristine, vinblastine, cisplatin, mitomycin and mithramycin.
Up to 30% of extravasation reactions may be complicated by the development of ulceration.
When an extravasation reaction is suspected, the infusion should be stopped and the infusing device aspirated. The extremity should be elevated. As a general rule cold compresses have been shown to reduce the incidence of subsequent ulceration with doxorubicin. Warm compresses have been found to be beneficial in extravasation of vinca alkaloids. Dimethylsulfoxide may be infused in some cases, ideally within 5 hours of the event occurring. No conclusive evidence exists to support the use of corticosteroids or sodium bicarbonate for extravasation injuries.
Extravasation of total parenteral nutrition solutions is usually managed by the local administration of hyaluronidase to the infusion site.
A 68 year old man presents with an ulcerated lesion on his right cheek. It is excised and on histological assessment a squamous cell carcinoma is diagnosed. It measures 25mm in diameter and is 4mm deep. Which of the following statements relating to this condition is false?
In this particular case margins of at least 6mm are required
Use of cryosurgery to treat this patients lesion would have been unsafe
Use of radiotherapy to treat this lesion would have been unsafe
This patients local recurrence rate may approach 15%
The disease usually spreads via lymphatics
Analysis of Statements Regarding Squamous Cell Carcinoma
To determine which statement is false regarding the squamous cell carcinoma (SCC) diagnosed in the 68-year-old man, we will analyze each statement based on current medical knowledge and guidelines.
In this particular case margins of at least 6mm are required:
For SCC, especially those that are larger or have certain risk factors, surgical excision with clear margins is critical to minimize the risk of recurrence. The recommended margin for low-risk SCCs can be around 4-5mm, but for high-risk lesions (which may include those that are larger than 2cm or deeper than 4mm), a margin of at least 6mm is often recommended. Given that this lesion measures 25mm in diameter and is 4mm deep, it would be prudent to require a margin of at least 6mm. Therefore, this statement is true.
Use of cryosurgery to treat this patient’s lesion would have been unsafe:
Cryosurgery can be used for superficial skin cancers; however, it is generally not recommended for invasive SCCs due to the potential for incomplete treatment and local recurrence. Given the size and depth of this lesion (25mm and 4mm respectively), using cryosurgery would indeed be considered unsafe as it may not adequately address the invasive nature of the cancer. Thus, this statement is true.
Use of radiotherapy to treat this lesion would have been unsafe:
Radiotherapy can be an effective treatment modality for SCC, particularly in cases where surgery is not feasible or when there are concerns about surgical margins. It can also be used post-operatively if there are positive margins or other high-risk features present. Therefore, stating that radiotherapy would be unsafe in treating this lesion does not align with current practices; thus, this statement is false.
This patient’s local recurrence rate may approach 15%:
Local recurrence rates for SCC can vary widely based on several factors including tumor size, depth, location, and adequacy of surgical margins. For high-risk lesions such as large or deeply invasive SCCs, local recurrence rates can indeed approach or exceed 15%. This aligns with clinical data suggesting higher recurrence rates in such cases; therefore, this statement is true.
The disease usually spreads via lymphatics:
SCC typically spreads through lymphatic channels rather than hematogenous routes (bloodstream). This characteristic behavior is well-documented in medical literature regarding squamous cell carcinoma; hence, this statement is also true.
Based on the analysis above:
The false statement among those provided regarding the management and characteristics of squamous cell carcinoma in this patient scenario is:
Use of radiotherapy to treat this lesion would have been unsafe
Poor prognostic factors in SCC:
Size >20mm (local recurrence rate of up to 15%)
Depth greater than 4mm (risk of metastasis up to 30%)
This man has an SCC with significant risk of metastasis. Although cryotherapy may be used to treat SCC it would be most unsafe in this setting as the lesion extends deeply. However, radiotherapy is a safe treatment modality for SCC and may be used in selected cases. It is unwise to use radiotherapy in areas prone to radionecrosis e.g. the nose.
Squamous cell carcinoma of the skin
Second most common skin malignancy
Derived from epidermal keratinocytes
Commonest in fair skinned individuals in sun exposed sites
May occur in perianal and genital skin especially in association with Human Papilloma Virus 16 and 18 infections.
Groups at high risk
Renal transplant and on immunosuppression
Individuals with HIV
Those who have received psoralen UVA therapy
Chronic wounds (Marjolins ulcer)
Xeroderma pigmentosum
Oculocutaneous albinism
Prognosis
Good Prognosis Poor prognosis
Well differentiated tumours Poorly differentiated tumours
<20mm diameter >20mm in diameter
<2mm deep >4mm deep
No associated diseases Immunosupression for whatever reason
Treatment
Surgical excision with 4mm margins if lesion <20mm in diameter. If tumour >20mm then margins should be 6mm.
A 20 year old man is admitted to the intensive care unit with an isolated severe head injury. A CT scan shows multiple intracerebral bleeds but no midline shift. He is intubated and ventilated. His pupils are dilated and react sluggishly to light. His heart rate is 50 beats/minute blood pressure 170/110 mmHg and his respiratory rate is set at 10 breaths/minute. The rising blood pressure is likely to be caused by:
Aortic and carotid baroreceptor stimulation
Cortisol stimulation
Renin angiotensin stimulation
Sympathetic stimulation related to blood loss
Sympathetic stimulation related to increased intra cranial pressure
Clinical Presentation and Context
In this scenario, we have a 20-year-old man with a severe head injury, as evidenced by the CT scan showing multiple intracerebral bleeds. The clinical signs include dilated pupils that react sluggishly to light, bradycardia (heart rate of 50 beats/minute), hypertension (blood pressure of 170/110 mmHg), and a controlled respiratory rate set at 10 breaths/minute due to intubation and ventilation.
Understanding the Physiological Responses
Intracranial Pressure (ICP): Severe head injuries often lead to increased intracranial pressure due to swelling or bleeding within the cranial cavity. This increase in ICP can stimulate various physiological responses.
Sympathetic Nervous System Activation: In response to elevated ICP, the body may activate the sympathetic nervous system as a compensatory mechanism. This activation can lead to increased systemic vascular resistance and subsequent hypertension.
Cushing’s Reflex: The combination of bradycardia and hypertension is indicative of Cushing’s reflex, which is a physiological response to increased ICP. This reflex involves:
Increased blood pressure due to sympathetic stimulation.
Bradycardia as a result of baroreceptor reflexes responding to high blood pressure.
Baroreceptor Response: While baroreceptors in the carotid arteries and aorta respond to changes in blood pressure, in this case, the primary driver for the observed hypertension is likely related to increased intracranial pressure rather than direct stimulation from baroreceptors alone.
Other Factors Considered:
Cortisol Stimulation: While cortisol can influence blood pressure regulation, it is not the primary factor in acute head trauma scenarios.
Renin-Angiotensin Stimulation: This system typically responds to low blood volume or low blood pressure situations rather than isolated increases in ICP.
Sympathetic Stimulation Related to Blood Loss: There is no indication of significant blood loss in this scenario; thus, this option is less relevant.
Given these considerations, the most plausible explanation for the rising blood pressure in this patient is:
Sympathetic stimulation related to increased intracranial pressure
This conclusion aligns with established medical knowledge regarding responses following severe head injuries and their effects on cardiovascular dynamics.
The changes seen are the result of raised ICP, its rare for head injuries to cause sufficient blood loss to affect the circulating volume.
Cushing reflex
When intra cranial pressure exceeds mean arterial pressure, compression of cerebral arterioles occurs. This results in cerebral ischaemia.
Increases in ICP results in stepwise activation of the sympathetic nervous system initially. This raises peripheral vascular resistance and results in hypertension. Cardiac output is also increased. These haemodynamic changes are detected by aortic arch baroreceptors and this results in activation of the parasympathetic nervous system. These parasympathetic effects comprise the second stage of the Cushing reflex. The Cushing reflex is a serious development that indicates imminent coning or other terminal events if not resolved quickly.
An 83 year old lady attends the endoscopy department for a flexible sigmoidoscopy. The endoscopist administers 7mg of intravenous midazolam for sedation. Approximately 3 minutes later the patient is noted to be hypoxic and has a respiratory rate of 5 breaths per minute. What is the most useful agent to administer at this point?
Intravenous naloxone
Oral naloxone
Intravenous flumazenil
Intravenous doxapram
Nebulised adrenaline
Benzodiazepine overdose with marked suppression of respiratory activity should be managed with IV flumazenil.
Midazolam
Midazolam is a short acting benzodiazepine. It is usually administered intravenously, when administered orally, dosing is less reliable. It is eliminated via the cytochrome P450 pathway in the liver. It is often used for procedural related sedation purposes. It is generally administered in doses of 2-5mg for this purpose. It is often given in combination with a short acting opiate such as fentanyl for analgesic purposes. One of the main reasons that midazolam is preferred for procedural related sedation is due to a degree of amnesia that can accompany it. If administered in excess, most patients can be managed with simple airway support measures an monitoring. However, if there is significant respiratory compromise, it can be reversed with the agent flumazenil which is a selective GABA receptor antagonist.
A 73 year old lady presents with large bowel obstruction. On examination, she has a rectal cancer 6cm from the anal verge which has occluded the colonic lumen. An abdominal x-ray shows a caecal diameter of 7cm. Which of the management strategies outlined below is the most appropriate?
Construction of a loop ileostomy
Construction of a loop colostomy
Construction of a venting caecostomy
Abdomino-perineal resection of the colon and rectum
Low anterior resection and loop ileostomy
Bowel obstruction due to RECTAL cancer should be treated by loop colostomy.
Bowel obstruction due to obstructing left sided COLON cancer is usually treated by resection of the primary lesion and formation of colostomy.
This patient should be defunctioned, definitive surgery should wait until staging is completed. A loop ileostomy will not satisfactorily decompress an acutely obstructed colon. Low rectal cancers that are obstructed should not usually be primarily resected. The obstructed colon that would be used for anastomosis would carry a high risk of anastomotic dehiscence. In addition, as this is an emergency presentation, staging may not be completed, an attempted resection may therefore compromise the circumferential resection margin, with an associated risk of local recurrence.
Colorectal cancer treatment
Patients diagnosed as having colorectal cancer should be completely staged using CT of the chest/ abdomen and pelvis. Their entire colon should have been evaluated with colonoscopy or CT colonography. Patients whose tumours lie below the peritoneal reflection should have their mesorectum evaluated with MRI.
Once their staging is complete patients should be discussed within a dedicated colorectal MDT meeting and a treatment plan formulated.
Treatment of colonic cancer
Cancer of the colon is nearly always treated with surgery. Stents, surgical bypass and diversion stomas may all be used as palliative adjuncts. Resectional surgery is the only option for cure in patients with colon cancer. The procedure is tailored to the patient and the tumour location. The lymphatic drainage of the colon follows the arterial supply and therefore most resections are tailored around the resection of particular lymphatic chains (e.g. ileo-colic pedicle for right sided tumours). Some patients may have confounding factors that will govern the choice of procedure, for example a tumour in a patient from a HNPCC family may be better served with a panproctocolectomy rather than segmental resection. Following resection the decision has to be made regarding restoration of continuity. For an anastomosis to heal the key technical factors include; adequate blood supply, mucosal apposition and no tissue tension. Surrounding sepsis, unstable patients and inexperienced surgeons may compromise these key principles and in such circumstances it may be safer to construct an end stoma rather than attempting an anastomosis.
When a colonic cancer presents with an obstructing lesion; the options are to either stent it or resect. In modern practice it is unusual to simply defunction a colonic tumour with a proximal loop stoma. This differs from the situation in the rectum (see below).
Following resection patients with risk factors for disease recurrence are usually offered chemotherapy, a combination of 5FU and oxaliplatin is common.
Rectal cancer
The management of rectal cancer is slightly different to that of colonic cancer. This reflects the rectum’s anatomical location and the challenges posed as a result. Tumours located in the rectum can be surgically resected with either an anterior resection or an abdomino - perineal resection. The technical aspects governing the choice between these two procedures can be complex to appreciate and the main point to appreciate for the MRCS is that involvement of the sphincter complex or very low tumours require APER. In the rectum a 2cm distal clearance margin is required and this may also impact on the procedure chosen. In addition to excision of the rectal tube an integral part of the procedure is a meticulous dissection of the mesorectal fat and lymph nodes (total mesorectal excision/ TME). In rectal cancer surgery invovlement of the cirumferential resection margin carries a high risk of disease recurrence. Because the rectum is an extraperitoneal structure (until you remove it that is!) it is possible to irradiate it, something which cannot be offered for colonic tumours. This has a major impact in rectal cancer treatment and many patients will be offered neoadjuvent radiotherapy (both long and short course) prior to resectional surgery. Patients with T1, 2 and 3 /N0 disease on imaging do not require irradiation and should proceed straight to surgery. Patients with T4 disease will typically have long course chemo radiotherapy. Patients presenting with large bowel obstruction from rectal cancer should not undergo resectional surgery without staging as primary treatment (very different from colonic cancer). This is because rectal surgery is more technically demanding, the anastomotic leak rate is higher and the danger of a positive resection margin in an unstaged patient is high. Therefore patients with obstructing rectal cancer should have a defunctioning loop colostomy.
Summary of procedures
The operations for cancer are segmental resections based on blood supply and lymphatic drainage. These commonly performed procedures are core knowledge for the MRCS and should be understood.
Site of cancer Type of resection Anastomosis Risk of leak
Right colon Right hemicolectomy Ileo-colic Low <5%
Transverse Extended right hemicolectomy Ileo-colic Low <5%
Splenic flexure Extended right hemicolectomy Ileo-colic Low <5%
Splenic flexure Left hemicolectomy Colo-colon 2-5%
Left colon Left hemicolectomy Colo-colon 2-5%
Sigmoid colon High anterior resection Colo-rectal 5%
Upper rectum Anterior resection (TME) Colo-rectal 5%
Low rectum Anterior resection (Low TME) Colo-rectal
(+/- Defunctioning stoma) 10%
Anal verge Abdomino-perineal excision of colon and rectum None n/a
In the emergency setting, where the bowel has perforated, the risk of an anastomotic breakdown is much greater, particularly when the anastomosis is colon-colon. In this situation, an end colostomy is often safer and can be reversed later. When resection of the sigmoid colon is performed and an end colostomy is fashioned the operation is referred to as a Hartmans procedure. Whilst left sided resections are more risky, ileo-colic anastomoses are relatively safe even in the emergency setting and do not need to be defunctioned.
References
A review of the diagnosis and management of colorectal cancer and a summary of the UK National Institute of Clinical Excellence guidelines is provided in:
Poston G, et al . Diagnosis and management of colorectal cancer: summary of NICE guidance. BMJ 2011: 343: d 6751.
A 27 year old lady presents with a breast lump. She has previously undergone a breast augmentation with an implant. Which of these imaging techniques is the most appropriate next step?
Ultrasound
CT scanning
MRI
PET CT
Mammography
Unless there are concerns about implant rupture, the imaging of a breast lump in a young patient with implants would be USS initially. If this is not conclusive then MRI should be performed. Remember that MRI is the modality of choice where an implant related complication is suspected or there are concerns about malignant or pre malignant changes and none of these apply here.
Breast imaging
Breast imaging is undertaken with a combination of USS and mammography in most women presenting with a palpable lump. In younger patients, the denser breasts encountered in this group may make mammography less informative.
In women who present with breast implants with a palpable lump, the imaging of choice remains the same. However, specialized imaging techniques may be needed to obtain optimal mammographic views. Where there are specific concerns about a breast implant, rather than a lump, the imaging modality of choice is MRI scanning.
MRI scanning may be beneficial in screening younger patients with a family history and also in patients with lobular cancers who are being considered for breast conserving surgery.
The pathogenicity of the tubercle bacillus is due to which of the following?
Necrosis caused by expanding granulomas
Ability to multiply within fibroblasts
Delayed hypersensitivity reaction against bacteria
Effect of antibody response
Direct toxic effect on host cells
Answer: Delayed hypersensitivity reaction against bacteria
Pathogenicity of the Tubercle Bacillus
The pathogenicity of the tubercle bacillus, scientifically known as Mycobacterium tuberculosis, is a complex process that involves several mechanisms. To determine which option best describes the primary cause of its pathogenicity, we will analyze each provided choice in detail.
Necrosis caused by expanding granulomas:
Granulomas are organized structures formed in response to chronic inflammation, typically seen in infections like tuberculosis. They consist of macrophages, lymphocytes, and other immune cells that attempt to contain the infection. While necrosis can occur within granulomas due to cell death and tissue damage, it is not the primary mechanism through which M. tuberculosis exerts its pathogenic effects.
Ability to multiply within fibroblasts:
Fibroblasts are connective tissue cells that play a role in wound healing and tissue repair. However, M. tuberculosis primarily resides and replicates within macrophages rather than fibroblasts. The ability to survive and multiply inside macrophages is crucial for its pathogenicity but does not specifically highlight fibroblasts as a significant factor.
Delayed hypersensitivity reaction against bacteria:
The immune response to M. tuberculosis includes delayed-type hypersensitivity (DTH), which is mediated by T cells and leads to the formation of granulomas. This immune response is an important aspect of how the body attempts to control the infection; however, it is more a consequence of infection rather than a direct cause of pathogenicity.
Effect of antibody response:
Antibodies play a role in the immune response against many pathogens; however, M. tuberculosis has evolved mechanisms to evade antibody-mediated responses effectively. The bacterium’s cell wall contains mycolic acids that protect it from being effectively targeted by antibodies.
Direct toxic effect on host cells:
While M. tuberculosis does not produce traditional toxins like some other bacteria, it can induce cytotoxic effects indirectly through immune-mediated mechanisms and inflammation caused by its presence in host tissues.
After analyzing these options, it becomes clear that while all these factors contribute to the overall disease process associated with tuberculosis, none directly encapsulate the primary mechanism behind the pathogenicity of M. tuberculosis. However, if we consider how this organism survives and thrives within host tissues despite an immune response aimed at eliminating it, we find that:
The most relevant factor among those listed would be related to its ability to evade destruction within macrophages (not explicitly mentioned but implied) leading to chronic infection and delayed hypersensitivity reactions as part of the immune response.
Thus, considering all aspects discussed:
The pathogenicity of the tubercle bacillus is primarily due to its ability to multiply within host cells (macrophages) and elicit a delayed hypersensitivity reaction against itself, but since “ability to multiply within fibroblasts” was one option presented:
Mycobacteria stimulate a specific T cell response of cell mediated immunity. This is effective in reducing the infection, the delayed hypersensitivity also damages tissues. Necrosis occurs in TB but is usually within the granuloma.
Tuberculosis pathology
Is a form of primary chronic inflammation, caused by the inability of macrophages to kill the Mycobacterium tuberculosis.
The macrophages often migrate to regional lymph nodes, the lung lesion plus affected lymph nodes is referred to as a Ghon complex.
This leads to the formation of a granuloma which is a collection of epithelioid histiocytes.
There is the presence of caseous necrosis in the centre.
The inflammatory response is mediated by a type 4 hypersensitivity reaction.
In healthy individuals the disease may be contained, in the immunocompromised disseminated (miliary TB) may occur.
Diagnosis
Waxy membrane of mycobacteria prevents binding with normal stains. Ziehl - Neelsen staining is typically used.
Culture based methods take far longer.
Which of the following is a recognised feature of ketamine when used as an anaesthetic agent?
Malignant hyperpyrexia
Adrenal suppression
Myocardial depression
Dissociative anaesthesia
Marked respiratory depression
Unlike most anaesthetic agents ketamine does not cause myocardial or marked respiratory depression. It is not associated with the adrenal suppression that may occur with etomidate. It is however, associated with a state of dissociative anaesthesia which patients may find distressing.
Anaesthetic agents
The table below summarises some of the more commonly used IV induction agents
Agent Specific features
Propofol
Rapid onset of anaesthesia
Pain on IV injection
Rapidly metabolised with little accumulation of metabolites
Proven anti emetic properties
Moderate myocardial depression
Widely used especially for maintaining sedation on ITU, total IV anaesthesia and for daycase surgery
Sodium thiopentone
Extremely rapid onset of action making it the agent of choice for rapid sequence of induction
Marked myocardial depression may occur
Metabolites build up quickly
Unsuitable for maintenance infusion
Little analgesic effects
Ketamine
May be used for induction of anaesthesia
Has moderate to strong analgesic properties
Produces little myocardial depression making it a suitable agent for anaesthesia in those who are haemodynamically unstable
May induce state of dissociative anaesthesia resulting in nightmares
Etomidate
Has favorable cardiac safety profile with very little haemodynamic instability
No analgesic properties
Unsuitable for maintaining sedation as prolonged (and even brief) use may result in adrenal suppression
Post operative vomiting is common
During an Ivor Lewis Oesophagectomy for carcinoma of the lower third of the oesophagus which structure is divided to allow mobilisation of the oesophagus?
Vagus nerve
Azygos vein
Right inferior lobar bronchus
Phrenic nerve
Pericardiophrenic artery
The azygos vein is routinely divided during an oesophagectomy to allow mobilisation. It arches anteriorly to insert into the SVC on the right hand side.
Treatment of oesophageal cancer
- In general resections are not offered to those patients with distant metastasis, and usually not to those with N2 disease.
Local nodal involvement is not in itself a contra indication to resection.
Surgical resection is the mainstay of treatment.
Neoadjuvent chemotherapy is given in most cases prior to surgery.
In situ disease may be managed by endoscopic mucosal resection, although this is still debated.
In patients with lower third lesions an Ivor - Lewis type procedure is most commonly performed. Very distal tumours may be suitable to a transhiatal procedure. Which is an attractive option as the penetration of two visceral cavities required for an Ivor- Lewis type procedure increases the morbidity considerably.
More proximal lesions will require a total oesphagectomy (Mckeown type) with anastomosis to the cervical oesophagus.
Patients with unresectable disease may derive benefit from local ablative procedures, palliative chemotherapy or stent insertion.
Operative details of Ivor- Lewis procedure
Combined laparotomy and right thoracotomy
Indication
Lower and middle third oesophageal tumours
Preparation
Staging with a combination of CT chest abdomen and pelvis- if no metastatic disease detected then patients will undergo a staging laparoscopy to detect peritoneal disease.
If both these modalities are negative then patients will finally undergo a PET CT scan to detect occult metastatic disease. Only in those whom no evidence of advanced disease is detected will proceed to resection.
Patients receive a GA, double lumen endotracheal tube to allow for lung deflation, CVP and arterial monitoring.
Procedure
A rooftop incision is made to access the stomach and duodenum.
Laparotomy To mobilize the stomach
The greater omentum is incised away from its attachment to the right gastroepiploic vessels along the greater curvature of the stomach.
Then the short gastric vessels are ligated and detached from the greater curvature from the spleen.
The lesser omentum is incised, preserving the right gastric artery.
The retroperitoneal attachments of the duodenum in its second and third portions are incised, allowing the pylorus to reach the oesophageal hiatus. Some surgeons perform a pyloroplasty at this point to facilitate gastric emptying.
The left gastric vessels are then ligated, avoiding any injury to the common hepatic or splenic arteries. Care must be taken to avoid inadvertently devascularising the liver owing to variations in anatomy.
Right Thoracotomy Oesophageal resection and oesophagogastric anastomosis
Through 5th intercostal space
Dissection performed 10cm above the tumour
This may involve transection of the azygos vein.
The oesophagus is then removed with the stomach creating a gastric tube.
An anastomosis is created.
The chest is closed with underwater seal drainage and tube drains to the abdominal cavity.
Post operatively
Patients will typically recover in ITU initially.
A nasogastric tube will have been inserted intraoperatively and must remain in place during the early phases of recovery.
Post operatively these patients are at relatively high risk of developing complications:
- Atelectasis- due to the effects of thoracotomy and lung collapse
- Anastomotic leakage. The risk is relatively high owing to the presence of a relatively devascularised stomach. Often the only blood supply is from the gastroepiploic artery as all others will have been divided. If a leak does occur then many will attempt to manage conservatively with prolonged nasogastric tube drainage and TPN. The reality is that up to 50% of patients developing an anastomotic leak will not survive to discharge.
- Delayed gastric emptying (may be avoided by performing a pyloroplasty).
A 28 year old man has a long history of recurrent chest infections. On examination, he is noted to have no palpable vas deferens. However, both testes are located within the scrotum. What is the most likely underlying disease association?
Kleinfelters syndrome
Kallmann syndrome
Cystic fibrosis
Coeliac disease
Gardners syndrome
99% of males with cystic fibrosis will have absent vas.
Underlying Disease Association in a 28-Year-Old Man with Recurrent Chest Infections and No Palpable Vas Deferens
To determine the most likely underlying disease association for the clinical scenario presented, we need to analyze the symptoms and their implications step by step.
Clinical Features
Recurrent Chest Infections: This symptom suggests an underlying issue with respiratory function or immune response. In particular, recurrent infections can be associated with conditions that affect mucous production or ciliary function.
No Palpable Vas Deferens: The absence of the vas deferens is a significant finding. This condition is known as congenital bilateral absence of the vas deferens (CBAVD), which is commonly associated with certain genetic disorders.
Testes Located Within the Scrotum: The presence of both testes in the scrotum indicates that there is no issue with testicular descent, which rules out some conditions like cryptorchidism.
Differential Diagnosis
Now let’s evaluate each of the potential associations provided:
Klinefelter Syndrome: This genetic condition (47,XXY) typically presents with hypogonadism, gynecomastia, and infertility but does not specifically correlate with recurrent chest infections or absent vas deferens.
Kallmann Syndrome: This condition involves hypogonadotropic hypogonadism and anosmia (loss of smell). While it can lead to infertility due to low testosterone levels, it does not typically present with recurrent chest infections or absent vas deferens.
Cystic Fibrosis: Cystic fibrosis is a genetic disorder caused by mutations in the CFTR gene, leading to thickened secretions in various organs including the lungs and pancreas. Patients often experience recurrent respiratory infections due to obstructed airways and may also have CBAVD due to abnormal development of the reproductive tract.
Coeliac Disease: This autoimmune disorder primarily affects the gastrointestinal system and does not typically present with absent vas deferens or recurrent chest infections as primary symptoms.
Gardner’s Syndrome: This is a genetic disorder characterized by multiple polyps in the colon along with other tumors and skin lesions but does not correlate directly with respiratory issues or absent vas deferens.
Conclusion
Given that this patient has recurrent chest infections along with congenital bilateral absence of the vas deferens, cystic fibrosis emerges as the most likely underlying disease association due to its well-documented connection between respiratory complications and reproductive tract anomalies such as CBAVD.
Therefore, based on this analysis:
Cystic fibrosis
Absence of the vas deferens
Absence of the vas may be unilateral or bilateral
Cystic fibrosis CFTR gene mutations are the cause in 40% of cases
Some non CF cases are due to unilateral renal agenesis
Sperm harvesting may allow for assisted conception
An infant is admitted with symptoms and signs of respiratory infection and is found to have several posterior rib fractures on chest radiograph. He was born prematurely at 37 weeks’ gestation and was observed overnight on the special care baby unit for tachypnoea which settled by the following day. On assessment, it is also apparent that his head circumference has increased at an excessive rate and has crossed 3 centiles since birth. What is the most likely underlying issue?
Accidental fracture
Pagets disease
Myeloproliferative disorder
Non accidental injury
Osteomalacia
Assessment of the Clinical Scenario
In this clinical scenario, we have an infant presenting with respiratory infection symptoms and posterior rib fractures identified on a chest radiograph. The infant was born prematurely at 37 weeks’ gestation and had a transient episode of tachypnoea that resolved quickly. Additionally, there is a notable increase in head circumference that has crossed three centiles since birth.
Step 1: Analyzing the Symptoms
Respiratory Infection: This can be common in infants, especially those born prematurely due to underdeveloped lungs.
Posterior Rib Fractures: The presence of rib fractures in an infant raises concern for potential underlying issues. In infants, rib fractures can occur due to various reasons, including accidental trauma or non-accidental injury (abuse).
Increased Head Circumference: A rapid increase in head circumference could indicate conditions such as hydrocephalus or other intracranial pathology.
Step 2: Evaluating Possible Causes
Accidental Fracture: While possible, the specific location of the fractures (posterior ribs) and the context suggest a higher likelihood of non-accidental injury.
Paget’s Disease: This condition primarily affects older adults and is characterized by abnormal bone remodeling. It is not relevant in this case involving an infant.
Myeloproliferative Disorder: These disorders involve excessive production of blood cells but do not typically present with rib fractures or increased head circumference as primary symptoms.
Non-Accidental Injury: Given the combination of posterior rib fractures and rapid increase in head circumference, non-accidental injury (child abuse) becomes a significant consideration. Rib fractures are often associated with physical abuse in infants, particularly when they are located posteriorly.
Osteomalacia: This condition involves softening of bones due to vitamin D deficiency but would not typically present with isolated rib fractures or increased head circumference without other systemic signs.
Step 3: Conclusion
Considering all these factors, the most likely underlying issue for this infant is non-accidental injury. The combination of posterior rib fractures and excessive increase in head circumference strongly suggests that there may be underlying trauma or abuse rather than benign causes.
Final Answer: Non accidental injury
Posterior rib fractures are extremely unusual in neonates. The change in head size may be accounted for by hydrocephalus which may occur as a sequelae from head injury.
Paediatric fractures
Paediatric fracture types
Type Injury pattern
Complete fracture Both sides of cortex are breached
Toddlers fracture Oblique tibial fracture in infants
Plastic deformity Stress on bone resulting in deformity without cortical disruption
Greenstick fracture Unilateral cortical breach only
Buckle fracture Incomplete cortical disruption resulting in periosteal haematoma only
Growth plate fractures
In paediatric practice fractures may also involve the growth plate and these injuries are classified according to the Salter- Harris system (given below):
Type Injury pattern
I Fracture through the physis only (x-ray often normal)
II Fracture through the physis and metaphysis
III Fracture through the physis and epiphyisis to include the joint
IV Fracture involving the physis, metaphysis and epiphysis
V Crush injury involving the physis (x-ray may resemble type I, and appear normal)
As a general rule it is safer to assume that growth plate tenderness is indicative of an underlying fracture even if the x-ray appears normal. Injuries of Types III, IV and V will usually require surgery. Type V injuries are often associated with disruption to growth.
Non accidental injury
Delayed presentation
Delay in attaining milestones
Lack of concordance between proposed and actual mechanism of injury
Multiple injuries
Injuries at sites not commonly exposed to trauma
Children on the at risk register
Pathological fractures
Genetic conditions, such as osteogenesis imperfecta, may cause pathological fractures.
Osteogenesis imperfecta
Defective osteoid formation due to congenital inability to produce adequate intercellular substances like osteoid, collagen and dentine.
Failure of maturation of collagen in all the connective tissues.
Radiology may show translucent bones, multiple fractures, particularly of the long bones, wormian bones (irregular patches of ossification) and a trefoil pelvis.
Subtypes
Type I The collagen is normal quality but insufficient quantity.
Type II- Poor collagen quantity and quality.
Type III- Collagen poorly formed. Normal quantity.
Type IV- Sufficient collagen quantity but poor quality.
Osteopetrosis
Bones become harder and more dense.
Autosomal recessive condition.
It is commonest in young adults.
Radiology reveals a lack of differentiation between the cortex and the medulla described as marble bone.
Secretions from which of the following will contain the highest levels of potassium?
Rectum
Ileum
Gallbladder
Pancreas
Stomach
The rectum has the potential to generate secretions rich in potassium. This is the rationale behind administration of resins for hyperkalaemia and the development of hypokalaemia in patients with villous adenoma of the rectum.
Potassium secretion -GI tract
Potassium secretions
Salivary glands Variable may be up to 60mmol/L
Stomach 10 mmol/L
Bile 5 mmol/L
Pancreas 4-5 mmol/L
Small bowel 10 mmol/L
Rectum 30 mmol/L
The above table provides average figures only and the exact composition varies depending upon the existence of disease, serum aldosterone levels and serum pH.
A key point to remember for the exam is that gastric potassium secretions are low. Hypokalaemia may occur in vomiting, usually as a result of renal wasting of potassium, not because of potassium loss in vomit.
A 28 year old rugby player injures his right humerus and on examination is noted to have a minor sensory deficit overlying the point of deltoid insertion into the humerus. Which of the nerves listed below is most likely to have been affected?
Radial
Axillary
Musculocutaneous
Median
Subscapular
This patch of skin is supplied by the axillary nerve
Axillary nerve
Terminal branch of the posterior cord of the brachial plexus
Root values C5 and C6
Descends posterior to the axillary artery at the lower border of subscapularis and then passes through quadrangular space with the posterior circumflex humeral vessels
Divides into anterior and posterior branches
Innervates deltoid muscle and small patch of skin over deltoid
A 53 year old man with a chronically infected right kidney is due to undergo a nephrectomy. Which of the following structures would be encountered first during a posterior approach to the hilum of the right kidney?
Right renal artery
Ureter
Right renal vein
Inferior vena cava
Right testicular vein
The ureter is the most posterior structure at the hilum of the right kidney and would therefore be encountered first during a posterior approach.
Renal arteries
The right renal artery is longer than the left renal artery
The renal vein/artery/pelvis enter the kidney at the hilum
Relations
Right Anterior- IVC, right renal vein, the head of the pancreas, and the descending part of the duodenum
Left Anterior- left renal vein, the tail of the pancreas
Branches
The renal arteries are direct branches off the aorta (upper border of L2- right side and L1 - left side)
In 30% there may be accessory arteries (mainly left side). Instead of entering the kidney at the hilum, they usually pierce the upper or lower part of the organ.
Before reaching the hilum of the kidney, each artery divides into four or five segmental branches (renal vein anterior and ureter posterior); which then divide within the sinus into lobar arteries supplying each pyramid and cortex.
Each vessel gives off some small inferior suprarenal branches to the suprarenal gland, the ureter, and the surrounding cellular tissue and muscles.
Which virus is associated with Kaposi’s sarcoma?
Human herpes virus 8
Human papillomavirus 16
Human T-lymphotropic virus 1
Epstein-Barr virus
Human papillomavirus 18
Virus Associated with Kaposi’s Sarcoma
Kaposi’s sarcoma (KS) is a type of cancer that primarily affects the skin, but can also involve other organs. It is characterized by the development of tumors that appear as purple or brown lesions on the skin and can be associated with significant morbidity. Understanding the viral etiology of Kaposi’s sarcoma is crucial for both diagnosis and treatment.
Human Herpes Virus 8 (HHV-8)
The primary virus associated with Kaposi’s sarcoma is Human herpes virus 8 (HHV-8), also known as Kaposi’s sarcoma-associated herpesvirus (KSHV). HHV-8 was discovered in 1994 and has since been established as a necessary factor in the development of KS, particularly in immunocompromised individuals, such as those with HIV/AIDS.
Mechanism of Action: HHV-8 infects endothelial cells, which line blood vessels, leading to their proliferation and the formation of vascular tumors characteristic of KS. The virus encodes several proteins that interfere with normal cellular processes, promoting cell survival and proliferation while inhibiting apoptosis (programmed cell death).
Epidemiology: The prevalence of HHV-8 varies geographically and demographically. It is more common in certain populations, including men who have sex with men and individuals from regions such as sub-Saharan Africa where KS is endemic.
Clinical Presentation: In patients who are immunocompromised, particularly those with HIV/AIDS, KS can present aggressively. In contrast, in immunocompetent individuals, KS may remain asymptomatic or present more indolently.
Other Viruses Considered
While HHV-8 is the primary virus linked to Kaposi’s sarcoma, other viruses listed in the question have different associations:
Human Papillomavirus 16 & 18 (HPV): These viruses are primarily associated with cervical cancer and other anogenital cancers but are not directly linked to Kaposi’s sarcoma.
Human T-Lymphotropic Virus 1 (HTLV-1): This virus is associated with adult T-cell leukemia/lymphoma but does not have a direct connection to KS.
Epstein-Barr Virus (EBV): While EBV has been implicated in various lymphoproliferative disorders and some forms of lymphoma, it does not have a direct causal relationship with Kaposi’s sarcoma.
In conclusion, after analyzing the associations between these viruses and Kaposi’s sarcoma:
The correct answer is: Human herpes virus 8
Oncoviruses
Viruses which cause cancer
These may be detected on blood test and prevented by vaccine
These are the main types of oncoviruses and their diseases:
Oncovirus Cancer
Epstein-Barr virus Burkitt’s lymphoma
Hodgkin’s lymphoma
Post transplant lymphoma
Nasopharyngeal carcinoma
Human papillomavirus 16/18 Cervical cancer
Anal cancer
Penile cancer
Vulval cancer
Oropharyneal cancer
Human herpes virus 8 Kaposi’s sarcoma
Hepatitis B virus Hepatocellular carcinoma
Hepatitis C virus Hepatocellular carcinoma
Human T-lymphotropic virus 1 Tropical spastic paraparesis
Adult T cell leukaemia
A 57 year old lady has suffered from lymphoedema for many years. The left leg is swollen to the mid thigh. Severe limb deformity has developed as a result of process and in spite of compression hosiery. Lymphoscintography shows no patent lymphatics in the proximal leg. The overlying skin is healthy. Which of the following options would give the best long term outcome?
Multilayer compression bandages
Charles operation
Homans operation
Long term loop diuretic therapy
Amputation
Surgery is indicated in less than 10% of cases. However, severe deformity is one of the indications for surgery. Lymphovenous anastomosis is indicated where the proximal lymphatics are not patent. When the overlying skin is healthy (and limb deformity a problem), a Homans procedure is a reasonable first line operative option.
Lymphoedema
- Due to impaired lymphatic drainage in the presence of normal capillary function.
Lymphoedema causes the accumulation of protein rich fluid, subdermal fibrosis and dermal thickening.
Characteristically fluid is confined to the epifascial space (skin and subcutaneous tissues); muscle compartments are free of oedema. It involves the foot, unlike other forms of oedema. There may be a ‘buffalo hump’ on the dorsum of the foot and the skin cannot be pinched due to subcutaneous fibrosis.
Causes of lymphoedema
Primary
Congenital < 1 year: sporadic, Milroy’s disease
Onset 1-35 years: sporadic, Meige’s disease
> 35 years: Tarda
Secondary
Bacterial/fungal/parasitic infection (filariasis)
Lymphatic malignancy
Radiotherapy to lymph nodes
Surgical resection of lymph nodes
DVT
Thrombophlebitis
Indications for surgery
Marked disability or deformity from limb swelling
Lymphoedema caused by proximal lymphatic obstruction with patent distal lymphatics suitable for a lymphatic drainage procedure
Lymphocutaneous fistulae and megalymphatics
Procedures
Homans operation Reduction procedure with preservation of overlying skin (which must be in good condition). Skin flaps are raised and the underlying tissue excised. Limb circumference typically reduced by a third.
Charles operation All skin and subcutaneous tissue around the calf are excised down to the deep fascia. Split skin grafts are placed over the site. May be performed if overlying skin is not in good condition. Larger reduction in size than with Homans procedure.
Lymphovenous anastamosis Identifiable lymphatics are anastomosed to sub dermal venules. Usually indicated in 2% of patients with proximal lymphatic obstruction and normal distal lymphatics.
Which option is not recommended during the management of compartment syndrome?
Anticoagulation
Keep limb level with the body
Intravenous fluids
Pain control
Fasciotomy
Anticoagulation will worsen compartment syndrome.
Compartment syndrome
- This is a particular complication that may occur following fractures (or following ischaemia re-perfusion injury in vascular patients). It is characterised by raised pressure within a closed anatomical space.
The raised pressure within the compartment will eventually compromise tissue perfusion resulting in necrosis. The two main fractures carrying this complication include supracondylar fractures and tibial shaft injuries.
Symptoms and signs
Pain, especially on movement (even passive)
Parasthesiae
Pallor may be present
Arterial pulsation may still be felt as the necrosis occurs as a result of microvascular compromise
Paralysis of the muscle group may occur
Diagnosis
Is made by measurement of intracompartmental pressure measurements. Pressures in excess of 20mmHg are abnormal and >40mmHg is diagnostic.
Treatment
This is essentially prompt and extensive fasciotomies
In the lower limb the deep muscles may be inadequately decompressed by the inexperienced operator when smaller incisions are performed
Myoglobinuria may occur following fasciotomy and result in renal failure and for this reason these patients require aggressive IV fluids
Where muscle groups are frankly necrotic at fasciotomy they should be debrided and amputation may have to be considered
Death of muscle groups may occur within 4-6 hours
As it exits the axilla the radial nerve passes under the inferior border of which of the muscles listed below?
Supraspinatus
Infraspinatus
Teres major
Deltoid
Pectoralis major
The radial nerve passes through the triangular interval to leave the axilla. The superior border of this is bounded by the teres major muscle to which the radial nerve is closely related.
Radial nerve
Continuation of posterior cord of the brachial plexus (root values C5 to T1)
Path
In the axilla: lies posterior to the axillary artery on subscapularis, latissimus dorsi and teres major.
Enters the arm between the brachial artery and the long head of triceps (medial to humerus).
Spirals around the posterior surface of the humerus in the groove for the radial nerve.
At the distal third of the lateral border of the humerus it then pierces the intermuscular septum and descends in front of the lateral epicondyle.
At the lateral epicondyle it lies deeply between brachialis and brachioradialis where it then divides into a superficial and deep terminal branch.
Deep branch crosses the supinator to become the posterior interosseous nerve.
Regions innervated
Motor (main nerve)
Triceps
Anconeus
Brachioradialis
Extensor carpi radialis
Motor (posterior interosseous branch)
Supinator
Extensor carpi ulnaris
Extensor digitorum
Extensor indicis
Extensor digiti minimi
Extensor pollicis longus and brevis
Abductor pollicis longus
Sensory The area of skin supplying the proximal phalanges on the dorsal aspect of the hand is supplied by the radial nerve (this does not apply to the little finger and part of the ring finger)
Muscular innervation and effect of denervation
Anatomical location Muscle affected Effect of paralysis
Shoulder Long head of triceps Minor effects on shoulder stability in abduction
Arm Triceps Loss of elbow extension
Forearm Supinator
Brachioradialis
Extensor carpi radialis longus and brevis
Through which of the structures listed below does the axillary nerve pass?
Quadrangular space
Triangular space
Subclavicular space
Sub pectoral space
Intercostal space
Axillary nerve passes through the quadrangular space
Axillary nerve
Terminal branch of the posterior cord of the brachial plexus
Root values C5 and C6
Descends posterior to the axillary artery at the lower border of subscapularis and then passes through quadrangular space with the posterior circumflex humeral vessels
Divides into anterior and posterior branches
Innervates deltoid muscle and small patch of skin over deltoid
A 75 year old man presents with locally advanced carcinoma of the prostate and vertebral body metastasis with impending spinal cord compression. Which of the following agents (if used in isolation) carries the greatest risk of worsening his symptoms in the short term?
Surgical orchidectomy
Cyproterone acetate
Luteinising hormone releasing hormone analogues
Flutamide
None of the above
LHRH analogues may cause flare of metastatic disease and anti androgens should be administered to counter this. Surgical orchidectomy reduces testosterone levels within 8 hours (but fails to reduce adrenal androgen release). Cyproterone and flutamide are androgen blockers that may be considered as add on therapy to reduce the risk of tumour flare when commencing treatment with LH RH analogues.
Prostate Cancer
Prostate Cancer
This is a common condition and up to 30,000 men are diagnosed with the condition each year. Up to 9,000 will die in in the UK from the condition per year.
Diagnosis
Early prostate cancers have few symptoms.
Metastatic disease may present as bone pain.
Locally advanced disease may present as pelvic pain or with urinary symptoms.
Prostate specific antigen measurement
Digital rectal examination
Trans rectal USS (+/- biopsy)
MRI/ CT and bone scan for staging.
PSA Test
The normal upper limit for PSA is 4ng/ml. However, in this group will lie patients with benign disease and some with localised prostate cancer. False positives may be due to prostatitis, UTI, BPH, vigorous DRE.
The percentage of free: total PSA may help to distinguish benign disease from cancer. Values of <20% are suggestive of cancer and biopsy is advised.
Pathology
95% adenocarcinoma
In situ malignancy is sometimes found in areas adjacent to cancer. Multiple biopsies needed to call true in situ disease.
Often multifocal- 70% lie in the peripheral zone.
Graded using the Gleason grading system, two grades awarded 1 for most dominant grade (on scale of 1-5) and 2 for second most dominant grade (scale 1-5). The two added together give the Gleason score. Where 2 is best prognosis and 10 the worst.
Lymphatic spread occurs first to the obturator nodes and local extra prostatic spread to the seminal vesicles is associated with distant disease.
Treatment
Watch and wait- Elderly, multiple co-morbidities, low Gleason score
Radiotherapy (External)- Both potentially curative and palliative therapy possible. However, radiation proctitis and rectal malignancy are late problems. Brachytherapy is a modification allowing internal radiotherapy.
Surgery- Radical prostatectomy. Surgical removal of the prostate is the standard treatment for localised disease. The robot is being used increasingly for this procedure. As well as the prostate the obturator nodes are also removed to complement the staging process. Erectile dysfunction is a common side effect. Survival may be better than with radiotherapy (see references). Functional outcomes are better when a robotic approach is used.
Hormonal therapy- Testosterone stimulates prostate tissue and prostatic cancers usually show some degree of testosterone dependence. 95% of testosterone is derived from the testis and bilateral orchidectomy may be used for this reason. Pharmacological alternatives include LHRH analogues and anti androgens (which may be given in combination).
In the UK the National Institute for Clinical Excellence (NICE) suggests that active surveillance is the preferred option for low risk men. It is particularly suitable for men with clinical stage T1c, Gleason score 3+3 and PSA density < 0.15 ng/ml/ml who have cancer in less than 50% of their biopsy cores, with < 10 mm of any core involved.
Candidates for active surveillance should:
have had at least 10 biopsy cores taken
have at least one re-biopsy.
If men on active surveillance show evidence of disease progression, offer radical treatment. Treatment decisions should be made with the man, taking into account co-morbidities and life expectancy.
During a difficult femoro-popliteal bypass operation the surgeon inadvertently places a clamp across the femoral nerve. It remains there for most of the procedure. At the end of the operation the nerve is inspected, it is in continuity but has evidence of being crushed. Which of the following is most likely to occur over the following weeks?
Wallerian degeneration
Rapid restoration of neuronal function because the axon itself is intact
Normal but delayed neuronal transmission due to disruption of the myelin
Absence of neuroma formation
None of the above
A neuronal injury such as this will result in Wallerian degeneration even though the nerve remains in continuity. Neuromas may well form.
Nerve injury
There are 3 types of nerve injury:
Neuropraxia
Nerve intact but electrical conduction is affected
Full recovery
Autonomic function preserved
Wallerian degeneration does not occur
Axonotmesis
Axon is damaged and the myelin sheath is preserved. The connective tissue framework is not affected.
Wallerian degeneration occurs.
Neurotmesis
Disruption of the axon, myelin sheath and surrounding connective tissue.
Wallerian degeneration occurs.
Wallerian Degeneration
Axonal degeneration distal to the site of injury.
Typically begins 24-36 hours following injury.
Axons are excitable prior to degeneration occurring.
Myelin sheath degenerates and is phagocytosed by tissue macrophages.
Nerve repair
Neuronal repair may only occur physiologically where nerves are in direct contact. Where a large defect is present, the process of nerve regeneration is hampered. It may not occur at all or result in the formation of a neuroma. Where nerve regrowth occurs it is typically at a rate of 1mm per day.
A 25 year old man is undergoing respiratory spirometry. He takes a maximal inspiration and maximally exhales. Which of the following measurements will best illustrate this process?
Functional residual capacity
Vital capacity
Inspiratory capacity
Maximum voluntary ventilation
Tidal volume
The maximum voluntary ventilation is the maximal ventilation over the course of 1 minute.
Definitions
Tidal volume (TV)
Is the volume of air inspired and expired during each ventilatory cycle at rest.
It is normally 500mls in males and 340mls in females.
Inspiratory reserve volume (IRV)
Is the maximum volume of air that can be forcibly inhaled following a normal inspiration. 3000mls.
Expiratory reserve volume (ERV)
Is the maximum volume of air that can be forcibly exhaled following a normal expiration. 1000mls.
Residual volume (RV)
Is that volume of air remaining in the lungs after a maximal expiration.
RV = FRC - ERV. 1500mls.
Functional residual capacity (FRC)
Is the volume of air remaining in the lungs at the end of a normal expiration.
FRC = RV + ERV. 2500mls.
Vital capacity (VC)
Is the maximal volume of air that can be forcibly exhaled after a maximal inspiration.
VC = TV + IRV + ERV. 4500mls in males, 3500mls in females.
Total lung capacity (TLC)
Is the volume of air in the lungs at the end of a maximal inspiration.
TLC = FRC + TV + IRV = VC + RV. 5500-6000mls.
Forced vital capacity (FVC)
The volume of air that can be maximally forcefully exhaled.
Which of the following laryngeal tumours will not typically metastasise to the cervical lymph nodes?
Glottic
Supraglottic
Subglottic
Transglottic
Aryepiglottic fold
A-The vocal cords have no lymphatic drainage and therefore this region serves as a lymphatic watershed. The supraglottic part drains to the upper deep cervical nodes through vessels piercing the thyrohyoid membrane. The sub glottic part drains to the pre laryngeal, pre tracheal and inferior deep cervical nodes. The aryepiglottic and vestibular folds have a rich lymphatic drainage and will metastasise early.
Larynx
The larynx lies in the anterior part of the neck at the levels of C3 to C6 vertebral bodies. The laryngeal skeleton consists of a number of cartilagenous segments. Three of these are paired; arytenoid, corniculate and cuneiform. Three are single; thyroid, cricoid and epiglottic. The cricoid cartilage forms a complete ring (the only one to do so).
The laryngeal cavity extends from the laryngeal inlet to the level of the inferior border of the cricoid cartilage.
Divisions of the laryngeal cavity
Laryngeal vestibule Superior to the vestibular folds
Laryngeal ventricle Lies between vestibular folds and superior to the vocal cords
Infraglottic cavity Extends from vocal cords to inferior border of the cricoid cartilage
The vocal folds (true vocal cords) control sound production. The apex of each fold projects medially into the laryngeal cavity. Each vocal fold includes:
Vocal ligament
Vocalis muscle (most medial part of thyroarytenoid muscle)
The glottis is composed of the vocal folds, processes and rima glottidis. The rima glottidis is the narrowest potential site within the larynx, as the vocal cords may be completely opposed, forming a complete barrier.
Muscles of the larynx
Muscle Origin Insertion Innervation Action
Posterior cricoarytenoid Posterior aspect of lamina of cricoid Muscular process of arytenoid Recurrent Laryngeal Abducts vocal fold
Lateral cricoarytenoid Arch of cricoid Muscular process of arytenoid Recurrent laryngeal Adducts vocal fold
Thyroarytenoid Posterior aspect of thyroid cartilage Muscular process of arytenoid Recurrent laryngeal Relaxes vocal fold
Transverse and oblique arytenoids Arytenoid cartilage Contralateral arytenoid Recurrent laryngeal Closure of intercartilagenous part of the rima glottidis
Vocalis Depression between lamina of thyroid cartilage Vocal ligament and vocal process of arytenoid cartilage Recurrent laryngeal Relaxes posterior vocal ligament, tenses anterior part
Cricothyroid Anterolateral part of cricoid Inferior margin and horn of thyroid cartilage External laryngeal Tenses vocal fold
Blood supply
Arterial supply is via the laryngeal arteries, branches of the superior and inferior thyroid arteries. The superior laryngeal artery is closely related to the internal laryngeal nerve. The inferior laryngeal artery is related to the inferior laryngeal nerve. Venous drainage is via superior and inferior laryngeal veins, the former draining into the superior thyroid vein and the latter draining into the middle thyroid vein, or thyroid venous plexus.
Lymphatic drainage
The vocal cords have no lymphatic drainage and this site acts as a lymphatic watershed.
Supraglottic part Upper deep cervical nodes
Subglottic part Prelaryngeal and pretracheal nodes and inferior deep cervical nodes
The aryepiglottic fold and vestibular folds have a dense plexus of lymphatics associated with them and malignancies at these sites have a greater propensity for nodal metastasis.
How many valves lie between the superior vena cava and the right atrium?
None
One
Two
Three
Four
There are no valves which is why it is relatively easy to insert a CVP line from the internal jugular vein into the right atrium.
Superior vena cava
Drainage
Head and neck
Upper limbs
Thorax
Part of abdominal walls
Formation
Subclavian and internal jugular veins unite to form the right and left brachiocephalic veins
These unite to form the SVC
Azygos vein joins the SVC before it enters the right atrium
Relations
Anterior Anterior margins of the right lung and pleura
Posteromedial Trachea and right vagus nerve
Posterolateral Posterior aspects of right lung and pleura
Pulmonary hilum is posterior
Right lateral Right phrenic nerve and pleura
Left lateral Brachiocephalic artery and ascending aorta
Developmental variations
Anomalies of the connection of the SVC are recognised. In some individuals a persistent left sided SVC drains into the right atrium via an enlarged orifice of the coronary sinus. More rarely the left sided vena cava may connect directly with the superior aspect of the left atrium, usually associated with an un-roofing of the coronary sinus. The commonest lesion of the IVC is for its abdominal course to be interrupted, with drainage achieved via the azygos venous system. This may occur in patients with left sided atrial isomerism.
A cohort study is being designed to look at the relationship between smoking and breast cancer. What is the usual outcome measure in a cohort study?
Odds ratio
Experimental event rate
Relative risk
Absolute risk increase
Numbers needed to harm
Cohort studies - relative risk
Study design
The following table highlights the main features of the main types of study:
Randomised controlled trial Participants randomly allocated to intervention or control group (e.g. standard treatment or placebo)
Practical or ethical problems may limit use
Cohort study Observational and prospective. Two (or more) are selected according to their exposure to a particular agent (e.g. medicine, toxin) and followed up to see how many develop a disease or other outcome.
The usual outcome measure is the relative risk.
Examples include Framingham Heart Study
Case-control study Observational and retrospective. Patients with a particular condition (cases) are identified and matched with controls. Data is then collected on past exposure to a possible causal agent for the condition.
The usual outcome measure is the odds ratio.
Inexpensive, produce quick results
Useful for studying rare conditions
Prone to confounding
Cross-sectional survey Provide a ‘snapshot’, sometimes called prevalence studies
Provide weak evidence of cause and effect
An 8 week old infant is brought to clinic with a history of 18 days of jaundice. The mother is breast feeding. He was a full term baby. There is no family history of liver disease. What is the most appropriate next step?
Liver USS
Unconjugated bilirubin measurement
Conjugated bilirubin measurement
Reassure and discharge
ERCP
This baby is a full term and has > 14 days of jaundice, therefore needs an urgent conjugated bilirubin check to rule out biliary atresia. If physiological jaundice the unconjugated bilirubin levels will be increased. Isotope scanning may be used in diagnosis, but a definitive diagnosis is normally made during a laparotomy.
Biliary atresia
1 in 17000 affected
Biliary tree lumen is obliterated by an inflammatory cholangiopathy causing progressive liver damage
Clinical features
Infant well in 1st few weeks of life
No family history of liver disease
Jaundice in infants > 14 days in term infants (>21 days in pre term infants)
Pale stool, yellow urine (colourless in babies)
Associated with cardiac malformations, polysplenia, situs inversus
Investigation
Conjugated bilirubin (prolonged physiological jaundice or breast milk jaundice will cause a rise in unconjugated bilirubin, whereas those with obstructive liver disease will have a rise in conjugated bilirubin)
Ultrasound of the liver (excludes extrahepatic causes, in biliary atresia infant may have tiny or invisible gallbladder)
Hepato-iminodiacetic acid radionuclide scan (good uptake but no excretion usually seen)
Management
Early recognition is important to prevent liver transplantation.
Nutritional support.
Roux-en-Y portojejunostomy (Kasai procedure).
If Kasai procedure fails or late recognition, a liver transplant becomes the only option.
A patient is due to undergo a right hemicolectomy for a carcinoma of the caecum. Which of the following vessels will require high ligation to provide optimal oncological control?
Middle colic artery
Inferior mesenteric artery
Superior mesenteric artery
Ileo-colic artery
None of the above
The ileo - colic artery supplies the caecum and would require high ligation during a right hemicolectomy. The middle colic artery should generally be preserved when resecting a caecal lesion.
This question is essentially asking you to name the vessel supplying the caecum. The SMA does not directly supply the caecum, it is the ileocolic artery which does this.
Caecum
Location
Proximal right colon below the ileocaecal valve
Intraperitoneal
Posterior relations
Psoas
Iliacus
Femoral nerve
Genitofemoral nerve
Gonadal vessels
Anterior relations Greater omentum
Arterial supply Ileocolic artery
Lymphatic drainage Mesenteric nodes accompany the venous drainage
The caecum is the most distensible part of the colon and in complete large bowel obstruction with a competent ileocaecal valve the most likely site of eventual perforation.
A 33 year old man sustains an injury to his forearm and wrist. When examined in clinic he is unable to adduct his thumb. What is the most likely underlying nerve lesion?
Radial nerve
Superficial branch of the ulnar nerve
Median nerve
Posterior interosseous nerve
Deep branch of the ulnar nerve
Damage to the deep branch of the ulnar nerve may result in an inability to adduct the thumb. This is tested clinically by trying to withdraw a piece of paper from a patients hand grasped between thumb and index finger.
Adductor pollicis
Origin Insertion Nerve supply Actions
Tendon sheath of flexor carpi radialis
Bases of second, third and fourth metacarpals
Anterior aspect of the trapezoid and capitate bones
Transverse head comes from the longitudinal ride of the third metacarpal
A 6 year old boy presents with groin pain. He is known to be disruptive in class. He reports that he is bullied for being short. On examination, he has an antalgic gait and pain on internal rotation of the right hip. What is the most likely diagnosis?
Perthes disease
Transient synovitis
Slipped upper femoral epiphysis
Developmental dysplasia of the hip
Septic arthritis
This child is short, has hyperactivity (disruptive behaviour) and is within the age range for Perthes disease. Hyperactivity and short stature are associated with Perthes disease.
Paediatric orthopaedics
Diagnosis Mode of presentation Treatment Radiology
Developmental dysplasia of the hip
Usually diagnosed in infancy by screening tests. May be bilateral, when disease is unilateral there may be leg length inequality. As disease progresses child may limp and then early onset arthritis. More common in extended breech babies. Splints and harnesses or traction. In later years osteotomy and hip realignment procedures may be needed. In arthritis a joint replacement may be needed. However, this is best deferred if possible as it will almost certainly require revision. Initially no obvious change on plain films and USS gives best resolution until 3 months of age. On plain films Shentons line should form a smooth arc
Perthes Disease Hip pain (may be referred to the knee) usually occurring between 5 and 12 years of age. Bilateral disease in 20%. Remove pressure from joint to allow normal development. Physiotherapy. Usually self-limiting if diagnosed and treated promptly. X-rays will show flattened femoral head. Eventually in untreated cases the femoral head will fragment.
Slipped upper femoral epiphysis Typically seen in obese male adolescents. Pain is often referred to the knee. Limitation to internal rotation is usually seen. Knee pain is usually present 2 months prior to hip slipping. Bilateral in 20%. Bed rest and non-weight bearing. Aim to avoid avascular necrosis. If severe slippage or risk of it occurring then percutaneous pinning of the hip may be required. X-rays will show the femoral head displaced and falling inferolaterally (like a melting ice cream cone) The Southwick angle gives indication of disease severity
Inspection of the left ventricle reveals all except which of the following?
Papillary muscles
Trabeculae carnae
Chordae tendinae
Conus arteriosus
Openings of the venae cordis minimae
The conus arteriosus (infundibulum) is the smooth walled outflow tract of the right ventricle leading to the pulmonary trunk.
A 23 year old lady with troublesome axillary hyperhidrosis is undergoing a thorascopic sympathectomy to treat the condition. Which of the following structures will need to be divided to access the sympathetic trunk?
Intercostal vein
Intercostal artery
Parietal pleura
Visceral pleura
None of the above
The sympathetic chain lies posterior to the parietal pleura. During a thorascopic sympathetomy this structure will need to be divided. The intercostal vessels lie posteriorly. They may be damaged with troublesome bleeding but otherwise are best left alone as deliberate division will not improve surgical access
A 44 year old man undergoes a distal gastrectomy for cancer. He is slightly anaemic and therefore receives a transfusion of 4 units of packed red cells to cover both the existing anaemia and associated perioperative blood loss. He is noted to develop ECG changes that are not consistent with ischaemia. What is the most likely cause?
Hyponatraemia
Hyperkalaemia
Hypercalcaemia
Metabolic alkalosis
Hypernatraemia
The transfusion of packed red cells has been shown to increase serum potassium levels. The risk is higher with large volume transfusions and with old blood.
In which space is a lumbar puncture performed?
Subdural space
Epidural space
Subarachnoid space
Extradural space
Intraventricular space
Samples of CSF are normally obtained by inserting a needle between the third and fourth lumbar vertebrae. The tip of the needle lies in the sub arachnoid space, the spinal cord terminates at L1 and is not at risk of injury. Clinical evidence of raised intracranial pressure is a contraindication to lumbar puncture.
A 56 year old lady with idiopathic thrombocytopenic purpura has a platelet count of 50. She is due to undergo a splenectomy. What is the optimal timing of a platelet transfusion in this case?
24 hours pre-operatively
2 hours pre-operatively
Whilst making the skin incision
After ligation of the splenic artery
On removal of the spleen
ITP causes splenic sequestration of platelets. Therefore a platelet transfusion should be carefully timed. Too soon and it will be ineffective. Too late and unnecessary bleeding will occur. The optimal time is after the splenic artery has been ligated.
Which is true about post-splencetomy changes:
Platelets will rise first (therefore in ITP should be given after splenic artery clamped)
Blood film will change over following weeks, Howell Jolly bodies will appear
Other blood film changes include target cells and Pappenheimer bodies
Increased risk of post splenectomy sepsis, therefore prophylactic antibiotics and pneumococcal vaccine should be given.
All of the above
All of the above
A 21 year old man is stabbed in the antecubital fossa. A decision is made to surgically explore the wound. At operation the surgeon dissects down onto the brachial artery. A nerve is identified medially, which nerve is it likely to be?
Radial
Recurrent branch of median
Anterior interosseous
Ulnar
Median
Median Median nerve
The median nerve is formed by the union of a lateral and medial root respectively from the lateral (C5,6,7) and medial (C8 and T1) cords of the brachial plexus; the medial root passes anterior to the third part of the axillary artery. The nerve descends lateral to the brachial artery, crosses to its medial side (usually passing anterior to the artery). It passes deep to the bicipital aponeurosis and the median cubital vein at the elbow.
It passes between the two heads of the pronator teres muscle, and runs on the deep surface of flexor digitorum superficialis (within its fascial sheath).
Near the wrist it becomes superficial between the tendons of flexor digitorum superficialis and flexor carpi radialis, deep to palmaris longus tendon. It passes deep to the flexor retinaculum to enter the palm, but lies anterior to the long flexor tendons within the carpal tunnel.
Branches
Region Branch
Upper arm No branches, although the nerve commonly communicates with the musculocutaneous nerve
Forearm Pronator teres
Pronator quadratus
Flexor carpi radialis
Palmaris longus
Flexor digitorum superficialis
Flexor pollicis longus
Flexor digitorum profundus (only the radial half)
Distal forearm Palmar cutaneous branch
Hand (Motor) Motor supply (LOAF)
Lateral 2 lumbricals
Opponens pollicis
Abductor pollicis brevis
Flexor pollicis brevis
Hand (Sensory)
Over thumb and lateral 2 ½ fingers
On the palmar aspect this projects proximally, on the dorsal aspect only the distal regions are innervated with the radial nerve providing the more proximal cutaneous innervation.
Patterns of damage
Damage at wrist
e.g. carpal tunnel syndrome
paralysis and wasting of thenar eminence muscles and opponens pollicis (ape hand deformity)
sensory loss to palmar aspect of lateral (radial) 2 ½ fingers
Damage at elbow, as above plus:
unable to pronate forearm
weak wrist flexion
ulnar deviation of wrist
Anterior interosseous nerve (branch of median nerve)
leaves just below the elbow
results in loss of pronation of forearm and weakness of long flexors of thumb and index finger
Survival in HCC:
% 5 at 5 years
% 10 at 5 years
% 15 at 5 years
% 30 at 5 years
% 15 at 5 years
Treatment of cholangiocarcinoma, which is true?
Surgical resection offers the best chance of cure.
Local invasion of peri hilar tumours is a particular problem
Lobar atrophy will often contra indicate surgical resection.
Palliation of jaundice is important,
Metallic stents should be avoided in those considered for resection.
None of the above
All of the above
All of the above
Survival in cholangiocarcinoma:
% 1-5 at 5 years
% 5-10 at 5 years
% 20 at 5 years
% 25 at 5 years
% 5-10 at 5 years
A 56 year old man has long standing chronic pancreatitis and develops pancreatic insufficiency. Which of the following will be absorbed normally?
Fat
Protein
Folic acid
Vitamin B12
None of the above
Pancreatic lipase is required for digestion of fat, Proteases facilitate protein and B12 absorption. Folate digestion is independent of the pancreas.
Pancreas exocrine physiology
Composition of pancreatic secretions
Pancreatic secretions are usually 1000-1500ml per 24 hours and have a pH of 8.
Secretion Source Substances secreted
Enzymic Acinar cells Trypsinogen
Procarboxylase
Amylase
Elastase
Aqueous Ductal and Centroacinar cells Sodium
Bicarbonate
Water
Potassium
Chloride
NB: Sodium and potassium reflect their plasma levels; chloride and bicarbonate vary with flow rate
Regulation
The cephalic and gastric phases (neuronal and physical) are less important in regulating the pancreatic secretions. The effect of digested material in the small bowel stimulates CCK release and ACh which stimulate acinar and ductal cells. Of these CCK is the most potent stimulus. In the case of the ductal cells these are potently stimulated by secretin which is released by the S cells of the duodenum. This results in an increase in bicarbonate.
Enzyme activation
Trypsinogen is converted via enterokinase to active trypsin in the duodenum. Trypsin then activates the other inactive enzymes
A 65 year old man with long standing atrial fibrillation develops an embolus to the lower leg. The decision is made to perform an embolectomy, utilising a trans popliteal approach. After incising the deep fascia, which of the following structures will the surgeons encounter first on exploring the central region of the popliteal fossa?
Popliteal vein
Common peroneal nerve
Popliteal artery
Tibial nerve
None of the above
The tibial nerve lies superior to the vessels in the inferior aspect of the popliteal fossa. In the upper part of the fossa the tibial nerve lies lateral to the vessels, it then passes superficial to them to lie medially. The popliteal artery is the deepest structure in the popliteal fossa.
A 73 year old lady is admitted with a brisk rectal bleed. She is otherwise well and the bleed settles. On examination her abdomen is soft and non tender. Elective colonoscopy shows a small erythematous lesion in the right colon, but no other abnormality. Diagnosis?
A.Haemorrhoids
B.Meckels diverticulum
C.Angiodysplasia
D.Colonic cancer
E.Diverticular bleed
F.Ulcerative colitis
G.Ischaemic colitis
The correct answer is Angiodysplasia
Angiodysplasia can be difficult to identify and treat. The colonoscopic stigmata are easily missed by poor bowel preparation.
Lower Gastrointestinal bleeding
Colonic bleeding
This typically presents as bright red or dark red blood per rectum. Colonic bleeding rarely presents as malaena type stool, this is because blood in the colon has a powerful laxative effect and is rarely retained long enough for transformation to occur and because the digestive enzymes present in the small bowel are not present in the colon. Up to 15% of patients presenting with haemochezia will have an upper gastrointestinal source of haemorrhage.
As a general rule right sided bleeds tend to present with darker coloured blood than left sided bleeds. Haemorrhoidal bleeding typically presents as bright red rectal bleeding that occurs post defecation either onto toilet paper or into the toilet pan. It is very unusual for haemorrhoids alone to cause any degree of haemodynamic compromise.
Causes
Cause Presenting features
Colitis Bleeding may be brisk in advanced cases, diarrhoea is commonly present. Abdominal x-ray may show featureless colon.
Diverticular disease Acute diverticulitis often is not complicated by major bleeding and diverticular bleeds often occur sporadically. 75% all will cease spontaneously within 24-48 hours. Bleeding is often dark and of large volume.
Cancer Colonic cancers often bleed and for many patients this may be the first sign of the disease. Major bleeding from early lesions is uncommon
Haemorrhoidal bleeding Typically bright red bleeding occurring post defecation. Although patients may give graphic descriptions bleeding of sufficient volume to cause haemodynamic compromise is rare.
Angiodysplasia Apart from bleeding, which may be massive, these arteriovenous lesions cause little in the way of symptoms. The right side of the colon is more commonly affected.
Management
Prompt correction of any haemodynamic compromise is required. Unlike upper gastrointestinal bleeding the first line management is usually supportive. This is because in the acute setting endoscopy is rarely helpful.
When haemorrhoidal bleeding is suspected a proctosigmoidoscopy is reasonable as attempts at full colonoscopy are usually time consuming and often futile.
In the unstable patient the usual procedure would be an angiogram (either CT or percutaneous), when these are performed during a period of haemodynamic instability they may show a bleeding point and may be the only way of identifying a patch of angiodysplasia.
In others who are more stable the standard procedure would be a colonoscopy in the elective setting. In patients undergoing angiography attempts can be made to address the lesion in question such as coiling. Otherwise surgery will be necessary.
In patients with ulcerative colitis who have significant haemorrhage the standard approach would be a sub total colectomy, particularly if medical management has already been tried and is not effective.
Indications for surgery
Patients > 60 years
Continued bleeding despite endoscopic intervention
Recurrent bleeding
Known cardiovascular disease with poor response to hypotension
Surgery
Selective mesenteric embolisation if life threatening bleeding. This is most helpful if conducted during a period of relative haemodynamic instability. If all haemodynamic parameters are normal then the bleeding is most likely to have stopped and any angiography normal in appearance. In many units a CT angiogram will replace selective angiography but the same caveats will apply.
If the source of colonic bleeding is unclear; perform a laparotomy, on table colonic lavage and following this attempt a resection. A blind sub total colectomy is most unwise, for example bleeding from an small bowel arterio-venous malformation will not be treated by this manoeuvre.
Summary of Acute Lower GI bleeding recommendations
Consider admission if:
* Over 60 years
* Haemodynamically unstable/profuse PR bleeding
* On aspirin or NSAID
* Significant co morbidity
Management
All patients should have a history and examination, PR and proctoscopy
Colonoscopic haemostasis aimed for in post polypectomy or diverticular bleeding
References
http://www.sign.ac.uk/guidelines/fulltext/105/index.html
A 39 year old man has suffered from terminal ileal Crohns disease for the past 20 years. Which condition is he least likely to develop?
Gallstones
Malabsorption
Pyoderma gangrenosum
Amyloidosis
Feltys syndrome
Step 1: Understanding Crohn’s Disease and Its Complications
Crohn’s disease is a type of inflammatory bowel disease (IBD) that can affect any part of the gastrointestinal tract but most commonly affects the terminal ileum. It is characterized by chronic inflammation, which can lead to various complications over time. Patients with Crohn’s disease are at risk for several conditions due to the nature of their illness.
Step 2: Evaluating Each Condition
Gallstones: Patients with Crohn’s disease, particularly those with ileal involvement, are at an increased risk for gallstones. This is primarily due to malabsorption of bile salts, which can lead to changes in bile composition and promote gallstone formation.
Malabsorption: Malabsorption is a common complication in patients with Crohn’s disease, especially when the terminal ileum is affected. The terminal ileum is responsible for absorbing vitamin B12 and bile acids; thus, its dysfunction can lead to significant nutritional deficiencies.
Pyoderma Gangrenosum: This is a rare skin condition associated with IBD, including Crohn’s disease. While it can occur in patients with Crohn’s, it is not as common as other complications.
Amyloidosis: This condition involves the deposition of amyloid proteins in various tissues and organs and can occur in patients with chronic inflammatory diseases like Crohn’s disease. However, it is relatively rare compared to other complications.
Felty’s Syndrome: This syndrome consists of rheumatoid arthritis, splenomegaly, and neutropenia (low white blood cell count). It is primarily associated with rheumatoid arthritis rather than Crohn’s disease or other forms of IBD.
Step 3: Determining the Least Likely Condition
Given the above evaluations:
Gallstones are likely due to malabsorption.
Malabsorption itself is very likely.
Pyoderma gangrenosum may occur but isn’t as prevalent.
Amyloidosis could develop but is less common.
Felty’s syndrome has no direct association with Crohn’s disease and occurs primarily in rheumatoid arthritis patients.
Thus, among these options, Felty’s syndrome stands out as the least likely condition for a patient suffering from terminal ileal Crohn’s disease.
Answer: Felty’s syndrome
A 56 year old male presents to the acute surgical take with severe abdominal pain. He is normally fit and well. He has no malignancy. The biochemistry laboratory contacts the ward urgently, his corrected calcium result is 3.6 mmol/l. What is the medication of choice to treat this abnormality?
IV Pamidronate
Oral Alendronate
Dexamethasone
Vitamin D
Resonium salts
IV Pamidronate is the drug of choice as it most effective and has long lasting effects. Calcitonin would need to be given with another agent, to ensure that the hypercalcaemia is treated once its short term effects wear off. IV zoledronate is preferred in scenarios associated with malignancy.
A 53 year old man is undergoing a distal pancreatectomy for trauma. Which of the following vessels is responsible for the arterial supply to the tail of the pancreas?
Splenic artery
Pancreaticoduodenal artery
Gastric artery
Hepatic artery
Superior mesenteric artery
Pancreatic head is supplied by the pancreaticoduodenal artery
Pancreatic tail is supplied by branches of the splenic artery
There is an arterial watershed in the supply between the head and tail of the pancreas. The head is supplied by the pancreaticoduodenal artery and the tail is supplied by branches of the splenic artery.
A 43 year old lady presents with varicose veins and undergoes a saphenofemoral disconnection, long saphenous vein stripping to the ankle and isolated hook phlebectomies. Post operatively she notices an area of numbness superior to her ankle. What is the most likely cause for this?
Sural nerve injury
Femoral nerve injury
Saphenous nerve injury
Common peroneal nerve injury
Superficial peroneal nerve injury
The most likely cause for the postoperative numbness superior to the ankle in this case is a Saphenous nerve injury. The Saphenous nerve runs alongside the long saphenous vein, which is typically addressed during procedures like saphenofemoral disconnection and vein stripping. Nerve injuries during these surgeries can lead to sensory disturbances such as numbness. It’s advisable for the patient to discuss this with their healthcare provider for a more accurate assessment.
The sural nerve is related to the short saphenous vein. The saphenous nerve is related to the long saphenous vein below the knee and for this reason full length stripping of the vein is no longer advocated.
Course of the long saphenous, which one is true?
1st digit where the dorsal vein merges with the dorsal venous arch of the foot.
Passes anterior to the medial malleolus and runs up the medial side of the leg.
At the knee it runs over the posterior border of the medial epicondyle of the femur.
Passes laterally to lie on the anterior surface of the thigh before entering the saphenous opening in the fascia lata.
Joins the femoral vein in the femoral triangle at the SFJ
All
Tributaries of the saphenous vein, which one is true?
Medial marginal
Superficial epigastric
Superficial iliac circumflex
Superficial external pudendal veins
Tributaries of the Saphenous Vein
The saphenous vein is a major superficial vein in the leg, primarily responsible for draining blood from the lower extremities. It has several tributaries that contribute to its function. To determine which of the listed veins are true tributaries of the saphenous vein, we will analyze each option step by step.
Medial Marginal Vein: The medial marginal vein runs along the medial side of the foot and is known to drain into the great saphenous vein. Therefore, this option is indeed a true tributary.
Superficial Epigastric Vein: This vein arises from the femoral vein and travels upward towards the abdomen, draining into the external iliac vein rather than directly into the saphenous system. Thus, it is not a tributary of the saphenous vein.
Superficial Iliac Circumflex Vein: Similar to the superficial epigastric vein, this vessel also drains into the external iliac vein and does not connect with or drain into the saphenous system. Therefore, it is not a tributary of the saphenous vein.
Superficial External Pudendal Veins: These veins also originate from areas around the groin and drain into either the femoral or external iliac veins rather than directly contributing to the saphenous system. Hence, they are not considered tributaries of the saphenous vein.
After analyzing all four options:
The only true tributary of the saphenous vein among those listed is Medial Marginal Vein.
Thus, based on this detailed examination:
Answer: Medial marginal
A 58 year old man is reviewed in the clinic following a successful cadaveric renal transplant the previous year. He has been able to return to work as a swimming instructor. Over the past week he reports that he has been suffering from recurrent episodes of diarrhoea. It has made him feel lethargic and exhausted. Stool microscopy shows evidence of cysts.
A.Giardia Infection
B.Cryptosporidium infection
C.Clonorchis sinensis infection
D.Ancylostoma duodenale infection
E.Ascaris lumbricoides infection
Cryptosporidium infection
Cryptosporidium is associated with infection, particularly in those who are immunocompromised. Diarrhoea is the main disease. The cysts are typically identified on stool microscopy.
A 23 year old women has undergone a pan proctocolectomy and ileoanal pouch because she suffers from familial adenomatous polyposis coli. What is the commonest extra colonic lesion in this disorder?
Gastric fundal polyps
Trichilemmomas
Duodenal polyps
Fibrocystic disease of the breast
Skull osteomas
Duodenal polyps occur in up to 100% of patients with FAP if follow up is continued for long enough. Duodenal cancer has an incidence of 4-10%.
Duodenal polyps are the commonest extra colonic lesion in FAP. Gastric fundal polyps are seen in 50% of patients. Skull osteomas are seen in Gardeners syndrome which is a variant of FAP.
The oxygen-haemoglobin dissociation curve is shifted to the right in which of the following scenarios?
Hypothermia
Respiratory alkalosis
Low altitude
Decreased 2,3-DPG in transfused red cells
Chronic iron deficiency anaemia
Hypothermia:
Hypothermia typically causes a leftward shift in the oxygen-haemoglobin dissociation curve. This is because lower temperatures increase hemoglobin’s affinity for oxygen, making it hold onto oxygen more tightly rather than releasing it.
Respiratory Alkalosis:
Respiratory alkalosis occurs when there is a decrease in carbon dioxide (CO2) levels due to hyperventilation, leading to an increase in blood pH. This condition also results in a leftward shift of the curve, as higher pH increases hemoglobin’s affinity for oxygen.
Low Altitude:
At low altitudes, the partial pressure of oxygen is higher compared to high altitudes; however, this does not directly cause a rightward shift in the dissociation curve. The body generally maintains normal hemoglobin-oxygen affinity at low altitudes.
Decreased 2,3-DPG in Transfused Red Cells:
2,3-Diphosphoglycerate (2,3-DPG) is a metabolite produced by red blood cells that decreases hemoglobin’s affinity for oxygen when present in higher concentrations. In transfused red cells where 2,3-DPG levels are decreased (as stored blood has lower levels), there would be a leftward shift rather than a rightward one.
Chronic Iron Deficiency Anaemia:
Chronic iron deficiency anemia can lead to changes in hemoglobin structure and function that may result in a rightward shift of the curve due to increased production of 2,3-DPG as the body attempts to enhance tissue oxygen delivery despite reduced overall hemoglobin levels.
Conclusion
Based on the analysis above:
The only scenario among those listed that leads to a rightward shift of the oxygen-haemoglobin dissociation curve is Chronic Iron Deficiency Anaemia.
Thus, the answer is:
Chronic iron deficiency anaemia
Mnemonic to remember causes of right shift of the oxygen dissociation curve:
CADET face RIGHT
C O2
A cidosis
2,3-DPG
E xercise
T emperature
The curve is shifted to the right when there is an increased oxygen requirement by the tissue. This includes:
Increased temperature
Acidosis
Increased DPG:
DPG is found in erythrocytes and is increased during glycolysis. It binds to the Hb molecule, thereby releasing oxygen to tissues. DPG is increased in conditions associated with poor oxygen delivery to tissues, such as anaemia and high altitude.
A pathologist is examining a histological section and identifies Hassall’s corpuscles. With what are they most commonly associated?
Follicular carcinoma of the thyroid
Medulla of the thymus
Medulla of the spleen
Medulla of the kidney
Fundus of the stomach
Hassall’s corpuscles are the concentric ring of epithelial cells seen in the medulla of the thymus.
A 25 year old man is stabbed in the upper arm. The brachial artery is lacerated at the level of the proximal humerus, and is being repaired. A nerve lying immediately lateral to the brachial artery is also lacerated. Which of the following is the nerve most likely to be?
Ulnar nerve
Median nerve
Radial nerve
Intercostobrachial nerve
Axillary nerve
The brachial artery begins at the lower border of teres major and terminates in the cubital fossa by branching into the radial and ulnar arteries. In the upper arm the median nerve lies closest to it in the lateral position. In the cubital fossa it lies medial to it.
A 63 year old man undergoes an upper GI endoscopy and adrenaline injection for a large actively bleeding duodenal ulcer. He remains stable for 6 hours and the nurses then call because he has passed 400ml malaena and has become tachycardic (pulse rate 120) and hypotensive (Bp 80/40). What is the best option?
Reassure that blood trapped in the upper portion of the gastrointestinal system will pass and that this episode will resolve with phosphate enema
Perform a repeat upper GI endoscopy
Perform a laparotomy and under-running of the ulcer
Administer tranexamic acid and intravenous proton pump inhibitors
Insert a Minnesota tube
The decision as to how best to manage patients with re-bleeding is difficult. Whilst it is tempting to offer repeat endoscopy, this intervention is best used on those with small ulcers. Large posteriorly sited duodenal ulcers are at high risk for re-bleeding and the timeframe of this event suggests that primary endoscopic haemostasis was inadequate. Surgery thus represents the safest way forward.
A 64 year old man presents to the clinic with right upper quadrant discomfort. He has never attended the hospital previously and is usually well. He has just retired from full time employment as a machinist in a PVC factory. CT scanning shows a large irregular tumour in the right lobe of his liver. Which of the following lesions is the most likely?
Liposarcoma
Angiosarcoma
Hamartoma
Hyatid liver disease
Benign angioma
Angiosarcoma of the liver is a rare tumour. However, it is linked to working with vinyl chloride, as in this case. Although modern factories minimise the exposure to this agent, this has not always been the case.
A 45 year old man undergoes a sub total colectomy and formation of end ileostomy. What is the most likely sodium content per litre of ileostomy fluid?
120 mmol
60 mmol
20 mmol
210 mmol
180 mmol
Investigators in the 1960’s dehydrated and measured the sodium content of ileostomy effluent and determined this concentration. Not an experiment many would care to repeat! (120)
Which of the following muscle relaxants will tend to incite neuromuscular excitability following administration?
Atracurium
Suxamethonium
Vecuronium
Pancuronium
None of the above
Suxamethonium may induce generalised muscular contractions following administration. This may raise serum potassium levels.
Which of the following blood products can be administered to a non ABO matched recipient?
Whole blood
Platelets
Packed red cells
Stem cells
Cryoprecipitate
In the UK, platelets either come from pooling of the platelet component from four units of whole donated blood, called random donor platelets, or by plasmapharesis from a single donor. The platelets are suspended in 200-300 ml of plasma and may be stored for up to 4 days in the transfusion laboratory where they are continually agitated at 22oC to preserve function. One adult platelet pool raises the normal platelet count by 30,000 to 60,000 platelets litre. ABO identical or compatible platelets are preferred but not necessary in adults; but rhesus compatibility is required in recipients who are children and women of childbearing age to prevent haemolytic disease of the newborn.
Removal of all plasma from a blood unit and substitution with:
Sodium chloride
Adenine
Anhydrous glucose
Mannitol
Up to 4 units of SAG M Blood may be administered. Thereafter whole blood is preferred. After 8 units, clotting factors and platelets should be considered.
Which of the following is not a content of the cavernous sinus?
Oculomotor nerve
Internal carotid artery
Opthalmic nerve
Abducens nerve
Optic nerve
Mnemonic for contents of cavernous sinus:
O TOM CAT
Occulomotor nerve (III)
Trochlear nerve (IV)
Ophthalmic nerve (V1)
Maxillary nerve (V2)
Carotid artery
Abducent nerve (VI)
T
OTOM=lateral wall components
CA= components within sinus
The optic nerve lies above and outside the cavernous sinus.
A 44 year old man with end stage renal failure undergoes a live donor renal transplant. During the immediate post operative period a good urine output is recorded. However, on return to the ward the nursing staff notice that the urinary catheter is no longer draining. However, the urostomy is continuing to drain urine.
A.Acute tubular necrosis
B.Renal artery thrombosis
C.Bladder occlusion
D.Ureteric occlusion
E.Acute rejection
F.Acute on chronic rejection
G.Hyperacute rejection
Bladder occlusion
The most likely explanation for this event is a blocked catheter. This may be the result of blood clot from the ureteric anastomosis. Bladder irrigation will usually resolve the problem.
A 43 year old man undergoes a live donor renal transplant. The donor’s right kidney is anastomosed to the recipient. On removal of the arterial clamps there is good urinary flow noted and the wounds are closed. On return to the ward the nurses notice that the patient suddenly becomes anuric and irrigation of the bladder does not improve the situation.
A.Acute tubular necrosis
B.Renal artery thrombosis
C.Bladder occlusion
D.Ureteric occlusion
E.Acute rejection
F.Acute on chronic rejection
G.Hyperacute rejection
The correct answer is Renal artery thrombosis
Right sided live donor transplants are extremely rare. This is because the vena cava precludes mobilisation of the right renal artery. The short right renal artery that is produced therefore presents a major challenge. The sudden cessation of urine output in this context is highly suggestive of an acute thrombosis. Delay in thrombectomy beyond 1 hour almost inevitably results in graft loss.
A 73 year old lady is admitted for a laparoscopic cholecystectomy. During her pre-operative assessment it is noted that she is receiving furosemide for the treatment of hypertension. Where is the site of action of this diuretic?
Proximal convoluted tubule
Descending limb of the loop of Henle
Ascending limb of the loop of Henle
Distal convoluted tubule
Collecting ducts
Action of furosemide = ascending limb of the loop of Henle
Furosemide and bumetanide are loop diuretics that act by inhibiting the Na-K-Cl cotransporter in the thick ascending limb of the loop of Henle, reducing the absorption of NaCl.
Diuretic agents
The diuretic drugs are divided into three major classes, which are distinguished according to the site at which they impair sodium reabsorption: loop diuretics in the thick ascending loop of Henle, thiazide type diuretics in the distal tubule and connecting segment; and potassium sparing diuretics in the aldosterone - sensitive principal cells in the cortical collecting tubule.
In the kidney, sodium is reabsorbed through Na+/ K+ ATPase pumps located on the basolateral membrane. These pumps return reabsorbed sodium to the circulation and maintain low intracellular sodium levels. This latter effect ensures a constant concentration gradient.
Physiological effects of commonly used diuretics
Site of action Diuretic Carrier or channel inhibited Percentage of filtered sodium excreted
Ascending limb of loop of Henle Frusemide Na+/K+ 2Cl - carrier Up to 25%
Distal tubule and connecting segment Thiazides Na+Cl- carrier Between 3 and 5%
Cortical collecting tubule Spironolactone Na+/K+ ATP ase pump Between 1 and 2%
A 56 year old man who drinks heavily is found collapsed by friends at his house. He was out drinking the previous night and following this was noted to have vomited repeatedly so his friends brought him home.
A.Schatzki ring
B.Plummer Vinson syndrome
C.Squamous cell carcinoma
D.Barretts oesophagus
E.Pharyngeal pouch
F.Adenocarcinoma
G.Leiomyoma
H.Oesophageal rupture
I.Diffuse oesophageal spasm
J.Hiatus hernia
Oesophageal rupture
Spontaneous rupture of the oesophagus may occur following an episode of vomiting. The subsequent mediastinitis can produce severe sepsis and death if not treated promptly. Adequate drainage of sepsis and early surgery are the cornerstones of management.
A 43 year old man has been troubled with dysphagia for many years. He is known to have achalasia and has had numerous dilatations. Over the past 6 weeks his dysphagia has worsened. At endoscopy a friable mass is noted in the oesophagus.
A.Schatzki ring
B.Plummer Vinson syndrome
C.Squamous cell carcinoma
D.Barretts oesophagus
Squamous cell carcinoma
The risk of squamous cell carcinoma of the oesophagus is increased in people with achalasia. The condition often presents late and has a poor prognosis.
A 73 year old lady is troubled by episodic swallowing difficulty and halitosis. An upper GI endoscopy is attempted and abandoned due to difficulty in achieving intubation.
A.Schatzki ring
B.Plummer Vinson syndrome
C.Squamous cell carcinoma
D.Barretts oesophagus
E.Pharyngeal pouch
Pharyngeal pouches occur when a defect occurs in killians dehiscence. Difficulty in intubation is a well recognised consequence and care must be taken to take the correct track during OGD to avoid perforation. Most cases are now treated with endoscopic stapling.
Surgical occlusion of which of these structures, will result in the greatest reduction in hepatic blood flow?
Portal vein
Common hepatic artery
Right hepatic artery
Coeliac axis
Left hepatic artery
The portal vein transports 70% of the blood supply to the liver, while the hepatic artery provides 30%. The portal vein contains the products of digestion. The arterial and venous blood is dispersed by sinusoids to the central veins of the liver lobules; these drain into the hepatic veins and then into the IVC. The caudate lobe drains directly into the IVC rather than into other hepatic veins.
A 55 year old man presents with symptoms of dyspepsia and on upper GI endoscopy an area of patchy erythematous tissue is identified protruding proximally from the gastro oesophageal junction. A biopsy is diagnostic of Barretts oesophagus with low grade dysplasia. Which of the following is the most appropriate management?
Distal oesophagectomy
Upper GI endoscopy with quadrantic biopsies from the region
Photodynamic therapy
Endoscopic sub mucosal resection of the area
Argon plasma coagulation
In Barrett’s surveillance the safest option is quadrantic (i.e. 4 biopsies, one from each quarter of the oesophagus at 2cm intervals)
Low grade dysplasia in conjunction with Barretts oesphagus should be monitored with regular (6 monthly) upper GI endoscopy and quadrantic biopsies. If the disease remains static at 2 years then the screening frequency may be decreased.
Which of the following would be the optimal fluid management option for a 45 year old man due to undergo an elective right hemicolectomy?
Remain “nil by mouth” for at least 6 hours pre-operatively and avoid intra venous fluids
Remain “nil by mouth” for at least 6 hours pre-operatively and receive supplementary intravenous 5% dextrose to replace lost calories
Allow him free access to oral fluids only until 30 minutes prior to surgery
Administer a carbohydrate based loading drink 3 hours pre operatively, and avoid intravenous fluids
Administer a carbohydrate based loading drink 6 hours pre-operatively and administer 5% dextrose saline thereafter
Administer a carbohydrate based loading drink 3 hours pre operatively, and avoid intravenous fluids
Patients for elective surgery should not have solids for 6 hours pre-operatively. However, clear fluids may be given up to 2 hours pre-operatively. Enhanced recovery programmes are now the standard of care in many countries around the world and involve administration of carbohydrate loading drinks.
The routine administration of 5% dextrose in the scenarios given above would convey little in the way of benefit and increase the risks of electrolyte derangement post operatively
A 23 year old man is undergoing an inguinal hernia repair. The surgeons mobilise the spermatic cord and place it in a hernia ring. A small slender nerve is identified superior to the cord. Which nerve is it most likely to be?
Iliohypogastric nerve
Pudendal nerve
Femoral branch of the genitofemoral nerve
Ilioinguinal nerve
Obturator nerve
The ilioinguinal nerve passes through the inguinal canal and is the nerve most commonly identified during hernia surgery. The genitofemoral nerve splits into two branches, the genital branch passes through the inguinal canal within the cord structures. The femoral branch of the genitofemoral nerve enters the thigh posterior to the inguinal ligament, lateral to the femoral artery. The iliohypogastric nerve pierces the external oblique aponeurosis above the superficial inguinal ring.
Which of the following physiological changes do not occur following tracheostomy?
Alveolar ventilation is increased.
Anatomical dead space is reduced by 50%.
Work of breathing is increased.
Proportion of ciliated epithelial cells in the trachea may decrease.
Splinting of the larynx may lead to swallowing difficulties.
Work of breathing is decreased which is one reasons it is popular option for weaning ventilated patients. Humidified air in this setting helps to reduce the viscosity of mucous that forms.
Where does the spinal cord terminate in neonates?
L1
L2
L3
L4
L5
At the 3rd month the foetus’s spinal cord occupies the entire length of the vertebral canal. The vertebral column then grows longer exceeding the growth rate of the spinal cord. This results with the cord being at L3 at birth and L1-2 by adulthood.
A 45 year old man is undergoing a low anterior resection for a carcinoma of the rectum. Which of the following fascial structures will need to be divided to mobilise the mesorectum from the sacrum and coccyx?
Denonvilliers fascia
Colles fascia
Sibsons fascia
Waldeyers fascia
None of the above
Fascial layers surrounding the rectum:
Anteriorly lies the fascia of Denonvilliers
Posteriorly lies Waldeyers fascia
Waldeyers fascia separates the mesorectum from the sacrum and will need to be divided.
A 68 year old man with type 2 diabetes is admitted to hospital unwell. On examination he has features of septic shock and right upper quadrant tenderness. He is not jaundiced. Imaging shows a normal calibre bile duct and no stones in the gallbladder.
A.Uncomplicated biliary colic
B.Acute cholecystitis
C.Cholangitis
D.Gallbladder abscess
E.Acalculous cholecystitis
Acalculous cholecystitis
Acalculous cholecystitis is more common in patients with an underlying co-morbidity. The morbidity and mortality following intervention are higher than in conventional gallstone disease.
A 43 year old lady with known gallstones is admitted with a high fever and jaundice. On examination, she looks extremely unwell. Her abdomen is generally soft although there is some mild tenderness in the right upper quadrant.
A.Uncomplicated biliary colic
B.Acute cholecystitis
C.Cholangitis
D.Gallbladder abscess
E.Acalculous cholecystitis
Cholangitis
Features of jaundice, fever and systemic sepsis are typical of cholangitis.
A 34 year old lady is admitted with a 3 day history of colicky right upper quadrant pain which radiates to her back. The pain is now more constant. On examination she is not jaundiced, but has a temperature of 38.5oC. She has localised peritonism in the right upper quadrant.
A.Uncomplicated biliary colic
B.Acute cholecystitis
C.Cholangitis
D.Gallbladder abscess
E.Acalculous cholecystitis
F.Pancreatitis
Acute cholecystitis
The features of pain and fever with right upper quadrant pain are suggestive of acute cholecystitis. The short nature of the history makes an abscess less likely.
A 10 year old child has a grommet inserted for a glue ear. What type of epithelium is present on the external aspect of the tympanic membrane?
Stratified squamous
Ciliated columnar
Non ciliated columnar
Non stratified squamous
None of the above
The external aspect of the tympanic membrane is lined by stratified squamous epithelium. This is significant clinically in the development of middle ear infections when this type of epithelium may migrate inside the middle ear.
A 73 year old lady is admitted with acute mesenteric ischaemia. A CT angiogram is performed and a stenotic lesion is noted at the origin of the superior mesenteric artery. At which of the following levels does this branch from the aorta?
L1
L2
L3
L4
L5
The SMA leaves the aorta at L1. It passes under the neck of the pancreas prior to giving its first branch the inferior pancreatico-duodenal artery.
A 42 year old man from Southern India presents with chronic swelling of both lower legs, they are brawny and indurated with marked skin trophic changes. Which of the following organisms is the most likely origin of this disease process?
Loa loa
Wuchereria bancrofti
Trypanosoma cruzi
Trypanosoma gambiense
None of the above
W. Bancrofti is the commonest cause of filariasis leading to lymphatic obstruction. Infection with Loa loa typically occurs in the African sub continent and usually results in generalised sub cutaneous infections without lymphatic obstruction. Trypanosomal infections would not produce this clinical picture.
The following statements relating to the musculocutaneous nerve are true except?
It arises from the lateral cord of the brachial plexus
It provides cutaneous innervation to the lateral side of the forearm
If damaged, then extension of the elbow joint will be impaired
It supplies the biceps muscle
It runs beneath biceps
It supplies biceps, brachialis and coracobrachialis. If damaged then elbow flexion rather than extension will be impaired.
Which of the following structures does not pass through the foramen ovale?
Lesser petrosal nerve
Accessory meningeal artery
Maxillary nerve
Emissary veins
Otic ganglion
Mnemonic: OVALE
O tic ganglion
V3 (Mandibular nerve:3rd branch of trigeminal)
A ccessory meningeal artery
L esser petrosal nerve
E missary veins
Which of the following is not utilised as a descriptive statistic?
Mean
Median
Mode
Z score
Standard deviation
The z score is determined using the normal distribution and is not a descriptive statistic
Which of the cranial nerves listed below is least likely to carry parasympathetic fibres?
III
VII
IX
X
II
Cranial nerves carrying parasympathetic fibres
X IX VII III (1973)
The parasympathetic functions served by the cranial nerves include:
III (oculomotor)Pupillary constriction and accommodation
VII (facial)Lacrimal gland, submandibular and sublingual glands
IX (glossopharyngeal)Parotid
X (vagus)Heart and abdominal viscera
The optic nerve carries no parasympathetic fibres.
The cranial preganglionic parasympathetic nerves arise from specific nuclei in the CNS. These synapse at one of four parasympathetic ganglia; otic, pterygopalatine, ciliary and submandibular. From these ganglia the parasympathetic nerves complete their journey to their target tissues via CN V (trigeminal) branches (ophthalmic nerve CNV branch 1, Maxillary nerve CN V branch2, mandibular nerve CN V branch 3)
A 72 year old man is undergoing an open abdominal aortic aneurysm repair. The aneurysm is located in a juxtarenal location and surgical access to the neck of aneurysm is difficult. Which of the following structures may be divided to improve access?
Cisterna chyli
Transverse colon
Left renal vein
Superior mesenteric artery
Coeliac axis
The left renal vein will be stretched over the neck of the anuerysm in this location and is not infrequently divided. This adds to the nephrotoxic insult of juxtarenal aortic surgery as a supra renal clamp is also often applied. Deliberate division of the Cisterna Chyli will not improve access and will result in a chyle leak. Division of the transverse colon will not help at all and would result in a high risk of graft infection. Division of the SMA is pointless for a juxtarenal procedure.
Please select the most appropriate management for the injury type described. Each option may be used once, more than once or not at all.
A 20 year old woman trips over a step, injuring her ankle. Examination reveals tenderness over the lateral malleolus and an x-ray demonstrates an undisplaced fracture distal to the syndesmosis.
A.Surgical fixation
B.Below knee amputation
C.Application of below knee plaster
D.Application of ankle boot
E.Application of external fixation device
F.Application of compression dressing and physiotherapy
G.Immediate reduction and application of backslab
Application of ankle boot
This is a Weber A fracture. It is a stable ankle injury and can therefore be managed conservatively. Whilst this patient could also be treated in a below knee plaster, most clinicians would nowadays treat this injury in an ankle boot. Patients should be advised to mobilise in the ankle boot, as pain allows, and can wean themselves out of the boot as the symptoms improve.
A 30 year old man injures his ankle playing football. On examination he has tenderness over both medial and lateral malleoli. X-ray demonstrates a bimalleolar fracture with a displaced distal fibula fracture, at the level of the syndesmosis and fracture of the medial malleolus with talar shift. The ankle has been provisionally reduced and splinted in the emergency department.
A.Surgical fixation
B.Below knee amputation
C.Application of below knee plaster
D.Application of ankle boot
E.Application of external fixation device
F.Application of compression dressing and physiotherapy
G.Immediate reduction and application of backslab
The correct answer is Surgical fixation
This is an unstable fracture pattern with a Weber B fracture of the distal fibula and a fracture of the medial malleolus. Talar shift indicates loss of ankle mortice congruity. This injury should therefore be treated with surgical fixation.
A 50 year old female slips on wet floor injuring her ankle. On examination, she has tenderness over the lateral and medial malleolus. X-rays demonstrate an undisplaced fracture of the distal fiibula at the level of the syndesmosis and a congruent ankle mortice.
A.Surgical fixation
B.Below knee amputation
C.Application of below knee plaster
D.Application of ankle boot
E.Application of external fixation device
F.Application of compression dressing and physiotherapy
G.Immediate reduction and application of backslab
Application of below knee plaster
This is a Weber B fracture and therefore potentially unstable. Medial malleolar tenderness indicates deltoid ligament injury. As the fracture is currently undisplaced and the ankle mortice is congruent, the injury can be initially managed conservatively in a below knee plaster but the patient should be monitored in the outpatient clinic for fracture displacement in the first few weeks.
An occlusion of the anterior cerebral artery may compromise the blood supply to the following structures except:
Medial inferior surface of the frontal lobe
Corpus callosum
Medial surface of the frontal lobe
Olfactory bulb
Brocas area
Brocas area is usually supplied by branches from the middle cerebral artery.
A 32 year old Indian lady presents with a diffuse swelling of the left breast. She has a 4 month old child. Clinically, she has jaundice and there is erythema of the left breast.
A.Ductal carcinoma in situ
B.Lobular carcinoma in situ
C.Invasive ductal carcinoma
D.Invasive lobular carcinoma
E.Inflammatory carcinoma
Inflammatory carcinoma
Inflammatory breast cancers have an aggressive nature. Dissemination occurs early and is more resistant to adjuvent treatments than other types of breast cancer. Often occurs in pregnancy or lactation.
A 72 year old female presents with a painless breast lump. Clinically she has a 4cm diameter irregular breast mass, with no other palpable masses.
A.Ductal carcinoma in situ
B.Lobular carcinoma in situ
C.Invasive ductal carcinoma
D.Invasive lobular carcinoma
E.Inflammatory carcinoma
F.Phyllodes tumour
The correct answer is Invasive ductal carcinoma
A post menopausal woman is more likely to have a ductal carcinoma and they tend to occur at a single focus within the breast.
A 72 year old woman presents with 2 breast lumps. She has a history of breast cancer in the opposite breast 5 years ago.
A.Ductal carcinoma in situ
B.Lobular carcinoma in situ
C.Invasive ductal carcinoma
D.Invasive lobular carcinoma
E.Inflammatory carcinoma
F.Phyllodes tumour
G.Paget’s disease of the nipple
The correct answer is Invasive lobular carcinoma
This is likely to be an invasive lobular carcinoma, mainly due to the multifocal lesions and the history of previous breast cancer in the opposite breast.
Parasympathetic fibres innervating the parotid gland originate from which of the following?
Submandibular ganglion
Otic ganglion
Ciliary ganglion
Pterygopalatine ganglion
None of the above
Secretion of saliva by the parotid gland is controlled by nerve fibres originating in the inferior salivatory nucleus; these leave the brain via the tympanic nerve (branch of glossopharyngeal nerve (CN IX), travel through the tympanic plexus (located in the middle ear), and then form the lesser petrosal nerve until reaching the otic ganglion. After synapsing in the Otic ganglion, the postganglionic (postsynaptic) fibres travel as part of the auriculotemporal nerve (a branch of the mandibular nerve (V3) to reach the parotid gland.
Which of the following structures suspends the spinal cord in the dural sheath?
Filum terminale
Conus medullaris
Ligamentum flavum
Denticulate ligaments
Anterior longitudinal ligament
The spinal cord is approximately 45cm in men and 43cm in women. The denticulate ligament is a continuation of the pia mater (innermost covering of the spinal cord) which has intermittent lateral projections attaching the spinal cord to the dura mater.
Where is the ‘safe triangle’ for chest drain insertion located?
4th intercostal space, mid axillary line
5th intercostal space, mid axillary line
4th intercostal space, mid scapular line
5th intercostal space, mid scapular line
4th intercostal space, mid clavicular line
‘Safe Triangle’ for chest drain insertion:
5th intercostal space, mid axillary line
A 73 year old man develops disseminated intravascular coagulation following an abdominal aortic aneurysm repair. He receives an infusion of cryoprecipitate. What is the major constituent of this infusion?
Factor VIII
Factor IX
Protein C
Protein S
Factor V
Factor 8
A 49 -year-old male presents with discomfort in the fingers of his left hand. On examination, the ring and little fingers of his left hand are flexed and unable to extend completely. He is able to make a fist with the hand. Palpation reveals thickened nodules on the medial half of the palm.
A.de Quervain’s tenosynovitis
B.Dupuytren’s contracture
C.Bouchard’s nodes
D.Ganglion
E.Carpal tunnel syndrome
F.Radial nerve injury
G.Ulnar nerve injury
Dupuytren’s contracture
Discomfort of the hand is not uncommon in Dupuytrens contracture, true pain is unusual. The disease most commonly affects the ring and little fingers.
A 62 year old man presents after his wife commented on the unusual shape of his fingers. On examination, he has a hard swelling adjacent to the distal interphalangeal joint of his index finger of the right hand with lateral deviation of the finger tip. There is no sensory disturbance and the swelling is not tender.
A.de Quervain’s tenosynovitis
B.Dupuytren’s contracture
C.Bouchard’s nodes
D.Ganglion
E.Carpal tunnel syndrome
F.Radial nerve injury
G.Ulnar nerve injury
H.Heberden’s nodes
Heberden’s nodes
These are bony outgrowths that occur in the distal interphalangeal joint in association with osteoarthritis. They may skew the finger tip sideways. Bouchards nodes are similar, but affect the proximal interphalangeal joint.
A 57 year - old lady presents with a three month history of pins and needles in the fingers of the right hand, particularly at night. On examination, there is some loss of the sensation over the palmar aspect of the lateral three fingers and wasting of the thenar eminence.
A.de Quervain’s tenosynovitis
B.Dupuytren’s contracture
C.Bouchard’s nodes
D.Ganglion
E.Carpal tunnel syndrome
Carpal tunnel syndrome
Carpal tunnel syndrome commonly produces pain at night as the wrists are flexed during sleep. Compromise of the median nerve may produce wasting of the thenar eminence muscles.
Hard, bony outgrowths or gelatinous cysts on the proximal interphalangeal joints (the middle joints of fingers or toes.) They are a sign of osteoarthritis, and are caused by formation of calcific spurs of the articular cartilage
A.de Quervain’s tenosynovitis
B.Dupuytren’s contracture
C.Bouchard’s nodes
D.Ganglion
E.Carpal tunnel syndrome
F.Radial nerve injury
G.Ulnar nerve injury
H.Heberden’s nodes
I.Tendon sheath infection
Bouchards nodes
Typically develop in middle age, beginning either with a chronic swelling of the affected joints or the sudden painful onset of redness, numbness, and loss of manual dexterity. This initial inflammation and pain eventually subsides, and the patient is left with a permanent bony outgrowth that often skews the fingertip sideways. It typically affects the DIP joint.
A.de Quervain’s tenosynovitis
B.Dupuytren’s contracture
C.Bouchard’s nodes
D.Ganglion
E.Carpal tunnel syndrome
F.Radial nerve injury
G.Ulnar nerve injury
H.Heberden’s nodes
I.Tendon sheath infection
Based on the description provided, the condition being described is:
H. Heberden’s nodes
Let’s break down the key features that point to this diagnosis:
- Age of onset: The description mentions “typically develop in middle age,” which is consistent with Heberden’s nodes.
- Joint affected: The condition “typically affects the DIP joint” (Distal Interphalangeal joint). This is the defining characteristic of Heberden’s nodes.
- Presentation:
- Can begin with “chronic swelling of the affected joints” or “sudden painful onset of redness, numbness, and loss of manual dexterity.”
- The initial inflammation and pain eventually subsides.
- Results in “permanent bony outgrowth that often skews the fingertip sideways.”
- Nature of the condition: Heberden’s nodes are bony enlargements of the DIP joints, which matches the description of “permanent bony outgrowth.”
Heberden’s nodes are a sign of osteoarthritis in the hands. They are named after William Heberden, an English physician who first described them in 1802. These nodes represent osteophytes (bone spurs) that form as a result of the degenerative process in osteoarthritis.
It’s worth noting that while similar, this condition is different from:
- Bouchard’s nodes (option C), which affect the proximal interphalangeal (PIP) joints rather than the DIP joints.
- The other options listed do not match the specific description provided, particularly the involvement of the DIP joint and the characteristic bony outgrowth.
Which of the following is not characteristic of a granuloma?
Altered macrophages
Fused macrophages
Epithelioid cells
Mixture of chronic inflammatory cells
Polymorphnuclear leucocytes, cellular debris and fibrin
Polymorphnuclear leucocytes, cellular debris and fibrin
These are typical components of an abscess cavity. Polymorphonuclear leucocytes may be found in a granuloma if there is a focus of suppuration.
A 42 year old man presents with a painless lump in the left testicle that he noticed on self examination. Clinically there is a firm nodule in the left testicle, ultrasound appearances show an irregular mass lesion. His serum AFP and HCG levels are both within normal limits. What is the most likely diagnosis?
Yolk sack tumour
Seminoma
Testicular teratoma
Epididymo-orchitis
Adenomatoid tumour
Seminomas typically have normal AFP and HCG. These are usually raised in teratomas and yolk sac tumours
This man’s age, presenting symptoms and normal tumour markers make a seminoma the most likely diagnosis. Epididymo-orchitis does not produce irregular mass lesions which are painless.
Testicular disorders
Testicular cancer
Testicular cancer is the most common malignancy in men aged 20-30 years. Around 95% of cases of testicular cancer are germ-cell tumours. Germ cell tumours may essentially be divided into:
Tumour type Key features Tumour markers Pathology
Seminoma
Commonest subtype (50%)
Average age at diagnosis = 40
Even advanced disease associated with 5 year survival of 73%
AFP usually normal
HCG elevated in 10% seminomas
Lactate dehydrogenase; elevated in 10-20% seminomas (but also in many other conditions)
Sheet like lobular patterns of cells with substantial fibrous component. Fibrous septa contain lymphocytic inclusions and granulomas may be seen.
Non seminomatous germ cell tumours (42%)
Teratoma
Yolk sac tumour
Choriocarcinoma
Mixed germ cell tumours (10%)
Younger age at presentation =20-30 years
Advanced disease carries worse prognosis (48% at 5 years)
Retroperitoneal lymph node dissection may be needed for residual disease after chemotherapy
AFP elevated in up to 70% of cases
HCG elevated in up to 40% of cases
Other markers rarely helpful
Risk factors for testicular cancer
Cryptorchidism
Infertility
Family history
Klinefelter’s syndrome
Mumps orchitis
Features
A painless lump is the most common presenting symptom
Pain may also be present in a minority of men
Other possible features include hydrocele, gynaecomastia
Diagnosis
Ultrasound is first-line
CT scanning of the chest/ abdomen and pelvis is used for staging
Tumour markers (see above) should be measured
Management
Orchidectomy (Inguinal approach)
Chemotherapy and radiotherapy may be given depending on staging
Abdominal lesions >1cm following chemotherapy may require retroperitoneal lymph node dissection.
Prognosis is generally excellent
5 year survival for seminomas is around 95% if Stage I
5 year survival for teratomas is around 85% if Stage I
Benign disease
Epididymo-orchitis
Acute epididymitis is an acute inflammation of the epididymis, often involving the testis and usually caused by bacterial infection.
Infection spreads from the urethra or bladder. In men <35 years, gonorrhoea or chlamydia are the usual infections.
Amiodarone is a recognised non infective cause of epididymitis, which resolves on stopping the drug.
Tenderness is usually confined to the epididymis, which may facilitate differentiating it from torsion where pain usually affects the entire testis.
Testicular torsion
Twist of the spermatic cord resulting in testicular ischaemia and necrosis.
Most common in males aged between 10 and 30 (peak incidence 13-15 years)
Pain is usually severe and of sudden onset.
Cremasteric reflex is lost and elevation of the testis does not ease the pain.
Treatment is with surgical exploration. If a torted testis is identified then both testis should be fixed as the condition of bell clapper testis is often bilateral.
Hydrocele
Presents as a mass that transilluminates, usually possible to ‘get above’ it on examination.
In younger men it should be investigated with USS to exclude tumour.
In children it may occur as a result of a patent processus vaginalis.
Treatment in adults is with a Lords or Jabouley procedure.
Treatment in children is with trans inguinal ligation of PPV.
A 63 year old lady presents with an obstructing cancer of the sigmoid colon. She is not peritonitic and her imaging demonstrates a solitary liver metastasis.
A.Ileocolic bypass
B.Loop ileostomy
C.High anterior resection
D.Insertion of self expanding metallic stent
E.Left hemicolectomy and on table colonic lavage and primary anastomosis
F.Extended right hemicolectomy and ileocolic anastomosis
The correct answer is Insertion of self expanding metallic stent
Ideally, the distant disease should be managed first and then the primary lesion addressed. A self expanding stent is likely to achieve this and avoids a stoma.
A 65 year old man presents with absolute constipation and abdominal pain. On examination he has marked abdominal distension. A digital rectal examination reveals an empty rectum. A rectal contrast study shows an obstructing lesion of the proximal rectum.
A.Ileocolic bypass
B.Loop ileostomy
C.High anterior resection
D.Insertion of self expanding metallic stent
E.Left hemicolectomy and on table colonic lavage and primary anastomosis
F.Extended right hemicolectomy and ileocolic anastomosis
G.Low anterior resection
H.Loop colostomy of the transverse colon
I.Loop colostomy of the sigmoid colon
J.Right hemicolectomy
The correct answer is Loop colostomy of the sigmoid colon
Rectal cancers should not be primarily resected prior to definitive staging and a tumour of this nature is likely to have circumferential margin involvement. Whilst a sigmoid and transverse loop colostomy would both provide an equal relief of obstruction the former procedure has the added benefit of making a subsequent resection safer, since a transverse colostomy would have to be taken down and closed during the course of subsequent surgery.
A 70 year old lady presents with a two day history of constipation and vomiting. On examination she has right iliac fossa tenderness and little abdominal distension. A CT scan is performed and is suggestive of an obstructing carcinoma of the colonic hepatic flexure (stage T3).
A.Ileocolic bypass
B.Loop ileostomy
C.High anterior resection
D.Insertion of self expanding metallic stent
E.Left hemicolectomy and on table colonic lavage and primary anastomosis
F.Extended right hemicolectomy and ileocolic anastomosis
G.Low anterior resection
H.Loop colostomy of the transverse colon
I.Loop colostomy of the sigmoid colon
J.Right hemicolectomy
The correct answer is Right hemicolectomy
This lesion should be amenable to standard right hemicolectomy. Extending the resection to take the middle colic vessels and distal transverse colon is unlikely to provide additional oncological benefit.
A baby is born by normal vaginal delivery at 39 weeks gestation. Initially all appears well and then the clinical staff become concerned because the baby develops recurrent episodes of cyanosis. These are worse during feeding and improve dramatically when the baby cries. The most likely underlying diagnosis is:
Choanal atresia
Oesophageal reflux
Tetralogy of Fallot
Oesophageal atresia
Congenital diaphragmatic hernia
In Choanal atresia the episodes of cyanosis are usually worst during feeding. Improvement may be seen when the baby cries as the oropharyngeal airway is used.
A 32 year old man presents to the acute surgical unit with acute pancreatitis. Over the next few days he becomes dyspnoeic and his saturations are 89% on air. A CXR shows bilateral pulmonary infiltrates. His CVP pressure is 16mmHg. What is the most likely diagnosis?
Cardiac failure
Pneumococcal pneumonia
Staphylococcal pneumonia
Pneumocystis carinii
Adult respiratory distress syndrome
Acute pancreatitis is known to precipitate ARDS. ARDS is characterised by bilateral pulmonary infiltrates and hypoxaemia. Note that pulmonary oedema is excluded by the CVP reading < 18mmHg.
A 28 year old lady presents with a pigmented lesion on her calf. Excisional biopsy confirms a diagnosis of melanoma measuring 1cm in diameter with a Breslow thickness of 0.1mm. The lesion is less than 1 mm at all resection margins. Which of the following surgical resection margins is acceptable for this lesion?
5 cm
1 cm
0.5 cm
2 cm
3 cm A 28 year old lady presents with a pigmented lesion on her calf. Excisional biopsy confirms a diagnosis of melanoma measuring 1cm in diameter with a Breslow thickness of 0.1mm. The lesion is less than 1 mm at all resection margins. Which of the following surgical resection margins is acceptable for this lesion?
5 cm
1 cm
0.5 cm
2 cm
3 cm
1 cm
A 53 year old lady has undergone a bilateral breast augmentation procedure many years previously. The implants are tense and uncomfortable and are removed. During their removal the surgeon encounters a dense membrane surrounding the implants, it has a coarse granular appearance. The tissue is sent for histology and it demonstrates fibrosis with the presence of calcification. The underlying process responsible for these changes is:
Hyperplasia
Dysplasia
Metastatic calcification
Dystrophic calcification
Necrosis
Breast implants often become surrounded by a pseudocapsule and this may secondarily then be subjected to a process of dystrophic calcification.
A 72 year old female is found to have a malignant lesion in her left arm. She had a mastectomy of the left breast 10 years ago and has chronic lymph oedema of the left arm. What is the most likely cause of the malignancy?
Lymphangiosarcoma
Lymphoma
Myeloma
Angiomyolipoma
Giant cell tumour
Lymphangiosarcoma is a rare condition arising as a result of chronic oedema. It is an aggressive malignancy.
A 45 year old lady undergoes a renal transplant from a living related donor. She is well for several months but on review in the outpatient department is noted to have persistent hypertension and a slight deterioration in renal function.
A.Ureteric anastomotic leak
B.Renal vein thrombosis
C.Acute rejection
D.Chronic allograft nephropathy
E.Renal artery thrombosis
F.Renal artery stenosis
G.Lymphocele
H.Hyperacute rejection
Renal artery stenosis
Renal artery stenosis typically occurs over several months and will usually result in the development of hypertension. Most cases can be assessed using duplex scanning and managed with angioplasty.
Complications of renal transplantation
A number of complications may occur following renal transplantation. A critical aspect of post operative care is evaluation of graft function. Post operatively, urine output is the most readily available, and easily measured, indicator of graft function. If an individual was relatively anuric pre-transplant and has a good urine output following surgery then this is more useful than it would be in someone who had a higher volume diuresis prior to transplantation. Recipients can be divided into three main groups following renal transplantation, with regard to their graft function:
Immediate graft function; brisk diuresis and falling serum creatinine
Slow graft function; modest urine output and slowly falling creatinine levels
Delayed graft function; defined as need for dialysis post transplant
Decreased urine output following surgery can be the result of hypovolvaemia or a blocked catheter (commonest causes). Other important causes include rejection, or a vascular complication.
Vascular complications
These may involve the donor vessels, those of the recipient or both. Renal artery thrombosis usually occurs early post transplant, but is uncommon with an incidence of less than 1%. It typically results in graft loss. It usually occurs as a result of a technical problem such a vessel torsion or sub intimal flaps. The usual presenting feature is a sudden cessation of urine output. When suspected, the occlusion is usually well demonstrated with duplex scanning. Ideally immediate surgical re-exploration should occur. Sadly, the graft has usually been lost by this stage and will require graft nephrectomy. Renal vein thrombosis is not as common as arterial graft thrombosis and the usual presenting features include discomfort at the graft site and swelling of the graft associated with loss of urine output. Again, duplex scanning is indicated. Unfortunately, this complication is also associated with a high incidence of graft loss.
Over a longer time frame (typically months) some individuals will develop renal artery stenosis. These individuals will typically develop hypertension and over time graft function will decline as hypertensive nephropathy occurs. It is usually demonstrated by duplex scanning and is usually amenable to endovascular intervention.
Urological complications
Urinary tract complications manifesting as leakage or obstruction are common complications following renal transplantation and occur in up to 10% of patients. The main underlying cause is the relatively poor blood supply to the transplanted ureter. Patients typically present relatively early in the first 5 weeks following transplantation with pain and swelling at the graft site. Imaging with USS is often the initial test. Therapeutic options include surgical re-implantation of the ureter for large leaks and stent insertion and nephrostomy placement for smaller leaks.
Lymphocele
These do not generally occur until 2 weeks or longer after surgery. They are, however, relatively common and may be seen in up to 18% of patients. Symptoms usually occur as a result of mass effect with compression of adjacent structures. These include the vessels supplying both the graft, with deterioration in graft function, the ureter, with alteration in urine output and the recipients lower limb vessels, with development of leg swelling. Creation of a laparoscopic or open peritoneal window is a favored treatment.
Rejection
Four types of graft rejection are recognised; hyperacute, accelerated acute, acute and chronic.
Type of rejection Key features
Hyperacute Occurs within minutes of clamp release
Due to pre formed antibodies
Immediate loss of graft occurs
Accelerated acute Occurs in first few days following surgery
Involved both cellular and antibody mediated injury
Pre-sensitisation of the donor is a common cause
Acute Traditionally the most common type of rejection
Seen days to weeks after surgery
Predominantly a cell mediated process mediated by lymphocytes
Organ biopsy demonstrates cellular infiltrates and graft cell apoptosis
Chronic Increasingly common problem
Typically; graft atrophy and atherosclerosis are seen. Fibrosis often occurs as a late event
A 39 year old lady undergoes a live related renal transplant. She progresses well. Two weeks following the transplant she is noted to have swelling overlying the transplant site and swelling of the ipsilateral limb.Urine output is acceptable and creatinine unchanged.
A.Ureteric anastomotic leak
B.Renal vein thrombosis
C.Acute rejection
D.Chronic allograft nephropathy
E.Renal artery thrombosis
F.Renal artery stenosis
G.Lymphocele
H.Hyperacute rejection
Lymphocele
Swelling over the graft site is often due to a lymphocele and this is further suggested by the normal renal function. They cause symptoms through mass effect and limb swelling may occur. Treatment is often surgical.
Your consultant decides to perform an open inguinal hernia repair under local anaesthesia. Which of the following dermatomal levels will require blockade?
T10
T12
T11
S1
S2
T12
If a sample is normally distributed which of the following is true?
Mean = standard deviation
Mean = standard error of the mean
Mean = median
Mean = variance
The mode and standard error of the mean have the same value
In a normally distributed sample, the mean, median and mode are the same.
A syndrome consisting of a PTEN mutation and intestinal hamartomas.
A.Peutz-Jeghers syndrome
B.Cowden disease
C.Familial adenomatous polyposis coli
D.Lynch syndrome
E.MYH associated polyposis
Cowden disease
PTEN is a tumour supressor gene and loss of function mutations result in up regulation of the mTOR pathway.
A syndrome which may be present in a patient with multiple intestinal hamartomas and pigmentation spots around the mouth.
A.Peutz-Jeghers syndrome
B.Cowden disease
C.Familial adenomatous polyposis coli
D.Lynch syndrome
E.MYH associated polyposis
Peutz-Jeghers syndrome
A syndrome likely to be present in a 28 year old man who presents with a locally advanced mucinous carcinoma of the caecum. There are scanty polyps in the remaining colon. His father died from colorectal cancer aged 34.
A.Peutz-Jeghers syndrome
B.Cowden disease
C.Familial adenomatous polyposis coli
D.Lynch syndrome
E.MYH associated polyposis
The correct answer is Lynch syndrome
Lynch syndrome is likely when right sided colonic cancers occur at a young age. These tumours are often poorly differentiated and mucinous. The Amsterdam criteria can be used to identify families at risk who may benefit from genetic testing
A 48 year old lady has previously undergone a sigmoid colectomy for carcinoma. On follow up imaging she is found to have a 3cm foci of metastatic disease in segment IV of the liver. What is the most appropriate course of action?
Palliative chemotherapy
External beam radiotherapy
Brachytherapy
Surgical resection alone
Chemotherapy followed by surgical resection
The treatment of colorectal liver metastasis is usually with chemotherapy followed by surgical resection. Where surgery is performed for liver metastasis with curative intent, the 5 year survival is 20%. Palliation would generally only be considered if the patient were frail or widespread disease found on imaging. Radiotherapy is not part of the treatment of liver metastasis.
Colorectal cancer treatment
Patients diagnosed as having colorectal cancer should be completely staged using CT of the chest/ abdomen and pelvis. Their entire colon should have been evaluated with colonoscopy or CT colonography. Patients whose tumours lie below the peritoneal reflection should have their mesorectum evaluated with MRI.
Once their staging is complete patients should be discussed within a dedicated colorectal MDT meeting and a treatment plan formulated.
Treatment of colonic cancer
Cancer of the colon is nearly always treated with surgery. Stents, surgical bypass and diversion stomas may all be used as palliative adjuncts. Resectional surgery is the only option for cure in patients with colon cancer. The procedure is tailored to the patient and the tumour location. The lymphatic drainage of the colon follows the arterial supply and therefore most resections are tailored around the resection of particular lymphatic chains (e.g. ileo-colic pedicle for right sided tumours). Some patients may have confounding factors that will govern the choice of procedure, for example a tumour in a patient from a HNPCC family may be better served with a panproctocolectomy rather than segmental resection. Following resection the decision has to be made regarding restoration of continuity. For an anastomosis to heal the key technical factors include; adequate blood supply, mucosal apposition and no tissue tension. Surrounding sepsis, unstable patients and inexperienced surgeons may compromise these key principles and in such circumstances it may be safer to construct an end stoma rather than attempting an anastomosis.
When a colonic cancer presents with an obstructing lesion; the options are to either stent it or resect. In modern practice it is unusual to simply defunction a colonic tumour with a proximal loop stoma. This differs from the situation in the rectum (see below).
Following resection patients with risk factors for disease recurrence are usually offered chemotherapy, a combination of 5FU and oxaliplatin is common.
Rectal cancer
The management of rectal cancer is slightly different to that of colonic cancer. This reflects the rectum’s anatomical location and the challenges posed as a result. Tumours located in the rectum can be surgically resected with either an anterior resection or an abdomino - perineal resection. The technical aspects governing the choice between these two procedures can be complex to appreciate and the main point to appreciate for the MRCS is that involvement of the sphincter complex or very low tumours require APER. In the rectum a 2cm distal clearance margin is required and this may also impact on the procedure chosen. In addition to excision of the rectal tube an integral part of the procedure is a meticulous dissection of the mesorectal fat and lymph nodes (total mesorectal excision/ TME). In rectal cancer surgery invovlement of the cirumferential resection margin carries a high risk of disease recurrence. Because the rectum is an extraperitoneal structure (until you remove it that is!) it is possible to irradiate it, something which cannot be offered for colonic tumours. This has a major impact in rectal cancer treatment and many patients will be offered neoadjuvent radiotherapy (both long and short course) prior to resectional surgery. Patients with T1, 2 and 3 /N0 disease on imaging do not require irradiation and should proceed straight to surgery. Patients with T4 disease will typically have long course chemo radiotherapy. Patients presenting with large bowel obstruction from rectal cancer should not undergo resectional surgery without staging as primary treatment (very different from colonic cancer). This is because rectal surgery is more technically demanding, the anastomotic leak rate is higher and the danger of a positive resection margin in an unstaged patient is high. Therefore patients with obstructing rectal cancer should have a defunctioning loop colostomy.
Summary of procedures
The operations for cancer are segmental resections based on blood supply and lymphatic drainage. These commonly performed procedures are core knowledge for the MRCS and should be understood.
Site of cancer Type of resection Anastomosis Risk of leak
Right colon Right hemicolectomy Ileo-colic Low <5%
Transverse Extended right hemicolectomy Ileo-colic Low <5%
Splenic flexure Extended right hemicolectomy Ileo-colic Low <5%
Splenic flexure Left hemicolectomy Colo-colon 2-5%
Left colon Left hemicolectomy Colo-colon 2-5%
Sigmoid colon High anterior resection Colo-rectal 5%
Upper rectum Anterior resection (TME) Colo-rectal 5%
Low rectum Anterior resection (Low TME) Colo-rectal
(+/- Defunctioning stoma) 10%
Anal verge Abdomino-perineal excision of colon and rectum None n/a
In the emergency setting, where the bowel has perforated, the risk of an anastomotic breakdown is much greater, particularly when the anastomosis is colon-colon. In this situation, an end colostomy is often safer and can be reversed later. When resection of the sigmoid colon is performed and an end colostomy is fashioned the operation is referred to as a Hartmans procedure. Whilst left sided resections are more risky, ileo-colic anastomoses are relatively safe even in the emergency setting and do not need to be defunctioned.
References
A review of the diagnosis and management of colorectal cancer and a summary of the UK National Institute of Clinical Excellence guidelines is provided in:
NICE Guideline NG151. Last updated in 2020.
Which of the following are not characteristic features of central chemoreceptors in the control of ventilation?
They are located in the medulla oblongata
They are stimulated primarily by venous hypercapnia
They are relatively insensitive to hypoxia
They may be affected by changes in the pH of CSF
During acute hypercapnia the carotid receptors will be stimulated first
They are stimulated by arterial carbon dioxide. They take longer to equilibrate than the peripheral chemoreceptors located in the carotid. They are less exposed to acidity due to the blood brain barrier.
A 48 year old lady with end stage renal failure receives a cadaveric renal transplant. The organ is ABO group matched only. On completion of the vascular anastomoses the surgeons remove the clamps. Over the course of the next twelve minutes the donated kidney becomes dusky and swollen and appears non viable. Which of the following is the most likely process that has caused this event?
IgG anti HLA Class I antibodies in the recipient
IgM anti HLA Class I antibodies in the recipient
IgG anti HLA Class I antibodies from the donor
IgM anti HLA Class I antibodies from the donor
IgM anti HLA Class II antibodies from the recipient
Episodes of hyperacute rejection are typically due to preformed antibodies. ABO mismatch is the best example. However, IgG anti HLA Class I antibodies are another potential cause. These events are now seen less commonly because the cross matching process generally takes this possibility into account
IgG anti HLA Class I antibodies in the recipient
A 45 year old lady develops severe back pain and on examination is found to have clinical evidence of an L5/ S1 radiculopathy. Her symptoms deteriorate and eventually a laminectomy is performed. During a posterior surgical approach the surgeons encounter a tough ligamentous structure lying anterior to the spinous processes. This structure is most likely to be the
Transverse spinal ligament
Supraspinal ligament
Anterior longitudinal ligament
Ligamentum flavum
Posterior longitudinal ligament
Ligamentum flavum
A 30 year old woman presents with pain and swelling of the left shoulder. There is a large radiolucent lesion in the head of the humerus extending to the subchondral plate.
A.Osteosarcoma
B.Osteomalacia
C.Osteoporosis
D.Metastatic carcinoma
E.Osteoblastoma
F.Giant cell tumour
G.Ewing’s sarcoma
Giant cell tumour
Giant cell tumours on x-ray have a ‘soap bubble’ appearance. They present as pain or pathological fractures. They commonly metastasize to the lungs.
A 72 year old woman has a lumbar vertebral crush fracture. She has hypocalcaemia and a low urinary calcium.
A.Osteosarcoma
B.Osteomalacia
C.Osteoporosis
D.Metastatic carcinoma
E.Osteoblastoma
F.Giant cell tumour
G.Ewing’s sarcoma
Osteomalacia
Hypocalcemia and low urinary calcium are biochemical features of osteomalacia. Unfortunately surgeons do need to look at some blood results!
A 16 year old boy presents with severe groin pain after kicking a football. Imaging confirms a pelvic fracture. A previous pelvic x-ray performed 2 weeks ago shows a lytic lesion with ‘onion type’ periosteal reaction.
A.Osteosarcoma
B.Osteomalacia
C.Osteoporosis
D.Metastatic carcinoma
E.Osteoblastoma
F.Giant cell tumour
G.Ewing’s sarcoma
Ewing’s sarcoma
A Ewings sarcoma is most common in males between 10-20 years. It can occur in girls. A lytic lesion with a lamellated or onion type periosteal reaction is a classical finding on x-rays. Most patients present with metastatic disease with a 5 year prognosis between 5-10%.
The cell of origin in virtually all pancreatic carcinomas is which of the following?
The acinar cells
The islet beta cells
The islet alpha cells
The interstitial fibroblasts
The ductular epithelium
Over 90% of pancreatic carcinomas are adenocarcinomas and are thus of ductular epithelial origin.
Which of the following does not pass through the superior orbital fissure?
Lacrimal nerve
Abducens nerve
Opthalmic artery
Trochlear nerve
Superior opthalmic vein
Mnemonic for the nerves passing through the supraorbital fissure:
Live Frankly To See Absolutely No Insult
Lacrimal
Frontal
Trochlear
Superior Division of Oculomotor
Abducens
Nasociliary
Inferior Division of Oculomotor nerve
The opthalmic artery arises from the internal carotid immediately after it has pierced the dura and arachnoid. It runs through the optic canal below the optic nerve and within its dural and arachnoid sheaths. It terminates as the supratrochlear and dorsal nasal arteries
An 18 year old man undergoes a tonsillectomy for attacks of recurrent acute tonsillitis. Whilst in recovery he develops a post operative haemorrhage. Which of the following vessels is the most likely culprit?
Facial vein
External palatine vein
External carotid artery
Internal jugular vein
None of the above
The external palatine vein lies immediately lateral to the tonsil and if damaged may be a cause of reactionary haemorrhage following tonsillectomy.
A 25 year old male presents with epistaxis, the ENT SpR plans to cauterise the bleeding point with silver nitrate.
A.1% xylocaine with 1 in 200,000 adrenaline
B.1% Lignocaine
C.0.5% Bupivacaine with 1 in 200,000 adrenaline
D.0.5% Bupivicaine
E.Prilocaine 1%
F.Procaine 1%
G.Cocaine 25%
H.Cocaine 10%
The correct answer is 1% xylocaine with 1 in 200,000 adrenaline
Historically cocaine was popular for the management of epistaxis. Some surgeons will still routinely use cocaine paste for this indication. Its popularity stems from the fact that it causes vasospasm. However, systemic absorption carries the risk of adverse reactions. Where it is used the correct dose is 4%. Topically applied short acting local anaesthetic agents with adrenaline may produce similar effects, with lower risks of toxicity.
An 18 year old boy requires a Zadeks procedure
A.1% xylocaine with 1 in 200,000 adrenaline
B.1% Lignocaine
C.0.5% Bupivacaine with 1 in 200,000 adrenaline
D.0.5% Bupivicaine
E.Prilocaine 1%
F.Procaine 1%
G.Cocaine 25%
H.Cocaine 10%
The correct answer is 1% Lignocaine
This is excision of the toe nail and a fast acting local anaesthetic is indicated. Adrenaline should be avoided in this setting as it can cause digital ischaemia
A 72 year old woman fractured her distal radius. A Biers Block is planned to facilitate reduction of the fracture.
A.1% xylocaine with 1 in 200,000 adrenaline
B.1% Lignocaine
C.0.5% Bupivacaine with 1 in 200,000 adrenaline
D.0.5% Bupivicaine
E.Prilocaine 1%
F.Procaine 1%
G.Cocaine 25%
H.Cocaine 10%
Prilocaine 1%
This is the best local anaesthetic for this. Bupivacaine may cause cardiotoxicity and should be avoided.
Which of the nerves listed below is responsible for providing innervation to the lower molar teeth?
Greater palatine nerve
Nasopalatine nerve
Inferior alveolar nerve
Zygomatic nerve
Mandibular nerve
The branches of the lower molar and premolar teeth are supplied by branches of the inferior alveolar nerve. Those of the canine and incisors by the incisive branch of the same nerve. The gingiva and supporting structures are innervated by the lingual nerve.
Trigeminal nerve
The trigeminal nerve is the main sensory nerve of the head. In addition to its major sensory role, it also innervates the muscles of mastication.
Distribution of the trigeminal nerve
Sensory
Scalp
Face
Oral cavity (and teeth)
Nose and sinuses
Dura mater
Motor
Muscles of mastication
Mylohyoid
Anterior belly of digastric
Tensor tympani
Tensor palati
Autonomic connections (ganglia)
Ciliary
Sphenopalatine
Otic
Submandibular
Path
Originates at the pons
Sensory root forms the large, crescentic trigeminal ganglion within Meckel’s cave, and contains the cell bodies of incoming sensory nerve fibres. Here the 3 branches exit.
The motor root cell bodies are in the pons and the motor fibres are distributed via the mandibular nerve. The motor root is not part of the trigeminal ganglion.
Branches of the trigeminal nerve
Ophthalmic nerve Sensory only
Maxillary nerve Sensory only
Mandibular nerve Sensory and motor
Sensory
Ophthalmic Exits skull via the superior orbital fissure
Sensation of: scalp and forehead, the upper eyelid, the conjunctiva and cornea of the eye, the nose (including the tip of the nose, except alae nasi), the nasal mucosa, the frontal sinuses, and parts of the meninges (the dura and blood vessels).
Maxillary nerve Exit skull via the foramen rotundum
Sensation: lower eyelid and cheek, the nares and upper lip, the upper teeth and gums, the nasal mucosa, the palate and roof of the pharynx, the maxillary, ethmoid and sphenoid sinuses, and parts of the meninges.
Mandibular nerve Exit skull via the foramen ovale
Sensation: lower lip, the lower teeth and gums, the chin and jaw (except the angle of the jaw), parts of the external ear, and parts of the meninges.
Motor
Distributed via the mandibular nerve.
The following muscles of mastication are innervated:
Masseter
Temporalis
Medial pterygoid
Lateral pterygoid
Other muscles innervated include:
Tensor veli palatini
Mylohyoid
Anterior belly of digastric
Tensor tympani
Which of the anaesthetic agents below is most likely to induce adrenal suppression?
Sodium thiopentone
Midazolam
Propofol
Etomidate
Ketamine
Etomidate is a recognised cause of adrenal suppression, this has been associated with increased mortality when used as a sedation agent in the critically ill.
Rapid onset of anaesthesia
Pain on IV injection
Rapidly metabolised with little accumulation of metabolites
Proven anti emetic properties
Moderate myocardial depression
Widely used especially for maintaining sedation on ITU, total IV anaesthesia and for daycase surgery
Propofol
Extremely rapid onset of action making it the agent of choice for rapid sequence of induction
Marked myocardial depression may occur
Metabolites build up quickly
Unsuitable for maintenance infusion
Little analgesic effects
Sodium thiopentone
May be used for induction of anaesthesia
Has moderate to strong analgesic properties
Produces little myocardial depression making it a suitable agent for anaesthesia in those who are haemodynamically unstable
May induce state of dissociative anaesthesia resulting in nightmares
Ketamine
Has favorable cardiac safety profile with very little haemodynamic instability
No analgesic properties
Unsuitable for maintaining sedation as prolonged (and even brief) use may result in adrenal suppression
Post operative vomiting is common
Etomidate
A 22 year old female undergoes a thyroidectomy. The resected specimen shows a non encapsulated tumour with papillary projections and pale empty nuclei.
A.Follicular carcinoma
B.Anaplastic carcinoma
C.Medullary carcinoma
D.Papillary carcinoma
E.Lymphoma
F.Hashimotos thyroiditis
G.Graves disease
Papillary carcinoma
The presence of papillary structures together with the cytoplasmic features described is strongly suggestive of papillary carcinoma. They are seldom encapsulated.
32 year old lady undergoes a thyroidectomy for a mild goitre. The resected specimen shows an intense lymphocytic infiltrate with acinar destruction and fibrosis.
A.Follicular carcinoma
B.Anaplastic carcinoma
C.Medullary carcinoma
D.Papillary carcinoma
E.Lymphoma
F.Hashimotos thyroiditis
G.Graves disease
Hashimotos thyroiditis
Lymphocytic infiltrates and fibrosis are typically seen in Hashimotos thyroiditis. In Lymphoma only dense lymphatic type tissue is usually present.
A 7 year old boy develops a persistent fever following an open appendicectomy for gangrenous appendicitis. On examination he has erythema of the wound and some abdominal distension. For the following post operative scenarios please select the most appropriate investigation or management.
A.Trans anal ultrasound scan
B.Upper abdominal ultrasound scan
C.Total abdominal ultrasound scan
D.PET CT scan
E.ERCP
F.Small bowel MRI Scan
G.Abdominal CT scan with oral and IV contrast
H.Non contrast abdominal CT scan
I.Laparotomy
J.Laparoscopy
Total abdominal ultrasound scan
This patient has risk factors for a wound infection which is not in itself an indication for scanning. However, he also had abdominal distension and this, together with the history of distension would generally attract a recommendation for imaging. A USS will show an abdominal wall collection and more importantly any phrenic or pelvic collections. Unlike adult practice, CT scanning is rarely performed in children.
A 56 year old man is 8 days following a left hemicolectomy. He has developed a swinging pyrexia over the past 48 hours and has an ileus clinically.
A.Trans anal ultrasound scan
B.Upper abdominal ultrasound scan
C.Total abdominal ultrasound scan
D.PET CT scan
E.ERCP
F.Small bowel MRI Scan
G.Abdominal CT scan with oral and IV contrast
H.Non contrast abdominal CT scan
I.Laparotomy
J.Laparoscopy
Abdominal CT scan with oral and IV contrast
This would most likely be the result of an anastomotic leak with abscess formation. Detailed imaging is required to allow accurate diagnosis and planning of management.
A 43 year old lady underwent an acute cholecystectomy for cholecystitis. A drain is left during the procedure. Over the next 5 days the drain has been accumulating between 100-200ml of bile per 24 hour period.
A.Trans anal ultrasound scan
B.Upper abdominal ultrasound scan
C.Total abdominal ultrasound scan
D.PET CT scan
E.ERCP
F.Small bowel MRI Scan
G.Abdominal CT scan with oral and IV contrast
H.Non contrast abdominal CT scan
I.Laparotomy
J.Laparoscopy
ERCP
The most likely cause of a bile leak in this scenario would be a dislodged clip from the cystic duct. Whilst it may be tempting to try and plan to manage this surgically the anatomy is often unfavorable and the duct very difficult to identify. An ERCP has the advantage of demonstrating the cause of the leak and allowing placement of a stent. This will usually allow the resolution of most leaks without the need for surgery.
At which level does the aorta perforate the diaphragm?
T10
T9
T8
T11
T12
Memory aid:
T8 (8 letters) = vena cava
T10 (10 letters) = oesophagus
T12 (12 letters) = aortic hiatus
A 24 year old lady is stabbed in the buttock. Following the injury the wound is sutured in the emergency department. Eight weeks later she attends the clinic, as she walks into the clinic room she has a waddling gait and difficulty with thigh abduction. On examination she has buttock muscle wasting. Which nerve has been injured?
Superior gluteal nerve
Obturator nerve
Sciatic nerve
Femoral nerve
Inferior gluteal nerve
Damage to the superior gluteal nerve will result in a Trendelenburg gait.
Which of the following does not stimulate insulin release?
Gastrin
Atenolol
Protein
Secretin
Vagal cholinergic action
Beta blockers inhibit the release of insulin.
Stimulation of insulin release:
Glucose
Amino acid
Vagal cholinergic
Secretin/Gastrin/CCK
Fatty acids
Beta adrenergic drugs
During a radical neck dissection, division of which of the following fascial layers will expose the ansa cervicalis?
Pretracheal fascia
Carotid sheath
Prevertebral fascia
Investing layer of fascia
Sibsons fascia
The ansa cervicalis lies anterior to the carotid sheath. It may be exposed by division of the pretracheal fascia at the posterolateral aspect of the thyroid gland. The pre vertebral fascia lies more posteriorly and division of the investing layer of fascia will not expose this nerve.
Which one is true for pheochromacytoma?
10% of cases are bilateral.
10% occur in children.
11% are malignant (higher when tumour is located outside the adrenal).
10% will not be hypertensive.
All of the above
All of the above. Rule of 10s in phaeochromocytoma
An 8 year old boy falls onto an outstretched hand and is brought to the emergency department. He is examined by a doctor and a bony injury is cleared clinically. He re-presents a week later with pain in his hand. What is the most likely underlying injury?
Fracture of the distal radius
Fracture of the scaphoid
Dislocation of the lunate
Rupture of flexor pollicis longus tendon
Bennett’s fracture
Scaphoid fractures in children are rare, will usually involve the distal pole and are easily missed. The initial clinical examination (and sometimes x-rays) may be normal and repeated clinical examination and imaging is advised for this reason. Whilst the other injuries may be sustained from a fall onto an outstretched hand they are less likely to be overlooked on clinical examination. In the case of a Bennetts fracture, the injury mechanism is less compatible with this type of injury.
A 73 year old lady presents with symptoms of faecal incontinence. On examination she has weak anal sphincter muscles. What are the main nerve root values of the nerves supplying the external anal sphincter?
S2,3
L5, S1
S4,5
S5
S2,3,4
S2, 3, 4 Keeps the poo off the floor
The external anal sphincter is innervated by the inferior rectal branch of the pudendal nerve, this has root values of S2, 3 and the perineal branch of S4.
A 22 year old falls over and lands on a shard of glass. It penetrates the palmar aspect of his hand, immediately lateral to the pisiform bone. Which of the following structures is most likely to be injured?
Palmar cutaneous branch of the median nerve
Lateral tendons of flexor digitorum superficialis
Ulnar artery
Flexor carpi radialis tendons
Lateral tendons of flexor digitorum profundus
The ulnar nerve and artery are at most immediate risk in this injury. This is illustrated in the image below
A 63 year old man is admitted with severe headache, nausea and recent epileptic fit. Fundoscopy shows papilloedema. He is also noted to have diplopia.
A.Optic nerve
B.Oculomotor nerve
C.Trigeminal nerve
D.Facial nerve
E.Abducens nerve
F.Glossopharyngeal nerve
G.Vestibulocochlear nerve
H.Accessory nerve
I.Hypoglossal nerve
Abducens nerve
The long intracranial course of this nerve makes it susceptible to damage early in the course of raised ICP.
An 18 year old boy undergoes an uncomplicated tonsillectomy for recurrent attacks of tonsillitis. Post operatively he complains of otalgia.
A.Optic nerve
B.Oculomotor nerve
C.Trigeminal nerve
D.Facial nerve
E.Abducens nerve
F.Glossopharyngeal nerve
G.Vestibulocochlear nerve
H.Accessory nerve
I.Hypoglossal nerve
Glossopharyngeal nerve
The glossopharyngeal nerve supplies this area and the ear and otalgia may be the result of referred pain.
A 63 year old female is referred to the surgical clinic with an iron deficiency anaemia. Her past medical history includes a left hemi colectomy but no other co-morbidities. At what site is most dietary iron absorbed?
Stomach
Duodenum
Proximal ileum
Distal ileum
Colon
Iron is best absorbed from the proximal small bowel (duodenum and jejunum) in the Fe 2+ state. Iron is transported across the small bowel mucosa by a divalent membrane transporter protein (hence the improved absorption of Fe 2+). The intestinal cells typically store the bound iron as ferritin. Cells requiring iron will typically then absorb the complex as needed.
Which of the following structures lies posterior to the femoral nerve in the femoral triangle?
Adductor longus
Pectineus
Psoas major
Iliacus
None of the above
The iliacus lies posterior to the femoral nerve in the femoral triangle. The femoral sheath lies anterior to the iliacus and pectineus muscles.
A 46 year old lady presents with symptoms of diarrhoea, weight loss of 10 Kg and a skin rash of erythematous blisters involving the abdomen and buttocks. The blisters have an irregular border and both intact and ruptured vesicles. What is the most likely diagnosis?
Colonic adenocarcinoma
Pancreatic adenocarcinoma
Tropical sprue
Glucagonoma
Insulinoma
Glucagonoma is strongly associated with necrolytic migratory erythema.
You are assisting in an open right adrenalectomy for a large adrenal adenoma. The consultant is distracted and you helpfully pull the adrenal into the wound to improve the view. Unfortunately this is followed by brisk bleeding. The vessel responsible for this is most likely to be:
Portal vein
Phrenic vein
Right renal vein
Superior mesenteric vein
Inferior vena cava
It drains directly via a very short vessel. If the sutures are not carefully tied then it may be avulsed off the IVC. An injury best managed using a Satinsky clamp and a 6/0 prolene suture.
A 28 year old lady requires an episiotomy for a ventouse vaginal delivery. Which of the nerves listed below will usually be anaesthetised to allow the episiotomy?
Femoral
Ilioinguinal
Pudendal
Genitofemoral
Sacral plexus
The pudendal nerve innervates the posterior vulval area and is routinely blocked in procedures such as episiotomy.
An enthusiastic surgical registrar undertakes his first solo splenectomy. The operation is far more difficult than anticipated and the registrar leaves a tube drain to the splenic bed at the end of the procedure. Over the following 24 hours approximately 500ml of clear fluid has entered the drain. Biochemical testing of the fluid is most likely to reveal:
Elevated creatinine
Elevated triglycerides
Elevated glucagon
Elevated amylase
None of the above
During splenectomy the tail of the pancreas may be damaged. The pancreatic duct will then drain into the splenic bed, amylase is the most likely biochemical finding. Glucagon is not secreted into the pancreatic duct.
A 22 year old man returns to the UK from holiday in India. He presents with painless jaundice. On examination he is not deeply jaundiced and there is no organomegaly.
A.Gilberts syndrome
B.Crigler Najjar syndrome
C.Hepatocellular carcinoma
D.Mirizzi syndrome
E.Hepatitis A
F.Hepatitis E
G.Bile duct stones
H.Multi cystic liver disease
Hepatitis A
Infective hepatitis is the most likely cause. In the UK, foreign travel is a common cause of developing infectious hepatitis, of which hepatitis A is the most common.
A 56 year old man presents with jaundice. He has a long history of alcohol misuse. On examination he is jaundiced and ultrasound shows multiple echo dense lesions in both lobes of the liver. His alpha feto protein is elevated 6 times the normal range
A.Gilberts syndrome
B.Crigler Najjar syndrome
C.Hepatocellular carcinoma
D.Mirizzi syndrome
E.Hepatitis A
F.Hepatitis E
G.Bile duct stones
H.Multi cystic liver disease
Hepatocellular carcinoma
HCC may complicate cirrhosis. AFP is often raised in HCC.
A 32 year old man who has suffered from Crohns disease for many years presents with intermittent jaundice. When it occurs it is obstructive in nature. It then usually resolves spontaneously.
A.Gilberts syndrome
B.Crigler Najjar syndrome
C.Hepatocellular carcinoma
D.Mirizzi syndrome
E.Hepatitis A
F.Hepatitis E
G.Bile duct stones
H.Multi cystic liver disease
Bile duct stones
Bile salts are absorbed in the terminal ileum. When this process is impaired as in Crohns the patient may develop gallstones, if these pass into the CBD then obstructive jaundice will result.
A 48 year old lady is undergoing an axillary node clearance for breast cancer. Which of the structures listed below are most likely to be encountered during the axillary dissection?
Cords of the brachial plexus
Thoracodorsal trunk
Internal mammary artery
Thoracoacromial artery
None of the above
Beware of damaging the thoracodorsal trunk if a latissimus dorsi flap reconstruction is planned.
The thoracodorsal trunk runs through the nodes in the axilla. If injured it may compromise the function and blood supply to latissimus dorsi, which is significant if it is to be used as a flap for a reconstructive procedure.
A 56 year old lady is referred to the colorectal clinic with symptoms of pruritus ani. On examination a polypoidal mass is identified inferior to the dentate line. A biopsy confirms squamous cell carcinoma. To which of the following lymph node groups will the lesion potentially metastasise?
Internal iliac
External iliac
Mesorectal
Inguinal
None of the above
Lesions distal to the dentate line drain to the inguinal nodes. Occasionally this will result in the need for a block dissection of the groin.
A 56 year old man presents with symptoms of neuropathic facial pain and some weakness of the muscles of facial expression on the right side. On examination he has a hard mass approximately 6cm anterior to the right external auditory meatus. What is the most likely diagnosis?
Pleomorphic adenoma
Adenocarcinoma
Mucoepidermoid carcinoma
Adenoid cystic carcinoma
Lymphoma
The patient is most likely to have a malignant lesion within the parotid. Of the malignancies listed; adenoid cystic carcinoma has the greatest tendency to perineural invasion.
A 45 year old women with breast cancer is started on a chemotherapy regime containing epirubicin. What is the primary mode of action of this drug?
Intercalation of DNA
Antimetabolite
Monoclonal antibody to epidermal growth factor
Inhibition of DNA gyrase
Inhibition of topoisomerase 1
Intercalation of DNA
Which of the following cancers is not associated with the human papillomavirus?
Anal cancer
Oropharyngeal cancer
Tracheal cancer
Vulval cancer
Penile cancer
HPV is associated with:
Cervical cancer (HPV 16/18 most common)
Anal cancer
Penile cancer
Vulval cancer
Oropharyngeal cancer
A 28 year old man has a long history of recurrent chest infections. On examination, he is noted to have no palpable vas deferens. However, both testes are located within the scrotum. What is the most likely underlying disease association?
Kleinfelters syndrome
Kallmann syndrome
Cystic fibrosis
Coeliac disease
Gardners syndrome
99% of males with cystic fibrosis will have absent vas
A 63 year old man is admitted with obstructive jaundice that has developed over the past 3 weeks. He was previously well and on examination has a smooth mass in his right upper quadrant.
A.Carcinoma of the head of the pancreas
B.Bile duct stricture
C.Mirizzi syndrome
D.Bile duct stones
E.Chronic cholecystitis
F.Peri hilar lymphadenopathy
G.Fitz - Hugh Curtis syndrome
Carcinoma of the head of the pancreas
Carcinoma of the pancreas (Courvoisiers law!). The development of jaundice in association with a smooth right upper quadrant mass is typical of distal biliary obstruction secondary to pancreatic malignancy. A bile duct stricture would not present in this way, all the other choices are related to gallstones and Fitz Hugh Curtis syndrome is a complication of pelvic inflammatory disease.
A 41 year old lady is admitted with colicky right upper quadrant pain. On clinical examination she has a mild pyrexia and is clinically jaundiced. An ultrasound scan is reported as showing gallstones and the patient is taken to theatre for an open cholecystectomy. At operation, Calots triangle is almost completely impossible to delineate.
A.Carcinoma of the head of the pancreas
B.Bile duct stricture
C.Mirizzi syndrome
D.Bile duct stones
E.Chronic cholecystitis
F.Peri hilar lymphadenopathy
G.Fitz - Hugh Curtis syndrome
Mirizzi syndrome
In Mirizzi syndrome the gallstone becomes impacted in Hartmans pouch. Episodes of recurrent inflammation occur and this causes compression of the bile duct. In severe cases this then progresses to fistulation. Surgery is extremely difficult as Calots triangle is often completely obliterated and the risks of causing injury to the CBD are high.
A 72 year old man undergoes a distal gastrectomy for carcinoma of the stomach. He presents with jaundice approximately 8 months post operatively. Ultrasound of the liver and bile ducts shows no focal liver lesion and normal calibre common bile duct with intra hepatic duct dilatation.
A.Carcinoma of the head of the pancreas
B.Bile duct stricture
C.Mirizzi syndrome
D.Bile duct stones
E.Chronic cholecystitis
F.Peri hilar lymphadenopathy
G.Fitz - Hugh Curtis syndrome
Peri hilar lymphadenopathy
Unfortunately metastatic disease is the most likely event. Peri hilar lymphadenopathy would be a common culprit.
A 70 year old lady presents with a number of skin lesions that she describes as unsightly. On examination she has a number of raised lesions with a greasy surface located over her trunk. Apart from having a greasy surface the lesions also seem to have scattered keratin plugs located within them.
A.Basal cell carcinoma
B.Dermatofibroma
C.Pilar cyst
D.Epidermoid cyst
E.Spitz naevus
F.Seborrhoeic keratosis
G.Atypical naevus
H.Capillary cavernous haemangioma
Seborrhoeic keratosis
Seborrhoeic keratosis may have a number of appearances. However, the scaly, thick, greasy surface with scattered keratin plugs makes this the most likely diagnosis.
A 21 year old lady presents with a nodule on the posterior aspect of her right calf. It has been present at the site for the past 6 months and occurred at the site of a previous insect bite. Although the nodule appears small, on palpation it appears to be nearly twice the size it appears on examination. The overlying skin is faintly pigmented.
A.Basal cell carcinoma
B.Dermatofibroma
C.Pilar cyst
D.Epidermoid cyst
E.Spitz naevus
F.Seborrhoeic keratosis
G.Atypical naevus
H.Capillary cavernous haemangioma
Dermatofibroma
Dermatofibromas may be pigmented and are often larger than they appear. They frequently occur at sites of previous trauma.
Which of the following ligaments contains the artery supplying the head of femur in children?
Transverse ligament
Ligamentum teres
Iliofemoral ligament
Ischiofemoral ligament
Pubofemoral ligament
Ligamentum teres
What is the reciprocal of absolute risk reduction?
Odds ratio
Number needed to treat
False positive
False negative
None of the above
In epidemiology, the absolute risk reduction, or risk difference is the decrease in risk of a given activity or treatment in relation to a control activity or treatment. It is the inverse of the number needed to treat.
A 15 year old boy undergoes an emergency splenectomy for trauma. He makes a full recovery and is discharged home. Eight weeks post operatively the general practitioner performs a full blood count with a blood film. Which of the following is most likely to be present?
Myofibroblasts
Howell-Jolly bodies
Multinucleate giant cells
Reed Sternberg Cells
None of the above
Post splenectomy blood film features:
Howell- Jolly bodies
Pappenheimer bodies
Target cells
Irregular contracted erythrocytes
As the filtration function is the spleen is no longer present Howell-Jolly bodies are found.
The loss of splenic tissue results in the inability to readily remove immature or abnormal red blood cells from the circulation. The red cell count does not alter significantly. However, cytoplasmic inclusions may be seen e.g. Howell-Jolly bodies.
In the first few days after splenectomy target cells, siderocytes and reticulocytes will appear in the circulation. Immediately following splenectomy a granulocytosis (mainly composed of neutrophils) is seen, this is replaced by a lymphocytosis and monocytosis over the following weeks.
The platelet count is usually increased and this may be persistent, oral antiplatelet agents may be needed in some patients.
A 43 year old woman is identified as being a carrier of a BRCA 1 mutation. Apart from breast cancer, which of the following malignancies is she at greatest risk of developing?
Colonic cancer
Ovarian cancer
Follicular carcinoma of the thyroid
Pituitary adenoma
Phaeochromocytoma
BRCA 1 mutation patients are 55% more likely to get ovarian cancer. Those with BRCA 2 are 25% more likely. The risk of developing other malignancies is slightly increased but not to the same extent, and not enough to justify screening.
A 53 year old man is due to undergo a splenectomy as a treatment for refractory haemolytic anaemia. The underlying pathological basis for haemolytic anaemia is thought to be a Type 2 hypersensitivity response. Which of the following mechanisms best describes this process
Deposition of immune complexes
Cell mediated immune response
IgE mediated response
Formation of autoantibodies against cell surface antigens
None of the above
Formation of autoantibodies against cell surface antigens
Mnemonic for the reactions and the mediators involved
ACID EGG-T
Type 1 Anaphylactic
Type 2 Cytotoxic
Type 3 Immune complex
Type 4 Delayed type
EGG T (mediators)
IgE
IgG
IgG
T cells
Type 2 hypersensitivity reactions (which includes haemolytic anaemia) are associated with formation of antibody against cell surface antigens.
Hypersensitivity reactions
The Gell and Coombs classification divides hypersensitivity reactions into 4 types
Type I Type II Type III Type IV
Description Anaphylactic Cytotoxic Immune complex Delayed type
Mediator IgE IgG, IgM IgG, Ig A, IgM T-cells
Antigen Exogenous Cell surface Soluble Tissues
Response time Minutes Hours Hours 2-3 days
Examples Asthma
Hay fever Autoimmune haemolytic anaemia
Pemphigus
Goodpasture’s Serum sickness
SLE
Aspergillosis Graft versus host disease
Contact dermatitis
What is the commonest type of fistula in ano?
Trans-sphincteric
Supra levator
Complex supra levator
Intersphincteric
Suprasphincteric
Intersphincteric fistulas are the commonest type and the external opening may be internal or external. These are the classical type of fistula and will have an internal opening near the anal verge and obey Goodsalls rule. Primary fistulotomy in this situation usually poses little risk to continence.
A 43 year old lady is donating her left kidney to her sister and the surgeons are harvesting the left kidney. Which of the following structures will lie most anteriorly at the hilum of the left kidney?
Left renal artery
Left renal vein
Left ureter
Left ovarian vein
Left ovarian artery
The renal veins lie most anteriorly, then artery and ureter lies posteriorly.
What is the sensory nerve supply to the angle of the jaw?
Maxillary branch of the trigeminal nerve
Mandibular branch of the trigeminal nerve
C3-C4
Greater auricular nerve (C2-C3)
Buccal branch of the facial nerve
The trigeminal nerve is the major sensory nerve to the face except over the angle of the jaw. The angle of the jaw is innervated by the greater auricular nerve.
A 63 year old man is undergoing a coronary artery bypass procedure. During the median sternotomy which structure would routinely require division?
Parietal pleura
Interclavicular ligament
Internal mammary artery
Brachiocephalic vein
Left vagus nerve
The interclavicular ligament lies at the upper end of a median sternotomy and is routinely divided to provide access. The pleural reflections are often encountered and should not be intentionally divided, if they are, then a chest drain will need to be inserted on the affected side as collections may then accumulate in the pleural cavity. Other structures encountered include the pectoralis major muscles, again if the incision is truly midline then these should not require formal division. The close relationship of the brachiocephalic vein should be borne in mind and it should be avoided, iatrogenic injury to this structure will result in considerable haemorrhage.
A 63 year old male is gardening when he trips and lands on a scythe. He sustains a deep laceration of his lateral thigh, it measures 3cm depth by 7cm length, it penetrates down to the bone, but no fracture is evident on imaging or examination. His co- morbidities include type II diabetes mellitus (diet controlled) and polymyalgia rheumatica (takes regular low dose prednisolone).
A.Immediate split thickness skin graft
B.Delayed split thickness skin graft
C.Primary closure
D.Delayed primary closure
E.Compression bandages
F.Myocutaneous flap
G.Random free flap
The correct answer is Delayed primary closure
Wounds which are contaminated or have the potential to become so are unsafe for immediate primary closure. The combination of diabetes and steroids makes wound complications more likely. Despite his high risk a primary skin graft or flap is unlikely to be a safer option. Either may be used at a later date in the event that delayed primary closure is unsuccessful.
A 71 year old lady trips over and falls landing on her left shin. She sustains a large pretibial laceration of her leg.
A.Immediate split thickness skin graft
B.Delayed split thickness skin graft
C.Primary closure
D.Delayed primary closure
E.Compression bandages
F.Myocutaneous flap
G.Random free flap
Delayed split thickness skin graft
Pretibial lacerations do not heal well. Simple apposition of skin edges almost always fails due to poor quality dermal tissues and underlying haematoma. Debridement of devitalised tissues prior to grafting usually gives the best results.
A 73 year old lady presents with an ulcer overlying her medial malleolus. It is painless and has been present for 4 months. She has oedema of the lower limbs and her ABPI measures 0.9.
A.Immediate split thickness skin graft
B.Delayed split thickness skin graft
C.Primary closure
D.Delayed primary closure
E.Compression bandages
F.Myocutaneous flap
G.Random free flap
Compression bandages
This is likely to be a venous leg ulcer. These are typically managed using compression bandages. Contra indications to this technique include peripheral vascular disease (not present here).
Which of the following structures separates the subclavian artery from the subclavian vein?
Scalenus anterior
Scalenus medius
Sternocleidomastoid
Pectoralis major
Pectoralis minor
The artery and vein are separated by scalenus anterior. This muscle runs from the transverse processes of C3,4,5 and 6 to insert onto the scalene tubercle of the first rib.
A 56 year old lady is due to undergo a left hemicolectomy for carcinoma of the splenic flexure. The surgeons decide to perform a high ligation of the inferior mesenteric vein. Into which of the following does this structure usually drain?
Portal vein
Inferior vena cava
Left renal vein
Left iliac vein
Splenic vein
Beware of ureteric injury in colonic surgery.
The inferior mesenteric vein drains into the splenic vein, this point of union lies close to the duodenum and this surgical maneouvre is a recognised cause of ileus.
Which nerve innervates the 1st and 2nd muscles of the thenar eminence?
A. Radial nerve
B. Ulnar nerve
C. Median nerve
D. Musculocutaneous nerve
E. Axillary nerve
C The median nerve innervates the 1st and 2nd muscles of the thenar eminence, including the abductor pollicis brevis and opponens pollicis.
Which part of the sternocleidomastoid muscle is attached to the upper manubrium sterni?
A. Muscular head
B. Mastoid process
C. Lateral area of the superior nuchal line
D. Spinal part of accessory nerve
E. Rounded tendon
E The rounded tendon of the sternocleidomastoid muscle is attached to the upper manubrium sterni.
Which nervous discharge causes erection?
A. Sympathetic discharge
B. Parasympathetic discharge
C. Somatic nerves
D. Autonomic discharge
E. Pelvic plexus
B Parasympathetic discharge causes erection, while sympathetic discharge causes ejaculation and detumescence.
What is the extensor retinaculum?
A. A tendon in the wrist
B. A thickening of the deep fascia across the back of the wrist
C. A ligament connecting the radius and ulnar styloid
D. A branch of the radial nerve
E. A branch of the ulnar nerve
B The extensor retinaculum is a thickening of the deep fascia that stretches across the back of the wrist and holds the long extensor tendons in position.
What is the arterial supply to the adrenal glands?
A. From inferior phrenic artery
B. From aorta
C. From renal arteries
D. From celiac artery
E. From superior mesenteric artery
A The arterial supply to the adrenal glands includes superior adrenal arteries from the inferior phrenic artery, middle adrenal arteries from the aorta, and inferior adrenal arteries from the renal arteries.
The knee joint is supplied by which nerve divisions?
A. Femoral, tibial and common peroneal divisions of the sciatic
B. Obturator and sciatic divisions
C. Femoral and obturator divisions
D. Tibial and common peroneal divisions of the sciatic
E. Femoral and tibial divisions of the sciatic
A The knee joint is supplied by the femoral, tibial and common peroneal divisions of the sciatic and by a branch from the obturator nerve.
A 43 year old lady is due to undergo an axillary node clearance as part of treatment for carcinoma of the breast. Which of the following fascial layers will be divided during the surgical approach to the axilla?
Sibsons fascia
Pre tracheal fascia
Waldayers fascia
Clavipectoral fascia
None of the above
The clavipectoral fascia is situated under the clavicular portion of pectoralis major. It protects both the axillary vessels and nodes. During an axillary node clearance for breast cancer the clavipectoral fascia is incised and this allows access to the nodal stations. The nodal stations are; level 1 nodes inferior to pectoralis minor, level 2 lie behind it and level 3 above it. During a Patey Mastectomy surgeons divide pectoralis minor to gain access to level 3 nodes. The use of sentinel node biopsy (and stronger assistants!) have made this procedure far less common.
What are the boundaries of the ‘safe triangle’ for chest drain insertion?
Bounded by trapezius, latissimus dorsi, and laterally by the vertebral border of the scapula
Bounded by latissimus dorsi, pectoralis major, line superior to the nipple and apex at the axilla
Bounded by latissimus dorsi, serratus anterior, line superior to the nipple and apex at the axilla
Bounded by trapezius, deltoid, rhomboid major and teres minor
Bounded by trapezius, deltoid and latissimus dorsi
Bounded by latissimus dorsi, pectoralis major, line superior to the nipple and apex at the axilla
Which of the following is not an oncogene?
ras
myc
sis
Ki 67
erb-B
Ki 67 is a nuclear proliferation marker (used in immunohistochemistry). Although, Ki67 positivity is a marker of malignancy, it is not itself, an oncogene.
Among the options you provided, Ki 67 is not an oncogene 123. Ki 67 is a protein that is expressed in proliferating cells and is used as a marker of cell proliferation 13. It is not a gene that has the potential to cause cancer 13.
On the other hand, ras, myc, sis, and erb-B are all oncogenes 14. Ras is a family of genes that encode proteins involved in cell signaling pathways that regulate cell growth and division 14. Mutations in ras genes can cause them to become permanently activated, leading to uncontrolled cell growth and division, and ultimately cancer 14. Myc is a transcription factor that regulates the expression of genes involved in cell growth and division 14. Overexpression of myc can lead to uncontrolled cell growth and division, and ultimately cancer 14. Sis is a gene that encodes a protein called platelet-derived growth factor (PDGF) 14. PDGF is involved in cell growth and division, and overexpression of sis can lead to uncontrolled cell growth and division, and ultimately cancer 14. Erb-B is a gene that encodes a receptor protein involved in cell signaling pathways that regulate cell growth and division 14. Mutations in erb-B can cause the receptor to become permanently activated, leading to uncontrolled cell growth and division, and ultimately cancer 14.
An otherwise fit 30 year old male donates 500ml of blood. Which of the processes outlined below is most likely to occur?
Oliguria
Activation of the renin angiotensin system
Sweating
Fall in mean arterial pressure
Tachypnoea
The loss of 500ml (assuming a 70 Kg male) will usually be sufficient to activate the renin angiotensin system. It is unlikely that it would cause any other physiological disturbance.
The vertebral artery traverses all of the following except?
Transverse process of C6
Transverse process of the axis
Vertebral canal
Foramen magnum
Intervertebral foramen
The vertebral artery passes through the foramina which are located in the transverse processes of the cervical vertebra, it does not traverse the intervertebral foramen.
The vertebral artery is a major artery in the neck that supplies blood to the brainstem and cerebellum. It arises from the subclavian artery and enters the transverse foramen of the cervical vertebrae, except for the seventh cervical vertebra (C7) 1. After passing through the transverse foramina of the cervical vertebrae, the vertebral artery ascends posteromedially towards the skull 2. It passes through the foramen magnum, which is a large opening in the occipital bone of the skull that allows the spinal cord to connect with the brain 2. The vertebral artery also passes through the subarachnoid space, which is the space between the arachnoid mater and the pia mater that surrounds the brain and spinal cord 3. However, the vertebral artery does not traverse the intervertebral foramen 45. The intervertebral foramen is the opening between adjacent vertebrae through which spinal nerves exit the spinal cord 6
A 25 year old man is injured in a road traffic accident. His right tibia is fractured and is managed by fasciotomies and application of an external fixator. Over the next 48 hours his serum creatinine rises and urine is sent for microscopy, muddy brown casts are identified. What is the most likely underlying diagnosis?
Acute interstitial nephritis
Acute tubular necrosis
Glomerulonephritis
IgA Nephropathy
Thin basement membrane disease
This patient is likely to have had compartment syndrome (tibial fracture + fasciotomies) which may produce myoglobinuria. The presence of worsening renal function, together with muddy brown casts is strongly suggestive of acute tubular necrosis. Acute interstitial nephritis usually arises from drug toxicity and does not usually produce urinary muddy brown casts. Thin basement membrane disease is an autosomal dominant condition that causes persistent microscopic haematuria, but not worsening renal function.
During an Ivor Lewis Oesophagectomy for carcinoma of the lower third of the oesophagus which structure is divided to allow mobilisation of the oesophagus?
Vagus nerve
Azygos vein
Right inferior lobar bronchus
Phrenic nerve
Pericardiophrenic artery
The azygos vein is routinely divided during an oesophagectomy to allow mobilisation. It arches anteriorly to insert into the SVC on the right hand side.
A 32 year old man is stabbed in the neck and the inferior trunk of his brachial plexus is injured. Which of the modalities listed below is least likely to be affected?
Initiating abduction of the shoulder
Abduction of the fingers
Flexion of the little finger
Sensation on the palmar aspect of the little finger
Gripping a screwdriver
Inferior trunk of brachial plexus.
C8 and T1 roots
Contributes to ulnar nerve and part of median nerve
The inferior trunk of the brachial plexus is rarely injured. Nerve roots C8 and T1 are the main contributors to this trunk. Therefore an injury to this site will most consistently affect the ulnar nerve. The inferior trunk also contributes to the median nerve by way of the medial cord and therefore some impairment of grip is almost inevitable.
Brachial plexus
The brachial plexus extends from the neck to the axilla. It is formed by the ventral rami of the fifth to the eighth cervical nerves with the ascending part of the first thoracic nerve.
Location of the plexus
The ventral rami which form the plexus enter the lower part of the posterior triangle of the neck in series with the ventral rami of the cervical plexus. The second part of the subclavian artery lies immediately anterior to the lower two rami. The upper three rami intermingle and pass inferolaterally towards the axilla and subclavian artery. They are enclosed within an extension of the prevertebral fascia. In the neck the plexus lies deep to platysma, the supraclavicular nerves, inferior belly of omohyoid and the transverse cervical artery. It then passes deep to the clavicle and the suprascapular vessels, to enter the axilla, and thence surround the second part of the axillary artery
Composition of the plexus
Ventral rami, the roots of the plexus, lie between scalenus medius and anterior.
As they enter the posterior triangle, the upper two (C5,6) and lower two (C8, T1) roots of the plexus unite to form the upper and lower trunks of the plexus respectively. Meanwhile, C7 continues as the middle trunk. The lower trunk may groove the superior surface of the first rib posterior to the subclavian artery, and the root from the first ventral ramus is always in contact with it.
Each trunk divides into ventral and dorsal divisions which are destined to supply the anterior (flexor) and posterior (extensor) parts of the upper limb.
The cords of the plexus are formed in the axilla. The dorsal divisions unite to form the posterior cord (C5-8). The ventral divisions of the upper and middle trunks unite to form the lateral cord (C5-7), while the ventral divisions of the lower trunk continues as the medial cord (C8-T1). The cords are named according to their relationship to the axillary artery. Each cord terminates by dividing into two main branches at the beginning of the third part of the artery.
Sympathetic communications
The fifth and sixth cervical ventral rami receive grey rami communicantes from the middle cervical ganglion, while the two or more grey rami communicantes pass from the inferior cervical ganglion to the seventh and eighth cervical ventral rami. The first thoracic ventral ramus receives its grey ramus from the cervicothoracic ganglion. Its for this reason that inferior plexus injury can be complicated by a Horners syndrome.
Summary
Origin Anterior rami of C5 to T1
Sections of the plexus
Roots, trunks, divisions, cords, branches
Mnemonic:Real Teenagers Drink Cold Beer
Roots
Located in the posterior triangle
Pass between scalenus anterior and medius
Trunks
Located posterior to middle third of clavicle
Upper and middle trunks related superiorly to the subclavian artery
Lower trunk passes over 1st rib posterior to the subclavian artery
Divisions Apex of axilla
Cords Related to axillary artery
Brown tumours of bone are associated with which of the following?
Hyperthyroidism
Hypothyroidism
Hyperparathyroidism
Hypoparathyroidism
Osteopetrosis
Brown tumors are tumors of bone that arise in settings of excess osteoclast activity, such as hyperparathyroidism, and consist of fibrous tissue, woven bone and supporting vasculature, but no matrix. They are radiolucent on x-ray. The osteoclasts consume the trabecular bone that osteoblasts lay down and this front of reparative bone deposition followed by additional resorption can expand beyond the usual shape of the bone, involving the periosteum thus causing bone pain. They appear brown because haemosiderin is deposited at the site.
Primary hyperparathyroidism
In exams, primary hyperparathyroidism is stereotypically seen in elderly females with an unquenchable thirst and an inappropriately normal or raised parathyroid hormone level. It is most commonly due to a solitary adenoma
Causes of primary hyperparathyroidism
80%: solitary adenoma
15%: hyperplasia
4%: multiple adenoma
1%: carcinoma
Features - ‘bones, stones, abdominal groans and psychic moans’
Polydipsia, polyuria
Peptic ulceration/constipation/pancreatitis
Bone pain/fracture
Renal stones
Depression
Hypertension
Associations
Hypertension
Multiple endocrine neoplasia: MEN I and II
Investigations
Raised calcium, low phosphate
PTH may be raised or normal
Technetium-MIBI subtraction scan
Treatment
Parathyroidectomy, if imaging suggests target gland then a focused approach may be used
Which of the following muscles does not attach to the radius?
Pronator quadratus
Biceps
Brachioradialis
Supinator
Brachialis
The brachialis muscle inserts into the ulna. The other muscles are all inserted onto the radius.
Radius
The radius is one of the two long forearm bones that extends from the lateral side of the elbow to the thumb side of the wrist. It has two expanded ends, of which the distal end is the larger. Key points relating to its topography and relations are outlined below;
Upper end
Articular cartilage- covers medial > lateral side
Articulates with radial notch of the ulna by the annular ligament
Muscle attachment- biceps brachii at the tuberosity
Shaft
Muscle attachment
Upper third of the body Supinator
Flexor digitorum superficialis
Flexor pollicis longus
Middle third of the body Pronator teres
Lower quarter of the body Pronator quadratus
Tendon of supinator longus
Lower end
Quadrilateral
Anterior surface- capsule of wrist joint
Medial surface- head of ulna
Lateral surface- ends in the styloid process
Posterior surface: 3 grooves containing:
1. Tendons of extensor carpi radialis longus and brevis
2. Tendon of extensor pollicis longus
3. Tendon of extensor indicis
A 27-year-old male presents to urology for investigation of pyelonephritis. He reports malaise, pyrexia, lymphadenopathy and a maculopapular rash. The Monospot test is negative. Given a history of recent high-risk sexual behaviour you are asked to exclude a HIV seroconversion illness. What is the most appropriate investigation?
Antibodies to HIV-2
gp120 polymerase chain reaction
p24 antigen test
CCR5 polymerase chain reaction
Antibodies to HIV-1
p24 antigen test
HIV testing
HIV seroconversion is symptomatic in 60-80% of patients and typically presents as a glandular fever type illness. Increased symptomatic severity is associated with poorer long term prognosis. It typically occurs 3-12 weeks after infection
Features
sore throat
lymphadenopathy
malaise, myalgia, arthralgia
diarrhoea
maculopapular rash
mouth ulcers
rarely meningoencephalitis
Diagnosis
antibodies to HIV may not be present
HIV PCR and p24 antigen tests can confirm diagnosis
HIV antibody test
most common and accurate test
usually consists of both a screening ELISA (Enzyme Linked Immuno-Sorbent Assay) test and a confirmatory Western Blot Assay
most people develop antibodies to HIV at 4-6 weeks but 99% do by 3 months
p24 antigen test
usually positive from about 1 week to 3 - 4 weeks after infection with HIV
sometimes used as an additional screening test in blood banks
An 83 year old lady is admitted with a fractured neck of femur and is managed with surgical fixation. On the day following surgery, she is noted to have a dense hemiparesis. Several days later, an MRI scan of the brain is performed which demonstrates a 3cm area of devitalised tissue within the right frontal lobe. Which of the pathological processes described below is most likely to account for these appearances?
Coagulative necrosis
Colliquative necrosis
Caseous necrosis
Fibrinoid necrosis
Apoptosis
Death of CNS tissue through vascular compromise tends to result in colliquative necrosis as the cells have less formal supporting stroma.
Cell death
Cells can die via two mechanisms; necrosis and apoptosis. These are outlined below:
Necrosis
Necrosis is characterised by bioenergetics failure. Loss of tissue perfusion results in hypoxia and an inability to generate ATP. The integrity of the cellular membrane is lost and the loss of ATP results in loss of energy dependent cellular transport mechanisms. There is an influx of water, ionic instability and cellular lysis. The release of intracellular contents may stimulate an inflammatory response. Several types of necrosis are recognised; coagulative, colliquative, caseous, gangrene, fibrinoid and fat necrosis. The type of tissue and the underlying cause determine the predominant necrosis pattern.
Coagulative necrosis
The commonest type, occurs in most organs
Tissue is initially firm, later becomes soft as tissue is digested by macrophages
In the early phases the histological appearances may demonstrate little change
In later stages cellular outlines are seen with loss of intracellular detail
Colliquative necrosis
Occurs in tissues with no supporting stroma
Dominant necrosis pattern in the CNS
Necrotic site may eventually become encysted
Caseous necrosis
No definable structure seen in the necrotic tissue
Amorphous eosinophilic tissue may be seen histologically
Classically seen in tuberculosis
Gangrene
Necrosis with putrefaction of tissue
May complicate ischaemia
Haemoglobin degenerates and results in the deposition of iron sulphide (which is why the tissue is black)
Both wet and dry gangrene may occur, in wet gangrene there is often a liquefactive component and super-added infection (which usually smells!)
Fibrinoid necrosis
Classically seen in arterioles in patients with hypertension (malignant type)
Necrosis of the smooth muscle wall occurs and plasma may extravasate into the media with fibrin deposition
Fat necrosis
Direct trauma to fat can result in rupture of adipocytes
Lipids incite a local inflammatory reaction
Inflammatory cells phagocytose the lipid with eventual fibrosis
Apoptosis
Also known as programmed cell death
Energy dependent pathways are activated via a number of intracellular signalling mechanisms
It is the result of the activation of caspases triggered by the bcl-2 family or the binding of the FAS ligand to its receptor
DNA fragments, mitochondrial function ceases, nuclear and cellular shrinkage occurs
Phagocytosis of the cell does not occur, instead the cell degenerates into apoptotic bodies
Into which of these veins does the middle thyroid vein drain?
Vertebral
External jugular
Internal jugular
Subclavian
Anterior jugular
It drains to the internal jugular vein. Which is one of the reasons why it bleeds so copiously if a ligature slips.
Thyroid gland
Right and left lobes connected by isthmus
Surrounded by sheath from pretracheal layer of deep fascia
Apex: Lamina of thyroid cartilage
Base: 4th-5th tracheal ring
Pyramidal lobe: from isthmus
May be attached to foramen caecum at the base of the tongue
Relations
Anteromedially
Sternothyroid
Superior belly of omohyoid
Sternohyoid
Anterior aspect of sternocleidomastoid
Posterolaterally Carotid sheath
Medially
Larynx
Trachea
Pharynx
Oesophagus
Cricothyroid muscle
External laryngeal nerve (near superior thyroid artery)
Recurrent laryngeal nerve (near inferior thyroid artery)
Posterior
Parathyroid glands
Anastomosis of superior and inferior thyroid arteries
Isthmus
Anteriorly: Sternothyroids, sternohyoids, anterior jugular veins
Posteriorly: 2nd, 3rd, 4th tracheal rings (attached via Ligament of Berry)
Blood Supply
Arterial
Superior thyroid artery (1st branch of external carotid)
Inferior thyroid artery (from thyrocervical trunk)
Thyroidea ima (in 10% of population -from brachiocephalic artery or aorta)
Venous
Superior and middle thyroid veins - into the IJV
Inferior thyroid vein - into the brachiocephalic veins
A 28 year old man falls and hits his head against a wall. There is a brief loss of consciousness. When assessed in accident and emergency he is alert and orientated with a GCS of 15, imaging shows no fracture of the skull. What is his risk of having an intracranial haematoma requiring removal?
1 in 6000
1 in 40
1 in 4
1 in 50,000
1 in 120
Head injury and haematoma
Risk of haematoma (requiring removal) in adults attending accident and emergency units following head injury.
Injury Conscious level Risk of haematoma requiring removal
Concussion, no skull fracture Orientated 1 in 6000
Concussion, no skull fracture Not orientated 1 in 120
Skull fracture Orientated 1 in 32
Skull fracture Not orientated 1 in 4
A 22 year old lady presents with an episode of renal colic and following investigation is suspected of suffering from MEN IIa. Which of the following abnormalities of the parathyroid glands are most often found in this condition?
Hypertrophy
Hyperplasia
Adenoma
Carcinoma
Metaplasia
MEN IIa
Medullary thyroid cancer
Hyperparathyroidism (usually hyperplasia)
Phaeochromocytoma
In MEN IIa the commonest lesion is medullary thyroid cancer, with regards to the parathyroid glands the most common lesion is hyperplasia. In MEN I a parathyroid adenoma is the most common lesion.
Multiple Endocrine Neoplasia
Multiple endocrine neoplasia (MEN) is inherited as an autosomal dominant disorder.
The table below summarises the three main types of MEN:
MEN type I MEN type IIa MEN type IIb
Mnemonic ‘three P’s’:
Parathyroid (95%): Parathyroid adenoma
Pituitary (70%): Prolactinoma/ACTH/Growth Hormone secreting adenoma
Pancreas (50%): Islet cell tumours/Zollinger Ellison syndrome
also: Adrenal (adenoma) and thyroid (adenoma) Phaeochromocytoma
Medullary thyroid cancer (70%)
Hyperparathyroidism (60%) Same as MEN IIa with addition of:
Marfanoid body habitus
Mucosal neuromas
MENIN gene (chromosome 11)
Most common presentation = hypercalcaemia RET oncogene (chromosome 10) RET oncogene (chromosome 10)
Which of the following is the first vessel to branch from the external carotid artery?
Superior thyroid artery
Inferior thyroid artery
Lingual artery
Facial artery
Occipital artery
Superior Thyroid Artery
Mnemonic
(Order in which they branch off)Some (sup thyroid)Attendings (Ascending Pharyngeal)Like (Lingual)Freaking (Facial)Out (Occipital)Potential (Post auricular)Medical (Maxillary)Students (Sup temporal) The first branch of the external carotid artery is the superior thyroid artery. The inferior thyroid artery is derived from the thyrocervical trunk.
External carotid artery
The external carotid commences immediately lateral to the pharyngeal side wall. It ascends and lies anterior to the internal carotid and posterior to the posterior belly of digastric and stylohyoid. More inferiorly it is covered by sternocleidomastoid, passed by hypoglossal nerves, lingual and facial veins.
It then pierces the fascia of the parotid gland finally dividing into its terminal branches within the gland itself.
Surface marking of the carotid
This is an imaginary line drawn from the bifurcation of the common carotid passing behind the angle of the jaw to a point immediately anterior to the tragus of the ear.
Branches of the external carotid artery
It has six main branches, three in front, two behind and one deep.
Three in front Superior thyroid
Lingual
Facial
Two behind Occipital
Posterior auricular
Deep Ascending pharyngeal
A 52 year old male presents with discomfort in the fingers of his left hand. On examination, the ring and little fingers of his left hand are flexed and unable to extend completely. He is able to make a fist with the hand. Palpation reveals thickened nodules on the medial half of the palm. What is the most likely diagnosis?
de Quervain’s tenosynovitis
Tendon sheath infection
Dupuytren’s contracture
Ganglion
Heberden’s nodes
Discomfort of the hand is not uncommon in Dupuytren’s contracture, true pain is unusual. The disease most commonly affects the ring and little fingers.
Hand diseases
Dupuytrens contracture
Fixed flexion contracture of the hand where the fingers bend towards the palm and cannot be fully extended.
Caused by underlying contractures of the palmar aponeurosis . The ring finger and little finger are the fingers most commonly affected. The middle finger may be affected in advanced cases, but the index finger and the thumb are nearly always spared.
Progresses slowly and is usually painless. In patients with this condition, the tissues under the skin on the palm of the hand thicken and shorten so that the tendons connected to the fingers cannot move freely. The palmar aponeurosis becomes hyperplastic and undergoes contracture.
Commonest in males over 40 years of age.
Association with liver cirrhosis and alcoholism. However, many cases are idiopathic.
Treatment is surgical and involves fasciectomy. However, the condition may recur and many surgical therapies are associated with risk of neurovascular damage to the digital nerves and arteries.
Carpal tunnel syndrome
Idiopathic median neuropathy at the carpal tunnel.
Characterised by altered sensation of the lateral 3 fingers.
The condition is commoner in females and is associated with other connective tissue disorders such as rheumatoid disease. It may also occur following trauma to the distal radius.
Symptoms occur mainly at night in early stages of the condition.
Examination may demonstrate wasting of the muscles of the thenar eminence and symptoms may be reproduced by Tinels test (compression of the contents of the carpal tunnel).
Formal diagnosis is usually made by electrophysiological studies.
Treatment is by surgical decompression of the carpal tunnel, a procedure achieved by division of the flexor retinaculum. Non - surgical options include splinting and bracing.
Miscellaneous hand lumps
Osler’s nodes Osler’s nodes are painful, red, raised lesions found on the hands and feet. They are the result of the deposition of immune complexes.
Bouchard’s nodes Hard, bony outgrowths or gelatinous cysts on the proximal interphalangeal joints (the middle joints of fingers or toes.) They are a sign of osteoarthritis, and are caused by formation of calcific spurs of the articular cartilage.
Heberden’s nodes Typically develop in middle age, beginning either with a chronic swelling of the affected joints or the sudden painful onset of redness, numbness, and loss of manual dexterity. This initial inflammation and pain eventually subsides, and the patient is left with a permanent bony outgrowth that often skews the fingertip sideways. It typically affects the DIP joint.
Ganglion Swelling in association with a tendon sheath commonly near a joint. They are common lesions in the wrist and hand. Usually they are asymptomatic and cause little in the way of functional compromise. They are fluid filled although the fluid is similar to synovial fluid it is slightly more viscous. When the cysts are troublesome they may be excised.
What are the most likely effects of the release of vasopressin from the pituitary?
Vasoconstriction of the afferent glomerular arteriole
Increased permeability of the mesangial cells to glucose
Reduced permeability of the inner medullary portion of the collecting duct to urea
Increased secretion of aldosterone from the macula densa
Increased water permeability of the distal tubule cells of the kidney
ADH (vasopressin) results in the insertion of aquaporin channels in apical membrane of the distal tubule and collecting ducts.
Renal Physiology
Overview
Each nephron is supplied with blood from an afferent arteriole that opens onto the glomerular capillary bed.
Blood then flows to an efferent arteriole, supplying the peritubular capillaries and medullary vasa recta.
The kidney receives up to 25% of resting cardiac output.
Control of blood flow
The kidney is able to autoregulate its blood flow between systolic pressures of 80- 180mmHg so there is little variation in renal blood flow.
This is achieved by myogenic control of arteriolar tone, both sympathetic input and hormonal signals (e.g. renin) are responsible.
Glomerular structure and function
Blood inside the glomerulus has considerable hydrostatic pressure.
The basement membrane has pores that will allow free diffusion of smaller solutes, larger negatively charged molecules such as albumin are unable to cross.
The glomerular filtration rate (GFR) is equal to the concentration of a solute in the urine, times the volume of urine produced per minute, divided by the plasma concentration (assuming that the solute is freely diffused e.g. inulin).
In clinical practice creatinine is used because it is subjected to very little proximal tubular secretion.
Although subject to variability, the typical GFR is 125ml per minute.
Glomerular filtration rate = Total volume of plasma per unit time leaving the capillaries and entering the bowman’s capsule
Renal clearance = volume plasma from which a substance is removed per minute by the kidneys
Substances used to measure GFR have the following features:
1. Inert
2. Free filtration from the plasma at the glomerulus (not protein bound)
3. Not absorbed or secreted at the tubules
4. Plasma concentration constant during urine collection
Examples: inulin, creatinine
plasma concentration (mmol/l)
The clearance of a substance is dependent not only on its diffusivity across the basement membrane but also subsequent tubular secretion and / or reabsorption.
So glucose which is freely filtered across the basement membrane is usually reabsorbed from tubules giving a clearance of zero.
Tubular function
Reabsorption and secretion of substances occurs in the tubules.
In the proximal tubule substrates such as glucose, amino acids and phosphate are co-transported with sodium across the semi permeable membrane.
Glucose is reabsorbed by both active and passive processes. The former uses sodium–glucose co-transporters (SGLTs) in the proximal convoluted tubule. The latter achieved through facilitated and passive process through GLUT transporters.
Up to two thirds of filtered water is reabsorbed in the proximal tubules.
This will lead to increase in urea concentration in the distal tubule allowing for its increased diffusion.
Substances to be secreted into the tubules are taken up from the peritubular blood by tubular cells.
Solutes such as paraaminohippuric acid are cleared with a single passage through the kidneys and this is why it is used to measure renal plasma flow. Ions such as calcium and phosphate will have a tubular reabsorption that is influenced by plasma PTH levels.
Potassium may be both secreted and re-absorbed and is co-exchanged with sodium.
Loop of Henle
Approximately 60 litres of water containing 9000mmol sodium enters the descending limb of the loop of Henle in 24 hours.
Loops from the juxtamedullary nephrons run deep into the medulla.
The osmolarity of fluid changes and is greatest at the tip of the papilla.
The thick ascending limb is impermeable to water, but highly permeable to sodium and chloride ions.
This loss means that at the beginning of the thick ascending limb the fluid is hypo osmotic compared with adjacent interstitial fluid.
In the thick ascending limb the reabsorption of sodium and chloride ions occurs by both facilitated and passive diffusion pathways.
The loops of Henle are co-located with vasa recta, these will have similar solute compositions to the surrounding extracellular fluid so preventing the diffusion and subsequent removal of this hypertonic fluid.
The energy dependent reabsorption of sodium and chloride in the thick ascending limb helps to maintain this osmotic gradient.
A 42 year old man is admitted to surgery with acute appendicitis. He is known to have hypertension, psoriatic arthropathy and polymyalgia rheumatica. His medical therapy includes:
Paracetamol 1g qds
Codeine phosphate 30mg qds
Bendrofluazide 2.5 mg od
Ramipril 10mg od
Methotrexate 7.5mg once a week
Prednisolone 5mg od
You are called by the core surgical trainee to assess this man as he has become delirious and hypotensive 2 hours after surgery. His blood results reveal:
Na+ 132 mmol/l
K+ 5.2 mmol/l
Urea 10 mmol/l
Creatinine 111 µmol/l
Glucose 3.5
CRP 158
Hb 10.2 g/dl
Platelets 156 * 109/l
WBC 14 * 109/l
What is the most likely diagnosis?
Septic shock secondary to appendicitis
Neutropenic sepsis
Phaeochromocytoma
Perforated bowel
Addisonian crisis
Features of an addisonian crisis:
Hyponatraemia
Hyperkalaemia
Hypoglycaemia
This man is on steroids for polymyalgia rheumatica. Surgery can precipitate acute adrenal deficiency. The diagnosis is further confirmed by the blood results of hyponatraemia, hyperkalaemia and hypoglycaemia. This patient urgently needs hydrocortisone.
Addisonian crisis
Causes
Sepsis or surgery causing an acute exacerbation of chronic insufficiency (Addison’s, Hypopituitarism)
Adrenal haemorrhage eg Waterhouse-Friderichsen syndrome (fulminant meningococcemia)
Steroid withdrawal
Management
Hydrocortisone 100 mg im or iv
1 litre normal saline infused over 30-60 mins or with dextrose if hypoglycaemic
Continue hydrocortisone 6 hourly until the patient is stable. No fludrocortisone is required because high cortisol exerts weak mineralocorticoid action
Oral replacement may begin after 24 hours and be reduced to maintenance over 3-4 days
Which of the following conditions is least likely to exhibit the Koebner phenomenon?
Vitiligo
Molluscum contagiosum
Lichen planus
Psoriasis
Lupus vulgaris
Lupus vulgaris is not associated with the Koebner phenomenon.
Koebner phenomenon
The Koebner phenomenon describes skin lesions which appear at the site of injury. It is seen in:
Psoriasis
Vitiligo
Warts
Lichen planus
Lichen sclerosus
Molluscum contagiosum
Which of the following statements relating to the posterior cerebral artery is false?
It supplies the visual cortex
It is closely related to the 3rd cranial nerve
It is a branch of the basilar artery
It is connected to the circle of Willis via the superior cerebellar artery
When occluded may result in contralateral loss of field of vision
The posterior cerebral arteries are formed at the bifurcation of the basilar artery and is connected to the circle of Willis via the posterior communicating artery.
The posterior cerebral arteries supply the occipital lobe and part of the temporal lobe.
Circle of Willis
The two internal carotid arteries and two vertebral arteries form an anastomosis known as the Circle of Willis on the inferior surface of the brain. Each half of the circle is formed by:
1. Anterior communicating artery
2. Anterior cerebral artery
3. Internal carotid artery
4. Posterior communicating artery
5. Posterior cerebral arteries and the termination of the basilar artery
The circle and its branches supply; the corpus striatum, internal capsule, diencephalon and midbrain.
Vertebral arteries
Enter the cranial cavity via foramen magnum
Lie in the subarachnoid space
Ascend on anterior surface of medulla oblongata
Unite to form the basilar artery at the base of the pons
Branches:
Posterior spinal artery
Anterior spinal artery
Posterior inferior cerebellar artery
Basilar artery
Branches:
Anterior inferior cerebellar artery
Labyrinthine artery
Pontine arteries
Superior cerebellar artery
Posterior cerebral artery (at the point where it bifurcates)
Internal carotid arteries
Branches:
Posterior communicating artery
Anterior cerebral artery
Middle cerebral artery
Anterior choroid artery
A 67 year old lady presents with jaundice and abdominal pain. Her investigations show a dilated common bile duct, a carcinoma of the pancreatic head compressing the pancreatic duct. Her liver contains bi-lobar metastasis. What is the most appropriate course of action?
Undertake synchronous resection of liver metastases and pancreatoduodenectomy
Resection of liver metastases, chemotherapy and then resection of the primary lesion
Insertion of endoscopic biliary stent and consideration of palliative chemotherapy
Palliation alone
Insertion of PTC drain and palliation
The presence of metastatic disease in the context of pancreatic cancer renders this incurable and resection of metastatic disease is not appropriate.
Pancreatic cancer
- Adenocarcinoma
Risk factors: Smoking, diabetes, adenoma, familial adenomatous polyposis
Mainly occur in the head of the pancreas (70%)
Spread locally and metastasizes to the liver
Carcinoma of the pancreas should be differentiated from other periampullary tumours with better prognosis
Clinical features
Weight loss
Painless jaundice
Epigastric discomfort (pain usually due to invasion of the coeliac plexus is a late feature)
Pancreatitis
Trousseau’s sign: migratory superficial thrombophlebitis
Investigations
USS: May miss small lesions
CT Scanning (pancreatic protocol). If unresectable on CT then no further staging needed
PET/CT for those with operable disease on CT alone
ERCP/ MRI for bile duct assessment
Staging laparoscopy to exclude peritoneal disease
Management
Head of pancreas: Whipple’s resection (SE dumping and ulcers). Newer techniques include pylorus preservation and SMA/ SMV resection
Carcinoma body and tail: poor prognosis, distal pancreatectomy, if operable
Usually adjuvent chemotherapy for resectable disease
ERCP and stent for jaundice and palliation
Surgical bypass may be needed for duodenal obstruction
A motorcyclist is involved in a road traffic accident. He suffers a complex humeral shaft fracture which is plated. Post operatively he complains of an inability to extend his fingers. Which of the following structures is most likely to have been injured?
Ulnar nerve
Radial nerve
Median nerve
Axillary nerve
None of the above
Mnemonic for radial nerve muscles: BEST
B rachioradialis
E xtensors
S upinator
T riceps
Radial nerve
Continuation of posterior cord of the brachial plexus (root values C5 to T1)
Path
In the axilla: lies posterior to the axillary artery on subscapularis, latissimus dorsi and teres major.
Enters the arm between the brachial artery and the long head of triceps (medial to humerus).
Spirals around the posterior surface of the humerus in the groove for the radial nerve.
At the distal third of the lateral border of the humerus it then pierces the intermuscular septum and descends in front of the lateral epicondyle.
At the lateral epicondyle it lies deeply between brachialis and brachioradialis where it then divides into a superficial and deep terminal branch.
Deep branch crosses the supinator to become the posterior interosseous nerve.
Which muscle does not insert on the medial or superomedial surface of the greater trochanter?
Gemelli
Obturator internus
Piriformis
Quadratus femoris
Obturator externus
Mnemonic for muscle attachment on greater trochanter is POGO:
Piriformis
Obturator internus
Gemelli
Obturator externus
The quadratus femoris fibres pass laterally to be inserted into the quadrate tubercle on the intertrochanteric crest of the femur. The other muscles all insert on the trochanteric fossa lying medial to the greater trochanter.
Gluteal region
Gluteal muscles
Gluteus maximus: inserts to gluteal tuberosity of the femur and iliotibial tract
Gluteus medius: attach to lateral greater trochanter
Gluteus minimis: attach to anterior greater trochanter
All extend and abduct the hip
Deep lateral hip rotators
Piriformis
Gemelli
Obturator internus
Quadratus femoris
Nerves
Superior gluteal nerve (L4,L5, S1)
Gluteus medius
Gluteus minimis
Tensor fascia lata
Inferior gluteal nerve (L5, S1, S2) Gluteus maximus
Damage to the superior gluteal nerve will result in the patient developing a Trendelenberg gait. Affected patients are unable to abduct the thigh at the hip joint. During the stance phase, the weakened abductor muscles allow the pelvis to tilt down on the opposite side. To compensate, the trunk lurches to the weakened side to attempt to maintain a level pelvis throughout the gait cycle. The pelvis sags on the opposite side of the lesioned superior gluteal nerve.
Which of the following nerves is responsible for innervation of the triceps muscle?
Radial
Ulnar
Axillary
Median
None of the above
To remember nerve roots and their reflexes:
1-2 Ankle (S1-S2)
3-4 Knee (L3-L4)
5-6 Biceps (C5-C6)
7-8 Triceps (C7-C8)
The radial nerve innervates all three heads of triceps, with a separate branch to each head.
Triceps
Origin
Long head- infraglenoid tubercle of the scapula.
Lateral head- dorsal surface of the humerus, lateral and proximal to the groove of the radial nerve
Medial head- posterior surface of the humerus on the inferomedial side of the radial groove and both of the intermuscular septae
Insertion
Olecranon process of the ulna. Here the olecranon bursa is between the triceps tendon and olecranon.
Some fibres insert to the deep fascia of the forearm, posterior capsule of the elbow (preventing the capsule from being trapped between olecranon and olecranon fossa during extension)
Innervation Radial nerve
Blood supply Profunda brachii artery
Action Elbow extension. The long head can adduct the humerus and extend it from a flexed position
Relations The radial nerve and profunda brachii vessels lie between the lateral and medial heads
A 32 year old man requires venous access for chemotherapy for acute myeloid leukaemia. What is the best option?
Groshong line
Internal jugular central line (triple lumen)
14 G cannula in the dorsum of the hand
18 G cannula in the non dominant hand
Sub cuticular catheter
Groshong lines and Hickman lines are similar and knowledge of these systems is needed because surgeons are often called upon to either insert or remove them. Chemotherapy for AML requires long term therapy and multiple blood tests therefore an indwelling device is preferable.
Intravenous access
Venous access
A number of routes for establishing venous access are available.
Peripheral venous cannula
Easy to insert with minimal morbidity. Wide lumen cannulae can provide rapid fluid infusions. When properly managed infections may be promptly identified and the cannula easily re sited. Problems relate to their peripheral sites and they are unsuitable for the administration of vaso active drugs, such as inotropes and irritant drugs such as TPN (except in the very short term setting).
Central lines
Insertion is more difficult and most operators and NICE advocate the use of ultra sound. Coagulopathies may lead to haemorrhage following iatrogenic arterial injury. Femoral lines are easier to insert and iatrogenic injuries easier to manage in this site however they are prone to high infection rates. Internal jugular route is preferred. They have multiple lumens allowing for administration of multiple infusions. The lumens are relatively narrow and thus they do not allow particularly rapid rates of infusion.
Intraosseous access
This is typically undertaken at the anteromedial aspect of the proximal tibia and provides access to the marrow cavity and circulatory system. Although traditionally preferred in paediatric practice they may be used in adults and a wide range of fluids can be infused using these devices.
Tunneled lines
Tunneled lines such as Groshong and Hickman lines are popular devices for patients with long term therapeutic requirements. These devices are usually inserted using ultrasound guidance into the internal jugular vein and then tunneled under the skin. A cuff of woven material is sited near the end and helps to anchor the device into the tissues. These cuffs require formal dissection to allow the device to be removed. Tunneled lines can be linked to injection ports that are located under the skin. These are especially popular in paediatric practice.
Peripherally inserted central cannula
Referred to as PICC lines, these are popular methods for establishing central venous access. Because they are inserted peripherally they are less prone to major complications relating to device insertion than conventional central lines.
A 73 year old lady presents with a femoral hernia. Which of the following structures forms the lateral wall of the femoral canal?
Pubic tubercle
Femoral vein
Femoral artery
Conjoint tendon
Femoral nerve
The canal exists to allow for the physiological expansion of the femoral vein, which lies lateral to it.
Femoral canal
The femoral canal lies at the medial aspect of the femoral sheath. The femoral sheath is a fascial tunnel containing both the femoral artery laterally and femoral vein medially. The canal lies medial to the vein.
Borders of the femoral canal
Laterally Femoral vein
Medially Lacunar ligament
Anteriorly Inguinal ligament
Posteriorly Pectineal ligament
Contents
Lymphatic vessels
Cloquet’s lymph node
Physiological significance
Allows the femoral vein to expand to allow for increased venous return from the lower limbs.
Pathological significance
As a potential space, it is the site of femoral hernias. The relatively tight neck places these at high risk of strangulation.
A 77 year old man with symptoms of intermittent claudication is due to have his ankle brachial pressure indices measured. The dorsalis pedis artery is impalpable. Which of the following tendinous structures lies medial to it, that may facilitate its identification?
Extensor digitorum longus tendon
Peroneus tertius tendon
Extensor hallucis longus tendon
Extensor digitorum brevis tendon
Flexor digitorum longus tendon
The extensor hallucis longus tendon lies medial to the dorsalis pedis artery.
Foot- anatomy
Arches of the foot
The foot is conventionally considered to have two arches.
The longitudinal arch is higher on the medial than on the lateral side. The posterior part of the calcaneum forms a posterior pillar to support the arch. The lateral part of this structure passes via the cuboid bone and the lateral two metatarsal bones. The medial part of this structure is more important. The head of the talus marks the summit of this arch, located between the sustentaculum tali and the navicular bone. The anterior pillar of the medial arch is composed of the navicular bone, the three cuneiforms and the medial three metatarsal bones.
The transverse arch is situated on the anterior part of the tarsus and the posterior part of the metatarsus. The cuneiforms and metatarsal bases narrow inferiorly, which contributes to the shape of the arch.
Intertarsal joints
Sub talar joint Formed by the cylindrical facet on the lower surface of the body of the talus and the posterior facet on the upper surface of the calcaneus. The facet on the talus is concave anteroposteriorly, the other is convex. The synovial cavity of this joint does not communicate with any other joint.
Talocalcaneonavicular joint The anterior part of the socket is formed by the concave articular surface of the navicular bone, posteriorly by the upper surface of the sustentaculum tali. The talus sits within this socket
Calcaneocuboid joint Highest point in the lateral part of the longitudinal arch. The lower aspect of this joint is reinforced by the long plantar and plantar calcaneocuboid ligaments.
Transverse tarsal joint The talocalcaneonavicular joint and the calcaneocuboid joint extend across the tarsus in an irregular transverse plane, between the talus and calcaneus behind and the navicular and cuboid bones in front. This plane is termed the transverse tarsal joint.
Cuneonavicular joint Formed between the convex anterior surface of the navicular bone and the concave surface of the the posterior ends of the three cuneiforms.
Intercuneiform joints Between the three cuneiform bones.
Cuneocuboid joint Between the circular facets on the lateral cuneiform bone and the cuboid. This joint contributes to the tarsal part of the transverse arch.
A detailed knowledge of the joints is not required for MRCS Part A. However, the contribution they play to the overall structure of the foot should be appreciated
Nerves in the foot
Lateral plantar nerve
Passes anterolaterally towards the base of the 5th metatarsal between flexor digitorum brevis and flexor accessorius. On the medial aspect of the lateral plantar artery. At the base of the 5th metatarsal it splits into superficial and deep branches.
Medial plantar nerve
Passes forwards with the medial plantar artery under the cover of the flexor retinaculum to the interval between abductor hallucis and flexor digitorum brevis on the sole of the foot.
Plantar arteries
Arise under the cover of the flexor retinaculum, midway between the tip of the medial malleolus and the most prominent part of the medial side of the heel.
Medial plantar artery. Passes forwards medial to medial plantar nerve in the space between abductor hallucis and flexor digitorum brevis.Ends by uniting with a branch of the 1st plantar metatarsal artery.
Lateral plantar artery. Runs obliquely across the sole of the foot. It lies lateral to the lateral plantar nerve. At the base of the 5th metatarsal bone it arches medially across the foot on the metatarsals
Dorsalis pedis artery
This vessel is a direct continuation of the anterior tibial artery. It commences on the front of the ankle joint and runs to the proximal end of the first metatarsal space. Here is gives off the arcuate artery and continues forwards as the first dorsal metatarsal artery. It is accompanied by two veins throughout its length. It is crossed by the extensor hallucis brevis
Considering the pituitary gland, which of the following is false?
The anterior pituitary secretes thyroid stimulating hormone
The anterior pituitary develops from Rathkes pouch
Patients with craniopharyngioma may develop bi temporal hemianopia
The pituitary is in direct contact with the optic chiasm
The posterior pituitary releases oxytocin via a positive feedback loop
Although the optic chiasm is closely related to the pituitary, and craniopharyngiomas may compress this structure leading to bitemporal hemianopia, it is separated from the chiasm itself by a dural fold.
Pituitary Gland
The pituitary gland is located within the sella turcica within the sphenoid bone in the middle cranial fossa. It is covered by a dural fold and weighs around 0.5g. It is attached to the hypothalamus by the infundibulum. The anterior pituitary receives hormonal stimuli from the hypothalamus by way of the hypothalamo-pituitary portal system. It develops from a depression in the wall of the pharynx (Rathkes pouch).
Anterior pituitary hormones
Growth hormone
Thyroid stimulating hormone
ACTH
Prolactin
LH and FSH
Melanocyte releasing hormone
Posterior pituitary hormones
Oxytocin
Anti diuretic hormone
Which of the following nerves innervates the long head of the biceps femoris muscle?
Inferior gluteal nerve
Tibial division of sciatic nerve
Superior gluteal nerve
Common peroneal division of sciatic nerve
Obturator nerve
The short head of biceps femoris, which may occasionally be absent, is innervated by the common peroneal component of the sciatic nerve. The long head is innervated by the tibial division of the sciatic nerve.
Biceps femoris
The biceps femoris is one of the hamstring group of muscles located in the posterior upper thigh. It has two heads.
Long head
Origin Ischial tuberosity
Insertion Fibular head
Action Knee flexion, lateral rotation tibia, extension hip
Innervation Tibial division of sciatic nerve (L5, S1, S2)
Arterial supply Profunda femoris artery, inferior gluteal artery, and the superior muscular branches of popliteal artery
Short head
Origin Lateral lip of linea aspera, lateral supracondylar ridge of femur
Insertion Fibular head
Action Knee flexion, lateral rotation tibia
Innervation Common peroneal division of sciatic nerve (L5, S1, S2)
Arterial supply Profunda femoris artery, inferior gluteal art
During a neck dissection, a nerve is noted to pass posterior to the medial aspect of the first rib. Which of the nerves listed below is this most likely to be?
Medial cord of the brachial plexus
Long thoracic nerve
Nerve to subclavius
Medial pectoral nerve
Phrenic nerve
The key point is that the phrenic nerve runs posterior to the medial aspect of the first rib. Superiorly, it lies on the surface of scalenus anterior.
Phrenic nerve
Origin
C3,4,5
Supplies
Diaphragm, sensation central diaphragm and pericardium
Path
The phrenic nerve passes with the internal jugular vein across scalenus anterior. It passes superficial to prevertebral fascia of deep cervical fascia.
Left: crosses anterior to the 1st part of the subclavian artery.
Right: Anterior to scalenus anterior and crosses anterior to the 2nd part of the subclavian artery.
On both sides, the phrenic nerve runs posterior to the subclavian vein and posterior to the internal thoracic artery as it enters the thorax.
Right phrenic nerve
In the superior mediastinum: anterior to right vagus and laterally to superior vena cava
Middle mediastinum: right of pericardium
It passes over the right atrium to exit the diaphragm at T8
Left phrenic nerve
Passes lateral to the left subclavian artery, aortic arch and left ventricle
Passes anterior to the root of the lung
Pierces the diaphragm alone
Which of the following is a branch of the third part of the axillary artery?
Superior thoracic
Lateral thoracic
Dorsal scapular
Thoracoacromial
Posterior circumflex humeral
Posterior circumflex humeral
The other branches include:
Subscapular
Anterior circumflex humeral
The dorsal scapular artery arises from the third part of the subclavian artery in most cases
Axilla
Boundaries of the axilla
Medially Chest wall and Serratus anterior
Laterally Humeral head
Floor Subscapularis
Anterior aspect Lateral border of Pectoralis major
Fascia Clavipectoral fascia
Content:
Long thoracic nerve (of Bell) Derived from C5-C7 and passes behind the brachial plexus to enter the axilla. It lies on the medial chest wall and supplies serratus anterior. Its location puts it at risk during axillary surgery and damage will lead to winging of the scapula.
Thoracodorsal nerve and thoracodorsal trunk Innervate and vascularise latissimus dorsi.
Axillary vein Lies at the apex of the axilla, it is the continuation of the basilic vein. Becomes the subclavian vein at the outer border of the first rib.
Intercostobrachial nerves Traverse the axillary lymph nodes and are often divided during axillary surgery. They provide cutaneous sensation to the axillary skin.
Lymph nodes The axilla is the main site of lymphatic drainage for the breast.
Branches of Axillary Artery
FROM FIRST PART
Superior thoracic artery, a very small branch, originates near the subclavius, enters between the pectoralis major and minor muscles, and then these muscles go towards the medial wall of the axilla.
FROM SECOND PART
Thoraco-acromial artery (acromiothoracic artery) : emerges at the upper border of pectoralis minor, pierces clavipectoral fascia and soon breaks up into 4 branches:
(a) pectoral branch,
(b) deltoid branch,
(c) acromial branch, and
(d) clavicular branch.
These branches radiate at right angle to every other. The pectoral branch consists of pectoral muscles, deltoid branch, ends by joining anastomosis over the acromion, clavicular branch supplied sternoclavicular joint.
Lateral thoracic artery : emerges at and runs along the inferior border of pectoralis minor, supplying the branches to pectoralis major and minor and serratus anterior muscles. In the females, the lateral thoracic artery is large and gives significant supply to the breast via it’s lateral mammary branches.
FROM THIRD PART
Subscapular artery, the largest branch of axillary artery, runs along the lower border of the subscapularis and ends near the inferior angle of the scapula. It supplies a large branch, the circumflex scapular artery; which goes through upper triangular intermuscular space, winds round the lateral border of scapula to goes into infraspinous fossa. Additionally, it supplies numerous small branches.
Anterior circumflex humeral artery, a small branch, enters in front of surgical neck of humerus and anastomoses with the posterior circumflex humeral artery to create an arterial circle around the surgical neck of humerus. It supplies an ascending branch, which runs upwards into the intertubercular sulcus of humerus to supply the head of humerus and shoulder joint.
Posterior circumflex humeral artery, larger in relation to the anterior circumflex humeral artery, enters backwards, together with axillary nerve via the quadrangular intermuscular space, crosses the posterior aspect of surgical neck of humerus to anastomose with the anterior circumflex humeral artery. It supplies the deltoid muscle and shoulder joint.
Which statement relating to phaeochromocytoma is untrue?
They are tumours of chromaffin cells in the adrenal medulla.
They are bilateral in 10% of cases.
When located in an extra adrenal location have a higher incidence of malignancy.
May be associated with an elevated urinary VMA.
Up to 40% may have a blood pressure within the normal range.
Normotension is seen in around 10% cases. The remainder show a degree of hypertension.
Phaeochromocytoma and adrenal lesions
Phaeochromocytoma
Neuroendocrine tumour of the chromaffin cells of the adrenal medulla. Hypertension and hyperglycaemia are often found.
10% of cases are bilateral.
10% occur in children.
11% are malignant (higher when tumour is located outside the adrenal).
10% will not be hypertensive.
Familial cases are usually linked to the Multiple endocrine neoplasia syndromes (considered under its own heading).
Most tumours are unilateral (often right sided) and smaller than 10cm.
Diagnosis
Urine and plasma metanephrine levels.
CT and MRI scanning are both used to localise the lesion.
Treatment
Patients require medical therapy first. An irreversible alpha adrenoreceptor blocker should be given, although minority may prefer reversible blockade(1). Labetolol may be co-administered for cardiac chronotropic control. Isolated beta blockade should not be considered as it will lead to unopposed alpha activity.
These patients are often volume depleted and will often require moderate volumes of intra venous normal saline perioperatively.
Once medically optimised the phaeochromocytoma should be removed. Most adrenalectomies can now be performed using a laparoscopic approach(2). The adrenals are highly vascular structures and removal can be complicated by catastrophic haemorrhage in the hands of the inexperienced. This is particularly true of right sided resections where the IVC is perilously close. Should the IVC be damaged a laparotomy will be necessary and the defect enclosed within a Satinsky style vascular clamp and the defect closed with prolene sutures. Attempting to interfere with the IVC using any instruments other than vascular clamps will result in vessel trauma and make a bad situation much worse.
Incidental adrenal lesions
Adrenal lesions may be identified on CT scanning performed for other reasons(3). Factors suggesting benign disease on CT include(4):
Size less than 3cm
Homogeneous texture
Lipid rich tissue
Thin wall to lesion
All patients with incidental lesions should be managed jointly with an endocrinologist and full work up as described above. Patients with functioning lesions or those with adverse radiological features (Particularly size >3cm) should proceed to surgery.
A 67 year old man is investigated for biliary colic and a 4.8 cm abdominal aortic aneurysm is identified. Which of the following statements relating to this condition is untrue?
The wall will be composed of dense fibrous tissue only
The majority are located inferior to the renal arteries
They occur most often in current or former smokers
He should initially be managed by a process of active surveillance
Aortoduodenal fistula is a recognised complication following repair.
They are true aneurysms and have all 3 layers of arterial wall.
Abdominal aorta aneurysm
- Abdominal aortic aneurysms are a common problem in vascular surgery.
They may occur as either true or false aneurysm. With the former all 3 layers of the arterial wall are involved, in the latter only a single layer of fibrous tissue forms the aneurysm wall.
True abdominal aortic aneurysms have an approximate incidence of 0.06 per 1000 people. They are commonest in elderly men and for this reason the UK is now introducing the aneurysm screening program with the aim of performing an abdominal aortic ultrasound measurement in all men aged 65 years.
Causes
Several different groups of patients suffer from aneurysmal disease.
The commonest group is those who suffer from standard arterial disease, i.e. Those who are hypertensive and have been or are smokers.
Other patients such as those suffering from connective tissue diseases such as Marfan’s may also develop aneurysms. In patients with abdominal aortic aneurysms the extracellular matrix becomes disrupted with a change in the balance of collagen and elastic fibres.
Management
Most abdominal aortic aneurysms are an incidental finding.
Symptoms most often relate to rupture or impending rupture.
20% rupture anteriorly into the peritoneal cavity. Very poor prognosis.
80% rupture posteriorly into the retroperitoneal space
The risk of rupture is related to aneurysm size, only 2% of aneurysms measuring less than 4cm in diameter will rupture over a 5 year period. This contrasts with 75% of aneurysms measuring over 7cm in diameter.
This is well explained by Laplaces’ law which relates size to transmural pressure.
For this reason most vascular surgeons will subject patients with an aneurysm size of 5cm or greater to CT scanning of the chest, abdomen and pelvis with the aim of delineating anatomy and planning treatment. Depending upon co-morbidities, surgery is generally offered once the aneurysm is between 5.5cm and 6cm.
Indications for surgery
Symptomatic aneurysms (80% annual mortality if untreated)
Increasing size above 5.5cm if asymptomatic
Rupture (100% mortality without surgery)
Surgical procedures
Abdominal aortic aneurysm repair
Procedure:
GA
Invasive monitoring (A-line, CVP, catheter)
Incision: Midline or transverse
Bowel and distal duodenum mobilised to access aorta.
Aneurysm neck and base dissected out and prepared for cross clamp
Systemic heparinisation
Cross clamp (proximal first)
Longitudinal aortotomy
Atherectomy
Deal with back bleeding from lumbar vessels and inferior mesenteric artery
Insert graft either tube or bifurcated depending upon anatomy
Suture using Prolene (3/0 for proximal , distal anastomosis suture varies according to site)
Clamps off: End tidal CO2 will rise owing to effects of reperfusion, at this point major risk of myocardial events.
Haemostasis
Closure of aneurysm sac to minimise risk of aorto-enteric fistula
Closure: Loop 1 PDS or Prolene to abdominal wall
Skin- surgeons preference
Post operatively:
ITU (Almost all)
Greatest risk of complications following emergency repair
Complications: Embolic- gut and foot infarcts
Cardiac - owing to premorbid states, re-perfusion injury and effects of cross clamp
Wound problems
Later risks related to graft- infection and aorto-enteric fistula
Special groups
Supra renal AAA
These patients will require a supra renal clamp and this carries a far higher risk of complications and risk of renal failure.
Ruptured AAA
Pre-operatively the management depends upon haemodynamic instability. In patients with symptoms of rupture (typical pain, haemodynamic compromise and risk factors) then ideally prompt laparotomy. In those with vague symptoms and haemodynamic stability the ideal test is CT scan to determine whether rupture has occurred or not. Most common rupture site is retroperitoneal 80%. These patients will tend to develop retroperitoneal haematoma. This can be disrupted if Bp is allowed to rise too high so aim for Bp 100mmHg.
Operative details are similar to elective repair although surgery should be swift, blind rushing often makes the situation worse. Plunging vascular clamps blindly into a pool of blood at the aneurysm neck carries the risk of injury the vena cava that these patients do not withstand. Occasionally a supracoeliac clamp is needed to effect temporary control, although leaving this applied for more than 20 minutes tends to carry a dismal outcome.
EVAR
Increasingly patients are now being offered endovascular aortic aneurysm repair. This is undertaken by surgeons and radiologists working jointly. The morphology of the aneurysm is important and not all are suitable. Here is a typical list of those features favoring a suitable aneurysm:
Long neck
Straight iliac vessels
Healthy groin vessels
Clearly few AAA patients possess the above and compromise has to be made. The use of fenestrated grafts can allow supra renal AAA to be treated.
Procedure:
GA
Radiology or theatre
Bilateral groin incisions
Common femoral artery dissected out
Heparinisation
Arteriotomy and insertion of guide wire
Dilation of arteriotomy
Insertion of EVAR Device
Once in satisfactory position it is released
Arteriotomy closed once check angiogram shows good position and no endoleak
Complications:
Endoleaks depending upon site are either Type I or 2. These may necessitate re-intervention and all EVAR patients require follow up . Details are not needed for MRCS.
A 28 year old man is stabbed outside a nightclub in the upper arm. The median nerve is transected. Which of the following muscles will demonstrate impaired function as a result?
Palmaris brevis
Second and third interossei
Adductor pollicis
Abductor pollicis longus
Abductor pollicis brevis
Palmaris brevis - Ulnar nerve
Palmar interossei- Ulnar nerve
Adductor pollicis - Ulnar nerve
Abductor pollicis longus - Posterior interosseous nerve
Abductor pollicis brevis - Median nerve
The median nerve innervates all the short muscles of the thumb except the adductor and the deep head of the short flexor. Palmaris and the interossei are innervated by the ulnar nerve.
Median nerve
The median nerve is formed by the union of a lateral and medial root respectively from the lateral (C5,6,7) and medial (C8 and T1) cords of the brachial plexus; the medial root passes anterior to the third part of the axillary artery. The nerve descends lateral to the brachial artery, crosses to its medial side (usually passing anterior to the artery). It passes deep to the bicipital aponeurosis and the median cubital vein at the elbow.
It passes between the two heads of the pronator teres muscle, and runs on the deep surface of flexor digitorum superficialis (within its fascial sheath).
Near the wrist it becomes superficial between the tendons of flexor digitorum superficialis and flexor carpi radialis, deep to palmaris longus tendon. It passes deep to the flexor retinaculum to enter the palm, but lies anterior to the long flexor tendons within the carpal tunnel.
Branches
Region Branch
Upper arm No branches, although the nerve commonly communicates with the musculocutaneous nerve
Forearm Pronator teres
Pronator quadratus
Flexor carpi radialis
Palmaris longus
Flexor digitorum superficialis
Flexor pollicis longus
Flexor digitorum profundus (only the radial half)
Distal forearm Palmar cutaneous branch
Hand (Motor) Motor supply (LOAF)
Lateral 2 lumbricals
Opponens pollicis
Abductor pollicis brevis
Flexor pollicis brevis
Hand (Sensory)
Over thumb and lateral 2 ½ fingers
On the palmar aspect this projects proximally, on the dorsal aspect only the distal regions are innervated with the radial nerve providing the more proximal cutaneous innervation.
Patterns of damage
Damage at wrist
e.g. carpal tunnel syndrome
paralysis and wasting of thenar eminence muscles and opponens pollicis (ape hand deformity)
sensory loss to palmar aspect of lateral (radial) 2 ½ fingers
Damage at elbow, as above plus:
unable to pronate forearm
weak wrist flexion
ulnar deviation of wrist
Anterior interosseous nerve (branch of median nerve)
leaves just below the elbow
results in loss of pronation of forearm and weakness of long flexors of thumb and index finger
A 68 year old man presents to the plastics team with severe burns to his hands. He is not distressed by the burns. He has bilateral charcot joints. On examination; there is loss of pain and temperature sensation of the upper limbs.
A. Osteomyelitis
B. Potts disease of the spine
C. Scheuermanns disease
D. Transverse myelitis
E. Tabesdorsalis
F. Subacutedegenerationofthecord G. Brown-Sequard syndrome
H. Syringomyelia
I. Epiduralhaematoma
The correct answer is Syringomyelia
This patient has syringomyelia which selectively affects the spinotholamic tracts. Syringomyelia is a disorder in which a cystic cavity forms within the spinal cord. The commonest variant is the Arnold- Chiari malformation in which the cavity connects with a congenital malformation affecting the cerebellum. Acquired forms of the condition may occur as a result of previous meningitis, surgery or tumours. Many neurological manifestations have been reported, although the classical variety spares the dorsal columns and medial lemniscus and affecting only the spinothalamic tract with loss of pain and temperature sensation. The bilateral distribution of this patients symptoms would therefore favor syringomyelia over SCID or Brown Sequard syndrome. Osteomyelitis would tend to present with back pain and fever in addition to any neurological signs. Epidural haematoma large enough to produce neurological impairment will usually have motor symptoms in addition to any selective sensory loss, and the history is usually shorter.
A 24 year old man presents with localised spinal pain over 2 months which is worsened on movement. He is known to be an IVDU. He has no history suggestive of tuberculosis. The pain is now excruciating at rest and not improving with analgesia. He has a temperature of 39 oC. Which is the most likely diagnosis for the scenario given.
A. Osteomyelitis
B. Potts disease of the spine
C. Scheuermanns disease
D. Transverse myelitis
E. Tabesdorsalis
F. Subacutedegenerationofthecord G. Brown-Sequard syndrome
H. Syringomyelia
I. Epiduralhaematoma
The correct answer is Osteomyelitis
In an IVDU with back pain and pyrexia have a high suspicion for osteomylelitis. The most likely organism is staph aureus and the cervical spine is the most common region affected. TB tends to affect the thoracic spine and in other causes of osteomyelitis the lumbar spine is affected.
A 22 year man is shot in the back, in the lumbar region. He has increased tone and hyper-reflexia of his right leg. He cannot feel his left leg. Which is the most likely diagnosis for the scenario given.
A. Osteomyelitis
B. Potts disease of the spine
C. Scheuermanns disease
D. Transverse myelitis
E. Tabesdorsalis
F. Subacute degeneration of the cord
G. Brown-Sequard syndrome
H. Syringomyelia
I. Epidural haematoma
The correct answer is Brown-Sequard syndrome
Theme from January 2012 exam
Brown -Sequard syndrome is caused by hemisection of the spinal cord. It may result from stab injuries or lateral vertebral fractures. It results in ipsilateral paralysis (pyramidal tract) , and also loss of proprioception and fine discrimination (dorsal columns). Pain and temperature sensation are lost on the contra-lateral side. This is because the fibres of the spinothalamic tract have decussated below the level of the cord transection.
Spinal disorders
Dorsal column lesion
Spinothalamic tract lesion Central cord lesion
Infarction spinal cord
Loss vibration and proprioception Tabes dorsalis, SACD
Loss of pain, sensation and temperature Flaccid paralysis of the upper limbs
Dorsal column signs (loss of proprioception and fine discrimination)
Osteomyelitis
Normally progressive
Staph aureus in IVDU, normally cervical region affected Fungal infections in immunocompromised
Thoracic region affected in TB
Cord compression
UMN signs Malignancy Haematoma Fracture
Brown-sequard syndrome
Hemisection of the spinal cord
Ipsilateral paralysis
Ipsilateral loss of proprioception and fine discrimination Contralateral loss of pain and temperature
A 24 year old man is brought to the emergency department have suffered a crush injury to his forearm. Assessment demonstrates that the arm is tender, red and swollen. There is clinical evidence of an ulnar fracture and the patient cannot move their fingers. Which is the most appropriate course of action?
Application of an external fixation device Closed reduction
Debridement
Discharge and review in fracture clinic Fasciotomy
Theme from April 2012
The combination of a crush injury, limb swelling and inability to move digits should raise suspicion of a compartment syndrome that will require a fasciotomy. Paralysis is a very late sign.
Compartment syndrome
This is a particular complication that may occur following fractures (or following ischaemia re-perfusion injury in vascular patients). It is characterised by raised pressure within a closed anatomical space.
The raised pressure within the compartment will eventually compromise tissue perfusion resulting in necrosis. The two main fractures carrying this complication include supracondylar fractures and tibial shaft injuries.
Symptoms and signs
Pain, especially on movement (even passive)
Parasthesiae
Pallor may be present
Arterial pulsation may still be felt as the necrosis occurs as a result of microvascular compromise Paralysis of the muscle group may occur
Diagnosis
Is made by measurement of intracompartmental pressure measurements. Pressures in excess of 20mmHg are abnormal and >40mmHg is diagnostic.
Treatment
This is essentially prompt and extensive fasciotomies
In the lower limb the deep muscles may be inadequately decompressed by the inexperienced operator when smaller incisions are performed Myoglobinuria may occur following fasciotomy and result in renal failure and for this reason these patients require aggressive IV fluids
Where muscle groups are frankly necrotic at fasciotomy they should be debrided and amputation may have to be considered
Death of muscle groups may occur within 4-6 hours
Which of the following transplants is most susceptible to donor- recipient HLA mismatches?
Autologous skin graft
Renal allograft
Liver allograft
Corneal allograft
Cardiac valve allograft
Autologous transplant- same individual (genetically identical)
Allograft - Genetically different
The kidney is highly susceptible to HLA mismatches and hyperacute rejection may occur in patients with IgG anti HLA Class I antibodies. The liver is at far lower risk of rejection of this nature. Although the heart is sensitive to HLA mismatches this is less than the kidney. Cardiac valves and the cornea incite little immunological response.
Organ Transplant
A number of different organ and tissue transplants are now available. In many cases an allograft is performed, where an organ is transplanted from one individual to another. Allografts will elicit an immune response and this is one of the main reasons for organ rejection.
Graft rejection occurs because allografts have allelic differences at genes that code immunohistocompatability complex genes. The main antigens that give rise to rejection are:
ABO blood group
Human leucocyte antigens (HLA)
Minor histocompatability antigens
ABO Matching
ABO incompatibility will result in early organ rejection (hyperacute) because of pre existing antibodies to other groups. Group O donors can give organs to any type of ABO recipient whereas group AB donor can only donate to AB recipient.
HLA System
The four most important HLA alleles are:
HLA A
HLA B
HLA C
HLA DR
An ideal organ match would be one in which all 8 alleles are matched (remember 2 from each parent, four each = 8 alleles). Modern immunosuppressive regimes help to manage the potential rejection due to HLA mismatching. However, the greater the number of mismatches the worse the long term outcome will be. T lymphocytes will recognise antigens bound to HLA molecules and will then become activated. Clonal expansion then occurs with a response directed against that antigen.
Types of organ rejection
Hyperacute. This occurs immediately through presence of pre formed antibodies (such as ABO incompatibility).
Acute. Occurs during the first 6 months and is usually T cell mediated. Usually tissue infiltrates and vascular lesions.
Chronic. Occurs after the first 6 months. Vascular changes predominate.
Hyperacute
Renal transplants at greatest risk and liver transplants at least risk. Although ABO incompatibility and HLA Class I incompatible transplants will all fare worse in long term.
Acute
All organs may undergo acute rejection. Mononuclear cell infiltrates predominate. All types of transplanted organ are susceptible and it may occur in up to 50% cases.
Chronic
Again all transplants with HLA mismatch may suffer this fate. Previous acute rejections and other immunosensitising events all increase the risk. Vascular changes are most prominent with myointimal proliferation leading to organ ischaemia. Organ specific changes are also seen such as loss of acinar cells in pancreas transplants and rapidly progressive coronary artery disease in cardiac transplants.
Surgical overview-Renal transplantation
A brief overview of the steps involved in renal transplantation is given.
Patients with end stage renal failure who are dialysis dependent or likely to become so in the immediate future are considered for transplant. Exclusion criteria include; active malignancy, old age (due to limited organ availability). Patients are medically optimised.
Donor kidneys, these may be taken from live related donors and close family, members may have less HLA mismatch than members of the general population. Laparoscopic donor nephrectomy further minimises the operative morbidity for the donor. Other organs are typically taken from brain dead or dying patients who have a cardiac arrest and in whom resuscitation is futile. The key event is to minimise the warm ischaemic time in the donor phase.
The kidney once removed is usually prepared on the bench in theatre by the transplant surgeon immediately prior to implantation and factors such as accessory renal arteries and vessel length are assessed and managed.
For first time recipients the operation is performed under general anaesthesia. A Rutherford-Morison incision is made on the preferred side. This provides excellent extraperitoneal access to the iliac vessels. The external iliac artery and vein are dissected out and following systemic heparinisation are cross clamped. The vein and artery are anastamosed to the iliacs and the clamps removed. The ureter is then implanted into the bladder and a stent is usually placed to maintain patency. The wounds are then closed and the patient recovered from surgery.
In the immediate phase a common problem encountered in cadaveric kidneys is acute tubular necrosis and this tends to resolve.
Graft survival times from cadaveric donors are typically of the order of 9 years and monozygotic twin transplant (live donor) may survive as long as 25 years.
Which of the following statements relating to menisceal tears is false?
The medial meniscus is most often affected
True locking of the knee joint may occur
Most established tears will heal with conservative management
In the chronic setting there is typically little to find on examination if the knee is not locked
An arthroscopic approach may be used to treat most lesions
Menisci have no nerve or blood supply and thus heal poorly. Established tears with associated symptoms are best managed by arthroscopic menisectomy.
A 25 year old man is undergoing respiratory spirometry. He takes a maximal inspiration and maximally exhales. Which of the following measurements will best illustrate this process?
Functional residual capacity
Vital capacity
Inspiratory capacity
Maximum voluntary ventilation
Tidal volume
Vital capacity
The maximum voluntary ventilation is the maximal ventilation over the course of 1 minute.
A 58 year old male is referred to endocrinology clinic for a parathyroidectomy by the F1 in medicine. His corrected calcium is 2.85 (2.2-2.6), PTH 7.5 (3-7) and 24h urinary calcium is 1.5 (2.5-7.5). What is the diagnosis?
Primary hyperparathyroidism
Secondary hyperparathyroidism
Tertiary hyperparathyroidism
Familial hypocalciuric hypercalcaemia
Hypercalacemia associated with malignancy
This F1 should have spoken to his senior. This patient has familial hypocalciuric hypercalcaemia, which requires no further action. A calcium to creatinine clearance ratio of <0.01 will confirm this diagnosis.
A 65 year old lady presents with a lesion affecting her right breast. On examination she has a weeping, crusting lesion overlying the right nipple, the areolar region is not involved. There is no palpable mass lesion in the breast, there is a palpable axillary lymph node. The patient’s general practitioner has tried treating the lesion with 1% hydrocortisone cream, with no success. What is the most likely diagnosis?
Infection with Staphylococcus aureus
Pagets disease of the nipple
Phylloides tumour
Nipple eczema
Basal cell carcinoma
A weeping, crusty lesion such as this is most likely to represent Pagets disease of the nipple (especially since the areolar region is spared). Although no mass lesion is palpable, a proportion of patients will still have an underlying invasive malignancy (hence the lymphadenopathy).
Pagets disease differs from eczema of the nipple in that it involves the nipple primarily and only latterly spreads to the areolar (the opposite occurs in eczema).
Which of the following statements relating to quadratus lumborum is false?
Causes flexion of the thoracic spine
Causes the rib cage to be pulled down
Innervated by anterior primary rami of T12 and L1-3
Attached to the iliac crest
Inserts into the 12th rib
Quadratus lumborum
Origin: Medial aspect of iliac crest and iliolumbar ligament
Insertion: 12th rib
Action: Pulls the rib cage inferiorly. Lateral flexion.
Nerve supply: Anterior primary rami of T12 and L1-3
The rectus abdominis causes flexion of the thoracic spine and therefore the statement suggesting that quaratus lumborum does so is incorrect.
A 76 year old man presents with a painful left leg. The pain began suddenly and with no previous history. On examination he has a white left leg with no palpable femoral pulse and loss of sensation. The pulses in the contra lateral limb are normal. It is now three hours since the pain first started.
A.Primary amputation
B.Transfemoral embolectomy with prophylactic fasciotomy
C.Transpopliteal embolectomy without prophylactic fasciotomy
D.Transfemoral embolectomy without prophylactic fasciotomy
E.Transpopliteal embolectomy with prophylactic fasciotomy
F.Angiogram
G.Systemic heparin infusion
H.Peripheral thrombolysis
Transfemoral embolectomy without prophylactic fasciotomy
A limb which is acutely ischaemic and with normal contralateral pulses normally indicates an acute embolus. Whilst intra arterial thrombolysis may be an option there is a reasonable argument for immediate surgery. A fasciotomy is unlikely to be required.
A 56 year old man presents with a painful left leg. The pain has been present for the past 8 hours although it has also been present (though less severe) about a week ago. At that stage he noted that his hallux had turned blue. This resolved spontaneously. On examination he has a weakly palpable femoral pulse on the affected side but no pulses palpable distal to this. His sensation is mildly impaired.
A.Primary amputation
B.Transfemoral embolectomy with prophylactic fasciotomy
C.Transpopliteal embolectomy without prophylactic fasciotomy
D.Transfemoral embolectomy without prophylactic fasciotomy
E.Transpopliteal embolectomy with prophylactic fasciotomy
F.Angiogram
G.Systemic heparin infusion
H.Peripheral thrombolysis
The correct answer is Angiogram
The history favors a more chronic process and the great toe cyanotic spell may be indicative of previous embolism from pathology such as an aneurysm. In the ideal scenario a duplex scan would be performed. However, an angiogram would probably supply sufficient information and allow appropriate endovascular therapy.
A 78 year old lady is found by carers with a severely painful left leg. On examination she has no palpable pulses and the limb is cold, insensate and mottled. The mottling does not blanch with pressure.
A.Primary amputation
B.Transfemoral embolectomy with prophylactic fasciotomy
C.Transpopliteal embolectomy without prophylactic fasciotomy
D.Transfemoral embolectomy without prophylactic fasciotomy
E.Transpopliteal embolectomy with prophylactic fasciotomy
F.Angiogram
G.Systemic heparin infusion
H.Peripheral thrombolysis
Primary amputation
This is an unsalvagable limb and is best amputated primarily.
Which of the following does not decrease the functional residual capacity?
Obesity
Pulmonary fibrosis
Muscle relaxants
Laparoscopic surgery
Upright position
Increased FRC:
Erect position
Emphysema
Asthma
Decreased FRC:
Pulmonary fibrosis
Laparoscopic surgery
Obesity
Abdominal swelling
Muscle relaxants
When the patient is upright the diaphragm and abdominal organs put less pressure on the lung bases, allowing for an increase in the functional residual capacity (FRC). Other causes of increased FRC include:
Emphysema
Asthma
In addition to those listed above, causes of reduced FRC include:
Abdominal swelling
Pulmonary oedema
Reduced muscle tone of the diaphragm
Age
A 23 year old climber falls and fractures his humerus. The surgeons decide upon a posterior approach to the middle third of the bone. Which of the following nerves is at greatest risk in this approach?
Ulnar
Antebrachial
Musculocutaneous
Radial
Intercostobrachial
The radial nerve wraps around the humerus and may be injured during a posterior approach. An IM nail may be preferred as it avoids the complex dissection needed for direct bone exposure.
Which of the following pathological explanations best describes the initial pathological processes occurring in an abdominal aortic aneurysm in an otherwise well 65 year old, hypertensive male?
Loss of elastic fibres from the adventitia
Loss of collagen from the adventitia
Loss of collagen from the media
Loss of elastic fibres from the media
Decreased matrix metalloproteinases in the adventitia
Loss of elastic fibres from the media
In established aneurysmal disease there is dilation of all layers of the arterial wall and loss of both elastin and collagen. The primary event is loss of elastic fibres with subsequent degradation of collagen fibres.
In medical statistics, which of the following does a p value of 0.04 represent?
Risk of type 1 Error
Risk of type 2 Error
Size of power of the study
Sample size
Number of degrees of freedom
P values are related to the significance levels of a statistical test and therefore are in effect measuring the risk of a type 1 error.
Which of the following is the main site of dehydroepiandrosterone release?
Posterior pituitary
Zona reticularis of the adrenal gland
Zona glomerulosa of the adrenal gland
Juxtaglomerular apparatus of the kidney
Zona fasciculata of the adrenal gland
The zona fasciculata of the adrenal gland is the primary site responsible for the production of dehydroepiandrosterone (DHEA). This layer of the adrenal cortex synthesizes DHEA, which is an important precursor for sex hormones. On the other hand, the zona glomerulosa is primarily involved in the production of aldosterone, a mineralocorticoid hormone that regulates sodium and potassium balance in the body. So, when it comes to DHEA, think zona fasciculata!
A 34 year old lady undergoes an elective cholecystectomy for attacks of recurrent cholecystitis due to gallstones. Microscopic assessment of the gallbladder is most likely to show which of the following?
Dysplasia of the fundus
Widespread necrosis
Aschoff-Rokitansky sinuses
Metaplasia of the fundus
None of the above
Aschoff-Rokitansky sinuses are the result of hyperplasia and herniation of epithelial cells through the fibromuscular layer of the gallbladder wall. They may be macroscopic or microscopic. Ashoff-Rokitansky sinuses may be identified in cases of chronic cholecystitis and gallstones. Although gallstones may predispose to the development of gallbladder cancer the actual incidence of dysplasia and metaplastic change is rare. In the elective setting described above necrosis would be rare.
A 1-day-old baby girl is noted to become profoundly cyanotic whilst feeding and crying. A diagnosis of congenital heart disease is suspected. What is the most likely cause?
Transposition of the great arteries
Coarctation of the aorta
Patent ductus arteriosus
Hypoplastic left heart
Ventricular septal defect
Congenital heart disease
Cyanotic: TGA most common at birth, Fallot’s most common overall
Acyanotic: VSD most common cause
It is important to be aware of common congenital cardiac abnormalities. The main differentiating factor is whether the patient is cyanotic or acyanotic. In the neonate, TGA is the most common condition presenting with profound cyanosis.
The other options are causes of acyanotic congenital heart disease
A 15 year-old boy presents to the out-patient clinic with tiredness, recurrent throat and chest infections, and gradual loss of vision. Multiple x-rays show brittle bones with no differentiation between the cortex and the medulla.
A.Rickets
B.Craniocleidodysostosis
C.Achondroplasia
D.Scurvy
E.Pagets disease
F.Multiple myeloma
G.Osteogenesis imperfecta
H.Osteomalacia
I.Osteopetrosis
J.None of the above
Osteopetrosis
Osteopetrosis is an autosomal recessive condition. It is commonest in young adults. They may present with symptoms of anaemia or thrombocytopaenia due to decreased marrow space. Radiology reveals a lack of differentiation between the cortex and the medulla described as marble bone. These bones are very dense and brittle.
A 12 year-old boy who is small for his age presents to the clinic with poor muscular development and hyper-mobile fingers. His x-rays show multiple fractures of the long bones and irregular patches of ossification.
A.Rickets
B.Craniocleidodysostosis
C.Achondroplasia
D.Scurvy
E.Pagets disease
F.Multiple myeloma
G.Osteogenesis imperfecta
H.Osteomalacia
I.Osteopetrosis
J.None of the above
Osteogenesis imperfecta
Osteogenesis imperfecta is caused by defective osteoid formation due to congenital inability to produce adequate intercellular substances like osteoid, collagen and dentine. There is a failure of maturation of collagen in all the connective tissues.Radiology may show translucent bones, multiple fractures, particularly of the long bones, wormian bones (irregular patches of ossification) and a trefoil pelvis.
A 1 year-old is brought to the Emergency Department with a history of failure to thrive. On examination, the child is small for age and has a large head. X-ray shows a cupped appearance of the epiphysis of the wrist.
A.Rickets
B.Craniocleidodysostosis
C.Achondroplasia
D.Scurvy
E.Pagets disease
F.Multiple myeloma
G.Osteogenesis imperfecta
H.Osteomalacia
I.Osteopetrosis
J.None of the above
Rickets
Rickets is the childhood form of osteomalacia. It is due to the failure of the osteoid to ossify due to vitamin D deficiency. Symptoms start about the age of one. The child is small for age and there is a history of failure to thrive. Bony deformities include bowing of the femur and tibia, a large head, deformity of the chest wall with thickening of the costochondral junction (rickettary rosary), and a transverse sulcus in the chest caused by the pull of the diaphragm (Harrison’s sulcus). X- Rays show widening and cupping of the epiphysis of the long bones, most readily apparent in the wrist.
Which of the following are not true of Li-Fraumeni syndrome?
It consists of mutations to the p53 tumour suppressor gene
Is likely to be present in a teenager presenting with a liposarcoma
It has an autosomal dominant inheritance pattern
Affected individuals are unlikely to develop acute myeloid leukaemia
Adrenal malignancies are more common than in normal population
They are at high risk of developing leukaemia.
A 42 year old teacher is admitted with a fall. An x-ray confirms a fracture of the surgical neck of the humerus. Which nerve is at risk?
A.Median nerve
B.Ulnar nerve
C.Radial nerve
D.Posterior interosseous nerve
E.Anterior interosseous nerve
F.Musculocutaneous nerve
G.Axillary nerve
H.Brachial Trunks C5-6
I.Brachial trunks C6-7
J.Brachial Trunks C8-T1
Axillary nerve
The Axillary nerve winds around the bone at the neck of the humerus. The axillary nerve is also at risk during shoulder dislocation.
A 32 year old window cleaner is admitted after falling off the roof. He reports that he had slipped off the top of the roof and was able to cling onto the gutter for a few seconds. The patient has Horner’s syndrome.
A.Median nerve
B.Ulnar nerve
C.Radial nerve
D.Posterior interosseous nerve
E.Anterior interosseous nerve
F.Musculocutaneous nerve
G.Axillary nerve
H.Brachial Trunks C5-6
I.Brachial trunks C6-7
J.Brachial Trunks C8-T1
Brachial Trunks C8-T1
The patient has a Klumpke’s paralysis involving brachial trunks C8-T1. Classically there is weakness of the hand intrinsic muscles. Involvement of T1 may cause a Horner’s syndrome. It occurs as a result of traction injuries or during delivery.
A 32 year old rugby player is hit hard on the shoulder during a rough tackle. Clinically his arm is hanging loose on the side. It is pronated and medially rotated.
A.Median nerve
B.Ulnar nerve
C.Radial nerve
D.Posterior interosseous nerve
E.Anterior interosseous nerve
F.Musculocutaneous nerve
G.Axillary nerve
H.Brachial Trunks C5-6
I.Brachial trunks C6-7
J.Brachial Trunks C8-T1
Brachial Trunks C5-6
The patient has an Erb’s palsy involving brachial trunks C5-6.
A 24 year old motorist is involved in a road traffic accident in which he collides with the wall of a tunnel in a head on car crash, speed 85mph. He is wearing a seatbelt and the airbags have deployed. When rescuers arrive he is lucid and conscious and then dies suddenly.
A.Tension pneumothorax
B.Haemopericardium
C.Haemothorax
D.Aortic transection
E.Ruptured spleen
F.Duodeno-jejunal flexure disruption
G.Aorto iliac disruption
H.Ileo-colic junction disruption
Aortic transection
Aortic transections typically occur distal to the ligamentum arteriosum. A temporary haematoma may prevent the immediate death that usually occurs. This is a deceleration injury. A widened mediastinum may be seen on x-ray.
A 30 year old women is involved in a road traffic accident she is a passenger in a car involved in a head on collision with another vehicle. Her car is travelling at 60mph. She has been haemodynamically stable throughout with only minimal tachycardia. On examination she has marked abdominal tenderness and a large amount of intra abdominal fluid on CT scan
A.Tension pneumothorax
B.Haemopericardium
C.Haemothorax
D.Aortic transection
E.Ruptured spleen
F.Duodeno-jejunal flexure disruption
G.Aorto iliac disruption
H.Ileo-colic junction disruption
Duodeno-jejunal flexure disruption
This is another site of sudden deceleration injury. Given the large amount of free fluid, if it were blood, then a greater degree of haemodynamic instability would be expected.
A 17 year old boy is involved in a motorcycle accident in which he is thrown from his motorcycle. On admission he has distended neck veins and a weak pulse. The trachea is central.
A.Tension pneumothorax
B.Haemopericardium
C.Haemothorax
D.Aortic transection
E.Ruptured spleen
F.Duodeno-jejunal flexure disruption
G.Aorto iliac disruption
H.Ileo-colic junction disruption
Haemopericardium
This is most likely a cardiac tamponade produced by haemopericardium. As little as 100ml of blood may result in tamponade as the pericardial sac is not distensible. Diagnosis is suggested by muffled heart sounds, paradoxical pulse and jugular vein distension.
What is the typical stroke volume in a resting 70 Kg man?
10ml
150ml
125ml
45ml
70ml
70ml
Stroke volumes range from 55-100ml.
Which of the following statements relating to parathyroid neoplasms is incorrect?
15% of cases are due to parathyroid carcinoma
80% of cases are due to parathyroid adenomas
Parathyroid adenomas are often encapsulated
10% of parathyroid adenomas develop in ectopically located glands
85% of cases of primary hyperparathyroidism are due to solitary adenomas
Parathyroid carcinomas account for up to 5% of tumours. Adenomas are often encapsulated. Lesions that are fibrotic and densely adherent to the gland may be a carcinoma. 85% cases of primary hyperparathyroidism are due to a single adenoma and this is the reason some surgeons favour a focused parathyroidectomy.
A 20 year old girl presents with a thyroid cancer, she is otherwise well with no significant family history. On examination she has a nodule in the left lobe of the thyroid with a small discrete mass separate from the gland itself. Which of the following is the most likely cause?
Follicular carcinoma
Anaplastic carcinoma
Medullary carcinoma
Papillary carcinoma
B Cell Lymphoma
Papillary carcinoma is the most common subtype and may cause lymph node metastasis (mass separate from the gland itself) that is rare with follicular tumours. Anaplastic carcinoma would cause more local symptoms and would be rare in this age group.
What are the most likely effects of the release of vasopressin from the pituitary?
Vasoconstriction of the afferent glomerular arteriole
Increased permeability of the mesangial cells to glucose
Reduced permeability of the inner medullary portion of the collecting duct to urea
Increased secretion of aldosterone from the macula densa
Increased water permeability of the distal tubule cells of the kidney
Vasopressin, also known as antidiuretic hormone (ADH), is a crucial hormone released from the posterior pituitary gland. Let’s explore its effects:
Vasoconstriction of the Afferent Glomerular Arteriole: True. Vasopressin acts on the afferent arteriole of the glomerulus in the kidneys, causing vasoconstriction. This constriction reduces blood flow into the glomerulus, helping to regulate glomerular filtration rate (GFR) and maintain blood pressure1.
Increased Permeability of Mesangial Cells to Glucose: False. Vasopressin does not directly affect glucose permeability in mesangial cells.
Reduced Permeability of the Inner Medullary Portion of the Collecting Duct to Urea: True. Vasopressin increases the permeability of the collecting ducts in the kidney, allowing for greater reabsorption of water. This process concentrates urine and reduces urea loss2.
Increased Secretion of Aldosterone from the Macula Densa: False. Aldosterone is primarily regulated by the zona glomerulosa of the adrenal cortex, not vasopressin.
Increased Water Permeability of Distal Tubule Cells of the Kidney: True. Vasopressin acts on the distal tubules and collecting ducts to enhance water reabsorption. It promotes the insertion of aquaporin-2 channels in the tubular cells, allowing water to move from the tubules back into the bloodstream1.
In summary, vasopressin primarily affects water balance by regulating water reabsorption in the kidneys, leading to concentrated urine and maintaining blood pressure.
The most likely effect of vasopressin (antidiuretic hormone) is increased water permeability of the distal tubule cells of the kidney. Vasopressin acts on the distal tubules and collecting ducts, promoting the insertion of aquaporin-2 channels in the tubular cells. This allows water to move from the tubules back into the bloodstream, leading to water reabsorption and concentrated urine
Which of the following hormones is mainly responsible for sodium - potassium exchange in the salivary ducts?
Vasopressin
Angiotensin I
Aldosterone
Somatostatin
Cholecystokinin
Aldosterone is responsible for regulating ion exchange in salivary glands. It acts on a sodium / potassium ion exchange pump.It is a mineralocorticoid hormone derived from the zona glomerulosa of the adrenal gland.
A 44 year old lady presents with a mass in the upper outer quadrant of her right breast. Imaging, histology and clinical examination confirm a 1.5cm malignant mass lesion with no clinical evidence of axillary nodal disease.
A.Simple mastectomy alone
B.Radical mastectomy alone
C.Simple mastectomy and sentinel lymph node biopsy
D.Wide local excision and sentinel lymph node biopsy
E.Simple mastectomy and axillary node clearance
F.Radical mastectomy and axillary node clearance
G.Wide local excision and axillary node clearance
H.Wide local excision alone
Wide local excision and sentinel lymph node biopsy
A small peripheral lesion such as this would usually be suitable for breast conserving surgery. Since imaging and clinical examination is not suspicious for axillary disease, a sentinel lymph node biopsy should be performed.
A 44 year old lady presents with a mass lesion in the upper outer quadrant of the left breast. On clinical examination she has a 2cm mass lesion which on core biopsy is demonstrated to have invasive ductal carcinoma. An FNA of a bulky axillary lymph node contains malignant cells.
A.Simple mastectomy alone
B.Radical mastectomy alone
C.Simple mastectomy and sentinel lymph node biopsy
D.Wide local excision and sentinel lymph node biopsy
E.Simple mastectomy and axillary node clearance
F.Radical mastectomy and axillary node clearance
G.Wide local excision and axillary node clearance
H.Wide local excision alone
Wide local excision and axillary node clearance
Although the primary lesion is small enough for breast conserving surgery, the presence of overt axillary lymph node metastasis will attract a recommendation for axillary node clearance
A 39 year old lady presents with a mass lesion in her right breast. Clinical examination, biopsy and imaging confirm a 2.5 cm lesion in the upper inner quadrant of her right breast and a 1.5 cm lesion at the central aspect of the same breast. Her axilla shows lymphadenopathy and a fine needle aspirate from the node shows malignant cells.
A.Simple mastectomy alone
B.Radical mastectomy alone
C.Simple mastectomy and sentinel lymph node biopsy
D.Wide local excision and sentinel lymph node biopsy
E.Simple mastectomy and axillary node clearance
F.Radical mastectomy and axillary node clearance
G.Wide local excision and axillary node clearance
H.Wide local excision alone
Simple mastectomy and axillary node clearance
A combination of established axillary disease and multifocal invasive lesions attracts an indication for mastectomy and axillary clearance. A radical mastectomy is less frequently indicated in modern surgical practice, disease that is locally advanced is often best downstaged using medical therapy, rather than embarking on the operations for breast cancer that were first popularised over 100 years ago.
A 73 year old male presents with a collapse and is brought to the emergency department. On examination he has a cold, painful left hand and forearm.
A.Proximal brachial artery occlusion secondary to atheroma
B.Distal brachial artery occlusion secondary to atheroma
C.Axillary artery embolus
D.Axillary vein thrombosis
E.Cervical rib
F.Raynaud’s disease
G.Rheumatoid disease
Sudden arterial embolus will affect the axillary artery in up to 30% cases. Because of the acute nature of the condition there is no time for the development of a collateral circulation so the limb is usually pale and painful. Emboli occur usually occur as a result of atrial fibrillation. Fast atrial fibrillation can cause syncope and an acute embolus.
A 23 year old man presents with intermittent symptoms of altered sensation in his arm and discomfort when he uses his hands. He works as an electrician and his symptoms are worst when he is fitting light fixtures.
A.Proximal brachial artery occlusion secondary to atheroma
B.Distal brachial artery occlusion secondary to atheroma
C.Axillary artery embolus
D.Axillary vein thrombosis
E.Cervical rib
F.Raynaud’s disease
G.Rheumatoid disease
Cervical rib
Compression of the thoracic outlet by the fibrous band of the “rib” can result in both neurological and circulatory compromise. When manual tasks are performed in which the hand works overhead the signs and symptoms will be maximal and this is the basis of Adsons test.
A 19 year old lady presents with recurrent episodes of pain in her hands. She notices that her symptoms are worst in cold weather. When she gets the pain she notices that her hands are very pale, they then become dark blue in colour.
A.Proximal brachial artery occlusion secondary to atheroma
B.Distal brachial artery occlusion secondary to atheroma
C.Axillary artery embolus
D.Axillary vein thrombosis
E.Cervical rib
F.Raynaud’s disease
G.Rheumatoid disease
Raynaud’s disease
Raynauds disease is characterised by a series of colour changes and discomfort is often present. The young age at presentation coupled with the absence of a smoking history (in most cases) makes occlusive disease unlikely.
What is the most common site for upper limb emboli to lodge?
A-Brachial artery
B-Axillary artery
C-Radial artery
D-Ulnar artery
Brachial artery
Which muscle does not insert on the medial surface of the greater trochanter?
Gemelli
Obturator internus
Piriformis
Quadratus femoris
Obturator externus
The quadratus femoris fibres pass laterally to be inserted into the quadrate tubercle on the intertrochanteric crest of the femur. The other muscles all insert on the trochanteric fossa lying medial to the greater trochanter. The mnemonic for the muscles that insert on the medial surface of the greater trochanter is indeed “POGO”:
P: Piriformis
O: Obturator internus
G: Gemelli
O: Obturator externus
Which muscle is not innervated by the superior gluteal nerve?
Gluteus medius
Gluteus minimis
Tensor fascia lata
Obturator externus
Obturator externus
During a radical gastrectomy for carcinoma of the stomach the surgeons remove the omentum. What is the main source of its blood supply?
Ileocolic artery
Superior mesenteric artery
Gastroepiploic artery
Middle colic artery
Inferior mesenteric artery
The vessels supplying the omentum are the omental branches of the right and left gastro-epiploic arteries. The colonic vessels are not responsible for the arterial supply to the omentum. The left gastro-epiploic artery is a branch of the splenic artery and the right gastro-epiploic artery is a terminal branch of the gastroduodenal artery.
A 20 year old male presents with a tense, swollen knee joint. There is no history of antecedent trauma. On examination the joint is tense and swollen but there is no sign of injury. Plain x-rays show no fracture or arthritis. What is the most likely explanation?
Rupture of the anterior cruciate ligament
Rupture of the medial collateral ligament
Tibial plateau fracture
Haemophilia A
von Willebrands disease
Haemarthrosis without trauma is typically a feature of haemophilia A and B
Without a history of trauma, ligamentous rupture or tibial plateau fractures would be unusual.
Haemarthroses may occur in 45% of patients with severe von Willebrands disease. However, this is rare
A 38 year old lady is due to undergo a parathyroidectomy for hyperparathyroidism. At operation the inferior parathyroid gland is identified as being enlarged. A vessel is located adjacent to the gland laterally. This vessel is most likely to be the:
External carotid artery
Common carotid artery
Internal carotid artery
External jugular vein
None of the above
The common carotid artery is a lateral relation of the inferior parathyroid.
A 45 year old man has a long femoral line inserted to provide CVP measurements. The catheter passes from the common iliac vein into the inferior vena cava. At which of the following vertebral levels will this occur?
L5
L4
S1
L3
L2
The common iliac veins fuse with the IVC at L5.
Which of the following is not considered a major branch of the descending thoracic aorta?
Bronchial artery
Mediastinal artery
Inferior thyroid artery
Posterior intercostal artery
Oesophageal artery
The inferior thyroid artery is usually derived from the thyrocervical trunk, a branch of the subclavian artery.
Thoracic aorta
Origin T4
Terminates T12
Relations
Anteriorly (from top to bottom)-root of the left lung, the pericardium, the oesophagus, and the diaphragm
Posteriorly-vertebral column, azygos vein
Right- azygos veins, thoracic duct
Left- left pleura and lung
Branches
Lateral segmental branches: Posterior intercostal arteries
Lateral visceral: Bronchial arteries supply bronchial walls and lung excluding the alveoli
Midline branches: Oesophageal arteries
Where are the greatest proportion of musculi pectinati found?
Right ventricle
Left ventricle
Right atrium
Pulmonary valve
Aortic valve
The musculi pectinati are found in the atria, hence the reason that the atrial walls in the right atrium are irregular anteriorly.
The musculi pectinati of the atria are internal muscular ridges on the anterolateral surface of the chambers and they are only present in the area derived from the embryological true atrium.
Heart anatomy
The walls of each cardiac chamber comprise:
Epicardium
Myocardium
Endocardium
Cardiac muscle is attached to the cardiac fibrous skeleton.
Relations
The heart and roots of the great vessels within the pericardial sac are related to the posterior aspect of the sternum, medial ends of the 3rd to 5th ribs on the left and their associated costal cartilages. The heart and pericardial sac are situated obliquely two thirds to the left and one third to the right of the median plane.
The pulmonary valve lies at the level of the left third costal cartilage.
The mitral valve lies at the level of the fourth costal cartilage.
Coronary sinus
This lies in the posterior part of the coronary groove and receives blood from the cardiac veins. The great cardiac vein lies at its left and the middle and small cardiac veins lie on its right. The smallest cardiac vein (anterior cardiac vein) drains into the right atrium directly.
Aortic sinus
Right coronary artery arises from the right aortic sinus, the left is derived from the left aortic sinus, which lies posteriorly.
Features of the left ventricle as opposed to the right
Structure Left Ventricle
A-V Valve Mitral (double leaflet)
Walls Twice as thick as right
Trabeculae carnae Much thicker and more numerous
Right coronary artery
The RCA supplies:
Right atrium
Diaphragmatic part of the right ventricle
Usually the posterior third of the interventricular septum
The sino atrial node (60% cases)
The atrio ventricular node (80% cases)
Left coronary artery
The LCA supplies:
Left atrium
Most of left ventricle
Part of the right ventricle
Anterior two thirds of the inter ventricular septum
The sino atrial node (remaining 40% cases)
Innervation of the heart
Autonomic nerve fibres from the superficial and deep cardiac plexus. These lie anterior to the bifurcation of the trachea, posterior to the ascending aorta and superior to the bifurcation of the pulmonary trunk. The parasympathetic supply to the heart is from presynaptic fibres of the vagus nerves.
Valves of the heart
Mitral valve Aortic valve Pulmonary valve Tricuspid valve
2 cusps 3 cusps 3 cusps 3 cusps
First heart sound Second heart sound Second heart sound First heart sound
1 anterior cusp 2 anterior cusps 2 anterior cusps 2 anterior cusps
Attached to chordae tendinae No chordae No chordae Attached to chordae tendinae
The anterior interosseous nerve is a branch of which of the following?
Ulnar nerve
Superficial branch of the radial nerve
Axillary nerve
Deep branch of the radial nerve
Median nerve
Anterior interosseous nerve
The anterior interosseous nerve (volar interosseous nerve) is a branch of the median nerve that supplies the deep muscles on the front of the forearm, except the ulnar half of the flexor digitorum profundus.
It accompanies the anterior interosseous artery along the anterior of the interosseous membrane of the forearm, in the interval between the flexor pollicis longus and flexor digitorum profundus, supplying the whole of the former and (most commonly) the radial half of the latter, and ending below in the pronator quadratus and wrist joint.
Innervation
The anterior interosseous nerve classically innervates 2.5 muscles:
Flexor pollicis longus
Pronator quadratus
The radial half of flexor digitorum profundus (the lateral two out of the four tendons).
These muscles are in the deep level of the anterior compartment of the forearm.
Which of the structures listed below are most closely related to the axillary nerve within the quadrangular space?
Posterior circumflex humeral vessels
Axillary artery
Anterior circumflex humeral vessels
Radial artery
Acromiothoracic artery
The posterior circumflex humeral vessels which are branches of the axillary artery are related to the axillary nerve within the quadrangular space.
Axillary nerve
Terminal branch of the posterior cord of the brachial plexus
Root values C5 and C6
Descends posterior to the axillary artery at the lower border of subscapularis and then passes through quadrangular space with the posterior circumflex humeral vessels
Divides into anterior and posterior branches
Innervates deltoid muscle and small patch of skin over deltoid
Which of the following is not associated with Epstein-Barr virus?
Burkitt’s lymphoma
Post-transplantation lymphoma
Hodgkin’s lymphoma
Nasopharyngeal carcinoma
Mycosis fungoides
Mycosis fungoides
Oncoviruses
Viruses which cause cancer
These may be detected on blood test and prevented by vaccine
These are the main types of oncoviruses and their diseases:
Oncovirus Cancer
Epstein-Barr virus Burkitt’s lymphoma
Hodgkin’s lymphoma
Post transplant lymphoma
Nasopharyngeal carcinoma
Human papillomavirus 16/18 Cervical cancer
Anal cancer
Penile cancer
Vulval cancer
Oropharyneal cancer
Human herpes virus 8 Kaposi’s sarcoma
Hepatitis B virus Hepatocellular carcinoma
Hepatitis C virus Hepatocellular carcinoma
Human T-lymphotropic virus 1 Tropical spastic paraparesis
Adult T cell leukaemia
An 80 year old lady is brought to the emergency department by her carers. She has been unwell for the past few days. On examination, she has a cold, pulseless leg with fixed mottling, below the knee. A duplex scan shows a stenosis of the profunda femoris and popliteal artery with no flow distal to this. What is the most appropriate course of action.
Femoro-femoro cross over graft
Axillo- femoral bypass graft
Trans femoral amputation
Gritti Stokes amputation
Administration of intravenous unfractionated heparin
Fixed mottling is a sign of an unsalvageable limb and mandates either amputation or palliation. A through knee amputation (Gritti Stokes) is unlikely to heal in this setting.
Acute limb ischaemia
- Thrombosis of a pre-existing site of atherosclerosis is the commonest cause of acute limb ischaemia
Acute thrombosis of popliteal aneurysms poses the greatest threat to the limb
Sudden occlusion of a large proximal vessel results in the typical appearances of acute limb ischaemia
Clinical appearances
Less than 6 hours = White leg
At 6 -12 hours = Mottled limb with blanching on pressure
More than 12-24 hours = Fixed mottling
Management of acutely ischaemic leg
Clinical picture Treatment
White leg with sensorimotor deficit Surgery and embolectomy
Dusky leg, mild anaesthesia Angiography
Fixed mottling Primary amputation
Role of thrombolysis
Intra arterial thrombolysis is better than peripheral thrombolysis
Mainly indicated in acute on chronic thrombosis
Avoid if within 2 months of CVA or 2 weeks of surgery
Aspiration of clot may improve success rate if the thrombosis is large
Surgery
Both groins should be prepared
Transverse arteriotomy is easier to close
Poor inflow should be managed with iliac trawl- if this fails to improve then consider a femoro-femoral cross over or axillo-femoral cross over.
A check angiogram should be performed on table and prior to closure
Systemic heparinisation should follow surgery
Fasciotomy should be considered if the time between onset and surgery exceeds 6 hours
A 17 year old man undergoes an elective right hemicolectomy. Post operatively he receives a total of 6 litres of 0.9% sodium chloride solution, over 24 hours. Which of the following complications may ensue?
Hyperchloraemic acidosis
Hypochloraemic alkalosis
Hyperchloraemic alkalosis
Acute renal failure
None of the above
Excessive infusions of any intravenous fluid carry the risk of development of tissue oedema and potentially cardiac failure. Excessive administration of sodium chloride is a recognised cause of hyperchloraemic acidosis and therefore Hartmans solution may be preferred where large volumes of fluid are to be administered.
Post operative fluid management
Composition of commonly used intravenous fluids mmol-1
Na K Cl Bicarbonate Lactate
Plasma 137-147 4-5.5 95-105 22-25 -
0.9% Saline 153 - 153 - -
Dextrose / saline 30.6 - 30.6 - -
Hartmans 130 4 110 - 28
Post operative fluid management
In the UK the GIFTASUP and NICE (CG174 2017) guidelines (see reference below) were devised to try and provide some consensus guidance as to how intravenous fluids should be administered. Some time ago it was a commonly held belief that little harm would occur as a result of excessive administration of normal saline and many oliguric post operative patients received enormous quantities of IV fluids. As a result they developed hyperchloraemic acidosis. With greater understanding of this potential complication, the use of electrolyte balanced solutions (Ringers lactate/ Hartmans) is now favored over normal saline.
The other guidance includes:
Fluids given should be documented clearly and easily available
Assess the patient’s fluid status when they leave theatre
If a patient is haemodynamically stable and euvolaemic, aim to restart oral fluid intake as soon as possible
Review patients whose urinary sodium is < 20
If a patient is oedematous, hypovolaemia if present should be treated first. This should then be followed by a negative balance of sodium and water, monitored using urine Na excretion levels
Solutions such as Dextran 70 should be used in caution in patients with sepsis as there is a risk of developing acute renal injury
A 52 year female post hysterectomy attends clinic. She reports pain and reduced sensation over the medial aspect of her thigh. Clinically thigh adduction is weak. What is the most likely nerve injury?
Obturator nerve
Sciatic nerve
Femoral nerve
L3 cord compression
Deep peroneal nerve
The obturator nerve supplies sensation to the medial aspect of the thigh and causes adduction and internal rotation of the thigh.
Injury occurs during pelvic or abdominal surgery.
L3 cord compression is unlikely.
Obturator nerve
The obturator nerve arises from L2, L3 and L4 by branches from the ventral divisions of each of these nerve roots. L3 forms the main contribution and the second lumbar branch is occasionally absent. These branches unite in the substance of psoas major, descending vertically in its posterior part to emerge from its medial border at the lateral margin of the sacrum. It then crosses the sacroiliac joint to enter the lesser pelvis, it descends on obturator internus to enter the obturator groove. In the lesser pelvis the nerve lies lateral to the internal iliac vessels and ureter, and is joined by the obturator vessels lateral to the ovary or ductus deferens.
Supplies
Medial compartment of thigh
Muscles supplied: external obturator, adductor longus, adductor brevis, adductor magnus (not the lower part-sciatic nerve), gracilis
The cutaneous branch is often absent. When present, it passes between gracilis and adductor longus near the middle part of the thigh, and supplies the skin and fascia of the distal two thirds of the medial aspect.
Obturator canal
Connects the pelvis and thigh: contains the obturator artery, vein, nerve which divides into anterior and posterior branches.
A 28 year old lady with known von Willebrands disease is bleeding following the excision of a sebaceous cyst. Administration of which of the following agents is most likely to be beneficial?
Desmopressin
Factor IX concentrate
Factor VII concentrate
Factor X concentrate
Vasopressin
Desmopressin is useful in managing the bleeding from the commonest type of vWD. Vasopressin is less likely to be disease specific and at most would have a vasoconstrictor approach which is likely to be of brief duration.
von Willebrands disease
- Most common inherited bleeding disorder
All vWD is caused by mutations in the gene for von Willebrand factor. von Willebrand factor is an adhesive glycoprotein that is secreted by endothelium and megakaryocytes
von Willebrand factor promotes platelet adhesion to damaged endothelium and other platelets. It is also involved in the transport and stabilization of factor VIII
There are 7 subtypes of von Willebrand disease. The commonest is type I (autosomal dominant) which accounts for 80% of cases, type 2vWD (autosomal dominant or recessive) accounts for 15% of cases
There is a significant spectrum of severity ranging from spontaneous bleeding and epistaxis through to troublesome excessive bleeding following minor procedures
The test that is most commonly used are von willebrand factor assays
Treatments include administration of tranexamic acid for minor cases undergoing minor procedures. More significant bleeding or more significant procedures respond well to DDAVP. This is most effective in type I, less effective in type 2 and contraindicated in type 2B. Patients with type 3 disease do not respond to DDAVP as they lack the ability to secrete vWF
Individuals who cannot have DDAVP or in whom it is contra indicated usually receive factor VIII concentrates containing vWF
The following are true of the femoral nerve, except:
It is derived from L2, L3 and L4 nerve roots
It supplies sartorius
It supplies quadriceps femoris
It gives cutaneous innervations via the saphenous nerve
It supplies adductor longus
Adductor longus is supplied by the obturator nerve.
Femoral nerve
Root values L2, 3, 4
Innervates
Pectineus
Sartorius
Quadriceps femoris
Vastus lateralis/medialis/intermedius
Rectus femoris
Branches
Medial cutaneous nerve of thigh
Saphenous nerve
Intermediate cutaneous nerve of thigh
Path
Penetrates psoas major and exits the pelvis by passing under the inguinal ligament to enter the femoral triangle, lateral to the femoral artery and vein.
Mnemonic for femoral nerve supply
(don’t) M I S V Q Scan for PE
M edial cutaneous nerve of the thigh
I ntermediate cutaneous nerve of the thigh
S aphenous nerve
V astus
Q uadriceps femoris
S artorius
PE ectineus
A 2 week old baby is referred to the surgical team by the paediatricians. They are concerned because the child has a painful area of macerated tissue at the site of the umbilicus. On examination, a clear- yellowish fluid is seen to be draining from the umbilicus when the baby cries. What is the most likely diagnosis?
Omphalitis
Umbilical granuloma
Persistent vitello-intestinal duct
Patent urachus
Isolated cellular remnants
A patent urachus will present with umbilical urinary discharge. The skin may become macerated if not properly cared for. The discharge is most likely to be present when intra-abdominal pressure is raised. It is associated with posterior urethral valves.
Paediatric umbilical disorders
Embryology
During development the umbilicus has two umbilical arteries and one umbilical vein. The arteries are continuous with the internal iliac arteries and the vein is continuous with the falciform ligament (ductus venosus). After birth the cord dessicates and separates and the umbilical ring closes.
Umbilical hernia
Up to 20% of neonates may have an umbilical hernia, it is more common in premature infants. The majority of these hernias will close spontaneously (may take between 12 months and three years). Strangulation is rare.
Paraumbilical hernia
These are due to defects in the linea alba that are in close proximity to the umbilicus. The edges of a paraumbilical hernia are more clearly defined than those of an umbilical hernia. They are less likely to resolve spontaneously than an umbilical hernia.
Omphalitis
This condition consists of infection of the umbilicus. Infection with Staphylococcus aureus is the commonest cause. The condition is potentially serious as infection may spread rapidly through the umbilical vessels in neonates with a risk of portal pyaemia, and portal vein thrombosis. Treatment is usually with a combination of topical and systemic antibiotics.
Umbilical granuloma
These consist of cherry red lesions surrounding the umbilicus, they may bleed on contact and be a site of seropurulent discharge. Infection is unusual and they will often respond favorably to chemical cautery with topically applied silver nitrate.
Persistent urachus
This is characterised by urinary discharge from the umbilicus. It is caused by persistence of the urachus which attaches to the bladder. They are associated with other urogenital abnormalities.
Persistent vitello-intestinal duct
This will typically present as an umbilical discharge that discharges small bowel content. Complete persistence of the duct is a rare condition. Much more common is the persistence of part of the duct (Meckels diverticulum). Persistent vitello-intestinal ducts are best imaged using a contrast study to delineate the anatomy and are managed by laparotomy and surgical closure.
A 73 year old female is referred to the surgical clinic with an iron deficiency anaemia. As part of the diagnostic work up the doctor requests a serum ferritin level. Which of the conditions listed is most likely to lead to a falsely elevated result?
Locally perforated sigmoid colonic adenocarcinoma
Colonic angiodysplasia
Dieulafoy lesion of the stomach
Transitional cell carcinoma of the bladder
Endometrial adenocarcinoma
A locally perforated colonic tumour will typically cause an intense inflammatory response and if peritonitis is not present clinically then at the very least a localised abscess. This inflammatory process is the most likely (from the list) to falsely raise the serum ferritin level. Angiodysplasia and dieulafoy lesions are mucosal arteriovenous malformations and unlikely to result in considerable inflammatory activity.
Ferritin
Ferritin is an intracellular protein that binds iron and stores it to be released in a controlled fashion at sites where iron is required. Because iron and ferritin are bound the total body ferritin levels may be decreased in cases of iron deficiency anaemia. Measurement of serum ferritin levels can be useful in determining whether an apparently low haemoglobin and microcytosis is truly caused by an iron deficiency state.
Ferritin is an acute phase protein and may be synthesised in increased quantities in situations where inflammatory activity is ongoing. Falsely elevated results may therefore be encountered clinically and need to be taken in context of the clinical picture and full blood count results.
An 18 year old athlete attends orthopaedic clinic reporting pain and swelling over the medial aspect of the knee joint. The pain occurs when climbing the stairs, but is not present when walking on flat ground. Clinically there is pain over the medial, proximal tibia and the McMurray test is negative. What is the most likely cause of this patient’s symptoms?
Anterior cruciate ligament tear
Prepatellar bursitis
Medial meniscus injury
Pes Anserinus Bursitis
Fracture of tibia
Pes anserinus: GOOSE’S FOOT
Combination of sartorius, gracilis and semitendinous tendons inserting into the anteromedial proximal tibia.
Pes Anserinus Bursitis is common in sportsmen due to overuse injuries. The main sign is of pain in the medial proximal tibia. As the McMurray test is negative, medial meniscal injury is excluded.
Sartorius
- Longest strap muscle in the body
Most superficial muscle in the anterior compartment of the thigh
Origin Anterior superior iliac spine
Insertion Medial surface of the of the body of the tibia (upper part). It inserts anterior to gracilis and semitendinosus
Nerve Supply Femoral nerve (L2,3 -root values for sartorius)
Action
Flexor of the hip and knee, slight abducts the thigh and rotates it laterally
It assists with medial rotation of the tibia on the femur. For example it would play a pivotal role in placing the right heel onto the left knee ( and vice versa)
Important relations The middle third of this muscle, and its strong underlying fascia forms the roof of the adductor canal , in which lie the femoral vessels, the saphenous nerve and the nerve to vastus medialis.
A patient is referred due to the development of a third nerve palsy associated with a headache. On examination, meningism is present. Which one of the following diagnoses needs to be urgently excluded?
Weber’s syndrome
Internal carotid artery aneurysm
Multiple sclerosis
Posterior communicating artery aneurysm
Anterior communicating artery aneurysm
Painful third nerve palsy = posterior communicating artery aneurysm
Given the combination of a headache and third nerve palsy it is important to exclude a posterior communicating artery aneurysm
Third nerve palsy
Features
eye is deviated ‘down and out’
ptosis
pupil may be dilated (sometimes called a ‘surgical’ third nerve palsy)
Causes
diabetes mellitus
vasculitis e.g. temporal arteritis, SLE
false localizing sign* due to uncal herniation through tentorium if raised ICP
posterior communicating artery aneurysm (pupil dilated)
cavernous sinus thrombosis
Weber’s syndrome: ipsilateral third nerve palsy with contralateral hemiplegia -caused by midbrain strokes
other possible causes: amyloid, multiple sclerosis
*this term is usually associated with sixth nerve palsies but it may be used for a variety of neurological presentations
Which of the following would be a sensible volume for maintenance intravenous fluids in a 3 day old term neonate?
50ml/ kg/ hour
50ml/ kg/ day
100ml/kg/hour
100ml/kg/day
200ml/kg/day
Calculate routine maintenance IV fluid rates for children and young people using the HollidaySegar formula (100 ml/kg/day for the first 10 kg of weight, 50 ml/kg/day for the next 10 kg and 20 ml/kg/day for the weight over 20 kg). Be aware that over a 24hour period, males rarely need more than 2500 ml and females rarely need more than 2000 ml of fluids.
From birth to day 1: 50-60 ml/kg/day.
Day 2: 70-80 ml/kg/day.
Day 3: 80-100 ml/kg/day.
Day 4: 100-120 ml/kg/day.
Days 5-8: 120-150 ml/kg/day.
Paediatric fluid management
Since 2000 there have been at least 4 reported deaths from fluid induced hyponatraemia in children. This led to the National Patient Safety Agency introducing revised guidelines in 2007. These have been reviewed and extensively updated by NICE in 2015 and further modified in 2020.
Indications for IV fluids include:
Resuscitation and circulatory support
Replacing on-going fluid losses
Maintenance fluids for children for whom oral fluids are not appropriate
Correction of electrolyte disturbances
Fluids to be avoided
Outside the neonatal period saline / glucose solutions should not be given. The greatest risk is with saline 0.18 / glucose 4% solutions. The report states that 0.45% saline / 5% glucose may be used. But preference should be given to isotonic solutions and few indications exist for this solution either.
The key point emphasised in the NICE guidelines in the avoidance of glucose containing solutions and instead of routinely giving glucose IV to children and neonates, the blood glucose levels should be monitored. In neonates in the first few days of life, sodium levels in the range of 131-154 mmol/l may be too high and a neonatologist consulted on a case by case basis.
Fluids to be used
If children and young people need IV fluids for routine maintenance, initially use isotonic crystalloids that contain sodium in the range 131 to 154 mmol/litre
Potassium should be added to maintenance fluids according patients plasma potassium levels (which should be monitored).
Blood glucose levels should be monitored in individuals at risk of hypoglycaemia
Intraoperative fluid management
If children and young people need IV fluids for routine maintenance, initially use isotonic crystalloids that contain sodium in the range 131 to 154 mmol/litre
Blood glucose levels should be monitored
Maintenance fluids
Weight Water requirement/kg/day Na mmol/kg/day K mmol/kg/day
First 10Kg body weight 100ml 2-4 1.5-2.5
Second 10Kg body weight 50ml 1-2 0.5-1.5
Subsequent Kg 20ml 0.5-1.0 0.2-0.7
Blood glucose will need to be monitored
A 38 year old man presents to the clinic with shoulder weakness. On examination he has an inability to initiate shoulder abduction. Which of the nerves listed below is least likely to be functioning normally?
Suprascapular nerve
Medial pectoral nerve
Axillary nerve
Median nerve
Radial nerve
Suprascapular nerve
The suprascapular nerve arises from the upper trunk of the brachial plexus. It lies superior to the trunks of the brachial plexus and passes inferolaterally parallel to them. It passes through the scapular notch, deep to trapezius. It innervates both supraspinatus and infraspinatus and initiates abduction of the shoulder. If damaged, patients may be able to abduct the shoulder by leaning over the affected side and deltoid can then continue to abduct the shoulder.
A 70 year old man undergoes a revision total hip replacement. 30 days post operatively the hip dislocates and pus is discharging from the wound. He is systemically unwell with a temperature of 38.5 and WCC 19. What is the most appropriate course of action?
Lay open wound and apply a VAC dressing
Hindquater amputation
Revision arthroplasty
Removal of metalwork and bone grafting
Removal of metalwork and implantation of local antibiotics
Removal of metal work implantation of gentamicin beads and delayed revision is the mainstay of managing this complication after 4 weeks. In earlier infections, linear exchanges covered by prolonged courses of antibiotics can be used.
Osteomyelitis
Infection of the bone
Causes
S aureus and occasionally Enterobacter or Streptococcus species
In sickle cell: Salmonella species
Clinical features
Erythema
Pain
Fever
Investigation
X-ray: lytic centre with a ring of sclerosis
Bone biopsy and culture
Treatment
Prolonged antibiotics
Sequestra may need surgical removal
A 39 year old man notices a swelling in his left hemiscrotum. On examination he has a left sided varicocele. The ipsilateral testis is normal on palpation. What is the most appropriate course of action?
Scrotal exploration and ligation of the varicocele
Abdominal ultrasound
Scrotal ultrasound
Left orchidectomy
Discharge
A left sided varicocele is a recognised presenting sign of a renal tumour occluding the renal vein (into which the left testicular vein drains). An abdominal ultrasound should be undertaken to exclude this. Surgery for uncomplicated varicocele is usually unnecessary.
Renal tumours
Renal cell carcinoma
Renal cell carcinoma is an adenocarcinoma of the renal cortex and is believed to arise from the proximal convoluted tubule. They are usually solid lesions, up to 20% may be multifocal, 20% may be calcified and 20% may have either a cystic component or be wholly cystic. They are often circumscribed by a pseudocapsule of compressed normal renal tissue. Spread may occur either by direct extension into the adrenal gland, renal vein or surrounding fascia. More distant disease usually occurs via the haematogenous route to lung, bone or brain.
Renal cell carcinoma comprise up to 85% of all renal malignancies. Males are more commonly affected than females and sporadic tumours typically affect patients in their sixth decade.
Patients may present with a variety of symptoms including; haematuria (50%), loin pain (40%), mass (30%) and up to 25% may have symptoms of metastasis.Less than 10% have the classic triad of haematuria, pain and mass.
Investigation
Many cases will present as haematuria and be discovered during diagnostic work up. Benign renal tumours are rare, so renal masses should be investigated with multislice CT scanning. Most tumours are also characterised with an arterial phase CT, particularly if they may be suitable for partial nephrectomy.
CT scanning of the chest and abdomen to detect distant disease should also be undertaken.
Routine bone scanning is not indicated in the absence of symptoms.
Biopsy should not be performed when a nephrectomy is planned but is mandatory before any ablative therapies are undertaken.
Assessment of the functioning of the contra lateral kidney.
Management
T1 lesions may be managed by partial nephrectomy and this gives equivalent oncological results to total radical nephrectomy. Partial nephrectomy may also be performed when there is inadequate reserve in the remaining kidney. Ablative techniques may also be considered for small T1 lesions in unfit patients. In general, outcomes are less favorable than with surgical resection.
For T2 lesions and above a radical nephrectomy is standard practice and this may be performed via a laparoscopic or open approach. Preoperative embolisation is not indicated nor is resection of uninvolved adrenal glands. During surgery early venous control is mandatory to avoid shedding of tumour cells into the circulation.
Patients with completely resected disease do not benefit from adjuvant therapy with either chemotherapy or biological agents. These should not be administered outside the setting of clinical trials.
Patients with transitional cell cancer will require a nephroureterectomy with disconnection of the ureter at the bladder.
A 50 year old man is admitted after falling from scaffolding. He has an open fracture of his tibia with a 15 cm wound. He is neurovascularly intact. What is the best initial course of action?
Intravenous antibiotics, photography and application of saline soaked gauze with impermeable dressing
Thorough wound debridement in the emergency department
Combined skeletal and soft tissue reconstruction on a scheduled operating list
Application of external fixator and conversion to internal fixation after two weeks
Immediate skeletal stabilisation and application of negative pressure dressing
Note the question asks for initial management
The initial management of open fractures should include administration of intravenous antibiotics, photography of wound and application of a sterile soaked gauze and impermeable film. The wound should only be handled to remove gross contamination. The patient is then likely to require definitive skeletal and soft tissue reconstruction.
Open fractures
The term open fracture refers to a disruption of the bony cortex associated with a breach in the overlying skin. Any wound that is present in the same limb as a fracture should be suspected as being representative of an open fracture. One of the main problems with open fractures is the associated injuries to the surrounding soft tissues. Whilst the skin is usually relatively resistant to trauma, underlying muscle can be damaged or devitalised, nerves, blood vessels and periosteum may all be disrupted the degree to which this occurs correlates with the severity of the injury and the outcome. These can be graded using the Gustilo and Anderson system (see below).
Grade Injury
1 Low energy wound <1cm
2 Greater than 1cm wound with moderate soft tissue damage
3 High energy wound > 10cm with extensive soft tissue damage
3 A (sub group of 3) Adequate soft tissue coverage
3 B (sub group of 3) Inadequate soft tissue coverage
3 C (sub group of 3) Associated arterial injury
In Type IIIc injuries, the mangled extremity scoring system (MESS) can help to predict the need for primary amputation.
Key points for initial management
Remove obvious contaminants from very contaminated wounds in the ED
Wound lavage should usually be undertaken in the OR
Early wound photography should be performed
Consider transfer of complex cases to centres that provide orthoplastic care
All wounds should be managed within 24 hours and high velocity ones within 12 hours, those with vascular compromise should be managed immediately
CT angiography is useful in delineating the extent of concommitant vascular injury
In relation to operating in the elderly which statement is false?
A 30 minute increment in operation length is associated with increase in mortality in patients over the age of 80
Hypoalbuminaemia is associated with increased mortality
Statins given preoperatively reduce perioperative cardiac events
Elevated brain (or B-type) natriuretic peptide (BNP) levels before undergoing non cardiac surgery is associated with high risk of cardiac mortality and all cause mortality
Beta blockers should be stopped acutely prior to surgery due to risk of perioperative hypotension
Beta blockers should not be stopped acutely prior to surgery as there may be a rebound effect associated with increased complications.
Brain natriuretic peptide is a neurohormone synthesized in the cardiac ventricles. Levels have been used to assess prognosis in heart failure and acute coronary syndromes. Preoperative elevated brain natriuretic peptide levels identify patients undergoing non cardiac surgery at high risk of cardiac mortality and all cause mortality.
All patients with peripheral vascular disease should take statins prior to vascular surgery as studies have shown a 50% risk reduction and a reduction in perioperative cardiac events.
Proactive care of older people undergoing surgery (POPS)
Comprehensive geriatric assessment
MDT assessment preoperatively
Main predictors of complications are co-morbidities cardiac disease and reduced functional capacity - preoperative assessment is the key to preventing adverse postoperative outcomes
Patients screened for risk factors (albumin <30, co morbidities)
Management plan made and disseminated to all involved
Patients education: pain relief, post op exercises, nutrition
Outcomes:
Fewer postoperative medical complications
Reduced length of stay by 4.5 days
With respect to the basilic vein, which statement is false?
Its deep anatomical location makes it unsuitable for use as an arteriovenous access site in fistula surgery
It originates from the dorsal venous network on the hand
It travels up the medial aspect of the forearm
Halfway between the shoulder and the elbow it lies deep to muscle
It is joined by the brachial vein to form the axillary vein
It is used in arteriovenous fistula surgery during a procedure known as a basilic vein transposition.
Basilic vein
The basilic and cephalic veins both provide the main pathways of venous drainage for the arm and hand. It is continuous with the palmar venous arch distally and the axillary vein proximally.
Path
Originates on the medial side of the dorsal venous network of the hand, and passes up the forearm and arm.
Most of its course is superficial.
Near the region anterior to the cubital fossa the vein joins the cephalic vein.
Midway up the humerus the basilic vein passes deep under the muscles.
At the lower border of the teres major muscle, the anterior and posterior circumflex humeral veins feed into it.
It is often joined by the medial brachial vein before draining into the axillary vein.
A 32 year old lady is admitted with weakness, visual disturbance and peri orbital pain. On examination, she is noted to have mydriasis and diminished direct response to light shone into the affected eye. The consensual response is preserved when light is shone into the unaffected eye. Which of the cranial nerves listed below is responsible for the diminished direct response?
Abducens
Oculomotor
Optic
Trigeminal
Hypoglossal
This describes a relative afferent pupillary defect (RAPD). RAPD is a defect in the direct response to light. It is due to damage in optic nerve or severe retinal disease. If an optic nerve lesion is present the affected pupil will not constrict to light when light is shone in the that pupil during the swinging flashlight test. However, it will constrict if light is shone in the other eye (consensual response).
The most likely cause for this is an optic neuritis (not really surgical!). Other causes include ischemic optic disease or retinal disease, severe glaucoma causing trauma to optic nerve and direct optic nerve damage (trauma, radiation, tumor).
Cranial nerves
Cranial nerve lesions
Olfactory nerve May be injured in basal skull fractures or involved in frontal lobe tumour extension. Loss of olfactory nerve function in relation to major CNS pathology is seldom an isolated event and thus it is poor localiser of CNS pathology.
Optic nerve Problems with visual acuity may result from intra ocular disorders. Problems with the blood supply such as amaurosis fugax may produce temporary visual distortion. More important surgically is the pupillary response to light. The pupillary size may be altered in a number of disorders. Nerves involved in the resizing of the pupil connect to the pretectal nucleus of the high midbrain, bypassing the lateral geniculate nucleus and the primary visual cortex. From the pretectal nucleus neurones pass to the Edinger - Westphal nucleus, motor axons from here pass along with the oculomotor nerve. They synapse with ciliary ganglion neurones; the parasympathetic axons from this then innervate the iris and produce miosis. The miotic pupil is seen in disorders such as Horner’s syndrome or opiate overdose.
Mydriasis is the dilatation of the pupil in response to disease, trauma, drugs (or the dark!). It is pathological when light fails to induce miosis. The radial muscle is innervated by the sympathetic nervous system. Because the parasympathetic fibres travel with the oculomotor nerve they will be damaged by lesions affecting this nerve (e.g. cranial trauma).
The response to light shone in one eye is usually a constriction of both pupils. This indicates intact direct and consensual light reflexes. When the optic nerve has an afferent defect the light shining on the affected eye will produce a diminished pupillary response in both eyes. Whereas light shone on the unaffected eye will produce a normal pupillary response in both eyes. This is referred to as the Marcus Gunn pupil and is seen in conditions such as optic neuritis. In a total CN II lesion shining the light in the affected eye will produce no response.
Oculomotor nerve The pupillary effects are described above. In addition it supplies all ocular muscles apart from lateral rectus and superior oblique. Thus the affected eye will be deviated inferolaterally. Levator palpebrae superioris may also be impaired resulting in impaired ability to open the eye.
Trochlear nerve The eye will not be able to look down.
Trigeminal nerve Largest cranial nerve. Exits the brainstem at the pons. Branches are ophthalmic, maxillary and mandibular. Only the mandibular branch has both sensory and motor fibres. Branches converge to form the trigeminal ganglion (located in Meckels cave). It supplies the muscles of mastication and also tensor veli palatine, mylohyoid, anterior belly of digastric and tensor tympani. The detailed descriptions of the various sensory functions are described in other areas of the website. The corneal reflex is important and is elicited by applying a small tip of cotton wool to the cornea, a reflex blink should occur if it is intact. It is mediated by: the naso ciliary branch of the ophthalmic branch of the trigeminal (sensory component) and the facial nerve producing the motor response. Lesions of the afferent arc will produce bilateral absent blink and lesions of the efferent arc will result in a unilateral absent blink.
Abducens nerve The affected eye will have a deficit of abduction. This cranial nerve exits the brainstem between the pons and medulla. It thus has a relatively long intra cranial course which renders it susceptible to damage in raised intra cranial pressure.
Facial nerve Emerges from brainstem between pons and medulla. It controls muscles of facial expression and taste from the anterior 2/3 of the tongue. The nerve passes into the petrous temporal bone and into the internal auditory meatus. It then passes through the facial canal and exits at the stylomastoid foramen. It passes through the parotid gland and divides at this point. It does not innervate the parotid gland. Its divisions are considered in other parts of the website. Its motor fibres innervate orbicularis oculi to produce the efferent arm of the corneal reflex. In surgical practice it may be injured during parotid gland surgery or invaded by malignancies of the gland and a lower motor neurone on the ipsilateral side will result.
Vestibulo-cochlear nerve Exits from the pons and then passes through the internal auditory meatus. It is implicated in sensorineural hearing loss. Individuals with sensorineural hearing loss will localise the sound in webers test to the normal ear. Rinnes test will be reduced on the affected side but should still work. These two tests will distinguish sensorineural hearing loss from conductive deafness. In the latter condition webers test will localise to the affected ear and Rinnes test will be impaired on the affected side. Surgical lesions affecting this nerve include CNS tumours and basal skull fractures. It may also be damaged by the administration of ototoxic drugs (of which gentamicin is the most commonly used in surgical practice).
Glossopharyngeal nerve Exits the pons just above the vagus. Receives sensory fibres from posterior 1/3 tongue, tonsils, pharynx and middle ear (otalgia may occur following tonsillectomy). It receives visceral afferents from the carotid bodies. It supplies parasympathetic fibres to the parotid gland via the otic ganglion and motor function to stylopharyngeaus muscle. The sensory function of the nerve is tested using the gag reflex.
Vagus nerve Leaves the medulla between the olivary nucleus and the inferior cerebellar peduncle. Passes through the jugular foramen and into the carotid sheath. Details of the functions of the vagus nerve are covered in the website under relevant organ sub headings.
Accessory nerve Exists from the caudal aspect of the brainstem (multiple branches) supplies trapezius and sternocleidomastoid muscles. The distal portion of this nerve is most prone to injury during surgical procedures.
Hypoglossal nerve Emerges from the medulla at the preolivary sulcus, passes through the hypoglossal canal. It lies on the carotid sheath and passes deep to the posterior belly of digastric to supply muscles of the tongue (except palatoglossus). Its location near the carotid sheath makes it vulnerable during carotid endarterectomy surgery and damage will produce ipsilateral defect in muscle function.
Which of the following fascial structures encases the apex of the lungs?
Waldeyers fascia
Sibsons fascia
Pretracheal fascia
Clavipectoral fascia
None of the above
Sibson’s fascia overlies the apices of both lungs
The suprapleural fascia (Sibson’s fascia) runs from C7 to the first rib and overlies the apex of both lungs.It lies between the parietal pleura and the thoracic cage.
Lung anatomy
The right lung is composed of 3 lobes divided by the oblique and transverse fissures. The left lung has two lobes divided by the oblique fissure.The apex of both lungs is approximately 4cm superior to the sterno-costal joint of the first rib. Immediately below this is a sulcus created by the subclavian artery.
Peripheral contact points of the lung
Base: diaphragm
Costal surface: corresponds to the cavity of the chest
Mediastinal surface: Contacts the mediastinal pleura. Has the cardiac impression. Above and behind this concavity is a triangular depression named the hilum, where the structures which form the root of the lung enter and leave the viscus. These structures are invested by pleura, which, below the hilum and behind the pericardial impression, forms the pulmonary ligament
Right lung
Above the hilum is the azygos vein; Superior to this is the groove for the superior vena cava and right innominate vein; behind this, and nearer the apex, is a furrow for the innominate artery. Behind the hilum and the attachment of the pulmonary ligament is a vertical groove for the oesophagus; In front and to the right of the lower part of the oesophageal groove is a deep concavity for the extrapericardiac portion of the inferior vena cava.
The root of the right lung lies behind the superior vena cava and the right atrium, and below the azygos vein.
The right main bronchus is shorter, wider and more vertical than the left main bronchus and therefore the route taken by most foreign bodies.
Left lung
Above the hilum is the furrow produced by the aortic arch, and then superiorly the groove accommodating the left subclavian artery; Behind the hilum and pulmonary ligament is a vertical groove produced by the descending aorta, and in front of this, near the base of the lung, is the lower part of the oesophagus.
The root of the left lung passes under the aortic arch and in front of the descending aorta.
Inferior borders of both lungs
6th rib in mid clavicular line
8th rib in mid axillary line
10th rib posteriorly
The pleura runs two ribs lower than the corresponding lung level.
Bronchopulmonary segments
Segment number Right lung Left lung
1 Apical Apical
2 Posterior Posterior
3 Anterior Anterior
4 Lateral Superior lingular
5 Medial Inferior lingular
6 Superior (apical) Superior (apical)
7 Medial basal Medial basal
8 Anterior basal Anterior basal
9 Lateral basal Lateral basal
10 Posterior basal Posterior basal
A 63 year old man is admitted with severe headache, nausea and recent epileptic fit. Fundoscopy shows papilloedema. He is also noted to have diplopia. Which of the cranial nerves listed accounts for the latter?
Abducens
Optic
Oculomotor
Facial
Trigeminal
Abducens. The long intracranial course of this nerve makes it susceptible to damage early in the course of raised ICP.
Cranial nerves
Cranial nerve lesions
Olfactory nerve May be injured in basal skull fractures or involved in frontal lobe tumour extension. Loss of olfactory nerve function in relation to major CNS pathology is seldom an isolated event and thus it is poor localiser of CNS pathology.
Optic nerve Problems with visual acuity may result from intra ocular disorders. Problems with the blood supply such as amaurosis fugax may produce temporary visual distortion. More important surgically is the pupillary response to light. The pupillary size may be altered in a number of disorders. Nerves involved in the resizing of the pupil connect to the pretectal nucleus of the high midbrain, bypassing the lateral geniculate nucleus and the primary visual cortex. From the pretectal nucleus neurones pass to the Edinger - Westphal nucleus, motor axons from here pass along with the oculomotor nerve. They synapse with ciliary ganglion neurones; the parasympathetic axons from this then innervate the iris and produce miosis. The miotic pupil is seen in disorders such as Horner’s syndrome or opiate overdose.
Mydriasis is the dilatation of the pupil in response to disease, trauma, drugs (or the dark!). It is pathological when light fails to induce miosis. The radial muscle is innervated by the sympathetic nervous system. Because the parasympathetic fibres travel with the oculomotor nerve they will be damaged by lesions affecting this nerve (e.g. cranial trauma).
The response to light shone in one eye is usually a constriction of both pupils. This indicates intact direct and consensual light reflexes. When the optic nerve has an afferent defect the light shining on the affected eye will produce a diminished pupillary response in both eyes. Whereas light shone on the unaffected eye will produce a normal pupillary response in both eyes. This is referred to as the Marcus Gunn pupil and is seen in conditions such as optic neuritis. In a total CN II lesion shining the light in the affected eye will produce no response.
Oculomotor nerve The pupillary effects are described above. In addition it supplies all ocular muscles apart from lateral rectus and superior oblique. Thus the affected eye will be deviated inferolaterally. Levator palpebrae superioris may also be impaired resulting in impaired ability to open the eye.
Trochlear nerve The eye will not be able to look down.
Trigeminal nerve Largest cranial nerve. Exits the brainstem at the pons. Branches are ophthalmic, maxillary and mandibular. Only the mandibular branch has both sensory and motor fibres. Branches converge to form the trigeminal ganglion (located in Meckels cave). It supplies the muscles of mastication and also tensor veli palatine, mylohyoid, anterior belly of digastric and tensor tympani. The detailed descriptions of the various sensory functions are described in other areas of the website. The corneal reflex is important and is elicited by applying a small tip of cotton wool to the cornea, a reflex blink should occur if it is intact. It is mediated by: the naso ciliary branch of the ophthalmic branch of the trigeminal (sensory component) and the facial nerve producing the motor response. Lesions of the afferent arc will produce bilateral absent blink and lesions of the efferent arc will result in a unilateral absent blink.
Abducens nerve The affected eye will have a deficit of abduction. This cranial nerve exits the brainstem between the pons and medulla. It thus has a relatively long intra cranial course which renders it susceptible to damage in raised intra cranial pressure.
Facial nerve Emerges from brainstem between pons and medulla. It controls muscles of facial expression and taste from the anterior 2/3 of the tongue. The nerve passes into the petrous temporal bone and into the internal auditory meatus. It then passes through the facial canal and exits at the stylomastoid foramen. It passes through the parotid gland and divides at this point. It does not innervate the parotid gland. Its divisions are considered in other parts of the website. Its motor fibres innervate orbicularis oculi to produce the efferent arm of the corneal reflex. In surgical practice it may be injured during parotid gland surgery or invaded by malignancies of the gland and a lower motor neurone on the ipsilateral side will result.
Vestibulo-cochlear nerve Exits from the pons and then passes through the internal auditory meatus. It is implicated in sensorineural hearing loss. Individuals with sensorineural hearing loss will localise the sound in webers test to the normal ear. Rinnes test will be reduced on the affected side but should still work. These two tests will distinguish sensorineural hearing loss from conductive deafness. In the latter condition webers test will localise to the affected ear and Rinnes test will be impaired on the affected side. Surgical lesions affecting this nerve include CNS tumours and basal skull fractures. It may also be damaged by the administration of ototoxic drugs (of which gentamicin is the most commonly used in surgical practice).
Glossopharyngeal nerve Exits the pons just above the vagus. Receives sensory fibres from posterior 1/3 tongue, tonsils, pharynx and middle ear (otalgia may occur following tonsillectomy). It receives visceral afferents from the carotid bodies. It supplies parasympathetic fibres to the parotid gland via the otic ganglion and motor function to stylopharyngeaus muscle. The sensory function of the nerve is tested using the gag reflex.
Vagus nerve Leaves the medulla between the olivary nucleus and the inferior cerebellar peduncle. Passes through the jugular foramen and into the carotid sheath. Details of the functions of the vagus nerve are covered in the website under relevant organ sub headings.
Accessory nerve Exists from the caudal aspect of the brainstem (multiple branches) supplies trapezius and sternocleidomastoid muscles. The distal portion of this nerve is most prone to injury during surgical procedures.
Hypoglossal nerve Emerges from the medulla at the preolivary sulcus, passes through the hypoglossal canal. It lies on the carotid sheath and passes deep to the posterior belly of digastric to supply muscles of the tongue (except palatoglossus). Its location near the carotid sheath makes it vulnerable during carotid endarterectomy surgery and damage will produce ipsilateral defect in muscle function.
A surgical trainee is incising a groin ‘abscess’ in an intravenous drug abuser. Unfortunately the ‘abscess’ is a false aneurysm and torrential bleeding ensues. In the panic of the situation the doctor then stabs himself in the finger. It transpires that the patient is a Hepatitis B carrier and the doctor is not immunised! What type of virus is Hepatitis B?
Double stranded DNA virus
Single stranded DNA virus
Double stranded RNA virus
Single stranded RNA virus
Retrovirus
Hepatitis B
Hepatitis B is a double-stranded DNA virus and is spread through exposure to infected blood or body fluids, including vertical transmission from mother to child. The incubation period is 6-20 weeks.
Immunisation against hepatitis B
Contains HBsAg absorbed onto aluminium hydroxide adjuvant and is prepared from yeast cells using recombinant DNA technology
Most schedules give 3 doses of the vaccine with a recommendation for a one-off booster 5 years following the initial primary vaccination
At risk groups who should be vaccinated include: healthcare workers, intravenous drug users, sex workers, close family contacts of an individual with hepatitis B, individuals receiving blood transfusions regularly, chronic kidney disease patients who may soon require renal replacement therapy, prisoners, chronic liver disease patients
Around 10-15% of adults fail to respond or respond poorly to 3 doses of the vaccine. Risk factors include age over 40 years, obesity, smoking, alcohol excess and immunosuppression
Testing for anti-HBs is only recommended for those at risk of occupational exposure (i.e. Healthcare workers) and patients with chronic kidney disease. In these patients anti-HBs levels should be checked 1-4 months after primary immunisation
The table below shows how to interpret anti-HBs levels:
Anti-HBs level (mIU/ml) Response
> 100 Indicates adequate response, no further testing required. Should still receive booster at 5 years
10 - 100 Suboptimal response - one additional vaccine dose should be given. If immunocompetent no further testing is required
< 10 Non-responder. Test for current or past infection. Give further vaccine course (i.e. 3 doses again) with testing following. If still fails to respond then HBIG would be required for protection if exposed to the virus
Complications of hepatitis B infection
Chronic hepatitis (5-10%)
Fulminant liver failure (1%)
Hepatocellular carcinoma
Glomerulonephritis
Polyarteritis nodosa
Cryoglobulinaemia
Management of hepatitis B
Pegylated interferon-alpha used to be the only treatment available. It reduces viral replication in up to 30% of chronic carriers. A better response is predicted by being female, < 50 years old, low HBV DNA levels, non-Asian, HIV negative, high degree of inflammation on liver biopsy
However, due to the side-effects of pegylated interferon it is now used less commonly in clinical practice. Oral antiviral medication is increasingly used with an aim to suppress viral replication (not in dissimilar way to treating HIV patients)
Examples include lamivudine, tenofovir and entecavir
A 7 year old boy presents with right iliac fossa pain and there is a clinical suspicion that appendicitis is present. From which of the following embryological structures is the appendix derived?
Vitello-intestinal duct
Urachus
Foregut
Hindgut
Midgut
The appendix is derived from the midgut
It is derived from the midgut which is why early appendicitis may present with periumbilical pain.
Appendix
Location: Base of caecum.
Up to 10cm long.
Mainly lymphoid tissue (Hence mesenteric adenitis may mimic appendicitis).
Caecal taenia coli converge at base of appendix and form a longitudinal muscle cover over the appendix. This convergence should facilitate its identification at surgery if it is retrocaecal and difficult to find (which it can be when people start doing appendicectomies!)
Arterial supply: Appendicular artery (branch of the ileocolic).
It is intra peritoneal.
McBurney’s point
1/3 of the way along a line drawn from the Anterior Superior Iliac Spine to the Umbilicus
6 Positions:
Retrocaecal 74%
Pelvic 21%
Postileal
Subcaecal
Paracaecal
Preileal
Which of the following is not a branch of the posterior cord of the brachial plexus?
Thoracodorsal nerve
Axillary nerve
Radial nerve
Lower subscapular nerve
Musculocutaneous nerve
Mnemonic branches off the posterior cord
S ubscapular (upper and lower)
T horacodorsal
A xillary
R adial
The musculocutaneous nerve is a branch off the lateral cord.
Brachial plexus
The brachial plexus extends from the neck to the axilla. It is formed by the ventral rami of the fifth to the eighth cervical nerves with the ascending part of the first thoracic nerve.
Location of the plexus
The ventral rami which form the plexus enter the lower part of the posterior triangle of the neck in series with the ventral rami of the cervical plexus. The second part of the subclavian artery lies immediately anterior to the lower two rami. The upper three rami intermingle and pass inferolaterally towards the axilla and subclavian artery. They are enclosed within an extension of the prevertebral fascia. In the neck the plexus lies deep to platysma, the supraclavicular nerves, inferior belly of omohyoid and the transverse cervical artery. It then passes deep to the clavicle and the suprascapular vessels, to enter the axilla, and thence surround the second part of the axillary artery
Composition of the plexus
Ventral rami, the roots of the plexus, lie between scalenus medius and anterior.
As they enter the posterior triangle, the upper two (C5,6) and lower two (C8, T1) roots of the plexus unite to form the upper and lower trunks of the plexus respectively. Meanwhile, C7 continues as the middle trunk. The lower trunk may groove the superior surface of the first rib posterior to the subclavian artery, and the root from the first ventral ramus is always in contact with it.
Each trunk divides into ventral and dorsal divisions which are destined to supply the anterior (flexor) and posterior (extensor) parts of the upper limb.
The cords of the plexus are formed in the axilla. The dorsal divisions unite to form the posterior cord (C5-8). The ventral divisions of the upper and middle trunks unite to form the lateral cord (C5-7), while the ventral divisions of the lower trunk continues as the medial cord (C8-T1). The cords are named according to their relationship to the axillary artery. Each cord terminates by dividing into two main branches at the beginning of the third part of the artery.
Sympathetic communications
The fifth and sixth cervical ventral rami receive grey rami communicantes from the middle cervical ganglion, while the two or more grey rami communicantes pass from the inferior cervical ganglion to the seventh and eighth cervical ventral rami. The first thoracic ventral ramus receives its grey ramus from the cervicothoracic ganglion. Its for this reason that inferior plexus injury can be complicated by a Horners syndrome.
Summary
Origin Anterior rami of C5 to T1
Sections of the plexus
Roots, trunks, divisions, cords, branches
Mnemonic:Real Teenagers Drink Cold Beer
Roots
Located in the posterior triangle
Pass between scalenus anterior and medius
Trunks
Located posterior to middle third of clavicle
Upper and middle trunks related superiorly to the subclavian artery
Lower trunk passes over 1st rib posterior to the subclavian artery
Divisions Apex of axilla
Cords Related to axillary artery
A 58 year old man is due to undergo a right hemicolectomy to treat an ascending colon cancer. This is a planned procedure and the surgeon decides to enroll the patient in an enhanced recovery programme following the surgery. Which of the interventions listed below is associated with reduced efficacy of enhanced recovery regimes?
Carbohydrate loading drinks prior to surgery
Early light diet following the procedure
Early mobilisation
IV fluid loading regimes during the procedure
Oral fluids rather than IV fluids post procedure
Enhanced recovery regimes try and avoid excessive IV fluids. These increase the rates of ileus in GI surgery. The oral route is preferred where possible.
Enhanced recovery programmes
Many surgical specialties have developed or adopted an enhanced recovery approach following procedures. Whilst the exact aspects differ according to the specialty involved, the broad principles remain the same.
Pre admission
Aim to optimise nutrition and any pre existing conditions. Prehabilitation programmes are used in some centres. Unit visits can be helpful to ensure patients know what to expect following the procedure.
Admission
In many cases admission on the day of the procedure is desirable and as near to the time of the procedure as possible.
Procedure related preparation
Try to minimise starvation times if GA is to be used. In many cases a carbohydrate loading drink is given 2 hours pre procedure. This is usually omitted in diabetic patients who are on complex insulin regimes.
For GI surgery, it is desirable to avoid mechanical bowel preparation unless this is essential.
Procedure
Minimally invasive procedures are preferred and sufficient analgesia to facilitate early ambulation. Avoid excessive IV fluids during the procedure as this can result in interstitial oedema and increase complication rates.
Post procedure
Aim for early ambulation, early removal of drains, catheters and IV lines where possible. Restart oral nutrition as soon as possible. Encourage patients to dress and walk about ward.
What is the lymphatic drainage of the membranous urethra?
Deep inguinal nodes
Superficial inguinal nodes
Internal iliac nodes
External iliac nodes
Para-aortic nodes
The prostatic and membranous urethra drain to the internal iliac nodes.
Urethral anatomy
Female urethra
The female urethra is shorter and more acutely angulated than the male urethra. It is an extra-peritoneal structure and embedded in the endopelvic fascia. The neck of the bladder is subjected to transmitted intra-abdominal pressure and therefore deficiency in this area may result in stress urinary incontinence. Between the layers of the urogenital diaphragm the female urethra is surrounded by the external urethral sphincter, this is innervated by the pudendal nerve. It ultimately lies anterior to the vaginal orifice.
Male urethra
In males the urethra is much longer and is divided into four parts.
Pre-prostatic urethra Extremely short and lies between the bladder and prostate gland.It has a stellate lumen and is between 1 and 1.5cm long.Innervated by sympathetic noradrenergic fibres, as this region is composed of striated muscles bundles they may contract and prevent retrograde ejaculation.
Prostatic urethra This segment is wider than the membranous urethra and contains several openings for the transmission of semen (at the midpoint of the urethral crest).
Membranous urethra Narrowest part of the urethra and surrounded by external sphincter. It traverses the perineal membrane 2.5cm postero-inferior to the symphysis pubis.
Penile urethra Travels through the corpus spongiosum on the underside of the penis. It is the longest urethral segment.It is dilated at its origin as the infrabulbar fossa and again in the glans penis as the navicular fossa. The bulbo-urethral glands open into the spongiose section of the urethra 2.5cm below the perineal membrane.
The urothelium is transitional in nature near to the bladder and becomes squamous more distally.
What is the most appropriate method of delivering early post-operative analgesia to a 6 month old child following an orchidopexy?
TAP block
Caudal block
Wound analgesic infusion catheter
Spinal block
Epidural block
Orchidopexy can be quite uncomfortable immediately following surgery. A caudal block can be a very effective adjunct and provides good analgesia. A spinal block and epidural would be inappropriate. A TAP block may cover the inguinal canal but this is not generally reliable and wound catheters are not used.
Management of pain
World Health Organisation Analgesic Ladder
Initially peripherally acting drugs such as paracetamol or non-steroidal anti-inflammatory drugs (NSAIDs) are given.
If pain control is not achieved, the second part of the ladder is to introduce weak opioid drugs such as codeine or dextropropoxyphene together with appropriate agents to control and minimise side effects.
The final rung of the ladder is to introduce strong opioid drugs such as morphine. Analgesia from peripherally acting drugs may be additive to that from centrally-acting opioids and thus, the two are given together.
The World Federation of Societies of Anaesthesiologists (WFSA) Analgesic Ladder
For management of acute pain
Initially, the pain can be expected to be severe and may need controlling with strong analgesics in combination with local anaesthetic blocks and peripherally acting drugs.
The second rung on the postoperative pain ladder is the restoration of the use of the oral route to deliver analgesia. Strong opioids may no longer be required and adequate analgesia can be obtained by using combinations of peripherally acting agents and weak opioids.
The final step is when the pain can be controlled by peripherally acting agents alone.
Local anaesthetics
Infiltration of a wound with a long-acting local anaesthetic such as Bupivacaine
Analgesia for several hours
Further pain relief can be obtained with repeat injections or by infusions via a thin catheter
Blockade of plexuses or peripheral nerves will provide selective analgesia in those parts of the body supplied by the plexus or nerves
Can either be used to provide anaesthesia for the surgery or specifically for postoperative pain relief
Especially useful where a sympathetic block is needed to improve postoperative blood supply or where central blockade such as spinal or epidural blockade is contraindicated.
Spinal anaesthesia
Provides excellent analgesia for surgery in the lower half of the body and pain relief can last many hours after completion of the operation if long-acting drugs containing vasoconstrictors are used.
- Side effects of spinal anaesthesia include: hypotension, sensory and motor block, nausea and urinary retention.
Epidural anaesthesia
An indwelling epidural catheter inserted. This can then be used to provide a continuous infusion of analgesic agents. It can provide excellent analgesia. They are still the preferred option following major open abdominal procedures and help prevent post operative respiratory compromise resulting from pain.
- Disadvantages of epidurals is that they usually confine patients to bed, especially if a motor block is present. In addition an indwelling urinary catheter is required. Which may not only impair mobility but also serve as a conduit for infection. They are contraindicated in coagulopathies.
Transversus Abdominal Plane block (TAP)
In this technique an ultrasound is used to identify the correct muscle plane and local anaesthetic (usually bupivicaine) is injected. The agent diffuses in the plane and blocks many of the spinal nerves. It is an attractive technique as it provides a wide field of blockade but does not require the placement of any indwelling devices. There is no post operative motor impairment. For this reason it is the preferred technique when extensive laparoscopic abdominal procedures are performed. They will then provide analgesia immediately following surgery but as they do not confine the patient to bed, the focus on enhanced recovery can begin sooner.
-The main disadvantage is that their duration of action is limited to the half life of the local anaesthetic agent chosen. In addition some anaesthetists do not have the USS skills required to site the injections.
Patient Controlled Analgesia (PCA)
- Patients administer their own intravenous analgesia and titrate the dose to their own end-point of pain relief using a small microprocessor - controlled pump. Morphine is the most popular drug used.
Strong Opioids
Severe pain arising from deep or visceral structures requires the use of strong opioids
Morphine
Short half life and poor bioavailability.
Metabolised in the liver and clearance is reduced in patients with liver disease, in the elderly and the debilitated
Side effects include nausea, vomiting, constipation and respiratory depression.
Tolerance may occur with repeated dosage
Pethidine
Synthetic opioid which is structurally different from morphine but which has similar actions. Has 10% potency of morphine.
Short half life and similar bioavailability and clearance to morphine.
Short duration of action and may need to be given hourly.
Pethidine has a toxic metabolite (norpethidine) which is cleared by the kidney, but which accumulates in renal failure or following frequent and prolonged doses and may lead to muscle twitching and convulsions. Extreme caution is advised if pethidine is used over a prolonged period or in patients with renal failure.
Weak opioids
Codeine: markedly less active than morphine, has predictable effects when given orally and is effective against mild to moderate pain.
Non opioid analgesics
- Mild to moderate pain.
Paracetamol
Inhibits prostaglandin synthesis.
Analgesic and antipyretic properties but little anti-inflammatory effect
It is well absorbed orally and is metabolised almost entirely in the liver
Side effects in normal dosage and is widely used for the treatment of minor pain. It causes hepatotoxicity in over dosage by overloading the normal metabolic pathways with the formation of a toxic metabolite.
NSAIDs
Analgesic and anti-inflammatory actions
Inhibition of prostaglandin synthesis by the enzyme Cyclooxygenase which catalyses the conversion of arachidonic acid to the various prostaglandins that are the chief mediators of inflammation. All NSAIDs work in the same way and thus there is no point in giving more than one at a time. .
NSAIDs are, in general, more useful for superficial pain arising from the skin, buccal mucosa, joint surfaces and bone.
Relative contraindications: history of peptic ulceration, gastrointestinal bleeding or bleeding diathesis; operations associated with high blood loss, asthma, moderate to severe renal impairment, dehydration and any history of hypersensitivity to NSAIDs or aspirin.
Neuropathic pain
National Institute of Clinical Excellence (UK) guidelines:
First line: Amitriptyline (Imipramine if cannot tolerate) or pregabalin
Second line: Amitriptyline AND pregabalin
Third line: refer to pain specialist. Give tramadol in the interim (avoid morphine)
If diabetic neuropathic pain: Duloxetine
As of 1 April 2019, pregabalin and gabapentin are Class C controlled substances (under the Misuse of Drugs Act 1971) and scheduled under the Misuse of Drugs Regulations 2001 as Schedule 3. Evaluate patients carefully for a history of drug abuse before prescribing and observe patients for development of signs of abuse and dependence (MHRA, Drug Safety Update April 2019).
What is the course of the median nerve relative to the brachial artery in the upper arm?
Medial to anterior to lateral
Lateral to posterior to medial
Medial to posterior to lateral
Medial to anterior to medial
Lateral to anterior to medial
Relations of median nerve to the brachial artery:
Lateral -> Anterior -> Medial
The median nerve descends lateral to the brachial artery, it usually passes anterior to the artery to lie on its medial side. It passes deep to the bicipital aponeurosis and the median cubital vein at the elbow. It enters the forearm between the two heads of the pronator teres muscle.
Brachial artery
The brachial artery begins at the lower border of teres major as a continuation of the axillary artery. It terminates in the cubital fossa at the level of the neck of the radius by dividing into the radial and ulnar arteries.
Relations
Posterior relations include the long head of triceps with the radial nerve and profunda vessels intervening. Anteriorly it is overlapped by the medial border of biceps.
It is crossed by the median nerve in the middle of the arm.
In the cubital fossa it is separated from the median cubital vein by the bicipital aponeurosis.
The basilic vein is in contact at the most proximal aspect of the cubital fossa and lies medially.
Which of the following stimulates prolactin release or action?
Leutinising hormone
Dopamine
Thyrotropin releasing hormone
Oestrogen
Follicle stimulating hormone
Dopamine: Dopamine, acting as a PRL-inhibitory factor (PIF), inhibits prolactin release. When dopamine levels decrease (such as during suckling), there is a disinhibition effect, leading to prolactin secretion12.
Thyrotropin Releasing Hormone (TRH): TRH, produced by the hypothalamus, stimulates the release of prolactin. Additionally, oestrogens enhance lactotroph sensitivity to TRH and reduce their sensitivity to dopamine inhibition1.
In summary, dopamine suppresses prolactin, while TRH and oestrogens promote its release. The other hormones listed (luteinizing hormone and follicle-stimulating hormone) do not directly influence prolactin13.
Remember, prolactin plays a crucial role in breast development, milk production, and other physiological processes, particularly in females
Prolactin
Prolactin is a peptide hormone released from the anterior pituitary. It is under tonic dopamine inhibition, thyrotropin releasing hormone has a stimulatory effect on release. Prolactin release stimulates milk production but also reduces gonadal activity. It decreases GnRH pulsatility at the hypothalamic level and to a lesser extent, blocks the action of LH on the ovary or testis.
A 43 year old lady develops a cerebello-pontine angle lesion. Which of the nerves listed below is likely to be affected first?
CN X
CN III
CN V
CN IX
CN XII
The most likely lesion to occur in the cerebello-pontine angle is an acoustic neuroma.
The trigeminal nerve has a broad base and involvement of at least part of this nerve is the most likely initial finding. The defect may be subtle such as loss of the ipsilateral corneal reflex. Ipsilateral hearing loss will also occur. Untreated, progressive lesions, may ultimately affect cranial nerve roots in this region.
Cranial nerves
Cranial nerve lesions
Olfactory nerve May be injured in basal skull fractures or involved in frontal lobe tumour extension. Loss of olfactory nerve function in relation to major CNS pathology is seldom an isolated event and thus it is poor localiser of CNS pathology.
Optic nerve Problems with visual acuity may result from intra ocular disorders. Problems with the blood supply such as amaurosis fugax may produce temporary visual distortion. More important surgically is the pupillary response to light. The pupillary size may be altered in a number of disorders. Nerves involved in the resizing of the pupil connect to the pretectal nucleus of the high midbrain, bypassing the lateral geniculate nucleus and the primary visual cortex. From the pretectal nucleus neurones pass to the Edinger - Westphal nucleus, motor axons from here pass along with the oculomotor nerve. They synapse with ciliary ganglion neurones; the parasympathetic axons from this then innervate the iris and produce miosis. The miotic pupil is seen in disorders such as Horner’s syndrome or opiate overdose.
Mydriasis is the dilatation of the pupil in response to disease, trauma, drugs (or the dark!). It is pathological when light fails to induce miosis. The radial muscle is innervated by the sympathetic nervous system. Because the parasympathetic fibres travel with the oculomotor nerve they will be damaged by lesions affecting this nerve (e.g. cranial trauma).
The response to light shone in one eye is usually a constriction of both pupils. This indicates intact direct and consensual light reflexes. When the optic nerve has an afferent defect the light shining on the affected eye will produce a diminished pupillary response in both eyes. Whereas light shone on the unaffected eye will produce a normal pupillary response in both eyes. This is referred to as the Marcus Gunn pupil and is seen in conditions such as optic neuritis. In a total CN II lesion shining the light in the affected eye will produce no response.
Oculomotor nerve The pupillary effects are described above. In addition it supplies all ocular muscles apart from lateral rectus and superior oblique. Thus the affected eye will be deviated inferolaterally. Levator palpebrae superioris may also be impaired resulting in impaired ability to open the eye.
Trochlear nerve The eye will not be able to look down.
Trigeminal nerve Largest cranial nerve. Exits the brainstem at the pons. Branches are ophthalmic, maxillary and mandibular. Only the mandibular branch has both sensory and motor fibres. Branches converge to form the trigeminal ganglion (located in Meckels cave). It supplies the muscles of mastication and also tensor veli palatine, mylohyoid, anterior belly of digastric and tensor tympani. The detailed descriptions of the various sensory functions are described in other areas of the website. The corneal reflex is important and is elicited by applying a small tip of cotton wool to the cornea, a reflex blink should occur if it is intact. It is mediated by: the naso ciliary branch of the ophthalmic branch of the trigeminal (sensory component) and the facial nerve producing the motor response. Lesions of the afferent arc will produce bilateral absent blink and lesions of the efferent arc will result in a unilateral absent blink.
Abducens nerve The affected eye will have a deficit of abduction. This cranial nerve exits the brainstem between the pons and medulla. It thus has a relatively long intra cranial course which renders it susceptible to damage in raised intra cranial pressure.
Facial nerve Emerges from brainstem between pons and medulla. It controls muscles of facial expression and taste from the anterior 2/3 of the tongue. The nerve passes into the petrous temporal bone and into the internal auditory meatus. It then passes through the facial canal and exits at the stylomastoid foramen. It passes through the parotid gland and divides at this point. It does not innervate the parotid gland. Its divisions are considered in other parts of the website. Its motor fibres innervate orbicularis oculi to produce the efferent arm of the corneal reflex. In surgical practice it may be injured during parotid gland surgery or invaded by malignancies of the gland and a lower motor neurone on the ipsilateral side will result.
Vestibulo-cochlear nerve Exits from the pons and then passes through the internal auditory meatus. It is implicated in sensorineural hearing loss. Individuals with sensorineural hearing loss will localise the sound in webers test to the normal ear. Rinnes test will be reduced on the affected side but should still work. These two tests will distinguish sensorineural hearing loss from conductive deafness. In the latter condition webers test will localise to the affected ear and Rinnes test will be impaired on the affected side. Surgical lesions affecting this nerve include CNS tumours and basal skull fractures. It may also be damaged by the administration of ototoxic drugs (of which gentamicin is the most commonly used in surgical practice).
Glossopharyngeal nerve Exits the pons just above the vagus. Receives sensory fibres from posterior 1/3 tongue, tonsils, pharynx and middle ear (otalgia may occur following tonsillectomy). It receives visceral afferents from the carotid bodies. It supplies parasympathetic fibres to the parotid gland via the otic ganglion and motor function to stylopharyngeaus muscle. The sensory function of the nerve is tested using the gag reflex.
Vagus nerve Leaves the medulla between the olivary nucleus and the inferior cerebellar peduncle. Passes through the jugular foramen and into the carotid sheath. Details of the functions of the vagus nerve are covered in the website under relevant organ sub headings.
Accessory nerve Exists from the caudal aspect of the brainstem (multiple branches) supplies trapezius and sternocleidomastoid muscles. The distal portion of this nerve is most prone to injury during surgical procedures.
Hypoglossal nerve Emerges from the medulla at the preolivary sulcus, passes through the hypoglossal canal. It lies on the carotid sheath and passes deep to the posterior belly of digastric to supply muscles of the tongue (except palatoglossus). Its location near the carotid sheath makes it vulnerable during carotid endarterectomy surgery and damage will produce ipsilateral defect in muscle function.
Which of the agents listed below can be administered via the peripheral intra venous route in the non cardiac arrest setting?
Milrinone
Noradrenaline
Adrenaline
Metaraminol
Dobutamine
Metaraminol is an alpha receptor agonist.
As a general rule, inotropes and vasopressors can only be administered via a central vein. Metaraminol is an exception to this as it can be administered via a peripheral line.
Circulatory support of the critically ill
Circulatory support
Impaired tissue oxygenation may occur as a result of circulatory shock. Shock is considered further under its own topic heading.
Patients requiring circulatory support require haemodynamic monitoring. At its simplest level this may simply be in the form of regular urine output measurements and blood pressure monitoring. In addition ECG monitoring will allow the identification of cardiac arrhythmias. Pulse oximeter measurements will allow quick estimation haemoglobin oxygen saturation in arterial blood.
Invasive arterial blood pressure monitoring is undertaken by the use of an indwelling arterial line. Most arterial sites can be used although the radial artery is the commonest. It is important not to cannulate end arteries. The arterial trace can be tracked to ventilation phases and those patients whose systolic pressure varies with changes in intrathoracic pressure may benefit from further intravenous fluids.
Central venous pressure is measured using a CVP line that is usually sited in the superior vena cava via the internal jugular route. The CVP will demonstrate right atrial filling pressure and volume status. When adequate intra vascular volume is present a fluid challenge will typically cause a prolonged rise in CVP (usually greater than 6-8mmHg).
To monitor the cardiac output a Swan-Ganz catheter is traditionally inserted (other devices may be used and are less invasive). Inflation of the distal balloon will provide the pulmonary artery occlusion pressure and the pressure distal to the balloon will equate to the left atrial pressure. This gives a measure of left ventricular preload. Because the Swan-Ganz catheter can measure several variables it can be used to calculate:
Stroke volume
Systemic vascular resistance
Pulmonary artery resistance
Oxygen delivery (and consumption)
Inotropes
In patients with an adequate circulating volume but on-going circulatory compromise a vasoactive drug may be considered. These should usually be administered via the central venous route. Commonly used inotropes include:
Agent Mode of action Effect
Noradrenaline α agonist Vasopressor action, minimal effect on cardiac output
Adrenaline α and β receptor agonist Increases cardiac output and peripheral vascular resistance
Dopamine β1 agonist Increases contractility and rate
Dobutamine β1 and β2 agonist Increases cardiac output and decreases SVR
Milrinone Phosphodiesterase inhibitor Elevation of cAMP levels improves muscular contractility, short half life and acts as vasodilator
What is the most appropriate management for a 56 year old lady who has shooting pains in her arm following a mastectomy and axillary node clearance?
Carbamazepine
Pregabalin
Oramorph
Diclofenac
Chemical neurectomy
Pregabalin is generally the first line agent for neuropathic pain.
Management of pain
World Health Organisation Analgesic Ladder
Initially peripherally acting drugs such as paracetamol or non-steroidal anti-inflammatory drugs (NSAIDs) are given.
If pain control is not achieved, the second part of the ladder is to introduce weak opioid drugs such as codeine or dextropropoxyphene together with appropriate agents to control and minimise side effects.
The final rung of the ladder is to introduce strong opioid drugs such as morphine. Analgesia from peripherally acting drugs may be additive to that from centrally-acting opioids and thus, the two are given together.
The World Federation of Societies of Anaesthesiologists (WFSA) Analgesic Ladder
For management of acute pain
Initially, the pain can be expected to be severe and may need controlling with strong analgesics in combination with local anaesthetic blocks and peripherally acting drugs.
The second rung on the postoperative pain ladder is the restoration of the use of the oral route to deliver analgesia. Strong opioids may no longer be required and adequate analgesia can be obtained by using combinations of peripherally acting agents and weak opioids.
The final step is when the pain can be controlled by peripherally acting agents alone.
Local anaesthetics
Infiltration of a wound with a long-acting local anaesthetic such as Bupivacaine
Analgesia for several hours
Further pain relief can be obtained with repeat injections or by infusions via a thin catheter
Blockade of plexuses or peripheral nerves will provide selective analgesia in those parts of the body supplied by the plexus or nerves
Can either be used to provide anaesthesia for the surgery or specifically for postoperative pain relief
Especially useful where a sympathetic block is needed to improve postoperative blood supply or where central blockade such as spinal or epidural blockade is contraindicated.
Spinal anaesthesia
Provides excellent analgesia for surgery in the lower half of the body and pain relief can last many hours after completion of the operation if long-acting drugs containing vasoconstrictors are used.
- Side effects of spinal anaesthesia include: hypotension, sensory and motor block, nausea and urinary retention.
Epidural anaesthesia
An indwelling epidural catheter inserted. This can then be used to provide a continuous infusion of analgesic agents. It can provide excellent analgesia. They are still the preferred option following major open abdominal procedures and help prevent post operative respiratory compromise resulting from pain.
- Disadvantages of epidurals is that they usually confine patients to bed, especially if a motor block is present. In addition an indwelling urinary catheter is required. Which may not only impair mobility but also serve as a conduit for infection. They are contraindicated in coagulopathies.
Transversus Abdominal Plane block (TAP)
In this technique an ultrasound is used to identify the correct muscle plane and local anaesthetic (usually bupivicaine) is injected. The agent diffuses in the plane and blocks many of the spinal nerves. It is an attractive technique as it provides a wide field of blockade but does not require the placement of any indwelling devices. There is no post operative motor impairment. For this reason it is the preferred technique when extensive laparoscopic abdominal procedures are performed. They will then provide analgesia immediately following surgery but as they do not confine the patient to bed, the focus on enhanced recovery can begin sooner.
-The main disadvantage is that their duration of action is limited to the half life of the local anaesthetic agent chosen. In addition some anaesthetists do not have the USS skills required to site the injections.
Patient Controlled Analgesia (PCA)
- Patients administer their own intravenous analgesia and titrate the dose to their own end-point of pain relief using a small microprocessor - controlled pump. Morphine is the most popular drug used.
Strong Opioids
Severe pain arising from deep or visceral structures requires the use of strong opioids
Morphine
Short half life and poor bioavailability.
Metabolised in the liver and clearance is reduced in patients with liver disease, in the elderly and the debilitated
Side effects include nausea, vomiting, constipation and respiratory depression.
Tolerance may occur with repeated dosage
Pethidine
Synthetic opioid which is structurally different from morphine but which has similar actions. Has 10% potency of morphine.
Short half life and similar bioavailability and clearance to morphine.
Short duration of action and may need to be given hourly.
Pethidine has a toxic metabolite (norpethidine) which is cleared by the kidney, but which accumulates in renal failure or following frequent and prolonged doses and may lead to muscle twitching and convulsions. Extreme caution is advised if pethidine is used over a prolonged period or in patients with renal failure.
Weak opioids
Codeine: markedly less active than morphine, has predictable effects when given orally and is effective against mild to moderate pain.
Non opioid analgesics
- Mild to moderate pain.
Paracetamol
Inhibits prostaglandin synthesis.
Analgesic and antipyretic properties but little anti-inflammatory effect
It is well absorbed orally and is metabolised almost entirely in the liver
Side effects in normal dosage and is widely used for the treatment of minor pain. It causes hepatotoxicity in over dosage by overloading the normal metabolic pathways with the formation of a toxic metabolite.
NSAIDs
Analgesic and anti-inflammatory actions
Inhibition of prostaglandin synthesis by the enzyme Cyclooxygenase which catalyses the conversion of arachidonic acid to the various prostaglandins that are the chief mediators of inflammation. All NSAIDs work in the same way and thus there is no point in giving more than one at a time. .
NSAIDs are, in general, more useful for superficial pain arising from the skin, buccal mucosa, joint surfaces and bone.
Relative contraindications: history of peptic ulceration, gastrointestinal bleeding or bleeding diathesis; operations associated with high blood loss, asthma, moderate to severe renal impairment, dehydration and any history of hypersensitivity to NSAIDs or aspirin.
Neuropathic pain
National Institute of Clinical Excellence (UK) guidelines:
First line: Amitriptyline (Imipramine if cannot tolerate) or pregabalin
Second line: Amitriptyline AND pregabalin
Third line: refer to pain specialist. Give tramadol in the interim (avoid morphine)
If diabetic neuropathic pain: Duloxetine
As of 1 April 2019, pregabalin and gabapentin are Class C controlled substances (under the Misuse of Drugs Act 1971) and scheduled under the Misuse of Drugs Regulations 2001 as Schedule 3. Evaluate patients carefully for a history of drug abuse before prescribing and observe patients for development of signs of abuse and dependence (MHRA, Drug Safety Update April 2019).
References
1. http://guidance.nice.org.uk/CG173/Guidance/pdf/English
2. Lovich-Sapola J, Smith CE, Brandt CP. Post operative pain control. Surg Clin North Am. 2015 Apr;95(2):301-183. Finnerup N et al. Pharmacotherapy for neuropathic pain in adults: a systematic review and meta-analysis. Lancet Neurol. 2015 Feb;14(2):162-73.
A 32 year old lady presents with a 1.5cm pigmented lesion on her back. The surgeon is concerned that this may be a melanoma. What is the most appropriate course of action?
2mm punch biopsy from the centre of the lesion
4mm punch biopsy from the centre of the lesion
Wide excision of the lesion with 3cm margins
Excisional biopsy of the lesion
Wide excision of the lesion with 1cm margins
Suspicious naevi should NOT be partially sampled as histological interpretation is severely compromised. Complete excision is mandatory where lesions fulfil diagnostic criteria. However, wide excision for margins may be deferred until definitive histology is available.
Lesions that are suspicious for melanoma should be excised with complete margins. Radical excision is not routinely undertaken for diagnostic purposes and therefore if subsequent histopathological assessment determines that the lesion is a melanoma a re-exicision of margins may be required. Incisional punch biopsies of potential melanomas makes histological interpretation difficult and is best avoided
Malignant melanoma
The main diagnostic features (major criteria):
Change in size
Change in shape
Change in colour
Secondary features (minor criteria)
Diameter >6mm
Inflammation
Oozing or bleeding
Altered sensation
Treatment
Suspicious lesions should undergo excision biopsy. The lesion should be removed in completely as incision biopsy can make subsequent histopathological assessment difficult.
Once the diagnosis is confirmed the pathology report should be reviewed to determine whether further re-excision of margins is required (see below):
Margins of excision-Related to Breslow thickness
Lesions 0-1mm thick 1cm
Lesions 1-2mm thick 1- 2cm (Depending upon site and pathological features)
Lesions 2-4mm thick 2-3 cm (Depending upon site and pathological features)
Lesions >4 mm thick 3cm
Marsden J et al Revised UK guidelines for management of Melanoma. Br J Dermatol 2010 163:238-256.
Further treatments such as sentinel lymph node mapping, isolated limb perfusion and block dissection of regional lymph node groups should be selectively applied.
Guidelines
The UK NICE guidance is covered by Melanoma: assessment and management NICE guideline [NG14]. However, the complex oncological regimens are not extensively covered in this guidance though more formal guidance is provided by relevant oncological organisations.
What is the correct embryological origin of the stapes?
First pharyngeal arch
Second pharyngeal arch
Third pharyngeal arch
Fourth pharyngeal arch
Fifth pharyngeal arch
Embryological origin stapes = 2nd pharyngeal arch
The ectoderm covering the outer aspect of the second arch originates from a strip of ectoderm lateral to the metencephalic neural fold. The cartilaginous element to this, eponymously known as Reicherts cartilage extends from the otic capsule to the midline on each side. Its dorsal end separates and becomes enclosed in the tympanic cavity as the stapes.
The dorsal ends of the cartilages of the first and second pharyngeal arches articulate superior to the tubotympanic recess. These cartilages form the malleus, incus and stapes. At least part of the malleus is formed from the first arch and the stapes from the second arch. The incus is most likely to arise from the first arch.
Pharyngeal arches
These develop during the fourth week of embryonic growth from a series of mesodermal outpouchings of the developing pharynx.
They develop and fuse in the ventral midline. Pharyngeal pouches form on the endodermal side between the arches.
There are 6 pharyngeal arches, the fifth does not contribute any useful structures and often fuses with the sixth arch.
Pharyngeal arches
Pharyngeal arch Muscular contributions Skeletal contributions Endocrine Artery Nerve
First Muscles of mastication
Anterior belly of digastric
Mylohyoid
Tensor tympanic
Tensor veli palatini Maxilla
Meckels cartilage
Incus
Malleus n/a Maxillary
External carotid Mandibular
Second Buccinator
Platysma
Muscles of facial expression
Stylohyoid
Posterior belly of digastric
Stapedius Stapes
Styloid process
Lesser horn and upper body of hyoid n/a Inferior branch of superior thyroid artery
Stapedial artery Facial
Third Stylopharyngeus Greater horn and lower part of hyoid Thymus
Inferior parathyroids Common and internal carotid Glossopharyngeal
Fourth Cricothyroid
All intrinsic muscles of the soft palate Thyroid and epiglottic cartilages Superior parathyroids Right- subclavian artery, Left-aortic arch Vagus
Sixth All intrinsic muscles of the larynx (except cricothyroid) Cricoid, arytenoid and corniculate cartilages n/a Right -Pulmonary artery, Left- Pulmonary artery and ductus arteriosus Vagus and recurrent laryngeal nerve
A 20 year old woman sustains a Holstein-Lewis fracture. Which nerve is at risk?
Ulnar
Radial
Median
Musculocutaneous
Axillary
Since the distal humerus is affected, the radial is at risk.
Eponymous fractures
Colles’ fracture (dinner fork deformity)
Fall onto extended outstretched hand
Classical Colles’ fractures have the following 3 features:
- Transverse fracture of the radius
- 1 inch proximal to the radio-carpal joint
- Dorsal displacement and angulation
Smith’s fracture (reverse Colles’ fracture)
Volar angulation of distal radius fragment (Garden spade deformity)
Caused by falling backwards onto the palm of an outstretched hand or falling with wrists flexed
Bennett’s fracture
Intra-articular fracture of the first carpometacarpal joint
Impact on flexed metacarpal, caused by fist fights
Monteggia’s fracture
Dislocation of the proximal radioulnar joint in association with an ulna fracture
Fall on outstretched hand with forced pronation
Needs prompt diagnosis to avoid disability
Galeazzi fracture
Radial shaft fracture with associated dislocation of the distal radioulnar joint
Direct blow
Pott’s fracture
Bimalleolar ankle fracture
Forced foot eversion
Barton’s fracture
Distal radius fracture (Colles’/Smith’s) with associated radiocarpal dislocation
Fall onto extended and pronated wrist
Involvement of the joint is a defining feature
Holstein Lewis Fracture
A HolsteinLewis fracture is a fracture of the distal third of the humerus resulting in entrapment of the radial nerve.
The radial nerve is one of the major nerves of the upper limb. It innervates all of the muscles in the extensor compartments of the arm.
Conservative treatment includes reduction and use of a functional brace
Vascular injury may require open surgery
What is the level of the hyoid bone?
C1
C2
C3
C4
C5
Surface anatomy of the neck
In the midline from above down, the following structures are felt
Structure Level
Hyoid C3
Notch of the thyroid cartilage C4
Cricoid cartilage(termination) C6
The lower border of the cricoid cartilage corresponds to the commencement of the trachea and also to the following:
Junction of larynx with trachea
Junction of pharynx with oesophagus
Level at which the inferior thyroid artery enters the thyroid gland
The level at which the vertebral artery enters the transverse foramen in the 6th cervical vertebra
Level at which the superior belly of omohyoid crosses the carotid sheath
The level of the middle cervical sympathetic ganglion
The level at which the carotid artery can be compressed against the transverse process of C6 (carotid tubercle).
A person is diagnosed with Conns syndrome. If the parotid gland secretions were assayed, what change (if any) will be noted?
Decreased potassium
Decreased sodium
Increased sodium
Potassium levels unchanged, sodium levels increased
Potassium levels raised sodium levels unchanged
Decreased sodium.
Aldosterone affects the parotid gland secretions and has the effect of conserving sodium (so salivary sodium reduced) and potassium secretion increased.
Parotid gland secretions
- Parotid gland secretions account for around 25% of salivary secretions
Levels of sodium and chloride are lower than plasma, potassium and bicarbonate levels are higher
Parotid secretions are watery and have high enzyme concentrations
Parasympathetic stimulation produces a water rich, serous saliva. Sympathetic stimulation leads to the production of a low volume, enzyme-rich saliva.
Fluid that is secreted within the acini is isotonic with plasma and undergoes modification within the ducts via process of ion exchange
The ducts are affected by aldosterone and so will retain sodium and secrete potassium when aldosterone levels are elevated
Which of the following statements relating to the Cavernous Sinus is false?
The pituitary gland lies medially
The internal carotid artery passes through it
The temporal lobe of the brain is a lateral relation
The mandibular branch of the trigeminal and optic nerve lie on the lateral wall
The ophthalmic veins drain into the anterior aspect of the sinus
The veins that drain into the sinus are important as sepsis can cause cavernous sinus thrombosis. The maxillary branch of the trigeminal and not the mandibular branches pass through the sinus
Cavernous sinus
The cavernous sinuses are paired and are situated on the body of the sphenoid bone. It runs from the superior orbital fissure to the petrous temporal bone.
Relations
Medial Lateral
Pituitary fossa
Sphenoid sinus Temporal lobe
Contents
Lateral wall components (from top to bottom:)
Oculomotor nerve
Trochlear nerve
Ophthalmic nerve
Maxillary nerve
Contents of the sinus (from medial to lateral:)
Internal carotid artery (and sympathetic plexus)
Abducens nerve
Blood supply
Ophthalmic vein, superficial cortical veins, basilar plexus of veins posteriorly.
Drains into the internal jugular vein via: the superior and inferior petrosal sinuses
A 25 year old man is diagnosed as having an undisplaced fracture of the proximal pole of the scaphoid. What is the best course of action?
Immobilisation in future splint for 5 weeks
Arrange an MRI scan
Immobilisation in plaster cast for 4 weeks
Surgical fixation
Initial immobisation in plaster cast for 2 weeks with check radiographs at that stage
It is generally accepted that proximal pole fractures of the scaphoid should be surgically fixed as non union rates of up to 34% can be seen when cast immobilization alone is attempted.
Scaphoid fractures
- Incidence of scaphoid fractures in UK ranges from 12.4 per 100,000 to 29 per 100,000
Surface of scaphoid is covered by articular cartilage with small area available for blood vessels (fracture risks blood supply)
Forms floor of anatomical snuffbox
Risk of fracture associated with fall onto outstretched hand (tubercle, waist, or proximal third)
A series of 4 scaphoid radiographs should be undertaken (PA, pronated oblique, Ziter view and lateral view). The Ziter view is a PA view with the wrist in ulnar deviation and beam angulated at 20 degrees
Sensitivity of scaphoid radiographs in 1st week of injury is 80%
Immobilization of scaphoid fractures difficult
Repeat imaging should be done at 10 days. MRI should be done in cases of diagnostic uncertainty
Classification of scaphoid fractures
Scaphoid tubercle
Distal pole
Waist
Proximal pole
Management
Undisplaced fractures of the waist of the scaphoid and most distal pole fractures can be managed in a cast for 6 weeks with high rates of union.
Displaced scaphoid waist fractures (more than 1-2mm) should be viewed as unstable and surgically fixed.
All proximal pole fractures should be fixed surgically.
Complications
Non union of scaphoid
Avascular necrosis of the scaphoid
Scapholunate disruption and wrist collapse
Degenerative changes of the adjacent joint
Which of the following does not exit the pelvis through the greater sciatic foramen?
Superior gluteal artery
Internal pudendal vessels
Sciatic nerve
Obturator nerve
Inferior gluteal nerve
The obturator nerve exits through the obturator foramen.
Greater sciatic foramen
Contents
Nerves
Sciatic Nerve
Superior and Inferior Gluteal Nerves
Pudendal Nerve
Posterior Femoral Cutaneous Nerve
Nerve to Quadratus Femoris
Nerve to Obturator internus
Vessels
Superior Gluteal Artery and vein
Inferior Gluteal Artery and vein
Internal Pudendal Artery and vein
Piriformis
The piriformis is a landmark for identifying structures passing out of the sciatic notch
Above piriformis: Superior gluteal vessels
Below piriformis: Inferior gluteal vessels, sciatic nerve (10% pass through it, <1% above it), posterior cutaneous nerve of the thigh
Greater sciatic foramen boundaries
Anterolaterally Greater sciatic notch of the ilium
Posteromedially Sacrotuberous ligament
Inferior Sacrospinous ligament and the ischial spine
Superior Anterior sacroiliac ligament
Structures passing between both foramina (Medial to lateral)
Pudendal nerve
Internal pudendal artery
Nerve to obturator internus
Contents of the lesser sciatic foramen
Tendon of the obturator internus
Pudendal nerve
Internal pudendal artery and vein
Nerve to the obturator internus
Which part of the jugular venous waveform is associated with the closure of the tricuspid valve?
a wave
c wave
x descent
y descent
v wave
JVP: C wave - closure of the tricuspid valve
The c wave of the jugular venous waveform is associated with the closure of the tricuspid valve.
Jugular venous pressure
As well as providing information on right atrial pressure, the jugular vein waveform may provide clues to underlying valvular disease. A non-pulsatile JVP is seen in superior vena caval obstruction. Kussmaul’s sign describes a paradoxical rise in JVP during inspiration seen in constrictive pericarditis
‘a’ wave = atrial contraction
large if atrial pressure e.g. tricuspid stenosis, pulmonary stenosis, pulmonary hypertension
absent if in atrial fibrillation
Cannon ‘a’ waves
caused by atrial contractions against a closed tricuspid valve
are seen in complete heart block, ventricular tachycardia/ectopics, nodal rhythm, single chamber ventricular pacing
‘c’ wave
closure of tricuspid valve
not normally visible
‘v’ wave
due to passive filling of blood into the atrium against a closed tricuspid valve
giant v waves in tricuspid regurgitation
‘x’ descent = fall in atrial pressure during ventricular systole
‘y’ descent = opening of tricuspid valve
A 45 year old patient undergoes a CT scan of the abdomen and is noted to have a 6cm mass in the right adrenal gland. Urinary catecholamines and other endocrine investigations are negative. CT of the chest and remainder of the abdomen is otherwise normal. What is the most appropriate course of action?
Image guided FNAC of the adrenal gland
Image guided core biopsy of the adrenal gland
List the patient for an adrenalectomy
Organise surveillance of the lesion with CT scanning
Organise surveillance of the lesion with USS
Most surgeons would excise a mass of this size rather than attempt biopsy. Further information relating to adrenal masses is covered under this topic.
Tissue sampling
Tissue sampling is an important surgical process. Biopsy modalities vary according to the site, experience and subsequent planned therapeutic outcome
The modalities comprise:
-Fine needle aspiration cytology
-Core biopsy
-Excision biopsy
-Tru cut biopsy
-Punch biopsy
-Cytological smears
-Endoscopic or laparoscopic biopsy
When the lesion is superficial the decision needs to be taken as to whether complete excision is desirable or whether excision biopsy is acceptable. In malignant melanoma for example the need for safe margins will mean that a more radical surgical approach needs to be adopted after diagnostic confirmation from excision biopsy than would be the case in basal cell carcinoma. Punch biopsies are useful in gaining histological diagnosis of unclear skin lesions where excision biopsy is undesirable such as in establishing whether a skin lesion is vasculitic or not.
Fine needle aspiration cytology (FNAC) is an operator dependent procedure that may or may not be image guided and essentially involves passing a needle through a lesion whilst suction is applied to a syringe. The material thus obtained is expressed onto a slide and sent for cytological assessment. This test can be limited by operator inexperience and also by the lack of histological architectural information (e.g. Follicular carcinoma of the thyroid). Where a discharge is present a sample may be sent for cytology although in some sites (e.g. Nipple discharge ) the information gleaned may be meaningless.
Tissue samples may be obtained by both core and tru cut biopsy. A core biopsy is obtained by use of a spring loaded gun with a needle passing quickly through the lesion of interest. A tru cut biopsy achieves the same objective but the needle moved by hand. When performing these techniques image guidance may be desirable (e.g. In breast lesions). Consideration needs to be given to any planned surgical resection as it may be necessary to resect the biopsy tract along with the specimen (e.g. In sarcoma surgery).
Visceral lesions may be accessed percutaneously under image guidance such as ultrasound guided biopsy of liver metastases. Or under direct vision such as a colonoscopic biopsy.
A female neonate who is born at term has an episode of bilious vomiting. An upper GI contrast study is performed and it shows the duodeno-jejunal flexure lies to the right of the midline. What is the most appropriate course of action?
Undertake a Ramstedts pyloromyotomy
Perform a gastrojejunostomy
Undertake a Ladd’s procedure
Undertake a duodeno-duodenostomy
Undertake a Kasai procedure
Intestinal malrotation with volvulus is treated with a Ladd’s procedure.
Intestinal malrotation in neonates
Normal embryology
During the fourth week of embryogenesis the intestine moves into the abdomen via the base of the umbilicus. Through a combination of foetal growth and bowel maturation the bowel resides in the abdominal cavity and undergoes a 270o counterclockwise twist. At the conclusion of this process the ligament of Treitz lies to the left of the spine and the caecum in the right lower quadrant. Malrotation occurs when the rotational process described is incomplete. Typically the duodenal loop lies to the left of the caecum and therefore lacks 90 o of its 270o rotation. It becomes fixed in this position with peritoneal attachments (Ladds bands).
Symptoms
Bilious vomiting is the cardinal symptom and sign. In most cases there are no antecedent symptoms. If untreated then gut perfusion may be impaired with resultant development of further symptoms.
Diagnosis
The main problem is that the infant develops mid gut volvulus. In this situation the bowel undergoes a 720o twist, the bowel viability depends upon a narrow mesentery containing the superior mesenteric artery. To ascertain whether this has occurred the two main tests include an abdominal ultrasound scan to determine the relationship between the superior mesenteric artery and vein (normally SMA lies to the left of the SMV). This test is complemented with an upper GI contrast series and this aims to establish that the DJ flexure is correctly sited to the left of the vertebral bodies.
Treatment
Laparotomy and division of adhesional bands (Ladds procedure). The division of the congenital adhesions allows widening of the small bowel mesentery. The bowel is untwisted and assessed for viability. The bowel is returned to the abdominal cavity in the non rotated fashion with the small bowel on the right hand side and the large bowel on the left, the caecum is positioned in the left upper quadrant. Because the caecum is located in this new location many surgeons will also perform an appendicectomy due to the diagnostic difficulty posed in diagnosing appendicitis in this group of patients.
Which one of the following would cause a rise in the carbon monoxide transfer factor (TLCO)?
Emphysema
Pulmonary embolism
Pulmonary haemorrhage
Pneumonia
Pulmonary fibrosis
Transfer factor raised: asthma, haemorrhage, left-to-right shunts, polycythaemia
low: everything else
Where alveolar haemorrhage occurs the TLCO tends to increase due to the enhanced uptake of carbon monoxide by intra-alveolar haemoglobin.
Transfer factor
The transfer factor describes the rate at which a gas will diffuse from alveoli into blood. Carbon monoxide is used to test the rate of diffusion. Results may be given as the total gas transfer (TLCO) or that corrected for lung volume (transfer coefficient, KCO)
Causes of a raised TLCO
asthma
pulmonary haemorrhage (Wegener’s, Goodpasture’s)
left-to-right cardiac shunts
polycythaemia
hyperkinetic states
male gender, exercise
Causes of a lower TLCO
pulmonary fibrosis
pneumonia
pulmonary emboli
pulmonary oedema
emphysema
anaemia
low cardiac output
KCO also tends to increase with age. Some conditions may cause an increased KCO with a normal or reduced TLCO
pneumonectomy/lobectomy
scoliosis/kyphosis
neuromuscular weakness
ankylosis of costovertebral joints e.g. ankylosing spondylitis
A 75 year old man is admitted with sudden onset severe generalised abdominal pain, vomiting and a single episode of bloody diarrhoea. On examination, he looks unwell and is in uncontrolled atrial fibrillation. Although diffusely tender his abdomen is soft. What is the most likely diagnosis?
Pancreatitis
Infective diarrhoea
Ischaemic colitis
Crohns disease
Mesenteric infarction
Pain out of proportion to physical signs, AF and generalized abdominal pain suggest widespread infarction.
Acute abdominal pain-diagnoses
Conditions presenting with acute abdominal pain
Condition Features Investigations Management
Appendicitis History of migratory pain.
Fever.
Anorexia.
Evidence of right iliac fossa tenderness.
Mild pyrexia. Differential white cell count
Pregnancy test
C-Reactive protein
Amylase
Urine dipstick testing Appendicectomy
Mesenteric adenitis Usually recent upper respiratory tract infection.
High fever.
Generalised abdominal discomfort- true localised pain and signs are rare. Full blood count- may show slightly raised white cell count
Urine dipstick often normal
Abdominal ultrasound scan - usually no free fluid Conservative management- appendicectomy if diagnostic doubt
Mittelschmerz Only seen in females
Mid cycle pain
Usually occurs two weeks after last menstrual period
Pain usually has a supra-pubic location
Usually subsides over a 24-48 hour period. Full blood count- normal
Urine dipstick- normal
Abdominal and pelvic ultrasound- may show a trace of pelvic free fluid Manage conservatively if doubt or symptoms fail to settle then laparoscopy
Fitz-Hugh Curtis syndrome Disseminated infection with Chlamydia.
Usually seen in females.
Consists of evidence of pelvic inflammatory disease together with peri-hepatic inflammation and subsequent adhesion formation. Abdominal ultrasound scan- may show free fluid
High vaginal swabs - may show evidence of sexually transmitted infections Usually medically managed- doxycycline or azithromycin
Abdominal aortic aneurysm (ruptured) Sudden onset of abdominal pain radiating to the back in older adults (look for risk factors).
Collapse.
May be moribund on arrival in casualty, more stable if contained haematoma.
Careful clinical assessment may reveal pulsatile mass. Patients who are haemodynamically stable should have a CT scan Unstable patients should undergo immediate surgery (unless it is not in their best interests).
Those with evidence of contained leak on CT should undergo immediate surgery
Increasing unruptured aneurysmal size is an indication for urgent surgical intervention (that can wait until the next working day)
Perforated peptic ulcer Sudden onset of pain (usually epigastric).
Often preceding history of upper abdominal pain.
Soon develop generalised abdominal pain.
On examination may have clinical evidence of peritonitis. Erect CXR may show free air. A CT scan may be indicated where there is diagnostic doubt Laparotomy (laparoscopic surgery for perforated peptic ulcers is both safe and feasible in experienced hands)
Intestinal obstruction Colicky abdominal pain and vomiting (the nature of which depends on the level of the obstruction).
Abdominal distension and constipation (again depending upon site of obstruction).
Features of peritonism may occur where local necrosis of bowel loops is occurring. A plain abdominal film may help with making the diagnosis. A CT scan may be useful where diagnostic uncertainty exists In those with a virgin abdomen a lower and earlier threshold for laparotomy should exist than in those who may have adhesional obstruction
Mesenteric infarction Embolic events present with sudden pain and forceful evacuation.
Acute on chronic events usually have a longer history and previous weight loss.
On examination the pain is typically greater than the physical signs would suggest. Arterial pH and lactate
Arterial phase CT scanning is the most sensitive test
A 73 year old lady is admitted with brisk rectal bleeding. Despite attempts at resuscitation the bleeding proceeds to cause haemodynamic compromise. An upper GI endoscopy is normal. A mesenteric angiogram is performed and a contrast blush is seen in the region of the sigmoid colon. The radiologist decides to embolise the vessel supplying this area. At what spinal level does it leave the aorta?
L2
L1
L4
L3
T10
The inferior mesenteric artery leaves the aorta at L3. It supplies the left colon and sigmoid. Its proximal continuation to communicate with the middle colic artery is via the marginal artery.
Levels
Transpyloric plane
Level of the body of L1
Pylorus stomach
Left kidney hilum (L1- left one!)
Fundus of the gallbladder
Neck of pancreas
Duodenojejunal flexure
Superior mesenteric artery
Portal vein
Left and right colic flexure
Root of the transverse mesocolon
2nd part of the duodenum
Upper part of conus medullaris
Spleen
Can be identified by asking the supine patient to sit up without using their arms. The plane is located where the lateral border of the rectus muscle crosses the costal margin.
Anatomical planes
Subcostal plane Lowest margin of 10th costal cartilage
Intercristal plane Level of body L4 (highest point of iliac crest)
Intertubercular plane Level of body L5
Common level landmarks
Inferior mesenteric artery L3
Bifurcation of aorta into common iliac arteries L4
Formation of IVC L5 (union of common iliac veins)
Diaphragm apertures
Vena cava T8
Oesophagus T10
Aortic hiatus T12
A 74 year old male is admitted to the Emergency Department with a fall. He is known to have rheumatoid arthritis and is on methotrexate and paracetamol. He lives alone in a bungalow and enjoys playing golf. He is independent with his ADLs. He complains of left groin pain, therefore has a hip x-ray which confirms a displaced intracapsular fracture. What is the best course of action?
Cemented hemiarthroplasty
Uncemented hemiarthroplasty
Total hip replacement
Dynamic hip screw
Intramedullary nail
This patient has pre-existing joint disease, good level of activity and a relatively high life expectancy, therefore THR is preferable to hemiarthroplasty.
Hip fractures
Background
Neck of femur (NOF) fracture is a common orthopaedic presentation, with over 65000 fractures in the UK per year. Like many orthopaedic injuries, there is a bimodal age distribution. It is imperative to distinguish between the high energy injury in a young patient, and the low energy osteoporotic fracture in the elderly, as their management aims are very different:
Young patient - Usually high energy trauma (e.g road traffic accident, horse riding) and needs treating in accordance with Advanced Trauma Life Support (ATLS) principles. Will often have associated injuries. Aim is to retain the patients own anatomy, and optimise their function.
Elderly patient - Predominantly female, fall from standing height (fragility fracture). Often patients have multiple comorbidities that will ultimately dictate their prognosis. Aim of orthopaedic treatment is to immediately regain patient mobility so that morbidity (infection, thromboembolic events, pressure sores etc) and mortality associated with prolonged bed rest is avoided. Left untreated, a neck of femur fracture can be considered a terminal event. Historically, mortality associated with elderly hip fracture is 10% at one month, and 30% at one year. However, this has been improved in the UK with the introduction of multidisciplinary, orthogeriatric lead care and the National Hip Fracture Database and Best Practice Tariff.
Pertinent anatomy
Osteology - normal neck-shaft angle is 130 +/- 7 degrees, and 10 +/- 7 degrees of neck anteversion.
Vascular supply - The predominant blood supply to the femoral head and neck is from the medial and lateral femoral circumflex arteries (branches of profunda femoris). These anastomose and pierce the joint capsule at the base of the neck, mainly posteriorly. There is a small vascular contribution from the artery of the ligament teres. Understanding the blood supply is fundamental to the decision making process in treating NOF fractures.
Presentation and initial management
Typically, patients present with pain in the hip/groin, a shortened, abducted, externally rotated leg (due to the unopposed pull of the muscles that act across the hip joint) and the inability to straight-leg-raise. With undisplaced fractures, signs are more subtle.
High energy injuries should be treated in line with ATLS principles. All patients should be fluid resuscitated, have adequate pain relief (often with a fascio-iliiaca nerve block), and be optimised for surgery. In addition, elderly patients should be assessed by an orthogeriatrician.
Imaging
Anteroposterior and cross-table lateral plain radiographs are sufficient to diagnose the majority of NOF fractures. If the fracture extends below the level of the lesser trochanter, or there is any possibility of pathological fracture, full length femur views are essential to plan surgery.
Where there is a high index of suspicion of fracture, but plain radiographs are inconclusive, gold standard investigation is MRI. However, if unavailable within 24 hours, or if the patient will not tolerate MRI, CT is appropriate. The majority of fractures can be seen with modern CT techniques, and so this is becoming first line in many hospitals.
Classification
There has been a move away from named classification systems towards descriptive classification systems.
Two main types of NOF exist: Intra-capsular, and extra-capsular. Extra-capsular fractures are further divided into pertrochanteric or subtrochanteric (within 5cm distal to the lesser trochanter). All fractures are then described as undisplaced, minimally displaced, or displaced.
Femoral neck and head blood supply disruption is common with intracapsular NOF fractures, and rare with extracapsular fractures. This fundamental principle underpins the practise of arthroplasty for intracapsular fractures, and fixation for extracapsular fractures.
If you wish to use a named classification system, the most commonly used are below:
Elderly intracapsular - Garden Classification
Young intrasapsular - Pauvels Classification
Intertrochanteric - Evans
Subtrochanteric - Russell Taylor
Treatment
In general, NOF fractures are treated operatively except if the patient is deemed unlikely to survive an anaesthetic. Best Practice Tarif (BPT) dictates that surgery should happen within 36 hours, as delay of greater than 48 hours is associated with increased morbidity and mortality. Below are suggested algorithms for the treatment of NOF.
* The priority with the young patient is to retain the femoral head if possible, even with a displaced intracapsular fracture. The risk of avascular necrosis and non-union (and therefore revision surgery) associated with internal fixation needs weighing up against the sequelae of total hip replacement in the young (wear, dislocation, revision). Discussion is necessary with the patient, on a case by case basis.
** Undisplaced fractures in the elderly can be treated with internal fixation, often with cannulated screws. This is appropriate for valgus impacted subcapital fractures which are inherently stable, to prevent secondary displacement. This does still carry the risk of AVN or non-union, and therefore a future revision. For this reason, many surgeons advocate arthroplasty as a single surgery.
*** NICE guidance - patients who fulfil these criteria should be offered total hip replacement which conveys better function and prosthetic survivorship, compared with hemiarthroplasty, but at an increased risk of dislocation.
- Intertrochanteric fractures vary greatly in their stability. If the trochanter (and therefore lateral wall), and medial calcar is in tact, then the fracture configuration bears stability. This can be treated with a DHS, as collapse of the fracture is predictable. Where either or both structures are involved in the fracture, stability becomes compromised and many surgeons will favour using an intramedullary device. This is an ongoing debate, and difficult to test in an exam setting.
Post operative management
Patients should be mobilised fully weight bearing where possible. Care is multidisciplinary in its delivery. Elderly patients should have orthogeriatrician assessment of comorbidity, and bone health with secondary prevention measures if appropriate. There should be early involvement of physiotherapy and occupational therapy services. For further guidance see sources listed below.
NICE clinical guidance on hip fracture: https://www.nice.org.uk/guidance/cg124
Best Practice Tarif: www.nhfd.co.uk/20/hipfractureR…/Best%20Practice%20Tariff%20User%20Guide.pdf National Hip Fracture Database: www.nhfd.co.uk/
A 43 year old lady undergoes a live related renal transplant. At the conclusion of the operation she has a good urine output and the graft appeared well perfused. On the ward she suddenly becomes anuric. What is the most likely cause?
Renal artery stenosis
Renal vein thrombosis
Renal artery thrombosis
Hyperacute rejection
Acute rejection
Sudden loss of urine output is most commonly due to a blocked catheter. However, if this is excluded (and is not included in the options) the most worrisome cause is arterial thrombosis. This will often be a delayed diagnosis and the rate of graft loss is high.
Complications of renal transplantation
A number of complications may occur following renal transplantation. A critical aspect of post operative care is evaluation of graft function. Post operatively, urine output is the most readily available, and easily measured, indicator of graft function. If an individual was relatively anuric pre-transplant and has a good urine output following surgery then this is more useful than it would be in someone who had a higher volume diuresis prior to transplantation. Recipients can be divided into three main groups following renal transplantation, with regard to their graft function:
Immediate graft function; brisk diuresis and falling serum creatinine
Slow graft function; modest urine output and slowly falling creatinine levels
Delayed graft function; defined as need for dialysis post transplant
Decreased urine output following surgery can be the result of hypovolvaemia or a blocked catheter (commonest causes). Other important causes include rejection, or a vascular complication.
Vascular complications
These may involve the donor vessels, those of the recipient or both. Renal artery thrombosis usually occurs early post transplant, but is uncommon with an incidence of less than 1%. It typically results in graft loss. It usually occurs as a result of a technical problem such a vessel torsion or sub intimal flaps. The usual presenting feature is a sudden cessation of urine output. When suspected, the occlusion is usually well demonstrated with duplex scanning. Ideally immediate surgical re-exploration should occur. Sadly, the graft has usually been lost by this stage and will require graft nephrectomy. Renal vein thrombosis is not as common as arterial graft thrombosis and the usual presenting features include discomfort at the graft site and swelling of the graft associated with loss of urine output. Again, duplex scanning is indicated. Unfortunately, this complication is also associated with a high incidence of graft loss.
Over a longer time frame (typically months) some individuals will develop renal artery stenosis. These individuals will typically develop hypertension and over time graft function will decline as hypertensive nephropathy occurs. It is usually demonstrated by duplex scanning and is usually amenable to endovascular intervention.
Urological complications
Urinary tract complications manifesting as leakage or obstruction are common complications following renal transplantation and occur in up to 10% of patients. The main underlying cause is the relatively poor blood supply to the transplanted ureter. Patients typically present relatively early in the first 5 weeks following transplantation with pain and swelling at the graft site. Imaging with USS is often the initial test. Therapeutic options include surgical re-implantation of the ureter for large leaks and stent insertion and nephrostomy placement for smaller leaks.
Lymphocele
These do not generally occur until 2 weeks or longer after surgery. They are, however, relatively common and may be seen in up to 18% of patients. Symptoms usually occur as a result of mass effect with compression of adjacent structures. These include the vessels supplying both the graft, with deterioration in graft function, the ureter, with alteration in urine output and the recipients lower limb vessels, with development of leg swelling. Creation of a laparoscopic or open peritoneal window is a favored treatment.
Rejection
Four types of graft rejection are recognised; hyperacute, accelerated acute, acute and chronic.
Type of rejection Key features
Hyperacute Occurs within minutes of clamp release
Due to pre formed antibodies
Immediate loss of graft occurs
Accelerated acute Occurs in first few days following surgery
Involved both cellular and antibody mediated injury
Pre-sensitisation of the donor is a common cause
Acute Traditionally the most common type of rejection
Seen days to weeks after surgery
Predominantly a cell mediated process mediated by lymphocytes
Organ biopsy demonstrates cellular infiltrates and graft cell apoptosis
Chronic Increasingly common problem
Typically; graft atrophy and atherosclerosis are seen. Fibrosis often occurs as a late event
Which of the following statements relating to abnormal coagulation is false?
Warfarin affects the synthesis of factor 2,7,9,10
The prothrombin time is prolonged in Haemophilia A
Cholestatic jaundice can cause vitamin K deficiency
Disseminated intravascular coagulation is associated with thrombocytopenia
Massive transfusion is associated with reduced levels of factor 5 and 8
In haemophilia A the APTT is prolonged and there is reduced levels of factor 8:C. The bleeding time and PT are normal. Cholestatic jaundice prevents the absorption of the fat soluble vitamin K. Massive transfusion (>10u blood or equivalent to the blood volume of a person) puts the patient at risk of thrombocytopaenia, factor 5 and 8 deficiency.
Abnormal coagulation
Cause Factors affected
Heparin Prevents activation factors 2,9,10,11
Warfarin Affects synthesis of factors 2,7,9,10
DIC Factors 1,2,5,8,11
Liver disease Factors 1,2,5,7,9,10,11
Interpretation blood clotting test results
Disorder APTT PT Bleeding time
Haemophilia Increased Normal Normal
von Willebrand’s disease Increased Normal Increased
Vitamin K deficiency Increased Increased Normal
A neurosurgeon decides to explore the cavernous sinus via the extended endonasal route. Which of the structures listed below is unlikely to be encountered with such an approach.
Oculomotor nerve
Internal carotid artery
Opthalmic nerve
Abducens nerve
Optic nerve
Mnemonic for contents of cavernous sinus:
O TOM CAT
Occulomotor nerve (III)
Trochlear nerve (IV)
Ophthalmic nerve (V1)
Maxillary nerve (V2)
Carotid artery
Abducent nerve (VI)
T
OTOM=lateral wall components
CA= components within sinus
The optic nerve lies above and outside the cavernous sinus.
A 23 year old man is stabbed in the groin, several structures are injured and the adductor longus muscle has been lacerated. Which of the following nerves is responsible for the innervation of adductor longus?
Femoral nerve
Obturator nerve
Sciatic nerve
Common peroneal nerve
Ilioinguinal nerve
The adductors are innervated by the obturator nerve.
Adductor longus
Origin Anterior body of pubis
Insertion Middle third of linea aspera
Action Adducts and flexes the thigh, medially rotate the hip
Innervation Anterior division of obturator nerve (L2, L3, L4)
Which of the following statements relating to audit and governance is untrue?
An audit standard is a threshold of compliance with an audit criterion
Sample size calculations are an important part of audit planning
Clinical audit is part of clinical governance
Audits should be performed regularly when a novel surgical technique is introduced and where there is little knowledge of anticipated complications or outcomes
An audit criterion is a measurable outcome of care, aspect of practice or capacity
6 pillars of clinical governance:
Clinical effectiveness
Research and development
Openness
Risk management
Education and training
Clinical audit
Audits should compare performance against known standards. Where a novel technique is being introduced standards are unlikely to exist, sample sizes cannot therefore be accurately calculated. This is an example of research, which is not an audit.
A 55 year old man is attending a health screening centre and is concerned about risk factors for the development of gastric cancer. Which of the factors described below carries the least risk of developing the condition subsequently
Polya gastrectomy for antral ulcer
Atrophic gastritis
Intestinal metaplasia of columnar type at the gastric cardia
Gastric polyp showing medium grade dysplasia
Long term therapy with sucralfate
Although some acid lowering procedures increase the risk of gastric cancer the use of sucralfate does not, at the present time, seem to increase the risk.
Gastric cancer
Overview
There are 700,000 new cases of gastric cancer worldwide each year. It is most common in Japan and less common in western countries. It is more common in men and incidence rises with increasing age. The exact cause of many sporadic cancer is not known, however, familial cases do occur in HNPCC families. In addition, smoking and smoked or preserved foods increase the risk. Japanese migrants retain their increased risk (decreased in subsequent generations). The distribution of the disease in western countries is changing towards a more proximal location (perhaps due to rising obesity).
Pathology
There is some evidence of support a stepwise progression of the disease through intestinal metaplasia progressing to atrophic gastritis and subsequent dysplasia, through to cancer. The favoured staging system is TNM. The risk of lymph node involvement is related to size and depth of invasion; early cancers confined to submucosa have a 20% incidence of lymph node metastasis. Tumours of the gastro-oesophageal junction are classified as below:
Type 1 True oesophageal cancers and may be associated with Barrett’s oesophagus.
Type 2 Carcinoma of the cardia, arising from cardiac type epithelium
or short segments with intestinal metaplasia at the oesophagogastric junction.
Type 3 Sub cardial cancers that spread across the junction. Involve similar nodal stations to gastric cancer.
Groups for close endoscopic monitoring
Intestinal metaplasia of columnar type
Atrophic gastritis
Low to medium grade dysplasia
Patients who have previously undergone resections for benign peptic ulcer disease (except highly selective vagotomy).
Referral to endoscopy
Patients of any age with dyspepsia and any of the following Patients without dyspepsia Worsening dyspepsia
Chronic gastrointestinal bleeding Dysphagia Barretts oesophagus
Dysphagia Unexplained abdominal pain or weight loss Intestinal metaplasia
Weight loss Vomiting Dysplasia
Iron deficiency anaemia Upper abdominal mass Atrophic gastritis
Upper abdominal mass Jaundice Patient aged over 55 years with unexplained or persistent dyspepsia
Upper GI endoscopy performed for dyspepsia. The addition of dye spraying (as shown in the bottom right) may facilitate identification of smaller tumours
Staging
CT scanning of the chest abdomen and pelvis is the routine first line staging investigation in most centres.
Laparoscopy to identify occult peritoneal disease
PET CT (particularly for junctional tumours)
Treatment
Proximally sited disease greater than 5-10cm from the OG junction may be treated by sub total gastrectomy
Total gastrectomy if tumour is <5cm from OG junction
For type 2 junctional tumours (extending into oesophagus) oesophagogastrectomy is usual
Endoscopic sub mucosal resection may play a role in early gastric cancer confined to the mucosa and perhaps the sub mucosa (this is debated)
Lymphadenectomy should be performed. A D2 lymphadenectomy is widely advocated by the Japanese, the survival advantages of extended lymphadenectomy have been debated. However, the overall recommendation is that a D2 nodal dissection be undertaken.
Most patients will receive chemotherapy either pre or post operatively.
Prognosis
UK Data
Disease extent Percentage 5 year survival
All RO resections 54%
Early gastric cancer 91%
Stage 1 87%
Stage 2 65%
Stage 3 18%
Operative procedure
Total Gastrectomy , lymphadenectomy and Roux en Y anastomosis
General anaesthesia
Prophylactic intravenous antibiotics
Incision: Rooftop.
Perform a thorough laparotomy to identify any occult disease.
Mobilise the left lobe of the liver off the diaphragm and place a large pack over it. Insert a large self retaining retractor e.g. omnitract or Balfour (take time with this, the set up should be perfect). Pack the small bowel away.
Begin by mobilising the omentum off the transverse colon.
Proceed to detach the short gastric vessels.
Mobilise the pylorus and divide it at least 2cm distally using a linear cutter stapling device.
Continue the dissection into the lesser sac taking the lesser omentum and left gastric artery flush at its origin.
The lymph nodes should be removed en bloc with the specimen where possible.
Place 2 stay sutures either side of the distal oesophagus. Ask the anaesthetist to pull back on the nasogastric tube. Divide the distal oesophagus and remove the stomach.
The oesphago jejunal anastomosis should be constructed. Identify the DJ flexure and bring a loop of jejunum up to the oesophagus (to check it will reach). Divide the jejunum at this point. Bring the divided jejunum either retrocolic or antecolic to the oesophagus. Anastamose the oesophagus to the jejunum, using either interrupted 3/0 vicryl or a stapling device. Then create the remainder of the Roux en Y reconstruction distally.
Place a jejunostomy feeding tube.
Wash out the abdomen and insert drains (usually the anastomosis and duodenal stump). Help the anaesthetist insert the nasogastric tube (carefully!)
Close the abdomen and skin.
Enteral feeding may commence on the first post-operative day. However, most surgeons will leave patients on free NG drainage for several days and keep them nil by mouth.
A 43 year old lady with hypertension is suspected of having a phaeochromocytoma. Which of the following investigations is most likely to be beneficial in this situation?
Dexamethasone suppression test
Urinary 5-Hydroxyindoleacetic Acid (5-HIAA)
Histamine provocation test
Tyramine provocation test
Urinary metanephrines measurement
Urinary metanephrine measurements are not completely specific but constitute first line assessment and more accurate than urinary VMA measurements. Stimulation tests of any sort are not justified in first line assessments.
Phaeochromocytoma and adrenal lesions
Phaeochromocytoma
Neuroendocrine tumour of the chromaffin cells of the adrenal medulla. Hypertension and hyperglycaemia are often found.
10% of cases are bilateral.
10% occur in children.
11% are malignant (higher when tumour is located outside the adrenal).
10% will not be hypertensive.
Familial cases are usually linked to the Multiple endocrine neoplasia syndromes (considered under its own heading).
Most tumours are unilateral (often right sided) and smaller than 10cm.
Diagnosis
Urine and plasma metanephrine levels.
CT and MRI scanning are both used to localise the lesion.
Treatment
Patients require medical therapy first. An irreversible alpha adrenoreceptor blocker should be given, although minority may prefer reversible blockade(1). Labetolol may be co-administered for cardiac chronotropic control. Isolated beta blockade should not be considered as it will lead to unopposed alpha activity.
These patients are often volume depleted and will often require moderate volumes of intra venous normal saline perioperatively.
Once medically optimised the phaeochromocytoma should be removed. Most adrenalectomies can now be performed using a laparoscopic approach(2). The adrenals are highly vascular structures and removal can be complicated by catastrophic haemorrhage in the hands of the inexperienced. This is particularly true of right sided resections where the IVC is perilously close. Should the IVC be damaged a laparotomy will be necessary and the defect enclosed within a Satinsky style vascular clamp and the defect closed with prolene sutures. Attempting to interfere with the IVC using any instruments other than vascular clamps will result in vessel trauma and make a bad situation much worse.
Incidental adrenal lesions
Adrenal lesions may be identified on CT scanning performed for other reasons(3). Factors suggesting benign disease on CT include(4):
Size less than 3cm
Homogeneous texture
Lipid rich tissue
Thin wall to lesion
All patients with incidental lesions should be managed jointly with an endocrinologist and full work up as described above. Patients with functioning lesions or those with adverse radiological features (Particularly size >3cm) should proceed to surgery.
A healthy man has a blood pressure of 120/80 mmHg and an intra cranial pressure of 17 mmHg. What is the approximate cerebral perfusion pressure?
103 mmHg
63 mmHg
83 mmHg
91 mmHg
76 mmHg
Cerebral perfusion pressure= Mean arterial pressure - intra cranial pressure
The mean arterial pressure can be calculated as:
MAP= Diastolic pressure+ 0.333(Systolic pressure- Diastolic pressure)
In this situation the MAP = 93.
The ICP is subtracted from this value; 93 - 17 = 76
Cerebral perfusion pressure
The cerebral perfusion pressure (CPP) is defined as being the net pressure gradient causing blood flow to the brain. The CPP is tightly autoregulated to maximise cerebral perfusion. A sharp rise in CPP may result in a rising ICP, a fall in CPP may result in cerebral ischaemia. It may be calculated by the following equation:
CPP= Mean arterial pressure - Intra cranial pressure
Following trauma, the CPP has to be carefully controlled and the may require invasive monitoring of the ICP and MAP.
Which ligament keeps the head of the radius connected to the radial notch of the ulna?
Annular (orbicular) ligament
Quadrate ligament
Radial collateral ligament of the elbow
Ulnar collateral ligament
Radial collateral ligament
The annular ligament connects the radial head to the radial notch of the ulna.
Radius
The radius is one of the two long forearm bones that extends from the lateral side of the elbow to the thumb side of the wrist. It has two expanded ends, of which the distal end is the larger. Key points relating to its topography and relations are outlined below;
Upper end
Articular cartilage- covers medial > lateral side
Articulates with radial notch of the ulna by the annular ligament
Muscle attachment- biceps brachii at the tuberosity
Shaft
Muscle attachment
Upper third of the body Supinator
Flexor digitorum superficialis
Flexor pollicis longus
Middle third of the body Pronator teres
Lower quarter of the body Pronator quadratus
Tendon of supinator longus
Lower end
Quadrilateral
Anterior surface- capsule of wrist joint
Medial surface- head of ulna
Lateral surface- ends in the styloid process
Posterior surface: 3 grooves containing:
1. Tendons of extensor carpi radialis longus and brevis
2. Tendon of extensor pollicis longus
3. Tendon of extensor indicis
A 34 year old male is being examined in the pre-operative assessment clinic. A murmur is identified in the 4th intercostal space just next to the left side of the sternum. From where is it most likely to have originated?
Mitral valve
Aortic valve
Pulmonary valve
Right ventricular aneurysm
Tricuspid valve
The tricuspid valve is generally referred to being best auscultated adjacent to the sternum. The plane of projected sound from the mitral area is best heard in the region of the cardiac apex.
Heart sounds
Sites of auscultation
Valve Site
Pulmonary valve Left second intercostal space, at the upper sternal border
Aortic valve Right second intercostal space, at the upper sternal border
Mitral valve Left fifth intercostal space, just medial to mid clavicular line
Tricuspid valve Left fourth intercostal space, at the lower left sternal border
A 24 year old man is involved in a fight and his face is cut with a knife. The wound lies immediately anterior to the tragus of the ear and extends anteriorly. The wound is surgically explored and the laceration is found to be mainly superficial. It extends slightly more deeply immediately inferior to the main trunk of the facial nerve. Bleeding is observed, from which of the following is it most likely to originate?
External carotid artery
Retromandibular vein
Occipital artery
Maxillary artery
Ascending pharyngeal artery
The retromandibular vein lies slightly more deeply than the facial nerve in the parotid gland. It is formed from the maxillary and superficial temporal vein.
Retromandibular vein
Formed by a union of the maxillary vein and superficial temporal vein
It descends through the parotid gland and bifurcates within it
The anterior division passes forwards to join the facial vein, the posterior division is one of the tributaries of the external jugular vein
What type of visual field defect is most likely to be noted in a patient with a craniopharyngioma?
Lower bitemporal hemianopia
Upper bitemporal hemianopia
Right superior quadranopia
Right homonymous hemianopia
Left homonymous hemianopia
Lesions at the optic chiasm classically produce a bitemporal hemianopia, however note lesions that spread up from below ie pituitary tumours, the defect is worse in the upper fields and if a lesion spreads down from above ie craniopharyngiomas, the visual defect is worse in the lower quadrants. Therefore this patient is likely to have a lower bitemporal hemianopia.
Visual field defects
left homonymous hemianopia means visual field defect to the left, i.e. Lesion of right optic tract
homonymous quadrantanopias: PITS (Parietal-Inferior, Temporal-Superior)
incongruous defects = optic tract lesion; congruous defects = optic radiation lesion or occipital cortex
Homonymous hemianopia
Incongruous defects: lesion of optic tract
Congruous defects: lesion of optic radiation or occipital cortex
Macula sparing: lesion of occipital cortex
Homonymous quadrantanopias
Superior: lesion of temporal lobe
Inferior: lesion of parietal lobe
Mnemonic = PITS (Parietal-Inferior, Temporal-Superior)
Bitemporal hemianopia
Lesion of optic chiasm
Upper quadrant defect > lower quadrant defect = inferior chiasmal compression, commonly a pituitary tumour
Lower quadrant defect > upper quadrant defect = superior chiasmal compression, commonly a craniopharyngioma
Which of the following fractures names best accounts for the injury seen in a 14 year old boy who jumps off a 10 foot wall and lands on both feet and whose imaging shows a bimalleolar fracture of the right ankle?
Pott’s
Barton’s
Galeazzi
Colles’
Bennett’s
Eponymous fractures
Colles’ fracture (dinner fork deformity)
Fall onto extended outstretched hand
Classical Colles’ fractures have the following 3 features:
- Transverse fracture of the radius
- 1 inch proximal to the radio-carpal joint
- Dorsal displacement and angulation
Smith’s fracture (reverse Colles’ fracture)
Volar angulation of distal radius fragment (Garden spade deformity)
Caused by falling backwards onto the palm of an outstretched hand or falling with wrists flexed
Bennett’s fracture
Intra-articular fracture of the first carpometacarpal joint
Impact on flexed metacarpal, caused by fist fights
X-ray: triangular fragment at ulnar base of metacarpal
Monteggia’s fracture
Dislocation of the proximal radioulnar joint in association with an ulna fracture
Fall on outstretched hand with forced pronation
Needs prompt diagnosis to avoid disability
Galeazzi fracture
Radial shaft fracture with associated dislocation of the distal radioulnar joint
Direct blow
Pott’s fracture
Bimalleolar ankle fracture
Forced foot eversion
Barton’s fracture
Distal radius fracture (Colles’/Smith’s) with associated radiocarpal dislocation
Fall onto extended and pronated wrist
Involvement of the joint is a defining feature
Holstein Lewis Fracture
A HolsteinLewis fracture is a fracture of the distal third of the humerus resulting in entrapment of the radial nerve.
The radial nerve is one of the major nerves of the upper limb. It innervates all of the muscles in the extensor compartments of the arm.
Conservative treatment includes reduction and use of a functional brace
Vascular injury may require open surgery
At which of the following sites is the most water absorbed?
Right colon
Left colon
Stomach
Jejunum
Duodenum
Water absorption in the gastrointestinal tract predominantly occurs in the small bowel (jejunum and ileum). The colon is an important site of water absorption, however, its overall contribution is relatively small. The importance of the colonic component to water absorption may increase following extensive small bowel resections.
Water absorption
During a 24 hours period the average person will ingest up to 2000ml of liquid orally. In addition a further 8000ml of fluid will enter the small bowel as gastrointestinal secretions. Intestinal water absorption is a passive process and is related to solute load. In the jejunum the active absorption of glucose and amino acids will create a concentration gradient that water will flow across. In the ileum most water is absorbed by a process of facilitated diffusion (with sodium).
Approximately 150ml of water enters the colon daily, most is absorbed, the colon can adapt to, and increase this amount following resection.
A 28 year old Indian woman, who is 18 weeks pregnant, presents with increasing shortness of breath, chest pain and coughing clear sputum. She is apyrexial, blood pressure is 140/80 mmHg, heart rate 130 bpm and saturations 94% on 15L oxygen. On examination, there is a mid diastolic murmur, there are bibasal crepitations and mild pedal oedema. She suddenly deteriorates and has a respiratory arrest. Her chest x-ray shows a whiteout of both of her lungs. What is the most likely explanation?
Acute exacerbation asthma
Pulmonary embolus
Mitral regurgitation
Mitral valve stenosis
Aortic dissection
Mitral stenosis is the commonest cause of cardiac abnormality occurring in pregnant women. Mitral stenosis is becoming less common in the UK population, however should be considered in women from countries where there is a higher incidence of rheumatic heart disease. Mitral stenosis causes a mid diastolic murmur which may be difficult to auscultate unless the patient is placed into the left lateral position. These patients are at risk of atrial fibrillation (up to 40%), which can also contribute to rapid decompensation such as pulmonary oedema (hence cxr ‘whiteout’ of lungs). Physiological changes in pregnancy may cause an otherwise asymptomatic patient to suddenly deteriorate. Balloon valvuloplasty is the treatment of choice.
A 25 year old cyclist is hit by a bus traveling at 30mph. He was not wearing a helmet. He arrives with a GCS of 3/15 and is intubated. A CT scan shows evidence of cerebral contusion but no localising clinical signs are present. What is the most appropriate course of action?
Burr hole decompression
Decompressive craniotomy
Insertion of intra cranial pressure monitoring device
Administration of intravenous mannitol
Parietotemporal craniotomy
This patient may well develop raised ICP over the next few days and intracranial pressure monitoring will help with management.
Head injury management- NICE Guidelines
Summary of guidelines
All patients should be assessed within 15 minutes on arrival to A&E
Document all 3 components of the GCS
If GCS <8 or = to 8, consider stabilising the airway
Full spine immobilisation until assessment if:
- GCS < 15
- neck pain/tenderness
- paraesthesia extremities
- focal neurological deficit
- suspected c-spine injury
If a c-spine injury is suspected a 3 view c-spine x-ray is indicated. CT c-spine is preferred if:
- Intubated
- GCS <13
- Normal x-ray but continued concerns regarding c-spine injury
- Any focal neurology
- A CT head scan is being performed
- Initial plain films are abnormal
Immediate CT head (within 1 hour) if:
GCS < 12 on admission
GCS < 15 2 hours after admission
Suspected open or depressed skull fracture
Suspected skull base fracture (panda eyes, Battle’s sign, CSF from nose/ear, bleeding ear)
Focal neurology
Vomiting > 1 episode
Post traumatic seizure
Contact neurosurgeon if:
Persistent GCS < 8 or = 8
Unexplained confusion > 4h
Reduced GCS after admission
Progressive neurological signs
Incomplete recovery post seizure
Penetrating injury
Cerebrospinal fluid leak
Observations
1/2 hourly GCS until 15
In the latest guidance, there is a move to considering CT in those on anticoagulants rather than mandating it.
Reference
1. NICE Guidance NG 232 (Published 2023).
2. Hodgkinson S et al. Early management of head injury: summary of NICE guidance. BMJ 2014 (348):34-37.
A 73 year old lady is recovering from recent surgery and is resting. Whilst she is resting, which of these muscles will be the major contributor to ventilatory activity?
Innermost intercostal
Diaphragm
Internal intercostal
Sternocleidomastoid
Scalene
The diaphragm is the major muscle involved in this process and it is for this reason that diminished activity following major abdominal surgery (note surgery type not specified here) can result in atelectasis.
Alveolar ventilation
- Minute ventilation is the total volume of gas ventilated per minute.
MV (ml/min)= tidal volume x Respiratory rate (resps/min).
Dead space ventilation describes the volume of gas not involved in exchange in the blood.
There are 2 types:
- Anatomical dead space: 150mls
Volume of gas in the respiratory tree not involved in gaseous exchange: mouth, pharynx, trachea, bronchi up to terminal bronchioles
Measured by Fowlers method
Increased by:
Standing, increased size of person, increased lung volume and drugs causing bronchodilatation e.g. Adrenaline - Physiological dead space: normal 150 mls, increases in ventilation/perfusion mismatch e.g. PE, COPD, hypotension
Volume of gas in the alveoli and anatomical dead space not involved in gaseous exchange.
Alveolar ventilation is the volume of fresh air entering the alveoli per minute.
Alveolar ventilation = minute ventilation - Dead space volume
A 77 year old man is admitted with large bowel obstruction and on investigation with an abdominal CT scan is found to have an obstructing cancer of the sigmoid colon. What is the most appropriate course of action?
Laparotomy, sigmoid colectomy and formation of end colostomy
Laparotomy and loop colostomy
Laparotomy and loop ileostomy
Laparotomy, high anterior resection and colo-rectal anastomosis
Palliation
Obstructing sigmoid cancers can be resected or stented. If stented, then the patient may need definitive surgery later. If resected, then a resection and end colostomy (Hartmann’s ) procedure is usually undertaken because of the risks of anastomotic leak in the setting of anastomosing obstructed colon to rectum.
Large bowel obstruction
Colonic obstruction remains a common surgical problem. It is most commonly due to malignancy (60%) and diverticular disease (20%). Volvulus affecting the colon accounts for 5% of cases. Acute colonic pseudo-obstruction remains a potential differential diagnosis in all cases. Intussusception affecting the colon (most often due to tumours in the adult population) remains a rare but recognised cause.
The typical patient will present with gradual onset of progressive abdominal distension, colicky abdominal pain and either obstipation or absolute constipation.
On examination abdominal distension is present, the presence of caecal tenderness (assuming no overt evidence of peritonitis) is a useful sign to elicit. A digital rectal examination and rigid sigmoidoscopy should be performed.
A plain abdominal x-ray is the usual first line test and; the caecal diameter and ileocaecal valve competency should be assessed on this film.
Imaging modalities
Debate long surrounds the use of CT versus gastrograffin enemas. The latter investigation has always been the traditional method of determining whether a structural lesion is indeed present. However, in the UK the use of this technique has declined and in most units a CT scan will be offered as the first line investigation by the majority of radiologists (and is advocated by the ACPGBI). In most cases this will provide sufficient detail to allow operative planning, and since malignancy accounts for most presentations may also stage the disease. In the event that the radiologist cannot provide a clear statement of lesion site, the surgeon should have no hesitation in requesting a contrast enema.
Surgical options
The decision as to when to operate or not is determined firstly by the patients physiological status. Unstable patients require resuscitation prior to surgery and admission to a critical care unit for invasive monitoring and potential inotropic support may be needed. In patients who are otherwise stable the decision then rests on the radiological and clinical findings. As a general rule the old adage that the sun should not rise and set on unrelieved large bowel obstruction still holds true. A caecal diameter of 12cm or more in the presence of complete obstruction with a competent ileocaecal valve and caecal tenderness is a sign of impending perforation and a relative indication for prompt surgery.
Right sided and transverse lesions
Right sided lesions producing large bowel obstruction should generally be treated by right hemicolectomy or its extended variant if the lesion lies in the distal transverse colon or splenic flexure. In these cases an ileocolic anastomosis may be easily constructed and even in the emergency setting has a low risk of anastomotic leak.
Left sided lesions
The options here lie between sub total colectomy and anastomosis, left hemicolectomy with on table lavage and primary anastomosis, left hemicolectomy and end colostomy formation and finally colonic stent insertion.
The usefulness of colonic stents was the subject of a Cochrane review in 2011. The authors concluded that on the basis of the data that they reviewed, there was no benefit from the use of colonic stents over conventional surgical resection with a tendency to better outcomes seen in the surgical group (1). A more recently conducted meta analysis met with the same conclusion (2). However, the recently concluded CREST trial has suggested that self expanding metallic stents can improve outcomes and avoids a stoma.
Rectosigmoid lesions
Lesions below the peritoneal reflection that are causing obstruction should generally be treated with a loop colostomy. Primary resection of unstaged rectal cancer would most likely carry a high CRM positivity rate and cannot be condoned. Where the lesion occupies the distal sigmoid colon the usual practice would be to perform a high anterior resection. The decision surrounding restoration of intestinal continuity would lie with the operating surgeon.
References
1. Sagar J. Colorectal stents for the management of malignant colonic obstructions. Cochrane Database of Systematic Reviews 2011, Issue 11. Art. No.: CD007378. DOI: 10.1002/14651858.CD007378.pub2.
2. Cirrochi et al Safety and efficacy of endoscopic colonic stenting as a bridge to surgery in the management of intestinal obstruction due to left colon and rectal cancer: A systematic review and meta-analysis. Surg Oncol. 2013 Mar;22(1):14-21.
A 45 year old man has been admitted after being knocked off his bicycle. His ankle is grossly deformed with bilateral malleolar tenderness with severe ankle swelling and tenting of the medial soft tissues. What is the most appropriate initial management?
Application of compression dressing and physiotherapy
Application of external fixation device
Immediate reduction and application of backslab
Surgical fixation
Application of full leg plaster cast
This is an unstable ankle injury that is likely to require surgical fixation. The immediate management of a displaced ankle fracture is to reduce the fracture to prevent soft tissues compromise and help reduce swelling. This can be performed before an x-ray is obtained if performing the x-ray will significantly delay reduction.
Ankle injuries
An ankle fracture relates to a fracture around the tibio-talar joint. It generally refers to a fracture involving the lateral, and/or medial and/or posterior malleolus. Pilon and Tillaux fractures are also considered to be ankle fractures, but are not covered here.
Ankle fractures are common. They effect men and women in equal numbers, but men have a higher rate as young adults (sports and contact injuries), and women a higher rate post-menopausal (fragility type fracture).
Osseous anatomy
The ankle (or mortise) joint consists of the distal tibia (tibial plafond and posterior malleolus), the distal fibula (lateral malleolus), and the talus. The main movement at the ankle joint is plantar and dorsiflexion.
Ligamentous anatomy
Medial side: Deltoid ligament. This is divided into superficial and deep portions. It is the primary restraint to valgus tilting of the talus.
Lateral side: Lateral ligament complex consisting from anterior to posterior of the anterior talofibular ligament (ATFL), calcaneofibular ligament (CFL), and the posterior talofibular ligament (PTFL). Together they resist valgus stress to the ankle, and are a restraint to anterior translation of the talus within the mortise joint.
Syndesmosis: The syndesmosis is a ligament complex between the distal tibia and fibula, holding the two bones together. It is fundamental to the integrity of the ankle joint, and its disruption leads to instability. It consists of (from anterior to posterior) the anterior-inferior tibiofibular ligament (AITFL), the transverse tibiofibular ligament (TTFL), the interosseous membrane, and the posterior-inferior tibiofibular ligament (PITFL).
Presentation and initial management
Patients will present following a traumatic event with a painful, swollen ankle, and reluctance/inability to weight bear. The Ottawa rules can be applied to differentiate between an ankle fracture and sprain, but can be unreliable.
In high energy injuries, management should follow ATLS principles to identify more significant injuries first. Neurovascular status of the foot should be documented, and open injuries should be excluded. If an open injury is identified, it should be managed in line with BOAST 4 principles1. If an obvious deformity exists, it should be reduced as soon as possible with appropriate analgesia or conscious sedation. Radiographs of clearly deformed or dislocated joints are not necessary, and removing the pressure on the surrounding soft tissues from the underlying bony deformity is the priority. If the fracture pattern is not clinically obvious then plain radiographs are appropriate and will guide the subsequent manipulation during plaster-of-paris below knee backslab application.
Imaging
AP, lateral and mortise views (20 degrees internal rotation) are essential to evaluate fracture displacement and syndesmotic injury. Decreased tibiofibular overlap, medial joint clear space and lateral talar shift all indicate a syndesmotic injury. (In subtle cases of shift, imaging the uninjured ankle can be helpful as a proportion of the population have little or no tibiotalar overlap-reference 2.)
Where there is suspicion of syndesmosis involvement in the absence of radiographic evidence, stress radiographs can be diagnostic.
Complex fracture patterns (and increasingly posterior malleolar fractures) are best defined using CT.
Classification
The most commonly used classifications are Lauge-Hansen and Danis-Weber.
Lauge-Hansen
Comprises two parts: first part is the foot position, and the second part is the force applied. Useful for understanding the forces involved and therefore predict the ligamentous or bony injury. Results in four injury patterns:
Supination - Adduction (SA) - 10-20%
Supination - External rotation (SER) - 40-75%
Pronation - Abduction (PA) - 5-20%
Pronation - External rotation (PER) - 5-20%
Not often used in clinical practice but good for understanding the principles of ankle fracture.
Danis-Weber
Commonly used. Based on the level of the fibula fracture in relation to the syndesmosis. The more proximal, the greater the risk of syndesmotic injury and therefore fracture instability.
A - fracture below the level of the syndesmosis
B - fracture at the level of the syndesmosis / level of the tibial plafond
C - fracture above the level of the syndesmosis. This includes Maisonneuve fractures (proximal fibula fracture), which can be associated with ankle instability. Beware the high fibula fracture - it may be an ankle fracture!
The Weber classification is based purely on the the lateral side. All injuries can include a medial or posterior bony or ligamentous injury which also dictates fracture stability (bimalleolar and trimalleolar fractures are more unstable).
Treatment
When deciding upon treatment for an ankle fracture, one must consider both the fracture and the patient. Diabetic patients and smokers are at greater risk of post-operative complication, especially wound problems and infection. Likewise, the long term outcome of post-traumatic arthritis from a malunited ankle fracture is extremely important for a young patient, but not as relevant in the elderly. Therefore, normal surgical decision processes apply as with all fractures.
Defining stability of an ankle fracture underpins the treatment decision.
Weber A - Unimalleolar Weber A Weber fractures by definition are stable and therefore can be mobilised fully weight bearing in an ankle boot.
Weber C - Fractures tend to include syndesmotic disruption and are usually bimalleolar (either bony or ligamentous). They are therefore unstable and usually require operative fixation. In addition to the fracture fixation, the syndesmosis usually requires reconstruction/augmentation with screws to restore the joint integrity and function.
Weber B - B fractures vary greatly. They can be part of a trimalleolar injury and therefore extremely unstable, requiring fixation. Alternatively, a uni-malleolar Weber B fracture can be a stable injury, and therefore mobilised immediately in an ankle boot. Defining the stability can be challenging, and often involves stress radiographs, or a trial of mobilisation and repeat radiographs. However, treating undisplaced ankle fractures in a below knee plaster, non-weight bearing for six weeks is still widely practised, and a safe approach.
When operative fixation is appropriate, it is usually via open reduction and internal fixation using plates and screws. It must be carried out when soft tissue swelling has settled in order to minimise the risk of wound problems. This can often take a week to settle.
The use of fibula nails is expanding, but is not yet mainstream. Ankle fractures can also be treated with external fixation, or with a hind foot nail in patients who need fixation but where soft tissue or bone quality is poor.
Post operative management
Ankle fractures generally take 6 weeks to unite enough to prevent secondary displacement. This is therefore an appropriate time period to keep a cast on in a conservatively managed patient. Weight bearing post-operatively depends on the quality of the fixation and bone quality, and preference varies between surgeons, ranging from aggressive early mobilisation to a period of non-weight bearing. Return to activities takes approximately three months, and often requires assistance of a physiotherapist to improve range-of-movement and muscle strengthening.
References
1. http://www.boa.ac.uk/publications/boa-standards-trauma-boasts/
2. Shah AS, Kadakia AR, Tan GJ, Karadsheh MS, Wolter TD, Sabb B. Radiographic evaluation of the normal distal tibiofibular syndesmosis. Foot Ankle Int. 2012;33(10):870-6
An 83 year old man with a long standing staghorn calculus presents with recurrent haematuria and investigation shows a mass of the left renal pelvis. Of the lesions listed below, the diagnosis is:
Squamous cell carcinoma
Adenocarcinoma
Transitional cell carcinoma
Sarcoma
Transitional metaplasia
SCC of the kidney usually arises in an area of chronic inflammation such as a staghorn calculus. They are rare.
Renal tumours
Renal cell carcinoma
Renal cell carcinoma is an adenocarcinoma of the renal cortex and is believed to arise from the proximal convoluted tubule. They are usually solid lesions, up to 20% may be multifocal, 20% may be calcified and 20% may have either a cystic component or be wholly cystic. They are often circumscribed by a pseudocapsule of compressed normal renal tissue. Spread may occur either by direct extension into the adrenal gland, renal vein or surrounding fascia. More distant disease usually occurs via the haematogenous route to lung, bone or brain.
Renal cell carcinoma comprise up to 85% of all renal malignancies. Males are more commonly affected than females and sporadic tumours typically affect patients in their sixth decade.
Patients may present with a variety of symptoms including; haematuria (50%), loin pain (40%), mass (30%) and up to 25% may have symptoms of metastasis.Less than 10% have the classic triad of haematuria, pain and mass.
Investigation
Many cases will present as haematuria and be discovered during diagnostic work up. Benign renal tumours are rare, so renal masses should be investigated with multislice CT scanning. Most tumours are also characterised with an arterial phase CT, particularly if they may be suitable for partial nephrectomy.
CT scanning of the chest and abdomen to detect distant disease should also be undertaken.
Routine bone scanning is not indicated in the absence of symptoms.
Biopsy should not be performed when a nephrectomy is planned but is mandatory before any ablative therapies are undertaken.
Assessment of the functioning of the contra lateral kidney.
Management
T1 lesions may be managed by partial nephrectomy and this gives equivalent oncological results to total radical nephrectomy. Partial nephrectomy may also be performed when there is inadequate reserve in the remaining kidney. Ablative techniques may also be considered for small T1 lesions in unfit patients. In general, outcomes are less favorable than with surgical resection.
For T2 lesions and above a radical nephrectomy is standard practice and this may be performed via a laparoscopic or open approach. Preoperative embolisation is not indicated nor is resection of uninvolved adrenal glands. During surgery early venous control is mandatory to avoid shedding of tumour cells into the circulation.
Patients with completely resected disease do not benefit from adjuvant therapy with either chemotherapy or biological agents. These should not be administered outside the setting of clinical trials.
Patients with transitional cell cancer will require a nephroureterectomy with disconnection of the ureter at the bladder.
References
Capitanio U, Montorsi F. Renal cancer. Lancet. 2016 Feb 27;387(10021):894-906.
Capitanio U et al. Epidemiology of Renal Cell Carcinoma. Eur Urol. 2019 Jan;75(1):74-84.
Which of the following features are not typically seen in a venous stasis ulcer?
Located above the medial malleolus
Haemosiderin deposits
Variable scarring
Exophytic granulation tissue
Varicose veins
Whilst the base of the ulcer may be lined by granulation tissue, this is seldom exophytic. One of the concerns would be that this represents a malignant transformation.
Chronic venous insufficiency and varicose veins
Wide spectrum of disease ranging from minor cosmetic problem through to ulceration and disability. It is commoner in women than men and is worse during pregnancy. Varicose veins are best considered as being a saccular dilation of veins (WHO). Chronic venous insufficiency is a series of tissue changes which occur in relation to pooling of blood in the extremities with associated venous hypertension occurring as a result of incompetent deep vein valves.
The veins of the lower limb consist of an interconnected network of superficial and deep venous systems. Varices occur because of localised weakness in the vein wall resulting in dilatation and reflux of blood due to non union of valve cusps. Histologically, the typical changes include fibrous scar tissue dividing smooth muscle within media in the vessel wall.
Tissue damage in chronic venous insufficiency occurs because of perivascular cytokine leakage resulting in localised tissue damage coupled with impaired lymphatic flow.
Diagnosis
Typical symptoms of varicose veins include:
Cosmetic appearance
Aching
Ankle swelling that worsens as the day progresses
Episodic thrombophlebitis
Bleeding
Itching
Symptoms of chronic venous insufficiency include:
Dependant leg pain
Prominent leg swelling
Oedema extending beyond the ankle
Venous stasis ulcers
The typical venous stasis ulcer is:
Located above the medial malleolus
Indolent appearance with basal granulation tissue
Variable degree of scarring
Non ischaemic edges
Haemosiderin deposition in the gaiter area (and also lipodermatosclerosis).
Differential diagnosis
Lower limb arterial disease
Marjolins ulcer
Claudication
Spinal stenosis
Swelling due to medical causes e.g. CCF.
Exclusion of these differentials is by means of physical examination and ankle brachial pressure index measurement.
Examination
Assess for dilated short saphenous vein (popliteal fossa) and palpate for saphena varix medial to the femoral artery
Brodie-Trendelenburg test: to assess level of incompetence
Perthes’ walking test: assess if deep venous system competent
Investigation
Doppler exam: if incompetent a biphasic signal due to retrograde flow is detected
Duplex scanning: to ensure patent deep venous system (do if DVT or trauma)
All patients should have a Doppler assessment to assess for venous reflux and should be classified as having uncomplicated varicose veins or varicose veins with associated chronic venous insufficiency. In the history establishing a previous thrombotic event (DVT/ lower limb fracture) is important and patients with such a history and all who have evidence of chronic venous insufficiency should have a duplex scan performed.
Owing to litigation patients with saphenopopliteal incompetence should have a duplex scan performed and the site marked by scan on the day of surgery.
Treatment
Indications for surgery:
Cosmetic: majority
Lipodermatosclerosis causing venous ulceration
Recurrent superficial thrombophlebitis
Bleeding from ruptured varix
Condition Therapy
Minor varicose veins - no complications Reassure/ cosmetic therapy
Symptomatic uncomplicated varicose veins In those without deep venous insufficiency options include; endothermal ablation, foam sclerotherapy, saphenofemoral / popliteal disconnection, stripping and avulsions, compression stockings
Varicose veins with skin changes Therapy as above (if compression minimum is formal class I stockings)
Chronic venous insufficiency or ulcers Class 2-3 compression stockings (ensure no arterial disease).
Application of formal compression stockings (usually class II/III). In patients who have suffered ulceration, compression stockings should be worn long term. Where ulceration is present and established saphenofemoral reflux exists this should be addressed surgically for durable relief of symptoms, either at the outset or following ulcer healing.
Injection sclerotherapy (5% Ethanolamine oleate), foam is increasingly popular, though transient blindness has been reported. Endo venous laser therapy is another minimally invasive option
Sapheno-femoral or sapheno-popliteal ligation, in the case of the LSV; stripping and multiple phlebectomies
Current best practice guidance
In the United Kingdom the National Institute of Clinical Excellence guidance on varicose veins suggests that for patients with symptomatic varicose veins the first line procedure of choice should be endothermal ablation (see reference for more information). Where this is unavailable or unsuitable then foam sclerotherapy should be the second line option. Surgery is currently the third line treatment option.
Trendelenburg procedure (sapheno-femoral junction ligation)
Head tilt 15 degrees and legs abducted
Oblique incision 1cm medial from artery
Tributaries ligated (Superficial circumflex iliac vein, Superficial inferior epigastric vein, Superficial and deep external pudendal vein)
SF junction double ligated
Saphenous vein stripped to level of knee/upper calf. NB increased risk of saphenous neuralgia if stripped more distally
References
Marsden G et al. Diagnosis of management of varicose veins in the the legs: summary of NICE guidance. BMJ 2013 (347): 30-31.
How many compartments are there in the lower leg?
2
1
3
5
4
The posterior compartment of the lower leg has both superficial and deep posterior layers, together with the anterior and lateral compartments this allows for four compartments. Decompression of the deep posterior compartment during fasciotomy may be overlooked with significant sequelae.
Fascial compartments of the leg
Compartments of the thigh
Formed by septae passing from the femur to the fascia lata.
Compartment Nerve Muscles Blood supply
Anterior compartment Femoral
Iliacus
Tensor fasciae latae
Sartorius
Quadriceps femoris
Femoral artery
Medial compartment Obturator
Adductor longus/magnus/brevis
Gracilis
Obturator externus
Profunda femoris artery and obturator artery
Posterior compartment (2 layers) Sciatic
Semimembranosus
Semitendinosus
Biceps femoris
Branches of Profunda femoris artery
Compartments of the lower leg
Separated by the interosseous membrane (anterior and posterior compartments), anterior fascial septum (separate anterior and lateral compartments) and posterior fascial septum (separate lateral and posterior compartments)
Compartment Nerve Muscles Blood supply
Anterior compartment Deep peroneal nerve
Tibialis anterior
Extensor digitorum longus
Extensor hallucis longus
Peroneus tertius
Anterior tibial artery
Posterior compartment Tibial
Muscles: deep and superficial compartments (separated by deep transverse fascia)
Deep: Flexor hallucis longus, Flexor digitalis longus, Tibialis posterior, Popliteus
Superficial: Gastrocnemius, Soleus, Plantaris
Posterior tibial
Lateral compartment Superficial peroneal
Peroneus longus/brevis
Peroneal artery
A 6 year old boy presents with pain in the hip it is present on activity and has been worsening over the past few weeks. There is no history of trauma. He was born by normal vaginal delivery at 38 weeks gestation On examination he has an antalgic gait and limitation of active and passive movement of the hip joint in all directions. C-reactive protein is mildly elevated at 10 but the white cell count is normal. What is the most likely diagnosis?
Perthes disease
Septic arthritis
Slipped upper femoral epiphysis
Developmental dysplasia of the hip
Osteoarthritis
Early plain x-ray changes in Perthes Disease:
Widening of the joint space.
Sub chondral linear lucency.
This is a typical presentation for Perthes disease. X-ray may show flattening of the femoral head or fragmentation in more advanced cases.
Paediatric orthopaedics
Diagnosis Mode of presentation Treatment Radiology
Developmental dysplasia of the hip Usually diagnosed in infancy by screening tests. May be bilateral, when disease is unilateral there may be leg length inequality. As disease progresses child may limp and then early onset arthritis. More common in extended breech babies. Splints and harnesses or traction. In later years osteotomy and hip realignment procedures may be needed. In arthritis a joint replacement may be needed. However, this is best deferred if possible as it will almost certainly require revision Initially no obvious change on plain films and USS gives best resolution until 3 months of age. On plain films Shentons line should form a smooth arc
Perthes Disease Hip pain (may be referred to the knee) usually occurring between 5 and 12 years of age. Bilateral disease in 20%. Remove pressure from joint to allow normal development. Physiotherapy. Usually self-limiting if diagnosed and treated promptly. X-rays will show flattened femoral head. Eventually in untreated cases the femoral head will fragment.
Slipped upper femoral epiphysis Typically seen in obese male adolescents. Pain is often referred to the knee. Limitation to internal rotation is usually seen. Knee pain is usually present 2 months prior to hip slipping. Bilateral in 20%. Bed rest and non-weight bearing. Aim to avoid avascular necrosis. If severe slippage or risk of it occurring then percutaneous pinning of the hip may be required. X-rays will show the femoral head displaced and falling inferolaterally (like a melting ice cream cone) The Southwick angle gives indication of disease severity
Which of the following substances is released from the sympathetic nervous system to stimulate the adrenal medulla?
Noradrenaline
Acetyl choline
Substance P
Tyrosine
Arginine
In the autonomic nervous system, noradrenaline is the commonly used neurotransmitter. However, in the adrenal medulla, Acetylcholine is released to stimulate adrenaline release.
Adrenal physiology
Adrenal medulla
The chromaffin cells of the adrenal medulla secrete the catecholamines noradrenaline and adrenaline. The medulla is innervated by the splanchnic nerves; the preganglionic sympathetic fibres secrete acetylcholine causing the chromaffin cells to secrete their contents by exocytosis.
Phaeochromocytomas are derived from these cells and will secrete both adrenaline and nor adrenaline.
Adrenal cortex
Three histologically distinct zones are recognised:
Zone Location Hormone Secreted
Zona glomerulosa Outer zone Aldosterone
Zona fasiculata Middle zone Glucocorticoids
Zona reticularis Inner zone Androgens
The glucocorticoids and aldosterone are mostly bound to plasma proteins in the circulation. Glucocorticoids are inactivated and excreted by the liver.
A 68 year old man is involved in a road traffic accident that is relatively minor and sustains a minor head injury with small associated scalp laceration. Apart from atrial fibrillation for which he is treated with bisoprolol and dabigatran, he is well. On examination, he is drowsy and GCS is 12 with no localizing neurological signs. What is the most appropriate course of action?
Arrange a skull x-ray
Arrange an MRI scan of the brain
Admit for observation with GCS measurements every 30 minutes
Admit for observation with GCS measurements every 60 minutes
Arrange a CT scan of the head
Head injury with anticoagulants= CT head
Head injury management- NICE Guidelines
Summary of guidelines
All patients should be assessed within 15 minutes on arrival to A&E
Document all 3 components of the GCS
If GCS <8 or = to 8, consider stabilising the airway
Full spine immobilisation until assessment if:
- GCS < 15
- neck pain/tenderness
- paraesthesia extremities
- focal neurological deficit
- suspected c-spine injury
If a c-spine injury is suspected a 3 view c-spine x-ray is indicated. CT c-spine is preferred if:
- Intubated
- GCS <13
- Normal x-ray but continued concerns regarding c-spine injury
- Any focal neurology
- A CT head scan is being performed
- Initial plain films are abnormal
Immediate CT head (within 1 hour) if:
GCS < 12 on admission
GCS < 15 2 hours after admission
Suspected open or depressed skull fracture
Suspected skull base fracture (panda eyes, Battle’s sign, CSF from nose/ear, bleeding ear)
Focal neurology
Vomiting > 1 episode
Post traumatic seizure
Contact neurosurgeon if:
Persistent GCS < 8 or = 8
Unexplained confusion > 4h
Reduced GCS after admission
Progressive neurological signs
Incomplete recovery post seizure
Penetrating injury
Cerebrospinal fluid leak
Observations
1/2 hourly GCS until 15
In the latest guidance, there is a move to considering CT in those on anticoagulants rather than mandating it.
Reference
1. NICE Guidance NG 232 (Published 2023).
2. Hodgkinson S et al. Early management of head injury: summary of NICE guidance. BMJ 2014 (348):34-37.
A 43 year old lady presents with varicose veins and undergoes a saphenofemoral disconnection, long saphenous vein stripping to the ankle and isolated hook phlebectomies. Post operatively she notices an area of numbness superior to her ankle. What is the most likely cause for this?
Sural nerve injury
Femoral nerve injury
Saphenous nerve injury
Common peroneal nerve injury
Superficial peroneal nerve injury
The sural nerve is related to the short saphenous vein. The saphenous nerve is related to the long saphenous vein below the knee and for this reason full length stripping of the vein is no longer advocated.
Saphenous vein
Long saphenous vein
This vein may be harvested for bypass surgery, or removed as treatment for varicose veins with saphenofemoral junction incompetence.
Originates at the 1st digit where the dorsal vein merges with the dorsal venous arch of the foot
Passes anterior to the medial malleolus and runs up the medial side of the leg
At the knee, it runs over the posterior border of the medial epicondyle of the femur bone
Then passes laterally to lie on the anterior surface of the thigh before entering an opening in the fascia lata called the saphenous opening
It joins with the femoral vein in the region of the femoral triangle at the saphenofemoral junction
Tributaries
Medial marginal
Superficial epigastric
Superficial iliac circumflex
Superficial external pudendal veins
Short saphenous vein
Originates at the 5th digit where the dorsal vein merges with the dorsal venous arch of the foot, which attaches to the great saphenous vein.
It passes around the lateral aspect of the foot (inferior and posterior to the lateral malleolus) and runs along the posterior aspect of the leg (with the sural nerve)
Passes between the heads of the gastrocnemius muscle, and drains into the popliteal vein, approximately at or above the level of the knee joint.
Which of the following are not typical of Lynch syndrome?
It is inherited in an autosomal recessive manner
Affected patients are more likely to develop right colon mucinous tumours than the general population
Affected individuals have an 80% lifetime risk of colon cancer
Endometrial cancer is seen in 80% of women
Gastric cancers are more common
Lynch syndrome is inherited in an autosomal dominant fashion. It is characterised by microsatellite instability in the DNA mismatch repair genes. Colonic tumours in patients with Lynch syndrome are more likely to be right sided tumours and to be poorly differentiated.
Genetics and surgical disease
Some of the more commonly occurring genetic conditions occurring in surgical patients are presented here.
Li-Fraumeni Syndrome
Autosomal dominant
Consists of germline mutations to p53 tumour suppressor gene
High incidence of malignancies particularly sarcomas and leukaemias
Diagnosed when:
*Individual develops sarcoma under 45 years
*First degree relative diagnosed with any cancer below age 45 years and another family member develops malignancy under 45 years or sarcoma at any age
BRCA 1 and 2
Carried on chromosome 17 (BRCA 1) and Chromosome 13 (BRCA 2)
Linked to developing breast cancer (60%) risk.
Associated risk of developing ovarian cancer (55% with BRCA 1 and 25% with BRCA 2).
Lynch Syndrome
Autosomal dominant
Develop colonic cancer and endometrial cancer at young age
80% of affected individuals will get colonic and/ or endometrial cancer
High risk individuals may be identified using the Amsterdam criteria
Amsterdam criteria
Three or more family members with a confirmed diagnosis of colorectal cancer, one of whom is a first degree (parent, child, sibling) relative of the other two.
Two successive affected generations.
One or more colon cancers diagnosed under age 50 years.
Familial adenomatous polyposis (FAP) has been excluded.
Gardners syndrome
Autosomal dominant familial colorectal polyposis
Multiple colonic polyps
Extra colonic diseases include: skull osteoma, thyroid cancer and epidermoid cysts
Desmoid tumours are seen in 15%
Mutation of APC gene located on chromosome 5
Due to colonic polyps most patients will undergo colectomy to reduce risk of colorectal cancer
Now considered a variant of familial adenomatous polyposis coli
A 60 year old man presents with recurrent renal stones. He is found to have a calcium of 2.72 (elevated) and a PTH of 12 (elevated). What is the most appropriate long term management plan?
Neck exploration and parathyroidectomy
CT scanning of the neck
MRI scanning of the neck
Medical treatment alone
External beam radiotherapy to the neck
This patient has primary hyperparathyroidism and nephrolithiasis, which is an indication for parathyroidectomy.
Parathyroid glands and disorders of calcium metabolism
Hyperparathyroidism
Disease type Hormone profile Clinical features Cause
Primary hyperparathyroidism
PTH (Elevated)
Ca2+ (Elevated)
Phosphate (Low)
Urine calcium : creatinine clearance ratio > 0.01
May be asymptomatic if mild
Recurrent abdominal pain (pancreatitis, renal colic)
Changes to emotional or cognitive state
Most cases due to solitary adenoma (80%), multifocal disease occurs in 10-15% and parathyroid carcinoma in 1% or less
Secondary hyperparathyroidism
PTH (Elevated)
Ca2+ (Low or normal)
Phosphate (Elevated)
Vitamin D levels (Low)
May have few symptoms
Eventually may develop bone disease, osteitis fibrosa cystica and soft tissue calcifications
Parathyroid gland hyperplasia occurs as a result of low calcium, almost always in a setting of chronic renal failure
Tertiary hyperparathyroidism
Ca2+ (Normal or high)
PTH (Elevated)
Phosphate levels (Decreased or Normal)
Vitamin D (Normal or decreased)
Alkaline phosphatase (Elevated)
Metastatic calcification
Bone pain and / or fracture
Nephrolithiasis
Pancreatitis
Occurs as a result of ongoing hyperplasia of the parathyroid glands after correction of underlying renal disorder, hyperplasia of all 4 glands is usually the cause
Differential diagnoses
It is important to consider the rare but relatively benign condition of benign familial hypocalciuric hypercalcaemia, caused by an autosomal dominant genetic disorder. Diagnosis is usually made by genetic testing and concordant biochemistry (urine calcium : creatinine clearance ratio <0.01-distinguished from primary hyperparathyroidism).
Treatment
Primary hyperparathyroidism
Indications for surgery
Elevated serum Calcium > 1mg/dL above normal
Hypercalciuria > 400mg/day
Creatinine clearance < 30% compared with normal
Episode of life threatening hypercalcaemia
Nephrolithiasis
Age < 50 years
Neuromuscular symptoms
Reduction in bone mineral density of the femoral neck, lumbar spine, or distal radius of more than 2.5 standard deviations below peak bone mass (T score lower than -2.5)
Secondary hyperparathyroidism
Usually managed with medical therapy.
Indications for surgery in secondary (renal) hyperparathyroidism:
Bone pain
Persistent pruritus
Soft tissue calcifications
Tertiary hyperparathyroidism
Allow 12 months to elapse following transplant as many cases will resolve
The presence of an autonomously functioning parathyroid gland may require surgery. If the culprit gland can be identified then it should be excised. Otherwise total parathyroidectomy and re-implantation of part of the gland may be required.
References
1. Insogna K. Primary Hyperparathyroidism. N Engl J Med. 2018 Sep 13;379(11):1050-1059.
- van der Plas W Y et al. Secondary and Tertiary Hyperparathyroidism: A Narrative Review. Scand J Surg. 2020 Dec;109(4):271-278.
A 34 year old man undergoes a sub total colectomy to treat fulminant ulcerative colitis. What type of stoma is most likely to be fashioned?
End colostomy
Loop colostomy
End ileostomy
Loop ileostomy
End jejunostomy
A sub total colectomy involves the removal of the entire right, transverse, left and part of the sigmoid colon. The rectal stump is closed and an end ileostomy fashioned in the right iliac fossa.
Abdominal stomas
Stomas may be sited during a range of abdominal procedures and involve bringing the lumen or visceral contents onto the skin. In most cases this applies to the bowel. However, other organs or their contents may be diverted in case of need.
With bowel stomas the type method of construction and to a lesser extent the site will be determined by the contents of the bowel. In practice, small bowel stomas should be spouted so that their irritant contents are not in contact with the skin. Colonic stomas do not need to be spouted as their contents are less irritant.
In the ideal situation the site of the stoma should be marked with the patient prior to surgery. Stoma siting is important as it will ultimately influence the ability of the patient to manage their stoma and also reduce the risk of leakage. Leakage of stoma contents and subsequent maceration of the surrounding skin can rapidly progress into a spiraling loss of control of stoma contents.
Types of stomas
Name of stoma Use Common sites
Gastrostomy
Gastric decompression or fixation
Feeding
Epigastrium
Loop jejunostomy
Seldom used as very high output
May be used following emergency laparotomy with planned early closure
Any location according to need
Percutaneous jejunostomy
Usually performed for feeding purposes and site in the proximal bowel
Usually left upper quadrant
Loop ileostomy
Defunctioning of colon e.g. following rectal cancer surgery
Does not decompress colon (if ileocaecal valve competent)
Usually right iliac fossa
End ileostomy
Usually following complete excision of colon or where ileo-colic anastomosis is not planned
May be used to defunction colon, but reversal is more difficult
Usually right iliac fossa
End colostomy Where a colon is diverted or resected and anastomosis is not primarily achievable or desirable Either left or right iliac fossa
Loop colostomy
To defunction a distal segment of colon
Since both lumens are present the distal lumen acts as a vent
May be located in any region of the abdomen, depending upon colonic segment used
Caecostomy Stoma of last resort where loop colostomy is not possible Right iliac fossa
Mucous fistula
To decompress a distal segment of bowel following colonic division or resection
Where closure of a distal resection margin is not safe or achievable
May be located in any region of the abdomen according to clinical need
Which of the following is not an intrinsic muscle of the hand?
Opponens pollicis
Palmaris longus
Flexor pollicis brevis
Flexor digiti minimi brevis
Opponens digiti minimi
Mnemonic for intrinsic hand muscles
‘A OF A OF A’
A bductor pollicis brevis
O pponens pollicis
F lexor pollicis brevis
A dductor pollicis (thenar muscles)
O pponens digiti minimi
F lexor digiti minimi brevis
A bductor digiti minimi (hypothenar muscles)
Palmaris longus originates in the forearm.
Hand
Anatomy of the hand
Bones
8 Carpal bones
5 Metacarpals
14 phalanges
Intrinsic Muscles 8 Interossei - Supplied by ulnar nerve
4 palmar-adduct fingers
4 dorsal- abduct fingers
Intrinsic muscles Lumbricals
Flex MCPJ and extend the IPJ.
Origin deep flexor tendon and insertion dorsal extensor hood mechanism.
Innervation: 1st and 2nd- median nerve, 3rd and 4th- deep branch of the ulnar nerve.
Thenar eminence
Abductor pollicis brevis
Opponens pollicis
Flexor pollicis brevis
Hypothenar eminence
Opponens digiti minimi
Flexor digiti minimi brevis
Abductor digiti minimi
Fascia and compartments of the palm
The fascia of the palm is continuous with the antebrachial fascia and the fascia of the dorsum of the hand. The palmar fascia is thin over the thenar and hypothenar eminences. In contrast, the central palmar fascia is relatively thick. The palmar aponeurosis covers the soft tissues and overlies the flexor tendons. The apex of the palmar aponeurosis is continuous with the flexor retinaculum and the palmaris longus tendon. Distally, it forms four longitudinal digital bands that attach to the bases of the proximal phalanges, blending with the fibrous digital sheaths.
A medial fibrous septum extends deeply from the medial border of the palmar aponeurosis to the 5th metacarpal. Lying medial to this are the hypothenar muscles. In a similar fashion, a lateral fibrous septum extends deeply from the lateral border of the palmar aponeurosis to the 3rd metacarpal. The thenar compartment lies lateral to this area.
Lying between the thenar and hypothenar compartments is the central compartment. It contains the flexor tendons and their sheaths, the lumbricals, the superficial palmar arterial arch and the digital vessels and nerves.
The deepest muscular plane is the adductor compartment, which contains adductor pollicis.
Short muscles of the hand
These comprise the lumbricals and interossei. The four slender lumbrical muscles flex the fingers at the metacarpophalangeal joints and extend the interphalangeal joint. The four dorsal interossei are located between the metacarpals and the four palmar interossei lie on the palmar surface of the metacarpals in the interosseous compartment of the hand.
Long flexor tendons and sheaths in the hand
The tendons of FDS and FDP enter the common flexor sheath deep to the flexor retinaculum. The tendons enter the central compartment of the hand and fan out to their respective digital synovial sheaths. Near the base of the proximal phalanx, the tendon of FDS splits to permit the passage of FDP. The FDP tendons are attached to the margins of the anterior aspect of the base of the distal phalanx.
The fibrous digital sheaths contain the flexor tendons and their synovial sheaths. These extend from the heads of the metacarpals to the base of the distal phalanges.
Palmar Interossei
Note that there are 4 palmar interossei. The first is a small slip of muscle which arises from the ulnar side of the base of the first metacarpal and passes between the head of the first dorsal interosseous and the oblique head of adductor pollicis to insert into the ulnar base of the of the proximal phalanx of the thumb. The second arises from the ulnar side of the body of the second metacarpal and is inserted into the ulnar side of the extensor hood of the index. The third and fourth palmar interossei arise from the radial sides of the bodies of the 4th and 5th metacarpals respectively and insert into the radial sides of the extensor hoods of the ring and little fingers.
Which of the following statements relating to alveolar ventilation is untrue?
Anatomical dead space is measured by helium dilution
Physiological dead space is increased in PE
Alveolar ventilation is defined as the volume of fresh air entering the alveoli per minute
Anatomical dead space is increased by adrenaline
Type 2 pneumocytes in the alveoli secrete surfactant
Anatomical dead space is measured by Fowlers method.
A patient inhales 100% oxygen to empty the conducting zone gases of nitrogen and then exhales through a mouthpiece which analyses the nitrogen concentration at the mouth. Initially the exhaled gases contain no nitrogen as this is dead space gas; the nitrogen concentration will increase as the alveolar gases are exhaled. Nitrogen which is measured following the breath of 100% oxygen must then have come only from gas exchanging areas of the lung and not dead space.
Alveolar ventilation
- Minute ventilation is the total volume of gas ventilated per minute.
MV (ml/min)= tidal volume x Respiratory rate (resps/min).
Dead space ventilation describes the volume of gas not involved in exchange in the blood.
There are 2 types:
- Anatomical dead space: 150mls
Volume of gas in the respiratory tree not involved in gaseous exchange: mouth, pharynx, trachea, bronchi up to terminal bronchioles
Measured by Fowlers method
Increased by:
Standing, increased size of person, increased lung volume and drugs causing bronchodilatation e.g. Adrenaline - Physiological dead space: normal 150 mls, increases in ventilation/perfusion mismatch e.g. PE, COPD, hypotension
Volume of gas in the alveoli and anatomical dead space not involved in gaseous exchange.
Alveolar ventilation is the volume of fresh air entering the alveoli per minute.
Alveolar ventilation = minute ventilation - Dead space volume Alveolar ventilation
A 25 year old man sustains a severe middle cranial fossa basal skull fracture. Once he has recovered it is noticed that he has impaired tear secretion. This is most likely to be the result of damage to which of the following?
Stellate ganglion
Ciliary ganglion
Otic ganglion
Trigeminal nerve
Greater petrosal nerve
The greater petrosal nerve may be injured and carries fibres for lacrimation (see below).
Lacrimal system
Lacrimal gland
Consists of an orbital part and palpebral part. They are continuous posterolaterally around the concave lateral edge of the levator palpebrae superioris muscle.
The ducts of the lacrimal gland open into the superior fornix. Those from the orbital part penetrate the aponeurosis of levator palpebrae superioris to join those from the palpebral part. Therefore excision of the palpebral part is functionally similar to excision of the entire gland.
Blood supply
Lacrimal branch of the opthalmic artery. Venous drainage is to the superior opthalmic vein.
Innervation
The gland is innervated by the secretomotor parasympathetic fibres from the pterygopalatine ganglion which in turn may reach the gland via the zygomatic or lacrimal branches of the maxillary nerve or pass directly to the gland. The preganglionic fibres travel to the ganglion in the greater petrosal nerve (a branch of the facial nerve at the geniculate ganglion).
Nasolacrimal duct
Descends from the lacrimal sac to open anteriorly in the inferior meatus of the nose.
Lacrimation reflex
Occurs in response to conjunctival irritation (or emotional events). The conjunctiva will send signals via the opthalmic nerve. These then pass to the superior salivary centre. The efferent signals pass via the greater petrosal nerve (parasympathetic preganglionic fibres) and the deep petrosal nerve which carries the post ganglionic sympathetic fibres. The parasympathetic fibres will relay in the pterygopalatine ganglion, the sympathetic fibres do not synapse. They in turn will relay to the lacrimal apparatus.
A 65 year old lady is recovering from major abdominal surgery in ITU and, as part of her clinical care a clotting screen is performed. The prothrombin time is noted to be increased. Which of the conditions below is unlikely to account for this finding?
Cholestatic jaundice
Disseminated intravascular coagulation
Prolonged antibiotic treatment
Liver disease
Acquired factor 12 deficiency
Acquired factor 12 deficiency
Vitamin K deficiency results from cholestatic jaundice and prolonged antibiotic therapy. Acquired factor 12 deficiency causes prolonged APTT.
Abnormal coagulation
Cause Factors affected
Heparin Prevents activation factors 2,9,10,11
Warfarin Affects synthesis of factors 2,7,9,10
DIC Factors 1,2,5,8,11
Liver disease Factors 1,2,5,7,9,10,11
Interpretation blood clotting test results
Disorder APTT PT Bleeding time
Haemophilia Increased Normal Normal
von Willebrand’s disease Increased Normal Increased
Vitamin K deficiency Increased Increased Normal
A 35 year old type 1 diabetic presents with difficulty mobilising and back pain radiating to the thigh. He has a temperature of 39 oC and has pain on extension of the hip. He is diagnosed with an iliopsoas abscess. Which of the following statements is false in relation to his diagnosis?
Staphylococcus aureus is the most likely primary cause
Recurrence occurs in 60% cases
More common in males
Crohn’s is the most likely secondary cause in patients within the UK
CT guided drainage is preferable first line management
Recurrence occurs in 60% cases
Classical features include: a limp, back pain and fever. Recurrence rates are about 15-20%. In the UK, Staphylococcus is the commonest primary cause, others include Streptococcus and E.coli. Management is ideally by CT guided drainage.
Iliopsoas abscess
- Collection of pus in iliopsoas compartment (iliopsoas and iliacus)
Causes:
Primary
Haematogenous spread of bacteria
Staphylococcus aureus: most common
Secondary
Crohn’s (commonest cause in this category)
Diverticulitis, Colorectal cancer
UTI, GU cancers
Vertebral osteomyelitis
Femoral catheter, lithotripsy
Endocarditis
Note the mortality rate can be up to 19-20% in secondary iliopsoas abscesses compared with 2.4% in primary abscesses.
Clinical features
Fever
Back/flank pain
Limp
Weight loss
Clinical examination
Patient in the supine position with the knee flexed and the hip mildly externally rotated
Specific tests to diagnose iliopsoas inflammation:
Place hand proximal to the patient’s ipsilateral knee and ask patient to lift thigh against your hand. This will cause pain due to contraction of the psoas muscle.
Lie the patient on the normal side and hyperextend the affected hip. In inflammation this should elicit pain as the psoas muscle is stretched.
Investigation
CT is gold standard
Management
Antibiotics
Percutaneous drainage
Surgery is indicated if:
- Failure of percutaneous drainage
- Presence of an another intra-abdominal pathology which requires surgery
Surgical approach
The authors technique for draining these collections is given here.
Review the CT scans and plan surgical approach. An extraperitoneal approach is important.
The collection usually extends inferiorly and can be accessed from an incision at a level of L4 on the affected side.
GA
Transverse laterally placed incision.
Incise external oblique.
Split the subsequent muscle layers.
As you approach the peritoneum use blunt dissection to pass laterally around it.
Remember the ureter and gonadal veins lie posterior at this level.
Eventually you will enter the abscess cavity, a large amount of pus is usually released at this point. Drain the area with suction and washout with saline.
Place a corrugated drain well into the abscess cavity.
If you have made a small skin incision it is reasonable to bring the drain up through the skin wound. Otherwise place a lateral exit site and close the skin and external oblique. If you do this ensure that you use interrupted sutures.
Anchor the drain with strong securely tied silk sutures (it is extremely tiresome if it falls out!)
Reference
Iliopsoas abscesses
I H Mallick, M H Thoufeeq, T P Rajendran
Postgrad Med J 2004;80:459-462
Which of the following statements relating to malignant mesothelioma is false?
It may be treated by extrapleural pneumonectomy.
It is linked to asbestos exposure.
It is linked to cigarette smoking independent of asbestos exposure.
It may occur intra abdominally.
It is relatively resistant to radiotherapy
It is not linked to cigarette smoking. When identified at an early stage a radical resection is the favored option. Radiotherapy is often given perioperatively. However, it is not a particularly radiosensitive tumour. Combination chemotherapy gives some of the best results and most regimes are cisplatin based.
Occupational cancers
Occupational cancers accounted for 5.3% cancer deaths in 2005.
In men the main cancers include:
Mesothelioma
Bladder cancer
Non melanoma skin cancer
Lung cancer
Sino nasal cancer
Occupations with high levels of occupational tumours include:
Construction industry
Working with coal tar and pitch
Mining
Metalworkers
Working with asbestos (accounts for 98% of all mesotheliomas)
Working in rubber industry
Shift work has been linked to breast cancer in women (Health and safety executive report RR595).
The latency between exposure and disease is typically 15 years for solid tumours and 20 for leukaemia.
Many occupational cancers are otherwise rare. For example sino nasal cancer is an uncommon tumour, 50% will be SCC. They are linked to conditions such as wood dust exposure and unlike lung cancer is not strongly linked to cigarette smoking. Another typical occupational tumour is angiosarcoma of the liver which is linked to working with vinyl chloride. Again in the non occupational context this is an extremely rare sporadic tumour.
Where are the reticulo-endothelial cells concentrated within the spleen?
Within the capsule
Within the red pulp
Within the trabeculae
Within the white pulp
Within the splenic cords
The reticuloendothelial cells are concerned with the immune functions of the spleen and these are therefore concentrated in the white pulp.
Spleen- function
The spleen is a reticuloendothelial organ, it develops in the dorsal mesogastrium at around 5 weeks gestation. At this stage it has an irregular surface and migrates to the left upper quadrant over the following weeks. In most cases the irregular hillocks on the spleen surface unify, when they fail to do so, accessory spleens may develop and are found in around 20% of people. The spleen is composed of both red and white pulp. In the red pulp, blood filled venous sinuses are found. In the white pulp, reticuloendothelial cords and white lymphoid follicles are present. Blood flows into the spleen at a rate of 150ml per minute. As blood passes through the spleen, the erythrocytes have to pass through fine endothelial fenestrations, older and less deformable erythrocytes are trapped during this process and destroyed. Red cell inclusion bodies such as parasites or residual nuclear components are split off during this process which is termed pitting.
Functions of spleen
Maintenance of the quality of erythrocytes in the red pulp by removal of senescent and dysfunctional cells
Antibody production in the white pulp
Removal of antibody coated bacteria and blood cells from the circulation
Because of these important functions, the spleen is seldom removed. Indications for splenectomy include major trauma and uncontrollable haemorrhage and the treatment of haemolytic anaemia.
The absence of a spleen has minimal long term effects on the haematologic profile. In the immediate post operative period, both leucocytosis and thrombocytosis are seen. In the longer term, the main manifestations are visible on the blood film and include visible nuclear remnants (Howell Jolly bodies), denatured haemoglobin (Heinz bodies), basophilic stippling and occasional nucleated erythrocytes.
The main risk following splenectomy is overwhelming sepsis with encapsulated organisms. Patients should receive the pneumococcal vaccine and long term antibiotic prophylaxis.
Which of the structures listed below is not typically transmitted via the superior orbital fissure?
Lacrimal nerve
Abducens nerve
Zygomatic nerve
Superior opthalmic vein
Superior division of the oculomotor nerve
The zygomatic nerve is transmitted via the inferior orbital fissure.
A useful mnemonic for the structures that pass through the superior orbital fissure:
SUPERIOR ORBITAL FISSURE:
Outside
L - lacrimal F - frontal T - trochlear done
Inside
N - nasociliary A - abducens nerve O - oculomotor nerve
LFT done outside No Abnormality Occurred inside
Orbital foramina
Foramina Structures transmitted
Superior orbital fissure Recurrent meningeal artery*
Lacrimal nerve
Trochlear nerve
Abducens nerve
Superior ophthalmic vein
Superior division of the oculomotor nerve
Inferior orbital fissure Maxillary nerve
Inferior ophthalmic vein
Zygomatic nerve
Optic foramen Optic nerve
Ophthalmic artery
*=It may help to know that it is also termed the anastomotic branch ( of the middle meningeal artery). It enters the orbit lateral in the superior orbital fissure, it anastomoses with the recurrent branch of the lacrimal artery.
A 53 year old lady is recovering following a difficult mastectomy and axillary nodal clearance for carcinoma of the breast. She complains of shoulder pain and on examination has obvious winging of the scapula. Loss of innervation to which of the following is the most likely underlying cause?
Latissimus dorsi
Serratus anterior
Pectoralis minor
Pectoralis major
Rhomboids
Serratus anterior
Winging of the scapula is most commonly the result of long thoracic nerve injury or dysfunction. Iatrogenic damage during the course of the difficult axillary dissection is the most likely cause in this scenario. Damage to the rhomboids may produce winging of the scapula but would be rare in the scenario given.
Long thoracic nerve
Derived from ventral rami of C5, C6, and C7 (close to their emergence from intervertebral foramina)
It runs downward and passes either anterior or posterior to the middle scalene muscle
It reaches upper tip of serratus anterior muscle and descends on outer surface of this muscle, giving branches into it
Winging of Scapula occurs in long thoracic nerve injury (most common) or from spinal accessory nerve injury (which denervates the trapezius) or a dorsal scapular nerve injury
A 75-year-old man presents with hepatomegaly and ascites. A CT scan shows evidence of post hepatic portal hypertension. The inferior vena cava passes through the diaphragm at which vertebral level?
T5
T8
T9
T10
T11
It passes through the diaphragm at T8.
Inferior vena cava
Origin
L5
Path
Left and right common iliac veins merge to form the IVC.
Passes right of midline
Paired segmental lumbar veins drain into the IVC throughout its length
The right gonadal vein empties directly into the cava and the left gonadal vein generally empties into the left renal vein.
The next major veins are the renal veins and the hepatic veins
Pierces the central tendon of diaphragm at T8
Right atrium
Relations
Anteriorly Small bowel, first and third part of duodenum, head of pancreas, liver and bile duct, right common iliac artery, right gonadal artery
Posteriorly Right renal artery, right psoas, right sympathetic chain, coeliac ganglion
Levels
Level Vein
T8 Hepatic vein, inferior phrenic vein, pierces diaphragm
L1 Right suprarenal vein, renal vein
L2 Gonadal vein
L1-5 Lumbar veins
L5 Common iliac vein, formation of IVC
A man is undergoing excision of a sub mandibular gland. As the gland is mobilised, a vessel is injured lying between the gland and the mandible. Which of the following is this vessel most likely to be?
Lingual artery
Occipital artery
Superior thyroid artery
Facial artery
External jugular vein
The high salivary viscosity of submandibular gland secretions favors stone formation. Most stones are radio-opaque.
The marginal mandibular nerve is the most superficial structure.
The facial artery lies between the gland and mandible and is often ligated during excision of the gland. The lingual artery may be encountered but this is usually later in the operative process as Whartons duct is mobilised.
Submandibular gland
Relations of the submandibular gland
Superficial Platysma, deep fascia and mandible
Submandibular lymph nodes
Facial vein (facial artery near mandible)
Marginal mandibular nerve
Cervical branch of the facial nerve
Deep Facial artery (inferior to the mandible)
Mylohyoid muscle
Sub mandibular duct
Hyoglossus muscle
Lingual nerve
Submandibular ganglion
Hypoglossal nerve
Submandibular duct (Wharton’s duct)
Opens lateral to the lingual frenulum on the anterior floor of mouth.
5 cm length
Lingual nerve wraps around Wharton’s duct. As the duct passes forwards it crosses medial to the nerve to lie above it and then crosses back, lateral to it, to reach a position below the nerve.
Innervation
Sympathetic innervation- Derived from superior cervical ganglion
Parasympathetic innervation- Submandibular ganglion via lingual nerve
Arterial supply
Branch of the facial artery. The facial artery passes through the gland to groove its deep surface. It then emerges onto the face by passing between the gland and the mandible.
Venous drainage
Anterior facial vein (lies deep to the Marginal Mandibular nerve)
Lymphatic drainage
Deep cervical and jugular chains of nodes
A 44 year old man is stabbed in the back and the left kidney is injured. A haematoma forms, which of the following fascial structures will contain the haematoma?
Waldeyers fascia
Sibsons fascia
Bucks fascia
Gerotas fascia
Denonvilliers fascia
Waldeyers fascia- Posterior ano-rectum
Sibsons fascia- Lung apex
Bucks fascia- Base of penis
Gerotas fascia- Surrounding kidney
Denonvilliers fascia- Between rectum and prostate
Renal anatomy
Each kidney is about 11cm long, 5cm wide and 3cm thick. They are located in a deep gutter alongside the projecting vertebral bodies, on the anterior surface of psoas major. In most cases the left kidney lies approximately 1.5cm higher than the right. The upper pole of both kidneys approximates with the 11th rib (beware pneumothorax during nephrectomy). On the left hand side the hilum is located at the L1 vertebral level and the right kidney at level L1-2. The lower border of the kidneys is usually alongside L3.
The table below shows the anatomical relations of the kidneys:
Relations
Relations Right Kidney Left Kidney
Posterior Quadratus lumborum, diaphragm, psoas major, transversus abdominis Quadratus lumborum, diaphragm, psoas major, transversus abdominis
Anterior Hepatic flexure of colon Stomach, Pancreatic tail
Superior Liver, adrenal gland Spleen, adrenal gland
Fascial covering
Each kidney and suprarenal gland is enclosed within a common layer of investing fascia, derived from the transversalis fascia. It is divided into anterior and posterior layers (Gerotas fascia).
Renal structure
Kidneys are surrounded by an outer cortex and an inner medulla which usually contains between 6 and 10 pyramidal structures. The papilla marks the innermost apex of these. They terminate at the renal pelvis, into the ureter.
Lying in a hollow within the kidney is the renal sinus. This contains:
1. Branches of the renal artery
2. Tributaries of the renal vein
3. Major and minor calyces’s
4. Fat
Structures at the renal hilum
The renal vein lies most anteriorly, then renal artery (it is an end artery) and the ureter lies most posterior.
An 82 year old lady is taken to theatre for a common bile duct exploration. She has a stone impacted at the distal aspect of the common bile duct and despite best efforts it proves impossible to remove it. What is the best course of action?
Close the bile duct over a T Tube and arrange for a stent to be placed
Undertake a choledochoduodenostomy
Arrange for a repeat ERCP
Construct a hepaticojejunostomy
Bypass the gallbladder onto the jejunum
If a stone cannot be removed at surgery then the chances of succeeding at ERCP are slim. In this case, its probably best to bypass the distal bile duct and a choledochoduodenostomy is the best way of achieving this. There are long term risks of cholangitis which are less of a concern in older patients.
Biliary disease
Diagnosis Typical features Pathogenesis
Gallstones Typically history of biliary colic or episodes of chlolecystitis. Obstructive type history and test results. Usually small calibre gallstones which can pass through the cystic duct. In Mirizzi syndrome the stone may compress the bile duct directly- one of the rare times that cholecystitis may present with jaundice
Cholangitis Usually obstructive and will have Charcot’s triad of symptoms (pain, fever, jaundice) Ascending infection of the bile ducts usually by E. coli and by definition occurring in a pool of stagnant bile.
Pancreatic cancer Typically painless jaundice with palpable gallbladder (Courvoisier’s Law) Direct occlusion of distal bile duct or pancreatic duct by tumour. Sometimes nodal disease at the portal hepatis may be the culprit in which case the bile duct may be of normal calibre.
TPN (total parenteral nutrition) associated jaundice Usually follows long term use and is usually painless with non obstructive features Often due to hepatic dysfunction and fatty liver which may occur with long term TPN usage.
Bile duct injury Depending upon the type of injury may be of sudden or gradual onset and is usually of obstructive type Often due to a difficult laparoscopic cholecystectomy when anatomy in Calots triangle is not appreciated. In the worst scenario the bile duct is excised and jaundice develops rapidly post operatively. More insidious is that of bile duct stenosis which may be caused by clips or diathermy injury.
Cholangiocarcinoma Gradual onset obstructive pattern Direct occlusion by disease and also extrinsic compression by nodal disease at the porta hepatis.
Septic surgical patient Usually hepatic features Combination of impaired biliary excretion and drugs such as ciprofloxacin which may cause cholestasis.
Metastatic disease Mixed hepatic and post hepatic Combination of liver synthetic failure (late) and extrinsic compression by nodal disease and anatomical compression of intra hepatic structures (earlier)
A gallbladder may develop a thickened wall in chronic cholecystitis, microscopically Roikitansky-Aschoff Sinuses may be seen
A 72 year old man with carcinoma of the lung is undergoing a left pneumonectomy. The left main bronchus is divided. Which of the following thoracic vertebrae lies posterior to this structure?
T3
T7
T6
T10
T1
The left main bronchus lies at T6. Topographical anatomy of the thorax is important as it helps surgeons to predict the likely structures to be injured in trauma scenarios (so popular with examiners)
Lung anatomy
The right lung is composed of 3 lobes divided by the oblique and transverse fissures. The left lung has two lobes divided by the oblique fissure.The apex of both lungs is approximately 4cm superior to the sterno-costal joint of the first rib. Immediately below this is a sulcus created by the subclavian artery.
Peripheral contact points of the lung
Base: diaphragm
Costal surface: corresponds to the cavity of the chest
Mediastinal surface: Contacts the mediastinal pleura. Has the cardiac impression. Above and behind this concavity is a triangular depression named the hilum, where the structures which form the root of the lung enter and leave the viscus. These structures are invested by pleura, which, below the hilum and behind the pericardial impression, forms the pulmonary ligament
Right lung
Above the hilum is the azygos vein; Superior to this is the groove for the superior vena cava and right innominate vein; behind this, and nearer the apex, is a furrow for the innominate artery. Behind the hilum and the attachment of the pulmonary ligament is a vertical groove for the oesophagus; In front and to the right of the lower part of the oesophageal groove is a deep concavity for the extrapericardiac portion of the inferior vena cava.
The root of the right lung lies behind the superior vena cava and the right atrium, and below the azygos vein.
The right main bronchus is shorter, wider and more vertical than the left main bronchus and therefore the route taken by most foreign bodies.
Left lung
Above the hilum is the furrow produced by the aortic arch, and then superiorly the groove accommodating the left subclavian artery; Behind the hilum and pulmonary ligament is a vertical groove produced by the descending aorta, and in front of this, near the base of the lung, is the lower part of the oesophagus.
The root of the left lung passes under the aortic arch and in front of the descending aorta.
Inferior borders of both lungs
6th rib in mid clavicular line
8th rib in mid axillary line
10th rib posteriorly
The pleura runs two ribs lower than the corresponding lung level.
Bronchopulmonary segments
Segment number Right lung Left lung
1 Apical Apical
2 Posterior Posterior
3 Anterior Anterior
4 Lateral Superior lingular
5 Medial Inferior lingular
6 Superior (apical) Superior (apical)
7 Medial basal Medial basal
8 Anterior basal Anterior basal
9 Lateral basal Lateral basal
10 Posterior basal Posterior basal
A 19 year old man undergoes an open inguinal hernia repair. The cord is mobilised and the deep inguinal ring identified. Which of the following structures forms its lateral wall?
External oblique aponeurosis
Transversalis fascia
Conjoint tendon
Inferior epigastric artery
Inferior epigastric vein
The transversalis fascia forms the superolateral edge of the deep inguinal ring. The epigastric vessels form its inferomedial wall.
Inguinal canal
Location
Above the inguinal ligament
The inguinal canal is 4cm long
The superficial ring is located anterior to the pubic tubercle
The deep ring is located approximately 1.5-2cm above the half way point between the anterior superior iliac spine and the pubic tubercle
Boundaries of the inguinal canal
Floor
External oblique aponeurosis
Inguinal ligament
Lacunar ligament
Roof
Internal oblique
Transversus abdominis
Anterior wall External oblique aponeurosis
Posterior wall
Transversalis fascia
Conjoint tendon
Laterally
Internal ring
Transversalis fascia
Fibres of internal oblique
Medially
External ring
Conjoint tendon
Contents
Males Spermatic cord and ilioinguinal nerve As it passes through the canal the spermatic cord has 3 coverings:
External spermatic fascia from external oblique aponeurosis
Cremasteric fascia
Internal spermatic fascia
Females Round ligament of uterus and ilioinguinal nerve
A 55 year old lady presents with discomfort in the right breast. On clinical examination a small lesion is identified and clinical appearances suggest fibroadenoma. Imaging confirms the presence of a fibroadenoma alone. A core biopsy is taken, this confirms the presence of the fibroadenoma. However, the pathologist notices that a small area of lobular carcinoma in situ is also present in the biopsy. What is the most appropriate course of action?
Whole breast irradiation
Simple mastectomy
Mastectomy and sentinal lymph node biopsy
Wide local excision and sentinel lymph node biopsy
Breast MRI scan
Lobular carcinoma in situ has a low association with invasive malignancy. It is seldom associated with microcalcification and therefore MRI is the best tool for determining disease extent. Resection of in situ disease is not generally recommended and most surgeons would simply pursue a policy of close clinical and radiological follow up.
Lobular carcinoma of the breast
Lobular breast cancers are less common than their ductal counterparts. They typically present differently, the mass is usually more diffuse and less obvious on the usual imaging modalities of ultrasound and mammography. This is significant since the disease may be understaged resulting in inadequate treatment when wide local excision is undertaken.
In women with invasive lobular carcinoma it is usually safest to perform an MRI scan of the breast, if breast conserving surgery is planned.
Lobular carcinomas are also more likely to be multifocal and metastasise to the contralateral breast.
Lobular carcinoma in situ is occasionally diagnosed incidentally on core biopsies. Unlike DCIS, lobular carcinoma in situ is far less strongly associated with foci of invasion and is usually managed by close monitoring.
Administration of which of the following may facilitate the identification of parathyroid glands intra operatively?
Patent V dye intravenously
Methylene blue intravenously
Indigocarmine dye intravenously
India ink intravenously
Intravenous rifampicin
Methylene blue stains the parathyroid glands and can be useful in facilitating their identification.
Preparation for surgery
Elective and emergency patients require different preparation.
Elective cases
Consider pre admission clinic to address medical issues.
Blood tests including FBC, U+E, LFT’s, Clotting, Group and Save
Urine analysis
Pregnancy test
Sickle cell test
ECG/ Chest x-ray
Exact tests to be performed will depend upon the proposed procedure and patient fitness.
Risk factors for development of deep vein thrombosis should be assessed and a plan for thromboprophylaxis formulated.
Diabetes
Diabetic patients have greater risk of complications.
Poorly controlled diabetes carries high risk of wound infections.
Patients with diet or tablet controlled diabetes may be managed using a policy of omitting medication and checking blood glucose levels regularly. Diabetics who are poorly controlled or who take insulin may require a intravenous sliding scale. Potassium supplementation should also be given.
Diabetic cases should be operated on first.
Emergency cases
Stabilise and resuscitate where needed.
Consider whether antibiotics are needed and when and how they should be administered.
Inform blood bank if major procedures planned particularly where coagulopathies are present at the outset or anticipated (e.g. Ruptured AAA repair)
Don’t forget to consent and inform relatives.
Special preparation
Some procedures require special preparation:
Thyroid surgery; vocal cord check.
Parathyroid surgery; consider methylene blue to identify gland.
Sentinel node biopsy; radioactive marker/ patent blue dye.
Surgery involving the thoracic duct; consider administration of cream.
Pheochromocytoma surgery; will need alpha and beta blockade.
Surgery for carcinoid tumours; will need covering with octreotide.
Colorectal cases; bowel preparation (especially left sided surgery)
Thyrotoxicosis; lugols iodine/ medical therapy.
A 30 year old man is suspected of having appendicitis. At operation an inflamed Meckels diverticulum is found. Which of the following vessels is responsible for the blood supply to a Meckels diverticulum?
Right colic artery
Vitelline artery
Appendicular artery
Internal iliac artery
External iliac artery
The vitelline arteries supply a Meckels these are usually derived from the ileal arcades.
Meckel’s diverticulum
- Congenital abnormality resulting in incomplete obliteration of the vitello-intestinal duct
Normally, in the foetus, there is an attachment between the vitello-intestinal duct and the yolk sac.This disappears at 6 weeks gestation.
The tip is free in majority of cases.
Associated with enterocystomas, umbilical sinuses, and omphaloileal fistulas.
Arterial supply: omphalomesenteric artery.
2% of population, 2 inches long, 2 feet from the ileocaecal valve.
Typically lined by ileal mucosa but ectopic gastric mucosa can occur, with the risk of peptic ulceration. Pancreatic and jejunal mucosa can also occur.
Clinical
Normally asymptomatic and an incidental finding.
Complications are the result of obstruction, ectopic tissue, or inflammation.
Removal if narrow neck or symptomatic. Options are between wedge excision or formal small bowel resection and anastomosis.
Which one of the following is least associated with Tetralogy of Fallot?
Right ventricular outflow tract obstruction
Overriding aorta
Pan systolic murmur
Left-to-right shunt
Right ventricular hypertrophy
Right-to-left shunting is characteristic of Fallot’s. In some patients there can be bidirectional shunting (if there is mild pulmonary stenosis) and a few patients can even have pink tetralogy when there is a predominant shunt from left to right due to minimal infundibular stenosis.
Tetralogy of Fallot
Tetralogy of Fallot (TOF) is the most common cause of cyanotic congenital heart disease*. It typically presents at around 1-2 months, although may not be picked up until the baby is 6 months old
The four characteristic features are:
ventricular septal defect (VSD)
right ventricular hypertrophy
right ventricular outflow tract obstruction, pulmonary stenosis
overriding aorta
The severity of the right ventricular outflow tract obstruction determines the degree of cyanosis and clinical severity
Other features
cyanosis
causes a right-to-left shunt
ejection systolic murmur due to pulmonary stenosis (the VSD doesn’t usually cause a murmur)
a right-sided aortic arch is seen in 25% of patients
chest x-ray shows a ‘boot-shaped’ heart, ECG shows right ventricular hypertrophy
Management
surgical repair is often undertaken in two parts
cyanotic episodes may be helped by beta-blockers to reduce infundibular spasm
*however, at birth transposition of the great arteries is the more common lesion as patients with TOF generally present at around 1-2 months
A 48 year old lady undergoes an ERCP for jaundice. 36 hours following the procedure she develops a fever and rigors. A blood culture is taken, which of the following organisms is most likely to be cultured?
Pseudomonas aeruginosa
Streptococcus
Enterobacter
Staphylococcus
Escherichia coli
Charcots triad = Surgical emergency.
Patients need: Biliary decompression and broad spectrum antibiotics.
E Coli is the most common organism implicated in cholangitis infections. Whilst enterobacter is occasionally cultured it is not the most common organism.
Cholangitis
- Combination of bacterial infection and biliary obstruction
Most common organisms are: (most frequent at top of list)
Escherichia coli
Klebsiella species
Enterococcus species
Streptococcus species
Clinical features
Charcot’s triad:
Fever (90% cases)
Right upper quadrant pain
Jaundice
Reynolds pentad: Above plus confusion and hypotension
Investigations
USS 1st line
CT scan
ERCP: may be 1st line if high clinical suspicion and suitable for treatment
Treatment
ERCP -usually after 72 hours of antibiotics
Percutaneous transhepatic cholangiogram and biliary drain
Which of the following anatomical structures lies within the spiral groove of the humerus?
Median nerve
Radial nerve
Tendon of triceps
Musculocutaneous nerve
Axillary nerve
The radial nerve lies in this groove and may be compromised by fractures involving the shaft.
Humerus
The humerus extends from the scapula to the elbow joint. It has a body and two ends. It is almost completely covered with muscle but can usually be palpated throughout its length. The smooth rounded surface of the head articulates with the shallow glenoid cavity. The head is connected to the body of the humerus by the anatomical neck. The surgical neck is the region below the head and tubercles and where they join the shaft and is the commonest site of fracture. The capsule of the shoulder joint is attached to the anatomical neck superiorly but extends down to 1.5cm on the surgical neck.
The greater tubercle is the prominence on the lateral side of the upper end of the bone. It merges with the body below and can be felt through the deltoid inferior to the acromion. The tendons of the supraspinatus and infraspinatus are inserted into impressions on its superior aspect. The lesser tubercle is a distinct prominence on the front of the upper end of the bone. It can be palpated through the deltoid just lateral to the tip of the coracoid process.
The intertubercular groove passes on the body between the greater and lesser tubercles, continuing down from the anterior borders of the tubercles to form the edges of the groove. The tendon of biceps within its synovial sheath passes through this groove, held within it by a transverse ligament.
The posterior surface of the body is marked by a spiral groove for the radial nerve which runs obliquely across the upper half of the body to reach the lateral border below the deltoid tuberosity. Within this groove lie the radial nerve and brachial vessels and both may be affected by fractures involving the shaft of the humerus.
The lower end of the humerus is wide and flattened anteroposteriorly, and inclined anteriorly. The middle third of the distal edge forms the trochlea. Superior to this are indentations for the coronoid fossa anteriorly and olecranon fossa posteriorly. Lateral to the trochlea is a rounded capitulum which articulates with the radius.
The medial epicondyle is very prominent with a smooth posterior surface which contains a sulcus for the ulnar nerve and collateral vessels. It’s distal margin gives attachment for the ulnar collateral ligament and, in front of this, the anterior surface has an impression for the common flexor tendon.
A 72 year old man presents with lower urinary tract symptoms. On digital rectal examination, benign prostatic hyperplasia is suspected. Which of the following treatments is associated with a reduction in the risk of urinary retention?
Alfuzosin
Finasteride
Prazosin
Tamsulosin
Terazosin
5 alpha reductase inhibitors reduce the risk of urinary retention.
In the PLESS study, data show a reduction in the risk of urinary retention although the absolute risk reduction was small.
Finasteride is a medication used to treat benign prostatic hyperplasia (BPH) by reducing the size of the prostate gland. It works by inhibiting the conversion of testosterone to dihydrotestosterone (DHT), which is a hormone that contributes to the growth of the prostate gland. Alfuzosin, tamsulosin, and terazosin are medications that belong to a class of drugs called alpha-blockers. These medications work by relaxing the smooth muscle in the prostate gland and bladder neck, which can improve urine flow and decrease urinary symptoms. While alpha-blockers are effective in improving urinary symptoms associated with BPH, they do not reduce the size of the prostate gland and are not associated with a reduction in the risk of urinary retention. Prazosin is an alpha-blocker that is used to treat high blood pressure and post-traumatic stress disorder (PTSD). It is not typically used to treat urinary symptoms associated with BPH
Reference
McConnell J et al. The effect of finasteride on the risk of urinary retention and the need for surgical intervention amongst men with benign prostatic hyperplasia. N Engl J Med 338:557-563
A 56 year old man is day one following a laparoscopic appendicectomy. He suddenly develops marked dyspnoea following administration of cyclizine to relieve post operative nausea. On arrival, the patient is breathless with a pulse rate of 122, Bp 86/48, oxygen saturations on air are 85%. Which of these agents should be administered initially?
Adrenaline
Chlorpheniramine
IV 0.9% saline
Hydrocortisone
Salbutamol
As this patient most likely has anaphylaxis, administration of adrenaline is most appropriate. Anaphylactic shock
- Suspect if there has been exposure to an allergen
Management
- Remove allergen
- ABCD
- Drugs:
Adrenaline 1:1000 0.5ml INTRAMUSCULARLY (not IV). Repeat after 5 mins if no response.
Consider antihistamine if ongoing local symptoms.
In the refractory anaphylaxis guidance, consideration is given to adrenaline infusions.
Reference
Emergency treatment of anaphylactic reactions. Guidelines for healthcare providers. Working Group of the Resuscitation Council (UK).2021
Which of the following is not a feature of a Charcot foot?
Bounding foot pulses in the early phases
Often occurs in the complete absence of trauma
Erythema of the foot in the early phase
Autonomic neuropathy
Peripheral neuropathy
Do not confuse the early phase of Charcot foot with cellulitis
Trauma (even if only minor) is a prerequisite. Patients cannot usually recall the traumatic event. The associated neuropathy means that patients continue to walk on the affected foot with subsequent deformity developing over time.
Peripheral vascular disease
Indications for surgery to revascularise the lower limb
Intermittent claudication
Critical ischaemia
Ulceration
Gangrene
Intermittent claudication that is not disabling may provide a relative indication, whilst the other complaints are often absolute indications depending upon the frailty of the patient.
Assessment
Clinical examination
Ankle brachial pressure index measurement
Duplex arterial ultrasound
Angiography (standard, CT or MRI): usually performed only if intervention being considered.
Angioplasty
In order for angioplasty to be undertaken successfully the artery has to be accessible. The lesion relatively short and reasonable distal vessel runoff. Longer lesions may be amenable to sub-intimal angioplasty.
Surgery
Surgery will be undertaken where attempts at angioplasty have either failed or are unsuitable. Bypass essentially involves bypassing the affected arterial segment by utilising a graft to run from above the disease to below the disease. As with angioplasty good runoff improves the outcome.
Some key concepts with bypass surgery
Superficial femoral artery occlusion to the above knee popliteal
In the ideal scenario, vein (either in situ or reversed LSV) would be used as a conduit. However, prosthetic material has reasonable 5 year patency rates and some would advocate using this in preference to vein so that vein can be used for other procedures in the future. In general terms either technique is usually associated with an excellent outcome (if run off satisfactory).
Procedure
Artery dissected out, IV heparin 3,000 units given and then the vessels are cross clamped
Longitudinal arteriotomy
Graft cut to size and tunneled to arteriotomy sites
Anastomosis to femoral artery usually with 5/0 ‘double ended’ Prolene suture
Distal anastomosis usually using 6/0 ‘double ended’ Prolene
Distal disease
Femoro-distal bypass surgery takes longer to perform, is more technically challenging and has higher failure rates.
In elderly diabetic patients with poor runoff a primary amputation may well be a safer and more effective option. There is no point in embarking on this type of surgery in patients who are wheelchair bound.
In femorodistal bypasses vein gives superior outcomes to PTFE.
Rules
Vein mapping 1st to see whether there is suitable vein (the preferred conduit). Sub intimal hyperplasia occurs early when PTFE is used for the distal anastomosis and will lead to early graft occlusion and failure.
Essential operative procedure as for above knee fem-pop.
If there is insufficient vein for the entire conduit then vein can be attached to the end of the PTFE graft and then used for the distal anastomosis. This type of ‘vein boot’ is technically referred to as a Miller Cuff and is associated with better patency rates than PTFE alone.
Remember the more distal the arterial anastomosis the lower the success rate.
References
Peach G et al. Diagnosis and management of peripheral arterial disease. BMJ 2012; 345: 36-41.
What is the main impact of the Montgomery ruling in consenting patients for surgical procedures?
It clearly states that all risks however minor with a frequency of 10% or greater be disclosed
It states whether a reasonable person in the patients position would be likely to attach significance to the risk, or the doctor is or should reasonably be aware that the particular patient would be likely to attach significance to it
It states all procedures should be performed by a consultant
It covers the decision making for minors who need to be consented by proxy
It replaces the Gillick competency test
The new legal judgement recognises this individual approach to warning patients about risk. Rather than taking into account the percentage possibility of a risk arising, doctors need to bear in mind the significance of a given risk for that particular patient, and the nature of the risk, such as the effect it would have on the patient’s life if it were to occur. The assessment is therefore considered to be both fact-sensitive and sensitive to the characteristics of the particular patient.
Consent
There are 3 types of consent:
- Informed
- Expressed
- Implied
Consent forms used in UK NHS
Consent Form 1 For competent adults who are able to consent for themselves where consciousness may be impaired (e.g. GA)
Consent Form 2 For an adult consenting on behalf of a child where consciousness is impaired
Consent Form 3 For an adult or child where consciousness is not impaired
Consent Form 4 For adults who lack capacity to provide informed consent
Capacity
Key points include:
1. Understand and retain information
2. Patient believes the information to be true
3. Patient is able to weigh the information to make a decision
All patients must be assumed to have capacity
Consent in minors
Young children and older children who are not Gillick competent cannot consent for themselves. In British law the patients biological mother can always provide consent. The child’s father can consent if the parents are married (and the father is the biological father), or if the father is named on the birth certificate (irrespective of marital status). If parents are not married and the father is not named on the birth certificate then the father cannot consent.
A 56 year old man is admitted with passage of a large volume of blood per rectum. On examination, he is tachycardic, his abdomen is soft, although he has marked dilated veins on his abdominal wall. Proctoscopy reveals large dilated veins with stigmata of recent haemorrhage. What is the most appropriate treatment?
IV terlipressin
Excisional haemorrhoidectomy
Injection sclerotherapy
Proctectomy
Rectal pack insertion
Rectal varices are a recognised complication of portal hypertension. In the first instance they can be managed with medical therapy to lower pressure in the portal venous system. TIPSS may be considered. Whilst band ligation is an option, attempting to inject these in same way as haemorroids would carry a high risk of precipitating further haemorrhage.
Lower Gastrointestinal bleeding
Colonic bleeding
This typically presents as bright red or dark red blood per rectum. Colonic bleeding rarely presents as malaena type stool, this is because blood in the colon has a powerful laxative effect and is rarely retained long enough for transformation to occur and because the digestive enzymes present in the small bowel are not present in the colon. Up to 15% of patients presenting with haemochezia will have an upper gastrointestinal source of haemorrhage.
As a general rule right sided bleeds tend to present with darker coloured blood than left sided bleeds. Haemorrhoidal bleeding typically presents as bright red rectal bleeding that occurs post defecation either onto toilet paper or into the toilet pan. It is very unusual for haemorrhoids alone to cause any degree of haemodynamic compromise.
Causes
Cause Presenting features
Colitis Bleeding may be brisk in advanced cases, diarrhoea is commonly present. Abdominal x-ray may show featureless colon.
Diverticular disease Acute diverticulitis often is not complicated by major bleeding and diverticular bleeds often occur sporadically. 75% all will cease spontaneously within 24-48 hours. Bleeding is often dark and of large volume.
Cancer Colonic cancers often bleed and for many patients this may be the first sign of the disease. Major bleeding from early lesions is uncommon
Haemorrhoidal bleeding Typically bright red bleeding occurring post defecation. Although patients may give graphic descriptions bleeding of sufficient volume to cause haemodynamic compromise is rare.
Angiodysplasia Apart from bleeding, which may be massive, these arteriovenous lesions cause little in the way of symptoms. The right side of the colon is more commonly affected.
Management
Prompt correction of any haemodynamic compromise is required. Unlike upper gastrointestinal bleeding the first line management is usually supportive. This is because in the acute setting endoscopy is rarely helpful.
When haemorrhoidal bleeding is suspected a proctosigmoidoscopy is reasonable as attempts at full colonoscopy are usually time consuming and often futile.
In the unstable patient the usual procedure would be an angiogram (either CT or percutaneous), when these are performed during a period of haemodynamic instability they may show a bleeding point and may be the only way of identifying a patch of angiodysplasia.
In others who are more stable the standard procedure would be a colonoscopy in the elective setting. In patients undergoing angiography attempts can be made to address the lesion in question such as coiling. Otherwise surgery will be necessary.
In patients with ulcerative colitis who have significant haemorrhage the standard approach would be a sub total colectomy, particularly if medical management has already been tried and is not effective.
Indications for surgery
Patients > 60 years
Continued bleeding despite endoscopic intervention
Recurrent bleeding
Known cardiovascular disease with poor response to hypotension
Surgery
Selective mesenteric embolisation if life threatening bleeding. This is most helpful if conducted during a period of relative haemodynamic instability. If all haemodynamic parameters are normal then the bleeding is most likely to have stopped and any angiography normal in appearance. In many units a CT angiogram will replace selective angiography but the same caveats will apply.
If the source of colonic bleeding is unclear; perform a laparotomy, on table colonic lavage and following this attempt a resection. A blind sub total colectomy is most unwise, for example bleeding from an small bowel arterio-venous malformation will not be treated by this manoeuvre.
Summary of Acute Lower GI bleeding recommendations
Consider admission if:
* Over 60 years
* Haemodynamically unstable/profuse PR bleeding
* On aspirin or NSAID
* Significant co morbidity
Management
All patients should have a history and examination, PR and proctoscopy
Colonoscopic haemostasis aimed for in post polypectomy or diverticular bleeding
References
http://www.sign.ac.uk/guidelines/fulltext/105/index.html
The oxygen-haemoglobin dissociation curve is shifted to the right in which of the following scenarios?
Hypothermia
Respiratory alkalosis
Low altitude
Decreased 2,3-DPG in transfused red cells
Chronic iron deficiency anaemia
Chronic iron deficiency anemia
Mnemonic to remember causes of right shift of the oxygen dissociation curve:
CADET face RIGHT
C O2
A cidosis
2,3-DPG
E xercise
T emperature
The curve is shifted to the right when there is an increased oxygen requirement by the tissue. This includes:
Increased temperature
Acidosis
Increased DPG:
DPG is found in erythrocytes and is increased during glycolysis. It binds to the Hb molecule, thereby releasing oxygen to tissues. DPG is increased in conditions associated with poor oxygen delivery to tissues, such as anaemia and high altitude.
A 45 year old lady presents with a pathological fracture of her femoral shaft. She is a poor historian, but it transpires that she underwent a thyroidectomy 1 year previously. She has no other illness or co-morbidities. What is the most likely underlying diagnosis?
Hyperparathyroidism
Metastatic papillary carcinoma of the thyroid
Metastatic medullary carcinoma of the thyroid
Metastatic follicular carcinoma of the thyroid
None of the above
Follicular carcinomas are a recognised cause of bone metastasis. Papillary lesions typically spread via the lymphatics.
Thyroid malignancy
Papillary carcinoma
Commonest sub-type
Accurately diagnosed on fine needle aspiration cytology
Histologically, they may demonstrate psammoma bodies (areas of calcification) and so called ‘orphan Annie’ nuclei
They typically metastasise via the lymphatics and thus laterally located apparently ectopic thyroid tissue is usually a metastasis from a well differentiated papillary carcinoma
Follicular carcinoma
Are less common than papillary lesions
Like papillary tumours, they may present as a discrete nodule. Although they appear to be well encapsulated macroscopically there is invasion on microscopic evaluation
Lymph node metastases are uncommon and these tumours tend to spread haematogenously. This translates into a higher mortality rate
Follicular lesions cannot be accurately diagnosed on fine needle aspiration cytology and thus all follicular FNA’s (THY 3f) will require at least a hemi thyroidectomy
Anaplastic carcinoma
Less common and tend to occur in elderly females
Disease is usually advanced at presentation and often only palliative decompression and radiotherapy can be offered.
Medullary carcinoma
These are tumours of the parafollicular cells ( C Cells) and are of neural crest origin.
The serum calcitonin may be elevated which is of use when monitoring for recurrence.
They may be familial and occur as part of the MEN -2A disease spectrum.
Spread may be either lymphatic or haematogenous and as these tumours are not derived primarily from thyroid cells they are not responsive to radioiodine.
Lymphoma
These respond well to combined chemoradiotherapy
Radical surgery is unnecessary once the disease has been diagnosed on biopsy material. Such biopsy material is not generated by an FNA and thus a core biopsy has to be obtained (with care!).
A 17 year old male has a suspected testicular torsion and the scrotum is to be explored surgically. The surgeon incises the skin and then the dartos muscle. What is the next tissue layer that will be encountered during the dissection?
Visceral layer of the tunica vaginalis
Cremasteric fascia
Parietal layer of the tunica vaginalis
External spermatic fascia
Internal spermatic fascia
The layers that will be encountered are (in order):
1. Skin
2. Dartos fascia and muscle
3. External spermatic fascia
4. Cremasteric muscle and fascia
5. Internal spermatic fascia
6. Parietal layer of the tunica vaginalis
The layers of the spermatic cord and scrotum are a popular topic in the MRCS exam.
A mnemonic which may help:
Some Damned Examiner Called It The Testes (skin dartos external fascia cremaster internal fascia tunica Testes)
Scrotal and testicular anatomy
Spermatic cord
Formed by the vas deferens and is covered by the following structures:
Layer Origin
Internal spermatic fascia Transversalis fascia
Cremasteric fascia From the fascial coverings of internal oblique
External spermatic fascia External oblique aponeurosis
Contents of the cord
Vas deferens Transmits sperm and accessory gland secretions
Testicular artery Branch of abdominal aorta supplies testis and epididymis
Artery of vas deferens Arises from inferior vesical artery
Cremasteric artery Arises from inferior epigastric artery
Pampiniform plexus Venous plexus, drains into right or left testicular vein
Sympathetic nerve fibres Lie on arteries, the parasympathetic fibres lie on the vas
Genital branch of the genitofemoral nerve Supplies cremaster
Lymphatic vessels Drain to lumbar and para-aortic nodes
Scrotum
Composed of skin and closely attached dartos fascia.
Arterial supply from the anterior and posterior scrotal arteries
Lymphatic drainage to the inguinal lymph nodes
Parietal layer of the tunica vaginalis is the innermost layer
Testes
The testes are surrounded by the tunica vaginalis (closed peritoneal sac). The parietal layer of the tunica vaginalis adjacent to the internal spermatic fascia.
The testicular arteries arise from the aorta immediately inferiorly to the renal arteries.
The pampiniform plexus drains into the testicular veins, the left drains into the left renal vein and the right into the inferior vena cava.
Lymphatic drainage is to the para-aortic nodes.
A 46 year old lady presents with symptoms of diarrhoea, weight loss of 10 Kg and a skin rash of erythematous blisters involving the abdomen and buttocks. The blisters have an irregular border and both intact and ruptured vesicles. What is the most likely diagnosis?
Colonic adenocarcinoma
Pancreatic adenocarcinoma
Tropical sprue
Glucagonoma
Insulinoma
Glucagonoma is strongly associated with necrolytic migratory erythema.
Glucagonoma
Rare pancreatic tumours arising from the alpha cells of the pancreas.
Glucagon levels markedly elevated.
Symptoms include diarrhoea, weight loss and necrolytic migratory erythema.
A serum level of glucagon >1000pg/ml usually suggests the diagnosis, imaging with CT scanning is also required.
Treatment is with surgical resection. However, careful staging is required for these tumours are usually malignant and non resectable.
A 63 year old man with end stage osteoarthritis of the hip is due to undergo a total hip replacement. The skin has been prepared and antibiotics given. What is the single most important modality to reduce the risks of infection?
Laminar flow theatre
Exhaust suits
Skin shaving on the ward
Total body scrubbing of the surgical team
Extended antibiotic chemoprophylaxis as routine
A laminar flow is the single most important intervention, many units will also use exhaust suits but these are less essential. Shaving skin on the ward increases infection rates. Extended chemoprophylaxis increases risks of antibiotic associated diarrhea.
Surgical site infection
- Surgical site infections may occur following a breach in tissue surfaces and allow normal commensals and other pathogens to initiate infection. They are a major cause of morbidity and mortality.
Surgical site infections (SSI) comprise up to 20% of all healthcare associated infections and at least 5% of patients undergoing surgery will develop an SSI as a result.
In many cases the organisms are derived from the patient’s own body. Measures that may increase the risk of SSI include:
Shaving the wound using a razor (disposable clipper preferred)
Using a non iodine impregnated incise drape if one is deemed to be necessary
Tissue hypoxia
Delayed administration of prophylactic antibiotics in tourniquet surgery
Preoperatively
Don’t remove body hair routinely
If hair needs removal, use electrical clippers with single use head (razors increase infection risk)
Antibiotic prophylaxis if:
- placement of prosthesis or valve
- clean-contaminated surgery
- contaminated surgery
Use local formulary
Aim to give single dose IV antibiotic on anaesthesia
If a tourniquet is to be used, give prophylactic antibiotics earlier
Intraoperatively
Prepare the skin with alcoholic chlorhexidine (Lowest incidence of SSI)
Cover surgical site with dressing
A recent meta analysis has confirmed that administration of supplementary oxygen does not reduce the risk of wound infection. In contrast to previous individual RCT’s(1)
Wound edge protectors do not appear to confer benefit (2)
Post operatively
Tissue viability advice for management of surgical wounds healing by secondary intention
Use of diathermy for skin incisions
In the NICE guidelines the use of diathermy for skin incisions is not advocated(3). Several randomised controlled trials have been undertaken and demonstrated no increase in risk of SSI when diathermy is used(4).
References
1. Brar M et al.. Perioperative supplemental oxygen in colorectal patients: a meta analysis. J Surg Res 2011 (166): 227 -235.
2. Pinkney T et al. Impact of wound edge protection devices on surgical site infection after laparotomy: impact of a multicentre randomised controlled trial (ROSSINI Trial). BMJ 2013 (347):10.
3. http://www.nice.org.uk/CG74
4. Ahmad N and Ahmed A. Meta-analysis of the effectiveness of surgical scalpel or diathermy in making abdominal skin incisions. Ann Surg 2011, 253(1):8-13.
Which of the following pairings of foramina and their contents is not correct?
Superior orbital fissure and the oculomotor nerve
Foramina rotundum and the maxillary nerve
Jugular foramen and the hypoglossal nerve
Foramina spinosum and the middle meningeal artery
Carotid canal and the internal carotid artery
The hypoglossal nerve passes through the hypoglossal canal.
Foramina of the base of the skull
Foramen Location Contents
Foramen ovale Sphenoid bone Otic ganglion
V3 (Mandibular nerve:3rd branch of
trigeminal)
Accessory meningeal artery
Lesser petrosal nerve
Emissary veins
Foramen spinosum Sphenoid bone Middle meningeal artery
Meningeal branch of the Mandibular nerve
Foramen rotundum Sphenoid bone Maxillary nerve (V2)
Foramen lacerum/ carotid canal Located between the sphenoid, the apex of the petrous temporal and the basilar part of the occipital Base of the medial pterygoid plate.
Internal carotid artery*
Nerve and artery of the pterygoid canal
Jugular foramen Temporal bone Anterior: inferior petrosal sinus
Intermediate: glossopharyngeal, vagus, and accessory nerves.
Posterior: sigmoid sinus (becoming the internal jugular vein) and some meningeal branches from the occipital and ascending pharyngeal arteries.
Foramen magnum Occipital bone Anterior and posterior spinal arteries
Vertebral arteries
Medulla oblongata
Stylomastoid foramen Temporal bone Stylomastoid artery
Facial nerve
Superior orbital fissure Sphenoid bone Oculomotor nerve (III)
Recurrent meningeal artery
Trochlear nerve (IV)
Lacrimal, frontal and nasociliary branches of opthalmic nerve (V1)
Abducent nerve (VI)
Superior ophthalmic vein
*= In life the foramen lacerum is occluded by a cartilagenous plug. The ICA initially passes into the carotid canal which ascends superomedially to enter the cranial cavity through the foramen lacerum.
A 26 year old man presents to the emergency department with a swelling over his left elbow after a fall on an outstretched hand. On examination, he has tenderness over the proximal part of his forearm, and has severely restricted supination and pronation movements. What is the most likely injury?
Fracture of the olecranon
Fracture of the radial head
Galeazzi fracture
Fracture of the shaft of the radius and ulnar
Fracture of the coronoid process
Fracture of the radial head is common in young adults. It is usually caused by a fall on the outstretched hand. On examination, there is marked local tenderness over the head of the radius, impaired movements at the elbow, and a sharp pain at the lateral side of the elbow at the extremes of rotation (pronation and supination).
Upper limb fractures
Colles’ fracture
Fall onto extended outstretched hands
Described as a dinner fork type deformity
Classical Colles’ fractures have the following 3 features:
Features of the injury
1. Transverse fracture of the radius
2. 1 inch proximal to the radio-carpal joint
3. Dorsal displacement and angulation
Smith’s fracture (reverse Colles’ fracture)
Volar angulation of distal radius fragment (Garden spade deformity)
Caused by falling backwards onto the palm of an outstretched hand or falling with wrists flexed
Bennett’s fracture
Intra-articular fracture of the first carpometacarpal joint
Impact on flexed metacarpal, caused by fist fights
X-ray: triangular fragment at ulnar base of metacarpal
Monteggia’s fracture
Dislocation of the proximal radioulnar joint in association with an ulna fracture
Fall on outstretched hand with forced pronation
Needs prompt diagnosis to avoid disability
Galeazzi fracture
Radial shaft fracture with associated dislocation of the distal radioulnar joint
Occur after a fall on the hand with a rotational force superimposed on it.
On examination, there is bruising, swelling and tenderness over the lower end of the forearm.
X Rays reveal the displaced fracture of the radius and a prominent ulnar head due to dislocation of the inferior radio-ulnar joint.
Barton’s fracture
Distal radius fracture (Colles’/Smith’s) with associated radiocarpal dislocation
Fall onto extended and pronated wrist
Scaphoid fractures
Scaphoid fractures are the commonest carpal fractures.
Surface of scaphoid is covered by articular cartilage with small area available for blood vessels (fracture risks blood supply)
Forms floor of anatomical snuffbox
Risk of fracture associated with fall onto outstretched hand (tubercle, waist, or proximal 1/3)
The main physical signs are swelling and tenderness in the anatomical snuff box, and pain on wrist movements and on longitudinal compression of the thumb.
Ulnar deviation AP needed for visualization of scaphoid
Immobilization of scaphoid fractures difficult
Radial head fracture
Fracture of the radial head is common in young adults.
It is usually caused by a fall on the outstretched hand.
On examination, there is marked local tenderness over the head of the radius, impaired movements at the elbow, and a sharp pain at the lateral side of the elbow at the extremes of rotation (pronation and supination).
A 35 year old man is admitted to hospital with vomiting, nausea and severe headaches. An MRI scan reveals a tumour of the cerebellopontine angle. Which one of the following pairs of cranial nerves is most likely to be compressed by this tumour?
Accessory and vagus
Facial and vagus
Facial and vestibulocochlear
Glossopharyngeal and vestibulocochlear
Vagus and vestibulocochlear
The cerebellopontine angle is located between the superior and inferior limbs of the angular cerebellopontine fissure formed by the petrosal cerebellar surface folding around the pons and middle cerebellar peduncle. The cerebellopontine fissure opens medially and has superior and inferior limbs that meet at a lateral apex. The fourth through the eleventh cranial nerves are located near or within the angular space between the two limbs commonly referred to as the cerebellopontine angle. The commonest lesion to affect this site is an acoustic neuroma. Therefore the vestibulocochlear nerve is commonly compromised. Larger lesions may also affect the facial nerve which lies closest to this site.
Cranial nerves
Cranial nerve lesions
Olfactory nerve May be injured in basal skull fractures or involved in frontal lobe tumour extension. Loss of olfactory nerve function in relation to major CNS pathology is seldom an isolated event and thus it is poor localiser of CNS pathology.
Optic nerve Problems with visual acuity may result from intra ocular disorders. Problems with the blood supply such as amaurosis fugax may produce temporary visual distortion. More important surgically is the pupillary response to light. The pupillary size may be altered in a number of disorders. Nerves involved in the resizing of the pupil connect to the pretectal nucleus of the high midbrain, bypassing the lateral geniculate nucleus and the primary visual cortex. From the pretectal nucleus neurones pass to the Edinger - Westphal nucleus, motor axons from here pass along with the oculomotor nerve. They synapse with ciliary ganglion neurones; the parasympathetic axons from this then innervate the iris and produce miosis. The miotic pupil is seen in disorders such as Horner’s syndrome or opiate overdose.
Mydriasis is the dilatation of the pupil in response to disease, trauma, drugs (or the dark!). It is pathological when light fails to induce miosis. The radial muscle is innervated by the sympathetic nervous system. Because the parasympathetic fibres travel with the oculomotor nerve they will be damaged by lesions affecting this nerve (e.g. cranial trauma).
The response to light shone in one eye is usually a constriction of both pupils. This indicates intact direct and consensual light reflexes. When the optic nerve has an afferent defect the light shining on the affected eye will produce a diminished pupillary response in both eyes. Whereas light shone on the unaffected eye will produce a normal pupillary response in both eyes. This is referred to as the Marcus Gunn pupil and is seen in conditions such as optic neuritis. In a total CN II lesion shining the light in the affected eye will produce no response.
Oculomotor nerve The pupillary effects are described above. In addition it supplies all ocular muscles apart from lateral rectus and superior oblique. Thus the affected eye will be deviated inferolaterally. Levator palpebrae superioris may also be impaired resulting in impaired ability to open the eye.
Trochlear nerve The eye will not be able to look down.
Trigeminal nerve Largest cranial nerve. Exits the brainstem at the pons. Branches are ophthalmic, maxillary and mandibular. Only the mandibular branch has both sensory and motor fibres. Branches converge to form the trigeminal ganglion (located in Meckels cave). It supplies the muscles of mastication and also tensor veli palatine, mylohyoid, anterior belly of digastric and tensor tympani. The detailed descriptions of the various sensory functions are described in other areas of the website. The corneal reflex is important and is elicited by applying a small tip of cotton wool to the cornea, a reflex blink should occur if it is intact. It is mediated by: the naso ciliary branch of the ophthalmic branch of the trigeminal (sensory component) and the facial nerve producing the motor response. Lesions of the afferent arc will produce bilateral absent blink and lesions of the efferent arc will result in a unilateral absent blink.
Abducens nerve The affected eye will have a deficit of abduction. This cranial nerve exits the brainstem between the pons and medulla. It thus has a relatively long intra cranial course which renders it susceptible to damage in raised intra cranial pressure.
Facial nerve Emerges from brainstem between pons and medulla. It controls muscles of facial expression and taste from the anterior 2/3 of the tongue. The nerve passes into the petrous temporal bone and into the internal auditory meatus. It then passes through the facial canal and exits at the stylomastoid foramen. It passes through the parotid gland and divides at this point. It does not innervate the parotid gland. Its divisions are considered in other parts of the website. Its motor fibres innervate orbicularis oculi to produce the efferent arm of the corneal reflex. In surgical practice it may be injured during parotid gland surgery or invaded by malignancies of the gland and a lower motor neurone on the ipsilateral side will result.
Vestibulo-cochlear nerve Exits from the pons and then passes through the internal auditory meatus. It is implicated in sensorineural hearing loss. Individuals with sensorineural hearing loss will localise the sound in webers test to the normal ear. Rinnes test will be reduced on the affected side but should still work. These two tests will distinguish sensorineural hearing loss from conductive deafness. In the latter condition webers test will localise to the affected ear and Rinnes test will be impaired on the affected side. Surgical lesions affecting this nerve include CNS tumours and basal skull fractures. It may also be damaged by the administration of ototoxic drugs (of which gentamicin is the most commonly used in surgical practice).
Glossopharyngeal nerve Exits the pons just above the vagus. Receives sensory fibres from posterior 1/3 tongue, tonsils, pharynx and middle ear (otalgia may occur following tonsillectomy). It receives visceral afferents from the carotid bodies. It supplies parasympathetic fibres to the parotid gland via the otic ganglion and motor function to stylopharyngeaus muscle. The sensory function of the nerve is tested using the gag reflex.
Vagus nerve Leaves the medulla between the olivary nucleus and the inferior cerebellar peduncle. Passes through the jugular foramen and into the carotid sheath. Details of the functions of the vagus nerve are covered in the website under relevant organ sub headings.
Accessory nerve Exists from the caudal aspect of the brainstem (multiple branches) supplies trapezius and sternocleidomastoid muscles. The distal portion of this nerve is most prone to injury during surgical procedures.
Hypoglossal nerve Emerges from the medulla at the preolivary sulcus, passes through the hypoglossal canal. It lies on the carotid sheath and passes deep to the posterior belly of digastric to supply muscles of the tongue (except palatoglossus). Its location near the carotid sheath makes it vulnerable during carotid endarterectomy surgery and damage will produce ipsilateral defect in muscle function.
A 50 year old male presents with painless frank haematuria. Clinical examination is unremarkable. Routine blood tests reveal a haemoglobin of 18g/dl but are otherwise normal. What is the most likely underlying diagnosis?
Squamous cell carcinoma of the bladder
Adenocarcinoma of the prostate
Adenocarcinoma of the kidney
Wilms tumour
Transitional cell carcinoma of the renal pelvis
Polycythaemia is a recognised feature of renal cell carcinoma. Wilms tumours most commonly occur in children.
Haematuria
Causes of haematuria
Trauma
Injury to renal tract
Renal trauma commonly due to blunt injury (others penetrating injuries)
Ureter trauma rare: iatrogenic
Bladder trauma: due to RTA or pelvic fractures
Infection
Remember TB
Malignancy
Renal cell carcinoma (remember paraneoplastic syndromes): painful or painless
Urothelial malignancies: 90% are transitional cell carcinoma, can occur anywhere along the urinary tract. Painless haematuria.
Squamous cell carcinoma and adenocarcinoma: rare bladder tumours
Prostate cancer
Penile cancers: SCC
Renal disease
Glomerulonephritis
Stones
Microscopic haematuria common
Structural abnormalities
Benign prostatic hyperplasia (BPH) causes haematuria due to hypervascularity of the prostate gland
Cystic renal lesions e.g. polycystic kidney disease
Vascular malformations
Renal vein thrombosis due to renal cell carcinoma
Coagulopathy
Causes bleeding of underlying lesions
Drugs
Cause tubular necrosis or interstitial nephritis: aminoglycosides, chemotherapy
Interstitial nephritis: penicillin, sulphonamides, and NSAIDs
Anticoagulants
Benign
Exercise
Gynaecological
Endometriosis: flank pain, dysuria, and haematuria that is cyclical
Iatrogenic
Catheterisation
Radiotherapy; cystitis, severe haemorrhage, bladder necrosis
Pseudohaematuria For example following consumption of beetroot
References
Http://bestpractice.bmj.com/best-practice/monograph/316/overview/aetiology.html
A surgical unit are conducting a study to determine whether patients who have bowel preparation have a lower risk of colonic anastomotic leakage than those having none. The planned sample size is 25. Which of the tests below is most appropriate?
Paired T test
Unpaired T test
Fishers exact test
Chi squared test
LSD post hoc test
It is likely to be underpowered with the number provided. However, it would be possible to classify such data into a 2x2 contingency table. However, when the sample size is small, the Chi squared test is not suitable and in these situations the Fishers exact test is used.
Statistics
Statistics is a topic that generally strikes fear and dread into most surgeons hearts. The MRCS is not an examination designed to test mathematical skill but the examiners do expect you to have working knowledge of commonly used tests so that you can appraise the literature properly.
Data types
Before selecting a method of statistical analysis it is imperative that the type of data to be analysed is correctly categorised. Commonly used terms include nominal, ordinal, interval and continuous.
Term Interpretation
Nominal Data can be allocated a numerical code that is arbitrary. For example allocating people as alive or dead using codes of 0 or 1
Ordinal data Data using numbers that can be used on a scale. Severity of pain is often measured in this way
Interval scale Data is measured numerically. However, the zero point is arbitrary
Continuous Data is measured numerically where the numerical value is a real number and may be any value. Examples include height and weight
Analysing data
Having ascribed the data it is then possible to begin the process of analysis. Nominal data is often tabulated into categories because of the nature of the underlying data sets. Continuous data may be displayed graphically often as individual data points. When the sample size is large enough, continuous data can be analysed to determine the distribution of the data points. Often, but not always these will be in the form of a gaussian distribution. Determining whether data is normally distributed or not is key to making sense of the subsequent statistical tests. Parametric tests are used to test normally distributed data, the T Test is one of the best examples. Data which is not normally distributed cannot be analysed in this way and a non parametric test must be used. Examples of such tests include Chi Squared and Mann Whitney U tests. Chi squared tests often appear in the medical literature. There are some assumptions that are made in relation to Chi squared tests; these include the need to use 2 degrees of freedom (usually) and the minimum sample size. Where the sample size is small then a different test is appropriate and the Fishers exact test is often used.
In situations where data is normally distributed and paired samples are taken from the same individuals (such as following an intervention) then the paired T Test may be used.
Multiple testing and post hoc analysis
In the ideal world statistical analysis is conducted on data that is collected prospectively according to pre set power calculations and defined end points. Occasionally, data does not produce an expected outcome or a certain type of patient appears to have a different result. Subsequent analysis of such groups is termed a post hoc analysis. This can be perfectly legitimate, alternatively it can represent the last ditch attempt of a researcher to try and find any aspect of the data that is worthwhile. This can lead to errors and false rejection of a null hypothesis. A statistically significant result is more likely to occur if the same dataset is subjected to multiple analyses. To counteract this problem some researchers will apply a Bonferroni correction, this adjusts the analysis to allow for multiple testing.
A 43 year old lady complains of significant varicose veins that are increasingly symptomatic. On examination, there is clinical signs of sapheno femoral junction incompetence and associated truncal varicosities. What is the most appropriate course of action?
Undertake a venous duplex scan
Undertake a venous doppler study
Perform a Trendelenberg test
Offer the patient radiofrequency ablative therapy
Offer the patient surgical excision of the varicosities
In modern venous surgery, decision making is based on venous imaging to determine which treatment modality is best. Few surgeons would proceed straight to surgery and excision without this test. A venous doppler would show incompetence of the valve but not more than that.
Vascular investigations
Venous disease
Venous Doppler
The simplest investigation for assessment of venous junctional incompetence is a Doppler assessment. This involves the patient standing and manual compression of the limb distal to the junction of interest. Flow should normally occur in one direction only. Where junctional incompetence is present reverse flow will occur and is relatively easy to identify.
Venograms and duplex scans
Structural venous information is historically obtained using a venogram. This is an invasive test and rarely required in modern clinical practice. The most helpful test is a venous duplex scan which will provide information relating to flow and vessel characteristics. Duplex is also useful in providing vein maps for bypass surgery.
Arterial disease
Ankle-brachial pressure
The ankle brachial pressure index measurement is an important investigation as it will allow classification of the severity of the flow compromise present. False readings may occur in those with calcified vessels such as diabetics and results in such settings should be interpreted with caution. When auscultating the vessel note should be made of the character of the signal. Monophasic signals are associated with a proximal stenosis and reduction in flow. Triphasic signals provide reassurance of a healthy vessel.
Arterial Duplex
As with the vein the duplex scan can provide a substantial amount of information about arterial patency and flow patterns. In skilled hands they can provide insight as to the state of proximal vessels that are anatomically inaccessible to duplex (e.g. Iliacs). Through assessment of distal flow patterns. It is an operator dependent test.
Conventional angiogram
Vessel puncture and catheter angiography is the gold standard method of assessing arteries. High quality information can usually be obtained. Limitations of the technique include the risk of contrast toxicity and risks of vessel damage. Severely calcified vessels may be difficult to puncture and in this situation a remote access site (e.g. brachial) may be used. This technique is particularly useful in providing a distal arterial roadmap prior to femoro-distal bypass.
CT angiography
These tests provide a considerable amount of structural and flow information. They require contrast and thus carry the risks associated with this. They are particularly useful in the setting of GI bleeding as they are rapidly available and can be performed by a non vascular radiologist. However, they lack the facility for endovascular intervention. In general they do not provide high enough resolution for distal arterial surgery.
Magnetic resonance angiography
This has the advantage of being non-invasive and not using nephrotoxic contrast. Movement artifact remains a problem in some sites and distal arterial resolution is imperfect.
A 22 year old woman has recently undergone a surgical excision of the submandibular gland. She presents to the follow up clinic with a complaint of tongue weakness on the ipsilateral side to her surgery. Which nerve has been damaged?
Hypoglossal nerve
Lingual nerve
Inferior alveolar nerve
Facial nerve
Lesser petrosal nerve
Three cranial nerves may be injured during submandibular gland excision.
Marginal mandibular branch of the facial nerve
Lingual nerve
Hypoglossal nerve
Hypoglossal nerve damage may result in paralysis of the ipsilateral aspect of the tongue. The nerve itself lies deep to the capsule surrounding the gland and should not be injured during an intracapsular dissection. The lingual nerve is probably at greater risk of injury. However, the effects of lingual nerve injury are sensory rather than motor.
Submandibular gland
Relations of the submandibular gland
Superficial Platysma, deep fascia and mandible
Submandibular lymph nodes
Facial vein (facial artery near mandible)
Marginal mandibular nerve
Cervical branch of the facial nerve
Deep Facial artery (inferior to the mandible)
Mylohyoid muscle
Sub mandibular duct
Hyoglossus muscle
Lingual nerve
Submandibular ganglion
Hypoglossal nerve
Submandibular duct (Wharton’s duct)
Opens lateral to the lingual frenulum on the anterior floor of mouth.
5 cm length
Lingual nerve wraps around Wharton’s duct. As the duct passes forwards it crosses medial to the nerve to lie above it and then crosses back, lateral to it, to reach a position below the nerve.
Innervation
Sympathetic innervation- Derived from superior cervical ganglion
Parasympathetic innervation- Submandibular ganglion via lingual nerve
Arterial supply
Branch of the facial artery. The facial artery passes through the gland to groove its deep surface. It then emerges onto the face by passing between the gland and the mandible.
Venous drainage
Anterior facial vein (lies deep to the Marginal Mandibular nerve)
Lymphatic drainage
Deep cervical and jugular chains of nodes
A 68 year old man with poorly controlled diabetes presents with severe otalgia and headaches. On examination, there is granulation tissue within the external auditory meatus. What is the most likely underlying infective agent?
Pseudomonas aeruginosa
Streptococcus pyogenes
Staphylococcus aureus
Actinomyces
Bacteroides fragilis
Malignant otitis externa is caused by Pseudomonas aeruginosa
Severe pain, headaches and granulation tissue within the external auditory meatus are key features of malignant otitis externa. Diabetes mellitus is one of the commonest risk factors.
Malignant otitis externa
- Uncommon type of otitis externa that is found in immunocompromised individuals (90% cases found in diabetics)
Infective organism is usually Pseudomonas aeruginosa
Infection commences in the soft tissues of the external auditory meatus, then progresses to involve the soft tissues and into the bony ear canal
Progresses to temporal bone osteomyelitis
Key features in history
Diabetes (90%) or immunosuppression (illness or treatment related)
Severe, unrelenting, deep-seated otalgia
Temporal headaches
Purulent otorrhea
Possibly dysphagia, hoarseness, and/or facial nerve dysfunction
Treatment
Anti pseudomonal antimicrobial agents
Topical agents
Hyperbaric oxygen is sometimes used in refractory cases
A 38 year old man undergoes an OGD to investigate dyspepsia. Following intubation of the duodenum, the ampulla of Vater is identified. At which of these sites, is the it most likely to be located?
Inferior aspect of the 1st part of the duodenum
Medial aspect of the 3rd part of the duodenum
Lateral aspect of the 2nd part of the duodenum
Lateral aspect of the 3rd part of the duodenum
Medial aspect of the 2nd part of the duodenum
The Ampulla of Vater is usually located 8-10cm from the pylorus and 2-3cm inferior to the opening of the accessory pancreatic duct of the lesser duodenal papilla. Knowledge of the location of the Ampulla is important when cannulating it at ERCP.
Duodenum
This is the first and widest part of the small bowel. It has a diameter of around 4-5cm. Its commencement is immediately distal to the pylorus and it runs for around 25cm where it becomes the jejunum at the region of the duodenojejunal flexure. It comprises four parts, superior, descending, horizontal and ascending. Of these, the horizontal is the longest segment. The first 2-3cm of the superior duodenum are intraperitoneal. The remainder is largely retroperitoneal with the exception of the final 1-2cm.
Medial relations of the duodenum include the superior pancreatico-duodenal artery and the pancreatic head. The descending duodenum is closely related to the commencement of the transverse colon which has little in the way of mesentery at this area. Posterior to the descending duodenum lies the right kidney.
The horizontal part passes transversely to the left with an upward deflection as it does so. From right to left it crosses in front of the right ureter, right psoas major, right gonadal vessels and IVC. It terminates anterior to the aorta. Anteriorly, it’s relations include the superior mesenteric vessels and the root of the small bowel.
The ascending part runs to the left of the aorta and upwards to the level of L2. It terminates by binding abruptly forwards as the duodenojejunal flexure. Posteriorly, are the left sympathetic trunk, left psoas major and left gonadal vessels. Anteriorly, it gives attachment to the root of the mesentery, while the left kidney lies laterally and the uncinate process of the pancreas lies medially. The region of the duodenojenunal flexure is fixed in position by the suspensory muscle of the duodenum. This fibromuscular band blends with the musculature of the flexure and passes upwards deep to the pancreas to gain attachment to the right crus of the diaphragm. It is referred to eponymously as the ligament of Treitz.
The following statements relating to the ankle joint are true except?
Three groups of ligaments provide mechanical stability
The sural nerve lies medial to the Achilles tendon at its point of insertion
Eversion of the foot occurs at the sub talar joint
The flexor hallucis longus tendon is the most posterior structure at the medial malleolus
The saphenous nerve crosses the ankle joint.
The sural nerve lies behind the distal fibula. Inversion and eversion are sub talar movements. The structures passing behind the medial malleolus from anterior to posterior include: tibialis posterior, flexor digitorum longus, posterior tibial vein, posterior tibial artery, nerve, flexor hallucis longus.
Ankle joint
The ankle joint is a synovial joint composed of the tibia and fibula superiorly and the talus inferiorly.
Ligaments of the ankle joint
Deltoid ligament (medially)
Lateral collateral ligament
Talofibular ligaments (both anteriorly and posteriorly)
The calcaneofibular ligament is separate from the fibrous capsule of the joint. The two talofibular ligaments are fused with it.
The components of the syndesmosis are
Antero-inferior tibiofibular ligament
Postero-inferior tibiofibular ligament
Inferior transverse tibiofibular ligament
Interosseous ligament
Movements at the ankle joint
Plantar flexion (55 degrees)
Dorsiflexion (35 degrees)
Inversion and eversion movements occur at the level of the sub talar joint
Nerve supply
Branches of deep peroneal and tibial nerves.
References
Golano P et al. Anatomy of the ankle ligaments: a pictorial essay. Knee Surg Sports Traumatol Arthrosc. 2010 May;18(5):557-69
In examining a biopsy of a primary tumour, the clearest evidence of malignancy is provided by:
Absence of a capsule
Basophilia of the cytoplasm
Invasion of surrounding structures
Excess of mitoses
Nuclear aberrations
Invasion is the hallmark of malignancy. The others may occur in insitu disease or dysplastic lesions.
Tissue sampling
Tissue sampling is an important surgical process. Biopsy modalities vary according to the site, experience and subsequent planned therapeutic outcome
The modalities comprise:
-Fine needle aspiration cytology
-Core biopsy
-Excision biopsy
-Tru cut biopsy
-Punch biopsy
-Cytological smears
-Endoscopic or laparoscopic biopsy
When the lesion is superficial the decision needs to be taken as to whether complete excision is desirable or whether excision biopsy is acceptable. In malignant melanoma for example the need for safe margins will mean that a more radical surgical approach needs to be adopted after diagnostic confirmation from excision biopsy than would be the case in basal cell carcinoma. Punch biopsies are useful in gaining histological diagnosis of unclear skin lesions where excision biopsy is undesirable such as in establishing whether a skin lesion is vasculitic or not.
Fine needle aspiration cytology (FNAC) is an operator dependent procedure that may or may not be image guided and essentially involves passing a needle through a lesion whilst suction is applied to a syringe. The material thus obtained is expressed onto a slide and sent for cytological assessment. This test can be limited by operator inexperience and also by the lack of histological architectural information (e.g. Follicular carcinoma of the thyroid). Where a discharge is present a sample may be sent for cytology although in some sites (e.g. Nipple discharge ) the information gleaned may be meaningless.
Tissue samples may be obtained by both core and tru cut biopsy. A core biopsy is obtained by use of a spring loaded gun with a needle passing quickly through the lesion of interest. A tru cut biopsy achieves the same objective but the needle moved by hand. When performing these techniques image guidance may be desirable (e.g. In breast lesions). Consideration needs to be given to any planned surgical resection as it may be necessary to resect the biopsy tract along with the specimen (e.g. In sarcoma surgery).
Visceral lesions may be accessed percutaneously under image guidance such as ultrasound guided biopsy of liver metastases. Or under direct vision such as a colonoscopic biopsy.
A 23 year old man undergoes an orchidectomy. The right testicular vein is ligated; into which structure does it drain?
Right renal vein
Inferior vena cava
Common iliac vein
Internal iliac vein
External iliac vein
The testicular venous drainage begins in the septa and these veins together with those of the tunica vasculosa converge on the posterior border of the testis as the pampiniform plexus. The pampiniform plexus drains to the testicular vein. The left testicular vein drains into the left renal vein. The right testicular vein drains into the inferior vena cava.
Scrotal and testicular anatomy
Spermatic cord
Formed by the vas deferens and is covered by the following structures:
Layer Origin
Internal spermatic fascia Transversalis fascia
Cremasteric fascia From the fascial coverings of internal oblique
External spermatic fascia External oblique aponeurosis
Contents of the cord
Vas deferens Transmits sperm and accessory gland secretions
Testicular artery Branch of abdominal aorta supplies testis and epididymis
Artery of vas deferens Arises from inferior vesical artery
Cremasteric artery Arises from inferior epigastric artery
Pampiniform plexus Venous plexus, drains into right or left testicular vein
Sympathetic nerve fibres Lie on arteries, the parasympathetic fibres lie on the vas
Genital branch of the genitofemoral nerve Supplies cremaster
Lymphatic vessels Drain to lumbar and para-aortic nodes
Scrotum
Composed of skin and closely attached dartos fascia.
Arterial supply from the anterior and posterior scrotal arteries
Lymphatic drainage to the inguinal lymph nodes
Parietal layer of the tunica vaginalis is the innermost layer
Testes
The testes are surrounded by the tunica vaginalis (closed peritoneal sac). The parietal layer of the tunica vaginalis adjacent to the internal spermatic fascia.
The testicular arteries arise from the aorta immediately inferiorly to the renal arteries.
The pampiniform plexus drains into the testicular veins, the left drains into the left renal vein and the right into the inferior vena cava.
Lymphatic drainage is to the para-aortic nodes.
Which of the tumour markers listed below is most likely to be elevated in a patient with pancreatic cancer?
CEA
CA19-9
AFP
PSA
CA15-3
Tumour markers
Tumour markers may be divided into:
monoclonal antibodies against carbohydrate or glycoprotein tumour antigens
tumour antigens
enzymes (alkaline phosphatase, neurone specific enolase)
hormones (e.g. calcitonin, ADH)
It should be noted that tumour markers usually have a low specificity
Monoclonal antibodies
Tumour marker Association
CA 125 Ovarian cancer
CA 19-9 Pancreatic cancer
CA 15-3 Breast cancer
NB: The breast cancer tumour marker is not specific or sensitive enough to be used routinely.
Tumour antigens
Tumour marker Association
Prostate specific antigen (PSA) Prostatic carcinoma
Alpha-feto protein (AFP) Hepatocellular carcinoma, teratoma
Carcinoembryonic antigen (CEA) Colorectal cancer
An 18 year old man is cutting some plants when a small piece of vegetable matter enters his eye. His eye becomes watery. Which of the following is responsible for relaying parasympathetic neuronal signals to the lacrimal apparatus?
Pterygopalatine ganglion
Otic ganglion
Submandibular ganglion
Ciliary ganglion
None of the above
The parasympathetic fibres to the lacrimal apparatus transit via the pterygopalatine ganglion.
Lacrimal system
Lacrimal gland
Consists of an orbital part and palpebral part. They are continuous posterolaterally around the concave lateral edge of the levator palpebrae superioris muscle.
The ducts of the lacrimal gland open into the superior fornix. Those from the orbital part penetrate the aponeurosis of levator palpebrae superioris to join those from the palpebral part. Therefore excision of the palpebral part is functionally similar to excision of the entire gland.
Blood supply
Lacrimal branch of the opthalmic artery. Venous drainage is to the superior opthalmic vein.
Innervation
The gland is innervated by the secretomotor parasympathetic fibres from the pterygopalatine ganglion which in turn may reach the gland via the zygomatic or lacrimal branches of the maxillary nerve or pass directly to the gland. The preganglionic fibres travel to the ganglion in the greater petrosal nerve (a branch of the facial nerve at the geniculate ganglion).
Nasolacrimal duct
Descends from the lacrimal sac to open anteriorly in the inferior meatus of the nose.
Lacrimation reflex
Occurs in response to conjunctival irritation (or emotional events). The conjunctiva will send signals via the opthalmic nerve. These then pass to the superior salivary centre. The efferent signals pass via the greater petrosal nerve (parasympathetic preganglionic fibres) and the deep petrosal nerve which carries the post ganglionic sympathetic fibres. The parasympathetic fibres will relay in the pterygopalatine ganglion, the sympathetic fibres do not synapse. They in turn will relay to the lacrimal apparatus.
Which of the following is not a typical feature of neuropraxia?
Transient delay in neuronal transmission
Axonal degeneration distal to the site of injury
Absence of neuroma formation
Preservation of autonomic function
Absence of axonal degeneration proximal to the site of injury
Axonal degeneration distal to the site of injury
Full recovery may occur 6-8 weeks after nerve injury in neuropraxia.
Wallerian degeneration does not usually occur in simple neuropraxia.
Autonomic function is usually preserved.
Nerve injury
There are 3 types of nerve injury:
Neuropraxia
Nerve intact but electrical conduction is affected
Full recovery
Autonomic function preserved
Wallerian degeneration does not occur
Axonotmesis
Axon is damaged and the myelin sheath is preserved. The connective tissue framework is not affected.
Wallerian degeneration occurs.
Neurotmesis
Disruption of the axon, myelin sheath and surrounding connective tissue.
Wallerian degeneration occurs.
Wallerian Degeneration
Axonal degeneration distal to the site of injury.
Typically begins 24-36 hours following injury.
Axons are excitable prior to degeneration occurring.
Myelin sheath degenerates and is phagocytosed by tissue macrophages.
Nerve repair
Neuronal repair may only occur physiologically where nerves are in direct contact. Where a large defect is present, the process of nerve regeneration is hampered. It may not occur at all or result in the formation of a neuroma. Where nerve regrowth occurs it is typically at a rate of 1mm per day.
A 55 year old man presents with a soft, fluctuant lesion overlying his right scapula. The surgeon suspects the lesion may be a lipoma. Which of the following, if present, may be indicative of an alternative diagnosis?
Located in superficial tissues
Size greater than 5cm
Presence of multiple similar lesions at other anatomical sites
Increased mobility of the lesion
Lobulated appearance during surgical excision
Lipomas are typically small and mobile lesions. They may be multiple. Lesions >5cm may be indicative of a soft tissue sarcoma and additional diagnostic tests may be required prior to excision.
Lipomata
Benign tumour of adipocytes
Occur in middle aged adults
Smooth, mobile, painless
Subtypes: Angiolipoma, Angiolipoleiomyoma
Malignant transformation is rare
Features suggestive of sarcomatous change
Size >5cm
Increasing size
Pain
Deep anatomical location
In one series the presence of all 4 features was associated with up to 85% being sarcomatous (1).
Lipoma removed surgically. Benign lesions are often small and well encapsulated. Deep seated or lesions larger than 5cm are at increased risk of being associated with sarcomatous change
Which of the following is not found within the deep perineal pouch in an adult male?
Pudendal nerve
Dorsal nerve of the penis
Sphincter urethrae
Urethral artery
Obturator nerve
Obturator nerve
Deep perineal pouch
The perineal pouch is surrounded inferiorly by the inferior fascia of the urogenital diaphragm. This fascial boundary extends laterally to form the medial wall of the ischiorectal fossa. The pouch is bounded superiorly by the superior fascia of the urogenital diaphragm and this lies beneath the levator ani muscle.
Contents of the deep perineal pouch
Urethral sphincter
Transversus perinei
Dorsal nerve of penis, muscular branches of the perineal nerve
Deep and dorsal arteries of penis, stem of origin of artery to the bulb of penis, urethral artery.
A 22 year old man has a long history of ulcerative colitis. His symptoms are well controlled with steroids. However, attempts at steroid weaning and use of steroid sparing drugs have repeatedly failed. He wishes to avoid a permanent stoma. Which of the following is the best operative option?
Pan proctocolectomy and end ileostomy
Abdomino perineal excision of the colon and rectum and end colostomy
Abdomino perineal excision of the colon and rectum and construction of an ileo anal pouch
Pan proctocolectomy and construction of an ileo anal pouch
Sub total colectomy and construction of an ileo anal pouch
Don’t confuse AP resection and proctectomy. The former is a cancer related procedure.
In patients with UC where medical management is not successful, surgical resection may offer a chance of cure. Those patients wishing to avoid a permanent stoma may be considered for an ileoanal pouch. However, this procedure is only offered in the elective setting.
Surgery for inflammatory bowel disease
Patients with inflammatory bowel disease (UC and Crohns) frequently present in surgical practice. Ulcerative colitis may be cured by surgical resection (Proctocolectomy), this is not the case in Crohns disease which may recur and affect other areas of the gastrointestinal tract.
Ulcerative colitis
Elective indications for surgery include disease that is requiring maximal therapy, or prolonged courses of steroids.
Longstanding UC is associated with a risk of malignant transformation. Dysplastic transformation of the colonic epithelium with associated mass lesions is an absolute indication for a proctocolectomy.
Emergency presentations of poorly controlled colitis that fails to respond to medical therapy should usually be managed with a sub total colectomy. Excision of the rectum is a procedure with a higher morbidity and is not generally performed in the emergency setting. An end ileostomy is usually created and the rectum either stapled off and left in situ, or, if the bowel is very oedematous, may be brought to the surface as a mucous fistula.
Patients with IBD have a high incidence of DVT and appropriate thromboprophylaxis is mandatory.
Restorative options in UC include an ileoanal pouch. This procedure can only be performed whilst the rectum is in situ and cannot usually be undertaken as a delayed procedure following proctectomy.
Ileoanal pouch complications include, anastomotic dehiscence, pouchitis and poor physiological function with seepage and soiling.
Crohns disease
Surgical resection of Crohns disease does not equate with cure, but may produce substantial symptomatic improvement.
Indications for surgery include complications such as fistulae, abscess formation and strictures.
Extensive small bowel resections may result in short bowel syndrome and localised stricturoplasty may allow preservation of intestinal length.
Staging of Crohns will usually involve colonoscopy and a small bowel study (e.g. MRI enteroclysis).
Complex perianal fistulae are best managed with long term draining seton sutures, complex attempts at fistula closure e.g. advancement flaps, may be complicated by non healing and fistula recurrence.
Severe perianal and / or rectal Crohns may require proctectomy. Ileoanal pouch reconstruction in Crohns carries a high risk of fistula formation and pouch failure and is not recommended.
Terminal ileal Crohns remains the commonest disease site and these patients may be treated with limited ileocaecal resections.
Terminal ileal Crohns may affect enterohepatic bile salt recycling and increase the risk of gallstones.
Which of the following is responsible for the rapid depolarisation phase of the myocardial action potential?
Rapid sodium influx
Rapid sodium efflux
Slow efflux of calcium
Efflux of potassium
Rapid calcium influx
Electrical activity of the heart
Myocardial action potential
Phase Description Mechanism
0 Rapid depolarisation Rapid sodium influx
These channels automatically deactivate after a few ms
1 Early repolarisation Efflux of potassium
2 Plateau Slow influx of calcium
3 Final repolarisation Efflux of potassium
4 Restoration of ionic concentrations Resting potential is restored by Na+/K+ ATPase
There is slow entry of Na+ into the cell decreasing the potential difference until the threshold potential is reached, triggering a new action potential
NB cardiac muscle remains contracted 10-15 times longer than skeletal muscle
Conduction velocity
Atrial conduction Spreads along ordinary atrial myocardial fibres at 1 m/sec
AV node conduction 0.05 m/sec
Ventricular conduction Purkinje fibres are of large diameter and achieve velocities of 2-4 m/sec (this allows a rapid and coordinated contraction of the ventricles
A 56 year old man is admitted with acute retention of urine. He has had a recent urinary tract infection. An USS shows bilateral hydronephrosis. What is the best course of action?
Antegrade ureteric stents
Retrograde ureteric stents
Urethral catheter
Bilateral nephrostomy
Suprapubic catheter
Establishing bladder drainage will often correct the situation. These patients often have a significant diuresis with associated electrolyte disturbance. The urethral route should be tried first.
Hydronephrosis
Causes of hydronephrosis
Unilateral: PACT
Pelvic-ureteric obstruction (congenital or acquired)
Aberrant renal vessels
Calculi
Tumours of renal pelvis
Bilateral: SUPER
Stenosis of the urethra
Urethral valve
Prostatic enlargement
Extensive bladder tumour
Retro-peritoneal fibrosis
Investigation
USS- identifies presence of hydronephrosis and can assess the kidneys
IVU- assess the position of the obstruction
Antegrade or retrograde pyelography- allows treatment
If renal colic suspected: non contrast CT scan (majority of stones are detected this way)
Management
Remove the obstruction and drainage of urine
Acute upper urinary tract obstruction: Nephrostomy tube
Chronic upper urinary tract obstruction: Ureteric stent or a pyeloplasty
A 55 year old lady has undergone a wide local excision and sentinel lymph node biopsy for breast cancer. The histology report shows a completely excised 1.3cm grade 1 invasive ductal carcinoma. The sentinel node contained no evidence of metastatic disease. The tumour is oestrogen receptor positive. What is the next course of action?
Monitor in clinic with annual review and mammography
Arrange radiotherapy
Arranged combined chemoradiotherapy
Arrange chemotherapy
Administration of GnRH analogue
Radiotherapy is routine following breast conserving surgery. Without irradiation the local recurrence rates are approximately 40%. These rates are potentially lower in older patients who receive endocrine therapy and who have small low grade tumours. Alongside this, an anti oestrogen will be needed.
Breast cancer treatment
Treatment Indication
Endocrine therapy
Oestrogen receptor positive tumours
Downstaging primary lesions
Definitive treatment in old, infirm patients
Irradiation
Wide local excision
Large lesion, high grade or marked vascular invasion following mastectomy
Chemotherapy
Downstaging advanced lesions to facilitate breast conserving surgery
Patients with grade 3 lesions or axillary nodal disease
Endocrine agents
Tamoxifen is used and works as a partial oestrogen receptor agonist. It will typically block activity at the breast. It does, however, stimulate the receptors at other sites and it is this that accounts for its association with endometrial cancer. In post menopausal women the process of aromatisation accounts for most oestrogen production. Therefore in this group aromatase inhibitors are the preferred agents. Women who are perimenopausal start on tamoxifen and switch at 3 years.
More recent studies (aTTom and ATLAS) have demonstrated benefits for continuing the drug for 10 years. In pre-menopausal women, there is increasing preference for the use of Exemestane over tamoxifen.
Chemotherapy
The FEC regime is most commonly used (Fluorouracil, epirubicin and cyclophosphamide). This was found to be superior to the older CMF regime. The Taxanes are commonly used in high risk patients and in this setting a regime of docetaxal, doxorubicin and cyclophosphamide may be used. The anthracycline class drugs have marked cardiotoxicity (a property that they share with trastuzumab) and this can limit their use.
A 34 year old man presents with symptoms attributable to a fistula in ano. He is examined in the lithotomy position and the external opening of the fistula is identified in the 7 o’clock position. At which of the following locations is the internal opening most likely to be identified?
7 o’clock
12 o’clock
9 o’clock
3 o’clock
6 o’clock
Goodsals rule:
Anterior fistulae will tend to have an internal opening opposite the external opening.
Posterior fistulae will tend to have a curved track that passes towards the midline.
According to Goodsalls rule the track of a posteriorly sited fistula will track to the posterior midline (i.e. 6 o’clock)
Fistulas
A fistula is defined as an abnormal connection between two epithelial surfaces.
There are many types ranging from Branchial fistulae in the neck to entero-cutaneous fistulae abdominally.
In general surgical practice the abdominal cavity generates the majority and most of these arise from diverticular disease and Crohn’s.
As a general rule all fistulae will resolve spontaneously as long as there is no distal obstruction. This is particularly true of intestinal fistulae.
The four types of fistulae are:
Enterocutaneous
These link the intestine to the skin. They may be high (>500ml) or low output (<250ml) depending upon source. Duodenal /jejunal fistulae will tend to produce high volume, electrolyte rich secretions which can lead to severe excoriation of the skin. Colo-cutaneous fistulae will tend to leak faeculent material. Both fistulae may result from the spontaneous rupture of an abscess cavity onto the skin (such as following perianal abscess drainage) or may occur as a result of iatrogenic input. In some cases it may even be surgically desirable e.g. mucous fistula following sub total colectomy for colitis.
Suspect if there is excess fluid in the drain.
Enteroenteric or Enterocolic
This is a fistula that involves the large or small intestine. They may originate in a similar manner to enterocutaneous fistulae. A particular problem with this fistula type is that bacterial overgrowth may precipitate malabsorption syndromes. This may be particularly serious in inflammatory bowel disease.
Enterovaginal
Aetiology as above.
Enterovesical
This type of fistula goes to the bladder. These fistulas may result in frequent urinary tract infections, or the passage of gas from the urethra during urination.
Management
Some rules relating to fistula management:
They will heal provided there is no underlying inflammatory bowel disease and no distal obstruction, so conservative measures may be the best option
Where there is skin involvement, protect the overlying skin, often using a well fitted stoma bag- skin damage is difficult to treat
A high output fistula may be rendered more easily managed by the use of octreotide, this will tend to reduce the volume of pancreatic secretions.
Nutritional complications are common especially with high fistula (e.g. high jejunal or duodenal) these may necessitate the use of TPN to provide nutritional support together with the concomitant use of octreotide to reduce volume and protect skin.
When managing perianal fistulae surgeons should avoid probing the fistula where acute inflammation is present, this almost always worsens outcomes.
When perianal fistulae occur secondary to Crohn’s disease the best management option is often to drain acute sepsis and maintain that drainage through the judicious use of setons whilst medical management is implemented.
Always attempt to delineate the fistula anatomy, for abscesses and fistulae that have an intra abdominal source the use of barium and CT studies should show a track. For perianal fistulae surgeons should recall Goodsall’s rule in relation to internal and external openings.
Which of the following structures suspends the spinal cord in the dural sheath?
Filum terminale
Conus medullaris
Ligamentum flavum
Denticulate ligaments
Anterior longitudinal ligament
The spinal cord is approximately 45cm in men and 43cm in women. The denticulate ligament is a continuation of the pia mater (innermost covering of the spinal cord) which has intermittent lateral projections attaching the spinal cord to the dura mater.
Spinal cord
- Located in a canal within the vertebral column that affords it structural support.
Rostrally it continues to the medulla oblongata of the brain and caudally it tapers at a level corresponding to the L1-2 interspace (in the adult), a central structure, the filum terminale anchors the cord to the first coccygeal vertebra.
The spinal cord is characterised by cervico-lumbar enlargements and these, broadly speaking, are the sites which correspond to the brachial and lumbar plexuses respectively.
There are some key points to note when considering the surgical anatomy of the spinal cord:
- During foetal growth the spinal cord becomes shorter than the spinal canal, hence the adult site of cord termination at the L1-2 level.
- Due to growth of the vertebral column the spine segmental levels may not always correspond to bony landmarks as they do in the cervical spine.
- The spinal cord is incompletely divided into two symmetrical halves by a dorsal median sulcus and ventral median fissure. Grey matter surrounds a central canal that is continuous rostrally with the ventricular system of the CNS.
- The grey matter is sub divided cytoarchitecturally into Rexeds laminae.
- Afferent fibres entering through the dorsal roots usually terminate near their point of entry but may travel for varying distances in Lissauers tract. In this way they may establish synaptic connections over several levels
- At the tip of the dorsal horn are afferents associated with nociceptive stimuli. The ventral horn contains neurones that innervate skeletal muscle.
The key point to remember when revising CNS anatomy is to keep a clinical perspective in mind. So it is worth classifying the ways in which the spinal cord may become injured. These include:
Trauma either direct or as a result of disc protrusion
Neoplasia either by direct invasion (rare) or as a result of pathological vertebral fracture
Inflammatory diseases such as Rheumatoid disease, or OA (formation of osteophytes compressing nerve roots etc.
Vascular either as a result of stroke (rare in cord) or as complication of aortic dissection
Infection historically diseases such as TB, epidural abscesses.
The anatomy of the cord will, to an extent dictate the clinical presentation. Some points/ conditions to remember:
Brown- Sequard syndrome-Hemisection of the cord producing ipsilateral loss of proprioception and upper motor neurone signs, plus contralateral loss of pain and temperature sensation. The explanation of this is that the fibres decussate at different levels.
Lesions below L1 will tend to present with lower motor neurone signs
A footballer sustains a knee injury in a match and is being assessed in the outpatient department. On examination, he has a positive valgus stress test and minimal joint effusion. What is the most likely underlying injury?
Injury to the lateral collateral ligament
Injury to the medial collateral ligament
Injury to the anterior cruciate ligament
Injury to the posterior cruciate ligament
Injury to the patellar tendon
A knee injury in the footballer with a positive valgus stress test is usually associated with MCL injury.
Knee collateral ligament
Anatomy
The tibial collateral ligament is a broad, flat band. Its upper end has an extensive attachment to the medial epicondyle of the femur with some fibres projecting onto the adductor magnus tendon. The ligament passes downwards and forwards to the medial side of the tibia. The deepest fibres are fused with the medial meniscus.
The fibular collateral ligament is round and cord like and stands clear of the thin, lateral part of the fibrous capsule. It is enclosed within the fascia lata. It passes from the lateral epicondyle of the femur to the head of the fibula in front of its highest point and splits the tendon of biceps femoris. On the lateral side of the joint the fibres are short and weak and bridge the interval between the femoral and tibial condyles. The popliteus tendon intervenes between the lateral meniscus and the capsule.
The tibial and fibular collateral ligaments prevent disruption of the joint at the sides. They are most tightly stretched in extension, and then their direction- the fibular ligament downwards and backwards, the tibial downwards and forwards- prevents rotation of the tibia laterally or the femur medially. Rotation may be demonstrated in the flexed knee.
Injury
The collateral ligaments are commonly injured, the medial is most often affected. It requires a significant force such as sporting tackle or motor vehicle to strike the side of the leg. Associated injuries to both the tibial plateau or menisci are not uncommon.
Grading and treatment
Grade of injury Features Treatment
1 Minor tearing of ligament fibres
Negative instability tests Conservative (analgesia and physiotherapy)
2 Ligament laxity (seen with knee in 30o flexion)
Knee stable when joint extended Usually splinting or casting for 4-6 weeks
3 Ligament completely torn
Joint instability Surgical ligament reconstruction
Which of the following statements relating to cerebrospinal fluid is untrue?
The choroid plexus is only present in the lateral ventricles
Total CSF volume is 100-150ml
CSF pressure is usually 10-15mmHg
The cerebral aqueduct connects the third and fourth ventricle
The foramen of Luschka are paired and lie laterally in the fourth ventricle
The choroid plexus lies in all ventricles.
Cerebrospinal fluid
The CSF fills the space between the arachnoid mater and pia mater (covering surface of the brain). The total volume of CSF in the brain is approximately 150ml. Approximately 500 ml is produced by the ependymal cells in the choroid plexus (70%), or blood vessels (30%). It is reabsorbed via the arachnoid granulations which project into the venous sinuses.
Circulation
1. Lateral ventricles (via foramen of Munro)
2. 3rd ventricle
3. Cerebral aqueduct (aqueduct of Sylvius)
4. 4th ventricle
5. Subarachnoid space (via foramina of Magendie and Luschka)
6. Reabsorbed into the venous system via arachnoid granulations into superior sagittal sinus
Composition
Glucose: 50-80mg/dl
Protein: 15-40 mg/dl
Red blood cells: Nil
White blood cells: 0-3 cells/ mm3
Which of the following statements relating to large volume blood loss in trauma is incorrect?
Tranexamic acid reduces the incidence of rebleeding following surgery
Hypocalcaemia may complicate resuscitation
Colloids are preferred initially as they reduce the incidence of coagulopathy
When patients receive over 5 units of whole blood mortality increases when blood products greater than 3 weeks old are utilised
In the battlefield setting a ratio of 1:1:1 for blood, plasma and platelets is used
Fresh blood is the fluid of choice when large volume blood loss complicates trauma. Mortality is doubled when blood >3 weeks old is used.
Trauma management
The cornerstone of trauma management is embodied in the principles of ATLS.
Following trauma there is a trimodal death distribution:
Immediately following injury. Typically as result of brain or high spinal injuries, cardiac or great vessel damage. Salvage rate is low.
In early hours following injury. In this group deaths are due to phenomena such as splenic rupture, sub dural haematomas and haemopneumothoraces
In the days following injury. Usually due to sepsis or multi organ failure.
Aspects of trauma management
ABCDE approach.
Tension pneumothoraces will deteriorate with vigorous ventilation attempts.
External haemorrhage is managed as part of the primary survey. As a rule tourniquets should not be used. Blind application of clamps will tend to damage surrounding structures and packing is the preferred method of haemorrhage control.
Urinary catheters and naso gastric tubes may need inserting. Be wary of basal skull fractures and urethral injuries.
Patients with head and neck trauma should be assumed to have a cervical spine injury until proven otherwise.
Thoracic injuries
Simple pneumothorax
Mediastinal traversing wounds
Tracheobronchial tree injury
Haemothorax
Blunt cardiac injury
Diaphragmatic injury
Aortic disruption
Pulmonary contusion
Management of thoracic trauma
Simple pneumothorax insert chest drain. Aspiration is risky in trauma as pneumothorax may be from lung laceration and convert to tension pneumothorax.
Mediastinal traversing wounds These result from situations like stabbings. Exit and entry wounds in separate hemithoraces. The presence of a mediastinal haematoma indicates the likelihood of a great vessel injury. All patients should undergo CT angiogram and oesophageal contrast swallow. Indications for thoracotomy are largely related to blood loss and will be addressed below.
Tracheobronchial tree injury Unusual injuries. In blunt trauma most injuries occur within 4cm of the carina. Features suggesting this injury include haemoptysis and surgical emphysema. These injuries have a very large air leak and may have tension pneumothorax.
Haemothorax Usually caused by laceration of lung vessel or internal mammary artery by rib fracture. Patients should all have a wide bore 36F chest drain. Indications for thoracotomy include loss of more than 1.5L blood initially or ongoing losses of >200ml per hour for >2 hours.
Cardiac contusions Usually cardiac arrhythmias, often overlying sternal fracture. Perform echocardiography to exclude pericardial effusions and tamponade. Risk of arrhythmias falls after 24 hours.
Diaphragmatic injury Usually left sided. Direct surgical repair is performed.
Traumatic aortic disruption Commonest cause of death after RTA or falls. Usually incomplete laceration near ligamentum arteriosum. All survivors will have contained haematoma. Only 1-2% of patients with this injury will have a normal chest x-ray.
Pulmonary contusion Common and lethal. Insidious onset. Early intubation and ventilation.
Abdominal trauma
Deceleration injuries are common.
In blunt trauma requiring laparotomy the spleen is most commonly injured (40%)
Stab wounds traverse structures most commonly liver (40%)
Gunshot wounds have variable effects depending upon bullet type. Small bowel is most commonly injured (50%)
Patients with stab wounds and no peritoneal signs up to 25% will not enter the peritoneal cavity
Blood at urethral meatus suggests a urethral tear
High riding prostate on PR = urethral disruption
Mechanical testing for pelvic stability should only be performed once
Investigations in abdominal trauma
Diagnostic Peritoneal Lavage Abdominal CT scan USS
Indication Document bleeding if hypotensive Document organ injury if normotensive Document fluid if hypotensive
Advantages Early diagnosis and sensitive; 98% accurate Most specific for localising injury; 92 to 98% accurate Early diagnosis, non invasive and repeatable; 86 to 95% accurate
Disadvantages Invasive and may miss retroperitoneal and diaphragmatic injury Location of scanner away from facilities, time taken for reporting, need for contrast Operator dependent and may miss retroperitoneal injury
Amylase may be normal following pancreatic trauma
Urethrography if suspected urethral injury
A 28 year old woman, who is 30 weeks pregnant, presents with sudden onset chest pain associated with loss of consciousness. Her blood pressure is 170/90 mmHg, saturations on 15L oxygen 93%, heart rate 120 bpm and she is apyrexial. On examination, there is an early diastolic murmur, occasional bibasal creptitations and mild pedal oedema. An ECG shows ST elevation in leads II, III and aVF. What is the most likely diagnosis?
Pulmonary embolism
Aortic dissection
Mitral valve stenosis
Pneumonia
Pneumothorax
Aortic dissection is associated with the 3rd trimester of pregnancy, connective tissue disorders (Marfan’s, Ehlers- Danlos) and bicuspid valve. Patients may complain of a tearing chest pain or syncope. Clinically they may be hypertensive. The right coronary artery may become involved in the dissection, causing myocardial infarct in up to 2% cases (hence ST elevation in the inferior leads). An aortic regurgitant murmur may be auscultated.
Chest pain in pregnancy
Aortic dissection
Predisposing factors in pregnancy are hypertension, congenital heart disease and Marfan’s syndrome
Mainly Stanford type A dissections
Sudden tearing chest pain, transient syncope
Patient may be cold and clammy, hypertensive and have an aortic regurgitation murmur
Involvement of the right coronary artery may cause inferior myocardial infarction
Surgical management
Gestational timeframe Management
< 28/40 Aortic repair with the fetus kept in utero
28-32/40 Dependent on fetal condition
> 32/40 Primary Cesarean section followed by aortic repair at the same operation
Mitral stenosis
Most cases associated with rheumatic heart disease
Becoming less common in British women; suspect in women who have moved recently to the country
Commonest cardiac condition in pregnancy
Commonly associated with mortality
Valve surgery; balloon valvuloplasty preferable
Pulmonary embolism
Leading cause of mortality in pregnancy
Half dose scintigraphy; CT chest if underlying lung disease, should aid diagnosis
Treatment with low molecular weight heparin throughout pregnancy and 4-6 weeks after childbirth
Warfarin is contra indicated in pregnancy (though may be continued in women with mechanical heart valves due to the significant risk of thromboembolism)
References
1. Bates S.M. and Ginsberg J.S. How we manage venous thromboembolism during pregnancy. Blood 2002 (100): 3470-3478.
- Scarsbrook A.Fand Gleeson V. Investigating suspected pulmonary embolism in pregnancy. BMJ 2007 (326) : 1135 doi: 10.1136/bmj.7399.1135.
- Morley C. A. and Lim B. A. Lesson of the Week: The risks of delay in diagnosis of breathlessness in pregnancy. BMJ 1995 (311) : 1083.
Which of the following drugs is least likely to cause syndrome of inappropriate anti diuretic hormone release?
Haloperidol
Carbamazepine
Amitriptylline
Cyclophosphamide
5 Flurouracil
5 Flurouracil
Drugs causing SIADH: ABCD
A nalgesics: opioids, NSAIDs
B arbiturates
C yclophosphamide/ Chlorpromazine/ Carbamazepine
D iuretic (thiazides)
Hyponatraemia
This is commonly tested in the MRCS (despite most surgeons automatically seeking medical advice if this occurs!). The most common cause in surgery is the over administration of 5% dextrose.
Hyponatraemia may be caused by water excess or sodium depletion. Causes of pseudohyponatraemia include hyperlipidaemia (increase in serum volume) or a taking blood from a drip arm. Urinary sodium and osmolarity levels aid making a diagnosis.
Classification
Urinary sodium > 20 mmol/l Sodium depletion, renal loss
Patient often hypovolaemic
Diuretics (thiazides)
Addison’s
Diuretic stage of renal failure
SIADH (serum osmolality low, urine osmolality high, urine Na high)
Patient often euvolaemic
Mnemonic: Syndrome of INAPPropriate Anti-Diuretic Hormone:
In creased
Na (sodium)
PP (urine)
Urinary sodium < 20 mmol/l Sodium depletion, extra-renal loss
Diarrhoea, vomiting, sweating
Burns, adenoma of rectum (if villous lesion and large)
Water excess (patient often hypervolaemic and oedematous)
Secondary hyperaldosteronism: CCF, cirrhosis
Reduced GFR: renal failure
IV dextrose, psychogenic polydipsia
Management
Symptomatic Hyponatremia :
Acute hyponatraemia with Na <120: immediate therapy. Central Pontine Myelinolisis, may occur from overly rapid correction of serum sodium. Aim to correct until the Na is > 125 at a rate of 1 mEq/h. Normal saline with frusemide is an alternative method.
The sodium requirement can be calculated as follows :
(125 - serum sodium) x 0.6 x body weight = required mEq of sodium
A 39 year old man has suffered from terminal ileal Crohns disease for the past 20 years. Which condition is he least likely to develop?
Gallstones
Malabsorption
Pyoderma gangrenosum
Amyloidosis
Feltys syndrome
Feltys syndrome:
Rheumatoid disease
Splenomegaly
Neutropenia
Feltys syndrome is associated with rheumatoid disease. Individuals with long standing Crohns disease are at risk of gallstones because of impairment of the enterohepatic recycling of bile salts. Formation of entero-enteric fistulation may produce malabsorption. Amyloidosis may complicate chronic inflammatory states.
Crohns disease
Crohns disease is a chronic transmural inflammation of a segment(s) of the gastrointestinal tract and may be associated with extra intestinal manifestations. Frequent disease patterns observed include ileal, ileocolic and colonic disease. Peri-anal disease may occur in association with any of these. The disease is often discontinuous in its distribution. Inflammation may cause ulceration, fissures, fistulas and fibrosis with stricturing. Histology reveals a chronic inflammatory infiltrate that is usually patchy and transmural.
Ulcerative colitis Vs Crohns
Crohn’s disease Ulcerative colitis
Distribution Mouth to anus Rectum and colon
Macroscopic changes Cobblestone appearance, apthoid ulceration Contact bleeding
Depth of disease Transmural inflammation Superficial inflammation
Distribution pattern Patchy Continuous
Histological features Granulomas (non caseating epithelioid cell aggregates with Langhans’ giant cells) Crypt abscesses, Inflammatory cells in the lamina propria
Extraintestinal manifestations of Crohns
Related to disease extent Unrelated to disease extent
Aphthous ulcers (10%) Sacroiliiitis (10-15%)
Erythema nodosum (5-10%) Ankylosing spondylitis (1-2%)
Pyoderma gangrenosum (0.5%) Primary sclerosing cholangitis (Rare)
Acute arthropathy (6-12%) Gallstones (up to 30%)
Ocular complications (up to 10%) Renal calculi (up to 10%)
Diarrhoea in Crohns
Diarrhoea in Crohns may be multifactorial since actual inflammation of the colon is not common. Causes therefore include the following:
Bile salt diarrhoea secondary to terminal ileal disease
Entero-colic fistula
Short bowel due to multiple resections
Bacterial overgrowth
Surgical interventions in Crohns disease
The commonest disease pattern in Crohns is stricturing terminal ileal disease and this often culminates in an ileocaecal resection. Other procedures performed include segmental small bowel resections and stricturoplasty. Colonic involvement in patients with Crohns is not common and, where found, distribution is often segmental. However, despite this distribution segmental resections of the colon in patients with Crohns disease are generally not advocated because the recurrence rate in the remaining colon is extremely high. As a result, the standard options of colonic surgery in Crohns patients are generally; sub total colectomy, panproctocolectomy and staged sub total colectomy and proctectomy. Restorative procedures such as ileoanal pouch have no role in therapy.
Crohns disease is notorious for the developmental of intestinal fistulae; these may form between the rectum and skin (peri anal) or the small bowel and skin. Fistulation between loops of bowel may also occur and result in bacterial overgrowth and malabsorption. Management of enterocutaneous fistulae involves controlling sepsis, optimising nutrition, imaging the disease and planning definitive surgical management.
A 34 year old man presents with varicose veins and it is suspected that these are part of the Klippel-Trenaunay syndrome. Which of the following is not a characteristic of this condition?
Presence of varicose veins
Gigantism of a limb
Long saphenous vein involvement
Port wine stains with clear borders
Low flow rates through capillary anomalies
The Klippel-Trenaunay vein is a large, lateral, superficial vein sometimes seen at birth. This vein begins in the foot or the lower leg and travels proximally until it enters the thigh or the gluteal area. Otherwise, varicosities may not be clinically evident until the child begins to ambulate.
Varicosities may be extensive, though they often spare the saphenous distribution. They are seen below the knee, laterally above the knee, and occasionally in the pelvic region. Varicosities may affect the superficial, deep, and perforating venous systems.
Flow rates through the vascular anomalies are low flow rates in contrast to Parkes Weber syndrome where high flow rates are seen.
Surgical exploration has demonstrated atresia and agenesis of deep veins, compression due to fibrous bands, aberrant arteries, abnormal muscles, or venous sheaths.
Rarely, varicosities have been found in the bladder, the colon, and the pulmonary vessels
Klippel-Trenaunay syndrome is a rare condition that affects the development of blood vessels, soft tissues, and bones. The disorder has three characteristic features: a red birthmark called a port-wine stain, abnormal overgrowth of soft tissues and bones, and vein malformations12
The cause of Klippel-Trenaunay syndrome is not well understood, but it is not directly inherited from parents. It may be related to problems with how the blood vessel system develops during pregnancy23
The symptoms of Klippel-Trenaunay syndrome vary from person to person, but they usually affect only one limb, most often the leg. The port-wine stain is often the first sign of the condition, and it may cover a large area of the skin. The affected limb may also be longer and wider than the other, and have varicose veins or lymphatic malformations12
There is no cure for Klippel-Trenaunay syndrome, but treatments can help manage the symptoms and prevent complications. These may include compression garments, laser therapy, surgery, or medication24
A 6 month old boy is brought to the clinic with difficulty breathing and weight loss. On examination, he is found to have a firm para testicular mass. What is the most likely diagnosis?
Teratoma
Seminoma
Adenocarcinoma
Rhabdomyosarcoma
Malignant fibrous histiocytoma
Rhabdomyosarcomas as one of the more common malignant solid tumours in children (though all are rare). They have an aggressive behavior pattern and metastases are common. Teratomas are nearly always benign in younger children. Seminomas are very rare indeed. Malignant fibrous histiocytomas are almost never found in this location.
Sarcomas
Malignant tumours of mesenchymal origin
Types
May be either bone or soft tissue in origin.
Bone sarcoma include:
Osteosarcoma
Ewings sarcoma (although non bony sites recognised)
Chondrosarcoma - originate from Chondrocytes
Soft tissue sarcoma are a far more heterogeneous group and include:
Liposarcoma-adipocytes
Rhabdomyosarcoma-striated muscle
Leiomyosarcoma-smooth muscle
Synovial sarcomas- close to joints (cell of origin not known but not synovium)
Malignant fibrous histiocytoma is now referred to as undifferentiated pleomorphic sarcoma. Careful histological assessment of lesions now allows more accurate categorisation of sarcoma subtypes than was previously possible.
Features
Certain features of a mass or swelling should raise suspicion for a sarcoma these include:
Large >5cm soft tissue mass
Deep tissue location or intra muscular location
Rapid growth
Painful lump
Assessment
Imaging of suspicious masses should utilise a combination of MRI, CT and USS. Blind biopsy should not be performed prior to imaging and where required should be done in such a way that the biopsy tract can be subsequently included in any resection.
Ewings sarcoma
Commoner in males
Incidence of 0.3 / 1, 000, 000
Onset typically between 10 and 20 years of age
Location by femoral diaphysis is commonest site
Histologically it is a small round tumour
Blood borne metastasis is common and chemotherapy is often combined with surgery
Osteosarcoma
Mesenchymal cells with osteoblastic differentiation
20% of all primary bone tumours
Incidence of 5 per 1,000,000
Peak age 15-30, commoner in males
Limb preserving surgery may be possible and many patients will receive chemotherapy
Liposarcoma
Malignancy of adipocytes
Rare, approximately 2.5 per 1,000,000. They are the most common soft tissue sarcoma
Typically located in deep locations such as retroperitoneum
Affect older age group usually >40 years of age
May be well differentiated and thus slow growing although may undergo de-differentiation and disease progression
Many tumours will have a pseudocapsule that can misleadingly allow surgeons to feel that they can ‘shell out’ these lesions. In reality, tumour may invade at the edge of the pseudocapsule and result in local recurrence if this strategy is adopted
Usually resistant to radiotherapy, although this is often used in a palliative setting
Malignant Fibrous Histiocytoma
Tumour with large number of histiocytes
Also described as undifferentiated pleomorphic sarcoma NOS (i.e. Cell of origin is not known)
Four major subtypes are recognised: storiform-pleomorphic (70% cases), myxoid (less aggressive), giant cell and inflammatory
Treatment is usually with surgical resection and adjuvant radiotherapy as this reduces the likelihood of local recurrence
An ENT surgeon is undertaking a nasendoscopy to evaluate symptoms of recurrent sinusitis. As she performs the procedure, she notices that there is lesion within the sphenoethmoidal recess. Of the structures outlined below, which is most likely to be compromised as a result?
Sphenoidal sinus
Fontal sinus
Maxillary sinus
Anterior ethmoidal sinus
Posterior ethmoidal sinus
The sphenoidal sinus drains into the sphenoethmoidal recess and is likely to be affected. The other sinuses drain into the superior, middle and inferior meatus.
Nasal cavity
There are two nasal cavities separated by the median nasal septum. This structure consists of septal cartilage and two bony parts: the vomer and the perpendicular plate of the ethmoid bone. Each cavity is approximately 5cm high and 7cm long, from the nares anteriorly to the choanae posteriorly. The lateral walls of each cavity are mainly made up of the maxilla. However, the lacrimal, ethmoid and palatine bones also contribute. The ethmoid gives rise to the two upper protrusions from the lateral walls, the superior and middle conchae. The inferior concha is a separate bone. Above the superior concha lies the sphenoethmoidal recess. The spaces inferior to conchae are the superior, middle and inferior meatus. The structures draining into each space are shown below:
Sphenoethmoidal recess Sphenoidal sinus
Superior meatus Posterior ethmoidal sinus
Middle meatus Frontal sinus, maxillary sinus ,anterior and middle ethmoidal sinus
Inferior meatus Nasolacrimal duct
The main arterial supply to the septum and lateral walls is the sphenopalatine artery, a branch of the maxillary artery. The olfactory nerve conveys special sensation. Common sensation from the nasal mucosa is conveyed by the trigeminal nerve.
Which of the following is not contained within the deep posterior compartment of the lower leg?
Tibialis posterior muscle
Posterior tibial artery
Tibial nerve
Sural nerve
Flexor hallucis longus
The deep posterior compartment lies anterior to soleus. The sural nerve is superficially sited and therefore not contained within it. Lower limb- Muscular compartments
Anterior compartment
Muscle Nerve Action
Tibialis anterior Deep peroneal nerve Dorsiflexes ankle joint, inverts foot
Extensor digitorum longus Deep peroneal nerve Extends lateral four toes, dorsiflexes ankle joint
Peroneus tertius Deep peroneal nerve Dorsiflexes ankle, everts foot
Extensor hallucis longus Deep peroneal nerve Dorsiflexes ankle joint, extends big toe
Peroneal compartment
Muscle Nerve Action
Peroneus longus Superficial peroneal nerve Everts foot, assists in plantar flexion
Peroneus brevis Superficial peroneal nerve Plantar flexes the ankle joint
Superficial posterior compartment
Nerve Action
Gastrocnemius Tibial nerve Plantar flexes the foot, may also flex the knee
Soleus Tibial nerve Plantar flexor
Deep posterior compartment
Muscle Nerve Action
Flexor digitorum longus Tibial Flexes the lateral four toes
Flexor hallucis longus Tibial Flexes the great toe
Tibialis posterior Tibial Plantar flexor, inverts the foot
A 52 year old man develops septic shock following a Hartmans procedure for perforated diverticular disease. He is started on an adrenaline infusion. Which of the following is least likely to occur?
Peripheral vasoconstriction
Coronary artery vasospasm
Gluconeogenesis
Lipolysis
Tachycardia
It’s cardiac effects are mediated via β 1 receptors. The coronary arteries which have β 2 receptors are unaffected.
Adrenaline
Fight or Flight response
- Catecholamine (phenylalanine and tyrosine)
- Neurotransmitter and hormone
- Released by the adrenal glands
- Effects on α 1 and 2, β 1 and 2 receptors
- Effect on β 2 receptors in skeletal muscle vessels-causing vasodilation
- Increase cardiac output and total peripheral resistance
- Vasoconstriction in the skin and kidneys causing a narrow pulse pressure
Actions
α adrenergic receptors:
Inhibits insulin secretion by the pancreas
Stimulates glycogenolysis in the liver and muscle
Stimulates glycolysis in muscle
β adrenergic receptors:
Stimulates glucagon secretion in the pancreas
Stimulates ACTH
Stimulates lipolysis by adipose tissue
A 32 year old man is diagnosed as having a carcinoma of the caecum. On questioning, his mother developed uterine cancer at the age of 39 and his maternal uncle died from colonic cancer aged 38. His older brother developed a colonic cancer with micro satellite instability aged 37. What is the most appropriate operative treatment?
Limited ileocaecal resection
Right hemicolectomy
Extended right hemicolectomy
Panproctocolectomy
Sub total colectomy
The likely diagnosis is one of a familial cancer syndrome and now that he has developed a colonic cancer the safest operative strategy is a total colectomy and end ileostomy.
Polyposis syndromes
Syndrome Genetic defect Features Screening and management Associated disorders
Familial adenomatous polyposis Mutation of APC gene (80%) cases, dominant Typically over 100 colonic adenomas
Cancer risk of 100%
20% are new mutations If known to be at risk then predictive genetic testing as teenager
Annual flexible sigmoidoscopy from 15 years
If no polyps found then 5 yearly colonoscopy started at age 20
Polyps found = resectional surgery (resection and pouch Vs sub total colectomy and IRA) Gastric fundal polyps (50%).
Duodenal polyps 90%.
If severe duodenal polyposis cancer risk of 30% at 10 years.
Abdominal desmoid tumours.
MYH associated polyposis Biallelic mutation of mut Y human homologue (MYH) on chromosome 1p, recessive Multiple colonic polyps
Later onset right sided cancers more common than in FAP
100% cancer risk by age 60 Once identified resection and ileoanal pouch reconstruction is recommended
Attenuated phenotype - regular colonoscopy Duodenal polyposis in 30%
Associated with increased risk of breast cancer (self examination)
Peutz -Jeghers syndrome STK11 (LKB1) mutation on chromosome 19 in some (but not all) cases, dominant Multiple benign intestinal hamartomas
Episodic obstruction and intussceception
Increased risk of GI cancers (colorectal cancer 20%, gastric 5%)
Increased risk of breast, ovarian, cervical pancreatic and testicular cancers Annual examination
Pan intestinal endoscopy every 2-3 years Malignancies at other sites
Classical pigmentation pattern
Cowden disease Mutation of PTEN gene on chromosome 10q22, dominant Macrocephaly
Multiple intestinal hamartomas
Multiple trichilemmomas
89% risk of cancer at any site
16% risk of colorectal cancer Targeted individualised screening Breast cancer (81% risk)
Thyroid cancer and non toxic goitre
Uterine cancer
HNPCC (Lynch syndrome) Germline mutations of DNA mismatch repair genes Colo rectal cancer 30-70%
Endometrial cancer 30-70%
Gastric cancer 5-10%
Scanty colonic polyps may be present
Colonic tumours likely to be right sided and mucinous Colonoscopy every 1-2 years from age 25
Consideration of prophylactic surgery
Extra colonic surveillance recommended Extra colonic cancers
A 70 year old man undergoes an oesophagectomy and an oesophagogastric anastomosis is constructed. The arterial supply to the gastric component is mainly provided by which of these vessels?
Short gastric arteries
Left gastric artery
Right gastroepiploic artery
Left gastroepiploic artery
Common hepatic artery
The resection of the proximal stomach which is performed in these cases together with the gastric mobilisation required results in the division of all vessels except the right gastro-epiploic artery which then supplies the gastric remnant. It is this precarious supply that can result in conduit necrosis.
Stomach anatomy
The abdominal oesophagus enters the stomach through the cardiac orifice, it tends to lie around the level of T11 although this can vary considerably. The pyloric orifice tends to lie on a level of L1 (the transpyloric plane) although this, too, can vary.
Structurally, the stomach comprises fundic, body and pyloric regions. The body is comprises most of the gastric surface area. The anterior and posterior surfaces of the stomach unite at the greater and lesser curvatures. The peritoneum on the anterior and posterior surfaces of the stomach unites at the greater curvature to form the gastrosplenic ligament above and the anterior layer of the greater omentum below.
There are three muscular layers to the stomach wall; circular muscle fibres, longitudinal fibres and oblique fibres.
Blood supply
Left gastric artery
Short gastric vessels
Gastro-epiploic arteries
The veins draining the stomach accompany the arteries. The right and left gastric veins drain directly into the portal vein. The right gastro-epiploic vein either drains into the SMV or middle colic vein. The left gastro-epiploic and short gastric veins drain into the splenic vein.
Innervation
Sympathetic nerves arise from the coeliac ganglia with post ganglionic fibres accompanying the arteries.
Parasympathetic innervation is from the anterior and posterior vagal trunks
Which of the following is not linked to excess glucocorticoids?
Osteonecrosis
Osteoporosis
Hypokalaemia
Hyponatraemia
Growth retardation in children
D- Hyponatraemia
There are many adverse effects associated with excess glucocorticoids. Thinning of the skin, osteonecrosis and osteoporosis are all common. Steroids are associated with retention of sodium and water. Potassium loss may occur and hypokalaemic alkalosis has been reported.
Cortisol
Glucocorticoid
Released by zona fasiculata of the adrenal gland
90% protein bound; 10% active
Circadian rhythm: High in the mornings
Negative feedback via ACTH
Actions
Glycogenolysis
Gluconeogenesis
Protein catabolism
Lipolysis
Stress response
Anti-inflammatory
Decrease protein in bones
Increase gastric acid
Increases neutrophils/platelets/red blood cells
Inhibits fibroblastic activity
A 19 year old student is involved in a head on car collision. He complains of severe chest pain. A Chest x-ray performed as part of a trauma series shows widening of the mediastinum. Which is the most likely injury in this scenario?
Rupture of the distal oesophagus
Rupture of the left main bronchus
Rupture of the aorta proximal to the left subclavian artery
Rupture of the aorta distal to the left subclavian artery
Rupture of the inferior vena cava
The aorta may be injured in deceleration accidents. In the setting of deceleration injury, chest pain and mediastinal widening the most likely problem is aortic rupture. This will typically occur distal to the left subclavian artery. Rupture of the proximal aorta may occur. However, survival is unlikely. It is important to note that the question uses the term Most likely injury as this is the component that distinguishes an ascending rupture from a descending rupture.
Thoracic aorta rupture
Mechanism of injury: Decelerating force i.e. RTA, fall from a great height
Most people die at scene
Survivors may have an incomplete laceration at the ligamentum arteriosum of the aorta.
Clinical features
Contained haematoma: persistent hypotension
Detected mainly by history, CXR changes
CXR changes
Widened mediastinum
Trachea/Oesophagus to right
Depression of left main stem bronchus
Widened paratracheal stripe/paraspinal interfaces
Space between aorta and pulmonary artery obliterated
Rib fracture/left haemothorax
Diagnosis
Angiography, usually CT aortogram.
Treatment
Repair or replacement. Ideally they should undergo endovascular repair.
A 40 year old professional singer is admitted for a right thyroid lobectomy. Post operatively, she is unable to sing high notes. Which of the following muscles is likely to demonstrate impaired function?
Thyroarytenoid
Posterior cricoarytenoid
Cricothyroid
Thyrohyoid
Lateral cricoarytenoid
The most likely injury is to the superior laryngeal nerve which innervates the cricothyroid muscle. Since this tenses the vocal cords, singing high notes becomes a problem.
Voice production
There are 2 main nerves involved:
Superior laryngeal nerve (SLN)
Innervates the cricothyroid muscle
Since the cricothyroid muscle is involved in adjusting the tension of the vocal fold for high notes during singing, SLN paresis and paralysis result in:
a. Abnormalities in pitch
b. Inability to sing with smooth change to each higher note (glissando or pitch glide)
Recurrent laryngeal nerve (RLN)/Inferior laryngeal nerve
Innervates intrinsic larynx muscles
a. Opening vocal folds (as in breathing, coughing)
b. Closing vocal folds for vocal fold vibration during voice use
c. Closing vocal folds during swallowing
Which of these structures are linked by the ductus arteriosus?
Iliac vessels and inferior vena cava
Umbilical vein and inferior vena cava
Iliac arteries and umbilical arteries
Pulmonary artery and aorta
Pulmonary vein and inferior vena cava
The ductus arteriosus connects the pulmonary artery to the first part of the descending aorta. The ductus venosus connects the umbilical vein to the inferior vena cava and bypasses the liver. The volume transferred by this route decreases with gestation. Persistent patent ductus arteriosus is treated with NSAIDS in most cases.
Ductus arteriosus
The ductus arteriosus is formed from the left 6th aortic arch during embryonic development and attaches to the final part of the aortic arch (the isthmus of aorta) and the first part of the pulmonary artery.
It serves to allow blood in the developing foetus to bypass the lungs. It usually closes soon after birth. Should it not do so, it can be stimulated to close by administration of indomethacin.
Which of the following types of growth plate fractures may have similar radiological appearances?
Salter Harris types 1 and 5
Salter Harris types 4 and 5
Salter Harris types 3 and 5
Salter Harris types 1 and 2
Salter Harris types 1 and 3
Mnemonic: SALTER
S (Type 1): Straight through the growth plate
A (Type 2): Above - through growth plate and Above involving the metaphysis
L (Type 3): Lower -through growth plate and beLow involving the epiphysis
T (Type 4):Through - Through both metaphysis, epiphysis and growth plate
E (Type 5): Everything - Crush / compression injury
R (Type 5): Ruined
As recommended by one of our users
Salter Harris injury types 1 and 5 (transverse fracture through growth plate Vs. Compression fracture) may mimic each other radiologically. Type 5 injuries have the worst outcomes. Radiological signs of type 5 injuries are subtle and may include narrowing of the growth plate.
Epiphyseal fractures
Fractures involving the growth plate in children are classified using the Salter - Harris system.
There are 5 main types.
Salter Harris Classification
Type Description
Type 1 Transverse fracture through the growth plate
Type 2 Fracture through the growth plate to the metaphysis (commonest type)
Type 3 Fracture through the growth plate and the epiphysis with metaphysis spared
Type 4 Fracture involving the growth plate, metaphysis and epiphysis
Type 5 Compression fracture of the growth plate (worst outcome)
Management
Non displaced type 1 injuries can generally be managed conservatively. Unstable or more extensive injuries will usually require surgical reduction and/ or fixation, as proper alignment is crucial.
A 59 year old man is diagnosed as having carcinoma of the pancreas with two malignant deposits in the right lobe of the liver. What is the most appropriate treatment?
Palliative chemotherapy
Liver resection followed by chemotherapy
Simultaneous resection of liver metastasis and en bloc segmental pancreatic resection
Pancreatic resection followed by liver resection once recovered
Radical radiotherapy followed by surgery
Pancreatic cancer has a poor prognosis and most cases have metastatic disease at presentation. There is no role in pancreatic cancer for liver resection together with pancreatic surgery as there is no survival benefit. Most centres will offer palliative chemotherapy which has improved both longevity and quality of life.
Pancreatic cancer
- Adenocarcinoma
Risk factors: Smoking, diabetes, adenoma, familial adenomatous polyposis
Mainly occur in the head of the pancreas (70%)
Spread locally and metastasizes to the liver
Carcinoma of the pancreas should be differentiated from other periampullary tumours with better prognosis
Clinical features
Weight loss
Painless jaundice
Epigastric discomfort (pain usually due to invasion of the coeliac plexus is a late feature)
Pancreatitis
Trousseau’s sign: migratory superficial thrombophlebitis
Investigations
USS: May miss small lesions
CT Scanning (pancreatic protocol). If unresectable on CT then no further staging needed
PET/CT for those with operable disease on CT alone
ERCP/ MRI for bile duct assessment
Staging laparoscopy to exclude peritoneal disease
Management
Head of pancreas: Whipple’s resection (SE dumping and ulcers). Newer techniques include pylorus preservation and SMA/ SMV resection
Carcinoma body and tail: poor prognosis, distal pancreatectomy, if operable
Usually adjuvent chemotherapy for resectable disease
ERCP and stent for jaundice and palliation
Surgical bypass may be needed for duodenal obstruction
A 35-year-old female is admitted to hospital with hypovolaemic shock. CT abdomen reveals a haemorrhagic lesion in the right kidney. Following surgery and biopsy this is shown to be an angiomyolipomata. What is the most likely underlying diagnosis?
Neurofibromatosis
Budd-Chiari syndrome
Hereditary haemorrhagic telangiectasia
Von Hippel-Lindau syndrome
Tuberous sclerosis
Tuberous sclerosis
Tuberous sclerosis (TS) is a genetic condition of autosomal dominant inheritance. Like neurofibromatosis, the majority of features seen in TS are neuro-cutaneous
Cutaneous features
depigmented ‘ash-leaf’ spots which fluoresce under UV light
roughened patches of skin over lumbar spine (Shagreen patches)
adenoma sebaceum: butterfly distribution over nose
fibromata beneath nails (subungual fibromata)
café-au-lait spots* may be seen
Neurological features
developmental delay
epilepsy (infantile spasms or partial)
intellectual impairment
Also
retinal hamartomas: dense white areas on retina (phakomata)
rhabdomyomas of the heart
gliomatous changes can occur in the brain lesions
polycystic kidneys, renal angiomyolipomata
*these of course are more commonly associated with neurofibromatosis. However a 1998 study of 106 children with TS found café-au-lait spots in 28% of patients
Which of the following is the equivalent of cardiac preload?
End diastolic volume
Stroke volume
Systemic vascular resistance
Mean arterial pressure
Peak systolic arterial pressure
Preload is the same as end diastolic volume. When it is increased slightly there is an associated increase in cardiac output (Frank Starling principle). When it is markedly increased e.g. over 250ml then cardiac output falls.
Cardiac physiology
- The heart has four chambers ejecting blood into both low pressure and high pressure systems.
The pumps generate pressures of between 0-25mmHg on the right side and 0-120 mmHg on the left.
At rest diastole comprises 2/3 of the cardiac cycle.
The product of the frequency of heart rate and stroke volume combine to give the cardiac output which is typically 5-6L per minute.
Detailed descriptions of the various waveforms are often not a feature of MRCS A (although they are on the syllabus). However, they are a very popular topic for surgical physiology in the MRCS B exam.
Electrical properties
Intrinsic myogenic rhythm within cardiac myocytes means that even the denervated heart is capable of contraction.
In the normal situation the cardiac impulse is generated in the sino atrial node in the right atrium and conveyed to the ventricles via the atrioventricular node.
The sino atrial node is also capable of spontaneous discharge and in the absence of background vagal tone will typically discharge around 100x per minute. Hence the higher resting heart rate found in cardiac transplant cases. In the SA and AV nodes the resting membrane potential is lower than in surrounding cardiac cells and will slowly depolarise from -70mV to around -50mV at which point an action potential is generated.
Differences in the depolarisation slopes between SA and AV nodes help to explain why the SA node will depolarise first. The cells have a refractory period during which they cannot be re-stimulated and this period allows for adequate ventricular filling. In pathological tachycardic states this time period is overridden and inadequate ventricular filling may then occur, cardiac output falls and syncope may ensue.
Parasympathetic fibres project to the heart via the vagus and will release acetylcholine. Sympathetic fibres release nor adrenaline and circulating adrenaline comes from the adrenal medulla. Noradrenaline binds to β 1 receptors in the SA node and increases the rate of pacemaker potential depolarisation.
Mid diastole: AV valves open. Ventricles hold 80% of final volume. Outflow valves shut. Aortic pressure is high.
Late diastole: Atria contract. Ventricles receive 20% to complete filling. Typical end diastolic volume 130-160ml.
Early systole: AV valves shut. Ventricular pressure rises. Isovolumetric ventricular contraction. AV Valves bulge into atria (c-wave). Aortic and pulmonary pressure exceeded- blood is ejected. Shortening of ventricles pulls atria downwards and drops intra atrial pressure (x-descent).
Late systole: Ventricular muscles relax and ventricular pressures drop. Although ventricular pressure drops the aortic pressure remains constant owing to peripheral vascular resistance and elastic property of the aorta. Brief period of retrograde flow that occurs in aortic recoil shuts the aortic valve. Ventricles will contain 60ml end systolic volume. The average stroke volume is 70ml (i.e. Volume ejected).
Early diastole: All valves are closed. Isovolumetric ventricular relaxation occurs. Pressure wave associated with closure of the aortic valve increases aortic pressure. The pressure dip before this rise can be seen on arterial waveforms and is called the incisura. During systole the atrial pressure increases such that it is now above zero (v- wave). Eventually atrial pressure exceed ventricular pressure and AV valves open - atria empty passively into ventricles and atrial pressure falls (y -descent )
The negative atrial pressures are of clinical importance as they can allow air embolization to occur if the neck veins are exposed to air. This patient positioning is important in head and neck surgery to avoid this occurrence if veins are inadvertently cut, or during CVP line insertion.
Mechanical properties
Preload = end diastolic volume
Afterload = aortic pressure
It is important to understand the principles of Laplace’s law in surgery.
It states that for hollow organs with a circular cross section, the total circumferential wall tension depends upon the circumference of the wall, multiplied by the thickness of the wall and on the wall tension.
The total luminal pressure depends upon the cross sectional area of the lumen and the transmural pressure. Transmural pressure is the internal pressure minus external pressure and at equilibrium the total pressure must counterbalance each other.
In terms of cardiac physiology the law explains that the rise in ventricular pressure that occurs during the ejection phase is due to physical change in heart size. It also explains why a dilated diseased heart will have impaired systolic function.
Starlings law
Increase in end diastolic volume will produce larger stroke volume.
This occurs up to a point beyond which cardiac fibres are excessively stretched and stroke volume will fall once more. It is important for the regulation of cardiac output in cardiac transplant patients who need to increase their cardiac output.
Baroreceptor reflexes
Baroreceptors located in aortic arch and carotid sinus.
Aortic baroreceptor impulses travel via the vagus and from the carotid via the glossopharyngeal nerve.
They are stimulated by arterial stretch.
Even at normal blood pressures they are tonically active.
Increase in baroreceptor discharge causes:
*Increased parasympathetic discharge to the SA node.
*Decreased sympathetic discharge to ventricular muscle causing decreased contractility and fall in stroke volume.
*Decreased sympathetic discharge to venous system causing increased compliance.
*Decreased peripheral arterial vascular resistance
Atrial stretch receptors
Located in atria at junction between pulmonary veins and vena cava.
Stimulated by atrial stretch and are thus low pressure sensors.
Increased blood volume will cause increased parasympathetic activity.
Very rapid infusion of blood will result in increase in heart rate mediated via atrial receptors: the Bainbridge reflex.
Decreases in receptor stimulation results in increased sympathetic activity this will decrease renal blood flow-decreases GFR-decreases urinary sodium excretion-renin secretion by juxtaglomerular apparatus-Increase in angiotensin II.
Increased atrial stretch will also result in increased release of atrial natriuretic peptide.
Which of the following statements relating to the basilar artery and its branches is false?
The superior cerebellar artery may be decompressed to treat trigeminal neuralgia
Occlusion of the posterior cerebral artery causes contralateral loss of the visual field
The oculomotor nerve lies between the superior cerebellar and posterior cerebral arteries
The posterior inferior cerebellar artery is the largest of the cerebellar arteries arising from the basilar artery
The labyrinthine branch is accompanied by the facial nerve
The posterior inferior cerebellar artery is the largest of the cerebellar arteries arising from the vertebral artery. The labyrinthine artery is long and slender and may arise from the lower part of the basilar artery. It accompanies the facial and vestibulocochlear nerves into the internal auditory meatus. The posterior cerebral artery is often larger than the superior cerebellar artery and it is separated from the vessel, near it’s origin, by the oculomotor nerve. Arterial decompression is a well established therapy for trigeminal neuralgia.
Circle of Willis
The two internal carotid arteries and two vertebral arteries form an anastomosis known as the Circle of Willis on the inferior surface of the brain. Each half of the circle is formed by:
1. Anterior communicating artery
2. Anterior cerebral artery
3. Internal carotid artery
4. Posterior communicating artery
5. Posterior cerebral arteries and the termination of the basilar artery
The circle and its branches supply; the corpus striatum, internal capsule, diencephalon and midbrain.
Vertebral arteries
Enter the cranial cavity via foramen magnum
Lie in the subarachnoid space
Ascend on anterior surface of medulla oblongata
Unite to form the basilar artery at the base of the pons
Branches:
Posterior spinal artery
Anterior spinal artery
Posterior inferior cerebellar artery
Basilar artery
Branches:
Anterior inferior cerebellar artery
Labyrinthine artery
Pontine arteries
Superior cerebellar artery
Posterior cerebral artery (at the point where it bifurcates)
Internal carotid arteries
Branches:
Posterior communicating artery
Anterior cerebral artery
Middle cerebral artery
Anterior choroid artery
A patient has an appendicectomy and a 1.2cm carcinoid tumour is identified in the tip of the appendix. What is the most appropriate management?
Watchful waiting
Discharge
Right hemicolectomy
Limited ileocaecal resection
Radioisotope scan
Individuals with small carcinoids can be discharged (<2cm and limited to the appendix). Larger tumours should have a radioisotope scan. Where the resection margin is positive or where the isotope scan suggests lymphatic metastasis a right hemicolectomy should be performed.
Carcinoid syndrome
Carcinoid tumours secrete serotonin
Originate in neuroendocrine cells mainly in the intestine (midgut-distal ileum/appendix)
Can occur in the rectum, bronchi
Hormonal symptoms mainly occur when disease spreads outside the bowel
Clinical features
Onset: insidious over many years
Flushing face
Palpitations
Pulmonary valve stenosis and tricuspid regurgitation causing dyspnoea
Asthma
Severe diarrhoea (secretory, persists despite fasting)
Investigation
5-HIAA in a 24-hour urine collection
Somatostatin receptor scintigraphy
CT scan
Blood testing for chromogranin A
Treatment
Octreotide
Surgical removal
Which of the nerves listed below provides sensory innervation to the skin overlying the lateral aspect of the nose?
Infratrochlear nerve
Zygomatic nerve
Nasopalatine nerve
Lateral nasal branches of the ethmoidal nerve
Frontal nerve
The lateral aspect of the external nose is innervated by lateral nasal branches of the anterior ethmoidal nerve. The ethmoidal nerve is a branch of the nasociliary nerve which is one of the divisions of the trigeminal.
Trigeminal nerve
The trigeminal nerve is the main sensory nerve of the head. In addition to its major sensory role, it also innervates the muscles of mastication.
Distribution of the trigeminal nerve
Sensory
Scalp
Face
Oral cavity (and teeth)
Nose and sinuses
Dura mater
Motor
Muscles of mastication
Mylohyoid
Anterior belly of digastric
Tensor tympani
Tensor palati
Autonomic connections (ganglia)
Ciliary
Sphenopalatine
Otic
Submandibular
Path
Originates at the pons
Sensory root forms the large, crescentic trigeminal ganglion within Meckel’s cave, and contains the cell bodies of incoming sensory nerve fibres. Here the 3 branches exit.
The motor root cell bodies are in the pons and the motor fibres are distributed via the mandibular nerve. The motor root is not part of the trigeminal ganglion.
Branches of the trigeminal nerve
Ophthalmic nerve Sensory only
Maxillary nerve Sensory only
Mandibular nerve Sensory and motor
Sensory
Ophthalmic Exits skull via the superior orbital fissure
Sensation of: scalp and forehead, the upper eyelid, the conjunctiva and cornea of the eye, the nose (including the tip of the nose, except alae nasi), the nasal mucosa, the frontal sinuses, and parts of the meninges (the dura and blood vessels).
Maxillary nerve Exit skull via the foramen rotundum
Sensation: lower eyelid and cheek, the nares and upper lip, the upper teeth and gums, the nasal mucosa, the palate and roof of the pharynx, the maxillary, ethmoid and sphenoid sinuses, and parts of the meninges.
Mandibular nerve Exit skull via the foramen ovale
Sensation: lower lip, the lower teeth and gums, the chin and jaw (except the angle of the jaw), parts of the external ear, and parts of the meninges.
Motor
Distributed via the mandibular nerve.
The following muscles of mastication are innervated:
Masseter
Temporalis
Medial pterygoid
Lateral pterygoid
Other muscles innervated include:
Tensor veli palatini
Mylohyoid
Anterior belly of digastric
Tensor tympani
A 52 year old male attends renal transplant clinic for a post operative assessment. You note that he is on ciclosporin and that a recent blood test shows that the ciclosporin level is elevated. Of the adverse effects noted below, which is the greatest concern?
Hyperthyroidism
Diabetes
Alopecia
Hypothermia
Nephrotoxicity
Ciclosporin- nephrotoxicity
This patient is at risk of nephrotoxicity and should be referred to the renal team as soon as possible. Alopecia is associated with azathioprine and diabetes is associated with long term use.
Organ transplantation: immunosuppressants
A number of drugs are available which help to mitigate the processes resulting in acute rejection. Cyclosporin and tacrolimus are commonly used drugs.
Example regime
Initial: ciclosporin/tacrolimus with a monoclonal antibody
Maintenance: ciclosporin/tacrolimus with MMF or sirolimus
Add steroids if more than one steroid responsive acute rejection episode
Ciclosporin
Inhibits calcineurin, a phosphatase involved in T cell activation
Nephrotoxic
Monitor levels
Azathioprine
Metabolised to form 6 mercaptopurine which inhibits DNA synthesis and cell division
Side effects include myelosupression, alopecia and nausea
Tacrolimus
Lower incidence of acute rejection compared to ciclosporin
Also less hypertension and hyperlipidaemia
However, high incidence of impaired glucose tolerance and diabetes
Mycophenolate mofetil (MMF)
Blocks purine synthesis by inhibition of IMPDH
Therefore inhibits proliferation of B and T cells
Side-effects: GI and marrow suppression
Sirolimus (rapamycin)
Blocks T cell proliferation by blocking the IL-2 receptor
Can cause hyperlipidaemia
Monoclonal antibodies
Selective inhibitors of IL-2 receptor
Daclizumab
Basilximab
A 22 year man is shot in the back, in the lumbar region. He has increased tone and hyper-reflexia of his right leg. He cannot feel his left leg. What is the most likely explanation?
Epidural haematoma
Osteomyelitis
Transverse myelitis
Brown-Sequard syndrome
Tabes dorsalis
Brown -Sequard syndrome is caused by hemisection of the spinal cord. It may result from stab injuries or lateral vertebral fractures. It results in ipsilateral paralysis (pyramidal tract) , and also loss of proprioception and fine discrimination (dorsal columns). Pain and temperature sensation are lost on the contra-lateral side. This is because the fibres of the spinothalamic tract have decussated below the level of the cord transection.
Spinal disorders
Dorsal column lesion
Loss vibration and proprioception
Tabes dorsalis, SACD
Spinothalamic tract lesion
Loss of pain, sensation and temperature
Central cord lesion
Flaccid paralysis of the upper limbs
Osteomyelitis
Normally progressive
Staph aureus in IVDU, normally cervical region affected
Fungal infections in immunocompromised
Thoracic region affected in TB
Infarction spinal cord
Dorsal column signs (loss of proprioception and fine discrimination)
Cord compression
UMN signs
Malignancy
Haematoma
Fracture
Brown-sequard syndrome
Hemisection of the spinal cord
Ipsilateral paralysis
Ipsilateral loss of proprioception and fine discrimination
Contralateral loss of pain and temperature
Dermatomes
C2 to C4 The C2 dermatome covers the occiput and the top part of the neck. C3 covers the lower part of the neck to the clavicle. C4 covers the area just below the clavicle.
C5 to T1 Situated in the arms. C5 covers the lateral arm at and above the elbow. C6 covers the forearm and the radial (thumb) side of the hand. C7 is the middle finger, C8 is the medial aspect of the hand, and T1 covers the medial side of the forearm.
T2 to T12 The thoracic covers the axillary and chest region. T3 to T12 covers the chest and back to the hip girdle. The nipples are situated in the middle of T4. T10 is situated at the umbilicus. T12 ends just above the hip girdle.
L1 to L5 The cutaneous dermatome representing the hip girdle and groin area is innervated by L1 spinal cord. L2 and 3 cover the front part of the thighs. L4 and L5 cover medial and lateral aspects of the lower leg.
S1 to S5 S1 covers the heel and the middle back of the leg. S2 covers the back of the thighs. S3 cover the medial side of the buttocks and S4-5 covers the perineal region. S5 is of course the lowest dermatome and represents the skin immediately at and adjacent to the anus.
Myotomes
Upper limb
Elbow flexors/Biceps C5
Wrist extensors C6
Elbow extensors/Triceps C7
Long finger flexors C8
Small finger abductors T1
Lower limb
Hip flexors (psoas) L1 and L2
Knee extensors (quadriceps) L3
Ankle dorsiflexors (tibialis anterior) L4 and L5
Toe extensors (hallucis longus) L 5
Ankle plantar flexors (gastrocnemius) S1
The anal sphincter is innervated by S2,3,4
Which of the following statements relating to sternocleidomastoid is untrue?
The external jugular vein lies posteromedially.
It is supplied by the accessory nerve.
It has two heads of origin
It inserts into the lateral aspect of the mastoid process.
It marks the anterior border of the posterior triangle.
The external jugular vein lies lateral (i.e. superficial) to the sternocleidomastoid.
Sternocleidomastoid
Anatomy
Origin Rounded tendon attached to upper manubrium sterni and muscular head attached to medial third of the clavicle
Insertion Mastoid process of the temporal bone and lateral area of the superior nuchal line of the occipital bone
Innervation Spinal part of accessory nerve and anterior rami of C2 and C3 (proprioception)
Action
Both: extend the head at atlanto-occipital joint and flex the cervical vertebral column. Accessory muscles of inspiration.
Single: lateral flexion of neck, rotates head so face looks upward to the opposite side
Sternocleidomastoid divides the anterior and posterior triangles of the neck.
A motor cyclist is involved in a road traffic accident causing severe right shoulder injuries. He is found to have an adducted, medially rotated shoulder. The elbow is fully extended and the forearm pronated. Which is the most likely diagnosis?
C8, T1 root lesion
C5, C6 root lesion
Radial nerve lesion
Ulnar nerve lesion
Axillary nerve lesion
Erbs Palsy C5, C6 lesion
The features include:
Waiter’s tip position
Loss of shoulder abduction (deltoid and supraspinatus paralysis)
Loss of external rotation of the shoulder (paralysis of infraspinatus)
Loss of elbow flexion (paralysis of biceps, brachialis and brachioradialis)
Loss of forearm supination (paralysis of Biceps)
The motorcyclist has had an Erb’s palsy (C5, C6 root lesion). This is commonly known to be associated with birth injury when a baby has a shoulder dystocia.
Brachial plexus
The brachial plexus extends from the neck to the axilla. It is formed by the ventral rami of the fifth to the eighth cervical nerves with the ascending part of the first thoracic nerve.
Location of the plexus
The ventral rami which form the plexus enter the lower part of the posterior triangle of the neck in series with the ventral rami of the cervical plexus. The second part of the subclavian artery lies immediately anterior to the lower two rami. The upper three rami intermingle and pass inferolaterally towards the axilla and subclavian artery. They are enclosed within an extension of the prevertebral fascia. In the neck the plexus lies deep to platysma, the supraclavicular nerves, inferior belly of omohyoid and the transverse cervical artery. It then passes deep to the clavicle and the suprascapular vessels, to enter the axilla, and thence surround the second part of the axillary artery
Composition of the plexus
Ventral rami, the roots of the plexus, lie between scalenus medius and anterior.
As they enter the posterior triangle, the upper two (C5,6) and lower two (C8, T1) roots of the plexus unite to form the upper and lower trunks of the plexus respectively. Meanwhile, C7 continues as the middle trunk. The lower trunk may groove the superior surface of the first rib posterior to the subclavian artery, and the root from the first ventral ramus is always in contact with it.
Each trunk divides into ventral and dorsal divisions which are destined to supply the anterior (flexor) and posterior (extensor) parts of the upper limb.
The cords of the plexus are formed in the axilla. The dorsal divisions unite to form the posterior cord (C5-8). The ventral divisions of the upper and middle trunks unite to form the lateral cord (C5-7), while the ventral divisions of the lower trunk continues as the medial cord (C8-T1). The cords are named according to their relationship to the axillary artery. Each cord terminates by dividing into two main branches at the beginning of the third part of the artery.
Sympathetic communications
The fifth and sixth cervical ventral rami receive grey rami communicantes from the middle cervical ganglion, while the two or more grey rami communicantes pass from the inferior cervical ganglion to the seventh and eighth cervical ventral rami. The first thoracic ventral ramus receives its grey ramus from the cervicothoracic ganglion. Its for this reason that inferior plexus injury can be complicated by a Horners syndrome.
Summary
Origin Anterior rami of C5 to T1
Sections of the plexus
Roots, trunks, divisions, cords, branches
Mnemonic:Real Teenagers Drink Cold Beer
Roots
Located in the posterior triangle
Pass between scalenus anterior and medius
Trunks
Located posterior to middle third of clavicle
Upper and middle trunks related superiorly to the subclavian artery
Lower trunk passes over 1st rib posterior to the subclavian artery
Divisions Apex of axilla
Cords Related to axillary artery
A 55 year old man presents with symptoms of dyspepsia and on upper GI endoscopy an area of patchy erythematous tissue is identified extending proximally from the gastro oesophageal junction. A biopsy is diagnostic of Barretts oesophagus with low grade dysplasia. Which of the following is the most appropriate next step?
Distal oesophagectomy
Upper GI endoscopy with quadrantic biopsies from the region
Photodynamic therapy
Endoscopic sub mucosal resection of the area
Argon plasma coagulation
In Barrett’s surveillance the safest option is quadrantic (i.e. 4 biopsies, one from each quarter of the oesophagus at 2cm intervals)
Low grade dysplasia in conjunction with Barretts oesphagus should be monitored with regular (6 monthly) upper GI endoscopy and quadrantic biopsies. If the disease remains static at 2 years then the screening frequency may be decreased.
Barrett’s oesophagus
Barretts oesophagus is a condition characterised by the metaplastic transformation of squamous oesophageal epithelium to columnar gastric type epithelium. Three types of this metaplastic process are recognised; intestinal (high risk), cardiac and fundic. The latter two categories may cause difficulties in diagnosis. The most concrete diagnosis can be made when endoscopic features of Barretts oesophagus are present together with a deep biopsy that demonstrates not just goblet cell metaplasia but also oesophageal glands.
Barrett’s can be sub divided into short (<3cm) and long (>3cm). The length of the affected segment correlates strongly with the chances of identifying metaplasia. The overall prevalence of Barrett’s oesophagus is difficult to determine but may be in the region of 1 in 20 and is identified in up to 12% of those undergoing endoscopy for reflux.
A proportion of patients with metaplasia will progress to dysplasia and for this reason individuals identified as having Barrett’s should undergo endoscopic surveillance (every 2-5 years). Biopsies should be quadrantic and taken at 2-3cm intervals. Biopsies need to be adequate. Where mass lesions are present consideration should be given to endoscopic sub mucosal resection. Up to 40% of patients will be upstaged from high grade dysplasia to invasive malignancy with such techniques.
Treatment
Long term proton pump inhibitor
Consider pH and manometry studies in younger patients who may prefer to consider an anti reflux procedure
Regular endoscopic monitoring (more frequently if moderate dysplasia). With quadrantic biopsies every 2-3 cm
If severe dysplasia be very wary of small foci of cancer
References
A consensus statement of the British approach is provided by:
Bennett C et al Consensus Statements for Management of Barrett’s Dysplasia and Early-Stage Esophageal Adenocarcinoma, Based on a Delphi Process. Gastroenterology Volume 143, Issue 2 , Pages 336-346, August 2012.
A 74 year old gentleman presents with an obstructing carcinoma of the splenic flexure. Attempts at placement of a colonic stent have failed. Which of the operative options listed below offers the best solution to this problem?
Sub total colectomy
Extended right hemicolectomy
Standard right hemicolectomy
Standard left hemicolectomy
Transverse colectomy
Standard right hemicolectomy involves colonic division to the right of the middle colic vessels
Extended right hemicolectomy involves division of the middle colic vessels and usually resection of the splenic flexure as well.
The question always causes confusion and to understand it the information needs to be carefully read. Firstly, the tumour is definitely at the splenic flexure and the second point is that the operation is definitely an extended right hemicolectomy. A left hemicolectomy or even the older operation of a transverse colectomy could be considered if the patient was not obstructed. However, when obstruction is present, an extended right hemicolectomy (which involves an ileocolic anastomosis) is relatively safe even in the obstructed setting.
Colorectal cancer treatment
Patients diagnosed as having colorectal cancer should be completely staged using CT of the chest/ abdomen and pelvis. Their entire colon should have been evaluated with colonoscopy or CT colonography. Patients whose tumours lie below the peritoneal reflection should have their mesorectum evaluated with MRI.
Once their staging is complete patients should be discussed within a dedicated colorectal MDT meeting and a treatment plan formulated.
Treatment of colonic cancer
Cancer of the colon is nearly always treated with surgery. Stents, surgical bypass and diversion stomas may all be used as palliative adjuncts. Resectional surgery is the only option for cure in patients with colon cancer. The procedure is tailored to the patient and the tumour location. The lymphatic drainage of the colon follows the arterial supply and therefore most resections are tailored around the resection of particular lymphatic chains (e.g. ileo-colic pedicle for right sided tumours). Some patients may have confounding factors that will govern the choice of procedure, for example a tumour in a patient from a HNPCC family may be better served with a panproctocolectomy rather than segmental resection. Following resection the decision has to be made regarding restoration of continuity. For an anastomosis to heal the key technical factors include; adequate blood supply, mucosal apposition and no tissue tension. Surrounding sepsis, unstable patients and inexperienced surgeons may compromise these key principles and in such circumstances it may be safer to construct an end stoma rather than attempting an anastomosis.
When a colonic cancer presents with an obstructing lesion; the options are to either stent it or resect. In modern practice it is unusual to simply defunction a colonic tumour with a proximal loop stoma. This differs from the situation in the rectum (see below).
Following resection patients with risk factors for disease recurrence are usually offered chemotherapy, a combination of 5FU and oxaliplatin is common.
Rectal cancer
The management of rectal cancer is slightly different to that of colonic cancer. This reflects the rectum’s anatomical location and the challenges posed as a result. Tumours located in the rectum can be surgically resected with either an anterior resection or an abdomino - perineal resection. The technical aspects governing the choice between these two procedures can be complex to appreciate and the main point to appreciate for the MRCS is that involvement of the sphincter complex or very low tumours require APER. In the rectum a 2cm distal clearance margin is required and this may also impact on the procedure chosen. In addition to excision of the rectal tube an integral part of the procedure is a meticulous dissection of the mesorectal fat and lymph nodes (total mesorectal excision/ TME). In rectal cancer surgery invovlement of the cirumferential resection margin carries a high risk of disease recurrence. Because the rectum is an extraperitoneal structure (until you remove it that is!) it is possible to irradiate it, something which cannot be offered for colonic tumours. This has a major impact in rectal cancer treatment and many patients will be offered neoadjuvent radiotherapy (both long and short course) prior to resectional surgery. Patients with T1, 2 and 3 /N0 disease on imaging do not require irradiation and should proceed straight to surgery. Patients with T4 disease will typically have long course chemo radiotherapy. Patients presenting with large bowel obstruction from rectal cancer should not undergo resectional surgery without staging as primary treatment (very different from colonic cancer). This is because rectal surgery is more technically demanding, the anastomotic leak rate is higher and the danger of a positive resection margin in an unstaged patient is high. Therefore patients with obstructing rectal cancer should have a defunctioning loop colostomy.
Summary of procedures
The operations for cancer are segmental resections based on blood supply and lymphatic drainage. These commonly performed procedures are core knowledge for the MRCS and should be understood.
Site of cancer Type of resection Anastomosis Risk of leak
Right colon Right hemicolectomy Ileo-colic Low <5%
Transverse Extended right hemicolectomy Ileo-colic Low <5%
Splenic flexure Extended right hemicolectomy Ileo-colic Low <5%
Splenic flexure Left hemicolectomy Colo-colon 2-5%
Left colon Left hemicolectomy Colo-colon 2-5%
Sigmoid colon High anterior resection Colo-rectal 5%
Upper rectum Anterior resection (TME) Colo-rectal 5%
Low rectum Anterior resection (Low TME) Colo-rectal
(+/- Defunctioning stoma) 10%
Anal verge Abdomino-perineal excision of colon and rectum None n/a
In the emergency setting, where the bowel has perforated, the risk of an anastomotic breakdown is much greater, particularly when the anastomosis is colon-colon. In this situation, an end colostomy is often safer and can be reversed later. When resection of the sigmoid colon is performed and an end colostomy is fashioned the operation is referred to as a Hartmans procedure. Whilst left sided resections are more risky, ileo-colic anastomoses are relatively safe even in the emergency setting and do not need to be defunctioned.
References
A review of the diagnosis and management of colorectal cancer and a summary of the UK National Institute of Clinical Excellence guidelines is provided in:
Poston G, et al . Diagnosis and management of colorectal cancer: summary of NICE guidance. BMJ 2011: 343: d 6751.
Which statement is false about the foramina of the skull?
The hypoglossal canal transmits the hypoglossal nerve
The foramen spinosum is at the base of the medial pterygoid plate.
The jugular foramen transmits the accessory nerve
The foramen lacerum is located between the sphenoid and temporal bones
The stylomastoid foramen transmits the facial nerve
Foramina of the base of the skull
Foramen Location Contents
Foramen ovale Sphenoid bone Otic ganglion
V3 (Mandibular nerve:3rd branch of
trigeminal)
Accessory meningeal artery
Lesser petrosal nerve
Emissary veins
Foramen spinosum Sphenoid bone Middle meningeal artery
Meningeal branch of the Mandibular nerve
Foramen rotundum Sphenoid bone Maxillary nerve (V2)
Foramen lacerum/ carotid canal Located between the sphenoid, the apex of the petrous temporal and the basilar part of the occipital Base of the medial pterygoid plate.
Internal carotid artery*
Nerve and artery of the pterygoid canal
Jugular foramen Temporal bone Anterior: inferior petrosal sinus
Intermediate: glossopharyngeal, vagus, and accessory nerves.
Posterior: sigmoid sinus (becoming the internal jugular vein) and some meningeal branches from the occipital and ascending pharyngeal arteries.
Foramen magnum Occipital bone Anterior and posterior spinal arteries
Vertebral arteries
Medulla oblongata
Stylomastoid foramen Temporal bone Stylomastoid artery
Facial nerve
Superior orbital fissure Sphenoid bone Oculomotor nerve (III)
Recurrent meningeal artery
Trochlear nerve (IV)
Lacrimal, frontal and nasociliary branches of opthalmic nerve (V1)
Abducent nerve (VI)
Superior ophthalmic vein
*= In life the foramen lacerum is occluded by a cartilagenous plug. The ICA initially passes into the carotid canal which ascends superomedially to enter the cranial cavity through the foramen lacerum.
Which nerve directly innervates the sinoatrial node?
Superior cardiac nerve
Right vagus nerve
Left vagus nerve
Inferior cardiac nerve
None of the above
No single one of the above nerves is responsible for direct cardiac innervation (which those who have handled the heart surgically will appreciate).
The heart receives its nerves from the superficial and deep cardiac plexuses. The cardiac plexuses send small branches to the heart along the major vessels, continuing with the right and left coronary arteries. The vagal efferent fibres emerge from the brainstem in the roots of the vagus and accessory nerves, and run to ganglia in the cardiac plexuses and within the heart itself.
The background vagal discharge serves to limit heart rate, and loss of this background vagal tone accounts for the higher resting heart rate seen following cardiac transplant.
Sinoatrial node
Located in the wall of the right atrium in the upper part of the sulcus terminalis from which it extends anteriorly over the opening of the superior vena cava.
In most cases it is supplied by the right coronary artery.
It has a complicated nerve supply from the cardiac nerve plexus that takes both sympathetic and parasympathetic fibres that run alongside the main vessels.
A 22 year old lady receives intravenous morphine for acute abdominal pain. Which of the following best accounts for its analgesic properties?
Binding to δ opioid receptors in the brainstem
Binding to δ opioid receptors at peripheral nerve sites
Binding to β opioid receptors within the CNS
Binding to α opioid receptors within the CNS
Binding to µ opioid receptors within the CNS
Binding to µ opioid receptors within the CNS
4 Types of opioid receptor:
δ (located in CNS)- Accounts for analgesic and antidepressant effects
k (mainly CNS)- analgesic and dissociative effects
µ (central and peripheral) - causes analgesia, miosis, decreased gut motility
Nociceptin receptor (CNS)- Affect of appetite and tolerance to µ agonists.
Morphine
Strong opiate analgesic. It is a pro- type narcotic drug and its effects mediated via the 4 types of opioid receptor. Its clinical effects stem from binding to these receptor sites within the CNS and gastrointestinal tract. Unwanted side effects include nausea, constipation, respiratory depression and, if used long term, addiction .
It may be administered orally or intravenously. It can be reversed with naloxone.
A 22 year old intravenous drug user is found to have a femoral abscess. The nursing staff contact the on call doctor as the patient has a temperature of 39oC. He is found to have a pan systolic murmur loudest at the left sternal edge at the 4th intercostal space. What is the most likely underlying lesion?
Aortic regurgitation
Mitral regurgitation
Aortic stenosis
Tricuspid stenosis
Tricuspid regurgitation
Intravenous drug users are at high risk of right sided cardiac valvular endocarditis. The character of the murmur fits with a diagnosis of tricuspid valve endocarditis.
Cardiac murmurs
Type of Murmur Conditions
Ejection systolic Aortic stenosis
Pulmonary stenosis, HOCM
ASD, Fallot’s
Pan-systolic Mitral regurgitation
Tricuspid regurgitation
VSD
Late systolic Mitral valve prolapse
Coarctation of aorta
Early diastolic Aortic regurgitation
Graham-Steel murmur (pulmonary regurgitation)
Mid diastolic Mitral stenosis
Austin-Flint murmur (severe aortic regurgitation)
A 73 year old lady is undergoing surgery to remove the middle lobe of the right lung. Which of these sites should the incision be centered on to gain access to the apex of the right middle lobe?
6th interspace in the mid axillary line
4th interspace in the mid clavicular line
6th interspace posterior to the mid axillary line
4th interspace posterior to the mid axillary line
3rd interspace in the mid axillary line
The right middle lobe is a wedge shaped structure. Its apex is posterior and it lies deep to the 6th rib in the mid axillary line. Anteriorly, it is in contact with the 5th, 6th and 7th costal cartiladges.
Lung anatomy
The right lung is composed of 3 lobes divided by the oblique and transverse fissures. The left lung has two lobes divided by the oblique fissure.The apex of both lungs is approximately 4cm superior to the sterno-costal joint of the first rib. Immediately below this is a sulcus created by the subclavian artery.
Peripheral contact points of the lung
Base: diaphragm
Costal surface: corresponds to the cavity of the chest
Mediastinal surface: Contacts the mediastinal pleura. Has the cardiac impression. Above and behind this concavity is a triangular depression named the hilum, where the structures which form the root of the lung enter and leave the viscus. These structures are invested by pleura, which, below the hilum and behind the pericardial impression, forms the pulmonary ligament
Right lung
Above the hilum is the azygos vein; Superior to this is the groove for the superior vena cava and right innominate vein; behind this, and nearer the apex, is a furrow for the innominate artery. Behind the hilum and the attachment of the pulmonary ligament is a vertical groove for the oesophagus; In front and to the right of the lower part of the oesophageal groove is a deep concavity for the extrapericardiac portion of the inferior vena cava.
The root of the right lung lies behind the superior vena cava and the right atrium, and below the azygos vein.
The right main bronchus is shorter, wider and more vertical than the left main bronchus and therefore the route taken by most foreign bodies.
Left lung
Above the hilum is the furrow produced by the aortic arch, and then superiorly the groove accommodating the left subclavian artery; Behind the hilum and pulmonary ligament is a vertical groove produced by the descending aorta, and in front of this, near the base of the lung, is the lower part of the oesophagus.
The root of the left lung passes under the aortic arch and in front of the descending aorta.
Inferior borders of both lungs
6th rib in mid clavicular line
8th rib in mid axillary line
10th rib posteriorly
The pleura runs two ribs lower than the corresponding lung level.
Bronchopulmonary segments
Segment number Right lung Left lung
1 Apical Apical
2 Posterior Posterior
3 Anterior Anterior
4 Lateral Superior lingular
5 Medial Inferior lingular
6 Superior (apical) Superior (apical)
7 Medial basal Medial basal
8 Anterior basal Anterior basal
9 Lateral basal Lateral basal
10 Posterior basal Posterior basal
A 19 year old man is playing rugby when he suddenly notices a severe pain at the posterolateral aspect of his right thigh. Which of the following muscle groups is most likely to have been injured?
Semimembranosus
Semitendinosus
Long head of biceps femoris
Gastrocnemius
Soleus
The biceps femoris is the laterally located hamstring muscle. The semitendinosus and semimembranosus are located medially. Rupture of gastrocnemius and soleus may occur but is less common.
Biceps femoris
The biceps femoris is one of the hamstring group of muscles located in the posterior upper thigh. It has two heads.
Long head
Origin Ischial tuberosity
Insertion Fibular head
Action Knee flexion, lateral rotation tibia, extension hip
Innervation Tibial division of sciatic nerve (L5, S1, S2)
Arterial supply Profunda femoris artery, inferior gluteal artery, and the superior muscular branches of popliteal artery
Short head
Origin Lateral lip of linea aspera, lateral supracondylar ridge of femur
Insertion Fibular head
Action Knee flexion, lateral rotation tibia
Innervation Common peroneal division of sciatic nerve (L5, S1, S2)
Arterial supply Profunda femoris artery, inferior gluteal artery, and the superior muscular branches of popliteal artery
Which of the following features are not typical of Crohns disease?
Complex fistula in ano
Small bowel strictures
Skip lesions
‘Rose thorn ulcers’ on barium studies
Pseudopolyps on colonoscopy
Pseudopolyps are a feature of ulcerative colitis and occur when there is severe mucosal ulceration. The remaining islands of mucosa may then appear to be isolated and almost polypoidal.
Crohns disease
Crohns disease is a chronic transmural inflammation of a segment(s) of the gastrointestinal tract and may be associated with extra intestinal manifestations. Frequent disease patterns observed include ileal, ileocolic and colonic disease. Peri-anal disease may occur in association with any of these. The disease is often discontinuous in its distribution. Inflammation may cause ulceration, fissures, fistulas and fibrosis with stricturing. Histology reveals a chronic inflammatory infiltrate that is usually patchy and transmural.
Ulcerative colitis Vs Crohns
Crohn’s disease Ulcerative colitis
Distribution Mouth to anus Rectum and colon
Macroscopic changes Cobblestone appearance, apthoid ulceration Contact bleeding
Depth of disease Transmural inflammation Superficial inflammation
Distribution pattern Patchy Continuous
Histological features Granulomas (non caseating epithelioid cell aggregates with Langhans’ giant cells) Crypt abscesses, Inflammatory cells in the lamina propria
Extraintestinal manifestations of Crohns
Related to disease extent Unrelated to disease extent
Aphthous ulcers (10%) Sacroiliiitis (10-15%)
Erythema nodosum (5-10%) Ankylosing spondylitis (1-2%)
Pyoderma gangrenosum (0.5%) Primary sclerosing cholangitis (Rare)
Acute arthropathy (6-12%) Gallstones (up to 30%)
Ocular complications (up to 10%) Renal calculi (up to 10%)
Diarrhoea in Crohns
Diarrhoea in Crohns may be multifactorial since actual inflammation of the colon is not common. Causes therefore include the following:
Bile salt diarrhoea secondary to terminal ileal disease
Entero-colic fistula
Short bowel due to multiple resections
Bacterial overgrowth
Surgical interventions in Crohns disease
The commonest disease pattern in Crohns is stricturing terminal ileal disease and this often culminates in an ileocaecal resection. Other procedures performed include segmental small bowel resections and stricturoplasty. Colonic involvement in patients with Crohns is not common and, where found, distribution is often segmental. However, despite this distribution segmental resections of the colon in patients with Crohns disease are generally not advocated because the recurrence rate in the remaining colon is extremely high. As a result, the standard options of colonic surgery in Crohns patients are generally; sub total colectomy, panproctocolectomy and staged sub total colectomy and proctectomy. Restorative procedures such as ileoanal pouch have no role in therapy.
Crohns disease is notorious for the developmental of intestinal fistulae; these may form between the rectum and skin (peri anal) or the small bowel and skin. Fistulation between loops of bowel may also occur and result in bacterial overgrowth and malabsorption. Management of enterocutaneous fistulae involves controlling sepsis, optimising nutrition, imaging the disease and planning definitive surgical management.
During a right hemicolectomy the caecum is mobilised. As the bowel is retracted medially a vessel is injured, posterior to the colon. Which of the following is the most likely vessel?
Right colic artery
Inferior vena cava
Aorta
External iliac artery
Gonadal vessels
The key in this question is that its during the caecal mobilization. The gonadal vessels and ureter are important posterior relations that are at risk during a right hemicolectomy. During latter stages of the procedure, the ileocolic artery and vein are traced along the anterior aspect of the duodenum. At this point it is possible to injure these, the superior mesenteric vein or the middle colic vein, injury to any of these can result in torrential bleeding that is very difficult to control.
Caecum
Location
Proximal right colon below the ileocaecal valve
Intraperitoneal
Posterior relations
Psoas
Iliacus
Femoral nerve
Genitofemoral nerve
Gonadal vessels
Anterior relations Greater omentum
Arterial supply Ileocolic artery
Lymphatic drainage Mesenteric nodes accompany the venous drainage
The caecum is the most distensible part of the colon and in complete large bowel obstruction with a competent ileocaecal valve the most likely site of eventual perforation.
The oxygen-haemoglobin dissociation curve is shifted to the left in:
With decreased 2,3-DPG in transfused red cells
Respiratory acidosis
Sudden move to high altitude
Pyrexia
Haemolytic anaemia
With decreased 2,3-DPG in transfused red cells
S shaped curve
The curve is shifted to the left when there is a decreased oxygen requirement by the tissue. This includes:
1. Hypothermia
2. Alkalosis
3. Reduced levels of DPG:
DPG is found in erythrocytes and is reduced in non exercising muscles, i.e. when there is reduced glycolysis.
4. Polycythaemia
Oxygen Transport
Oxygen transport
Almost all oxygen is transported within erythrocytes. It has limited solubility and only 1% is carried as solution. Therefore the amount of oxygen transported will depend upon haemoglobin concentration and its degree of saturation.
Haemoglobin
Globular protein composed of 4 subunits. Haem consists of a protoporphyrin ring surrounding an iron atom in its ferrous state. The iron can form two additional bonds; one with oxygen and the other with a polypeptide chain. There are two alpha and two beta subunits to this polypeptide chain in an adult and together these form globin. Globin cannot bind oxygen but is able to bind to carbon dioxide and hydrogen ions, the beta chains are able to bind to 2,3 diphosphoglycerate. The oxygenation of haemoglobin is a reversible reaction. The molecular shape of haemoglobin is such that binding of one oxygen molecule facilitates the binding of subsequent molecules.
Oxygen dissociation curve
The oxygen dissociation curve describes the relationship between the percentage of saturated haemoglobin and partial pressure of oxygen in the blood. It is not affected by haemoglobin concentration.
Chronic anaemia causes 2, 3 DPG levels to increase, hence shifting the curve to the right
Haldane effect
Shifts to left = for given oxygen tension there is increased saturation of Hb with oxygen i.e. Decreased oxygen delivery to tissues
Bohr effect
Shifts to right = for given oxygen tension there is reduced saturation of Hb with oxygen i.e. Enhanced oxygen delivery to tissues
Shifts to Left = Lower oxygen delivery
HbF, methaemoglobin, carboxyhaemoglobin
low [H+] (alkali)
low pCO2
low 2,3-DPG
low temperature
Shifts to Right = Raised oxygen delivery
raised [H+] (acidic)
raised pCO2
raised 2,3-DPG*
raised temperature
*2,3-diphosphoglycerate
A 78 year old man is due to undergo an endarterectomy of the internal carotid artery. Which of the following nervous structures are most at risk during the dissection?
Recurrent laryngeal nerve
Sympathetic chain
Hypoglossal nerve
Phrenic nerve
Lingual nerve
Nerves at risk during a carotid endarterectomy:
Hypoglossal nerve
Greater auricular nerve
Superior laryngeal nerve
During a carotid endarterectomy the sternocleidomastoid muscle is dissected, with ligation of the common facial vein and then the internal jugular is dissected exposing the common and the internal carotid arteries. The nerves at risk during the operation include:
Hypoglossal nerve
Greater auricular nerve
Superior laryngeal nerve
The sympathetic chain lies posteriorly and is less prone to injury in this procedure.
Internal carotid artery
The internal carotid artery is formed from the common carotid opposite the upper border of the thyroid cartilage. It extends superiorly to enter the skull via the carotid canal. From the carotid canal it then passes through the cavernous sinus, above which it divides into the anterior and middle cerebral arteries.
Relations in the neck
Posterior
Longus capitis
Pre-vertebral fascia
Sympathetic chain
Superior laryngeal nerve
Medially
External carotid (near origin)
Wall of pharynx
Ascending pharyngeal artery
Laterally
Internal jugular vein (moves posteriorly at entrance to skull)
Vagus nerve (most posterolaterally)
Anteriorly
Sternocleidomastoid
Lingual and facial veins
Hypoglossal nerve
Relations in the carotid canal
Internal carotid plexus
Cochlea and middle ear cavity
Trigeminal ganglion (superiorly)
Leaves canal lies above the foramen lacerum
Path and relations in the cranial cavity
The artery bends sharply forwards in the cavernous sinus, the aducens nerve lies close to its inferolateral aspect. The oculomotor, trochlear, opthalmic and, usually, the maxillary nerves lie in the lateral wall of the sinus. Near the superior orbital fissure it turns posteriorly and passes postero-medially to pierce the roof of the cavernous sinus inferior to the optic nerve. It then passes between the optic and oculomotor nerves to terminate below the anterior perforated substance by dividing into the anterior and middle cerebral arteries.
Branches
Anterior and middle cerebral artery
Ophthalmic artery
Posterior communicating artery
Anterior choroid artery
Meningeal arteries
Hypophyseal arteries
A 19 year old sportswoman presents with knee pain which is worse on walking down the stairs and when sitting still. On examination, there is wasting of the quadriceps and pseudolocking of the knee. What is the diagnosis?
Osteoarthritis
Quadriceps tendon rupture
Undisplaced fracture patella
Chondromalacia patellae
Osgood Schlatters disease
A teenage girl with knee pain on walking down the stairs is characteristic for chondromalacia patellae (anterior knee pain). Most cases are managed with physiotherapy.
Knee injury
Types of injury
Ruptured anterior cruciate ligament
Sport injury
Mechanism: high twisting force applied to a bent knee
Typically presents with: loud crack, pain and RAPID joint swelling (haemoarthrosis)
Poor healing
Management: intense physiotherapy or surgery
Ruptured posterior cruciate ligament
Mechanism: hyperextension injuries
Tibia lies back on the femur
Paradoxical anterior draw test
Rupture of medial collateral ligament
Mechanism: leg forced into valgus via force outside the leg
Knee unstable when put into valgus position
Menisceal tear
Rotational sporting injuries
Delayed knee swelling
Joint locking (Patient may develop skills to ‘unlock’ the knee
Recurrent episodes of pain and effusions are common, often following minor trauma
Chondromalacia patellae
Teenage girls, following an injury to knee e.g. Dislocation patella
Typical history of pain on going downstairs or at rest
Tenderness, quadriceps wasting
Dislocation of the patella
Most commonly occurs as a traumatic primary event, either through direct trauma or through severe contraction of quadriceps with knee stretched in valgus and external rotation
Genu valgum, tibial torsion and high riding patella are risk factors
Skyline x-ray views of patella are required, although displaced patella may be clinically obvious
An osteochondral fracture is present in 5%
The condition has a 20% recurrence rate
Fractured patella
2 types:
i. Direct blow to patella causing undisplaced fragments
ii. Avulsion fracture
Tibial plateau fracture
Occur in the elderly (or following significant trauma in young)
Mechanism: knee forced into valgus or varus, but the knee fractures before the ligaments rupture
Varus injury affects medial plateau and if valgus injury, lateral plateau depressed fracture occurs
Classified using the Schatzker system (see below)
Schatzker Classification system for tibial plateau fractures
Type Anatomical description Features
1 Vertical split of lateral condyle Fracture through dense bone, usually in the young. It may be virtually undisplaced, or the condylar fragment may be pushed inferiorly and tilted
2 Vertical split of the lateral condyle combined with an adjacent load bearing part of the condyle The wedge fragment (which may be of variable size), is displaced laterally; the joint is widened. Untreated, a valgus deformity may develop
3 Depression of the articular surface with intact condylar rim The split does not extend to the edge of the plateau. Depressed fragments may be firmly embedded in subchondral bone, the joint is stable
4 Fragment of the medial tibial condyle Two injuries are seen in this category; (1) a depressed fracture of osteoporotic bone in the elderly. (2) a high energy fracture resulting in a condylar split that runs from the intercondylar eminence to the medial cortex. Associated ligamentous injury may be severe
5 Fracture of both condyles Both condyles fractured but the column of the metaphysis remains in continuity with the tibial shaft
6 Combined condylar and subcondylar fractures High energy fracture with marked comminution
With which of the following blood products is iatrogenic septicaemia with a gram positive organism most likely?
Cryoprecipitate
Platelets
Packed red cells
Factor VIII concentrate
Factor IX concentrate
Platelets are stored at room temperature and must be used soon after collection. This places them at increased risk of culturing gram positive organisms. Iatrogenic infection with gram negative organisms is more likely with packed red cells as these are stored at 4 degrees.
Infections with blood products of this nature are both rare.
Blood products
Whole blood fractions
Fraction Key points
Packed red cells Used for transfusion in chronic anaemia and cases where infusion of large volumes of fluid may result in cardiovascular compromise. Product obtained by centrifugation of whole blood.
Platelet rich plasma Usually administered to patients who are thrombocytopaenic and are bleeding or require surgery. It is obtained by low speed centrifugation.
Platelet concentrate Prepared by high speed centrifugation and administered to patients with thrombocytopaenia.
Fresh frozen plasma
Prepared from single units of blood.
Contains clotting factors, albumin and immunoglobulin.
Unit is usually 200 to 250ml.
Usually used in correcting clotting deficiencies in patients with hepatic synthetic failure who are due to undergo surgery.
Usual dose is 12-15ml/Kg-1.
It should not be used as first line therapy for hypovolaemia.
Cryoprecipitate
Formed from supernatant of FFP.
Rich source of Factor VIII and fibrinogen.
Allows large concentration of factor VIII to be administered in small volume.
SAG-Mannitol Blood Removal of all plasma from a blood unit and substitution with:
Sodium chloride
Adenine
Anhydrous glucose
Mannitol
Up to 4 units of SAG M Blood may be administered. Thereafter whole blood is preferred. After 8 units, clotting factors and platelets should be considered.
Cell saver devices
These collect patients own blood lost during surgery and then re-infuse it. There are two main types:
Those which wash the blood cells prior to re-infusion. These are more expensive to purchase and more complicated to operate. However, they reduce the risk of re-infusing contaminated blood back into the patient.
Those which do not wash the blood prior to re-infusion.
Their main advantage is that they avoid the use of infusion of blood from donors into patients and this may reduce risk of blood borne infection. It may be acceptable to Jehovah’s witnesses. It is contraindicated in malignant disease for risk of facilitating disease dissemination.
Blood products used in warfarin reversal
In some surgical patients the use of warfarin can pose specific problems and may require the use of specialised blood products
Immediate or urgent surgery in patients taking warfarin(1) (2):
- Stop warfarin
- Vitamin K (reversal within 4-24 hours)
-IV takes 4-6h to work (at least 5mg)
-Oral can take 24 hours to be clinically effective - Fresh frozen plasma
Used less commonly now as 1st line warfarin reversal
-30ml/kg-1
-Need to give at least 1L fluid in 70kg person (therefore not appropriate in fluid overload)
-Need blood group
-Only use if human prothrombin complex is not available - Human Prothrombin Complex (reversal within 1 hour)
-Bereplex 50 u/kg
-Rapid action but factor 6 short half life, therefore give with vitamin K
References
1. Dentali, F., C. Marchesi, et al. (2011). ‘Safety of prothrombin complex concentrates for rapid anticoagulation reversal of vitamin K antagonists. A meta-analysis.’ Thromb Haemost 106(3): 429-438.
- http://www.transfusionguidelines.org/docs/pdfs/bbt-03warfarin-reversal-flowchart-2006.pdf
Which of the following statements relating to avascular necrosis is false?
When associated with fracture may occur despite the radiological evidence of fracture union.
Pain and stiffness will typically precede radiological evidence of the condition.
Drilling of affected bony fragments may be used to facilitate angiogenesis where arthroplasty is not warranted.
The earliest detectable radiological evidence is a radiolucency of the affected area coupled with subchondral collapse.
It is less likely when prompt anatomical alignment of fracture fragments is achieved.
Avascular necrosis- radiological changes occur late.
Radiolucency and subchondral collapse are late changes. The earliest evidence on plain films is the affected area appearing as being more radio-opaque due to hyperaemia and resorption of the neighboring area. It may be diagnosed earlier using bone scans and MRI.
Avascular necrosis
Cellular death of bone components due to interruption of the blood supply, causing bone destruction
Main joints affected are hip, scaphoid, lunate and the talus.
It is not the same as non union. The fracture has usually united.
Radiological evidence is slow to appear.
Vascular ingrowth into the affected bone may occur. However, many joints will develop secondary osteoarthritis.
Causes
P ancreatitis
L upus
A lcohol
S teroids
T rauma
I diopathic, infection
C aisson disease, collagen vascular disease
R adiation, rheumatoid arthritis
A myloid
G aucher disease
S ickle cell disease
Presentation
Usually pain. Often despite apparent fracture union.
Investigation
MRI scanning will show changes earlier than plain films.
Treatment
In fractures at high risk sites anticipation is key. Early prompt and accurate reduction is essential.
Non weight bearing may help to facilitate vascular regeneration.
Joint replacement may be necessary, or even the preferred option (e.g. Hip in the elderly).
Inferior alveolar nerve block may result in?
Numbness of lower lip on injected side
Ineffective block for the lower incisor teeth
Numbness of hard palate
Inability to clench jaws
Transient ipsilateral facial weakness
An inferior alveolar nerve block is a nerve block technique that anesthetizes the areas of the mouth and face innervated by one of the inferior alveolar nerves1. It may result in various symptoms, such as numbness of the lower lip and chin, parts of the tongue and gingiva, and transient facial weakness2.
The best answer to your question is:
Numbness of lower lip on injected side
This is because the inferior alveolar nerve block affects the mental nerve, which is a branch of the inferior alveolar nerve that supplies the skin and mucous membranes of the lower lip and chin on one side1. The other options are either incorrect or less likely to occur. For example, an inferior alveolar nerve block is usually effective for the lower incisor teeth, unless there is anatomical variation or accessory innervation3. Numbness of the hard palate is not caused by an inferior alveolar nerve block, but by a maxillary nerve block. Inability to clench jaws is not a common symptom of an inferior alveolar nerve block, but it may happen if the nerve block also affects the motor branches of the mandibular nerve. Transient ipsilateral facial weakness is a rare complication of an inferior alveolar nerve block, and it may occur due to diffusion of the anesthetic agent into the parotid gland, affecting the facial nerve.
An 88 year old lady presents with a large mass in the upper inner quadrant of her right breast. Investigations confirm an oestrogen receptor positive, invasive ductal carcinoma. She has declined operative treatment. What is the best course of action?
Combined chemoradiotherapy
Radical radiotherapy
Administration of letrozole
Best supportive care
Chemotherapy alone
Elderly patients may be managed using endocrine therapy alone. Eventually most will escape hormonal control. In post menopausal women oestrogens are produced by the peripheral aromatization of androgens and aromatase inhibitors are therefore the most popular agent in this age group.
Breast cancer treatment
Treatment Indication
Endocrine therapy
Oestrogen receptor positive tumours
Downstaging primary lesions
Definitive treatment in old, infirm patients
Irradiation
Wide local excision
Large lesion, high grade or marked vascular invasion following mastectomy
Chemotherapy
Downstaging advanced lesions to facilitate breast conserving surgery
Patients with grade 3 lesions or axillary nodal disease
Endocrine agents
Tamoxifen is used and works as a partial oestrogen receptor agonist. It will typically block activity at the breast. It does, however, stimulate the receptors at other sites and it is this that accounts for its association with endometrial cancer. In post menopausal women the process of aromatisation accounts for most oestrogen production. Therefore in this group aromatase inhibitors are the preferred agents. Women who are perimenopausal start on tamoxifen and switch at 3 years.
More recent studies (aTTom and ATLAS) have demonstrated benefits for continuing the drug for 10 years. In pre-menopausal women, there is increasing preference for the use of Exemestane over tamoxifen.
Chemotherapy
The FEC regime is most commonly used (Fluorouracil, epirubicin and cyclophosphamide). This was found to be superior to the older CMF regime. The Taxanes are commonly used in high risk patients and in this setting a regime of docetaxal, doxorubicin and cyclophosphamide may be used. The anthracycline class drugs have marked cardiotoxicity (a property that they share with trastuzumab) and this can limit their use.
Which of the following is not a content of the porta hepatis?
Portal vein
Hepatic artery
Cystic duct
Lymph nodes
None of the above
The cystic duct lies outside the porta hepatis and is an important landmark in laparoscopic cholecystectomy. The structures in the porta hepatis are:
Portal vein
Hepatic artery
Common hepatic duct
These structures divide immediately after or within the porta hepatis to supply the functional left and right lobes of the liver.
The porta hepatis is also surrounded by lymph nodes, that may enlarge to produce obstructive jaundice and parasympathetic nervous fibres that travel along vessels to enter the liver.
Liver
Structure of the liver
Right lobe
Supplied by right hepatic artery
Contains Couinaud segments V to VIII (-/+Sg I)
Left lobe
Supplied by the left hepatic artery
Contains Couinaud segments II to IV (+/- Sg1)
Quadrate lobe
Part of the right lobe anatomically, functionally is part of the left
Couinaud segment IV
Porta hepatis lies behind
On the right lies the gallbladder fossa
On the left lies the fossa for the umbilical vein
Caudate lobe
Supplied by both right and left hepatic arteries
Couinaud segment I
Lies superior to the porta hepatis
Anterior and medial to the inferior vena cava
Bile from the caudate lobe drains into both right and left hepatic ducts
Detailed knowledge of Couinaud segments is not required for MRCS
Between the liver lobules are portal canals which contain the portal triad: Hepatic Artery, Portal Vein, tributary of Bile Duct.
Relations of the liver
Anterior Postero inferiorly
Diaphragm Oesophagus
Xiphoid process Stomach
Duodenum
Hepatic flexure of colon
Right kidney
Gallbladder
Inferior vena cava
Porta hepatis
Location Postero inferior surface, it joins nearly at right angles with the left sagittal fossa, and separates the caudate lobe behind from the quadrate lobe in front
Transmits
Common hepatic duct
Hepatic artery
Portal vein
Sympathetic and parasympathetic nerve fibres
Lymphatic drainage of the liver (and nodes)
Ligaments
Falciform ligament
2 layer fold peritoneum from the umbilicus to anterior liver surface
Contains ligamentum teres (remnant umbilical vein)
On superior liver surface it splits into the coronary and left triangular ligaments
Ligamentum teres Joins the left branch of the portal vein in the porta hepatis
Ligamentum venosum Remnant of ductus venosus
Arterial supply
Hepatic artery
Venous
Hepatic veins
Portal vein
Nervous supply
Sympathetic and parasympathetic trunks of coeliac plexus
A 55 year old accountant has jaundice and a temperature of 39oC. He is known to have gallstones. Blood cultures have grown a gram negative bacilli. Imaging shows a bile duct measuring 1.2cm in diameter. What is the best treatment option?
PTC and stent
PTC and drain
ERCP and stent
MRCP
USS
You should suspect cholangitis in a patient with fevers and jaundice. Charcot’s triad may only be present in 20% of patients.
Note the question states treatment option, this excludes MRCP and USS which are not treatments. Whilst PTC may access the duct, the drains may displace easily and the transduodenal route is the preferred first line access.
Management of Pancreatitis
Management of Acute Pancreatitis in the UK
Diagnosis
Traditionally hyperamylasaemia has been utilised with amylase being elevated three times the normal range.
However, amylase may give both false positive and negative results.
Serum lipase is both more sensitive and specific than serum amylase. It also has a longer half life.
Serum amylase levels do not correlate with disease severity.
Differential causes of hyperamylasaemia
Acute pancreatitis
Pancreatic pseudocyst
Mesenteric infarct
Perforated viscus
Acute cholecystitis
Diabetic ketoacidosis
Assessment of severity
Glasgow, Ranson scoring systems and APACHE II
Biochemical scoring e.g. using CRP
Features that may predict a severe attack within 48 hours of admission to hospital
Initial assessment
Clinical impression of severity
Body mass index >30
Pleural effusion
APACHE score >8
24 hours after admission
Clinical impression of severity
APACHE II >8
Glasgow score of 3 or more
Persisting multiple organ failure
CRP>150
48 hours after admission
Glasgow Score of >3
CRP >150
Persisting or progressive organ failure
Table adapted from UK guidelines for management of acute pancreatitis. GUT 2005, 54 suppl III
Management
Nutrition
There is reasonable evidence to suggest that the use of enteral nutrition does not worsen the outcome in pancreatitis
Most trials to date were underpowered to demonstrate a conclusive benefit.
The rationale behind feeding is that it helps to prevent bacterial translocation from the gut, thereby contributing to the development of infected pancreatic necrosis.
Use of antibiotic therapy
Many UK surgeons administer antibiotics to patients with acute pancreatitis. However, there is very little evidence to support this practice.
A recent Cochrane review highlights the potential benefits of administering Imipenem to patients with established pancreatic necrosis in the hope of averting the progression to infection.
There are concerns that the administration of antibiotics in mild attacks of pancreatitis will not affect outcome and may contribute to antibiotic resistance and increase the risks of antibiotic associated diarrhoea.
Surgery
Patients with acute pancreatitis due to gallstones should undergo early cholecystectomy.
Patients with obstructed biliary system due to stones should undergo early ERCP.
Patients with extensive necrosis where infection is suspected should usually undergo FNA for culture.
Patients with infected necrosis should undergo either radiological drainage or surgical necrosectomy. The choice of procedure depends upon local expertise.
References
www.bsg.org.uk/pdfworddocs/pancreatic.pdf
Antibiotic therapy for prophylaxis against infection of pancreatic necrosis in acute pancreatitis. Villatoro et al. Cochrane Library DOI: 10.1002/14651858.CD002941.pub3. 2010 version.
You are working as an anatomy demonstrator and the medical students decide to test your knowledge on the Circle of Willis. Which of the following comments is false?
The anterior communicating artery links the right and left sides
Asymmetry of the circle of willis is a risk factor for the development of intracranial aneurysms
Majority of blood passing through the vessels mix together
Includes the anterior communicating artery
The circle surrounds the stalk of the pituitary gland
There is minimum mixing of blood passing through the vessels.
Circle of Willis
The two internal carotid arteries and two vertebral arteries form an anastomosis known as the Circle of Willis on the inferior surface of the brain. Each half of the circle is formed by:
1. Anterior communicating artery
2. Anterior cerebral artery
3. Internal carotid artery
4. Posterior communicating artery
5. Posterior cerebral arteries and the termination of the basilar artery
The circle and its branches supply; the corpus striatum, internal capsule, diencephalon and midbrain.
Vertebral arteries
Enter the cranial cavity via foramen magnum
Lie in the subarachnoid space
Ascend on anterior surface of medulla oblongata
Unite to form the basilar artery at the base of the pons
Branches:
Posterior spinal artery
Anterior spinal artery
Posterior inferior cerebellar artery
Basilar artery
Branches:
Anterior inferior cerebellar artery
Labyrinthine artery
Pontine arteries
Superior cerebellar artery
Posterior cerebral artery (at the point where it bifurcates)
Internal carotid arteries
Branches:
Posterior communicating artery
Anterior cerebral artery
Middle cerebral artery
Anterior choroid artery
A 65 year old lady is admitted with large bowel obstruction. On investigation with CT, she is found to have a tumour of the mid rectum with no evidence of metastatic disease. What is the most appropriate course of action?
Formation of a loop colostomy
Laparotomy and Hartmanns procedure
Pan proctocolectomy and end ileostomy
Low anterior resection and covering loop ileostomy
Low anterior resection and end colostomy
Avoid emergency resections in large bowel obstruction due to rectal cancer
This patient has presented with large bowel obstruction. However, in the case of rectal cancer, she is incompletely staged as ability to completely resect the lesion can only be determined with MRI scanning and this information is not provided. Even if the lesion were resectable, in the emergency setting, it is often safer to undertake a simple procedure such as a loop colostomy and then complete surgery at a later date. A low anterior resection and loop ileostomy in this situation would almost certainly leak (and for the reasons outlined above, may be incomplete).
Large bowel obstruction
Colonic obstruction remains a common surgical problem. It is most commonly due to malignancy (60%) and diverticular disease (20%). Volvulus affecting the colon accounts for 5% of cases. Acute colonic pseudo-obstruction remains a potential differential diagnosis in all cases. Intussusception affecting the colon (most often due to tumours in the adult population) remains a rare but recognised cause.
The typical patient will present with gradual onset of progressive abdominal distension, colicky abdominal pain and either obstipation or absolute constipation.
On examination abdominal distension is present, the presence of caecal tenderness (assuming no overt evidence of peritonitis) is a useful sign to elicit. A digital rectal examination and rigid sigmoidoscopy should be performed.
A plain abdominal x-ray is the usual first line test and; the caecal diameter and ileocaecal valve competency should be assessed on this film.
Imaging modalities
Debate long surrounds the use of CT versus gastrograffin enemas. The latter investigation has always been the traditional method of determining whether a structural lesion is indeed present. However, in the UK the use of this technique has declined and in most units a CT scan will be offered as the first line investigation by the majority of radiologists (and is advocated by the ACPGBI). In most cases this will provide sufficient detail to allow operative planning, and since malignancy accounts for most presentations may also stage the disease. In the event that the radiologist cannot provide a clear statement of lesion site, the surgeon should have no hesitation in requesting a contrast enema.
Surgical options
The decision as to when to operate or not is determined firstly by the patients physiological status. Unstable patients require resuscitation prior to surgery and admission to a critical care unit for invasive monitoring and potential inotropic support may be needed. In patients who are otherwise stable the decision then rests on the radiological and clinical findings. As a general rule the old adage that the sun should not rise and set on unrelieved large bowel obstruction still holds true. A caecal diameter of 12cm or more in the presence of complete obstruction with a competent ileocaecal valve and caecal tenderness is a sign of impending perforation and a relative indication for prompt surgery.
Right sided and transverse lesions
Right sided lesions producing large bowel obstruction should generally be treated by right hemicolectomy or its extended variant if the lesion lies in the distal transverse colon or splenic flexure. In these cases an ileocolic anastomosis may be easily constructed and even in the emergency setting has a low risk of anastomotic leak.
Left sided lesions
The options here lie between sub total colectomy and anastomosis, left hemicolectomy with on table lavage and primary anastomosis, left hemicolectomy and end colostomy formation and finally colonic stent insertion.
The usefulness of colonic stents was the subject of a Cochrane review in 2011. The authors concluded that on the basis of the data that they reviewed, there was no benefit from the use of colonic stents over conventional surgical resection with a tendency to better outcomes seen in the surgical group (1). A more recently conducted meta analysis met with the same conclusion (2). However, the recently concluded CREST trial has suggested that self expanding metallic stents can improve outcomes and avoids a stoma.
Rectosigmoid lesions
Lesions below the peritoneal reflection that are causing obstruction should generally be treated with a loop colostomy. Primary resection of unstaged rectal cancer would most likely carry a high CRM positivity rate and cannot be condoned. Where the lesion occupies the distal sigmoid colon the usual practice would be to perform a high anterior resection. The decision surrounding restoration of intestinal continuity would lie with the operating surgeon.
References
1. Sagar J. Colorectal stents for the management of malignant colonic obstructions. Cochrane Database of Systematic Reviews 2011, Issue 11. Art. No.: CD007378. DOI: 10.1002/14651858.CD007378.pub2.
2. Cirrochi et al Safety and efficacy of endoscopic colonic stenting as a bridge to surgery in the management of intestinal obstruction due to left colon and rectal cancer: A systematic review and meta-analysis. Surg Oncol. 2013 Mar;22(1):14-21.
During a gangland gunfight a man is shot in the chest. The bullet passes through the posterior mediastinum (from left to right). Which of the following structures is least likely to be injured
Thoracic duct
Oesophagus
Vagus nerve
Descending thoracic aorta
Arch of the azygos vein
The arch of the azygos vein lies in the middle mediastinum.
Mediastinum
Region between the pulmonary cavities.
It is covered by the mediastinal pleura. It does not contain the lungs.
It extends from the thoracic inlet superiorly to the diaphragm inferiorly.
Mediastinal regions
Superior mediastinum (between manubriosternal angle and T4/5)
Middle mediastinum
Posterior mediastinum
Anterior mediastinum
Region Contents
Superior mediastinum
Superior vena cava
Brachiocephalic veins
Arch of aorta
Thoracic duct
Trachea
Oesophagus
Thymus
Vagus nerve
Left recurrent laryngeal nerve
Phrenic nerve
Anterior mediastinum
Thymic remnants
Lymph nodes
Fat
Middle mediastinum
Pericardium
Heart
Aortic root
Arch of azygos vein
Main bronchi
Posterior mediastinum
Oesophagus
Thoracic aorta
Azygos vein
Thoracic duct
Vagus nerve
Sympathetic nerve trunks
Splanchnic nerves
A 43 year old female develops severe chest wall cellulitis following a mastectomy. On examination, the skin is markedly erythematous. Which of the acute inflammatory mediators listed below is least likely to produce vasodilation at this site?
Complement component C5a
Lysosomal compounds
Histamine
Serotonin
Prostaglandins
Erythema is a classical feature of acute inflammation. Potent mediators of vascular dilatation include; histamine, prostaglandins, nitric oxide, platelet activating factor, complement C5a (and C3a) and lysosomal compounds. Although serotonin is associated with acute inflammation it is a vasoconstrictor. The effects of serotonin are dependent upon the state of the vessels in the tissues. Intact and healthy tissues and vessels will respond to a serotonin infusion with vasodilation (hence the flushing seen in carcinoid syndrome). In contrast it worsens cardiac ischaemia in myocardial infarcts when released from damaged platelets.
Acute inflammation
Inflammation is the reaction of the tissue elements to injury. Vascular changes occur, resulting in the generation of a protein rich exudate. So long as the injury does not totally destroy the existing tissue architecture, the episode may resolve with restoration of original tissue architecture.
Vascular changes
Vasodilation occurs and persists throughout the inflammatory phase.
Inflammatory cells exit the circulation at the site of injury.
The equilibrium that balances Starlings forces within capillary beds is disrupted and a protein rich exudate will form as the vessel walls also become more permeable to proteins.
The high fibrinogen content of the fluid may form a fibrin clot. This has several important immunomodulatory functions.
Sequelae
Resolution
Typically occurs with minimal initial injury
Stimulus removed and normal tissue architecture results
Organisation
Delayed removal of exudate
Tissues undergo organisation and usually fibrosis
Suppuration
Typically formation of an abscess or an empyema
Sequestration of large quantities of dead neutrophils
Progression to chronic inflammation
Coupled inflammatory and reparative activities
Usually occurs when initial infection or suppuration has been inadequately managed
Causes
Infections e.g. Viruses, exotoxins or endotoxins released by bacteria
Chemical agents
Physical agents e.g. Trauma
Hypersensitivity reactions
Tissue necrosis
Presence of neutrophil polymorphs is a histological diagnostic feature of acute inflammation
The vertebral artery traverses all of the following except?
Transverse process of C6
Transverse process of the axis
Vertebral canal
Foramen magnum
Intervertebral foramen
The vertebral artery passes through the foramina which are located in the transverse processes of the cervical vertebra, it does not traverse the intervertebral foramen.
Vertebral artery
The vertebral artery is the first branch of the subclavian artery. Anatomically it is divisible into 4 regions:
The first part runs to the foramen in the transverse process of C6. Anterior to this part lies the vertebral and internal jugular veins. On the left side the thoracic duct is also an anterior relation.
The second part runs superiorly through the foramina of the the transverse processes of the upper 6 cervical vertebrae. Once it has passed through the transverse process of the axis it then turns superolaterally to the atlas. It is accompanied by a venous plexus and the inferior cervical sympathetic ganglion.
The third part runs posteromedially on the lateral mass of the atlas. It enters the sub occipital triangle, in the groove of the upper surface of the posterior arch of the atlas. It then passes anterior to the edge of the posterior atlanto-occipital membrane to enter the vertebral canal.
The fourth part passes through the spinal dura and arachnoid, running superiorly and anteriorly at the lateral aspect of the medulla oblongata. At the lower border of the pons it unites to form the basilar artery.
A 19 year old motorcyclist is involved in a road traffic accident. His chest movements are irregular. He is found to have multiple rib fractures, with 2 fractures in the 3rd rib and 3 fractures in the 4th rib. What is the underlying diagnosis?
Simple rib fractures
Flail chest injury
Cardiac tamponade
Pneumothorax
Aortic rupture
Multiple rib fractures with > or = 2 rib fractures in more than 2 ribs is diagnosed as a flail chest. This is associated with pulmonary contusion.
Thoracic trauma
Key points related to thoracic trauma
Less than 10% of blunt chest trauma and 15-30% of penetrating chest trauma requires operative intervention.
The physiologic consequences of thoracic trauma are hypoxia, hypercarbia, and acidosis. Contusion, hematoma, and alveolar collapse, or changes in intrathoracic pressure relationships (e.g., tension pneumothorax and open pneumothorax) cause hypoxia and lead to metabolic acidosis. Hypercarbia causes respiratory acidosis and most often follows inadequate ventilation caused by changes in intrathoracic pressure relationships and depressed level of consciousness.
Types of thoracic trauma
Tension pneumothorax
Often laceration to lung parenchyma with flap
Pressure develops in thorax
Most common cause is mechanical ventilation in patient with pleural injury
Symptoms overlap with cardiac tamponade, hyper-resonant percussion note is more likely in tension pnemothorax
Flail chest
Chest wall disconnects from thoracic cage
Multiple rib fractures (at least two fractures per rib in at least two ribs)
Associated with pulmonary contusion
Abnormal chest motion
Avoid over hydration and fluid overload
Pneumothorax
Most common cause is lung laceration with air leakage
Most traumatic pneumothoraces should have a chest drain
Patients with traumatic pneumothorax should never be mechanically ventilated until a chest drain is inserted
Haemothorax
Most commonly due to laceration of lung, intercostal vessel or internal mammary artery
Haemothoraces large enough to appear on CXR are treated with large bore chest drain
Surgical exploration is warranted if >1500ml blood drained immediately
Cardiac tamponade
Beck’s triad: elevated venous pressure, reduced arterial pressure, reduced heart sounds
Pulsus paradoxus
May occur with as little as 100ml blood
Pulmonary contusion
Most common potentially lethal chest injury
Arterial blood gases and pulse oximetry important
Early intubation within an hour if significant hypoxia
Blunt cardiac injury
Usually occurs secondary to chest wall injury
ECG may show features of myocardial infarction
Sequelae: hypotension, arrhythmias, cardiac wall motion abnormalities
Aorta disruption
Deceleration injuries
Contained haematoma
Widened mediastinum
Diaphragm disruption
Most due to motor vehicle accidents and blunt trauma causing large radial tears (laceration injuries result in small tears)
More common on left side
Insert gastric tube, may pass into intrathoracic stomach
Mediastinal traversing wounds
Entrance wound in one hemithorax and exit wound/foreign body in opposite hemithorax
Mediastinal haematoma or pleural cap suggests great vessel injury
Mortality is 20%
During short saphenous vein surgery for varicose veins which of the following nerves is particularly at risk?
Sural nerve
Popliteal nerve
Tibial nerve
Femoral nerve
Saphenous nerve
Saphenous vein
Long saphenous vein
This vein may be harvested for bypass surgery, or removed as treatment for varicose veins with saphenofemoral junction incompetence.
Originates at the 1st digit where the dorsal vein merges with the dorsal venous arch of the foot
Passes anterior to the medial malleolus and runs up the medial side of the leg
At the knee, it runs over the posterior border of the medial epicondyle of the femur bone
Then passes laterally to lie on the anterior surface of the thigh before entering an opening in the fascia lata called the saphenous opening
It joins with the femoral vein in the region of the femoral triangle at the saphenofemoral junction
Tributaries
Medial marginal
Superficial epigastric
Superficial iliac circumflex
Superficial external pudendal veins
Short saphenous vein
Originates at the 5th digit where the dorsal vein merges with the dorsal venous arch of the foot, which attaches to the great saphenous vein.
It passes around the lateral aspect of the foot (inferior and posterior to the lateral malleolus) and runs along the posterior aspect of the leg (with the sural nerve)
Passes between the heads of the gastrocnemius muscle, and drains into the popliteal vein, approximately at or above the level of the knee joint.
Where are sarcomas most likely to be found?
In the thorax
In the abdomen
Around the axial spine
In the extremities
In the head and neck
40% of sarcomas are found in the extremities.
Sarcomas
Malignant tumours of mesenchymal origin
Types
May be either bone or soft tissue in origin.
Bone sarcoma include:
Osteosarcoma
Ewings sarcoma (although non bony sites recognised)
Chondrosarcoma - originate from Chondrocytes
Soft tissue sarcoma are a far more heterogeneous group and include:
Liposarcoma-adipocytes
Rhabdomyosarcoma-striated muscle
Leiomyosarcoma-smooth muscle
Synovial sarcomas- close to joints (cell of origin not known but not synovium)
Malignant fibrous histiocytoma is now referred to as undifferentiated pleomorphic sarcoma. Careful histological assessment of lesions now allows more accurate categorisation of sarcoma subtypes than was previously possible.
Features
Certain features of a mass or swelling should raise suspicion for a sarcoma these include:
Large >5cm soft tissue mass
Deep tissue location or intra muscular location
Rapid growth
Painful lump
Assessment
Imaging of suspicious masses should utilise a combination of MRI, CT and USS. Blind biopsy should not be performed prior to imaging and where required should be done in such a way that the biopsy tract can be subsequently included in any resection.
Ewings sarcoma
Commoner in males
Incidence of 0.3 / 1, 000, 000
Onset typically between 10 and 20 years of age
Location by femoral diaphysis is commonest site
Histologically it is a small round tumour
Blood borne metastasis is common and chemotherapy is often combined with surgery
Osteosarcoma
Mesenchymal cells with osteoblastic differentiation
20% of all primary bone tumours
Incidence of 5 per 1,000,000
Peak age 15-30, commoner in males
Limb preserving surgery may be possible and many patients will receive chemotherapy
Liposarcoma
Malignancy of adipocytes
Rare, approximately 2.5 per 1,000,000. They are the most common soft tissue sarcoma
Typically located in deep locations such as retroperitoneum
Affect older age group usually >40 years of age
May be well differentiated and thus slow growing although may undergo de-differentiation and disease progression
Many tumours will have a pseudocapsule that can misleadingly allow surgeons to feel that they can ‘shell out’ these lesions. In reality, tumour may invade at the edge of the pseudocapsule and result in local recurrence if this strategy is adopted
Usually resistant to radiotherapy, although this is often used in a palliative setting
Malignant Fibrous Histiocytoma
Tumour with large number of histiocytes
Also described as undifferentiated pleomorphic sarcoma NOS (i.e. Cell of origin is not known)
Four major subtypes are recognised: storiform-pleomorphic (70% cases), myxoid (less aggressive), giant cell and inflammatory
Treatment is usually with surgical resection and adjuvant radiotherapy as this reduces the likelihood of local recurrence
A 30 year old male is hit on the side of the head with a bat. He now presents to Emergency Department with odd behaviour and complaining of a headache. Whilst waiting for a CT scan he becomes drowsy and unresponsive. What is the most likely underlying injury?
Intra cerebral haematoma
Sub dural haematoma
Extra dural haematoma
Intraventricular haemorrhage
Sub arachnoid haemorrhage
Extra dural haematoma
The middle meningeal artery is prone to damage when the temporal side of the head is hit.
Note that there may NOT be any initial LOC or lucid interval.
Head injury
Patients who suffer head injuries should be managed according to ATLS principles and extra cranial injuries should be managed alongside cranial trauma. Inadequate cardiac output will compromise CNS perfusion irrespective of the nature of the cranial injury.
Types of traumatic brain injury
Extradural haematoma Bleeding into the space between the dura mater and the skull. Often results from acceleration-deceleration trauma or a blow to the side of the head. The majority of extradural haematomas occur in the temporal region where skull fractures cause a rupture of the middle meningeal artery.
Features
Raised intracranial pressure
Some patients may exhibit a lucid interval
Subdural haematoma Bleeding into the outermost meningeal layer. Most commonly occur around the frontal and parietal lobes. May be either acute or chronic.
Risk factors include old age and alcoholism.
Slower onset of symptoms than a extradural haematoma.
Subarachnoid haemorrhage Usually occurs spontaneously in the context of a ruptured cerebral aneurysm, but may be seen in association with other injuries when a patient has sustained a traumatic brain injury.
Pathophysiology
Primary brain injury may be focal (contusion/ haematoma) or diffuse (diffuse axonal injury)
Diffuse axonal injury occurs as a result of mechanical shearing following deceleration, causing disruption and tearing of axons
Intra-cranial haematomas can be extradural, subdural or intracerebral, while contusions may occur adjacent to (coup) or contralateral (contre-coup) to the side of impact
Secondary brain injury occurs when cerebral oedema, ischaemia, infection, tonsillar or tentorial herniation exacerbates the original injury. The normal cerebral auto regulatory processes are disrupted following trauma rendering the brain more susceptible to blood flow changes and hypoxia
The Cushings reflex (hypertension and bradycardia) often occurs late and is usually a pre terminal event
Management
Where there is life threatening rising ICP such as in extra dural haematoma and whilst theatre is prepared or transfer arranged use of IV mannitol/ frusemide may be required.
Diffuse cerebral oedema may require decompressive craniotomy
Exploratory Burr Holes have little management in modern practice except where scanning may be unavailable and to thus facilitate creation of formal craniotomy flap
Depressed skull fractures that are open require formal surgical reduction and debridement, closed injuries may be managed non operatively if there is minimal displacement.
ICP monitoring is appropriate in those who have GCS 3-8 and normal CT scan.
ICP monitoring is mandatory in those who have GCS 3-8 and abnormal CT scan.
Hyponatraemia is most likely to be due to syndrome of inappropriate ADH secretion.
Minimum of cerebral perfusion pressure of 70mmHg in adults.
Minimum cerebral perfusion pressure of between 40 and 70 mmHg in children.
Interpretation of pupillary findings in head injuries
Pupil size Light response Interpretation
Unilaterally dilated Sluggish or fixed 3rd nerve compression secondary to tentorial herniation
Bilaterally dilated Sluggish or fixed
Poor CNS perfusion
Bilateral 3rd nerve palsy
Unilaterally dilated or equal Cross reactive (Marcus - Gunn) Optic nerve injury
Bilaterally constricted May be difficult to assess
Opiates
Pontine lesions
Metabolic encephalopathy
Unilaterally constricted Preserved Sympathetic pathway disruption
Which of the following statements relating to quadratus lumborum is false?
Causes flexion of the thoracic spine
Causes the rib cage to be pulled down
Innervated by anterior primary rami of T12 and L1-3
Attached to the iliac crest
Inserts into the 12th rib
Quadratus lumborum
Origin: Medial aspect of iliac crest and iliolumbar ligament
Insertion: 12th rib
Action: Pulls the rib cage inferiorly. Lateral flexion.
Nerve supply: Anterior primary rami of T12 and L1-3
The rectus abdominis causes flexion of the thoracic spine and therefore the statement suggesting that quadratus lumborum does so is incorrect.
Abdominal wall
The 2 main muscles of the abdominal wall are the rectus abdominis (anterior) and the quadratus lumborum (posterior).
The remaining abdominal wall consists of 3 muscular layers. Each muscle passes from the lateral aspect of the quadratus lumborum posteriorly to the lateral margin of the rectus sheath anteriorly. Each layer is muscular posterolaterally and aponeurotic anteriorly.
Muscles of abdominal wall
External oblique
Lies most superficially
Originates from 5th to 12th ribs
Inserts into the anterior half of the outer aspect of the iliac crest, linea alba and pubic tubercle
More medially and superiorly to the arcuate line, the aponeurotic layer overlaps the rectus abdominis muscle
The lower border forms the inguinal ligament
The triangular expansion of the medial end of the inguinal ligament is the lacunar ligament.
Internal oblique
Arises from the thoracolumbar fascia, the anterior 2/3 of the iliac crest and the lateral 2/3 of the inguinal ligament
The muscle sweeps upwards to insert into the cartilages of the lower 3 ribs
The lower fibres form an aponeurosis that runs from the tenth costal cartilage to the body of the pubis
At its lowermost aspect it joins the fibres of the aponeurosis of transversus abdominis to form the conjoint tendon.
Transversus abdominis
Innermost muscle
Arises from the inner aspect of the costal cartilages of the lower 6 ribs , from the anterior 2/3 of the iliac crest and lateral 1/3 of the inguinal ligament
Its fibres run horizontally around the abdominal wall ending in an aponeurosis. The upper part runs posterior to the rectus abdominis. Lower down the fibres run anteriorly only.
The rectus abdominis lies medially; running from the pubic crest and symphysis to insert into the xiphoid process and 5th, 6th and 7th costal cartilages. The muscles lies in a aponeurosis as described above.
Nerve supply: anterior primary rami of T7-12
Surgical notes
During abdominal surgery it is usually necessary to divide either the muscles or their aponeuroses. During a midline laparotomy it is desirable to divide the aponeurosis. This will leave the rectus sheath intact above the arcuate line and the muscles intact below it. Straying off the midline will often lead to damage to the rectus muscles, particularly below the arcuate line where they may often be in close proximity to each other.
A 23 year old man who plays rugby for a hobby presents with recurrent anterior dislocation of the shoulder. Which of the following abnormalities is most likely to be present to account for this?
Rotator cuff tear
Biceps tendon rupture
Bankart lesion
Axillary nerve injury
Infraspinatus tendinitis
A Bankart lesion is an injury of the anterior (inferior) glenoid labrum of the shoulder due to anterior shoulder dislocation. When this happens, a pocket at the front of the glenoid forms that allows the humeral head to dislocate into it.
Anterior dislocations are the most common. When recurrent, a Bankart lesion is the most common underlying abnormality. This is usually visualised by CT and MRI scanning and often repaired arthroscopically.
Shoulder disorders
Shoulder fractures and dislocations
Fractures
Proximal humerus
Background
Third most common fragility fracture in the elderly.
Results from low energy fall in predominantly elderly females, or from high energy trauma in young males.
Can be associated with nerve injury (commonly axillary), and fracture-dislocation of the humeral head. Detailed neurological assessment is essential for all upper limb injuries.
Anatomy
Osteology
Consists of articular head, greater tuberosity, lesser tuberosity, metaphysis and diaphysis. Between the articular head and the tuberosities is the anatomical neck (previous physis). Between the tuberosities and the metaphysis is the surgical neck.
The supraspinatus, infraspinatus and teres minor muscles attach to the greater tuberosity. The subscapularis muscle attaches to the lesser tuberosity.
Vascular Supply
Humeral head is supplied by the anterior and posterior humeral circumflex arteries. Anatomical neck fractures are at greatest risk of osteonecrosis.
Imaging
Imaging aims to both delineate the fracture pattern, and confirm/exlude the presence of an associated dislocation.
Radiographs - True anteroposterior (AP), axillary lateral and/or scapula Y view.
CT - indicated to better define intra-articular involvement and to aid pre-operative planning. MRI is not useful for fracture imaging.
Classification
Description of the fracture is often more useful than classification. Particular attention should be paid to humeral alignment, fracture displacement, and greater tuberosity position (rotator cuff will pull the GT supero-posterioly, which can cause impingement problems with malunion).
- Neer Classification: Most commonly used. Describes fracture as 2,3,or 4 part depending upon the number main fragments. Also comments on the degree of displacement. Fragments:
-greater tuberosity
-lesser tuberosity
- articular surface
- shaft
Displacement: >1cm or angulation >45 degrees.
Treatment
The vast majority of proximal humeral fractures are minimally displaced, and therefore can be managed conservatively. This involves immobilisation in a polysling, and progressive mobilisation. Pendular exercise can commence at 14 days, and active abduction from 4-6 weeks.
Irreducible fracture dislocation is an indication for operative management. Other indications include large displacement, younger patient, head splitting (intra-articular fractures). However, the recent PROFHER trial (1) has suggested no benefit to operative intervention on patient outcome (it must be applied cautiously as majority of patients were elderly with extraarticular fractures). Options available for surgical management include:
ORIF Most commonly used. Plate and screw fixation. Can reconstruct complex fractures.
Intramedullary nail Suitable for extra-articular configuration, predominantly surgical neck +/- GT fractures.
Hemiarthroplasty Used for un-reconstructable fractures in the older patient who has good glenoid quality.
Total shoulder arthroplasty Unconstructable fractures where high functioning shoulder is required (hemiarthroplasty will cause glenoid erosion)
Reverse shoulder arthroplasty Total shoulder arthroplasty that provides better functional outcome than conventional total shoulder replacement.
Scapula
Background
Uncommon fractures usually associated with high energy trauma. Most commonly involve scapula body or spine (50%), glenoid fossa and glenoid neck. Important to exclude associated life threatening injury.
Imaging
Plain radiographs should include true anteroposterior (AP), axillary lateral and/or scapula Y view. CT scanning is useful for defining intra-articular involvement, displacement and for three dimensional reconstruction.
Classification
Based on the location of the fracture (coracoid, acromion, glenoid neck, glenoid fossa, scapula body). Beware of ipsilateral glenoid neck and clavicle fracture -floating shoulder - where limb is effectively dissociated from axial skeleton.
Treatment
The vast majority of scapula fractures are amenable to conservative management, consisting of sling immobilisation for two weeks followed by early rehabilitation. Floating shoulder will usually require fixation, and consideration of surgery should also be given to intra-articular and displaced/angulated glenoid fractures.
Dislocations
Types
Dislocations around the shoulder joint include glenohumeral dislocation, acromioclavicular joint disruption and sternoclavicular dislocation. Only glenohumeral dislocation will be covered here.
Glenohumeral dislocation
Diagnosis, classification and management are covered here.
Background
Shoulder dislocation is commonly seen in A&E. It has a high recurrence rate that is as high as 80% in teenagers. Initial management requires emergent reduction to prevent lasting chondral damage.
Early assessment and management
Usually a traumatic cause (multi-directional instability in frequent dislocations requires discussion with orthopaedics and is not covered here). Careful history, examination and documentation of neurovascular status of the limb, in particular the axillary nerve (regimental badge sensation). This should be re-assessed post manipulation. Early radiographs to confirm direction of dislocation.
Initial management consists of emergent closed reduction under under entanox and analgesia, but often requires conscious sedation. Arm should then be immobilised in a polysling, and XR to confirm relocation.
Imaging - True anteroposterior (AP), axillary lateral and/or scapula Y view. Reduced humeral head should lie between acromion and coracoid on lateral/scapula view.
Types
Direction Features Cause Examination Reduction techniques
Anterior Most Common >90% Usually traumatic - anterior force on arm when shoulder is abducted, externally rotated Loss of shoulder contour - sulcus sign. Humeral head can be felt anteriorly.
Hippocratic.
Milch.
Stimson.
Kocher not advised due to complication of fracture
Posterior 50% missed in A&E 50% traumatic, but classically post seizure or electrocution Shoulder locked in internal rotation. XR may show lightbulb appearance. Gentle lateral traction to adducted arm.
Inferior Rare Associated with pectorals and rotator cuff tears, and glenoid fracture As for primary injury Management of primary injury
Superior Rare Associated with acrominon/clavicle fracture As for primary injury Management of primary injury
Associated injuries
Bankart lesion - avulsion of the anterior glenoid labrum with an anterior shoulder dislocation (reverse Bankart if poster labrum in posterior dislocation).
Hill Sachs defect - chondral impaction on posteriosuperior humeral head from contact with gleonoid rim. Can be large enough to lock shoulder, requiring open reduction. (Reverse Hill Sachs in posterior dislocation).
Rotator cuff tear - increases with age.
Greater or lesser tuberosity fracture - increases with age.
Humeral neck fracture - shoulder fracture dislocation. More common in high energy trauma and elderly. Should be discussed with orthopaedics prior to any attempted reduction.
Rotator Cuff Disease
Rotator cuff disease is a spectrum of conditions that ranges from subacromial impingement to rotator cuff tears and eventually to rotator cuff arthropathy (arthritis).
Anatomy
The rotator cuff is a group of four muscles that are important in shoulder movements, and maintenance of glenohumeral stability.
Muscle Scapular attachment Humeral attachment Action Innervation
Supraspinatus Supraspinatus fossa Superior facet of greater tuberosity Initiation of abduction of humerus Suprascapular nerve
Infraspinatus Infraspinatus fossa Posterior facet of greater tuberosity External rotation of humerus Suprascapular nerve
Teres Minor Lateral border Inferior facet of greater tuberosity External rotation of humerus Axillary Nerve
Subscapularis Subscapular fossa Lesser tuberosity Internal rotation of humerus Upper and lower subscapular nerve
The inferior rotator cuff muscles (infraspinatus, teres minor, and subscapularis) balance the superior pull of the deltoid. Injury/tear results in upward migration of the humeral head on the glenoid (can be seen on AP radiograph).
Likewise, the anterior muscles (subscapularis) are balanced with the posterior muscles (infraspinatus, teres minor).
Subacromial Impingement
The most common cause of shoulder pain, which results from impingement of the superior cuff on the undersurface of the acromion, and an inflammatory bursitis.
Associated with certain types of acromial morphology (Bigliani classification).
Presents as insidious pain which is exacerbated by overhead activities.
Rotator Cuff Tear
Often presents as an acute event on the background of chronic subacromial impingement in the older patient, but can present as an avulsion injury in younger patients.
Majority of tears are to the superior cuff (supraspinatus, infraspinatus, teres minor), though a tear to subscapularis is associated with subcoracoid impingement.
Tears present as pain and weakness when using the muscles in question.
Rotator Cuff Arthropathy
Defined as shoulder arthritis in the setting of rotator cuff dysfunction. Results from superior migration due to the loss of rotator cuff function and integrity. Unopposed deltoid pulls the humeral head superiorly.
Associated with massive chronic cuff tears.
Imaging
Plain radiographs
AP of the shoulder may show superior migration of the humerus with a cuff tear, and features of arthritis with arthropathy. Other causes of pain may also be identified (e.g. calcific tendonitis/fracture)
Outlet view is useful for defining the acromial morphology
USS
Allows dynamic imaging of the cuff, and is inexpensive. However, it is very user dependent.
MRI
Best imaging modality for cuff pathology.
Also allows imaging of the rest of the shoulder. When intra-articular pathology is suspected, can be combined with an arthrogram for improved sensitivity and specificity.
Treatment
Subacromial impingement
Physiotherapy, oral anti-inflammatory medication
Subacromial steroid injection can settle inflammation
Arthroscopic subacromial decompression by shaving away the undersurface of the acromion, more space is created for the rotator cuff. Cuff integrity is assessed also at time of surgery, and can be repaired if necessary.
Rotator cuff tear
When considering repair of a cuff tear, the age and activity of the patient, the nature of the tear (degenerative vs. acute traumatic), and the size and retraction of the tear should be considered when making a surgical plan.
Mild tears or tears in the elderly can be managed conservatively, as outlined above.
Moderate tears can be repaired arthroscopically. Massive or retracted tears will often require an open repair (occasionally with a tendon transfer). Subacromial decompression is performed at the same time to reduce impingement, symptoms and recurrence.
Calcific tendonitis
Calcific tendonitis involves calcific deposits within tendons anywhere in the body, but most commonly in the rotator cuff (specifically the supraspinatus tendon). When present in the shoulder, it is associated with subacromial impingement and pain.
Pathology
More common in women aged 30-60 years.
Association with diabetes and hypothyroidism
There are three stages of calcification
Formative phase characterized by calcific deposits
Resting phase deposit is stable, but presents with impingement problems
Resorptive phase phagocytic resorption. Most painful stage.
Presentation
Similar in presentation to subacromial impingement, with pain especially with over head activities. Atraumatic in nature.
Imaging
Plain radiographs show calcification of the rotator cuff, usually within 1.5cm of its insertion on the humerus. Supraspinatus outlet views can show level of impingment. Further imaging is rarely needed.
Treatment
Non-operative NSAIDS, steroid injection (controversial, but practiced) and physiotherapy. Approximately 75% will resolve by 6 months with conservative management.
Ultrasound guided or surgical needle barbotage can break down deposits and resolve symptoms. Occasionally surgical excision is required.
Adhesive capsulitis (Frozen Shoulder)
Pain and loss of movement of shoulder joint, which involves fibroplastic proliferation of capsular tissue, causing soft tissue scarring and contracture. Patients present with a painful and decreased arc of motion.
Associated with prolonged immobilization, previous surgery, thyroid disorders (AI) and diabetes
Classically three stages which can take up to two years to resolve:
Stage one the freezing and painful stage
Stage two the frozen and stiff stage
Stage three the thawing stage, where shoulder movement slowly improves
Imaging
Plain radiographs to exclude other causes of a painful shoulder
MRI arthrogram may show capsular contracture, and again may be used to exclude cuff pathology. However, often not performed as diagnosis is largely clinical.
Treatment
Non-operative NSAIDS, steroid injection and physiotherapy. Patience is required as condition can take up to 2 years to improve.
Operative MUA or arthroscopic adhesiolysis (release of adhesions) can expedite recovery, followed by intensive physiotherapy.
Glenohumeral Arthritis
Background
May be osteoarthritis (primary or secondary to cuff tear or trauma), rheumatoid arthritis, or as part of a spondyloarthropathy. Majority of those with RA will develop symptoms.
More common in the elderly
Presents like any other arthritis - pain at night and with movement
Imaging
AP and axillary radiographs will show features of arthritis.
CT/MRI is often useful to classify the shape of the glenoid and extent of bone loss when considering arthroplasty. MRI also essential to asses integrity of rotator cuff if considering shoulder replacement.
Treatment
Like all orthopaedics, start with simple measures:
NSAIDS, management of RA, physiotherapy, steroid injection.
Hemiarthroplasty can sometimes be considered if glenoid is in excellent condition or if patient has large comorbidity.
Arthroscopic debridement is useful if patient has isolated ACJ arthritis, but is rarely used for glenohumeral arthritis.
Total shoulder replacement is shown to produce superior outcome when compared to hemiarthroplasty in terms of pain relief, function and implant survival.
Total shoulder replacement can be anatomical (ball on humerus, with cup on glenoid), or reverse geometry (ball on glenoid, with cup on humerus). Anatomical TSR requires an in tact rotator cuff, so often reverse is preferable when the cuff if questionable in integrity.
References
1. JAMA. 2015;313(10):1037-1047. doi:10.1001/jama.2015.1629
A 32 year old attends neurology clinic complaining of tingling in his hand. He has radial deviation of his wrist and there is mild clawing of his fingers, with the 4th and 5th digits being relatively spared. What is the most likely lesion?
Ulnar nerve damage at the wrist
Ulnar nerve damage at the elbow
Radial nerve damage at the elbow
Median nerve damage at the wrist
Median nerve damage at the elbow
The ulnar paradox- the higher the lesion, the less the clawing of the fingers seen clinically.
At the elbow the ulnar nerve lesion affects the flexor carpi ulnaris and flexor digitorum profundus.
Ulnar nerve
Origin
C8, T1
Supplies (no muscles in the upper arm)
Flexor carpi ulnaris
Flexor digitorum profundus
Flexor digiti minimi
Abductor digiti minimi
Opponens digiti minimi
Adductor pollicis
Interossei muscle
Third and fourth lumbricals
Palmaris brevis
Path
Posteromedial aspect of upper arm to flexor compartment of forearm, then along the ulnar. Passes beneath the flexor carpi ulnaris muscle, then superficially over the flexor retinaculum into the palm of the hand.
Branches
Branch Supplies
Muscular branch Flexor carpi ulnaris
Medial half of the flexor digitorum profundus
Palmar cutaneous branch (Arises near the middle of the forearm) Skin on the medial part of the palm
Dorsal cutaneous branch Dorsal surface of the medial part of the hand
Superficial branch Cutaneous fibres to the anterior surfaces of the medial one and one-half digits
Deep branch Hypothenar muscles
All the interosseous muscles
Third and fourth lumbricals
Adductor pollicis
Medial head of the flexor pollicis brevis
Effects of injury
Damage at the wrist
Wasting and paralysis of intrinsic hand muscles (claw hand)
Wasting and paralysis of hypothenar muscles
Loss of sensation medial 1 and half fingers
Damage at the elbow
Radial deviation of the wrist
Clawing less in 4th and 5th digits
At what level does the aorta bifurcate into the left and right common iliac arteries?
L1
L2
L3
L4
L5
The aorta typically bifurcates at L4. This level is usually fairly constant and is often tested in the exam.
Levels
Transpyloric plane
Level of the body of L1
Pylorus stomach
Left kidney hilum (L1- left one!)
Fundus of the gallbladder
Neck of pancreas
Duodenojejunal flexure
Superior mesenteric artery
Portal vein
Left and right colic flexure
Root of the transverse mesocolon
2nd part of the duodenum
Upper part of conus medullaris
Spleen
Can be identified by asking the supine patient to sit up without using their arms. The plane is located where the lateral border of the rectus muscle crosses the costal margin.
Anatomical planes
Subcostal plane Lowest margin of 10th costal cartilage
Intercristal plane Level of body L4 (highest point of iliac crest)
Intertubercular plane Level of body L5
Common level landmarks
Inferior mesenteric artery L3
Bifurcation of aorta into common iliac arteries L4
Formation of IVC L5 (union of common iliac veins)
Diaphragm apertures
Vena cava T8
Oesophagus T10
Aortic hiatus T12
A 32 year old man presents with a painful swelling over the volar aspect of his hand after receiving a hard blow to his palm. On examination, he experiences pain on moving the wrist and on longitudinal compression of the thumb. What is the most likely injury?
Bennets fracture
Scaphoid fracture
5th metacarpal fracture
Ganglion
Bursitis
Scaphoid fractures usually occur as a result of direct hard blow to the palm or following a fall on the out-stretched hand. The main physical signs are swelling and tenderness in the anatomical snuff box, and pain on wrist movements and on longitudinal compression of the thumb
Upper limb fractures
Colles’ fracture
Fall onto extended outstretched hands
Described as a dinner fork type deformity
Classical Colles’ fractures have the following 3 features:
Features of the injury
1. Transverse fracture of the radius
2. 1 inch proximal to the radio-carpal joint
3. Dorsal displacement and angulation
Smith’s fracture (reverse Colles’ fracture)
Volar angulation of distal radius fragment (Garden spade deformity)
Caused by falling backwards onto the palm of an outstretched hand or falling with wrists flexed
Bennett’s fracture
Intra-articular fracture of the first carpometacarpal joint
Impact on flexed metacarpal, caused by fist fights
X-ray: triangular fragment at ulnar base of metacarpal
Monteggia’s fracture
Dislocation of the proximal radioulnar joint in association with an ulna fracture
Fall on outstretched hand with forced pronation
Needs prompt diagnosis to avoid disability
Galeazzi fracture
Radial shaft fracture with associated dislocation of the distal radioulnar joint
Occur after a fall on the hand with a rotational force superimposed on it.
On examination, there is bruising, swelling and tenderness over the lower end of the forearm.
X Rays reveal the displaced fracture of the radius and a prominent ulnar head due to dislocation of the inferior radio-ulnar joint.
Barton’s fracture
Distal radius fracture (Colles’/Smith’s) with associated radiocarpal dislocation
Fall onto extended and pronated wrist
Scaphoid fractures
Scaphoid fractures are the commonest carpal fractures.
Surface of scaphoid is covered by articular cartilage with small area available for blood vessels (fracture risks blood supply)
Forms floor of anatomical snuffbox
Risk of fracture associated with fall onto outstretched hand (tubercle, waist, or proximal 1/3)
The main physical signs are swelling and tenderness in the anatomical snuff box, and pain on wrist movements and on longitudinal compression of the thumb.
Ulnar deviation AP needed for visualization of scaphoid
Immobilization of scaphoid fractures difficult
Radial head fracture
Fracture of the radial head is common in young adults.
It is usually caused by a fall on the outstretched hand.
On examination, there is marked local tenderness over the head of the radius, impaired movements at the elbow, and a sharp pain at the lateral side of the elbow at the extremes of rotation (pronation and supination).
A man with lung cancer and bone metastasis in the thoracic spinal vertebral bodies, sustains a pathological fracture at the level of T4. The fracture is unstable and the spinal cord is severely compressed at this level. Which of the findings below will not be present 6 weeks after injury?
Extensor plantar reflexes
Spasticity of the lower limbs
Diminished patellar tendon reflex
Urinary incontinence
Sensory ataxia
A thoracic cord lesion causes spastic paraperesis, hyperrflexia and extensor plantar responses (UMN lesion), incontinence, sensory loss below the lesion and ‘sensory’ ataxia.These features typically manifest several weeks later, once spinal shock (in which areflexia predominates) has resolved.
Spinal cord
- Located in a canal within the vertebral column that affords it structural support.
Rostrally it continues to the medulla oblongata of the brain and caudally it tapers at a level corresponding to the L1-2 interspace (in the adult), a central structure, the filum terminale anchors the cord to the first coccygeal vertebra.
The spinal cord is characterised by cervico-lumbar enlargements and these, broadly speaking, are the sites which correspond to the brachial and lumbar plexuses respectively.
There are some key points to note when considering the surgical anatomy of the spinal cord:
- During foetal growth the spinal cord becomes shorter than the spinal canal, hence the adult site of cord termination at the L1-2 level.
- Due to growth of the vertebral column the spine segmental levels may not always correspond to bony landmarks as they do in the cervical spine.
- The spinal cord is incompletely divided into two symmetrical halves by a dorsal median sulcus and ventral median fissure. Grey matter surrounds a central canal that is continuous rostrally with the ventricular system of the CNS.
- The grey matter is sub divided cytoarchitecturally into Rexeds laminae.
- Afferent fibres entering through the dorsal roots usually terminate near their point of entry but may travel for varying distances in Lissauers tract. In this way they may establish synaptic connections over several levels
- At the tip of the dorsal horn are afferents associated with nociceptive stimuli. The ventral horn contains neurones that innervate skeletal muscle.
The key point to remember when revising CNS anatomy is to keep a clinical perspective in mind. So it is worth classifying the ways in which the spinal cord may become injured. These include:
Trauma either direct or as a result of disc protrusion
Neoplasia either by direct invasion (rare) or as a result of pathological vertebral fracture
Inflammatory diseases such as Rheumatoid disease, or OA (formation of osteophytes compressing nerve roots etc.
Vascular either as a result of stroke (rare in cord) or as complication of aortic dissection
Infection historically diseases such as TB, epidural abscesses.
The anatomy of the cord will, to an extent dictate the clinical presentation. Some points/ conditions to remember:
Brown- Sequard syndrome-Hemisection of the cord producing ipsilateral loss of proprioception and upper motor neurone signs, plus contralateral loss of pain and temperature sensation. The explanation of this is that the fibres decussate at different levels.
Lesions below L1 will tend to present with lower motor neurone signs
A 35 year old farm labourer injures the posterior aspect of his hand with a mechanical scythe. He severs some of his extensor tendons in this injury. How many tunnels lie in the extensor retinaculum that transmit the tendons of the extensor muscles?
One
Three
Four
Five
Six
There are six tunnels, each lined by its own synovial sheath.
Extensor retinaculum
The extensor retinaculum is a thickening of the deep fascia that stretches across the back of the wrist and holds the long extensor tendons in position.
Its attachments are:
The pisiform and triquetral medially
The end of the radius laterally
Structures related to the extensor retinaculum
Structures superficial to the retinaculum
Basilic vein
Dorsal cutaneous branch of the ulnar nerve
Cephalic vein
Superficial branch of the radial nerve
Structures passing deep to the extensor retinaculum
Extensor carpi ulnaris tendon
Extensor digiti minimi tendon
Extensor digitorum and extensor indicis tendon
Extensor pollicis longus tendon
Extensor carpi radialis longus tendon
Extensor carpi radialis brevis tendon
Abductor pollicis longus and extensor pollicis brevis tendons
Beneath the extensor retinaculum fibrous septa form six compartments that contain the extensor muscle tendons. Each compartment has its own synovial sheath.
The radial artery
The radial artery passes between the lateral collateral ligament of the wrist joint and the tendons of the abductor pollicis longus and extensor pollicis brevis.
Image illustrating the topography of tendons passing under the extensor retinaculum
Which of the following cell types is least likely to be found in a wound 1 week following injury?
Macrophages
Fibroblasts
Myofibroblasts
Endothelial cells
Neutrophils
Myofibroblasts are differentiated fibroblasts, in which the cytoskeleton contains actin filaments. These cell types facilitate wound contracture and are the hallmark of a mature wound. They are almost never found in wounds less than 1 month old.
Remember the question asks about the cell type asks about which cells are least likely to be found.
Phases of wound healing
Phase Key features Cells Timeframe
Haemostasis
Vasospasm in adjacent vessels
Platelet plug formation and generation of fibrin rich clot
Erythrocytes and platelets Seconds/ Minutes
Inflammation
Neutrophils migrate into wound (function impaired in diabetes).
Growth factors released, including basic fibroblast growth factor and vascular endothelial growth factor.
Fibroblasts replicate within the adjacent matrix and migrate into wound.
Macrophages and fibroblasts couple matrix regeneration and clot substitution.
Neutrophils, fibroblasts and macrophages Days
Regeneration
Platelet derived growth factor and transformation growth factors stimulate fibroblasts and epithelial cells.
Fibroblasts produce a collagen network.
Angiogenesis occurs and wound resembles granulation tissue.
Fibroblasts, endothelial cells, macrophages Weeks
Remodelling
Longest phase of the healing process and may last up to one year (or longer).
During this phase fibroblasts become differentiated (myofibroblasts) and these facilitate wound contraction.
Collagen fibres are remodelled.
Microvessels regress leaving a pale scar.
Myofibroblasts
Which main group of receptors does dobutamine bind to?
α-1
α-2
ß-1
ß-2
D-1
Dobutamine is a sympathomimetic with both alpha- and beta-agonist properties; it displays a considerable selectivity for beta1-cardiac receptors.
Inotropes and cardiovascular receptors
Inotropes are a class of drugs which work primarily by increasing cardiac output. They should be distinguished from vasoconstrictor drugs which are used specifically when the primary problem is peripheral vasodilatation.
Catecholamine type agents are commonly used and work by increasing cAMP levels by adenylate cyclase stimulation. This in turn increases intracellular calcium ion mobilisation and thus the force of contraction. Adrenaline works as a beta adrenergic receptor agonist at lower doses and an alpha receptor agonist at higher doses. Dopamine causes dopamine receptor mediated renal and mesenteric vascular dilatation and beta 1 receptor agonism at higher doses. This results in increased cardiac output. Since both heart rate and blood pressure are raised, there is less overall myocardial ischaemia. Dobutamine is a predominantly beta 1 receptor agonist with weak beta 2 and alpha receptor agonist properties. Noradrenaline is a catecholamine type agent and predominantly acts as an alpha receptor agonist and serves as a peripheral vasoconstrictor.
Phosphodiesterase inhibitors such as milrinone act specifically on the cardiac phosphodiesterase and increase cardiac output.
Inotrope Cardiovascular receptor action
Adrenaline α-1, α-2, β-1, β-2
Noradrenaline α-1,( α-2), (β-1), (β-2)
Dobutamine β-1, (β 2)
Dopamine (α-1), (α-2), (β-1), D-1,D-2
Minor receptor effects in brackets
Effects of receptor binding
α-1, α-2 vasoconstriction
β-1 increased cardiac contractility and HR
β-2 vasodilatation
D-1 renal and spleen vasodilatation
D-2 inhibits release of noradrenaline
A 66 year old male is admitted to the vascular ward for an amputation. He reports episodes of vertigo and dysarthria to the house officer. He suddenly collapses with a Glasgow Coma Score of 3. What is the most likely diagnosis?
Cerebral haemorrhage in left temporal parietal area
Opiate overdose
Cerebral haemorrhage in right temporal parietal area
Diazepam overdose
Basilar artery occlusion
Vertigo and dysarthria suggest a posterior circulation event. In the scenario of a patient complaining of posterior symptoms and a sudden deterioration in consciousness, the main differential diagnosis is of a basilar artery occlusion.
Stroke: types
Primary intracerebral haemorrhage (PICH, c. 10%)
Presents with headache, vomiting, loss of consciousness
Total anterior circulation infarcts (TACI, c. 15%)
Involves middle and anterior cerebral arteries
Hemiparesis/hemisensory loss
Homonymous hemianopia
Higher cognitive dysfunction e.g. Dysphasia
Partial anterior circulation infarcts (PACI, c. 25%)
Involves smaller arteries of anterior circulation e.g. upper or lower division of middle cerebral artery
Higher cognitive dysfunction or two of the three TACI features
Lacunar infarcts (LACI, c. 25%)
Involves perforating arteries around the internal capsule, thalamus and basal ganglia
Present with either isolated hemiparesis, hemisensory loss or hemiparesis with limb ataxia
Posterior circulation infarcts (POCI, c. 25%)
Vertebrobasilar arteries
Presents with features of brainstem damage
Ataxia, disorders of gaze and vision, cranial nerve lesions
Lateral medullary syndrome (posterior inferior cerebellar artery)
Wallenberg’s syndrome
Ipsilateral: ataxia, nystagmus, dysphagia, facial numbness, cranial nerve palsy
Contralateral: limb sensory loss
Weber’s syndrome
Ipsilateral III palsy
Contralateral weakness
Anterior cerebral artery
Contralateral hemiparesis and sensory loss, lower extremity > upper
Disconnection syndrome
Middle cerebral artery
Contralateral hemiparesis and sensory loss, upper extremity > lower
Contralateral hemianopia
Aphasia (Wernicke’s)
Gaze abnormalities
Posterior cerebral artery
Contralateral hemianopia with macular sparing
Disconnection syndrome
Lacunar
Present with either isolated hemiparesis, hemisensory loss or hemiparesis with limb ataxia
Lateral medulla (posterior inferior cerebellar artery)
Ipsilateral: ataxia, nystagmus, dysphagia, facial numbness, cranial nerve palsy e.g.
Horner’s
Contralateral: limb sensory loss
Pontine
VI nerve: horizontal gaze palsy
VII nerve
Contralateral hemiparesis
Which cranial nerve provides general sensation to the anterior two thirds of the tongue?
Facial
Trigeminal
Hypoglossal
Vagus
Glossopharyngeal
Taste to the anterior two thirds of the tongue is supplied by the facial nerve, the trigeminal supplies general sensation, this is mediated by the mandibular branch of the trigeminal nerve (via the lingual nerve).
Cranial nerves
Cranial nerve lesions
Olfactory nerve May be injured in basal skull fractures or involved in frontal lobe tumour extension. Loss of olfactory nerve function in relation to major CNS pathology is seldom an isolated event and thus it is poor localiser of CNS pathology.
Optic nerve Problems with visual acuity may result from intra ocular disorders. Problems with the blood supply such as amaurosis fugax may produce temporary visual distortion. More important surgically is the pupillary response to light. The pupillary size may be altered in a number of disorders. Nerves involved in the resizing of the pupil connect to the pretectal nucleus of the high midbrain, bypassing the lateral geniculate nucleus and the primary visual cortex. From the pretectal nucleus neurones pass to the Edinger - Westphal nucleus, motor axons from here pass along with the oculomotor nerve. They synapse with ciliary ganglion neurones; the parasympathetic axons from this then innervate the iris and produce miosis. The miotic pupil is seen in disorders such as Horner’s syndrome or opiate overdose.
Mydriasis is the dilatation of the pupil in response to disease, trauma, drugs (or the dark!). It is pathological when light fails to induce miosis. The radial muscle is innervated by the sympathetic nervous system. Because the parasympathetic fibres travel with the oculomotor nerve they will be damaged by lesions affecting this nerve (e.g. cranial trauma).
The response to light shone in one eye is usually a constriction of both pupils. This indicates intact direct and consensual light reflexes. When the optic nerve has an afferent defect the light shining on the affected eye will produce a diminished pupillary response in both eyes. Whereas light shone on the unaffected eye will produce a normal pupillary response in both eyes. This is referred to as the Marcus Gunn pupil and is seen in conditions such as optic neuritis. In a total CN II lesion shining the light in the affected eye will produce no response.
Oculomotor nerve The pupillary effects are described above. In addition it supplies all ocular muscles apart from lateral rectus and superior oblique. Thus the affected eye will be deviated inferolaterally. Levator palpebrae superioris may also be impaired resulting in impaired ability to open the eye.
Trochlear nerve The eye will not be able to look down.
Trigeminal nerve Largest cranial nerve. Exits the brainstem at the pons. Branches are ophthalmic, maxillary and mandibular. Only the mandibular branch has both sensory and motor fibres. Branches converge to form the trigeminal ganglion (located in Meckels cave). It supplies the muscles of mastication and also tensor veli palatine, mylohyoid, anterior belly of digastric and tensor tympani. The detailed descriptions of the various sensory functions are described in other areas of the website. The corneal reflex is important and is elicited by applying a small tip of cotton wool to the cornea, a reflex blink should occur if it is intact. It is mediated by: the naso ciliary branch of the ophthalmic branch of the trigeminal (sensory component) and the facial nerve producing the motor response. Lesions of the afferent arc will produce bilateral absent blink and lesions of the efferent arc will result in a unilateral absent blink.
Abducens nerve The affected eye will have a deficit of abduction. This cranial nerve exits the brainstem between the pons and medulla. It thus has a relatively long intra cranial course which renders it susceptible to damage in raised intra cranial pressure.
Facial nerve Emerges from brainstem between pons and medulla. It controls muscles of facial expression and taste from the anterior 2/3 of the tongue. The nerve passes into the petrous temporal bone and into the internal auditory meatus. It then passes through the facial canal and exits at the stylomastoid foramen. It passes through the parotid gland and divides at this point. It does not innervate the parotid gland. Its divisions are considered in other parts of the website. Its motor fibres innervate orbicularis oculi to produce the efferent arm of the corneal reflex. In surgical practice it may be injured during parotid gland surgery or invaded by malignancies of the gland and a lower motor neurone on the ipsilateral side will result.
Vestibulo-cochlear nerve Exits from the pons and then passes through the internal auditory meatus. It is implicated in sensorineural hearing loss. Individuals with sensorineural hearing loss will localise the sound in webers test to the normal ear. Rinnes test will be reduced on the affected side but should still work. These two tests will distinguish sensorineural hearing loss from conductive deafness. In the latter condition webers test will localise to the affected ear and Rinnes test will be impaired on the affected side. Surgical lesions affecting this nerve include CNS tumours and basal skull fractures. It may also be damaged by the administration of ototoxic drugs (of which gentamicin is the most commonly used in surgical practice).
Glossopharyngeal nerve Exits the pons just above the vagus. Receives sensory fibres from posterior 1/3 tongue, tonsils, pharynx and middle ear (otalgia may occur following tonsillectomy). It receives visceral afferents from the carotid bodies. It supplies parasympathetic fibres to the parotid gland via the otic ganglion and motor function to stylopharyngeaus muscle. The sensory function of the nerve is tested using the gag reflex.
Vagus nerve Leaves the medulla between the olivary nucleus and the inferior cerebellar peduncle. Passes through the jugular foramen and into the carotid sheath. Details of the functions of the vagus nerve are covered in the website under relevant organ sub headings.
Accessory nerve Exists from the caudal aspect of the brainstem (multiple branches) supplies trapezius and sternocleidomastoid muscles. The distal portion of this nerve is most prone to injury during surgical procedures.
Hypoglossal nerve Emerges from the medulla at the preolivary sulcus, passes through the hypoglossal canal. It lies on the carotid sheath and passes deep to the posterior belly of digastric to supply muscles of the tongue (except palatoglossus). Its location near the carotid sheath makes it vulnerable during carotid endarterectomy surgery and damage will produce ipsilateral defect in muscle function.
A 30 year old male is admitted electively for a right inguinal hernia repair under local anaesthesia. He is otherwise well but his grandfather died from a pulmonary embolism. What is the most appropriate form of thromboprophylaxis?
Administration of low dose low molecular weight heparin for 2 weeks
Administration of high dose low molecular weight heparin for 2 weeks
No prophylaxis
Low dose low molecular weight heparin and pneumatic compression stockings
High dose low molecular weight heparin and pneumatic compression stockings
Inguinal hernia repairs under local anaesthetic have a short operative time and patients are usually ambulant immediately afterwards. His family history is unlikely to be significant and he is at very low risk.
Thromboprophylaxis in surgical patients
Deep vein thrombosis may develop insidiously in many surgical patients. Untreated it may progress to result in pulmonary embolism.
The following surgical patients are at increased risk of deep vein thrombosis:
Surgery greater than 90 minutes at any site or greater than 60 minutes if the procedure involves the lower limbs or pelvis
Acute admissions with inflammatory process involving the abdominal cavity
Expected significant reduction in mobility
Age over 60 years
Known malignancy
Thrombophilia
Previous thrombosis
BMI >30
Taking hormone replacement therapy or the contraceptive pill
Varicose veins with phlebitis
Mechanical thromboprophylaxis
Early ambulation after surgery is cheap and is effective
Compression stockings (contra -indicated in peripheral arterial disease)
Intermittent pneumatic compression devices
Foot impulse devices
Therapeutic agents
Agent Mode of action Uses
Low molecular weight heparin Binds antithrombin causing inhibition of factor Xa Thromboprophylaxis or treatment of thromboembolic events in those with normal renal function. It is given as once daily subcutaneous injection
Unfractionated heparin Binds antithrombin III affecting thrombin and factor Xa Effective anticoagulation, administered intravenously it has a rapid onset and its therapeutic effects decline quickly on stopping and infusion. Its activity is measured using the APTT. If need be it can be reversed using protamine sulphate
Dabigatran Orally administered direct thrombin inhibitor Used prophylaxis in hip and knee surgery. It does not require therapeutic monitoring. It should not be used in any patient in whom there is a risk of active bleeding or imminent likelihood of surgery. It is reversed using Idarucizumab
A 23 year old man suffers a thermal injury to his left hand. It becomes red and painful. Which of the following mediators are not involved in this process?
Histamine
Free radicals
Prostaglandins
Leukotrienes
Serotonin
Acute inflammation is not mediated by free radicals
Chemical mediators facilitate the spread of inflammation into normal tissue
Chemical mediators include:
Lysosomal compounds
Chemokines such as serotinin and histamine (released by platelets and mast cells)
Other enzyme cascades producing inflammatory mediators include:
Complement, kinin, coagulation system and fibrinolytic system
Acute inflammation
Inflammation is the reaction of the tissue elements to injury. Vascular changes occur, resulting in the generation of a protein rich exudate. So long as the injury does not totally destroy the existing tissue architecture, the episode may resolve with restoration of original tissue architecture.
Vascular changes
Vasodilation occurs and persists throughout the inflammatory phase.
Inflammatory cells exit the circulation at the site of injury.
The equilibrium that balances Starlings forces within capillary beds is disrupted and a protein rich exudate will form as the vessel walls also become more permeable to proteins.
The high fibrinogen content of the fluid may form a fibrin clot. This has several important immunomodulatory functions.
Sequelae
Resolution
Typically occurs with minimal initial injury
Stimulus removed and normal tissue architecture results
Organisation
Delayed removal of exudate
Tissues undergo organisation and usually fibrosis
Suppuration
Typically formation of an abscess or an empyema
Sequestration of large quantities of dead neutrophils
Progression to chronic inflammation
Coupled inflammatory and reparative activities
Usually occurs when initial infection or suppuration has been inadequately managed
Causes
Infections e.g. Viruses, exotoxins or endotoxins released by bacteria
Chemical agents
Physical agents e.g. Trauma
Hypersensitivity reactions
Tissue necrosis
Presence of neutrophil polymorphs is a histological diagnostic feature of acute inflammation
A baby is found to have a Klumpke’s palsy post delivery. Which of the following is most likely to be present?
Partial loss of flexors of the wrist
Weak elbow flexion
Pronated forearm
Adducted shoulder
Shoulder medially rotated
Features of Klumpkes Paralysis
Claw hand (MCP joints extended and IP joints flexed)
Loss of sensation over medial aspect of forearm and hand
Horner’s syndrome
Loss of flexors of the wrist
A C8, T1 root lesion is called Klumpke’s paralysis and is caused by delivery with the arm extended.
Brachial plexus
The brachial plexus extends from the neck to the axilla. It is formed by the ventral rami of the fifth to the eighth cervical nerves with the ascending part of the first thoracic nerve.
Location of the plexus
The ventral rami which form the plexus enter the lower part of the posterior triangle of the neck in series with the ventral rami of the cervical plexus. The second part of the subclavian artery lies immediately anterior to the lower two rami. The upper three rami intermingle and pass inferolaterally towards the axilla and subclavian artery. They are enclosed within an extension of the prevertebral fascia. In the neck the plexus lies deep to platysma, the supraclavicular nerves, inferior belly of omohyoid and the transverse cervical artery. It then passes deep to the clavicle and the suprascapular vessels, to enter the axilla, and thence surround the second part of the axillary artery
Composition of the plexus
Ventral rami, the roots of the plexus, lie between scalenus medius and anterior.
As they enter the posterior triangle, the upper two (C5,6) and lower two (C8, T1) roots of the plexus unite to form the upper and lower trunks of the plexus respectively. Meanwhile, C7 continues as the middle trunk. The lower trunk may groove the superior surface of the first rib posterior to the subclavian artery, and the root from the first ventral ramus is always in contact with it.
Each trunk divides into ventral and dorsal divisions which are destined to supply the anterior (flexor) and posterior (extensor) parts of the upper limb.
The cords of the plexus are formed in the axilla. The dorsal divisions unite to form the posterior cord (C5-8). The ventral divisions of the upper and middle trunks unite to form the lateral cord (C5-7), while the ventral divisions of the lower trunk continues as the medial cord (C8-T1). The cords are named according to their relationship to the axillary artery. Each cord terminates by dividing into two main branches at the beginning of the third part of the artery.
Sympathetic communications
The fifth and sixth cervical ventral rami receive grey rami communicantes from the middle cervical ganglion, while the two or more grey rami communicantes pass from the inferior cervical ganglion to the seventh and eighth cervical ventral rami. The first thoracic ventral ramus receives its grey ramus from the cervicothoracic ganglion. Its for this reason that inferior plexus injury can be complicated by a Horners syndrome.
Summary
Origin Anterior rami of C5 to T1
Sections of the plexus
Roots, trunks, divisions, cords, branches
Mnemonic:Real Teenagers Drink Cold Beer
Roots
Located in the posterior triangle
Pass between scalenus anterior and medius
Trunks
Located posterior to middle third of clavicle
Upper and middle trunks related superiorly to the subclavian artery
Lower trunk passes over 1st rib posterior to the subclavian artery
Divisions Apex of axilla
Cords Related to axillary artery
A 43 year old lady presents with urinary incontinence. At which of the following locations is Onufs nucleus likely to be found?
Medulla oblongata
Anterior horn of L5 nerve roots
Micturition centre in the Pons
Anterior horn of S2 nerve roots
None of the above
Onufs nucleus is located in the anterior horn of S2 and is the origin of neurones to the external urethral sphincter.
Urinary incontinence
Involuntary passage of urine. Most cases are female (80%). It has a prevalence of 11% in those aged greater than 65 years. The commonest variants include:
Stress urinary incontinence (50%)
Urge incontinence (15%)
Mixed (35%)
Males
Males may also suffer from incontinence although it is a much rarer condition in men. A number of anatomical factors contribute to this. Males have 2 powerful sphincters; one at the bladder neck and the other in the urethra. Damage to the bladder neck mechanism is a factor in causing retrograde ejaculation following prostatectomy. The short segment of urethra passing through the urogenital diaphragm consists of striated muscle fibres (the external urethral sphincter) and smooth muscle capable of more sustained contraction. It is the latter mechanism that maintains continence following prostatectomy.
Females
The sphincter complex at the level of bladder neck is poorly developed in females. As a result the external sphincter complex is functionally more important, its composition being similar to that of males. Innervation is via the pudendal nerve and the neuropathy that may accompany obstetric events may compromise this and lead to stress urinary incontinence.
Innervation
Somatic innervation to the bladder is via the pudendal, hypogastric and pelvic nerves. Autonomic nerves travel in these nerve fibres too. Bladder filling leads to detrusor relaxation (sympathetic) coupled with sphincter contraction. The parasympathetic system causes detrusor contraction and sphincter relaxation. Overall control of micturition is centrally mediated via centres in the Pons.
Stress urinary incontinence
50% of cases, especially in females.
Damage (often obstetric) to the supporting structures surrounding the bladder may lead to urethral hypermobility.
Other cases due to sphincter dysfunction, usually from neurological disorders (e.g. Pudendal neuropathy, multiple sclerosis).
Urethral mobility:
Pressure not transmitted appropriately to the urethra resulting in involuntary passage of urine during episodes of raised intra-abdominal pressure.
Sphincter dysfunction:
Sphincter fails to adapt to compress urethra resulting in involuntary passage of urine. When the sphincter completely fails there is often to continuous passage of urine.
Urge incontinence
In these patients there is sense of urgency followed by incontinence. The detrusor muscle in these patients is unstable and urodynamic investigation will demonstrate overactivity of the detrusor muscle at inappropriate times (e.g. Bladder filling). Urgency may be seen in patients with overt neurological disorders and those without. The pathophysiology is not well understood but poor central and peripheral co-ordination of the events surrounding bladder filling are the main processes.
Assessment
Careful history and examination including vaginal examination for cystocele.
Bladder diary for at least 3 days
Consider flow cystometry if unclear symptomatology or surgery considered and diagnosis is unclear.
Exclusion of other organic disease (e.g. Stones, UTI, Cancer)
Management
Conservative measures should be tried first; Stress urinary incontinence or mixed symptoms should undergo 3 months of pelvic floor exercise. Over active bladder should have 6 weeks of bladder retraining.
Drug therapy for women with overactive bladder should be offered oxybutynin (or solifenacin if elderly) if conservative measures fail.
In women with detrusor instability who fail non operative therapy a trial of sacral neuromodulation may be considered, with conversion to permanent implant if good response. Augmentation cystoplasty is an alternative but will involve long term intermittent self catheterisation.
In women with stress urinary incontinence a urethral sling type procedure may be undertaken. Where cystocele is present in association with incontinence it should be repaired particularly if it lies at the introitus.
NICE guidelines
Initial assessment urinary incontinence should be classified as stress/urge/mixed.
At least 3/7 bladder diary if unable to classify easily.
Start conservative treatment before urodynamic studies if a diagnosis is obvious from the history
Urodynamic studies if plans for surgery.
Stress incontinence: Pelvic floor exercises 3/12, if fails consider surgery.
Urge incontinence: Bladder training >6/52, if fails for oxybutynin (antimuscarinic drugs) then sacral nerve stimulation.
Pelvic floor exercises offered to all women in their 1st pregnancy.
A cervical rib is due to which of the following?
Hyperplasia of the annulus fibrosus
Proliferation of the nucleus pulposus
Fusion of the transverse processes of the 6th and 7th cervical vertebrae
An accessory cervical vertebra
Elongation of the transverse processes of the 7th cervical vertebra
Cervical ribs occur as a result of the elongation of the transverse process of the 7th cervical vertebra. It is usually a fibrous band that attaches to the first thoracic rib.
Cervical ribs
0.2-0.4% incidence
Most cases present with neurological symptoms
Consist of an anomalous fibrous band that often originates from C7 and may arc towards, but rarely reaches the sternum
Congenital cases may present around the third decade, some cases are reported to occur following trauma
Bilateral in up to 70%
Compression of the subclavian artery may produce absent radial pulse on clinical examination and in particular may result in a positive Adsons test (lateral flexion of the neck towards the symptomatic side and traction of the symptomatic arm- leads to obliteration of radial pulse)
Treatment is most commonly undertaken when there is evidence of neurovascular compromise. A transaxillary approach is the traditional operative method for excision.
A 22 year old man suffers a compound fracture of the tibia. During attempted surgical repair the deep peroneal nerve is divided. Which of the following muscles will not be affected as a result?
Tibialis anterior
Peroneus longus
Extensor hallucis longus
Extensor digitorum longus
Peroneus tertius
Peroneus longus is innervated by the superficial peroneal nerve (L4, L5, S1).
A 50 year old lady is admitted having fallen down some stairs sustaining multiple rib fractures 36 hours previously. On examination, she is confused and agitated and has clinical evidence of lateralising signs. She deteriorates further and then dies with no response to resuscitation. What is the most likely explanation?
Intraventricular haemorrhage
Acute sub dural haemorrhage
Chronic sub dural haematoma
Sub arachnoid haemorrhage
Extra dural haematoma
The time frame of deterioration of an acute sub dural bleed would fit with this scenario. They are highly lethal and not uncommon injuries. As the bleed enlarges, lateralising signs may be seen and eventually coning and death will occur.
Intra cranial haemorrhage
Extradural haematoma Bleeding into the space between the dura mater and the skull. Often results from acceleration-deceleration trauma or a blow to the side of the head. The majority of extradural haematomas occur in the temporal region where skull fractures cause a rupture of the middle meningeal artery.
Features
Raised intracranial pressure
Some patients may exhibit a lucid interval
Subdural haematoma Bleeding into the outermost meningeal layer. Most commonly occur around the frontal and parietal lobes. May be either acute or chronic.
Risk factors include old age and alcoholism.
Slower onset of symptoms than a extradural haematoma.
Intracerebral haematoma Usually hyperdense lesions on CT scanning. Arise in areas of traumatic contusion which fuse to become a haematoma. Areas of clot and fresh blood may co-exist on the same CT scan (Swirl sign). Large haematomas and those associated with mass effect should be evacuated.
Subarachnoid haemorrhage Usually occurs spontaneously in the context of a ruptured cerebral aneurysm but may be seen in association with other injuries when a patient has sustained a traumatic brain injury
Intraventricular haemorrhage Haemorrhage that occurs into the ventricular system of the brain. It is relatively rare in adult surgical practice and when it does occur, it is typically associated with severe head injuries. In premature neonates it may occur spontaneously. The blood may clot and occlude CSF flow, hydrocephalus may result.
In neonatal practice the vast majority of IVH occur in the first 72 hours after birth, the aetiology is not well understood and it is suggested to occur as a result of birth trauma combined with cellular hypoxia, together with the delicate neonatal CNS.
An 18 year old man develops a severe spreading sepsis of the hand. The palm is explored surgically and the flexor digiti minimi brevis muscle is mobilised to facilitate drainage of the infection. Which of the following structures is not closely related to this muscle?
The hook of hamate
Median nerve
Superficial palmar arterial arch
Digital nerves arising from the ulnar nerve
None of the above
The flexor digiti minimi brevis originates from the Hamate, on its under- surface lie the ulnar contribution to the superficial palmar arterial arch and digital nerves derived from the ulnar nerve. The median nerve overlies the flexor tendons.
Hand
Anatomy of the hand
Bones
8 Carpal bones
5 Metacarpals
14 phalanges
Intrinsic Muscles 8 Interossei - Supplied by ulnar nerve
4 palmar-adduct fingers
4 dorsal- abduct fingers
Intrinsic muscles Lumbricals
Flex MCPJ and extend the IPJ.
Origin deep flexor tendon and insertion dorsal extensor hood mechanism.
Innervation: 1st and 2nd- median nerve, 3rd and 4th- deep branch of the ulnar nerve.
Thenar eminence
Abductor pollicis brevis
Opponens pollicis
Flexor pollicis brevis
Hypothenar eminence
Opponens digiti minimi
Flexor digiti minimi brevis
Abductor digiti minimi
Fascia and compartments of the palm
The fascia of the palm is continuous with the antebrachial fascia and the fascia of the dorsum of the hand. The palmar fascia is thin over the thenar and hypothenar eminences. In contrast, the central palmar fascia is relatively thick. The palmar aponeurosis covers the soft tissues and overlies the flexor tendons. The apex of the palmar aponeurosis is continuous with the flexor retinaculum and the palmaris longus tendon. Distally, it forms four longitudinal digital bands that attach to the bases of the proximal phalanges, blending with the fibrous digital sheaths.
A medial fibrous septum extends deeply from the medial border of the palmar aponeurosis to the 5th metacarpal. Lying medial to this are the hypothenar muscles. In a similar fashion, a lateral fibrous septum extends deeply from the lateral border of the palmar aponeurosis to the 3rd metacarpal. The thenar compartment lies lateral to this area.
Lying between the thenar and hypothenar compartments is the central compartment. It contains the flexor tendons and their sheaths, the lumbricals, the superficial palmar arterial arch and the digital vessels and nerves.
The deepest muscular plane is the adductor compartment, which contains adductor pollicis.
Short muscles of the hand
These comprise the lumbricals and interossei. The four slender lumbrical muscles flex the fingers at the metacarpophalangeal joints and extend the interphalangeal joint. The four dorsal interossei are located between the metacarpals and the four palmar interossei lie on the palmar surface of the metacarpals in the interosseous compartment of the hand.
Long flexor tendons and sheaths in the hand
The tendons of FDS and FDP enter the common flexor sheath deep to the flexor retinaculum. The tendons enter the central compartment of the hand and fan out to their respective digital synovial sheaths. Near the base of the proximal phalanx, the tendon of FDS splits to permit the passage of FDP. The FDP tendons are attached to the margins of the anterior aspect of the base of the distal phalanx.
The fibrous digital sheaths contain the flexor tendons and their synovial sheaths. These extend from the heads of the metacarpals to the base of the distal phalanges.
Palmar Interossei
Note that there are 4 palmar interossei. The first is a small slip of muscle which arises from the ulnar side of the base of the first metacarpal and passes between the head of the first dorsal interosseous and the oblique head of adductor pollicis to insert into the ulnar base of the of the proximal phalanx of the thumb. The second arises from the ulnar side of the body of the second metacarpal and is inserted into the ulnar side of the extensor hood of the index. The third and fourth palmar interossei arise from the radial sides of the bodies of the 4th and 5th metacarpals respectively and insert into the radial sides of the extensor hoods of the ring and little fingers.
A 78 year old man complains of a long history of shoulder pain and more recently weakness. On examination, active attempts at abduction are impaired. Passive movements are normal. What is the most likely diagnosis?
Rotator cuff tear
Osteoarthritis
Metastatic malignancy
Adhesive capsulitis
Calcific tendonitis
Rotator cuff tears are common in elderly people and may occur following minor trauma or as a result of long standing impingement. Tears greater than 2cm should generally be repaired surgically. The length of the history in this scenario is suggestive of a tear complicating impingement.
Shoulder disorders
Shoulder fractures and dislocations
Fractures
Proximal humerus
Background
Third most common fragility fracture in the elderly.
Results from low energy fall in predominantly elderly females, or from high energy trauma in young males.
Can be associated with nerve injury (commonly axillary), and fracture-dislocation of the humeral head. Detailed neurological assessment is essential for all upper limb injuries.
Anatomy
Osteology
Consists of articular head, greater tuberosity, lesser tuberosity, metaphysis and diaphysis. Between the articular head and the tuberosities is the anatomical neck (previous physis). Between the tuberosities and the metaphysis is the surgical neck.
The supraspinatus, infraspinatus and teres minor muscles attach to the greater tuberosity. The subscapularis muscle attaches to the lesser tuberosity.
Vascular Supply
Humeral head is supplied by the anterior and posterior humeral circumflex arteries. Anatomical neck fractures are at greatest risk of osteonecrosis.
Imaging
Imaging aims to both delineate the fracture pattern, and confirm/exlude the presence of an associated dislocation.
Radiographs - True anteroposterior (AP), axillary lateral and/or scapula Y view.
CT - indicated to better define intra-articular involvement and to aid pre-operative planning. MRI is not useful for fracture imaging.
Classification
Description of the fracture is often more useful than classification. Particular attention should be paid to humeral alignment, fracture displacement, and greater tuberosity position (rotator cuff will pull the GT supero-posterioly, which can cause impingement problems with malunion).
- Neer Classification: Most commonly used. Describes fracture as 2,3,or 4 part depending upon the number main fragments. Also comments on the degree of displacement. Fragments:
-greater tuberosity
-lesser tuberosity
- articular surface
- shaft
Displacement: >1cm or angulation >45 degrees.
Treatment
The vast majority of proximal humeral fractures are minimally displaced, and therefore can be managed conservatively. This involves immobilisation in a polysling, and progressive mobilisation. Pendular exercise can commence at 14 days, and active abduction from 4-6 weeks.
Irreducible fracture dislocation is an indication for operative management. Other indications include large displacement, younger patient, head splitting (intra-articular fractures). However, the recent PROFHER trial (1) has suggested no benefit to operative intervention on patient outcome (it must be applied cautiously as majority of patients were elderly with extraarticular fractures). Options available for surgical management include:
ORIF Most commonly used. Plate and screw fixation. Can reconstruct complex fractures.
Intramedullary nail Suitable for extra-articular configuration, predominantly surgical neck +/- GT fractures.
Hemiarthroplasty Used for un-reconstructable fractures in the older patient who has good glenoid quality.
Total shoulder arthroplasty Unconstructable fractures where high functioning shoulder is required (hemiarthroplasty will cause glenoid erosion)
Reverse shoulder arthroplasty Total shoulder arthroplasty that provides better functional outcome than conventional total shoulder replacement.
Scapula
Background
Uncommon fractures usually associated with high energy trauma. Most commonly involve scapula body or spine (50%), glenoid fossa and glenoid neck. Important to exclude associated life threatening injury.
Imaging
Plain radiographs should include true anteroposterior (AP), axillary lateral and/or scapula Y view. CT scanning is useful for defining intra-articular involvement, displacement and for three dimensional reconstruction.
Classification
Based on the location of the fracture (coracoid, acromion, glenoid neck, glenoid fossa, scapula body). Beware of ipsilateral glenoid neck and clavicle fracture -floating shoulder - where limb is effectively dissociated from axial skeleton.
Treatment
The vast majority of scapula fractures are amenable to conservative management, consisting of sling immobilisation for two weeks followed by early rehabilitation. Floating shoulder will usually require fixation, and consideration of surgery should also be given to intra-articular and displaced/angulated glenoid fractures.
Dislocations
Types
Dislocations around the shoulder joint include glenohumeral dislocation, acromioclavicular joint disruption and sternoclavicular dislocation. Only glenohumeral dislocation will be covered here.
Glenohumeral dislocation
Diagnosis, classification and management are covered here.
Background
Shoulder dislocation is commonly seen in A&E. It has a high recurrence rate that is as high as 80% in teenagers. Initial management requires emergent reduction to prevent lasting chondral damage.
Early assessment and management
Usually a traumatic cause (multi-directional instability in frequent dislocations requires discussion with orthopaedics and is not covered here). Careful history, examination and documentation of neurovascular status of the limb, in particular the axillary nerve (regimental badge sensation). This should be re-assessed post manipulation. Early radiographs to confirm direction of dislocation.
Initial management consists of emergent closed reduction under under entanox and analgesia, but often requires conscious sedation. Arm should then be immobilised in a polysling, and XR to confirm relocation.
Imaging - True anteroposterior (AP), axillary lateral and/or scapula Y view. Reduced humeral head should lie between acromion and coracoid on lateral/scapula view.
Types
Direction Features Cause Examination Reduction techniques
Anterior Most Common >90% Usually traumatic - anterior force on arm when shoulder is abducted, externally rotated Loss of shoulder contour - sulcus sign. Humeral head can be felt anteriorly.
Hippocratic.
Milch.
Stimson.
Kocher not advised due to complication of fracture
Posterior 50% missed in A&E 50% traumatic, but classically post seizure or electrocution Shoulder locked in internal rotation. XR may show lightbulb appearance. Gentle lateral traction to adducted arm.
Inferior Rare Associated with pectorals and rotator cuff tears, and glenoid fracture As for primary injury Management of primary injury
Superior Rare Associated with acrominon/clavicle fracture As for primary injury Management of primary injury
Associated injuries
Bankart lesion - avulsion of the anterior glenoid labrum with an anterior shoulder dislocation (reverse Bankart if poster labrum in posterior dislocation).
Hill Sachs defect - chondral impaction on posteriosuperior humeral head from contact with gleonoid rim. Can be large enough to lock shoulder, requiring open reduction. (Reverse Hill Sachs in posterior dislocation).
Rotator cuff tear - increases with age.
Greater or lesser tuberosity fracture - increases with age.
Humeral neck fracture - shoulder fracture dislocation. More common in high energy trauma and elderly. Should be discussed with orthopaedics prior to any attempted reduction.
Rotator Cuff Disease
Rotator cuff disease is a spectrum of conditions that ranges from subacromial impingement to rotator cuff tears and eventually to rotator cuff arthropathy (arthritis).
Anatomy
The rotator cuff is a group of four muscles that are important in shoulder movements, and maintenance of glenohumeral stability.
Muscle Scapular attachment Humeral attachment Action Innervation
Supraspinatus Supraspinatus fossa Superior facet of greater tuberosity Initiation of abduction of humerus Suprascapular nerve
Infraspinatus Infraspinatus fossa Posterior facet of greater tuberosity External rotation of humerus Suprascapular nerve
Teres Minor Lateral border Inferior facet of greater tuberosity External rotation of humerus Axillary Nerve
Subscapularis Subscapular fossa Lesser tuberosity Internal rotation of humerus Upper and lower subscapular nerve
The inferior rotator cuff muscles (infraspinatus, teres minor, and subscapularis) balance the superior pull of the deltoid. Injury/tear results in upward migration of the humeral head on the glenoid (can be seen on AP radiograph).
Likewise, the anterior muscles (subscapularis) are balanced with the posterior muscles (infraspinatus, teres minor).
Subacromial Impingement
The most common cause of shoulder pain, which results from impingement of the superior cuff on the undersurface of the acromion, and an inflammatory bursitis.
Associated with certain types of acromial morphology (Bigliani classification).
Presents as insidious pain which is exacerbated by overhead activities.
Rotator Cuff Tear
Often presents as an acute event on the background of chronic subacromial impingement in the older patient, but can present as an avulsion injury in younger patients.
Majority of tears are to the superior cuff (supraspinatus, infraspinatus, teres minor), though a tear to subscapularis is associated with subcoracoid impingement.
Tears present as pain and weakness when using the muscles in question.
Rotator Cuff Arthropathy
Defined as shoulder arthritis in the setting of rotator cuff dysfunction. Results from superior migration due to the loss of rotator cuff function and integrity. Unopposed deltoid pulls the humeral head superiorly.
Associated with massive chronic cuff tears.
Imaging
Plain radiographs
AP of the shoulder may show superior migration of the humerus with a cuff tear, and features of arthritis with arthropathy. Other causes of pain may also be identified (e.g. calcific tendonitis/fracture)
Outlet view is useful for defining the acromial morphology
USS
Allows dynamic imaging of the cuff, and is inexpensive. However, it is very user dependent.
MRI
Best imaging modality for cuff pathology.
Also allows imaging of the rest of the shoulder. When intra-articular pathology is suspected, can be combined with an arthrogram for improved sensitivity and specificity.
Treatment
Subacromial impingement
Physiotherapy, oral anti-inflammatory medication
Subacromial steroid injection can settle inflammation
Arthroscopic subacromial decompression by shaving away the undersurface of the acromion, more space is created for the rotator cuff. Cuff integrity is assessed also at time of surgery, and can be repaired if necessary.
Rotator cuff tear
When considering repair of a cuff tear, the age and activity of the patient, the nature of the tear (degenerative vs. acute traumatic), and the size and retraction of the tear should be considered when making a surgical plan.
Mild tears or tears in the elderly can be managed conservatively, as outlined above.
Moderate tears can be repaired arthroscopically. Massive or retracted tears will often require an open repair (occasionally with a tendon transfer). Subacromial decompression is performed at the same time to reduce impingement, symptoms and recurrence.
Calcific tendonitis
Calcific tendonitis involves calcific deposits within tendons anywhere in the body, but most commonly in the rotator cuff (specifically the supraspinatus tendon). When present in the shoulder, it is associated with subacromial impingement and pain.
Pathology
More common in women aged 30-60 years.
Association with diabetes and hypothyroidism
There are three stages of calcification
Formative phase characterized by calcific deposits
Resting phase deposit is stable, but presents with impingement problems
Resorptive phase phagocytic resorption. Most painful stage.
Presentation
Similar in presentation to subacromial impingement, with pain especially with over head activities. Atraumatic in nature.
Imaging
Plain radiographs show calcification of the rotator cuff, usually within 1.5cm of its insertion on the humerus. Supraspinatus outlet views can show level of impingment. Further imaging is rarely needed.
Treatment
Non-operative NSAIDS, steroid injection (controversial, but practiced) and physiotherapy. Approximately 75% will resolve by 6 months with conservative management.
Ultrasound guided or surgical needle barbotage can break down deposits and resolve symptoms. Occasionally surgical excision is required.
Adhesive capsulitis (Frozen Shoulder)
Pain and loss of movement of shoulder joint, which involves fibroplastic proliferation of capsular tissue, causing soft tissue scarring and contracture. Patients present with a painful and decreased arc of motion.
Associated with prolonged immobilization, previous surgery, thyroid disorders (AI) and diabetes
Classically three stages which can take up to two years to resolve:
Stage one the freezing and painful stage
Stage two the frozen and stiff stage
Stage three the thawing stage, where shoulder movement slowly improves
Imaging
Plain radiographs to exclude other causes of a painful shoulder
MRI arthrogram may show capsular contracture, and again may be used to exclude cuff pathology. However, often not performed as diagnosis is largely clinical.
Treatment
Non-operative NSAIDS, steroid injection and physiotherapy. Patience is required as condition can take up to 2 years to improve.
Operative MUA or arthroscopic adhesiolysis (release of adhesions) can expedite recovery, followed by intensive physiotherapy.
Glenohumeral Arthritis
Background
May be osteoarthritis (primary or secondary to cuff tear or trauma), rheumatoid arthritis, or as part of a spondyloarthropathy. Majority of those with RA will develop symptoms.
More common in the elderly
Presents like any other arthritis - pain at night and with movement
Imaging
AP and axillary radiographs will show features of arthritis.
CT/MRI is often useful to classify the shape of the glenoid and extent of bone loss when considering arthroplasty. MRI also essential to asses integrity of rotator cuff if considering shoulder replacement.
Treatment
Like all orthopaedics, start with simple measures:
NSAIDS, management of RA, physiotherapy, steroid injection.
Hemiarthroplasty can sometimes be considered if glenoid is in excellent condition or if patient has large comorbidity.
Arthroscopic debridement is useful if patient has isolated ACJ arthritis, but is rarely used for glenohumeral arthritis.
Total shoulder replacement is shown to produce superior outcome when compared to hemiarthroplasty in terms of pain relief, function and implant survival.
Total shoulder replacement can be anatomical (ball on humerus, with cup on glenoid), or reverse geometry (ball on glenoid, with cup on humerus). Anatomical TSR requires an in tact rotator cuff, so often reverse is preferable when the cuff if questionable in integrity.
References
1. JAMA. 2015;313(10):1037-1047. doi:10.1001/jama.2015.1629
A 21 year old man is involved in a road traffic accident. After a transient period of concussion he is found to have a GCS of 15 by the paramedics. On arrival at hospital he is monitored in a side room of the emergency department. When he is next observed he is noted to have a GCS of 3 and a blown right pupil. Which of the processes below best accounts for this deterioration?
Hydrocephalus
Intraventricular bleed
Sub dural bleed
Trans tentorial herniation
Sub arachnoid haemorrhage
The presence of a blown right pupil is a sign of a third cranial nerve compression. The most likely cause is an extradural bleed. However, since this option is not listed the process of trans tentorial herniation would be the most applicable answer. Intraventricular bleeds are typically more common in premature neonates, deterioration due to hydrocephalus is more chronic.
Head injuries
Head injury is the commonest cause of death and disability in people aged 1-40 years in the UK. In the UK 1.4 million people will attend emergency departments each year with a recent head injury.
The typical patterns are described below:
Extradural haematoma Bleeding into the space between the dura mater and the skull. Often results from acceleration-deceleration trauma or a blow to the side of the head. The majority of extradural haematomas occur in the temporal region where skull fractures cause a rupture of the middle meningeal artery.
Features
Raised intracranial pressure
Some patients may exhibit a lucid interval
Subdural haematoma Bleeding into the outermost meningeal layer. Most commonly occur around the frontal and parietal lobes. May be either acute or chronic.
Risk factors include old age and alcoholism.
Slower onset of symptoms than a extradural haematoma.
Subarachnoid haemorrhage Usually occurs spontaneously in the context of a ruptured cerebral aneurysm but may be seen in association with other injuries when a patient has sustained a traumatic brain injury
Which of the nerves listed below is responsible for the innervation of gluteus maximus?
Inferior gluteal nerve
Superior gluteal nerve
Posterior femoral cutaneous nerve
Sciatic nerve
Perineal nerve
Superior and inferior gluteal nerve
Superior gluteal nerve
Arises from dorsal surface of the sacral plexus (L4, L5, S1)
Passes into gluteal region together with superior gluteal vessels
Supplies gluteus medius and minimus
Inferior gluteal nerve
Arises from dorsal surface of sacral plexus (L5, S1 and S2)
Runs medial to the posterior femoral cutaneous nerve
Enters gluteal region at inferior border of piriformis
Supplies gluteus maximus
Which of the following statements relating to branchial cysts is untrue?
The greater auricular nerve may be divided during excision
They typically occur in young adults
They move upwards on swallowing
They are rare over the age of 40 years
They are usually located in the anterior triangle of the neck
Nerves at risk during branchial cyst excision: Mandibular branch of facial nerve, greater auricular nerve and accessory nerve.
They do not move on swallowing. They should be diagnosed with caution in those aged >40 years, as lumps in this age group may in fact be metastatic disease from oropharyngeal cancer.
Neck lumps
The table below gives characteristic exam question features for conditions causing neck lumps:
Reactive lymphadenopathy By far the most common cause of neck swellings. There may be a history of local infection or a generalised viral illness
Lymphoma Rubbery, painless lymphadenopathy
The phenomenon of pain whilst drinking alcohol is very uncommon
There may be associated night sweats and splenomegaly
Thyroid swelling May be hypo-, eu- or hyperthyroid symptomatically
Moves upwards on swallowing
Thyroglossal cyst More common in patients < 20 years old
Usually midline, between the isthmus of the thyroid and the hyoid bone
Moves upwards with protrusion of the tongue
May be painful if infected
Pharyngeal pouch More common in older men
Represents a posteromedial herniation between thyropharyngeus and cricopharyngeus muscles
Usually not seen, but if large then a midline lump in the neck that gurgles on palpation
Typical symptoms are dysphagia, regurgitation, aspiration and chronic cough
Cystic hygroma A congenital lymphatic lesion (lymphangioma) typically found in the neck, classically on the left side
Most are evident at birth, around 90% present before 2 years of age
Branchial cyst An oval, mobile cystic mass that develops between the sternocleidomastoid muscle and the pharynx
Develop due to failure of obliteration of the second branchial cleft in embryonic development
Usually present in early adulthood
Cervical rib More common in adult females
Around 10% develop thoracic outlet syndrome
Carotid aneurysm Pulsatile lateral neck mass which doesn’t move on swallowing
A 56 year old machinist has his arm entrapped in a steel grinder and is brought to the emergency department. On examination, he is unable to extend his metacarpophalangeal joints and abduct his shoulder. He has weakness of his elbow and wrist. What has been injured?
Ulnar nerve
Axillary nerve
Medial cord of brachial plexus
Lateral cord of brachial plexus
Posterior cord of brachial plexus
The posterior cord gives rise to:
Radial nerve ((innervates the triceps, brachioradialis, wrist extensors, and finger extensors)
Axillary nerve (innervates deltoid and teres minor)
Upper subscapular nerve (innervates subscapularis)
Lower subscapular nerve (innervates teres major and subscapularis)
Thoracodorsal nerve (innervates latissimus dorsi)
This is a description of a posterior cord lesion. Remember that the posterior cord gives rise to the axillary and radial nerve.
A man is stabbed in the chest to the right of the manubriosternal angle. Which structure is least likely to be injured in this case?
Right pleura
The trachea
Right phrenic nerve
Right recurrent laryngeal nerve
Brachiocephalic vein
The right recurrent laryngeal nerve branches off the right vagus more proximally and arches posteriorly round the subclavian artery. So of the structures given it is the least likely to be injured.
Mediastinum
Region between the pulmonary cavities.
It is covered by the mediastinal pleura. It does not contain the lungs.
It extends from the thoracic inlet superiorly to the diaphragm inferiorly.
Mediastinal regions
Superior mediastinum (between manubriosternal angle and T4/5)
Middle mediastinum
Posterior mediastinum
Anterior mediastinum
Region Contents
Superior mediastinum
Superior vena cava
Brachiocephalic veins
Arch of aorta
Thoracic duct
Trachea
Oesophagus
Thymus
Vagus nerve
Left recurrent laryngeal nerve
Phrenic nerve
Anterior mediastinum
Thymic remnants
Lymph nodes
Fat
Middle mediastinum
Pericardium
Heart
Aortic root
Arch of azygos vein
Main bronchi
Posterior mediastinum
Oesophagus
Thoracic aorta
Azygos vein
Thoracic duct
Vagus nerve
Sympathetic nerve trunks
Splanchnic nerves
A 45 year old man has widespread metastatic adenocarcinoma of the colon. Which of these tumour markers is most likely to be elevated?
CA19-9
Carcinoembryonic antigen
Alpha Feto Protein
CA 125
Beta HCG
Screening for colonic cancer using CEA is not justified
Carcinoembryonic antigen is elevated in colonic cancer, typically in relation to disease extent with highest serum levels noted in metastatic disease. It is falsely elevated in a number of non-malignant disease states such as cirrhosis and colitis and for this reason it has no role in monitoring colitics for colonic cancer[1].
Reference
1. Sturgeon, C.M., L.C. Lai, and M.J. Duffy. Serum tumour markers: how to order and interpret them. BMJ, 2009. 339: p. b3527.
Colorectal cancer screening and diagnosis
Overview
Most cancers develop from adenomatous polyps. Screening for colorectal cancer has been shown to reduce mortality by 16%
The NHS now has a national screening programme offering screening every 2 years to all men and women aged 60 to 69 years. Patients aged over 70 years may request screening
Eligible patients are sent faecal occult blood (FOB) tests through the post. This is being replaced by FIT testing.
Patients with abnormal results are offered a colonoscopy
- The NHS BOSS flexible sigmoidoscopy screening comprises a single flexible sigmoidoscopy to patients aged 55 years
At colonoscopy, approximately:
5 out of 10 patients will have a normal exam
4 out of 10 patients will be found to have polyps which may be removed due to their premalignant potential
1 out of 10 patients will be found to have cancer
Diagnosis
Essentially the following patients need referral:
- Altered bowel habit for more than six weeks
- New onset of rectal bleeding
- Symptoms of tenesmus
Colonoscopy is the gold standard, provided it is complete and good mucosal visualisation is achieved. Other options include double contrast barium enema and CT colonography.
Staging
Once a malignant diagnosis is made patients with colonic cancer will be staged using chest / abdomen and pelvic CT. Patients with rectal cancer will also undergo evaluation of the mesorectum with pelvic MRI scanning.
For examination purposes the Dukes and TNM systems are preferred.
Tumour markers
Carcinoembryonic antigen (CEA) is the main tumour marker in colorectal cancer. Not all tumours secrete this, and it may be raised in conditions such as IBD. However, absolute levels do correlate (roughly) with disease burden and it is once again being used routinely in follow up.
A 28 year old man has a carcinoid tumour identified in his appendix. Blood testing for which of the substances listed below is likely to be helpful during follow up?
CA19-9
Alkaline phosphatase
AFP
CEA
Chromogranin A
It is important to distinguish between blood and urine tests for carcinoid syndrome. Blood tests usually measure chromogranin A,neuron-specific enolase (NSE), substance P, and gastrin. Urine tests usually measure 5 HIAA, which is a metabolite of serotonin. Sometimes blood tests for 5 hydroxytryptamine (serotonin) are also performed.
Carcinoid syndrome
Carcinoid tumours secrete serotonin
Originate in neuroendocrine cells mainly in the intestine (midgut-distal ileum/appendix)
Can occur in the rectum, bronchi
Hormonal symptoms mainly occur when disease spreads outside the bowel
Clinical features
Onset: insidious over many years
Flushing face
Palpitations
Pulmonary valve stenosis and tricuspid regurgitation causing dyspnoea
Asthma
Severe diarrhoea (secretory, persists despite fasting)
Investigation
5-HIAA in a 24-hour urine collection
Somatostatin receptor scintigraphy
CT scan
Blood testing for chromogranin A
Treatment
Octreotide
Surgical removal
A 44 year old lady presents with a mass lesion in the upper outer quadrant of the left breast. On clinical examination she has a 2cm mass lesion which on core biopsy is demonstrated to have invasive ductal carcinoma. An FNA of a bulky axillary lymph node contains malignant cells. What is the correct course of action?
Wide local excision and axillary node clearance
Radical mastectomy and axillary node clearance
Simple mastectomy and sentinel node biopsy
Wide local excision and sentinel node biopsy
Excision biopsy and sentinel node biopsy
Although the primary lesion is small enough for breast conserving surgery, the presence of overt axillary lymph node metastasis will attract a recommendation for axillary node clearance. Note that an excision biopsy is not appropriate for malignant cases.
Breast cancer management
- Surgery is performed in most patients suffering from breast cancer.
Chemotherapy may be used to downstage tumours and allow breast conserving surgery. Hormonal therapy may also be used for the same purposes.
Radiotherapy is given to most patients who have undergone breast conserving surgery (some older patients receiving hormone treatment and who have small low grade tumours may safely avoid DXT.
Therapeutic mammoplasty is an option for some patients but requires symmetrizing surgery in most cases.
Patients who have undergone mastectomy may be offered a reconstructive procedure either in conjunction with their primary resection or as a staged procedure at a later date.
Surgical options
Mastectomy vs Wide local excision
Mastectomy Wide Local Excision
Multifocal tumour Solitary lesion
Central tumour Peripheral tumour
Large lesion in small breast Small lesion in large breast
DCIS >4cm DCIS <4cm
Patient Choice Patient choice
Central lesions may be managed using breast conserving surgery, where an acceptable cosmetic result may be obtained, this is rarely the case in small breasts
Axillary disease
As a minimum, all patients with invasive breast cancer should have their axilla staged. In those who do not have overt evidence of axillary nodal involvement this can be undertaken using sentinel lymph node biopsy.
Patients with a positive sentinel lymph node biopsy or who have imaging and cytological or histological evidence of axillary nodal metastasis should undergo axillary node clearance or axillary irradiation.
Axillary node clearance is associated with the development of lymphoedema, increased risk of cellulitis and frozen shoulder.
Which of the ABPI measurements shown below is most likely to be found in a 43 year old lady with long standing diabetes who complains of foot pain. It is worse at night and during minor exercise.
> 1.0
0.9
0.3
0.5
0.7
Diabetes may be complicated by vessel calcification and neuropathic pain. Therefore individuals may present with pain which is atypical for claudication both in terms of its tempo of onset and location.
Ankle-Brachial pressure index
Measurement of ankle- brachial pressure index (ABPI) is a commonly performed vascular investigation.
Calculated by dividing lower limb pressure by the highest upper limb pressure.
Results of ABPI
1.2 or greater Usually due to vessel calcification
1.0- 1.2 Normal
0.8-1.0 Minor stenotic lesion
Initiate risk factor management
0.50-0.8 Moderate stenotic lesion
Consider duplex
Risk factor management
If mixed ulcers present then avoid tight compression bandages
0.3 - 0.5 Likely significant stenosis
Duplex scanning to delineate lesions needed
Compression bandaging contra indicated
Less than 0.3 Indicative of critical ischaemia
Urgent detailed imaging required
Which of the following drugs causes hyperkalaemia?
Heparin
Ciprofloxacin
Salbutamol
Levothyroxine
Codeine phosphate
Both unfractionated and low-molecular weight heparin can cause hyperkalaemia. This is thought to be caused by inhibition of aldosterone secretion. Salbutamol is a recognised treatment for hyperkalaemia.
Hyperkalaemia
- Plasma potassium levels are regulated by a number of factors including aldosterone, acid-base balance and insulin levels.
Metabolic acidosis is associated with hyperkalaemia as hydrogen and potassium ions compete with each other for exchange with sodium ions across cell membranes and in the distal tubule.
ECG changes seen in hyperkalaemia include tall-tented T waves, small P waves, widened QRS leading to a sinusoidal pattern and asystole
Causes of hyperkalaemia
Acute renal failure
Drugs*: potassium sparing diuretics, ACE inhibitors, angiotensin 2 receptor blockers, spironolactone, ciclosporin, heparin**
Metabolic acidosis
Addison’s
Tissue necrosis/rhabdomyolysis: burns, trauma
Massive blood transfusion
Foods that are high in potassium
Salt substitutes (i.e. Contain potassium rather than sodium)
Bananas, oranges, kiwi fruit, avocado, spinach, tomatoes
*beta-blockers interfere with potassium transport into cells and can potentially cause hyperkalaemia in renal failure patients - remember beta-agonists, e.g. Salbutamol, are sometimes used as emergency treatment
**both unfractionated and low-molecular weight heparin can cause hyperkalaemia. This is thought to be caused by inhibition of aldosterone secretion
Which of the following is not a risk factor for developing tuberculosis?
Gastrectomy
Solid organ transplantation with immunosupression
Intravenous drug use
Haematological malignancy
Amiodarone
Risk factors for developing active tuberculosis include:
silicosis
chronic renal failure
HIV positive
solid organ transplantation with immunosuppression
intravenous drug use
haematological malignancy
anti-TNF treatment
previous gastrectomy
Tuberculosis
Tuberculosis (TB) is an infection caused by Mycobacterium tuberculosis that most commonly affects the lungs. Understanding the pathophysiology of TB can be difficult - the key is to differentiate between primary and secondary disease.
Primary tuberculosis
A non-immune host who is exposed to M. tuberculosis may develop primary infection of the lungs. A small lung lesion known as a Ghon focus develops. The Ghon focus is composed of tubercle-laden macrophages. The combination of a Ghon focus and hilar lymph nodes is known as a Ghon complex
In immunocompetent people the initial lesion usually heals by fibrosis. Those who are immunocompromised may develop disseminated disease (miliary tuberculosis).
Secondary (post-primary) tuberculosis
If the host becomes immunocompromised the initial infection may become reactivated. Reactivation generally occurs in the apex of the lungs and may spread locally or to more distant sites. Possible causes of immunocomprise include:
immunosuppressive drugs including steroids
HIV
malnutrition
The lungs remain the most common site for secondary tuberculosis. Extra-pulmonary infection may occur in the following areas:
central nervous system (tuberculous meningitis - the most serious complication)
vertebral bodies (Pott’s disease)
cervical lymph nodes (scrofuloderma)
renal
gastrointestinal tract
Which of the following is a content of the adductor canal?
Saphenous nerve
Sural nerve
Femoral nerve
Profunda branch of the femoral artery
Saphenous vein
It contains the saphenous nerve, femoral vein and the superficial branch of the femoral artery.
Adductor canal
Also called Hunter’s or subsartorial canal
Immediately distal to the apex of the femoral triangle, lying in the middle third of the thigh. Canal terminates at the adductor hiatus.
Borders Contents
Laterally Vastus medialis muscle Saphenous nerve
Posteriorly Adductor longus, adductor magnus Superficial femoral artery
Roof Sartorius Superficial femoral vein
Which of these statements relating to the external carotid is false?
It ends by bifurcating into the superficial temporal and ascending pharyngeal artery
Its first branch is the superior thyroid artery
The superior thyroid, lingual and facial arteries all arise from its anterior surface
The ascending pharyngeal artery is a medial branch
Initially it lies anteromedial to the internal carotid
It terminates by dividing into the superficial temporal and maxillary branches. The external carotid has eight branches, 3 from its anterior surface ; thyroid, lingual and facial. The pharyngeal artery is a medial branch. The posterior auricular and occipital are posterior branches.
External carotid artery
The external carotid commences immediately lateral to the pharyngeal side wall. It ascends and lies anterior to the internal carotid and posterior to the posterior belly of digastric and stylohyoid. More inferiorly it is covered by sternocleidomastoid, passed by hypoglossal nerves, lingual and facial veins.
It then pierces the fascia of the parotid gland finally dividing into its terminal branches within the gland itself.
Surface marking of the carotid
This is an imaginary line drawn from the bifurcation of the common carotid passing behind the angle of the jaw to a point immediately anterior to the tragus of the ear.
Branches of the external carotid artery
It has six main branches, three in front, two behind and one deep.
Three in front Superior thyroid
Lingual
Facial
Two behind Occipital
Posterior auricular
Deep Ascending pharyngeal
It terminates by dividing into the superficial temporal and maxillary arteries in the parotid gland.
A 22 year old man sustains a blow to the side of his head with a baseball bat during a fight. He is initially conscious. However, he subsequently loses consciousness and then dies. Post mortem examination shows an extradural haematoma. The most likely culprit vessel is a branch of which of the following?
Middle cerebral artery
Internal carotid artery
Anterior cerebral artery
Maxillary artery
Mandibular artery
The middle meningeal artery is the most likely source of the extradural haematoma in this setting. It is a branch of the maxillary artery. The middle cerebral artery does not give rise to the middle meningeal artery. Note that the question is asking for the vessel which gives rise to the middle meningeal artery (‘the likely culprit vessel is a branch of which of the following’)
Middle meningeal artery
Middle meningeal artery is typically the third branch of the first part of the maxillary artery, one of the two terminal branches of the external carotid artery. After branching off the maxillary artery in the infratemporal fossa, it runs through the foramen spinosum to supply the dura mater (the outermost meninges) .
The middle meningeal artery is the largest of the three (paired) arteries which supply the meninges, the others being the anterior meningeal artery and the posterior meningeal artery.
The middle meningeal artery runs beneath the pterion. It is vulnerable to injury at this point, where the skull is thin. Rupture of the artery may give rise to an extra dural hematoma.
In the dry cranium, the middle meningeal, which runs within the dura mater surrounding the brain, makes a deep indention in the calvarium.
The middle meningeal artery is intimately associated with the auriculotemporal nerve which wraps around the artery making the two easily identifiable in the dissection of human cadavers and also easily damaged in surgery.
A 73 year old man undergoes an excision biopsy of a lymph node that is closely applied to sternocleidomastoid. This muscle is mobilized and a nerve that is present is damaged. Which muscle below is most likely to be affected?
Trapezius
Rhomboid major
Deltoid
Supraspinatus
Rhomboid minor
The accessory nerve has a number of lymph nodes applied to it near the sternocleidomastoid muscle. It is particularly at risk if SCM is mobilized. If injured, the trapezius muscle and SCM will be paralysed.
Trapezius
Origin Medial third of the superior nuchal line of the occiput
External occipital protruberance
Ligamentum nuchae
Spines of C7 and all thoracic vertebrae and all intervening interspinous ligaments
Insertion Posterior border of the lateral third of the clavicle
Medial border of the acromion
Upper border of the crest of the spine of the scapula
Nerve supply Spinal portion of the accessory nerve
Actions Elevation of the shoulder girdle
Lateral rotation of the scapula
A 39 year old man notices a swelling in his left hemiscrotum. On examination he has a left sided varicocele. The ipsilateral testis is normal on palpation. What is the most appropriate course of action?
A-Scrotal exploration and ligation of the varicocele
B-Abdominal ultrasound
C-Scrotal ultrasound
D-Left orchidectomy
E-Discharge
B-Abdominal ultrasound
A left sided varicocele is a recognized presenting sign of a renal tumour occluding the renal vein (into which left testicular vein drains). An abdominal ultrasound should be undertaken to exclude this. Surgery for uncomplicated varicocele is usually unnecessary.
A 3 month old boy is brought to the clinic by his mother who has noticed a swelling in the right inguinal region. On examination, there is a firm mass affecting the right spermatic cord distally, the testis is felt separately from it. What is the most likely diagnosis?
A-Inguino scrotal hernia
B-Rhabdomyosarcoma
C-Lymphatic filariasis
D-Torsion of testicular hydatid
E-Hydrocele
B-Rhabdomyosarcoma
Rhabdomyosarcomas are paratesticular tumours with a bimodal distribution. Because the mass is felt separate to the testis, this is the more likely diagnosis.
• 5% of testicular tumors
• Most often arises of distal portion of spermatic cord and may invade testis or surounding tissues
• Bimodal age distribution - 3-4 months - 16 years
• Arises from mesenchymal tissue - 90% embryonal variant (better prognosis) - 30% - 50% have metastasis (usually lymph node) at diagnosis
An 8 year-old boy of Caribbean descent presents with periumbilical abdominal pain. He has vomited twice and is refusing fluids. His temperature is 38.1oC and blood tests are as follows: Haemoglobin 8 g/dl, WCC 13 x 109/l, with a neutrophilia. What is the most likely diagnosis?
Pancreatitis
Sickle cell crisis
Appendicitis
Intussusception
Spontaneous bacterial peritonitis
Anaemia is seldom seen in appendicitis and if present should prompt a search for an alternative underlying diagnosis.
Sickle cell anaemia is characterised by severe chronic haemolytic anaemia resulting from poorly formed erythrocytes. Painful crises result from vaso-occlusive episodes, which may occur spontaneously or may be precipitated by infection. Consider this diagnosis in all children of appropriate ethnic background.
Acute abdominal pain-diagnoses
Conditions presenting with acute abdominal pain
Condition Features Investigations Management
Appendicitis History of migratory pain.
Fever.
Anorexia.
Evidence of right iliac fossa tenderness.
Mild pyrexia. Differential white cell count
Pregnancy test
C-Reactive protein
Amylase
Urine dipstick testing Appendicectomy
Mesenteric adenitis Usually recent upper respiratory tract infection.
High fever.
Generalised abdominal discomfort- true localised pain and signs are rare. Full blood count- may show slightly raised white cell count
Urine dipstick often normal
Abdominal ultrasound scan - usually no free fluid Conservative management- appendicectomy if diagnostic doubt
Mittelschmerz Only seen in females
Mid cycle pain
Usually occurs two weeks after last menstrual period
Pain usually has a supra-pubic location
Usually subsides over a 24-48 hour period. Full blood count- normal
Urine dipstick- normal
Abdominal and pelvic ultrasound- may show a trace of pelvic free fluid Manage conservatively if doubt or symptoms fail to settle then laparoscopy
Fitz-Hugh Curtis syndrome Disseminated infection with Chlamydia.
Usually seen in females.
Consists of evidence of pelvic inflammatory disease together with peri-hepatic inflammation and subsequent adhesion formation. Abdominal ultrasound scan- may show free fluid
High vaginal swabs - may show evidence of sexually transmitted infections Usually medically managed- doxycycline or azithromycin
Abdominal aortic aneurysm (ruptured) Sudden onset of abdominal pain radiating to the back in older adults (look for risk factors).
Collapse.
May be moribund on arrival in casualty, more stable if contained haematoma.
Careful clinical assessment may reveal pulsatile mass. Patients who are haemodynamically stable should have a CT scan Unstable patients should undergo immediate surgery (unless it is not in their best interests).
Those with evidence of contained leak on CT should undergo immediate surgery
Increasing unruptured aneurysmal size is an indication for urgent surgical intervention (that can wait until the next working day)
Perforated peptic ulcer Sudden onset of pain (usually epigastric).
Often preceding history of upper abdominal pain.
Soon develop generalised abdominal pain.
On examination may have clinical evidence of peritonitis. Erect CXR may show free air. A CT scan may be indicated where there is diagnostic doubt Laparotomy (laparoscopic surgery for perforated peptic ulcers is both safe and feasible in experienced hands)
Intestinal obstruction Colicky abdominal pain and vomiting (the nature of which depends on the level of the obstruction).
Abdominal distension and constipation (again depending upon site of obstruction).
Features of peritonism may occur where local necrosis of bowel loops is occurring. A plain abdominal film may help with making the diagnosis. A CT scan may be useful where diagnostic uncertainty exists In those with a virgin abdomen a lower and earlier threshold for laparotomy should exist than in those who may have adhesional obstruction
Mesenteric infarction Embolic events present with sudden pain and forceful evacuation.
Acute on chronic events usually have a longer history and previous weight loss.
On examination the pain is typically greater than the physical signs would suggest. Arterial pH and lactate
Arterial phase CT scanning is the most sensitive test Immediate laparotomy and resection of affected segments, in acute embolic events SMA embolectomy may be needed.
A 45 year old man with recurrent episodes of confusion is found to have a 1.5cm insulinoma of the pancreatic head. What is the most appropriate management?
Whipples procedure
Total pancreatectomy and en bloc splenectomy
Pylorus preserving pancreatico duodenectomy
Enucleation of the lesion
External beam radiotherapy
Most insulinomas are benign and radical resection is therefore not justified.
Insulinoma
- Insulin producing tumours of the pancreatic β cells
Incidence of 1 per 1,000,000 per year
90% of lesions are benign
Most tumours less than 2cm in size
Between 5 and 10% have MEN type 1
75% of patients with MEN 1 will develop pancreatic islet cell tumours
Typical features of insulinoma
Symptomatic hypoglycaemia during fasting
Concomitant blood glucose of less than 3mmol/L
Relief of hypoglycaemia by use of glucose
Testing
When neuroglycopenic symptoms occur blood is taken for serum insulin levels, serum glucose, C-peptide and pro insulin concentrations. The plasma insulin concentration is >10 micro U/ml in patients with the disorder.
Tumour localisation
USS (25% accuracy), endoscopic USS better (75% accuracy)
CT scanning (pancreatic protocol=40% accuracy)
Malignant insulinomas are larger and diagnostic accuracy with MRI is nearly 100% in such cases
Somatostatin receptor scintigraphy (50% accuracy)
Treatment
Since the majority of tumours are benign; the blind segmental resection of the pancreas (e.g. Whipples) cannot be justified, this may be considered acceptable for malignant lesions. The best approach at laparotomy is to corroborate pre operative imaging with intraoperative ultrasonography to identify the lesion. Tumours may be close to the pancreatic duct and this must be appreciated by the operating surgeon. The perioperative use of octreotide reduces the amount of pancreatic drainage, but not overall complications.
Which of the following is not a change found on an ECG in acute pulmonary embolism?
No changes
J waves
P pulmonale
Right ventricular strain
T wave inversion in the inferior leads
S1, Q3, T3
J waves are pathognomonic of hypothermia.
Pulmonary Embolism: ECG changes
- No changes
S1, Q3, T3
Tall R waves: V1
P pulmonale (peaked P waves): inferior leads
Right axis deviation, Right bundle branch block
Atrial arrhythmias
T wave inversion: V1, V2, V3
Right ventricular strain: if identified is associated with adverse short-term outcome and adds prognostic value to echocardiographic evidence of right ventricular dysfunction in patients with acute pulmonary embolism and normal blood pressure.
References
Vanni S et al. Prognostic value of ECG among patients with acute pulmonary embolism and normal blood pressure. Am J Med. 2009 Mar;122(3):257-64.
A 23 year old man presents with blunt abdominal trauma and a splenic bleed is suspected. He is commenced on an infusion of tranexamic acid. Which of the following best describes its mechanism of action?
Inhibition of plasmin
Inhibition of thrombin
Inhibition of factor II
Inhibition of factor Xa
Activation of factor VIII
Tranexamic acid inhibits plasmin and this prevents fibrin degradation.
Tranexamic acid
Tranexamic acid is a synthetic derivative of lysine. Its primary mode of action is as an anti fibrinolytic that competitively inhibits the conversion of plasminogen to plasmin. Plasmin degrades fibrin and therefore rendering plasmin inactive slows this process.
The role of tranexamic acid in trauma was investigated in the CRASH 2 trial and has been shown to be of benefit in bleeding trauma when administered in the first 3 hours.
A 33 year old lady presents to the emergency department following a road traffic accident. Her evaluation demonstrates a mild degree of anaemia and on examination, the spleen is palpable. In her family history, her mother underwent a splenectomy in her fifth decade. Which of the following laboratory test findings is most likely?
Positive Paul Bunnell test
Blood film showing elliptical cells
Blood film showing infection with plasmodium vivax
Reed Sternberg cells within the splenic tissue
Spur cells
It is likely that the patient has hereditary spherocytosis. Given the familial nature, EBV infection is unlikely and so a Paul Bunnell test will be negative. Lymphoma would be unusual and Spur cells are seen in association with liver disease and / or renal failure.
Hereditary Spherocytosis
Most common disorder of the red cell membrane, it has an incidence of 1 in 5000. The abnormally shaped erythrocytes are prone to splenic sequestration and destruction. This can result in hyperbilirubinaemia, jaundice and splenomegaly. In older patients an intercurrent illness may increase the rate of red cell destruction resulting in more acute symptoms.
Severe cases may benefit from splenectomy.
A 62 year old man is identified as having a rectal cancer. Following diagnostic work up no metastatic disease is identified. The tumour is 2cm from the anal verge. On MRI, lesion is T2, N0. Which of the following represents the correct course of action?
Proceed to abdomino-perineal excision of the colon and rectum (ELAPE)
Undertake a low anterior resection and loop ileostomy
Undertake a Hartmanns procedure
Offer radical external beam radiotherapy followed by abdomino-perineal excision of the colon and rectum (ELAPE)
Offer radical radiotherapy followed by low anterior resection and loop ileostomy
The tumour is too low for restorative surgery to be considered with an acceptable functional outcome. The tumour will therefore require an ELAPE style abdomino perineal resection. Since the lesion is T2 there is no prognostic benefit from adding radiotherapy which will confer additional morbidity.
Colorectal cancer treatment
Patients diagnosed as having colorectal cancer should be completely staged using CT of the chest/ abdomen and pelvis. Their entire colon should have been evaluated with colonoscopy or CT colonography. Patients whose tumours lie below the peritoneal reflection should have their mesorectum evaluated with MRI.
Once their staging is complete patients should be discussed within a dedicated colorectal MDT meeting and a treatment plan formulated.
Treatment of colonic cancer
Cancer of the colon is nearly always treated with surgery. Stents, surgical bypass and diversion stomas may all be used as palliative adjuncts. Resectional surgery is the only option for cure in patients with colon cancer. The procedure is tailored to the patient and the tumour location. The lymphatic drainage of the colon follows the arterial supply and therefore most resections are tailored around the resection of particular lymphatic chains (e.g. ileo-colic pedicle for right sided tumours). Some patients may have confounding factors that will govern the choice of procedure, for example a tumour in a patient from a HNPCC family may be better served with a panproctocolectomy rather than segmental resection. Following resection the decision has to be made regarding restoration of continuity. For an anastomosis to heal the key technical factors include; adequate blood supply, mucosal apposition and no tissue tension. Surrounding sepsis, unstable patients and inexperienced surgeons may compromise these key principles and in such circumstances it may be safer to construct an end stoma rather than attempting an anastomosis.
When a colonic cancer presents with an obstructing lesion; the options are to either stent it or resect. In modern practice it is unusual to simply defunction a colonic tumour with a proximal loop stoma. This differs from the situation in the rectum (see below).
Following resection patients with risk factors for disease recurrence are usually offered chemotherapy, a combination of 5FU and oxaliplatin is common.
Rectal cancer
The management of rectal cancer is slightly different to that of colonic cancer. This reflects the rectum’s anatomical location and the challenges posed as a result. Tumours located in the rectum can be surgically resected with either an anterior resection or an abdomino - perineal resection. The technical aspects governing the choice between these two procedures can be complex to appreciate and the main point to appreciate for the MRCS is that involvement of the sphincter complex or very low tumours require APER. In the rectum a 2cm distal clearance margin is required and this may also impact on the procedure chosen. In addition to excision of the rectal tube an integral part of the procedure is a meticulous dissection of the mesorectal fat and lymph nodes (total mesorectal excision/ TME). In rectal cancer surgery invovlement of the cirumferential resection margin carries a high risk of disease recurrence. Because the rectum is an extraperitoneal structure (until you remove it that is!) it is possible to irradiate it, something which cannot be offered for colonic tumours. This has a major impact in rectal cancer treatment and many patients will be offered neoadjuvent radiotherapy (both long and short course) prior to resectional surgery. Patients with T1, 2 and 3 /N0 disease on imaging do not require irradiation and should proceed straight to surgery. Patients with T4 disease will typically have long course chemo radiotherapy. Patients presenting with large bowel obstruction from rectal cancer should not undergo resectional surgery without staging as primary treatment (very different from colonic cancer). This is because rectal surgery is more technically demanding, the anastomotic leak rate is higher and the danger of a positive resection margin in an unstaged patient is high. Therefore patients with obstructing rectal cancer should have a defunctioning loop colostomy.
Summary of procedures
The operations for cancer are segmental resections based on blood supply and lymphatic drainage. These commonly performed procedures are core knowledge for the MRCS and should be understood.
Site of cancer Type of resection Anastomosis Risk of leak
Right colon Right hemicolectomy Ileo-colic Low <5%
Transverse Extended right hemicolectomy Ileo-colic Low <5%
Splenic flexure Extended right hemicolectomy Ileo-colic Low <5%
Splenic flexure Left hemicolectomy Colo-colon 2-5%
Left colon Left hemicolectomy Colo-colon 2-5%
Sigmoid colon High anterior resection Colo-rectal 5%
Upper rectum Anterior resection (TME) Colo-rectal 5%
Low rectum Anterior resection (Low TME) Colo-rectal
(+/- Defunctioning stoma) 10%
Anal verge Abdomino-perineal excision of colon and rectum None n/a
In the emergency setting, where the bowel has perforated, the risk of an anastomotic breakdown is much greater, particularly when the anastomosis is colon-colon. In this situation, an end colostomy is often safer and can be reversed later. When resection of the sigmoid colon is performed and an end colostomy is fashioned the operation is referred to as a Hartmans procedure. Whilst left sided resections are more risky, ileo-colic anastomoses are relatively safe even in the emergency setting and do not need to be defunctioned.
References
A review of the diagnosis and management of colorectal cancer and a summary of the UK National Institute of Clinical Excellence guidelines is provided in:
Poston G, et al . Diagnosis and management of colorectal cancer: summary of NICE guidance. BMJ 2011: 343: d 6751.
Parasympathetic fibres innervating the parotid gland originate from which of the following?
Submandibular ganglion
Otic ganglion
Ciliary ganglion
Pterygopalatine ganglion
None of the above
Otic ganglion
Secretion of saliva by the parotid gland is controlled by nerve fibres originating in the inferior salivatory nucleus; these leave the brain via the tympanic nerve (branch of glossopharyngeal nerve (CN IX), travel through the tympanic plexus (located in the middle ear), and then form the lesser petrosal nerve until reaching the otic ganglion. After synapsing in the Otic ganglion, the postganglionic (postsynaptic) fibres travel as part of the auriculotemporal nerve (a branch of the mandibular nerve (V3) to reach the parotid gland.
Parotid gland
Anatomy of the parotid gland
Location Overlying the mandibular ramus; anterior and inferior to the ear.
Salivary duct Crosses the masseter, pierces the buccinator and drains adjacent to the 2nd upper molar tooth (Stensen’s duct).
Structures passing through the gland
Facial nerve (Mnemonic: The Zebra Buggered My Cat; Temporal Zygomatic, Buccal, Mandibular, Cervical)
External carotid artery
Retromandibular vein
Auriculotemporal nerve
Relations
Anterior: masseter, medial pterygoid, superficial temporal and maxillary artery, facial nerve, stylomandibular ligament
Posterior: posterior belly digastric muscle, sternocleidomastoid, stylohyoid, internal carotid artery, mastoid process, styloid process
Arterial supply Branches of external carotid artery
Venous drainage Retromandibular vein
Lymphatic drainage Deep cervical nodes
Nerve innervation
Parasympathetic-Secretomotor
Sympathetic-Superior cervical ganglion
Sensory- Greater auricular nerve
Parasympathetic stimulation produces a water rich, serous saliva. Sympathetic stimulation leads to the production of a low volume, enzyme-rich saliva.
A 63 year old man undergoes a sub total gastrectomy for carcinoma of the stomach. Which of the sequelae below is least likely to occur?
Metabolic bone disease
Bile reflux
Dumping syndrome
Zinc deficiency
B12 deficiency
Zinc is mainly absorbed in the duodenum and jejunum. Bile reflux may occur post gastrectomy. The risk of bile reflux is lower if a Roux en Y reconstruction is used.
Post gastrectomy syndromes
Post gastrectomy syndromes may vary slightly depending upon whether a total or partial gastrectomy is performed. A Roux en Y reconstruction generally gives the best functional outcomes. Where a gastrojejunostomy is performed as reconstruction following a distal gastrectomy the gastric emptying is generally better if the jejunal limbs are tunneled in the retrocolic plane.
The following may occur following gastrectomy:
Small capacity (early satiety)
Dumping syndrome
Bile gastritis
Afferent loop syndrome
Efferent loop syndrome
Anaemia (B12 deficiency)
Metabolic bone disease
An injured axillary artery is ligated between the thyrocervical trunk of the subclavian and subscapular artery. Subsequent collateral circulation is likely to result in reversal of blood flow in which of the vessels listed below?
Circumflex scapular artery
Transverse cervical artery
Posterior intercostal arteries
Suprascapular artery
Profunda brachii artery
It’s an easy question really, we just made the wording difficult (on purpose). It is asking about the branches of the axillary artery and knowledge of the fact that there is an extensive collateral network around the shoulder joint. As a result, the occlusion of the proximal aspect of the circumflex humeral inflow (from the axillary artery) ceases and there is then retrograde flow through it from collaterals.
The circumflex scapular artery is a branch of the subscapular artery and normally supplies the muscle on the dorsal aspect of the scapula. In this instance, flow is reversed in the circumflex scapular and subscapular arteries forming a collateral circulation around the scapula.
Axillary artery
The axillary artery extends from the outer border of the first rib to the lower border of teres major, where it becomes the brachial artery. The vessel is subdivided into three zones; the first part lies above pectoralis minor, the second part is behind the muscle and the third part lies inferior to it.
First part
Together with the axillary vein, the artery is enclosed within the cords of the brachial plexus. Both vessels are contained within the axillary sheath, a prolongation of the prevertebral fascia. Posteriomedial to the sheath lies the first intercostal space, the superior aspect of the serratus anterior and the long thoracic nerve. Within the sheath, the medial cord of the brachial plexus lies behind the artery. Anteriorly lies the clavipectoral fascia. Superolaterally, lie the lateral and posterior cords of the brachial plexus. Inferomedially lies the axillary vein.
Second part
Posterior to the second part lies the posterior cord of the brachial plexus and the subscapularis muscle. Anteriorly, lie pectoralis minor and major. The lateral cord of the brachial plexus lies laterally. Medially, lies the medial cord of the brachial plexus, here it separates the artery from the vein.
Third part
Posterior to the artery lie subscapularis, latissimus dorsi and teres major. Interspersed between the vessel and subscapularis are the axillary and radial nerves. Anterior to the vessel is the medial root of the median nerve. Laterally, the lies the median and musculocutaneous nerves and coracobrachialis. The axillary vein is related medially.
Branches of the axillary artery
Highest thoracic artery
Thoraco-acromial artery
Lateral thoracic artery
Subscapular artery
Posterior circumflex humeral artery
Anterior circumflex humeral artery
A 64 year old man has a suspected lymphoma and lymph node biopsy from the posterolateral aspect of the right neck is planned. Which of the nerves listed is at greatest risk?
Accessory
Long thoracic
External laryngeal
Facial
Vagus
The accessory nerve has a superficial course and is easily injured. It lies under platysma and may be divided during the early part of the procedure.
Accessory nerve
The cranial root of the accessory nerve arises from the caudal two thirds of the nucleus ambiguus and the caudal four fifths of the dorsal nucleus of the vagus. The cranial root emerges as four rootlets from the dorsolateral surface of the medulla oblongata below those of the vagus. It then traverses the jugular foramen. On exiting the jugular foramen it separates from its spinal part. Whereupon its cranial fibres joint those of the vagus to innervate some of the palatal muscles. The fibres arising from the spinal root exit near the junction between the spinal cord and the medulla. The fibres pass rostrally to unite with the cranial roots to exit through the jugular foramen. As outlined above these separate on exiting the foramen. The spinal part then crosses the transverse process of the atlas, and is crossed by the occipital artery as it does so.
It descends obliquely, medial to the styloid process, stylohyoid and the posterior belly of digastric. It then reaches the upper part of sternocleidomastoid to enter its upper surface. It typically exits this muscle a little above the midpoint of the posterior aspect of it. This point is usually 4-6cm below the tip of the mastoid process. It crosses the posterior triangle on the levator scapulae separated from it by the pre vertebral layer of deep cervical fascia. At this point, the nerve is superficial and related to the superficial cervical lymph nodes. Approximately 3-5 cm above the clavicle it passes behind the anterior border of trapezius which it innervates.
A 26 year old man who smokes heavily develops aching, crampy pains in his legs. On examination, distal limb pulses are diminished. What is the most likely cause?
Vasculitis
Steal syndrome
Clot embolus
Vasospasm
Arterial injury
This is likely to represent Buergers disease. It is commonest in young males who smoke heavily.
Arterial occlusions/ insufficiency
Arterial occlusions may occur as a result of a number of processes. The typical clinical scenarios are outlined below.
Cause of occlusion Typical picture
Embolus Sudden onset
Depending upon level of occlusion; limb may show typical features of pain, loss of pulses and pallor. Sensory perceptive changes may also be present
Thrombosis Usually known disease and prodromal symptoms e.g. claudication
Disruption to flow may be incomplete
If background disease process present then collaterals may be present and picture less dramatic
Vasospasm May be due to Raynauds and affect extremities
Symptoms are often temperature related
Discolouration of the hands may occur (pale, dark, red)
Symptoms improve during pregnancy (hyperdynamic circulation)
Steal syndromes Occur secondary to arteriovenous fistula, or partial arterial occlusions (e.g. cervical rib)
Pain and diminished pulses distal to fistula are seen
Vasculitis
Vessel diameter and vasculitis classification
Aorta and branches
Takayasu’s arteritis
Buergers disease
Giant cell arteritis
Large and medium sized arteries
Buergers disease
Giant cell arteritis
Polyarteritis nodosa
Medium sized muscular arteries
Polyarteritis nodosa
Wegeners granulomatosis
Small muscular arteries
Wegeners granulomatosis
Rheumatoid vasculitis
Specific conditions
Takyasu’s arteritis
Inflammatory, obliterative arteritis affecting aorta and branches
Females> Males
Symptoms may include upper limb claudication
Clinical findings include diminished or absent pulses
ESR often affected during the acute phase
Buergers disease
Segmental thrombotic occlusions of the small and medium sized lower limb vessels
Commonest in young male smokers
Proximal pulses usually present, but pedal pulses are lost
An acuter hypercellular occlusive thrombus is often present
Tortuous corkscrew shaped collateral vessels may be seen on angiography
Giant cell arteritis
Systemic granulomatous arteritis that usually affects large and medium sized vessels
Females > Males
Temporal arteritis is commonest type
Granulomatous lesions may be seen on biopsy (although up to 50% are normal)
Polyarteritis nodosa
Systemic necrotising vasculitis affecting small and medium sized muscular arteries
Most common in populations with high prevalence of hepatitis B
Renal disease is seen in 70% cases
Angiography may show saccular or fusiform aneurysms and arterial stenoses
Wegeners granulomatosis
Predominantly affects small and medium sized arteries
Systemic necrotising granulomatous vasculitis
Cutaneous vascular lesions may be seen (ulceration, nodules and purpura)
Sinus imaging may show mucosal thickening and air fluid levels
A 22 year old man is involved in a road traffic accident. He is found to have a pelvic fracture. While on the ward the nursing staff report that he is complaining of lower abdominal pain. On examination, you find a distended tender bladder and blood at the urethral meatus. What is the best management?
10 Ch foley urethral catheter
Suprapubic catheter
16 Ch foley urethral catheter
18 Ch coude tip urethral catheter
Pain relief and review in 1 hour
This patient has possible urethral injury based on the history. Urethral catheterisation is contraindicated in this situation.
Lower genitourinary tract trauma
Most bladder injuries occur due to blunt trauma
85% associated with pelvic fractures
Easily overlooked during assessment in trauma
Up to 10% of male pelvic fractures are associated with urethral or bladder injuries
Types of injury
Urethral injury
Mainly in males
Blood at the meatus (50% cases)
There are 2 types:
i.Bulbar rupture
- most common
- straddle type injury e.g. bicycles
- triad signs: urinary retention, perineal haematoma, blood at the meatus
ii. Membranous rupture
- can be extra or intraperitoneal
- commonly due to pelvic fracture
- Penile or perineal oedema/ hematoma
- PR: prostate displaced upwards (beware co-existing retroperitoneal haematomas as they may make examination difficult)
- Investigation: ascending urethrogram
- Management: suprapubic catheter (surgical placement, not percutaneously)
External genitalia injuries (i.e., the penis and the scrotum)
Secondary to injuries caused by penetration, blunt trauma, continence- or sexual pleasure-enhancing devices, and mutilation
Bladder injury
rupture is intra or extraperitoneal
presents with haematuria or suprapubic pain
history of pelvic fracture and inability to void: always suspect bladder or urethral injury
inability to retrieve all fluid used to irrigate the bladder through a Foley catheter indicates bladder injury
investigation- IVU or cystogram
management: laparotomy if intraperitoneal, conservative if extraperitoneal
A 50 year old female slips on wet floor injuring her ankle. On examination, she has tenderness over the lateral and medial malleolus. X-rays (stress views) demonstrate an undisplaced fracture of the distal fibula at the level of the syndesmosis and a congruent ankle mortise. What is the most appropriate management?
Application of full leg cast
Surgical fixation
Application of below knee walking boot
Application of external fixator
Bed rest, splinting and traction
Application of below knee walking boot
This is a Weber B fracture and therefore potentially unstable. Medial malleolar tenderness indicates deltoid ligament injury. As the fracture is currently undisplaced and the ankle mortise is congruent, the injury can be initially managed conservatively in a moon boot but the patient should be monitored in the outpatient clinic for fracture displacement in the first few weeks.
Ankle injuries
An ankle fracture relates to a fracture around the tibio-talar joint. It generally refers to a fracture involving the lateral, and/or medial and/or posterior malleolus. Pilon and Tillaux fractures are also considered to be ankle fractures, but are not covered here.
Ankle fractures are common. They effect men and women in equal numbers, but men have a higher rate as young adults (sports and contact injuries), and women a higher rate post-menopausal (fragility type fracture).
Osseous anatomy
The ankle (or mortise) joint consists of the distal tibia (tibial plafond and posterior malleolus), the distal fibula (lateral malleolus), and the talus. The main movement at the ankle joint is plantar and dorsiflexion.
Ligamentous anatomy
Medial side: Deltoid ligament. This is divided into superficial and deep portions. It is the primary restraint to valgus tilting of the talus.
Lateral side: Lateral ligament complex consisting from anterior to posterior of the anterior talofibular ligament (ATFL), calcaneofibular ligament (CFL), and the posterior talofibular ligament (PTFL). Together they resist valgus stress to the ankle, and are a restraint to anterior translation of the talus within the mortise joint.
Syndesmosis: The syndesmosis is a ligament complex between the distal tibia and fibula, holding the two bones together. It is fundamental to the integrity of the ankle joint, and its disruption leads to instability. It consists of (from anterior to posterior) the anterior-inferior tibiofibular ligament (AITFL), the transverse tibiofibular ligament (TTFL), the interosseous membrane, and the posterior-inferior tibiofibular ligament (PITFL).
Presentation and initial management
Patients will present following a traumatic event with a painful, swollen ankle, and reluctance/inability to weight bear. The Ottawa rules can be applied to differentiate between an ankle fracture and sprain, but can be unreliable.
In high energy injuries, management should follow ATLS principles to identify more significant injuries first. Neurovascular status of the foot should be documented, and open injuries should be excluded. If an open injury is identified, it should be managed in line with BOAST 4 principles1. If an obvious deformity exists, it should be reduced as soon as possible with appropriate analgesia or conscious sedation. Radiographs of clearly deformed or dislocated joints are not necessary, and removing the pressure on the surrounding soft tissues from the underlying bony deformity is the priority. If the fracture pattern is not clinically obvious then plain radiographs are appropriate and will guide the subsequent manipulation during plaster-of-paris below knee backslab application.
Imaging
AP, lateral and mortise views (20 degrees internal rotation) are essential to evaluate fracture displacement and syndesmotic injury. Decreased tibiofibular overlap, medial joint clear space and lateral talar shift all indicate a syndesmotic injury. (In subtle cases of shift, imaging the uninjured ankle can be helpful as a proportion of the population have little or no tibiotalar overlap-reference 2.)
Where there is suspicion of syndesmosis involvement in the absence of radiographic evidence, stress radiographs can be diagnostic.
Complex fracture patterns (and increasingly posterior malleolar fractures) are best defined using CT.
Classification
The most commonly used classifications are Lauge-Hansen and Danis-Weber.
Lauge-Hansen
Comprises two parts: first part is the foot position, and the second part is the force applied. Useful for understanding the forces involved and therefore predict the ligamentous or bony injury. Results in four injury patterns:
Supination - Adduction (SA) - 10-20%
Supination - External rotation (SER) - 40-75%
Pronation - Abduction (PA) - 5-20%
Pronation - External rotation (PER) - 5-20%
Not often used in clinical practice but good for understanding the principles of ankle fracture.
Danis-Weber
Commonly used. Based on the level of the fibula fracture in relation to the syndesmosis. The more proximal, the greater the risk of syndesmotic injury and therefore fracture instability.
A - fracture below the level of the syndesmosis
B - fracture at the level of the syndesmosis / level of the tibial plafond
C - fracture above the level of the syndesmosis. This includes Maisonneuve fractures (proximal fibula fracture), which can be associated with ankle instability. Beware the high fibula fracture - it may be an ankle fracture!
The Weber classification is based purely on the the lateral side. All injuries can include a medial or posterior bony or ligamentous injury which also dictates fracture stability (bimalleolar and trimalleolar fractures are more unstable).
Treatment
When deciding upon treatment for an ankle fracture, one must consider both the fracture and the patient. Diabetic patients and smokers are at greater risk of post-operative complication, especially wound problems and infection. Likewise, the long term outcome of post-traumatic arthritis from a malunited ankle fracture is extremely important for a young patient, but not as relevant in the elderly. Therefore, normal surgical decision processes apply as with all fractures.
Defining stability of an ankle fracture underpins the treatment decision.
Weber A - Unimalleolar Weber A Weber fractures by definition are stable and therefore can be mobilised fully weight bearing in an ankle boot.
Weber C - Fractures tend to include syndesmotic disruption and are usually bimalleolar (either bony or ligamentous). They are therefore unstable and usually require operative fixation. In addition to the fracture fixation, the syndesmosis usually requires reconstruction/augmentation with screws to restore the joint integrity and function.
Weber B - B fractures vary greatly. They can be part of a trimalleolar injury and therefore extremely unstable, requiring fixation. Alternatively, a uni-malleolar Weber B fracture can be a stable injury, and therefore mobilised immediately in an ankle boot. Defining the stability can be challenging, and often involves stress radiographs, or a trial of mobilisation and repeat radiographs. However, treating undisplaced ankle fractures in a below knee plaster, non-weight bearing for six weeks is still widely practised, and a safe approach.
When operative fixation is appropriate, it is usually via open reduction and internal fixation using plates and screws. It must be carried out when soft tissue swelling has settled in order to minimise the risk of wound problems. This can often take a week to settle.
The use of fibula nails is expanding, but is not yet mainstream. Ankle fractures can also be treated with external fixation, or with a hind foot nail in patients who need fixation but where soft tissue or bone quality is poor.
Post operative management
Ankle fractures generally take 6 weeks to unite enough to prevent secondary displacement. This is therefore an appropriate time period to keep a cast on in a conservatively managed patient. Weight bearing post-operatively depends on the quality of the fixation and bone quality, and preference varies between surgeons, ranging from aggressive early mobilisation to a period of non-weight bearing. Return to activities takes approximately three months, and often requires assistance of a physiotherapist to improve range-of-movement and muscle strengthening.
References
1. http://www.boa.ac.uk/publications/boa-standards-trauma-boasts/
2. Shah AS, Kadakia AR, Tan GJ, Karadsheh MS, Wolter TD, Sabb B. Radiographic evaluation of the normal distal tibiofibular syndesmosis. Foot Ankle Int. 2012;33(10):870-6
A 32 year old female hits her head on the steering wheel during a collision with another car. She has periorbital swelling and a flattened appearance of the face. What is the most likely injury?
Le Fort 1 fracture affecting maxilla
Le Fort 3 fracture affecting the maxilla
Mandibular fracture
Unilateral fracture of the zygoma
Isolated temporal bone injury
The flattened appearance of the face is a classical description of the dish/pan face associated with Le fort fracture 2 or 3 of the maxilla.
Craniomaxillofacial injuries
Craniomaxillofacial injuries in the UK are due to:
Interpersonal violence (52%)
Motor vehicle accidents (16%)
Sporting injuries (19%)
Falls (11%)
Le Fort Fractures
Grade Feature
Le Fort 1 The fracture extends from the nasal septum to the lateral pyriform rims, travels horizontally above the teeth apices, crosses below the zygomaticomaxillary junction, and traverses the pterygomaxillary junction to interrupt the pterygoid plates.
Le Fort 2 These fractures have a pyramidal shape and extend from the nasal bridge at or below the nasofrontal suture through the frontal process of the maxilla, inferolaterally through the lacrimal bones and inferior orbital floor and rim through or near the inferior orbital foramen, and inferiorly through the anterior wall of the maxillary sinus; it then travels under the zygoma, across the pterygomaxillary fissure, and through the pterygoid plates.
Le Fort 3 These fractures start at the nasofrontal and frontomaxillary sutures and extend posteriorly along the medial wall of the orbit through the nasolacrimal groove and ethmoid bones. The thicker sphenoid bone posteriorly usually prevents continuation of the fracture into the optic canal. Instead, the fracture continues along the floor of the orbit along the inferior orbital fissure and continues superolaterally through the lateral orbital wall, through the zygomaticofrontal junction and the zygomatic arch. Intranasally, a branch of the fracture extends through the base of the perpendicular plate of the ethmoid, through the vomer, and through the interface of the pterygoid plates to the base of the sphenoid. This type of fracture predisposes the patient to CSF rhinorrhea more commonly than the other types.
Ocular injuries
Superior orbital fissure syndrome
Severe force to the lateral wall of the orbit resulting in compression of neurovascular structures. Results in :
Complete opthalmoplegia and ptosis (Cranial nerves 3, 4, 6 and nerve to levator palpebrae superioris)
Relative afferent pupillary defect
Dilatation of the pupil and loss of accommodation and corneal reflexes
Altered sensation from forehead to vertex (frontal branch of trigeminal nerve)
Orbital blow out fracture
Typically occurs when an object of slightly larger diameter than the orbital rim strikes the incompressible eyeball. The bone fragment is displaced downwards into the antral cavity, remaining attached to the orbital periosteum. Periorbital fat may be herniated through the defect, interfering with the inferior rectus and inferior oblique muscles which are contained within the same fascial sheath. This prevents upward movement and outward rotation of the eye and the patient experiences diplopia on upward gaze. The initial bruising and swelling may make assessment difficult and patients should usually be reviewed 5 days later. Residual defects may require orbital floor reconstruction.
Nasal Fractures
Common injury
Ensure new and not old deformity
Control epistaxis
CSF rhinorrhoea implies that the cribriform plate has been breached and antibiotics will be required.
Usually best to allow bruising and swelling to settle and then review patient clinically. Major persistent deformity requires fracture manipulation, best performed within 10 days of injury.
Retrobulbar haemorrhage
Rare but important ocular emergency. Presents with:
Pain (usually sharp and within the globe)
Proptosis
Pupil reactions are lost
Paralysis (eye movements lost)
Visual acuity is lost (colour vision is lost first)
May be the result of Le Fort type facial fractures.
Management:
Mannitol 1g/Kg as 20% infusion, Osmotic diuretic, Contra-indicated in congestive heart failure and pulmonary oedema
Acetazolamide 500mg IV, (Monitor FBC/U+E) Reduces aqueous pressure by inhibition of carbonic anhydrase (used in glaucoma)
Dexamethasone 8mg orally or intravenously
In a traumatic setting an urgent cantholysis may be needed prior to definitive surgery.
Consider
Papaverine 40mg smooth muscle relaxant
Dextran 40 500mls IV improves perfusion
A 45 year old man with long standing ulcerative colitis and rectal dysplasia presents with a DALM lesion in the rectum. What is the most appropriate management option?
Snare polypectomy
Repeat endoscopy in 2 years
Discharge
Anterior resection
Panproctocolectomy
DALM lesions complicating ulcerative colitis should be managed with panproctocolectomy. An anterior resection is inadequate since it will only remove the rectum and ulcerative colitis affects the entire colon. Since many will be associated with invasion a snare polypectomy is not sufficient either.
Colonic lesions - DALM
The term DALM lesion refers to a Dysplasia Associated Lesion or Mass.
They may complicate dysplasia occurring in patients with longstanding ulcerative colitis.
They have a high incidence of invasive foci.
When they complicate longstanding ulcerative colitis, they should be treated by panproctocolectomy.
A 48 year old man is recovering on the high dependency unit following a long and complex laparotomy. His preoperative medication includes an ACE inhibitor for blood pressure control. For the past two hours he has been oliguric with a urine output of 10ml/hr-1. What the most appropriate immediate course of action?
Stop the ACE inhibitor
Administer a fluid challenge
Start an infusion of nor adrenaline
Administer intravenous frusemide
Insert a Swann-Ganz Catheter
Hypovolaemia is the most likely cause for oliguria and a fluid challenge is the most appropriate action. Blind administration of inotropes to hypovolaemic patients is unwise, with the possible exception of cardiac patients.
Hypovolaemia and the surgical patient
Hypovolaemia often represents the end point of multiple pathological processes. It may be divided into the following categories; overt compensated hypovolaemia, covert compensated hypovolaemia and decompensated hypovolaemia. Of these three categories the covert compensated subtype of hypovolaemia remains the commonest and is accounted for by the fact that class I shock will often produce no overtly discernible clinical signs. This is due, in most cases, to a degree of splanchnic autotransfusion. The most useful diagnostic test for detection of covert compensated hypovolaemia remains urinanalysis. This often shows increased urinary osmolality and decreased sodium concentration.
In overt compensated hypovolaemia the blood pressure is maintained although other haemodynamic parameters may be affected. This correlates to class II shock. In most cases assessment can be determined clinically. Where underlying cardiopulmonary disease may be present the placement of a CVP line may guide fluid resuscitation. Severe pulmonary disease may produce discrepancies between right and left atrial filling pressures. This problem was traditionally overcome through the use of Swann-Ganz catheters.
Untreated, hypovolaemia may ultimately become uncompensated with resultant end organ dysfunction. Microvascular hypoperfusion may result in acidosis with a subsequent myocardial depressive effect, thereby producing a vicious circle.
The treatment of hypovolaemia is with intravenous fluids. In the first instance a fluid challenge such as the rapid infusion of 250ml of crystalloid will often serve as both a diagnostic and resuscitative measure. In the event that this fails to produce the desired response the patient will need to be re-evaluated clinically. More fluid may be needed. However, it is important not to overlook mechanical ureteric obstruction in the anuric, normotensive patient.
A 56 year old man presents with lethargy, haematuria and haemoptysis. On examination he is hypertensive and has a right loin mass. A CT scan shows a lesion affecting the upper pole of the right kidney, it has a small cystic centre. Which of the options below is the most likely diagnosis?
Squamous cell carcinoma of the kidney
Nephroblastoma
Renal adenocarcinoma
Transitional cell carcinoma of the kidney
Polycystic kidney disease
Renal adenocarcinoma are the most common renal tumours. These will typically affect the renal parenchyma. Transitional cell carcinoma will usually affect urothelial surfaces. Nephroblastoma would be very rare in this age group. Renal adenocarcinoma may produce cannon ball metastasis in the lung which cause haemoptysis, this is not a feature of PKD.
Renal tumours
Renal cell carcinoma
Renal cell carcinoma is an adenocarcinoma of the renal cortex and is believed to arise from the proximal convoluted tubule. They are usually solid lesions, up to 20% may be multifocal, 20% may be calcified and 20% may have either a cystic component or be wholly cystic. They are often circumscribed by a pseudocapsule of compressed normal renal tissue. Spread may occur either by direct extension into the adrenal gland, renal vein or surrounding fascia. More distant disease usually occurs via the haematogenous route to lung, bone or brain.
Renal cell carcinoma comprise up to 85% of all renal malignancies. Males are more commonly affected than females and sporadic tumours typically affect patients in their sixth decade.
Patients may present with a variety of symptoms including; haematuria (50%), loin pain (40%), mass (30%) and up to 25% may have symptoms of metastasis.Less than 10% have the classic triad of haematuria, pain and mass.
Investigation
Many cases will present as haematuria and be discovered during diagnostic work up. Benign renal tumours are rare, so renal masses should be investigated with multislice CT scanning. Most tumours are also characterised with an arterial phase CT, particularly if they may be suitable for partial nephrectomy.
CT scanning of the chest and abdomen to detect distant disease should also be undertaken.
Routine bone scanning is not indicated in the absence of symptoms.
Biopsy should not be performed when a nephrectomy is planned but is mandatory before any ablative therapies are undertaken.
Assessment of the functioning of the contra lateral kidney.
Management
T1 lesions may be managed by partial nephrectomy and this gives equivalent oncological results to total radical nephrectomy. Partial nephrectomy may also be performed when there is inadequate reserve in the remaining kidney. Ablative techniques may also be considered for small T1 lesions in unfit patients. In general, outcomes are less favorable than with surgical resection.
For T2 lesions and above a radical nephrectomy is standard practice and this may be performed via a laparoscopic or open approach. Preoperative embolisation is not indicated nor is resection of uninvolved adrenal glands. During surgery early venous control is mandatory to avoid shedding of tumour cells into the circulation.
Patients with completely resected disease do not benefit from adjuvant therapy with either chemotherapy or biological agents. These should not be administered outside the setting of clinical trials.
Patients with transitional cell cancer will require a nephroureterectomy with disconnection of the ureter at the bladder.
References
Capitanio U, Montorsi F. Renal cancer. Lancet. 2016 Feb 27;387(10021):894-906.
Capitanio U et al. Epidemiology of Renal Cell Carcinoma. Eur Urol. 2019 Jan;75(1):74-84.
A 89 year old woman presents with long standing seborrhoeic warts of her abdominal wall , they have caused troublesome itching. What is the best treatment?
Administration of topical steroids
Shave excision and cautery
Excision and primary closure
Excision and skin graft
Excision biopsy
These lesions are often extensive and superficial. Shave excision will suffice, material must be sent for histology.
Skin Diseases
Skin lesions may be referred for surgical assessment, but more commonly will come via a dermatologist for definitive surgical management.
Skin malignancies include basal cell carcinoma, squamous cell carcinoma and malignant melanoma.
Basal Cell Carcinoma
Most common form of skin cancer.
Commonly occur on sun exposed sites apart from the ear.
Sub types include nodular, morphoeic, superficial and pigmented.
Typically slow growing with low metastatic potential.
Standard surgical excision, topical chemotherapy and radiotherapy are all successful.
As a minimum a diagnostic punch biopsy should be taken if treatment other than standard surgical excision is planned.
Squamous Cell Carcinoma
Again related to sun exposure.
May arise in pre - existing solar keratoses.
May metastasize if left.
Immunosupression (e.g. following transplant), increases risk.
Wide local excision is the treatment of choice and where a diagnostic excision biopsy has demonstrated SCC, repeat surgery to gain adequate margins may be required.
Malignant Melanoma
The main diagnostic features (major criteria):
Change in size
Change in shape
Change in colour
Secondary features (minor criteria)
Diameter >6mm
Inflammation
Oozing or bleeding
Altered sensation
Treatment
Suspicious lesions should undergo excision biopsy. The lesion should be removed completely as incision biopsy can make subsequent histopathological assessment difficult.
Once the diagnosis is confirmed the pathology report should be reviewed to determine whether further re-excision of margins is required (see below):
Margins of excision-Related to Breslow thickness
Lesions 0-1mm thick 1cm
Lesions 1-2mm thick 1- 2cm (Depending upon site and pathological features)
Lesions 2-4mm thick 2-3 cm (Depending upon site and pathological features)
Lesions >4 mm thick 3cm
Marsden J et al. Revised UK guidelines for management of Melanoma. Br J Dermatol 2010 163:238-256.
Further treatments such as sentinel lymph node mapping, isolated limb perfusion and block dissection of regional lymph node groups should be selectively applied.
Kaposi Sarcoma
Tumour of vascular and lymphatic endothelium.
Purple cutaneous nodules.
Associated with immuno supression.
Classical form affects elderly males and is slow growing.
Immunosupression form is much more aggressive and tends to affect those with HIV related disease.
Non malignant skin disease
Dermatitis Herpetiformis
Chronic itchy clusters of blisters.
Linked to underlying gluten enteropathy (coeliac disease).
Dermatofibroma
Benign lesion.
Firm elevated nodules.
Usually history of trauma.
Lesion consists of histiocytes, blood vessels and fibrotic changes.
Pyogenic granuloma
Overgrowth of blood vessels.
Red nodules.
Usually follow trauma.
May mimic amelanotic melanoma.
Acanthosis nigricans
Brown to black, poorly defined, velvety hyperpigmentation of the skin.
Usually found in body folds such as the posterior and lateral folds of the neck, the axilla, groin, umbilicus, forehead, and other areas.
The most common cause of acanthosis nigricans is insulin resistance, which leads to increased circulating insulin levels. Insulin spillover into the skin results in its abnormal increase in growth (hyperplasia of the skin).
In the context of a malignant disease, acanthosis nigricans is a paraneoplastic syndrome and is then commonly referred to as acanthosis nigricans maligna. Involvement of mucous membranes is rare and suggests a coexisting malignant condition.
A 78 year old man is referred to the clinic by his general practitioner. For many years he noticed a smooth swelling approximately 2cm anterior to the tragus of his right ear. Apart from being a heavy smoker he has no co-morbidities. What is the most likely diagnosis?
Pleomorphic adenoma
Liposarcoma
Warthins tumour
Adenocarcinoma
None of the above
Warthins tumours are most common in elderly smokers. They have a relatively benign and indolent course.
Parotid gland clinical
Benign neoplasms
Up to 80% of all salivary gland tumours occur in the parotid gland and up to 80% of these are benign. There is no consistent correlation between the rate of growth and the malignant potential of the lesion. However, benign tumours should not invade structures such as the facial nerve.
With the exception of Warthins tumours, they are commoner in women than men. The median age of developing a lesion is in the 5th decade of life.
Benign tumour types
Tumour type Features
Benign pleomorphic adenoma or benign mixed tumor Most common parotid neoplasm (80%)
Proliferation of epithelial and myoepithelial cells of the ducts and an increase in stromal components
Slow growing, lobular, and not well encapsulated
Recurrence rate of 1-5% with appropriate excision (parotidectomy)
Recurrence possibly secondary to capsular disruption during surgery
Malignant degeneration occurring in 2-10% of adenomas observed for long periods, with carcinoma ex-pleomorphic adenoma occurring most frequently as adenocarcinoma
Warthin tumor (papillary cystadenoma lymphoma or adenolymphoma) Second most common benign parotid tumor (5%)
Most common bilateral benign neoplasm of the parotid
Marked male as compared to female predominance
Occurs later in life (sixth and seventh decades)
Presents as a lymphocytic infiltrate and cystic epithelial proliferation
May represent heterotopic salivary gland epithelial tissue trapped within intraparotid lymph nodes
Incidence of bilaterality and multicentricity of 10%
Malignant transformation rare (almost unheard of)
Monomorphic adenoma Account for less than 5% of tumours
Slow growing
Consist of only one morphological cell type (hence term mono)
Include; basal cell adenoma, canalicular adenoma, oncocytoma, myoepitheliomas
Haemangioma Should be considered in the differential of a parotid mass in a child
Accounts for 90% of parotid tumours in children less than 1 year of age
Hypervascular on imaging
Spontaneous regression may occur and malignant transformation is almost unheard of
Malignant salivary gland tumours
Types of malignancy
Mucoepidermoid carcinoma 30% of all parotid malignancies
Usually low potential for local invasiveness and metastasis (depends mainly on grade)
Adenoid cystic carcinoma Unpredictable growth pattern
Tendency for perineural spread
Nerve growth may display skip lesions resulting in incomplete excision
Distant metastasis more common (visceral rather than nodal spread)
5 year survival 35%
Mixed tumours Often a malignancy occurring in a previously benign parotid lesion
Acinic cell carcinoma Intermediate grade malignancy
May show perineural invasion
Low potential for distant metastasis
5 year survival 80%
Adenocarcinoma Develops from secretory portion of gland
Risk of regional nodal and distant metastasis
5 year survival depends upon stage at presentation, may be up to 75% with small lesions with no nodal involvement
Lymphoma Large rubbery lesion, may occur in association with Warthins tumours
Diagnosis should be based on regional nodal biopsy rather than parotid resection
Treatment is with chemotherapy (and radiotherapy)
Diagnostic evaluation
Plain x-rays may be used to exclude calculi
Sialography may be used to delineate ductal anatomy
FNAC is used in most cases
Superficial parotidectomy may be either diagnostic or therapeutic depending upon the nature of the lesion
Where malignancy is suspected the primary approach should be definitive resection rather than excisional biopsy
CT/ MRI may be used in cases of malignancy for staging primary disease
Treatment
For nearly all lesions this consists of surgical resection, for benign disease this will usually consist of a superficial parotidectomy. For malignant disease a radical or extended radical parotidectomy is performed. The facial nerve is included in the resection if involved. The need for neck dissection is determined by the potential for nodal involvement.
Other parotid disorders
HIV infection
Lymphoepithelial cysts associated with HIV occur almost exclusively in the parotid
Typically presents as bilateral, multicystic, symmetrical swelling
Risk of malignant transformation is low and management usually conservative
Sjogren syndrome
Autoimmune disorder characterised by parotid enlargement, xerostomia and keratoconjunctivitis sicca
90% of cases occur in females
Second most common connective tissue disorder
Bilateral, non tender enlargement of the gland is usual
Histologically, the usual findings are of a lymphocytic infiltrate in acinar units and epimyoepithelial islands surrounded by lymphoid stroma
Treatment is supportive
There is an increased risk of subsequent lymphoma
Sarcoid
Parotid involvement occurs in 6% of patients with sarcoid
Bilateral in most cases
Gland is not tender
Xerostomia may occur
Management of isolated parotid disease is usually conservative
Which of the following associations are incorrect?
Afro-Caribbean ethnicity and keloid scarring
Extensive third degree burns and wound contraction
Chemotherapy and dehiscence of healed wounds
Poor healing at the site of previous radiotherapy
Zinc deficiency and delayed healing
The question asks for the incorrect association. Certain ethnic groups are at increased risk of keloid scar formation. Severe burns can be complicated by wound contracture that requires surgery. Radiotherapy and zinc deficiency are both recognized risk factors for delayed healing. However, fully healed wounds cannot breakdown with chemotherapy and this statement is therefore incorrect and represents the correct answer.
Wound healing
Surgical wounds are either incisional or excisional and either clean, clean contaminated or dirty. Although the stages of wound healing are broadly similar their contributions will vary according to the wound type.
The main stages of wound healing include:
Haemostasis
Minutes to hours following injury
Vasospasm in adjacent vessels, platelet plug formation and generation of fibrin rich clot.
Inflammation
Typically days 1-5
Neutrophils migrate into wound (function impaired in diabetes).
Growth factors released, including basic fibroblast growth factor and vascular endothelial growth factor.
Fibroblasts replicate within the adjacent matrix and migrate into wound.
Macrophages and fibroblasts couple matrix regeneration and clot substitution.
Regeneration
Typically days 7 to 56
Platelet derived growth factor and transformation growth factors stimulate fibroblasts and epithelial cells.
Fibroblasts produce a collagen network.
Angiogenesis occurs and wound resembles granulation tissue.
Remodeling
From 6 weeks to 1 year
Longest phase of the healing process and may last up to one year (or longer).
During this phase fibroblasts become differentiated (myofibroblasts) and these facilitate wound contraction.
Collagen fibres are remodeled.
Microvessels regress leaving a pale scar.
The above description represents an idealised scenario. A number of diseases may distort this process. Neovascularisation is an important early process. Endothelial cells may proliferate in the wound bed and recanalise to form a vessel. Vascular disease, shock and sepsis can all compromise microvascular flow and impair healing.
Conditions such as jaundice will impair fibroblast synthetic function and immunity with a detrimental effect in most parts of the healing process.
Problems with scars:
Hypertrophic scars
Excessive amounts of collagen within a scar. Nodules may be present histologically containing randomly arranged fibrils within and parallel fibres on the surface. The tissue itself is confined to the extent of the wound itself and is usually the result of a full thickness dermal injury. They may go on to develop contractures.
Keloid scars
Excessive amounts of collagen within a scar. Typically a keloid scar will pass beyond the boundaries of the original injury. They do not contain nodules and may occur following even trivial injury. They do not regress over time and may recur following removal.
Drugs which impair wound healing:
Non steroidal anti inflammatory drugs
Steroids
Immunosupressive agents
Anti neoplastic drugs
Closure
Delayed primary closure is the anatomically precise closure that is delayed for a few days but before granulation tissue becomes macroscopically evident.
Secondary closure refers to either spontaneous closure or to surgical closure after granulation tissue has formed.
A 56 year old man is undergoing a carotid endarterectomy. The internal carotid artery is mobilised. How many branches does this vessel give off in the neck?
0
1
2
3
6
The internal carotid does not have any branches in the neck.
Internal carotid artery
The internal carotid artery is formed from the common carotid opposite the upper border of the thyroid cartilage. It extends superiorly to enter the skull via the carotid canal. From the carotid canal it then passes through the cavernous sinus, above which it divides into the anterior and middle cerebral arteries.
Relations in the neck
Posterior
Longus capitis
Pre-vertebral fascia
Sympathetic chain
Superior laryngeal nerve
Medially
External carotid (near origin)
Wall of pharynx
Ascending pharyngeal artery
Laterally
Internal jugular vein (moves posteriorly at entrance to skull)
Vagus nerve (most posterolaterally)
Anteriorly
Sternocleidomastoid
Lingual and facial veins
Hypoglossal nerve
Relations in the carotid canal
Internal carotid plexus
Cochlea and middle ear cavity
Trigeminal ganglion (superiorly)
Leaves canal lies above the foramen lacerum
Path and relations in the cranial cavity
The artery bends sharply forwards in the cavernous sinus, the aducens nerve lies close to its inferolateral aspect. The oculomotor, trochlear, opthalmic and, usually, the maxillary nerves lie in the lateral wall of the sinus. Near the superior orbital fissure it turns posteriorly and passes postero-medially to pierce the roof of the cavernous sinus inferior to the optic nerve. It then passes between the optic and oculomotor nerves to terminate below the anterior perforated substance by dividing into the anterior and middle cerebral arteries.
Branches
Anterior and middle cerebral artery
Ophthalmic artery
Posterior communicating artery
Anterior choroid artery
Meningeal arteries
Hypophyseal arteries
The following muscles are supplied by the recurrent laryngeal nerve except:
Transverse arytenoid
Posterior crico-arytenoid
Cricothyroid
Oblique arytenoid
Thyroarytenoid
Innervates: all intrinsic larynx muscles (excluding cricothyroid)
The external branch of the superior laryngeal nerve innervates the cricothyroid muscle.
Recurrent laryngeal nerve
Branch of the vagus nerve
Path
Right
Arises anterior to the subclavian artery and ascends obliquely next to the trachea, behind the common carotid artery
It is either anterior or posterior to the inferior thyroid artery
Left
Arises left to the arch of the aorta
Winds below the aorta
Ascends along the side of the trachea
Then both
Pass in a groove between the trachea and oesophagus
Enters the larynx behind the articulation between the thyroid cartilage and cricoid
Distributed to larynx muscles
Branches to
Cardiac plexus
Mucous membrane and muscular coat of the oesophagus and trachea
Innervates
Intrinsic larynx muscles (excluding cricothyroid)
A male infant, born at term appears well following delivery. Six hours later, he is noted to have bilious vomiting by the paediatricians. On examination, he seems well and his abdomen is soft and non tender. What is the best course of action?
Arrange an abdominal x-ray
Undertake a test feed
Perform serial abdominal examinations
Arrange an upper GI contrast study
Arrange a laparotomy
Bilious vomiting in neonates is a surgical emergency and is intestinal malrotation and volvulus until otherwise proven. It is investigated with an upper GI contrast study. Contrast should be seen to exit the stomach and the location of the DJ flexure is noted (it lies to the left of the midline). If this is not the case, or the study is inconclusive, a laparotomy is performed.
Intestinal malrotation in neonates
Normal embryology
During the fourth week of embryogenesis the intestine moves into the abdomen via the base of the umbilicus. Through a combination of foetal growth and bowel maturation the bowel resides in the abdominal cavity and undergoes a 270o counterclockwise twist. At the conclusion of this process the ligament of Treitz lies to the left of the spine and the caecum in the right lower quadrant. Malrotation occurs when the rotational process described is incomplete. Typically the duodenal loop lies to the left of the caecum and therefore lacks 90 o of its 270o rotation. It becomes fixed in this position with peritoneal attachments (Ladds bands).
Symptoms
Bilious vomiting is the cardinal symptom and sign. In most cases there are no antecedent symptoms. If untreated then gut perfusion may be impaired with resultant development of further symptoms.
Diagnosis
The main problem is that the infant develops mid gut volvulus. In this situation the bowel undergoes a 720o twist, the bowel viability depends upon a narrow mesentery containing the superior mesenteric artery. To ascertain whether this has occurred the two main tests include an abdominal ultrasound scan to determine the relationship between the superior mesenteric artery and vein (normally SMA lies to the left of the SMV). This test is complemented with an upper GI contrast series and this aims to establish that the DJ flexure is correctly sited to the left of the vertebral bodies.
Treatment
Laparotomy and division of adhesional bands (Ladds procedure). The division of the congenital adhesions allows widening of the small bowel mesentery. The bowel is untwisted and assessed for viability. The bowel is returned to the abdominal cavity in the non rotated fashion with the small bowel on the right hand side and the large bowel on the left, the caecum is positioned in the left upper quadrant. Because the caecum is located in this new location many surgeons will also perform an appendicectomy due to the diagnostic difficulty posed in diagnosing appendicitis in this group of patients.
Which of the following is not typically associated with a degloving injury?
Overlying pallor of the skin
Abnormal motility of the overlying skin
History of friction type injury
Improved results when the degloved segment is left in situ as a temporary closure
Poor results when primary compression treatment is used in preference to skin grafting
Improved results when the degloved segment is left in situ as a temporary closure
Degloving injuries typically involve extremities and are usually friction injuries e.g. arm being run over. There is abnormal motility of the overlying skin, pallor, loss of sensation. Early treatment is key and should involve skin grafting which may use the degloved segment. This however, should be formally prepared for the role and simple compression bandaging gives poor results.
Wound healing
Surgical wounds are either incisional or excisional and either clean, clean contaminated or dirty. Although the stages of wound healing are broadly similar their contributions will vary according to the wound type.
The main stages of wound healing include:
Haemostasis
Minutes to hours following injury
Vasospasm in adjacent vessels, platelet plug formation and generation of fibrin rich clot.
Inflammation
Typically days 1-5
Neutrophils migrate into wound (function impaired in diabetes).
Growth factors released, including basic fibroblast growth factor and vascular endothelial growth factor.
Fibroblasts replicate within the adjacent matrix and migrate into wound.
Macrophages and fibroblasts couple matrix regeneration and clot substitution.
Regeneration
Typically days 7 to 56
Platelet derived growth factor and transformation growth factors stimulate fibroblasts and epithelial cells.
Fibroblasts produce a collagen network.
Angiogenesis occurs and wound resembles granulation tissue.
Remodeling
From 6 weeks to 1 year
Longest phase of the healing process and may last up to one year (or longer).
During this phase fibroblasts become differentiated (myofibroblasts) and these facilitate wound contraction.
Collagen fibres are remodeled.
Microvessels regress leaving a pale scar.
The above description represents an idealised scenario. A number of diseases may distort this process. Neovascularisation is an important early process. Endothelial cells may proliferate in the wound bed and recanalise to form a vessel. Vascular disease, shock and sepsis can all compromise microvascular flow and impair healing.
Conditions such as jaundice will impair fibroblast synthetic function and immunity with a detrimental effect in most parts of the healing process.
Problems with scars:
Hypertrophic scars
Excessive amounts of collagen within a scar. Nodules may be present histologically containing randomly arranged fibrils within and parallel fibres on the surface. The tissue itself is confined to the extent of the wound itself and is usually the result of a full thickness dermal injury. They may go on to develop contractures.
Intra cranial pressure is governed by the principles of the Monroe-Kellie doctrine. To which of the following does this concept not apply?
A 2 month old child
A 2 year old child
A 5 year old child
A 10 year old child
An adult
The Monroe-Kelly Doctrine assumes that the cranial cavity is a rigid box. In children with non fused fontanells this is not the case.
Applied neurophysiology
Pressure within the cranium is governed by the Monroe-Kelly doctrine. This considers the skull as a closed box. Increases in mass can be accommodated by loss of CSF. Once a critical point is reached (usually 100- 120ml of CSF lost) there can be no further compensation and ICP rises sharply. The next step is that pressure will begin to equate with MAP and neuronal death will occur. Herniation will also accompany this process.
The CNS can autoregulate its own blood supply. Vaso constriction and dilatation of the cerebral blood vessels is the primary method by which this occurs. Extremes of blood pressure can exceed this capacity resulting in risk of stroke. Other metabolic factors such as hypercapnia will also cause vasodilation, which is of importance in ventilating head injured patients.
The brain can only metabolise glucose, when glucose levels fall, consciousness will be impaired.
A 2 year old boy presents with vague neurological symptoms and is imaged with an MRI of the brain. This demonstrates a tumour that is located in the floor of the 4th ventricle and extends through the foramen of Magendie. What is the most likely diagnosis?
Glioblastoma
CNS lymphoma
Ependymoma
Schwannoma
Meningioma
Glioblastoma is rare in children. Ependymoma account for up to 33% of CNS tumours in those under age of 3. They commonly arise in the 4th ventricle and can grow through the foramina of Luschka and Magendie
CNS tumours
60% = Glioma and metastatic disease
20% = Meningioma
10% = Pituitary lesions
In paediatric practice medulloblastomas (neuroectodermal tumours) were the commonest lesions, astrocytomas now account for the majority.
Tumours arising in right temporal and frontal lobe may reach considerable size before becoming symptomatic. Whereas tumours in the speech and visual areas will typically produce early symptoms.
Most paediatric CNS tumours are infratentorial
Most adult CNS tumours are supratentorial
Diagnosis
MRI Scanning provides the best resolution.
Treatment
Usually surgery, even if tumour cannot be completely resected conditions such as rising ICP can be addressed with tumour debulking and survival and quality of life prolonged.
Curative surgery can usually be undertaken with lesions such as meningiomas. Gliomas have a marked propensity to invade normal brain and resection of these lesions is nearly always incomplete.
A 43 year old man is stabbed outside a nightclub. He suffers a transection of his median nerve just as it leaves the brachial plexus. Which of these features is least likely to ensue?
Ulnar deviation of the wrist
Complete loss of wrist flexion
Loss of pronation
Loss of flexion at the thumb joint
Inability to oppose the thumb
Loss of the median nerve will result in loss of function of the flexor muscles. However, flexor carpi ulnaris will still function and produce ulnar deviation and some residual wrist flexion. High median nerve lesions result in complete loss of flexion at the thumb joint.
Median nerve
The median nerve is formed by the union of a lateral and medial root respectively from the lateral (C5,6,7) and medial (C8 and T1) cords of the brachial plexus; the medial root passes anterior to the third part of the axillary artery. The nerve descends lateral to the brachial artery, crosses to its medial side (usually passing anterior to the artery). It passes deep to the bicipital aponeurosis and the median cubital vein at the elbow.
It passes between the two heads of the pronator teres muscle, and runs on the deep surface of flexor digitorum superficialis (within its fascial sheath).
Near the wrist it becomes superficial between the tendons of flexor digitorum superficialis and flexor carpi radialis, deep to palmaris longus tendon. It passes deep to the flexor retinaculum to enter the palm, but lies anterior to the long flexor tendons within the carpal tunnel.
Branches
Region Branch
Upper arm No branches, although the nerve commonly communicates with the musculocutaneous nerve
Forearm Pronator teres
Pronator quadratus
Flexor carpi radialis
Palmaris longus
Flexor digitorum superficialis
Flexor pollicis longus
Flexor digitorum profundus (only the radial half)
Distal forearm Palmar cutaneous branch
Hand (Motor) Motor supply (LOAF)
Lateral 2 lumbricals
Opponens pollicis
Abductor pollicis brevis
Flexor pollicis brevis
Hand (Sensory)
Over thumb and lateral 2 ½ fingers
On the palmar aspect this projects proximally, on the dorsal aspect only the distal regions are innervated with the radial nerve providing the more proximal cutaneous innervation.
Patterns of damage
Damage at wrist
e.g. carpal tunnel syndrome
paralysis and wasting of thenar eminence muscles and opponens pollicis (ape hand deformity)
sensory loss to palmar aspect of lateral (radial) 2 ½ fingers
Damage at elbow, as above plus:
unable to pronate forearm
weak wrist flexion
ulnar deviation of wrist
Anterior interosseous nerve (branch of median nerve)
leaves just below the elbow
results in loss of pronation of forearm and weakness of long flexors of thumb and index finger
A 67 year old man is undergoing an angiogram for gastro intestinal bleeding. The radiologist advances the catheter into the coeliac axis. At what spinal level does this vessel typically arise from the aorta?
T10
L3
L4
T12
None of the above
The coeliac axis lies at T12, it takes an almost horizontal angle off the aorta. It has three major branches.
Abdominal aortic branches
Branches Level Paired Type
Inferior phrenic T12 (Upper border) Yes Parietal
Coeliac T12 No Visceral
Superior mesenteric L1 No Visceral
Middle suprarenal L1 Yes Visceral
Renal L1-L2 Yes Visceral
Gonadal L2 Yes Visceral
Lumbar L1-L4 Yes Parietal
Inferior mesenteric L3 No Visceral
Median sacral L4 No Parietal
Common iliac L4 Yes Terminal
Which structure separates the cephalic vein and the brachial artery in the antecubital fossa?
Brachioradialis muscle
Bicipital aponeurosis
Origin of flexor digitorum profundus muscle
Pronator quadratus muscle
Origin of flexor digitorum superficialis muscle
The correct answer is bicipital aponeurosis. The bicipital aponeurosis is a thin sheet of connective tissue that extends from the biceps brachii muscle and covers the brachial artery and median nerve in the cubital fossa1. It separates these structures from the median cubital vein, which is a superficial vein that connects the cephalic vein (located laterally) and the basilic vein (located medially)2. The other options are muscles that are either located outside the cubital fossa or form the floor of the fossa, but do not separate the cephalic vein and the brachial artery.
A 62 year old man presents with dysphagia and on investigation is found to have a stenosing tumour of the mid oesophagus with a single metastasis in the right lobe of the liver (segment VI). What is the most appropriate treatment?
Radical radiotherapy to the oesophagus and liver resection
Insertion of self expanding metallic stent
Liver resection and subsequent oesophageal resection
Oesophageal resection and subsequent liver resection
Combined oesophageal and liver resection if cardiopulmonary exercise testing shows the patient is fit enough
Distant disease in patients with oesophageal cancer is a contra indication to a resectional strategy and downstaging with chemotherapy is not routinely undertaken in this age group as the results are poor. An expanding stent will provide rapid and durable palliation.
Oesophageal cancer - treatment
Treatments for SCC’s and adenocarcinomas of the oesophagus differ. This is primarily due to the positive outcomes that are observed when localised SCC’s (particularly of the proximal oesophagus) are treated with radical chemoradiotherapy (obviating the need for surgery).
Only those patients whose staging investigations are negative for metastatic disease should be considered for surgery.
Surgical options
Endoscopic mucosal resection Treatment for early localised adenocarcinoma of the distal oesophagus. Survival mirrors that of surgical resection for Tis and T1 disease
Transhiatal oeosphagectomy Most commonly used for junctional (type II) (1) tumours where limited thoracic oesophageal resection is required. Less morbidity than two field oesophagectomy
Ivor Lewis oesophagectomy Two stage approach for middle and distal tumours. Very commonly performed, intrathoracic anastomosis will result in mediastinitis in event of anastomotic leak. Lower incidence of recurrent laryngeal nerve injury
McKeown oesophagectomy Three field approach, may be useful for proximal tumours. Anastomotic leakage is less serious. Higher incidence of recurrent laryngeal nerve injury
Neoadjuvent and adjuvent treatment
Neoadjuvent radiotherapy alone prior to resection confers little benefit and is not routinely performed (2)
Preoperative chemotherapy is associated with a survival advantage (OE02 trial)
Peri operative (pre and post operative) chemotherapy confers a survival advantage in junctional tumours
Post operative chemotherapy is not generally recommended following oesophageal resections outside clinical trials
Palliation strategies
Combination chemotherapy improves quality of life and survival in non operable disease (3)
Trastuzumab may improve survival in patients with HER 2 positive tumours
Oesophageal intubation with self expanding metal stents is the treatment of choice in patients with occluding tumours >2cm from the cricopharyngeus
Covered metal stents are useful in cases of malignant fistulas
Laser therapy and argon plasma coagulation may be useful as therapies for tumour overgrowth and bleeding
Photodynamic therapy and ethanol injections confer little benefit and should not be routinely used
References
1. Hulscher JB, van Sandick JW, de Boer AG, et al. Extended transthoracic resection compared with limited transhiatal resection for adenocarcinoma of the esophagus. N Engl J Med 2002;347:1662-9
2. Deng HY, Wang WP, Wang YC, Hu WP, Ni PZ, Lin YD, Chen LQ. Neoadjuvant chemoradiotherapy or chemotherapy? A comprehensive systematic review and meta-analysis of the options for neoadjuvant therapy for treating oesophageal cancer. Eur J Cardiothorac Surg. 2017 Mar 1;51(3):421-431. doi: 10.1093/ejcts/ezw315.
3.Khin MO, Bromham N, Harrison M, Eadon H. Assessment and management of oesophago-gastric cancer: summary of NICE guidance. BMJ. 2018 Jan 29;360:k213. doi: 10.1136/bmj.k213.
A 30 year old male presents with a painless swelling of the testis. Histologically the stroma has a lymphocytic infiltrate. The most likely diagnosis is :
Differentiated teratoma
Malignant undifferentiated teratoma
Classical seminoma
Spermatocytic seminoma
Anaplastic seminoma
Seminoma is the commonest type of testicular tumour and is more common in males aged between 30-40 years. Classical seminoma is the commonest subtype and histology shows lymphocytic stromal infiltrate. Other subtypes include:
1. Spermatocytic: tumour cells resemble spermatocytes. Excellent prognosis.
2. Anaplastic
3. Syncytiotrophoblast giant cells: β HCG present in cells
A teratoma is more common in males aged 20-30 years.
Testicular disorders
Testicular cancer
Testicular cancer is the most common malignancy in men aged 20-30 years. Around 95% of cases of testicular cancer are germ-cell tumours. Germ cell tumours may essentially be divided into:
Tumour type Key features Tumour markers Pathology
Seminoma
Commonest subtype (50%)
Average age at diagnosis = 40
Even advanced disease associated with 5 year survival of 73%
AFP usually normal
HCG elevated in 10% seminomas
Lactate dehydrogenase; elevated in 10-20% seminomas (but also in many other conditions)
Sheet like lobular patterns of cells with substantial fibrous component. Fibrous septa contain lymphocytic inclusions and granulomas may be seen.
Non seminomatous germ cell tumours (42%)
Teratoma
Yolk sac tumour
Choriocarcinoma
Mixed germ cell tumours (10%)
Younger age at presentation =20-30 years
Advanced disease carries worse prognosis (48% at 5 years)
Retroperitoneal lymph node dissection may be needed for residual disease after chemotherapy
AFP elevated in up to 70% of cases
HCG elevated in up to 40% of cases
Other markers rarely helpful
Risk factors for testicular cancer
Cryptorchidism
Infertility
Family history
Klinefelter’s syndrome
Mumps orchitis
Features
A painless lump is the most common presenting symptom
Pain may also be present in a minority of men
Other possible features include hydrocele, gynaecomastia
Diagnosis
Ultrasound is first-line
CT scanning of the chest/ abdomen and pelvis is used for staging
Tumour markers (see above) should be measured
Management
Orchidectomy (Inguinal approach)
Chemotherapy and radiotherapy may be given depending on staging
Abdominal lesions >1cm following chemotherapy may require retroperitoneal lymph node dissection.
Prognosis is generally excellent
5 year survival for seminomas is around 95% if Stage I
5 year survival for teratomas is around 85% if Stage I
Benign disease
Epididymo-orchitis
Acute epididymitis is an acute inflammation of the epididymis, often involving the testis and usually caused by bacterial infection.
Infection spreads from the urethra or bladder. In men <35 years, gonorrhoea or chlamydia are the usual infections.
Amiodarone is a recognised non infective cause of epididymitis, which resolves on stopping the drug.
Tenderness is usually confined to the epididymis, which may facilitate differentiating it from torsion where pain usually affects the entire testis.
Testicular torsion
Twist of the spermatic cord resulting in testicular ischaemia and necrosis.
Most common in males aged between 10 and 30 (peak incidence 13-15 years)
Pain is usually severe and of sudden onset.
Cremasteric reflex is lost and elevation of the testis does not ease the pain.
Treatment is with surgical exploration. If a torted testis is identified then both testis should be fixed as the condition of bell clapper testis is often bilateral.
Hydrocele
Presents as a mass that transilluminates, usually possible to ‘get above’ it on examination.
In younger men it should be investigated with USS to exclude tumour.
In children it may occur as a result of a patent processus vaginalis.
Treatment in adults is with a Lords or Jabouley procedure.
Treatment in children is with trans inguinal ligation of PPV.
A 30 year old lady is found to have a thyroglossal cyst that has been recently infected and the patient requests treatment. What is the most appropriate course of action?
Surgical treatment with resection of cyst, associated track, central portion of the hyoid and wedge of tongue muscle behind the hyoid
Excision of the cyst alone with ligation of the track
Laying open of the track and excision of the cyst
Phenolisation of the cyst
Marsupialisation of the cyst
Recurrence following attempted resection of thyroglossal cysts is very common. Complete excision of the cyst and its track and origin is mandatory (Sistrunks procedure).
Thyroglossal cysts
The commonest midline neck mass in children is a congenital cyst of the thyroglossal duct. Embryologically, the cyst can arise at any point along the route of the thyroglossal duct, extending from the foramen caecum to the thyroid gland.
They are typically located below the hyoid and moves both on swallowing and tongue protrusion. Whilst most cysts are asymptomatic, associated infection may result in pain and swelling.
The usual treatment is excision of the cyst and track, this is a called a Sistrunk procedure.
Which of the following is not a feature of oesphageal atresia in neonates?
High incidence of polyhydramnios
Risk of recurrence in subsequent pregnancies of 80%
Distal tracheoesphageal fistula is the commonest variant
High incidence of associated imperforate anus
Absence of gastric fluid on antenatal ultrasound
Most are sporadic and risk in subsequent pregnancies is not increased.
Paediatric Gastrointestinal disorders
Pyloric stenosis
M>F
5-10% Family history in parents
Projectile non bile stained vomiting at 4-6 weeks of life
Diagnosis is made by test feed or USS
Treatment: Ramstedt pyloromyotomy (open or laparoscopic)
Acute appendicitis
Uncommon under 3 years
When occurs may present atypically
Mesenteric adenitis
Central abdominal pain and URTI
Conservative management
Intussusception
Telescoping bowel
Proximal to or at the level of, ileocaecal valve
6-9 months age
Colicky pain, diarrhoea and vomiting, sausage shaped mass, red jelly stool.
Treatment: reduction with air insufflation
Malrotation
High caecum at the midline
Feature in exomphalos, congenital diaphragmatic hernia, intrinsic duodenal atresia
May be complicated by development of volvulus, infant with volvulus may have bile stained vomiting
Diagnosis is made by upper GI contrast study and USS
Treatment is by laparotomy, if volvulus is present (or at high risk of occurring then a ladds procedure is performed
Hirschsprung’s disease
Absence of ganglion cells from myenteric and submucosal plexuses
Occurs in 1/5000 births
Full thickness rectal biopsy for diagnosis
Delayed passage of meconium and abdominal distension
Treatment is with rectal washouts initially, thereafter an anorectal pull through procedure
Oesophageal atresia
Associated with tracheo-oesophageal fistula and polyhydramnios
May present with choking and cyanotic spells following aspiration
VACTERL associations
Meconium ileus
Usually delayed passage of meconium and abdominal distension
Majority have cystic fibrosis
X-Rays may not show a fluid level as the meconium is viscid (depends upon feeding), PR contrast studies may dislodge meconium plugs and be therapeutic
Infants who do not respond to PR contrast and NG N-acetyl cysteine will require surgery to remove the plugs
Biliary atresia
Jaundice > 14 days
Increased conjugated bilirubin
Urgent Kasai procedure
Necrotising enterocolitis
Prematurity is the main risk factor
Early features include abdominal distension and passage of bloody stools
X-Rays may show pneumatosis intestinalis and evidence of free air
Increased risk when empirical antibiotics are given to infants beyond 5 days
Treatment is with total gut rest and TPN, babies with perforations will require laparotomy
Which of the following is not a feature of oesphageal atresia in neonates?
High incidence of polyhydramnios
Risk of recurrence in subsequent pregnancies of 80%
Distal tracheoesphageal fistula is the commonest variant
High incidence of associated imperforate anus
Absence of gastric fluid on antenatal ultrasound
Most are sporadic and risk in subsequent pregnancies is not increased.
Paediatric Gastrointestinal disorders
Pyloric stenosis
M>F
5-10% Family history in parents
Projectile non bile stained vomiting at 4-6 weeks of life
Diagnosis is made by test feed or USS
Treatment: Ramstedt pyloromyotomy (open or laparoscopic)
Acute appendicitis
Uncommon under 3 years
When occurs may present atypically
Mesenteric adenitis
Central abdominal pain and URTI
Conservative management
Intussusception
Telescoping bowel
Proximal to or at the level of, ileocaecal valve
6-9 months age
Colicky pain, diarrhoea and vomiting, sausage shaped mass, red jelly stool.
Treatment: reduction with air insufflation
Malrotation
High caecum at the midline
Feature in exomphalos, congenital diaphragmatic hernia, intrinsic duodenal atresia
May be complicated by development of volvulus, infant with volvulus may have bile stained vomiting
Diagnosis is made by upper GI contrast study and USS
Treatment is by laparotomy, if volvulus is present (or at high risk of occurring then a ladds procedure is performed
Hirschsprung’s disease
Absence of ganglion cells from myenteric and submucosal plexuses
Occurs in 1/5000 births
Full thickness rectal biopsy for diagnosis
Delayed passage of meconium and abdominal distension
Treatment is with rectal washouts initially, thereafter an anorectal pull through procedure
Oesophageal atresia
Associated with tracheo-oesophageal fistula and polyhydramnios
May present with choking and cyanotic spells following aspiration
VACTERL associations
Meconium ileus
Usually delayed passage of meconium and abdominal distension
Majority have cystic fibrosis
X-Rays may not show a fluid level as the meconium is viscid (depends upon feeding), PR contrast studies may dislodge meconium plugs and be therapeutic
Infants who do not respond to PR contrast and NG N-acetyl cysteine will require surgery to remove the plugs
Biliary atresia
Jaundice > 14 days
Increased conjugated bilirubin
Urgent Kasai procedure
Necrotising enterocolitis
Prematurity is the main risk factor
Early features include abdominal distension and passage of bloody stools
X-Rays may show pneumatosis intestinalis and evidence of free air
Increased risk when empirical antibiotics are given to infants beyond 5 days
Treatment is with total gut rest and TPN, babies with perforations will require laparotomy
Surgical treatment with resection of cyst, associated track, central portion of the hyoid and wedge of tongue muscle behind the hyoid
Excision of the cyst alone with ligation of the track
Laying open of the track and excision of the cyst
Phenolisation of the cyst
Marsupialisation of the cyst
Recurrence following attempted resection of thyroglossal cysts is very common. Complete excision of the cyst and its track and origin is mandatory (Sistrunks procedure).
Thyroglossal cysts
The commonest midline neck mass in children is a congenital cyst of the thyroglossal duct. Embryologically, the cyst can arise at any point along the route of the thyroglossal duct, extending from the foramen caecum to the thyroid gland.
They are typically located below the hyoid and moves both on swallowing and tongue protrusion. Whilst most cysts are asymptomatic, associated infection may result in pain and swelling.
The usual treatment is excision of the cyst and track, this is a called a Sistrunk procedure.
A 28 year old man is reviewed in the clinic. He has suffered from Crohns disease for many years, he has recently undergone a sub total colectomy. However, he has residual Crohns in his rectum and this is the cause of ongoing symptoms. Medical therapy is proving ineffective. What is the best course of action?
Abdomino perineal excision of the colon and rectum
Proctectomy
Hartmanns procedure
Ileo-rectal anastomosis
Formation of ileo-anal pouch
An abdomino-perineal excision of the colon and rectum is a cancer procedure and not appropriate in the context of inflammatory bowel disease. The only appropriate surgical option here is a proctectomy to remove the rectal stump and anal canal.
IBD
Ulcerative colitis Vs Crohns
Crohn’s disease Ulcerative colitis
Distribution Mouth to anus Rectum and colon
Macroscopic changes Cobblestone appearance, apthoid ulceration Contact bleeding
Depth of disease Transmural inflammation Superficial inflammation
Distribution pattern Patchy Continuous
Histological features Granulomas (non caseating epithelioid cell aggregates with Langerhans’ giant cells) Crypt abscesses, Inflammatory cells in the lamina propria
Surgical treatment
Ulcerative colitis
In UC the main place for surgery is when medical treatment has failed, in the emergency setting this will be a sub total colectomy, end ileostomy and a mucous fistula. Electively it will be a pan proctocolectomy, an ileoanal pouch may be a selected option for some. Remember that longstanding UC increases colorectal cancer risk.
Crohn’s disease
Unlike UC Crohn’s patients need to avoid surgeons, minimal resections are the rule. They should not have ileoanal pouches as they will do poorly with them. Management of Crohn’s ano rectal sepsis is with a minimal approach, simply drain sepsis and use setons to facilitate drainage. Definitive fistula surgery should be avoided.
A 74 year old man is admitted for an inguinal hernia repair. His past medical history includes transient ischaemic attacks for which he takes clopidogrel. This is stopped 36 hours prior to surgery. In the recovery room, he is noted to have signs of bleeding from the wound. What is the most likely explanation?
Reduced Factor Xa levels
Platelet dysfunction
Reduced Factor IX levels
Injury to the epigastric vein
Injury to the femoral artery
Platelet dysfunction. The clopidogrel should have been stopped sooner. Vascular injuries are usually evident intraoperatively.
Clopidogrel
Clopidogrel is an antiplatelet agent that is usually administered orally in doses of 75mg. It is converted by the P450 enzymes in the liver into the active drug. It decreases ADP induced platelet aggregation persisting for 120 hours after the final dose. This last point is especially relevant for surgeons as it means that it needs to be stopped around 5-7 days prior to surgery. Its effects are very potent and failure to adhere to this can result in significant increases in perioperative bleeding that can be difficult to control.
A 42 year old woman complains of a burning pain of her anterolateral thigh which worsens on walking. There is a positive tinel sign over the inguinal ligament. Which nerve is affected?
Ilioinguinal nerve
Genitofemoral nerve
Lateral cutaneous nerve of the thigh
Femoral nerve
Saphenous nerve
The lateral cutaneous nerve supplies sensation to the anterior and lateral aspect of the thigh. Entrapment is commonly due to intra and extra pelvic causes. Treatment involves local anaesthetic injections.
Lateral cutaneous nerve of the thigh
Cutaneous nerve arising from posterior surface of the second and third lumbar ventral rami
Emerges from the lateral border of psoas major anterior to the iliac crest, and passes between iliacus and iliac fascia
Enters thigh posterior to the lateral end of the inguinal ligament, medial to the anterior superior iliac spine
It pierces the fascia lata 10cm inferior to the anterior superior iliac spine and divides into 2 branches
Anterior branch supplies skin and fascia of the anterolateral surface of the knee
Smaller posterior branch supplies the skin and fascia on the lateral part of the upper leg between the greater trochanter and distal third of the thigh
A 56 year old man presents with a painless swelling in the upper part of the anterior triangle of his neck. On examination a mass lesion involving the sub mandibular gland is identified. On CT scanning this is shown to be a solid lesion. There is no regional lymphadenopathy. Two fine needle aspirates have failed to be diagnostic. Which of the following is the most appropriate management option?
Sub mandibular gland excision
Incisional biopsy of the mass
Manage conservatively and repeat the CT scan in 6 months
Sub mandibular gland excision and radical neck dissection
Diagnostic excision of the superficial lobe of the submandibular gland
There is a 50% risk that this lesion is malignant (in some series up to 70%). Therefore the gland should be excised entirely. At this stage a radical neck dissection is not justified.
Submandibular glands- disease
Physiology
The submandibular glands secrete approximately 800- 1000ml saliva per day. They typically produce mixed seromucinous secretions. When parasympathetic activity is dominant; the secretions will be more serous. The parasympathetic fibres are derived from the chorda tympani nerves and the submandibular ganglion. Sensory fibres are conveyed by the lingual branch of the mandibular nerve.
Sialolithiasis
80% of all salivary gland calculi occur in the submandibular gland
70% of the these calculi are radio-opaque
Stones are usually composed of calcium phosphate or calcium carbonate
Patients typically develop colicky pain and post prandial swelling of the gland
Investigation involves sialography to demonstrate the site of obstruction and associated other stones
Stones impacted in the distal aspect of Whartons duct may be removed orally, other stones and chronic inflammation will usually require gland excision
Sialadenitis
Usually occurs as a result of Staphylococcus aureus infection
Pus may be seen leaking from the duct, erythema may also be noted
Development of a sub mandibular abscess is a serious complication as it may spread through the other deep fascial spaces and occlude the airway
Submandibular tumours
Only 8% of salivary gland tumours affect the sub mandibular gland
Of these 50% are malignant (usually adenoid cystic carcinoma)
Diagnosis usually involves fine needle aspiration cytology
Imaging is with CT and MRI
In view of the high prevalence of malignancy, all masses of the submandibular glands should generally be excised.
A 43 year old man falls over landing on his left hand. Although there was anatomical snuffbox tenderness; no x-rays either at the time, or subsequently, have shown evidence of scaphoid fracture. He has been immobilised in a futura splint for two weeks and is now asymptomatic. What is the most appropriate course of action?
Application of tubigrip bandage and fracture clinic review
Admission and surgical debridement
Application of futura splint and fracture clinic review
Application of below elbow cast for 6 weeks
Discharge with reassurance
This patient is at extremely low risk of having sustained a scaphoid injury and may be discharged.
Scaphoid fractures
- Incidence of scaphoid fractures in UK ranges from 12.4 per 100,000 to 29 per 100,000
Surface of scaphoid is covered by articular cartilage with small area available for blood vessels (fracture risks blood supply)
Forms floor of anatomical snuffbox
Risk of fracture associated with fall onto outstretched hand (tubercle, waist, or proximal third)
A series of 4 scaphoid radiographs should be undertaken (PA, pronated oblique, Ziter view and lateral view). The Ziter view is a PA view with the wrist in ulnar deviation and beam angulated at 20 degrees
Sensitivity of scaphoid radiographs in 1st week of injury is 80%
Immobilization of scaphoid fractures difficult
Repeat imaging should be done at 10 days. MRI should be done in cases of diagnostic uncertainty
Classification of scaphoid fractures
Scaphoid tubercle
Distal pole
Waist
Proximal pole
Management
Undisplaced fractures of the waist of the scaphoid and most distal pole fractures can be managed in a cast for 6 weeks with high rates of union.
Displaced scaphoid waist fractures (more than 1-2mm) should be viewed as unstable and surgically fixed.
All proximal pole fractures should be fixed surgically.
Complications
Non union of scaphoid
Avascular necrosis of the scaphoid
Scapholunate disruption and wrist collapse
Degenerative changes of the adjacent joint
Reference
Berber O et al. Fractures of the scaphoid. BMJ 2020 (369): 414-416.
A 56 year old man is diagnosed with an abdominal aortic aneurysm and undergoes a CT scan to assess the size of the aorta. During the course of his investigations a lesion of the adrenal gland is identified. It measures 1.5 cm in diameter and the gland is otherwise normal. What is the most likely diagnosis?
Adrenal gland metastasis
Adrenal gland arterio-venous malformation
Adrenal cyst
Phaeochromocytoma
Adrenal cortical adenoma
Adrenal cortical adenoma
25% of all adrenal lesions >4cm in diameter are malignant
Incidentalomas of the adrenal gland are common and represent the most likely lesion in this scenario. Clearly the other lesions are all possibilities but are unlikely.
Adrenal lesions- Incidental
Incidentaloma of the adrenal glands have become increasingly common as CT scanning of the abdomen is widely undertaken. Prevalences range from 1.5-9% in autopsy studies. Overall, 75% will be non functioning adenomas. Investigation to exclude a functioning lesion is as below:
Investigation
Morning and midnight plasma cortisol measurements
Dexamethasone suppression test
24 hour urinary cortisol excretion
24 hour urinary excretion of catecholamines
Serum potassium, aldosterone and renin levels
Management
The risk of malignancy is related to the size of the lesion and 25% of all masses greater than 4cm will be malignant. Such lesions should usually be excised. Where a lesion is a suspected metastatic deposit a biopsy may be considered.
A 73 year old man undergoes a sub total oesophagectomy with anastomosis of the stomach to the cervical oesophagus. Which vessel will be primarily responsible for the arterial supply to the oesophageal portion of the anastomosis?
Superior thyroid artery
Internal carotid artery
Direct branches from the thoracic aorta
Inferior thyroid artery
Subclavian artery
The cervical oesophagus is supplied by the inferior thyroid artery. The thoracic oesophagus (removed in this case) is supplied by direct branches from the thoracic aorta.
Oesophagus
- 25cm long
Starts at C6 vertebra, pierces diaphragm at T10 and ends at T11
Squamous epithelium
Constrictions of the oesophagus
Structure Distance from incisors
Cricoid cartilage 15cm
Arch of the Aorta 22.5cm
Left principal bronchus 27cm
Diaphragmatic hiatus 40cm
Relations
Anteriorly
Trachea to T4
Recurrent laryngeal nerve
Left bronchus, Left atrium
Diaphragm
Posteriorly
Thoracic duct to left at T5
Hemiazygos to the left T8
Descending aorta
First 2 intercostal branches of aorta
Left
Thoracic duct
Left subclavian artery
Right
Azygos vein
Arterial, venous and lymphatic drainage of the oesophagus
Artery Vein Lymphatics Muscularis externa
Upper third Inferior thyroid Inferior thyroid Deep cervical Striated muscle
Mid third Aortic branches Azygos branches Mediastinal Smooth & striated muscle
Lower third Left gastric Left gastric Gastric Smooth muscle
Nerve supply
Upper half is supplied by recurrent laryngeal nerve
Lower half by oesophageal plexus (vagus)
Histology
Mucosa :Non-keratinized stratified squamous epithelium
Submucosa: glandular tissue
Muscularis externa (muscularis): composition varies. See table
Adventitia
A 41 year old man is admitted with peritonitis secondary to a perforated appendix. He is treated with a laparoscopic appendicectomy but has a stormy post operative course. He is now developing increasing abdominal pain and has been vomiting. A laparotomy is performed and at operation a large amount of small bowel shows evidence of patchy areas of infarction. What is the most likely cause?
Mesenteric venous thrombosis
Superior mesenteric artery embolus
Acute on chronic mesenteric ischaemia
Vasculitis
Median arcuate ligament syndrome
Mesenteric vein thrombosis may complicate severe intra abdominal sepsis and when it progresses may impair bowel perfusion. The serosa is quite resistant to ischaemia so in this case the appearances are usually patchy.
Mesenteric vessel disease
Mesenteric ischaemia accounts for 1 in 1000 acute surgical admissions. It is primarily caused by arterial embolism resulting in infarction of the colon. It is more likely to occur in areas such as the splenic flexure that are located at the borders of the territory supplied by the superior and inferior mesenteric arteries.
Types
Acute mesenteric embolus (commonest 50%)
Sudden onset abdominal pain followed by profuse diarrhoea.
May be associated with vomiting.
Rapid clinical deterioration.
Serological tests: WCC, lactate, amylase may all be abnormal particularly in established disease. These can be normal in the early phases.
Acute on chronic mesenteric ischaemia
Usually longer prodromal history.
Post prandial abdominal discomfort and weight loss are dominant features. Patients will usually present with an acute on chronic event, but otherwise will tend not to present until mesenteric flow is reduced by greater than 80%.
When acute thrombosis occurs presentation may be as above. In the chronic setting the symptoms will often be those of ischaemic colitis (mucosa is the most sensitive area to this insult).
Mesenteric vein thrombosis
Usually a history over weeks.
Overt abdominal signs and symptoms will not occur until venous thrombosis has reached a stage to compromise arterial inflow.
Thrombophilia accounts for 60% of cases.
Low flow mesenteric infarction
This occurs in patients with multiple co morbidities in whom mesenteric perfusion is significantly compromised by overuse of inotropes or background cardiovascular compromise.
The end result is that the bowel is not adequately perfused and infarcts occur from the mucosa outwards.
Diagnosis
Serological tests: WCC, lactate, CRP, amylase (can be normal in early disease).
Cornerstone for diagnosis of arterial AND venous mesenteric disease is CT angiography scanning in the arterial phase with thin slices (<5mm). Venous phase contrast is not helpful.
SMA duplex USS is useful in the evaluation of proximal SMA disease in patients with chronic mesenteric ischaemia.
MRI is of limited use due to gut peristalsis and movement artefact.
Management
Overt signs of peritonism: Laparotomy
Mesenteric vein thrombosis: If no peritonism: Medical management with IV heparin
At operation limited resection of frankly necrotic bowel with view to relook laparotomy at 24-48h. In the interim urgent bowel revascularisation via endovascular (preferred) or surgery.
Prognosis
Overall poor. Best outlook is from an acute ischaemia from an embolic event where surgery occurs within 12h. Survival may be 50%. This falls to 30% with treatment delay. The other conditions carry worse survival figures.
Which of the following structures passes through the quadrangular space near the humeral head?
Axillary artery
Radial nerve
Axillary nerve
Median nerve
Transverse scapular artery
The quadrangular space is bordered by the humerus laterally, subscapularis and teres minor superiorly, teres major inferiorly and the long head of triceps medially. It lies lateral to the triangular space. It transmits the axillary nerve and posterior circumflex humeral artery.
Shoulder joint
Shallow synovial ball and socket type of joint.
It is an inherently unstable joint, but is capable to a wide range of movement.
Stability is provided by muscles of the rotator cuff that pass from the scapula to insert in the greater tuberosity (all except sub scapularis-lesser tuberosity).
Glenoid labrum
Fibrocartilaginous rim attached to the free edge of the glenoid cavity
Tendon of the long head of biceps arises from within the joint from the supraglenoid tubercle, and is fused at this point to the labrum.
The long head of triceps attaches to the infraglenoid tubercle
Fibrous capsule
Attaches to the scapula external to the glenoid labrum and to the labrum itself (postero-superiorly)
Attaches to the humerus at the level of the anatomical neck superiorly and the surgical neck inferiorly
Anteriorly the capsule is in contact with the tendon of subscapularis, superiorly with the supraspinatus tendon, and posteriorly with the tendons of infraspinatus and teres minor. All these blend with the capsule towards their insertion.
Two defects in the fibrous capsule; superiorly for the tendon of biceps. Anteriorly there is a defect beneath the subscapularis tendon.
The inferior extension of the capsule is closely related to the axillary nerve at the surgical neck and this nerve is at risk in anteroinferior dislocations. It also means that proximally sited osteomyelitis may progress to septic arthritis.
Movements and muscles
Flexion Anterior part of deltoid
Pectoralis major
Biceps
Coracobrachialis
Extension Posterior deltoid
Teres major
Latissimus dorsi
Adduction Pectoralis major
Latissimus dorsi
Teres major
Coracobrachialis
Abduction Mid deltoid
Supraspinatus
Medial rotation Subscapularis
Anterior deltoid
Teres major
Latissimus dorsi
Lateral rotation Posterior deltoid
Infraspinatus
Teres minor
Important anatomical relations
Anteriorly Brachial plexus
Axillary artery and vein
Posterior Suprascapular nerve
Suprascapular vessels
Inferior Axillary nerve
Circumflex humeral vessels
A 35 year old male presents with haematuria. He is found to have bilateral masses in the flanks. He has a history of epilepsy and learning disability. Which of the lesions below is most likely?
Angiomyolipoma
Renal cortical cysts
Transitional cell cancer
Nephroblastomas
Staghorn calculi
This patient has tuberous sclerosis. This is associated with angiomyolipoma, which is present in 60-80% patients. It is a benign lesion.
Renal lesions
Lesion Disease specific features Treatment
Renal cell carcinoma
Most present with haematuria (50%)
Common renal tumour (85% cases)
Paraneoplastic features include hypertension and polycythaemia
Most commonly has haematogenous mestastasis
Usually radical or partial nephrectomy
Nephroblastoma
Rare childhood tumour
It accounts for 80% of all genitourinary malignancies in those under the age of 15 years
Up to 90% will have a mass
50% will be hypertensive
Diagnostic work up includes ultrasound and CT scanning
Surgical resection combined with chemotherapy (usually vincristine, actinomycin D and doxorubicin)
Neuroblastoma
Most common extracranial tumour of childhood
80% occur in those under 4 years of age
Tumour of neural crest origin (up to 50% occur in the adrenal gland)
The tumour is usually calcified and may be diagnosed using MIBG scanning
Staging is with CT
Surgical resection, radiotherapy and chemotherapy
Transitional cell carcinoma
Accounts for 90% of lower urinary tract tumours, but only 10% of renal tumours
Males affected 3x more than females
Occupational exposure to industrial dyes and rubber chemicals may increase risk
Up to 80% present with painless haematuria
Diagnosis and staging is with CT IVU
Radical nephroureterectomy
Angiomyolipoma
80% of these hamartoma type lesions occur sporadically, the remainder are seen in those with tuberous sclerosis
Tumour is composed of blood vessels, smooth muscle and fat
Massive bleeding may occur in 10% of cases
50% of patients with lesions >4cm will have symptoms and will require surgical resection
A 74 year old lady falls and injures her left arm. Following assessment she is found to have an impacted fracture affecting the surgical neck of the humerus. What is the most appropriate course of action?
Reduce the fracture and apply a plate to stabilize the fragments
Perform a hemiarthroplasty
Apply a collar and cuff system for three weeks and then commence physiotherapy
Apply an upper limb cast for 8 weeks
Apply an external fixator system
Impacted fractures of the surgical neck are stable injuries and usually heal without complication. It is rare to need to resort to surgery. Its important to start physiotherapy early.
Proximal humerus fractures
Very common injury. Usually through the surgical neck. Number of classification systems though for practical purposes describing the number of fracture fragments is probably easier. Some key points:
It is rare to have fractures through the anatomical neck.
Anatomical neck fractures which are displaced by >1cm carry a risk of avascular necrosis to the humeral head.
In children the commonest injury pattern is a greenstick fracture through the surgical neck.
Impacted fractures of the surgical neck are usually managed with a collar and cuff for 3 weeks followed by physiotherapy.
More significant displaced fractures may require open reduction and fixation or use of an intramedullary device.
A 45 year old man complains of sharp chest pain. He is due to have elective surgery to replace his left hip. He has been bed bound for 3 months. He suddenly collapses; his blood pressue is 70/40mmHg, heart rate 120 bpm and his saturations are 74% on air. He is deteriorating in front of you. What is the next best management plan?
Aspirin
Thrombolysis with Alteplase
Unfractionated heparin
Thrombolysis with streptokinase
Clopidogrel
This man is peri arrest with the diagnosis of pulmonary embolism (chest pain,bedbound, collapse, low saturations). He needs urgent thrombolysis with alteplase (he may not survive if you wait for the medical Spr/ITU to arrive!).
Pulmonary embolism: management
A summary of the British Thoracic Society guidelines
Heparin should be given if intermediate or high clinical probability before imaging.
Unfractionated heparin (UFH) should be considered (a) as a first dose bolus, (b) in massive PE, or (c) where rapid reversal of effect may be needed.
Otherwise, low molecular weight heparin (LMWH) should be considered as preferable to UFH, having equal efficacy and safety and being easier to use.
Oral anticoagulation should only be commenced once VTE has been reliably confirmed.
The target INR should be 2.0-3.0; when this is achieved, heparin can be discontinued.
The standard duration of oral anticoagulation is: 4 to 6 weeks for temporary risk factors, 3 months for first idiopathic, and at least 6 months for other; the risk of bleeding should be balanced with that of further VTE.
Massive PE
CTPA or echocardiography will reliably diagnose clinically massive PE.
Thrombolysis is 1st line for massive PE (ie circulatory failure) and may be instituted on clinical grounds alone if cardiac arrest is imminent; a 50 mg bolus of alteplase is recommended.
Invasive approaches (thrombus fragmentation and IVC filter insertion) should be considered where facilities and expertise are readily available.
Which of the following is the most sensitive blood test for diagnosis of acute pancreatitis?
Amylase
Lipase
C-peptide
Trypsin
Trysinogen
The serum amylase may rise and fall quite quickly and lead to a false negative result. Should the clinical picture not be concordant with the amylase level then serum lipase or a CT Scan should be performed.
Management of Pancreatitis
Management of Acute Pancreatitis in the UK
Diagnosis
Traditionally hyperamylasaemia has been utilised with amylase being elevated three times the normal range.
However, amylase may give both false positive and negative results.
Serum lipase is both more sensitive and specific than serum amylase. It also has a longer half life.
Serum amylase levels do not correlate with disease severity.
Differential causes of hyperamylasaemia
Acute pancreatitis
Pancreatic pseudocyst
Mesenteric infarct
Perforated viscus
Acute cholecystitis
Diabetic ketoacidosis
Assessment of severity
Glasgow, Ranson scoring systems and APACHE II
Biochemical scoring e.g. using CRP
Features that may predict a severe attack within 48 hours of admission to hospital
Initial assessment
Clinical impression of severity
Body mass index >30
Pleural effusion
APACHE score >8
24 hours after admission
Clinical impression of severity
APACHE II >8
Glasgow score of 3 or more
Persisting multiple organ failure
CRP>150
48 hours after admission
Glasgow Score of >3
CRP >150
Persisting or progressive organ failure
Table adapted from UK guidelines for management of acute pancreatitis. GUT 2005, 54 suppl III
Management
Nutrition
There is reasonable evidence to suggest that the use of enteral nutrition does not worsen the outcome in pancreatitis
Most trials to date were underpowered to demonstrate a conclusive benefit.
The rationale behind feeding is that it helps to prevent bacterial translocation from the gut, thereby contributing to the development of infected pancreatic necrosis.
Use of antibiotic therapy
Many UK surgeons administer antibiotics to patients with acute pancreatitis. However, there is very little evidence to support this practice.
A recent Cochrane review highlights the potential benefits of administering Imipenem to patients with established pancreatic necrosis in the hope of averting the progression to infection.
There are concerns that the administration of antibiotics in mild attacks of pancreatitis will not affect outcome and may contribute to antibiotic resistance and increase the risks of antibiotic associated diarrhoea.
Surgery
Patients with acute pancreatitis due to gallstones should undergo early cholecystectomy.
Patients with obstructed biliary system due to stones should undergo early ERCP.
Patients with extensive necrosis where infection is suspected should usually undergo FNA for culture.
Patients with infected necrosis should undergo either radiological drainage or surgical necrosectomy. The choice of procedure depends upon local expertise.
References
www.bsg.org.uk/pdfworddocs/pancreatic.pdf
Antibiotic therapy for prophylaxis against infection of pancreatic necrosis in acute pancreatitis. Villatoro et al. Cochrane Library DOI: 10.1002/14651858.CD002941.pub3. 2010 version.
In paediatric orthopaedic surgery, which of the following does not fulfill the Kocher criteria for septic arthritis?
ESR > 40mm/h
Positive blood culture
Fever
White cell count > 12, 000
Non weight bearing on the affected side
Kocher’s ‘WIFE’ is:
WCC >12
Inability to weight bear
Fever
ESR >40
= >90 chance of septic arthritis
Kocher criteria
1. Non weight bearing on affected side
2. ESR > 40 mm/hr
3. Fever
4. WBC count of >12,000 mm3
- When 4/4 criteria are met, there is a 99% chance that the child has septic arthritis
The Kocher criteria do not consider blood culture results.
Septic arthritis- Paediatric
Septic arthritis
Staph aureus commonest organism
Urgent washout and antibiotics otherwise high risk of joint destruction
Diagnosis
Plain x-rays
Consider aspiration
Utilise the Kocher criteria (see below)
Kocher criteria:
1. Non weight bearing on affected side
2. ESR > 40 mm/hr
3. Fever
4. WBC count of >12,000 mm3
- when 4/4 criteria are met, there is a 99% chance that the child has septic arthritis
Treatment
Surgical drainage of the affected joint is required, this should be done as soon as possible since permanent damage to the joint may occur. In some cases repeated procedures are necessary. Appropriate intravenous antibiotics should be administered.
A 43 year old lady with a metallic heart valve has just undergone an elective paraumbilical hernia repair. In view of her metallic valve, she is given unfractionated heparin perioperatively. How should the therapeutic efficacy be monitored, assuming her renal function is normal?
Therapeutic monitoring is not required
Measurement of APTT
Measurement of INR
Measurement of Prothromin time
None of the above
Unlike low molecular weight heparins that do not require monitoring unfractionated heparin does require monitoring, this is done by measuring the APTT.
Heparin
Causes the formation of complexes between antithrombin and activated thrombin/factors 7,9,10,11 & 12
Advantages of low molecular weight heparin
Better bioavailability
Lower risk of bleeding
Longer half life
Little effect on APTT at prophylactic dosages
Less risk of HIT
Complications
Bleeding
Osteoporosis
Heparin induced thrombocytopenia (HIT): occurs 5-14 days after 1st exposure
Anaphylaxis
In surgical patients that may need a rapid return to theatre, administration of unfractionated heparin is preferred; as low molecular weight heparins have a longer duration of action and are harder to reverse.
Which of the following is the main site of dehydroepiandrosterone release?
Posterior pituitary
Zona reticularis of the adrenal gland
Zona glomerulosa of the adrenal gland
Juxtaglomerular apparatus of the kidney
Zona fasciculata of the adrenal gland
Adrenal cortex mnemonic: GFR - ACD
DHEA possesses some androgenic activity and is almost exclusively released from the adrenal gland.
Renin-angiotensin-aldosterone system
Adrenal cortex (mnemonic GFR - ACD)
Zona glomerulosa (on outside): mineralocorticoids, mainly aldosterone
Zona fasciculata (middle): glucocorticoids, mainly cortisol
Zona reticularis (on inside): androgens, mainly dehydroepiandrosterone (DHEA)
Renin
Released by JGA cells in kidney in response to reduced renal perfusion, low sodium
Hydrolyses angiotensinogen to form angiotensin I
Factors stimulating renin secretion
Low BP
Hyponatraemia
Sympathetic nerve stimulation
Catecholamines
Erect posture
Angiotensin
ACE in lung converts angiotensin I → angiotensin II
Vasoconstriction leads to raised BP
Stimulates thirst
Stimulates aldosterone and ADH release
Aldosterone
Released by the zona glomerulosa in response to raised angiotensin II, potassium, and ACTH levels
Causes retention of Na+ in exchange for K+/H+ in distal tubule
Which of the following statements in relation to the p53 tumour suppressor protein is false?
It may induce necrosis of cells with non repairable DNA damage
It is affected in Li Fraumeni syndrome
It can induce DNA repair
It can halt the cell cycle
It may inhibit angiogenesis
When DNA cannot be repaired it will induce cellular apoptosis (not necrosis)
Genetics and surgical disease
Some of the more commonly occurring genetic conditions occurring in surgical patients are presented here.
Li-Fraumeni Syndrome
Autosomal dominant
Consists of germline mutations to p53 tumour suppressor gene
High incidence of malignancies particularly sarcomas and leukaemias
Diagnosed when:
*Individual develops sarcoma under 45 years
*First degree relative diagnosed with any cancer below age 45 years and another family member develops malignancy under 45 years or sarcoma at any age
BRCA 1 and 2
Carried on chromosome 17 (BRCA 1) and Chromosome 13 (BRCA 2)
Linked to developing breast cancer (60%) risk.
Associated risk of developing ovarian cancer (55% with BRCA 1 and 25% with BRCA 2).
Lynch Syndrome
Autosomal dominant
Develop colonic cancer and endometrial cancer at young age
80% of affected individuals will get colonic and/ or endometrial cancer
High risk individuals may be identified using the Amsterdam criteria
Amsterdam criteria
Three or more family members with a confirmed diagnosis of colorectal cancer, one of whom is a first degree (parent, child, sibling) relative of the other two.
Two successive affected generations.
One or more colon cancers diagnosed under age 50 years.
Familial adenomatous polyposis (FAP) has been excluded.
Gardners syndrome
Autosomal dominant familial colorectal polyposis
Multiple colonic polyps
Extra colonic diseases include: skull osteoma, thyroid cancer and epidermoid cysts
Desmoid tumours are seen in 15%
Mutation of APC gene located on chromosome 5
Due to colonic polyps most patients will undergo colectomy to reduce risk of colorectal cancer
Now considered a variant of familial adenomatous polyposis coli
A 28 year old man lacerates the posterolateral aspect of his wrist with a knife in an attempted suicide. On arrival in the emergency department the wound is inspected and found to be located over the lateral aspect of the extensor retinaculum (which is intact). Which of the following structures is at greatest risk of injury?
Superficial branch of the radial nerve
Radial artery
Dorsal branch of the ulnar nerve
Tendon of extensor carpi radialis brevis
Tendon of extensor digiti minimi
The superficial branch of the radial nerve passes superior to the extensor retinaculum in the position of this laceration and is at greatest risk of injury. The dorsal branch of the ulnar nerve and artery also pass superior to the extensor retinaculum but are located medially.
Extensor retinaculum
The extensor retinaculum is a thickening of the deep fascia that stretches across the back of the wrist and holds the long extensor tendons in position.
Its attachments are:
The pisiform and triquetral medially
The end of the radius laterally
Structures related to the extensor retinaculum
Structures superficial to the retinaculum
Basilic vein
Dorsal cutaneous branch of the ulnar nerve
Cephalic vein
Superficial branch of the radial nerve
Structures passing deep to the extensor retinaculum
Extensor carpi ulnaris tendon
Extensor digiti minimi tendon
Extensor digitorum and extensor indicis tendon
Extensor pollicis longus tendon
Extensor carpi radialis longus tendon
Extensor carpi radialis brevis tendon
Abductor pollicis longus and extensor pollicis brevis tendons
Beneath the extensor retinaculum fibrous septa form six compartments that contain the extensor muscle tendons. Each compartment has its own synovial sheath.
The radial artery
The radial artery passes between the lateral collateral ligament of the wrist joint and the tendons of the abductor pollicis longus and extensor pollicis brevis.
Image illustrating the topography of tendons passing under the extensor retinaculum
An 18 year old lady with troublesome hyperhidrosis of the hands and arms is due to undergo a sympathectomy to treat the condition. Which of the following should the surgeons divide to most effectively treat her condition?
Sympathetic ganglia at T1, T2 and T3
Sympathetic ganglia at T2 and T3
Sympathetic ganglia at T1 and T2
Stellate ganglion
Superior cervical ganglion
To treat hyperhidrosis the sympathetic ganglia at T2 and T3 should be divided. Dividing the other structures listed would either carry a risk of Horners syndrome or be ineffective.
Useful mnemonic: T2 & T3 Make the hands sweat-free
Sympathetic nervous system- anatomy
The cell bodies of the pre-ganglionic efferent neurones lie in the lateral horn of the grey matter of the spinal cord in the thoraco-lumbar regions.
The pre-ganglionic efferents leave the spinal cord at levels T1-L2. These pass to the sympathetic chain.
Lateral branches of the sympathetic chain connect it to every spinal nerve. These post ganglionic nerves will pass to structures that receive sympathetic innervation at the periphery.
Sympathetic chains
These lie on the vertebral column and run from the base of the skull to the coccyx.
Cervical region Lie anterior to the transverse processes of the cervical vertebrae and posterior to the carotid sheath.
Thoracic region Lie anterior to the neck of the upper ribs and and lateral sides of the lower thoracic vertebrae.They are covered by the parietal pleura
Lumbar region Enter by passing posterior to the medial arcuate ligament. Lie anteriorly to the vertebrae and medial to psoas major.
Sympathetic ganglia
Superior cervical ganglion lies anterior to C2 and C3.
Middle cervical ganglion (if present) C6
Stellate ganglion- anterior to transverse process of C7, lies posterior to the subclavian artery, vertebral artery and cervical pleura.
Thoracic ganglia are segmentally arranged.
There are usually 4 lumbar ganglia.
Clinical importance
Interruption of the head and neck supply of the sympathetic nerves will result in an ipsilateral Horners syndrome.
For treatment of hyperhidrosis the sympathetic denervation can be achieved by removing the second and third thoracic ganglia with their rami. Removal of T1 will cause a Horners syndrome and is therefore not performed.
In patients with vascular disease of the lower limbs a lumbar sympathetomy may be performed, either radiologically or (more rarely now) surgically. The ganglia of L2 and below are disrupted. If L1 is removed then ejaculation may be compromised (and little additional benefit conferred as the preganglionic fibres do not arise below L2.
Which of these muscles is not a component of the rotator cuff?
Subscapularis
Teres minor
Supraspinatus
Infraspinatus
Deltoid
Mnemonic for rotator cuff muscles (TISS)
Teres minor
Infraspinatus
Supraspinatus
Subscapularis
Deltoid may abduct the s houlder and is not a rotator cuff muscle.
Muscles of the rotator cuff
Muscle Innervation
Supraspinatus muscle Suprascapular nerve
Infraspinatus muscle Suprascapular nerve
Teres minor muscle Axillary nerve
Subscapularis muscle Superior and inferior subscapular nerves
A 67 year old patient is due to undergo a femoro-popliteal bypass graft. Which heparin regime should the surgeon ask for prior to cross clamping the femoral artery?
Single therapeutic dose of low molecular weight heparin on the ward prior to coming to theatre
Single therapeutic dose of low molecular weight heparin the night before surgery
Dose of 10,000 units of unfractionated heparin prior to induction of anaesthesia
Dose of 3,000 units of unfractionated heparin, 3 minutes prior to cross clamping
Dose of 30,000 units of unfractionated heparin, 3 minutes prior to cross clamping
As a rule most vascular surgeons will administer approximately 3,000 units of systemic heparin 3-5 minutes prior to cross clamping to help prevent further intra arterial thromboses. A dose of 30,000 units is given prior to going on cardiopulmonary bypass. Heparin given at induction will cause bleeding during routine dissection.
Heparin
Causes the formation of complexes between antithrombin and activated thrombin/factors 7,9,10,11 & 12
Advantages of low molecular weight heparin
Better bioavailability
Lower risk of bleeding
Longer half life
Little effect on APTT at prophylactic dosages
Less risk of HIT
Complications
Bleeding
Osteoporosis
Heparin induced thrombocytopenia (HIT): occurs 5-14 days after 1st exposure
Anaphylaxis
In surgical patients that may need a rapid return to theatre, administration of unfractionated heparin is preferred; as low molecular weight heparins have a longer duration of action and are harder to reverse.
A 55 year old man is admitted with a brisk haematemesis. He is taken to the endoscopy department and an upper GI endoscopy is performed by the gastroenterologist. He identifies an ulcer on the posterior duodenal wall and spends an eternity trying to control the bleeding with all the latest haemostatic techniques. He eventually asks the surgeons for help. A laparotomy and anterior duodenotomy are performed, as the surgeon opens the duodenum a vessel is spurting blood into the duodenal lumen. From which of the following does this vessel arise?
Left gastric artery
Common hepatic artery
Right hepatic artery
Superior mesenteric artery
Splenic artery
The vessel will be the gastroduodenal artery, this arises from the common hepatic artery.
Gastroduodenal artery
Supplies
Pylorus, proximal part of the duodenum, and indirectly to the pancreatic head (via the anterior and posterior superior pancreaticoduodenal arteries)
Path
The gastroduodenal artery most commonly arises from the common hepatic artery of the coeliac trunk. It terminates by bifurcating into the right gastroepiploic artery and the superior pancreaticoduodenal artery
A 25 year old lady presents with a swelling located at the anterior border of the sternocleidomastoid muscle. The swelling is intermittent and on examination it is soft and fluctuant. What is the most likely diagnosis?
Thyroglossal cyst
Branchial cyst
Carotid body tumour
Infection with Bartonella
Dental abscess
Branchial cysts are remnants of the branchial cleft. They may become infected.
Neck lumps
The table below gives characteristic exam question features for conditions causing neck lumps:
Reactive lymphadenopathy By far the most common cause of neck swellings. There may be a history of local infection or a generalised viral illness
Lymphoma Rubbery, painless lymphadenopathy
The phenomenon of pain whilst drinking alcohol is very uncommon
There may be associated night sweats and splenomegaly
Thyroid swelling May be hypo-, eu- or hyperthyroid symptomatically
Moves upwards on swallowing
Thyroglossal cyst More common in patients < 20 years old
Usually midline, between the isthmus of the thyroid and the hyoid bone
Moves upwards with protrusion of the tongue
May be painful if infected
Pharyngeal pouch More common in older men
Represents a posteromedial herniation between thyropharyngeus and cricopharyngeus muscles
Usually not seen, but if large then a midline lump in the neck that gurgles on palpation
Typical symptoms are dysphagia, regurgitation, aspiration and chronic cough
Cystic hygroma A congenital lymphatic lesion (lymphangioma) typically found in the neck, classically on the left side
Most are evident at birth, around 90% present before 2 years of age
Branchial cyst An oval, mobile cystic mass that develops between the sternocleidomastoid muscle and the pharynx
Develop due to failure of obliteration of the second branchial cleft in embryonic development
Usually present in early adulthood
Cervical rib More common in adult females
Around 10% develop thoracic outlet syndrome
Carotid aneurysm Pulsatile lateral neck mass which doesn’t move on swallowing
What type of stoma should be considered in a patient undergoing emergency operative intervention for large bowel obstruction as a result of a carcinoma 5cm from the anal verge?
End colostomy
Loop colostomy
End ileostomy
Loop jejunostomy
Loop ileostomy
Don’t confuse loop ileostomy and loop colostomy. A loop colostomy is the only safe option for an obstructing rectal cancer. Loop ileostomy in the context of rectal cancer is performed to mitigate the effects of anastomotic leak following a low anterior resection.
Large bowel obstruction resulting from carcinoma should be resected, stented or defunctioned. The first two options typically apply to tumours above the peritoneal reflection. Lower tumours should be defunctioned with a loop colostomy and then formal staging undertaken prior to definitive surgery. An emergency attempted rectal resection carries a high risk of involvement of the circumferential resection margin and is not recommended.
Abdominal stomas
Stomas may be sited during a range of abdominal procedures and involve bringing the lumen or visceral contents onto the skin. In most cases this applies to the bowel. However, other organs or their contents may be diverted in case of need.
With bowel stomas the type method of construction and to a lesser extent the site will be determined by the contents of the bowel. In practice, small bowel stomas should be spouted so that their irritant contents are not in contact with the skin. Colonic stomas do not need to be spouted as their contents are less irritant.
In the ideal situation the site of the stoma should be marked with the patient prior to surgery. Stoma siting is important as it will ultimately influence the ability of the patient to manage their stoma and also reduce the risk of leakage. Leakage of stoma contents and subsequent maceration of the surrounding skin can rapidly progress into a spiraling loss of control of stoma contents.
Types of stomas
Name of stoma Use Common sites
Gastrostomy
Gastric decompression or fixation
Feeding
Epigastrium
Loop jejunostomy
Seldom used as very high output
May be used following emergency laparotomy with planned early closure
Any location according to need
Percutaneous jejunostomy
Usually performed for feeding purposes and site in the proximal bowel
Usually left upper quadrant
Loop ileostomy
Defunctioning of colon e.g. following rectal cancer surgery
Does not decompress colon (if ileocaecal valve competent)
Usually right iliac fossa
End ileostomy
Usually following complete excision of colon or where ileo-colic anastomosis is not planned
May be used to defunction colon, but reversal is more difficult
Usually right iliac fossa
End colostomy Where a colon is diverted or resected and anastomosis is not primarily achievable or desirable Either left or right iliac fossa
Loop colostomy
To defunction a distal segment of colon
Since both lumens are present the distal lumen acts as a vent
May be located in any region of the abdomen, depending upon colonic segment used
Caecostomy Stoma of last resort where loop colostomy is not possible Right iliac fossa
Mucous fistula
To decompress a distal segment of bowel following colonic division or resection
Where closure of a distal resection margin is not safe or achievable
A 28 year old man falls on the back of his hand. On x-ray, he has a fractured distal radius demonstrating volar displacement of the fracture. What eponymous term is used to describe this?
Barton’s
Colles’
Smith’s
Pott’s
Galeazzi
This is a Smith fracture (reverse Colles’ fracture); unlike a Colles’ this is a high velocity injury and may require surgical correction. Note that Colles’ fractures are usually dorsally displaced.
Eponymous fractures
Colles’ fracture (dinner fork deformity)
Fall onto extended outstretched hand
Classical Colles’ fractures have the following 3 features:
- Transverse fracture of the radius
- 1 inch proximal to the radio-carpal joint
- Dorsal displacement and angulation
Smith’s fracture (reverse Colles’ fracture)
Volar angulation of distal radius fragment (Garden spade deformity)
Caused by falling backwards onto the palm of an outstretched hand or falling with wrists flexed
Bennett’s fracture
Intra-articular fracture of the first carpometacarpal joint
Impact on flexed metacarpal, caused by fist fights
X-ray: triangular fragment at ulnar base of metacarpal
Monteggia’s fracture
Dislocation of the proximal radioulnar joint in association with an ulna fracture
Fall on outstretched hand with forced pronation
Needs prompt diagnosis to avoid disability
Galeazzi fracture
Radial shaft fracture with associated dislocation of the distal radioulnar joint
Direct blow
Pott’s fracture
Bimalleolar ankle fracture
Forced foot eversion
Barton’s fracture
Distal radius fracture (Colles’/Smith’s) with associated radiocarpal dislocation
Fall onto extended and pronated wrist
Involvement of the joint is a defining feature
Holstein Lewis Fracture
A HolsteinLewis fracture is a fracture of the distal third of the humerus resulting in entrapment of the radial nerve.
The radial nerve is one of the major nerves of the upper limb. It innervates all of the muscles in the extensor compartments of the arm.
Conservative treatment includes reduction and use of a functional brace
Vascular injury may require open surgery
A 23 year old man is undergoing an inguinal hernia repair under local anaesthesia. The surgeon encounters a bleeding site which he manages with diathermy. About a minute or so later the patient complains that he is able to feel the burning pain of the heat at the operative site. Which of the following nerve fibres is responsible for the transmission of this signal?
A α fibres
A β fibres
B fibres
C fibres
None of the above
Slow transmission of mechanothermal stimuli is transmitted via C fibres.
A γ fibres transmit information relating to motor proprioception, A β fibres transmit touch and pressure and B fibres are autonomic fibres.
Pain - neuronal transmission
Somatic pain
Peripheral nociceptors are innervated by either small myelinated fibres (A-delta) fibres or by unmyelinated C fibres.
The A gamma fibres register high intensity mechanical stimuli. The C fibres usually register high intensity mechanothermal stimuli.
A 19 year old female is involved in an athletics event. She has just completed the high jump when she suddenly develops severe back pain and weakness affecting both her legs. On examination, she has a prominent sacrum and her lower back is painful. What is the most likely underlying cause?
Structural scoliosis
Ankylosing spondylitis
Scheuermanns disease
Spondylolisthesis
Spondylolysis
Young athletic females are the group most frequently affected by spondylolythesis who have a background of spondylolysis. Whilst the latter condition is a risk factor for spondylolythesis the former condition is most likely in a young athletic female who presents with sudden pain.
Diseases affecting the vertebral column
Ankylosing spondylitis
Chronic inflammatory disorder affecting the axial skeleton
Sacro-ilitis is a usually visible in plain films
Up to 20% of those who are HLA B27 positive will develop the condition
Affected articulations develop bony or fibrous changes
Typical spinal features include loss of the lumbar lordosis and progressive kyphosis of the cervico-thoracic spine
Scheuermann’s disease
Epiphysitis of the vertebral joints is the main pathological process
Predominantly affects adolescents
Symptoms include back pain and stiffness
X-ray changes include epiphyseal plate disturbance and anterior wedging
Clinical features include progressive kyphosis (at least 3 vertebrae must be involved)
Minor cases may be managed with physiotherapy and analgesia, more severe cases may require bracing or surgical stabilisation
Scoliosis
Consists of curvature of the spine in the coronal plane
Divisible into structural and non structural, the latter being commonest in adolescent females who develop minor postural changes only. Postural scoliosis will typically disappear on manoeuvres such as bending forwards
Structural scoliosis affects > 1 vertebral body and is divisible into idiopathic, congential and neuromuscular in origin. It is not correctable by alterations in posture
Within structural scoliosis, idiopathic is the most common type
Severe, or progressive structural disease is often managed surgically with bilateral rod stabilisation of the spine
Spina bifida
Non fusion of the vertebral arches during embryonic development
Three categories; myelomeningocele, spina bifida occulta and meningocele
Myelomeningocele is the most severe type with associated neurological defects that may persist in spite of anatomical closure of the defect
Up to 10% of the population may have spina bifida occulta, in this condition the skin and tissues (but not not bones) may develop over the distal cord. The site may be identifiable by a birth mark or hair patch
The incidence of the condition is reduced by use of folic acid supplements during pregnancy
Spondylolysis
Congenital or acquired deficiency of the pars interarticularis of the neural arch of a particular vertebral body, usually affects L4/ L5
May be asymptomatic and affects up to 5% of the population
Spondylolysis is the commonest cause of spondylolisthesis in children
Asymptomatic cases do not require treatment
Spondylolisthesis
This occurs when one vertebra is displaced relative to its immediate inferior vertebral body
May occur as a result of stress fracture or spondylolysis
Traumatic cases may show the classic ‘Scotty Dog’ appearance on plain films
Treatment depends upon the extent of deformity and associated neurological symptoms, minor cases may be actively monitored. Individuals with radicular symptoms or signs will usually require spinal decompression and stabilisation
A 23 year old lady with troublesome axillary hyperhidrosis is undergoing a thorascopic sympathectomy to treat the condition. Which of the following structures will need to be divided to access the sympathetic trunk?
Intercostal vein
Intercostal artery
Parietal pleura
Visceral pleura
None of the above
The sympathetic chain lies posterior to the parietal pleura. During a thorascopic sympathectomy this structure will need to be divided. The intercostal vessels lie posteriorly. They may be damaged with troublesome bleeding but otherwise are best left alone as deliberate division will not improve surgical access.
Sympathetic nervous system- anatomy
The cell bodies of the pre-ganglionic efferent neurones lie in the lateral horn of the grey matter of the spinal cord in the thoraco-lumbar regions.
The pre-ganglionic efferents leave the spinal cord at levels T1-L2. These pass to the sympathetic chain.
Lateral branches of the sympathetic chain connect it to every spinal nerve. These post ganglionic nerves will pass to structures that receive sympathetic innervation at the periphery.
Sympathetic chains
These lie on the vertebral column and run from the base of the skull to the coccyx.
Cervical region Lie anterior to the transverse processes of the cervical vertebrae and posterior to the carotid sheath.
Thoracic region Lie anterior to the neck of the upper ribs and and lateral sides of the lower thoracic vertebrae.They are covered by the parietal pleura
Lumbar region Enter by passing posterior to the medial arcuate ligament. Lie anteriorly to the vertebrae and medial to psoas major.
Sympathetic ganglia
Superior cervical ganglion lies anterior to C2 and C3.
Middle cervical ganglion (if present) C6
Stellate ganglion- anterior to transverse process of C7, lies posterior to the subclavian artery, vertebral artery and cervical pleura.
Thoracic ganglia are segmentally arranged.
There are usually 4 lumbar ganglia.
Clinical importance
Interruption of the head and neck supply of the sympathetic nerves will result in an ipsilateral Horners syndrome.
For treatment of hyperhidrosis the sympathetic denervation can be achieved by removing the second and third thoracic ganglia with their rami. Removal of T1 will cause a Horners syndrome and is therefore not performed.
In patients with vascular disease of the lower limbs a lumbar sympathetomy may be performed, either radiologically or (more rarely now) surgically. The ganglia of L2 and below are disrupted. If L1 is removed then ejaculation may be compromised (and little additional benefit conferred as the preganglionic fibres do not arise below L2.
The recurrent laryngeal is connected to which of the following nerves?
Trigeminal
Accessory
Hypoglossal
Vagus
Glossopharyngeal
Recurrent laryngeal nerve
Branch of the vagus nerve
Path
Right
Arises anterior to the subclavian artery and ascends obliquely next to the trachea, behind the common carotid artery
It is either anterior or posterior to the inferior thyroid artery
Left
Arises left to the arch of the aorta
Winds below the aorta
Ascends along the side of the trachea
Then both
Pass in a groove between the trachea and oesophagus
Enters the larynx behind the articulation between the thyroid cartilage and cricoid
Distributed to larynx muscles
Branches to
Cardiac plexus
Mucous membrane and muscular coat of the oesophagus and trachea
Innervates
Intrinsic larynx muscles (excluding cricothyroid)
Stimulation of the parasympathetic nerve supply to the bladder is most likely to result in which of the following actions?
Contraction of the internal urethral sphincter
Relaxation of the external urethral sphincter
Contraction of the external urethral sphincter
Contraction of the detrusor muscle
Relaxation of the detrusor muscle
The internal sphincter is formed by the circular arrangement of smooth muscle of the bladder wall at the neck of the bladder. The detrusor muscle and internal sphincter are under autonomic control. Parasympathetic fibres cause contraction of the detrusor muscle and relaxation of the internal sphincter. Sympathetic fibres cause relaxation of the detrusor muscle and contraction of the internal sphincter. The external sphincter is under voluntary control.
Bladder
The empty bladder is contained within the pelvic cavity. It is usually a three sided pyramid. The apex of the bladder points forwards towards the symphysis pubis and the base lies immediately anterior to the rectum or vagina. Continuous with the apex is the median umbilical ligament, during development this was the site of the urachus.
The inferior aspect of the bladder is retroperitoneal and the superior aspect covered by peritoneum. As the bladder distends it will tend to separate the peritoneum from the fascia of transversalis. For this reason a bladder that is distended due to acute urinary retention may be approached with a suprapubic catheter that avoids entry into the peritoneal cavity.
The trigone is the least mobile part of the bladder and forms the site of the ureteric orifices and internal urethral orifice. In the empty bladder the ureteric orifices are approximately 2-3cm apart, this distance may increase to 5cm in the distended bladder.
Arterial supply
The superior and inferior vesical arteries provide the main blood supply to the bladder. These are branches of the internal iliac artery.
Venous drainage
In males the bladder is drained by the vesicoprostatic venous plexus. In females the bladder is drained by the vesicouterine venous plexus. In both sexes this venous plexus will ultimately drain to the internal iliac veins.
Lymphatic drainage
Lymphatic drainage is predominantly to the external iliac nodes, internal iliac and obturator nodes also form sites of bladder lymphatic drainage.
Innervation
Parasympathetic nerve fibres innervate the bladder from the pelvic splanchnic nerves. Sympathetic nerve fibres are derived from L1 and L2 via the hypogastric nerve plexuses. The parasympathetic nerve fibres will typically cause detrusor muscle contraction and result in voiding. The muscle of the trigone is innervated by the sympathetic nervous system. The external urethral sphincter is under conscious control. During bladder filling the rate of firing of nerve impulses to the detrusor muscle is low and receptive relaxation occurs. At higher volumes and increased intra vesical pressures the rate of neuronal firing will increase and eventually voiding will occur.
A 43 year old lady undergoes a day case laparoscopic cholecystectomy. The operation is more difficult than anticipated and a drain is placed to the operative site. Whilst in recovery, the patient loses 1800ml of frank blood into the drain. Which of the following will not occur?
Release of aldosterone via the Bainbridge reflex
Reduced urinary sodium excretion
Increase in sympathetic discharge to ventricular muscle
Fall in parasympathetic discharge to the sino atrial node
Decreased stimulation from atrial pressure receptors
The Bainbridge reflex is the increase in heart rate mediated via atrial stretch receptors that occurs following a rapid infusion of blood. Note the question asks which will not occur and that is why it is A and not the other options.
Cardiac physiology
- The heart has four chambers ejecting blood into both low pressure and high pressure systems.
The pumps generate pressures of between 0-25mmHg on the right side and 0-120 mmHg on the left.
At rest diastole comprises 2/3 of the cardiac cycle.
The product of the frequency of heart rate and stroke volume combine to give the cardiac output which is typically 5-6L per minute.
Detailed descriptions of the various waveforms are often not a feature of MRCS A (although they are on the syllabus). However, they are a very popular topic for surgical physiology in the MRCS B exam.
Electrical properties
Intrinsic myogenic rhythm within cardiac myocytes means that even the denervated heart is capable of contraction.
In the normal situation the cardiac impulse is generated in the sino atrial node in the right atrium and conveyed to the ventricles via the atrioventricular node.
The sino atrial node is also capable of spontaneous discharge and in the absence of background vagal tone will typically discharge around 100x per minute. Hence the higher resting heart rate found in cardiac transplant cases. In the SA and AV nodes the resting membrane potential is lower than in surrounding cardiac cells and will slowly depolarise from -70mV to around -50mV at which point an action potential is generated.
Differences in the depolarisation slopes between SA and AV nodes help to explain why the SA node will depolarise first. The cells have a refractory period during which they cannot be re-stimulated and this period allows for adequate ventricular filling. In pathological tachycardic states this time period is overridden and inadequate ventricular filling may then occur, cardiac output falls and syncope may ensue.
Parasympathetic fibres project to the heart via the vagus and will release acetylcholine. Sympathetic fibres release nor adrenaline and circulating adrenaline comes from the adrenal medulla. Noradrenaline binds to β 1 receptors in the SA node and increases the rate of pacemaker potential depolarisation.
Mid diastole: AV valves open. Ventricles hold 80% of final volume. Outflow valves shut. Aortic pressure is high.
Late diastole: Atria contract. Ventricles receive 20% to complete filling. Typical end diastolic volume 130-160ml.
Early systole: AV valves shut. Ventricular pressure rises. Isovolumetric ventricular contraction. AV Valves bulge into atria (c-wave). Aortic and pulmonary pressure exceeded- blood is ejected. Shortening of ventricles pulls atria downwards and drops intra atrial pressure (x-descent).
Late systole: Ventricular muscles relax and ventricular pressures drop. Although ventricular pressure drops the aortic pressure remains constant owing to peripheral vascular resistance and elastic property of the aorta. Brief period of retrograde flow that occurs in aortic recoil shuts the aortic valve. Ventricles will contain 60ml end systolic volume. The average stroke volume is 70ml (i.e. Volume ejected).
Early diastole: All valves are closed. Isovolumetric ventricular relaxation occurs. Pressure wave associated with closure of the aortic valve increases aortic pressure. The pressure dip before this rise can be seen on arterial waveforms and is called the incisura. During systole the atrial pressure increases such that it is now above zero (v- wave). Eventually atrial pressure exceed ventricular pressure and AV valves open - atria empty passively into ventricles and atrial pressure falls (y -descent )
The negative atrial pressures are of clinical importance as they can allow air embolization to occur if the neck veins are exposed to air. This patient positioning is important in head and neck surgery to avoid this occurrence if veins are inadvertently cut, or during CVP line insertion.
Mechanical properties
Preload = end diastolic volume
Afterload = aortic pressure
It is important to understand the principles of Laplace’s law in surgery.
It states that for hollow organs with a circular cross section, the total circumferential wall tension depends upon the circumference of the wall, multiplied by the thickness of the wall and on the wall tension.
The total luminal pressure depends upon the cross sectional area of the lumen and the transmural pressure. Transmural pressure is the internal pressure minus external pressure and at equilibrium the total pressure must counterbalance each other.
In terms of cardiac physiology the law explains that the rise in ventricular pressure that occurs during the ejection phase is due to physical change in heart size. It also explains why a dilated diseased heart will have impaired systolic function.
Starlings law
Increase in end diastolic volume will produce larger stroke volume.
This occurs up to a point beyond which cardiac fibres are excessively stretched and stroke volume will fall once more. It is important for the regulation of cardiac output in cardiac transplant patients who need to increase their cardiac output.
Baroreceptor reflexes
Baroreceptors located in aortic arch and carotid sinus.
Aortic baroreceptor impulses travel via the vagus and from the carotid via the glossopharyngeal nerve.
They are stimulated by arterial stretch.
Even at normal blood pressures they are tonically active.
Increase in baroreceptor discharge causes:
*Increased parasympathetic discharge to the SA node.
*Decreased sympathetic discharge to ventricular muscle causing decreased contractility and fall in stroke volume.
*Decreased sympathetic discharge to venous system causing increased compliance.
*Decreased peripheral arterial vascular resistance
Atrial stretch receptors
Located in atria at junction between pulmonary veins and vena cava.
Stimulated by atrial stretch and are thus low pressure sensors.
Increased blood volume will cause increased parasympathetic activity.
Very rapid infusion of blood will result in increase in heart rate mediated via atrial receptors: the Bainbridge reflex.
Decreases in receptor stimulation results in increased sympathetic activity this will decrease renal blood flow-decreases GFR-decreases urinary sodium excretion-renin secretion by juxtaglomerular apparatus-Increase in angiotensin II.
Increased atrial stretch will also result in increased release of atrial natriuretic peptide.
A 22 year old man is involved in a fight outside a nightclub. He is stabbed in the back, on the left side, approximately 3cm below the 12th rib in the mid scapular line. The structure most likely to be injured first as a result is the:
Spleen
Left kidney
Left adrenal gland
Left ureter
None of the above
The left kidney lies in this location and is the most likely structure to be injured. The Spleen lies more superiorly, and the left adrenal and ureter are unlikely to be injured in isolation.
Levels
Transpyloric plane
Level of the body of L1
Pylorus stomach
Left kidney hilum (L1- left one!)
Fundus of the gallbladder
Neck of pancreas
Duodenojejunal flexure
Superior mesenteric artery
Portal vein
Left and right colic flexure
Root of the transverse mesocolon
2nd part of the duodenum
Upper part of conus medullaris
Spleen
Can be identified by asking the supine patient to sit up without using their arms. The plane is located where the lateral border of the rectus muscle crosses the costal margin.
Anatomical planes
Subcostal plane Lowest margin of 10th costal cartilage
Intercristal plane Level of body L4 (highest point of iliac crest)
Intertubercular plane Level of body L5
Common level landmarks
Inferior mesenteric artery L3
Bifurcation of aorta into common iliac arteries L4
Formation of IVC L5 (union of common iliac veins)
Diaphragm apertures
Vena cava T8
Oesophagus T10
Aortic hiatus T12
Which of the following represents the root values of the sciatic nerve?
L4 to S3
L1 to L4
L3 to S1
S1 to S4
L5 to S1
The sciatic nerve most commonly arises from L4 to S3.
Sciatic nerve
The sciatic nerve is formed from the sacral plexus and is the largest nerve in the body. It is the continuation of the main part of the plexus arising from ventral rami of L4 to S3. These rami converge at the inferior border of piriformis to form the nerve itself. It passes through the inferior part of the greater sciatic foramen and emerges beneath piriformis. Medially, lie the inferior gluteal nerve and vessels and the pudendal nerve and vessels. It runs inferolaterally under the cover of gluteus maximus midway between the greater trochanter and ischial tuberosity. It receives its blood supply from the inferior gluteal artery. The nerve provides cutaneous sensation to the skin of the foot and the leg. It also innervates the posterior thigh muscles and the lower leg and foot muscles. The nerve splits into the tibial and common peroneal nerves approximately half way down the posterior thigh. The tibial nerve supplies the flexor muscles and the common peroneal nerve supplies the extensor muscles and the evertor muscles of the foot.
Summary points
Origin Spinal nerves L4 - S3
Articular Branches Hip joint
Muscular branches in upper leg
Semitendinosus
Semimembranosus
Biceps femoris
Part of adductor magnus
Cutaneous sensation
Posterior aspect of thigh (via cutaneous nerves)
Gluteal region
Entire lower leg (except the medial aspect)
Terminates At the upper part of the popliteal fossa by dividing into the tibial and peroneal nerves
The nerve to the short head of the biceps femoris comes from the common peroneal part of the sciatic and the other muscular branches arise from the tibial portion.
The tibial nerve goes on to innervate all muscles of the foot except the extensor digitorum brevis (which is innervated by the common peroneal nerve).
Which of the structures listed below is not a content of the carotid sheath?
Internal jugular vein
Internal carotid artery
Vagus nerve
Recurrent laryngeal nerve
Common carotid artery
Contents of carotid sheath:
Common carotid artery
Internal carotid artery
Internal jugular vein
Vagus nerve
Common carotid artery
The right common carotid artery arises at the bifurcation of the brachiocephalic trunk, the left common carotid arises from the arch of the aorta. Both terminate at the level of the upper border of the thyroid cartilage (the lower border of the third cervical vertebra) by dividing into the internal and external carotid arteries.
Left common carotid artery
This vessel arises immediately to the left and slightly behind the origin of the brachiocephalic trunk. Its thoracic portion is 2.5- 3.5 cm in length and runs superolaterally to the sternoclavicular joint.
In the thorax
The vessel is in contact, from below upwards, with the trachea, left recurrent laryngeal nerve, left margin of the oesophagus. Anteriorly the left brachiocephalic vein runs across the artery, and the cardiac branches from the left vagus descend in front of it. These structures together with the thymus and the anterior margins of the left lung and pleura separate the artery from the manubrium.
In the neck
The artery runs superiorly deep to sternocleidomastoid and then enters the anterior triangle. At this point it lies within the carotid sheath with the vagus nerve and the internal jugular vein. Posteriorly the sympathetic trunk lies between the vessel and the prevertebral fascia. At the level of C7 the vertebral artery and thoracic duct lie behind it. The anterior tubercle of C6 transverse process is prominent and the artery can be compressed against this structure (it corresponds to the level of the cricoid).
Anteriorly at C6 the omohyoid muscle passes superficial to the artery.
Within the carotid sheath the jugular vein lies lateral to the artery.
Right common carotid artery
The right common carotid arises from the brachiocephalic artery. The right common carotid artery corresponds with the cervical portion of the left common carotid, except that there is no thoracic duct on the right. The oesophagus is less closely related to the right carotid than the left.
Summary points about the carotid anatomy
Path
Passes behind the sternoclavicular joint (12% patients above this level) to the upper border of the thyroid cartilage, to divide into the external (ECA) and internal carotid arteries (ICA).
Relations
Level of 6th cervical vertebra crossed by omohyoid
Then passes deep to the thyrohyoid, sternohyoid, sternomastoid muscles.
Passes anterior to the carotid tubercle (transverse process 6th cervical vertebra)-NB compression here stops haemorrhage.
The inferior thyroid artery passes posterior to the common carotid artery.
Then : Left common carotid artery crosses the thoracic duct, Right common carotid artery crossed by recurrent laryngeal nerve
Which of the following statements relating to necrotising enterocolitis is false?
It has a mortality of 30%
Most frequently presents in premature neonates less than 32 weeks gestation.
Should be managed by early laparotomy and segmental resections in most cases.
Pneumostosis intestinalis may be visible on plain abdominal x-ray.
May be minimised by use of breast milk over formula feeds.
Most cases will settle with conservative management with NG decompression and appropriate support. Laparotomy should be undertaken in patients do not who progress despite conservative management or in whom compelling indications for surgery exist (eg free air).
Paediatric Gastrointestinal disorders
Pyloric stenosis
M>F
5-10% Family history in parents
Projectile non bile stained vomiting at 4-6 weeks of life
Diagnosis is made by test feed or USS
Treatment: Ramstedt pyloromyotomy (open or laparoscopic)
Acute appendicitis
Uncommon under 3 years
When occurs may present atypically
Mesenteric adenitis
Central abdominal pain and URTI
Conservative management
Intussusception
Telescoping bowel
Proximal to or at the level of, ileocaecal valve
6-9 months age
Colicky pain, diarrhoea and vomiting, sausage shaped mass, red jelly stool.
Treatment: reduction with air insufflation
Malrotation
High caecum at the midline
Feature in exomphalos, congenital diaphragmatic hernia, intrinsic duodenal atresia
May be complicated by development of volvulus, infant with volvulus may have bile stained vomiting
Diagnosis is made by upper GI contrast study and USS
Treatment is by laparotomy, if volvulus is present (or at high risk of occurring then a ladds procedure is performed
Hirschsprung’s disease
Absence of ganglion cells from myenteric and submucosal plexuses
Occurs in 1/5000 births
Full thickness rectal biopsy for diagnosis
Delayed passage of meconium and abdominal distension
Treatment is with rectal washouts initially, thereafter an anorectal pull through procedure
Oesophageal atresia
Associated with tracheo-oesophageal fistula and polyhydramnios
May present with choking and cyanotic spells following aspiration
VACTERL associations
Meconium ileus
Usually delayed passage of meconium and abdominal distension
Majority have cystic fibrosis
X-Rays may not show a fluid level as the meconium is viscid (depends upon feeding), PR contrast studies may dislodge meconium plugs and be therapeutic
Infants who do not respond to PR contrast and NG N-acetyl cysteine will require surgery to remove the plugs
Biliary atresia
Jaundice > 14 days
Increased conjugated bilirubin
Urgent Kasai procedure
Necrotising enterocolitis
Prematurity is the main risk factor
Early features include abdominal distension and passage of bloody stools
X-Rays may show pneumatosis intestinalis and evidence of free air
Increased risk when empirical antibiotics are given to infants beyond 5 days
Treatment is with total gut rest and TPN, babies with perforations will require laparotomy
A patient has an arterial blood gas sample which provides the following result:
pH 7.20
pO2 7.5
Bicarbonate 22
pCO2 8.1
Chloride 10meq
What is the most likely cause?
Type II respiratory failure
Metabolic acidosis with increased anion gap
Metabolic alkalosis
Type I respiratory failure
Respiratory alkalosis
Type II respiratory failure
This is a sign of acute type 2 respiratory failure (non compensated). This is the result of carbon dioxide retention.
Metabolic acidosis
This is the most common surgical acid - base disorder.
Reduction in plasma bicarbonate levels.
Two mechanisms:
1. Gain of strong acid (e.g. diabetic ketoacidosis)
2. Loss of base (e.g. from bowel in diarrhoea)
- Classified according to the anion gap, this can be calculated by:
(Na+ + K+) - (Cl- + HCO3-).
- If a question supplies the chloride level then this is often a clue that the anion gap should be calculated. The normal range = 10-18 mmol/L
Normal anion gap ( = hyperchloraemic metabolic acidosis)
Gastrointestinal bicarbonate loss: diarrhoea, ureterosigmoidostomy, fistula
Renal tubular acidosis
Drugs: e.g. acetazolamide
Ammonium chloride injection
Addison’s disease
Raised anion gap
Lactate: shock, hypoxia
Ketones: diabetic ketoacidosis, alcohol
Urate: renal failure
Acid poisoning: salicylates, methanol
Metabolic acidosis secondary to high lactate levels may be subdivided into two types:
Lactic acidosis type A: (Perfusion disorders e.g.shock, hypoxia, burns)
Lactic acidosis type B: (Metabolic e.g. metformin toxicity)
Metabolic alkalosis
Usually caused by a rise in plasma bicarbonate levels.
Rise of bicarbonate above 24 mmol/L will typically result in renal excretion of excess bicarbonate.
Caused by a loss of hydrogen ions or a gain of bicarbonate. It is due mainly to problems of the kidney or gastrointestinal tract
Causes
Vomiting / aspiration (e.g. Peptic ulcer leading to pyloric stenosis, nasogastric suction)
Diuretics
Liquorice, carbenoxolone
Hypokalaemia
Primary hyperaldosteronism
Cushing’s syndrome
Bartter’s syndrome
Congenital adrenal hyperplasia
Mechanism of metabolic alkalosis
Activation of renin-angiotensin II-aldosterone (RAA) system is a key factor
Aldosterone causes reabsorption of Na+ in exchange for H+ in the distal convoluted tubule
ECF depletion (vomiting, diuretics) → Na+ and Cl- loss → activation of RAA system → raised aldosterone levels
In hypokalaemia, K+ shift from cells → ECF, alkalosis is caused by shift of H+ into cells to maintain neutrality
Respiratory acidosis
Rise in carbon dioxide levels usually as a result of alveolar hypoventilation
Renal compensation may occur leading to Compensated respiratory acidosis
Causes
COPD
Decompensation in other respiratory conditions e.g. Life-threatening asthma / pulmonary oedema
Sedative drugs: benzodiazepines, opiate overdose
Respiratory alkalosis
Hyperventilation resulting in excess loss of carbon dioxide
This will result in increasing pH
Causes
Psychogenic: anxiety leading to hyperventilation
Hypoxia causing a subsequent hyperventilation: pulmonary embolism, high altitude
Early salicylate poisoning*
CNS stimulation: stroke, subarachnoid haemorrhage, encephalitis
Pregnancy
*Salicylate overdose leads to a mixed respiratory alkalosis and metabolic acidosis. Early stimulation of the respiratory centre leads to a respiratory alkalosis whilst later the direct acid effects of salicylates (combined with acute renal failure) may lead to an acidosis
What is the most common cause of hypercalcaemia in the UK in hospitalised patients?
Thiazide use
Metastatic malignancy
Primary hyperparathyroidism
Osteogenic sarcoma
Sarcoidosis
Metastatic cancer accounts for most cases of hypercalcaemia in hospitalised patients. In the community primary hyperparathyroidism is the commonest cause.
Hypercalcaemia
Main causes
Malignancy (most common cause in hospital in-patients)
Primary hyperparathyroidism (commonest cause in non hospitalised patients)
Less common
Sarcoidosis (extrarenal synthesis of calcitriol )
Thiazides, lithium
Immobilisation
Pagets disease
Vitamin A/D toxicity
Thyrotoxicosis
MEN
Milk alkali syndrome
Clinical features
Stones, bones, abdominal groans, and psychic moans
High serum calcium levels result in decreased neuronal excitability. Therefore sluggish reflexes, muscle weakness and constipation may occur.
Which of the following features does not put a patient at risk of refeeding syndrome?
BMI < 16 kg/m
Alcohol abuse
Thyrotoxicosis
Chemotherapy
Diuretics
Thyrotoxicosis
Diuretics increase the risk of re-feeding syndrome through a process of increasing the risk of depletion of key electrolytes.
Nutrition - Refeeding syndrome
Refeeding syndrome describes the metabolic abnormalities which occur on feeding a person following a period of starvation. The metabolic consequences include:
Hypophosphataemia
Hypokalaemia
Hypomagnesaemia
Abnormal fluid balance
These abnormalities can lead to organ failure.
Re-feeding problems
If patient not eaten for > 5 days, aim to re-feed at < 50% energy and protein levels
High risk for re-feeding problems
If one or more of the following:
BMI < 16 kg/m2
Unintentional weight loss >15% over 3-6 months
Little nutritional intake > 10 days
Hypokalaemia, Hypophosphataemia or hypomagnesaemia prior to feeding (unless high)
If two or more of the following:
BMI < 18.5 kg/m2
Unintentional weight loss > 10% over 3-6 months
Little nutritional intake > 5 days
History of: alcohol abuse, drug therapy including insulin, chemotherapy, diuretics and antacids
Prescription
Start at up to 10 kcal/kg/day increasing to full needs over 4-7 days
Start immediately before and during feeding: oral thiamine 200-300mg/day, vitamin B co strong 1 tds and supplements
Give K+ (2-4 mmol/kg/day), phosphate (0.3-0.6 mmol/kg/day), magnesium (0.2-0.4 mmol/kg/day)
A 6 year old child presents with colicky abdominal pain, vomiting and the passage of red current jelly stool per rectum. On examination, the child has a tender abdomen and a palpable mass in the right upper quadrant. Imaging shows an intussusception. Which of the conditions below is least recognised as a precipitant?
Inflammation of Peyers patches
Cystic fibrosis
Meckels diverticulum
Mesenteric cyst
Mucosal polyps
Mesenteric cysts may be associated with intra abdominal catastrophes where these occur they are typically either intestinal volvulus or intestinal infarction. They seldom cause intussusception. Cystic fibrosis may lead to the formation of meconium ileus equivalent and plugs may occasionally serve as the lead points for an intussusception.
Intussusception- Paediatric
Intussusception typically presents with colicky abdominal pain and vomiting. The telescoping of the bowel produces mucosal ischaemia and bleeding may occur resulting in the passage of red current jelly stools. Recognised causes include lumenal pathologies such as polyps, lymphadenopathy and diseases such as cystic fibrosis. Idiopathic intussceception of the ileocaecal valve and terminal ileum is the most common variant and typically affects young children and toddlers.
The diagnosis is usually made by abdominal ultrasound investigation. The decision as to the optimal treatment is dictated by the patients physiological status and abdominal signs. In general, children who are unstable with localising peritoneal signs should undergo laparotomy, as should those in whom attempted radiological reduction has failed.
In relatively well children, without localising signs, attempted pneumatic reduction under fluroscopic guidance is the usual treatment. This is not done where there are concerns about impending perforation or if there is a small bowel intussusception.
Which of the following is not an extraintestinal feature of Crohns disease?
Iritis
Clubbing
Aphthous ulcers
Erythema multiforme
Pyoderma gangrenosum
Extraintestinal manifestation of inflammatory bowel disease: A PIE SAC
Aphthous ulcers
Pyoderma gangrenosum
Iritis
Erythema nodosum
Sclerosing cholangitis
Arthritis
Clubbing
Crohns disease
Crohns disease is a chronic transmural inflammation of a segment(s) of the gastrointestinal tract and may be associated with extra intestinal manifestations. Frequent disease patterns observed include ileal, ileocolic and colonic disease. Peri-anal disease may occur in association with any of these. The disease is often discontinuous in its distribution. Inflammation may cause ulceration, fissures, fistulas and fibrosis with stricturing. Histology reveals a chronic inflammatory infiltrate that is usually patchy and transmural.
Ulcerative colitis Vs Crohns
Crohn’s disease Ulcerative colitis
Distribution Mouth to anus Rectum and colon
Macroscopic changes Cobblestone appearance, apthoid ulceration Contact bleeding
Depth of disease Transmural inflammation Superficial inflammation
Distribution pattern Patchy Continuous
Histological features Granulomas (non caseating epithelioid cell aggregates with Langhans’ giant cells) Crypt abscesses, Inflammatory cells in the lamina propria
Extraintestinal manifestations of Crohns
Related to disease extent Unrelated to disease extent
Aphthous ulcers (10%) Sacroiliiitis (10-15%)
Erythema nodosum (5-10%) Ankylosing spondylitis (1-2%)
Pyoderma gangrenosum (0.5%) Primary sclerosing cholangitis (Rare)
Acute arthropathy (6-12%) Gallstones (up to 30%)
Ocular complications (up to 10%) Renal calculi (up to 10%)
Diarrhoea in Crohns
Diarrhoea in Crohns may be multifactorial since actual inflammation of the colon is not common. Causes therefore include the following:
Bile salt diarrhoea secondary to terminal ileal disease
Entero-colic fistula
Short bowel due to multiple resections
Bacterial overgrowth
Surgical interventions in Crohns disease
The commonest disease pattern in Crohns is stricturing terminal ileal disease and this often culminates in an ileocaecal resection. Other procedures performed include segmental small bowel resections and stricturoplasty. Colonic involvement in patients with Crohns is not common and, where found, distribution is often segmental. However, despite this distribution segmental resections of the colon in patients with Crohns disease are generally not advocated because the recurrence rate in the remaining colon is extremely high. As a result, the standard options of colonic surgery in Crohns patients are generally; sub total colectomy, panproctocolectomy and staged sub total colectomy and proctectomy. Restorative procedures such as ileoanal pouch have no role in therapy.
Crohns disease is notorious for the developmental of intestinal fistulae; these may form between the rectum and skin (peri anal) or the small bowel and skin. Fistulation between loops of bowel may also occur and result in bacterial overgrowth and malabsorption. Management of enterocutaneous fistulae involves controlling sepsis, optimising nutrition, imaging the disease and planning definitive surgical management.
A 56 year old man presents with urinary symptoms and on investigation is found to have a 2cm nodule in the left lobe of the prostate. Imaging with whole body MRI and pelvic CT/MRI demonstrates a likely cancer with no distant disease and no nodal metastasis. What is the most appropriate course of action?
Robotic prostatectomy
Open prostatectomy
Transvesical prostatectomy
Prostatic biopsy
Radical pelvic radiotherapy
Prostate cancers are typically biopsied prior to treatment.
Prostate Cancer
Prostate Cancer
This is a common condition and up to 30,000 men are diagnosed with the condition each year. Up to 9,000 will die in in the UK from the condition per year.
Diagnosis
Early prostate cancers have few symptoms.
Metastatic disease may present as bone pain.
Locally advanced disease may present as pelvic pain or with urinary symptoms.
Prostate specific antigen measurement
Digital rectal examination
Trans rectal USS (+/- biopsy)
MRI/ CT and bone scan for staging.
PSA Test
The normal upper limit for PSA is 4ng/ml. However, in this group will lie patients with benign disease and some with localised prostate cancer. False positives may be due to prostatitis, UTI, BPH, vigorous DRE.
The percentage of free: total PSA may help to distinguish benign disease from cancer. Values of <20% are suggestive of cancer and biopsy is advised.
Pathology
95% adenocarcinoma
In situ malignancy is sometimes found in areas adjacent to cancer. Multiple biopsies needed to call true in situ disease.
Often multifocal- 70% lie in the peripheral zone.
Graded using the Gleason grading system, two grades awarded 1 for most dominant grade (on scale of 1-5) and 2 for second most dominant grade (scale 1-5). The two added together give the Gleason score. Where 2 is best prognosis and 10 the worst.
Lymphatic spread occurs first to the obturator nodes and local extra prostatic spread to the seminal vesicles is associated with distant disease.
Treatment
Watch and wait- Elderly, multiple co-morbidities, low Gleason score
Radiotherapy (External)- Both potentially curative and palliative therapy possible. However, radiation proctitis and rectal malignancy are late problems. Brachytherapy is a modification allowing internal radiotherapy.
Surgery- Radical prostatectomy. Surgical removal of the prostate is the standard treatment for localised disease. The robot is being used increasingly for this procedure. As well as the prostate the obturator nodes are also removed to complement the staging process. Erectile dysfunction is a common side effect. Survival may be better than with radiotherapy (see references). Functional outcomes are better when a robotic approach is used.
Hormonal therapy- Testosterone stimulates prostate tissue and prostatic cancers usually show some degree of testosterone dependence. 95% of testosterone is derived from the testis and bilateral orchidectomy may be used for this reason. Pharmacological alternatives include LHRH analogues and anti androgens (which may be given in combination).
In the UK the National Institute for Clinical Excellence (NICE) suggests that active surveillance is the preferred option for low risk men. It is particularly suitable for men with clinical stage T1c, Gleason score 3+3 and PSA density < 0.15 ng/ml/ml who have cancer in less than 50% of their biopsy cores, with < 10 mm of any core involved.
Candidates for active surveillance should:
have had at least 10 biopsy cores taken
have at least one re-biopsy.
If men on active surveillance show evidence of disease progression, offer radical treatment. Treatment decisions should be made with the man, taking into account co-morbidities and life expectancy.
References
1. Prostate cancer pathway. NICE.(http://guidance.nice.org.uk/IPG424)
2. Sooriakumaran P et al. Comparative effectiveness of radical prostatectomy and radiotherapy in prostate cancer: observational study of mortality outcomes. BMJ 2014 (348):13. This study shows that in men with localised disease survival was greater in those offered surgery.
A 63 year old man has a history of claudication that has been present for many years. He is recently evaluated in the clinic and a duplex scan shows that he has an 85% stenosis of the superficial femoral artery. Two weeks later he presents with a 1 hour history of severe pain in his leg. On examination, he has absent pulses in the affected limb and it is much cooler than the contra-lateral limb. Which process best accounts for this presentation?
Thrombosis
Embolus
Atheroma growth
Sub intimal dissection
Anaemia
In an existing lesion a complication such as thrombosis is more likely than embolus. These patients should receive heparin and imaging with duplex scanning. Whilst an early surgical bypass or intra-arterial thrombolysis may be indicated, an embolectomy should not generally be performed as the lesion is not an embolus and the operation therefore ineffective.
Claudication
Claudication is a condition in which patients develop pain in a limb during periods of exercise. The underlying disorder is usually that of arterial insufficiency. Atheroma develops in the arterial wall and once this occludes >50-75% of the lumenal diameter the supply to metabolising tissues distally may become compromised. The typical claudicant complains of calf pain that is worse on exercise and relieved by rest. This typical description assumes that the SFA is the site of disease, more proximal disease may present with other symptoms such as buttock claudication and impotence.
The history is usually a progressive one, patients presenting as an emergency with severe pain, diminished sensation, pallor and absent pulses have critical limb ischaemia. This may complicate claudication and usually indicates a plaque related complication, such as thrombosis.
Risk factors
Risk factors for claudication include smoking, diabetes and hyperlipidaemia.
Diagnosis
Diagnostic work -up includes measurement of ankle- brachial pressure indices, duplex scanning and formal angiography.
Treatment
Those with long claudication distances, no ulceration or gangrene may be managed conservatively. Patients with rest pain, ulceration or gangrene will almost always require intervention. All patients should receive an antiplatelet agent and a statin unless there are compelling contra-indications.
A 17 year old male is admitted with lower abdominal discomfort. He has been suffering from intermittent right iliac fossa pain for the past few months. His past medical history includes a negative colonoscopy and gastroscopy undertaken as investigation for iron deficiency anaemia. The pain is worse after meals. Inflammatory markers are normal. What is the most likely cause?
Appendicitis
Crohns disease
Peptic ulcer disease
Meckels diverticulum
Irritable bowel syndrome
This scenario should raise suspicion for Meckels as these may contain ectopic gastric mucosa which may secrete acid with subsequent bleeding and ulceration. The iron deficiency anaemia makes a Meckels more likely than IBS.
Right iliac fossa pain
Differential diagnosis
Appendicitis
Pain radiating to right iliac fossa
Anorexia (very common)
Short history
Diarrhoea and profuse vomiting rare
Crohn’s disease
Often long history
Signs of malnutrition
Change in bowel habit, especially diarrhoea
Mesenteric adenitis
Mainly affects children
Causes include Adenoviruses, Epstein Barr Virus, beta-haemolytic Streptococcus, Staphylococcus spp., Escherichia coli, Streptococcus viridans and Yersinia spp.
Patients have a higher temperature than those with appendicitis
If laparotomy is performed, enlarged mesenteric lymph nodes will be present
Diverticulitis
Both left and right sided disease may present with right iliac fossa pain
Clinical history may be similar, although some change in bowel habit is usual
When suspected, a CT scan may help in refining the diagnosis
Meckel’s diverticulitis
A Meckel’s diverticulum is a congenital abnormality that is present in about 2% of the population
Typically 2 feet proximal to the ileocaecal valve
May be lined by ectopic gastric mucosal tissue and produce bleeding
Perforated peptic ulcer
This usually produces upper quadrant pain but pain may be lower
Perforations typically have a sharp sudden onset of pain in the history
Incarcerated right inguinal or femoral hernia
Usually only right iliac fossa pain if right sided or bowel obstruction.
Bowel perforation secondary to caecal or colon carcinoma
Seldom localised to right iliac fossa, although complete large bowel obstruction with caecal distension may cause pain prior to perforation.
Gynaecological causes
Pelvic inflammatory disease/salpingitis/pelvic abscess/Ectopic pregnancy/Ovarian torsion/Threatened or complete abortion/Mittelschmerz
Urological causes
Ureteric colic/UTI/Testicular torsion
Other causes
TB/Typhoid/Herpes Zoster/AAA/Situs inversus
A 25 year old man is found to have carcinoid syndrome. Which of the following hormones is released by carcinoids?
Serotonin
Dopamine
Nor adrenaline
Adrenaline
Aldosterone
Rule of thirds:
1/3 multiple
1/3 small bowel
1/3 metastasize
1/3 second tumour
Carcinoids secrete serotonin. Carcinoid syndrome will only occur in the presence of liver metastasis as the hormone released from primary lesions will typically be metabolised by the liver.
Carcinoid syndrome
Carcinoid tumours secrete serotonin
Originate in neuroendocrine cells mainly in the intestine (midgut-distal ileum/appendix)
Can occur in the rectum, bronchi
Hormonal symptoms mainly occur when disease spreads outside the bowel
Clinical features
Onset: insidious over many years
Flushing face
Palpitations
Pulmonary valve stenosis and tricuspid regurgitation causing dyspnoea
Asthma
Severe diarrhoea (secretory, persists despite fasting)
Investigation
5-HIAA in a 24-hour urine collection
Somatostatin receptor scintigraphy
CT scan
Blood testing for chromogranin A
Treatment
Octreotide
Surgical removal
A 59 year old lady is referred from the NHS breast screening program. A recent mammogram is reported as showing linear, branching microcalcification with coarse granules. Which disease process is the most likely underlying cause of these appearances?
Invasive lobular cancer
Lobular carcinoma in situ
Cribriform type ductal carcinoma in situ
Comedo type ductal carcinoma in situ
Fibroadenosis
Comedo type DCIS is usually associated with microcalcifications. Cribriform lesions are usually multifocal but less likely to form microcalcifications. Lobular cancers and in situ lesions rarely form microcalcifications and are difficult to detect using mammography.
Breast cancer - In situ disease
Breast cancer that has yet to invade the basement membrane is referred to as in situ disease. Both ductal and lobular in situ variants are recognised.
Ductal carcinoma in situ
Sub types include; comedo, cribriform, micropapillary and solid
Comdeo DCIS is most likely to form microcalcifications
Cribriform and micropapillary are most likely to be multifocal
Most lesions are mixed (composed of multiple subtypes)
High nuclear grade DCIS is associated with more malignant characteristics (loss of p53, increased erbB2 expression)
Local excision of low nuclear grade DCIS will usually produce satisfactory outcomes.
Multifocal lesions, large and high nuclear grade lesions will usually require mastectomy
Whole breast irradiation improves locoregional control when breast conserving surgery is performed
Lobular carcinoma in situ
Much rarer than DCIS
Does not form microcalcifications
Usually single growth pattern
When an invasive component is found it is less likely to be associated with axillary nodal metastasis than with invasive foci found within DCIS
Low grade LCIS is usually treated by monitoring rather than excision
Which of the following structures is not closely related to the carotid sheath?
Sternothyroid muscle
Sternohyoid muscle
Hypoglossal nerve
Superior belly of omohyoid muscle
Anterior belly of digastric muscle
At its lower end the carotid sheath is related to sternohyoid and sternothyroid. Opposite the cricoid cartilage the sheath is crossed by the superior belly of omohyoid. Above this level the sheath is covered by the sternocleidomastoid muscle. Above the level of the hyoid the vessels pass deep to the posterior belly of digastric and stylohyoid. Opposite the hyoid bone the sheath is crossed obliquely by the hypoglossal nerve.
Common carotid artery
The right common carotid artery arises at the bifurcation of the brachiocephalic trunk, the left common carotid arises from the arch of the aorta. Both terminate at the level of the upper border of the thyroid cartilage (the lower border of the third cervical vertebra) by dividing into the internal and external carotid arteries.
Left common carotid artery
This vessel arises immediately to the left and slightly behind the origin of the brachiocephalic trunk. Its thoracic portion is 2.5- 3.5 cm in length and runs superolaterally to the sternoclavicular joint.
In the thorax
The vessel is in contact, from below upwards, with the trachea, left recurrent laryngeal nerve, left margin of the oesophagus. Anteriorly the left brachiocephalic vein runs across the artery, and the cardiac branches from the left vagus descend in front of it. These structures together with the thymus and the anterior margins of the left lung and pleura separate the artery from the manubrium.
In the neck
The artery runs superiorly deep to sternocleidomastoid and then enters the anterior triangle. At this point it lies within the carotid sheath with the vagus nerve and the internal jugular vein. Posteriorly the sympathetic trunk lies between the vessel and the prevertebral fascia. At the level of C7 the vertebral artery and thoracic duct lie behind it. The anterior tubercle of C6 transverse process is prominent and the artery can be compressed against this structure (it corresponds to the level of the cricoid).
Anteriorly at C6 the omohyoid muscle passes superficial to the artery.
Within the carotid sheath the jugular vein lies lateral to the artery.
Right common carotid artery
The right common carotid arises from the brachiocephalic artery. The right common carotid artery corresponds with the cervical portion of the left common carotid, except that there is no thoracic duct on the right. The oesophagus is less closely related to the right carotid than the left.
Summary points about the carotid anatomy
Path
Passes behind the sternoclavicular joint (12% patients above this level) to the upper border of the thyroid cartilage, to divide into the external (ECA) and internal carotid arteries (ICA).
Relations
Level of 6th cervical vertebra crossed by omohyoid
Then passes deep to the thyrohyoid, sternohyoid, sternomastoid muscles.
Passes anterior to the carotid tubercle (transverse process 6th cervical vertebra)-NB compression here stops haemorrhage.
The inferior thyroid artery passes posterior to the common carotid artery.
Then : Left common carotid artery crosses the thoracic duct, Right common carotid artery crossed by recurrent laryngeal nerve
A 70 year old man falls and fractures his scaphoid bone. The fracture is displaced and the decision is made to insert a screw to fix the fracture. Which of the following structures lies directly medial to the scaphoid?
Lunate
Pisiform
Trapezoid
Trapezium
None of the above
The lunate lies medially in the anatomical plane. Fractures of the scaphoid that are associated with high velocity injuries may cause associated lunate dislocation.
Scaphoid bone
The scaphoid has a concave articular surface for the head of the capitate and at the edge of this is a crescentic surface for the corresponding area on the lunate.
Proximally, it has a wide convex articular surface with the radius. It has a distally sited tubercle that can be palpated. The remaining articular surface is to the lateral side of the tubercle. It faces laterally and is associated with the trapezium and trapezoid bones.
The narrow strip between the radial and trapezial surfaces and the tubercle gives rise to the radial collateral carpal ligament.
Blood supply
This is from the scaphoid tubercle vessels that comprise 20% and the dorsal ridge vessels that supply 80%.
Which of the following forms the medial wall of the femoral canal?
Pectineal ligament
Adductor longus
Sartorius
Lacunar ligament
Inguinal ligament
Lacunar ligament
The femoral canal and the femoral triangle are distinct anatomical structures. Do not confuse them, especially in the time pressured exam situation.
Femoral canal
The femoral canal lies at the medial aspect of the femoral sheath. The femoral sheath is a fascial tunnel containing both the femoral artery laterally and femoral vein medially. The canal lies medial to the vein.
Borders of the femoral canal
Laterally Femoral vein
Medially Lacunar ligament
Anteriorly Inguinal ligament
Posteriorly Pectineal ligament Contents
Lymphatic vessels
Cloquet’s lymph node
Physiological significance
Allows the femoral vein to expand to allow for increased venous return from the lower limbs.
Pathological significance
As a potential space, it is the site of femoral hernias. The relatively tight neck places these at high risk of strangulation.
An 18 year old male presents to casualty with a depressed skull fracture. This is managed surgically. Over the next few days he complains of double vision on walking down stairs and reading. On testing ocular convergence, the left eye faces downwards and medially, but the right side does not do so. Which of the nerves listed below is most likely to be responsible?
Facial
Oculomotor
Abducens
Trochlear
Trigeminal nerve
The trochlear nerve has a relatively long intracranial course and this makes it vulnerable to injury in head trauma. Head trauma is the commonest cause of an acute fourth nerve palsy. A 4th nerve palsy is the commonest cause of a vertical diplopia. The diplopia is at its worst when the eye looks medially which it usually does as part of the accommodation reflex when walking down stairs.
Disorders of the oculomotor system
Nerve Path Nerve palsy features
Oculomotor nerve
Large nucleus at the midbrain
Fibres pass through the red nucleus and the pyramidal tract; through the cavernous sinus into the orbit
Ptosis
Eye down and out
Unable to move the eye superiorly, inferiorly, medially
Pupil fixed and dilated
Trochlear nerve
Longest intracranial course
Only nerve to exit the dorsal aspect of brainstem
Nucleus at midbrain, passes between the posterior cerebral and superior cerebellar arteries, through the cavernous sinus into the orbit
Vertical diplopia (diplopia on descending the stairs)
Unable to look down and in
Abducens nerve Nucleus lies in the mid pons Convergence of eyes in primary position
Lateral diplopia towards side of lesion
Eye deviates medially
Which of the cranial nerves listed below is least likely to carry parasympathetic fibres?
III
VII
IX
X
II
Cranial nerves carrying parasympathetic fibres
X IX VII III (1973)
The parasympathetic functions served by the cranial nerves include:
III (oculomotor) Pupillary constriction and accommodation
VII (facial) Lacrimal gland, submandibular and sublingual glands
IX (glossopharyngeal) Parotid
X (vagus) Heart and abdominal viscera
The optic nerve carries no parasympathetic fibres.
The cranial preganglionic parasympathetic nerves arise from specific nuclei in the CNS. These synapse at one of four parasympathetic ganglia; otic, pterygopalatine, ciliary and submandibular. From these ganglia the parasympathetic nerves complete their journey to their target tissues via CN V (trigeminal) branches (ophthalmic nerve CNV branch 1, Maxillary nerve CN V branch2, mandibular nerve CN V branch 3)
Cranial nerves
Cranial nerve lesions
Olfactory nerve May be injured in basal skull fractures or involved in frontal lobe tumour extension. Loss of olfactory nerve function in relation to major CNS pathology is seldom an isolated event and thus it is poor localiser of CNS pathology.
Optic nerve Problems with visual acuity may result from intra ocular disorders. Problems with the blood supply such as amaurosis fugax may produce temporary visual distortion. More important surgically is the pupillary response to light. The pupillary size may be altered in a number of disorders. Nerves involved in the resizing of the pupil connect to the pretectal nucleus of the high midbrain, bypassing the lateral geniculate nucleus and the primary visual cortex. From the pretectal nucleus neurones pass to the Edinger - Westphal nucleus, motor axons from here pass along with the oculomotor nerve. They synapse with ciliary ganglion neurones; the parasympathetic axons from this then innervate the iris and produce miosis. The miotic pupil is seen in disorders such as Horner’s syndrome or opiate overdose.
Mydriasis is the dilatation of the pupil in response to disease, trauma, drugs (or the dark!). It is pathological when light fails to induce miosis. The radial muscle is innervated by the sympathetic nervous system. Because the parasympathetic fibres travel with the oculomotor nerve they will be damaged by lesions affecting this nerve (e.g. cranial trauma).
The response to light shone in one eye is usually a constriction of both pupils. This indicates intact direct and consensual light reflexes. When the optic nerve has an afferent defect the light shining on the affected eye will produce a diminished pupillary response in both eyes. Whereas light shone on the unaffected eye will produce a normal pupillary response in both eyes. This is referred to as the Marcus Gunn pupil and is seen in conditions such as optic neuritis. In a total CN II lesion shining the light in the affected eye will produce no response.
Oculomotor nerve The pupillary effects are described above. In addition it supplies all ocular muscles apart from lateral rectus and superior oblique. Thus the affected eye will be deviated inferolaterally. Levator palpebrae superioris may also be impaired resulting in impaired ability to open the eye.
Trochlear nerve The eye will not be able to look down.
Trigeminal nerve Largest cranial nerve. Exits the brainstem at the pons. Branches are ophthalmic, maxillary and mandibular. Only the mandibular branch has both sensory and motor fibres. Branches converge to form the trigeminal ganglion (located in Meckels cave). It supplies the muscles of mastication and also tensor veli palatine, mylohyoid, anterior belly of digastric and tensor tympani. The detailed descriptions of the various sensory functions are described in other areas of the website. The corneal reflex is important and is elicited by applying a small tip of cotton wool to the cornea, a reflex blink should occur if it is intact. It is mediated by: the naso ciliary branch of the ophthalmic branch of the trigeminal (sensory component) and the facial nerve producing the motor response. Lesions of the afferent arc will produce bilateral absent blink and lesions of the efferent arc will result in a unilateral absent blink.
Abducens nerve The affected eye will have a deficit of abduction. This cranial nerve exits the brainstem between the pons and medulla. It thus has a relatively long intra cranial course which renders it susceptible to damage in raised intra cranial pressure.
Facial nerve Emerges from brainstem between pons and medulla. It controls muscles of facial expression and taste from the anterior 2/3 of the tongue. The nerve passes into the petrous temporal bone and into the internal auditory meatus. It then passes through the facial canal and exits at the stylomastoid foramen. It passes through the parotid gland and divides at this point. It does not innervate the parotid gland. Its divisions are considered in other parts of the website. Its motor fibres innervate orbicularis oculi to produce the efferent arm of the corneal reflex. In surgical practice it may be injured during parotid gland surgery or invaded by malignancies of the gland and a lower motor neurone on the ipsilateral side will result.
Vestibulo-cochlear nerve Exits from the pons and then passes through the internal auditory meatus. It is implicated in sensorineural hearing loss. Individuals with sensorineural hearing loss will localise the sound in webers test to the normal ear. Rinnes test will be reduced on the affected side but should still work. These two tests will distinguish sensorineural hearing loss from conductive deafness. In the latter condition webers test will localise to the affected ear and Rinnes test will be impaired on the affected side. Surgical lesions affecting this nerve include CNS tumours and basal skull fractures. It may also be damaged by the administration of ototoxic drugs (of which gentamicin is the most commonly used in surgical practice).
Glossopharyngeal nerve Exits the pons just above the vagus. Receives sensory fibres from posterior 1/3 tongue, tonsils, pharynx and middle ear (otalgia may occur following tonsillectomy). It receives visceral afferents from the carotid bodies. It supplies parasympathetic fibres to the parotid gland via the otic ganglion and motor function to stylopharyngeaus muscle. The sensory function of the nerve is tested using the gag reflex.
Vagus nerve Leaves the medulla between the olivary nucleus and the inferior cerebellar peduncle. Passes through the jugular foramen and into the carotid sheath. Details of the functions of the vagus nerve are covered in the website under relevant organ sub headings.
Accessory nerve Exists from the caudal aspect of the brainstem (multiple branches) supplies trapezius and sternocleidomastoid muscles. The distal portion of this nerve is most prone to injury during surgical procedures.
Hypoglossal nerve Emerges from the medulla at the preolivary sulcus, passes through the hypoglossal canal. It lies on the carotid sheath and passes deep to the posterior belly of digastric to supply muscles of the tongue (except palatoglossus). Its location near the carotid sheath makes it vulnerable during carotid endarterectomy surgery and damage will produce ipsilateral defect in muscle function.
A 28 year old African man is admitted with acute severe abdominal pain. He has just flown into the UK long haul and the pain developed whilst in flight. On examination he is tender in the left upper quadrant. His blood tests are as shown.
Hb 6 g/dl
Reticulocyte count 15%.
Ultrasound shows a spleen with a heterogenous texture. The gallbladder shows a few small gallstones but is otherwise normal.
What is the most likely diagnosis?
Pancreatitis
Parvovirus infection
Sickle cell anaemia
Pulmonary embolism
Beta Thalassaemia minor
A combination of a high reticulocyte count and severe anaemia indicates sickle cell anaemia, however another differential can be of a transient aplastic crisis due to parvovirus. This is less likely as this causes a reticulocytopenia rather than a reticulocytosis.
Parvovirus B19 infects erythroid progenitor cells in the bone marrow and causes temporary cessation of red blood cell production, patients who have underlying hematologic abnormalities are at risk of cessation of red blood cell production if they become infected. This can result in a transient aplastic crisis. Thus, patients with sickle cell anaemia are at risk. Typically, these patients have a viral prodrome followed by anaemia, often with haemoglobin concentrations falling below 5.0 g/dL and reticulocytopenia.
Sickle cell anaemia
Autosomal recessive
Single base mutation
Deoxygenated cells become sickle in shape
Causes: short red cell survival, obstruction of microvessels and infarction
Sickling is precipitated by: dehydration, infection, hypoxia
Manifest at 6 months age
Africans, Middle East, Indian
Diagnosis: Hb electrophoresis
Sickle crises
Bone pain
Pleuritic chest pain: acute sickle chest syndrome commonest cause of death
CVA, seizures
Papillary necrosis
Splenic infarcts
Priapism
Hepatic pain
Hb does not fall during a crisis, unless there is
Aplasia: parvovirus
Acute sequestration
Haemolysis
Long-term complications
Infections: Streptococcus pnemoniae
Chronic leg ulcers
Gallstones: haemolysis
Aseptic necrosis of bone
Chronic renal disease
Retinal detachment, proliferative retinopathy
Surgical complications
Bowel ischaemia
Cholecystitis
Avascular necrosis
Management
Supportive
Hydroxyurea
Repeated transfusions pre operatively
Exchange transfusion in emergencies
Sickle cell trait
Heterozygous state
Asymptomatic
Symptoms associated with extreme situations ie anaesthesia complications
Protective against Plasmodium falciparum
Which statement is false about pethidine?
It is thirty times more lipid soluble than morphine
Structurally similar to morphine
Pethidine has a toxic metabolite (norpethidine) which is cleared by the kidney
Pethidine is metabolized by the liver
Can be given intramuscularly
It has a different structure. It is much more lipid soluble than morphine. It produces less biliary tract spasm than morphine.
Management of pain
World Health Organisation Analgesic Ladder
Initially peripherally acting drugs such as paracetamol or non-steroidal anti-inflammatory drugs (NSAIDs) are given.
If pain control is not achieved, the second part of the ladder is to introduce weak opioid drugs such as codeine or dextropropoxyphene together with appropriate agents to control and minimise side effects.
The final rung of the ladder is to introduce strong opioid drugs such as morphine. Analgesia from peripherally acting drugs may be additive to that from centrally-acting opioids and thus, the two are given together.
The World Federation of Societies of Anaesthesiologists (WFSA) Analgesic Ladder
For management of acute pain
Initially, the pain can be expected to be severe and may need controlling with strong analgesics in combination with local anaesthetic blocks and peripherally acting drugs.
The second rung on the postoperative pain ladder is the restoration of the use of the oral route to deliver analgesia. Strong opioids may no longer be required and adequate analgesia can be obtained by using combinations of peripherally acting agents and weak opioids.
The final step is when the pain can be controlled by peripherally acting agents alone.
Local anaesthetics
Infiltration of a wound with a long-acting local anaesthetic such as Bupivacaine
Analgesia for several hours
Further pain relief can be obtained with repeat injections or by infusions via a thin catheter
Blockade of plexuses or peripheral nerves will provide selective analgesia in those parts of the body supplied by the plexus or nerves
Can either be used to provide anaesthesia for the surgery or specifically for postoperative pain relief
Especially useful where a sympathetic block is needed to improve postoperative blood supply or where central blockade such as spinal or epidural blockade is contraindicated.
Spinal anaesthesia
Provides excellent analgesia for surgery in the lower half of the body and pain relief can last many hours after completion of the operation if long-acting drugs containing vasoconstrictors are used.
- Side effects of spinal anaesthesia include: hypotension, sensory and motor block, nausea and urinary retention.
Epidural anaesthesia
An indwelling epidural catheter inserted. This can then be used to provide a continuous infusion of analgesic agents. It can provide excellent analgesia. They are still the preferred option following major open abdominal procedures and help prevent post operative respiratory compromise resulting from pain.
- Disadvantages of epidurals is that they usually confine patients to bed, especially if a motor block is present. In addition an indwelling urinary catheter is required. Which may not only impair mobility but also serve as a conduit for infection. They are contraindicated in coagulopathies.
Transversus Abdominal Plane block (TAP)
In this technique an ultrasound is used to identify the correct muscle plane and local anaesthetic (usually bupivicaine) is injected. The agent diffuses in the plane and blocks many of the spinal nerves. It is an attractive technique as it provides a wide field of blockade but does not require the placement of any indwelling devices. There is no post operative motor impairment. For this reason it is the preferred technique when extensive laparoscopic abdominal procedures are performed. They will then provide analgesia immediately following surgery but as they do not confine the patient to bed, the focus on enhanced recovery can begin sooner.
-The main disadvantage is that their duration of action is limited to the half life of the local anaesthetic agent chosen. In addition some anaesthetists do not have the USS skills required to site the injections.
Patient Controlled Analgesia (PCA)
- Patients administer their own intravenous analgesia and titrate the dose to their own end-point of pain relief using a small microprocessor - controlled pump. Morphine is the most popular drug used.
Strong Opioids
Severe pain arising from deep or visceral structures requires the use of strong opioids
Morphine
Short half life and poor bioavailability.
Metabolised in the liver and clearance is reduced in patients with liver disease, in the elderly and the debilitated
Side effects include nausea, vomiting, constipation and respiratory depression.
Tolerance may occur with repeated dosage
Pethidine
Synthetic opioid which is structurally different from morphine but which has similar actions. Has 10% potency of morphine.
Short half life and similar bioavailability and clearance to morphine.
Short duration of action and may need to be given hourly.
Pethidine has a toxic metabolite (norpethidine) which is cleared by the kidney, but which accumulates in renal failure or following frequent and prolonged doses and may lead to muscle twitching and convulsions. Extreme caution is advised if pethidine is used over a prolonged period or in patients with renal failure.
Weak opioids
Codeine: markedly less active than morphine, has predictable effects when given orally and is effective against mild to moderate pain.
Non opioid analgesics
- Mild to moderate pain.
Paracetamol
Inhibits prostaglandin synthesis.
Analgesic and antipyretic properties but little anti-inflammatory effect
It is well absorbed orally and is metabolised almost entirely in the liver
Side effects in normal dosage and is widely used for the treatment of minor pain. It causes hepatotoxicity in over dosage by overloading the normal metabolic pathways with the formation of a toxic metabolite.
NSAIDs
Analgesic and anti-inflammatory actions
Inhibition of prostaglandin synthesis by the enzyme Cyclooxygenase which catalyses the conversion of arachidonic acid to the various prostaglandins that are the chief mediators of inflammation. All NSAIDs work in the same way and thus there is no point in giving more than one at a time. .
NSAIDs are, in general, more useful for superficial pain arising from the skin, buccal mucosa, joint surfaces and bone.
Relative contraindications: history of peptic ulceration, gastrointestinal bleeding or bleeding diathesis; operations associated with high blood loss, asthma, moderate to severe renal impairment, dehydration and any history of hypersensitivity to NSAIDs or aspirin.
Neuropathic pain
National Institute of Clinical Excellence (UK) guidelines:
First line: Amitriptyline (Imipramine if cannot tolerate) or pregabalin
Second line: Amitriptyline AND pregabalin
Third line: refer to pain specialist. Give tramadol in the interim (avoid morphine)
If diabetic neuropathic pain: Duloxetine
As of 1 April 2019, pregabalin and gabapentin are Class C controlled substances (under the Misuse of Drugs Act 1971) and scheduled under the Misuse of Drugs Regulations 2001 as Schedule 3. Evaluate patients carefully for a history of drug abuse before prescribing and observe patients for development of signs of abuse and dependence (MHRA, Drug Safety Update April 2019).
References
1. http://guidance.nice.org.uk/CG173/Guidance/pdf/English
2. Lovich-Sapola J, Smith CE, Brandt CP. Post operative pain control. Surg Clin North Am. 2015 Apr;95(2):301-183. Finnerup N et al. Pharmacotherapy for neuropathic pain in adults: a systematic review and meta-analysis. Lancet Neurol. 2015 Feb;14(2):162-73.
A 73 year old lady is hit by a car. She suffers a complex fracture of the distal aspect of her humerus with associated injury to the radial nerve. Which of the following movements will be most impaired as a result?
Elbow extension
Elbow flexion
Shoulder abduction
Wrist extension
None of the above
The triceps will not be affected so elbow extension will be preserved. Loss of wrist extension will be the most obvious effect.
Radial nerve
Continuation of posterior cord of the brachial plexus (root values C5 to T1)
Path
In the axilla: lies posterior to the axillary artery on subscapularis, latissimus dorsi and teres major.
Enters the arm between the brachial artery and the long head of triceps (medial to humerus).
Spirals around the posterior surface of the humerus in the groove for the radial nerve.
At the distal third of the lateral border of the humerus it then pierces the intermuscular septum and descends in front of the lateral epicondyle.
At the lateral epicondyle it lies deeply between brachialis and brachioradialis where it then divides into a superficial and deep terminal branch.
Deep branch crosses the supinator to become the posterior interosseous nerve.
Regions innervated
Motor (main nerve)
Triceps
Anconeus
Brachioradialis
Extensor carpi radialis
Motor (posterior interosseous branch)
Supinator
Extensor carpi ulnaris
Extensor digitorum
Extensor indicis
Extensor digiti minimi
Extensor pollicis longus and brevis
Abductor pollicis longus
Sensory The area of skin supplying the proximal phalanges on the dorsal aspect of the hand is supplied by the radial nerve (this does not apply to the little finger and part of the ring finger)
Muscular innervation and effect of denervation
Anatomical location Muscle affected Effect of paralysis
Shoulder Long head of triceps Minor effects on shoulder stability in abduction
Arm Triceps Loss of elbow extension
Forearm Supinator
Brachioradialis
Extensor carpi radialis longus and brevis Weakening of supination of prone hand and elbow flexion in mid prone position
A 55 year old motorcyclist is involved in a road traffic accident and sustained a Gustilo and Anderson IIIc type fracture to the distal tibia. He was trapped in the wreckage for 7 hours during which time he bled profusely from the fracture site. He has an established distal neurovascular deficit. What is the most appropriate course of action?
Amputation
Skeletal traction
Application of external fixator and arterial reconstruction
Insertion of intramedullary nail and arterial reconstruction
Application of plate to tibia and arterial reconstruction
This man is unstable, and at 7 hours after extraction, the limb is not viable. The safest option is primary amputation.
Fracture management
Bony injury resulting in a fracture may arise from trauma (excessive forces applied to bone), stress related (repetitive low velocity injury) or pathological (abnormal bone which fractures during normal use of following minimal trauma)
Diagnosis involves not just evaluating the fracture ; such as site and type of injury but also other associated injuries and distal neurovascular deficits. This may entail not just clinical examination but radiographs of proximal and distal joints.
When assessing x-rays it is important to assess for changes in length of the bone, the angulation of the distal bone, rotational effects, presence of material such as glass.
Fracture types
Fracture type Description
Oblique fracture Fracture lies obliquely to long axis of bone
Comminuted fracture >2 fragments
Segmental fracture More than one fracture along a bone
Transverse fracture Perpendicular to long axis of bone
Spiral fracture Severe oblique fracture with rotation along long axis of bone
Open Vs Closed
It is also important to distinguish open from closed injuries. The most common classification system for open fractures is the Gustilo and Anderson classification system (given below):
Grade Injury
1 Low energy wound <1cm
2 Greater than 1cm wound with moderate soft tissue damage
3 High energy wound > 10cm with extensive soft tissue damage
3 A (sub group of 3) Adequate soft tissue coverage
3 B (sub group of 3) Inadequate soft tissue coverage
3 C (sub group of 3) Associated arterial injury
Key points in management of fractures
Immobilise the fracture including the proximal and distal joints
Carefully monitor and document neurovascular status, particularly following reduction and immobilisation
Manage infection including tetanus prophylaxis
IV broad spectrum antibiotics for open injuries
As a general principle all open fractures should be thoroughly debrided ( and internal fixation devices avoided or used with extreme caution)
Open fractures constitute an emergency and should be debrided and lavaged within 6 hours of injury
Which muscle is responsible for causing flexion of the interphalangeal joint of the thumb?
Flexor pollicis longus
Flexor pollicis brevis
Flexor digitorum superficialis
Flexor digitorum profundus
Adductor pollicis
There are 8 muscles:
1. Two flexors (flexor pollicis brevis and flexor pollicis longus)
2. Two extensors (extensor pollicis brevis and longus)
3. Two abductors (abductor pollicis brevis and longus)
4. One adductor (adductor pollicis)
5. One muscle that opposes the thumb by rotating the CMC joint (opponens pollicis).
Flexor and extensor longus insert on the distal phalanx moving both the MCP and IP joints.
Hand
Anatomy of the hand
Bones
8 Carpal bones
5 Metacarpals
14 phalanges
Intrinsic Muscles 8 Interossei - Supplied by ulnar nerve
4 palmar-adduct fingers
4 dorsal- abduct fingers
Intrinsic muscles Lumbricals
Flex MCPJ and extend the IPJ.
Origin deep flexor tendon and insertion dorsal extensor hood mechanism.
Innervation: 1st and 2nd- median nerve, 3rd and 4th- deep branch of the ulnar nerve.
Thenar eminence
Abductor pollicis brevis
Opponens pollicis
Flexor pollicis brevis
Hypothenar eminence
Opponens digiti minimi
Flexor digiti minimi brevis
Abductor digiti minimi
Image sourced from Wikipedia
Fascia and compartments of the palm
The fascia of the palm is continuous with the antebrachial fascia and the fascia of the dorsum of the hand. The palmar fascia is thin over the thenar and hypothenar eminences. In contrast, the central palmar fascia is relatively thick. The palmar aponeurosis covers the soft tissues and overlies the flexor tendons. The apex of the palmar aponeurosis is continuous with the flexor retinaculum and the palmaris longus tendon. Distally, it forms four longitudinal digital bands that attach to the bases of the proximal phalanges, blending with the fibrous digital sheaths.
A medial fibrous septum extends deeply from the medial border of the palmar aponeurosis to the 5th metacarpal. Lying medial to this are the hypothenar muscles. In a similar fashion, a lateral fibrous septum extends deeply from the lateral border of the palmar aponeurosis to the 3rd metacarpal. The thenar compartment lies lateral to this area.
Lying between the thenar and hypothenar compartments is the central compartment. It contains the flexor tendons and their sheaths, the lumbricals, the superficial palmar arterial arch and the digital vessels and nerves.
The deepest muscular plane is the adductor compartment, which contains adductor pollicis.
Short muscles of the hand
These comprise the lumbricals and interossei. The four slender lumbrical muscles flex the fingers at the metacarpophalangeal joints and extend the interphalangeal joint. The four dorsal interossei are located between the metacarpals and the four palmar interossei lie on the palmar surface of the metacarpals in the interosseous compartment of the hand.
Long flexor tendons and sheaths in the hand
The tendons of FDS and FDP enter the common flexor sheath deep to the flexor retinaculum. The tendons enter the central compartment of the hand and fan out to their respective digital synovial sheaths. Near the base of the proximal phalanx, the tendon of FDS splits to permit the passage of FDP. The FDP tendons are attached to the margins of the anterior aspect of the base of the distal phalanx.
The fibrous digital sheaths contain the flexor tendons and their synovial sheaths. These extend from the heads of the metacarpals to the base of the distal phalanges.
Palmar Interossei
Note that there are 4 palmar interossei. The first is a small slip of muscle which arises from the ulnar side of the base of the first metacarpal and passes between the head of the first dorsal interosseous and the oblique head of adductor pollicis to insert into the ulnar base of the of the proximal phalanx of the thumb. The second arises from the ulnar side of the body of the second metacarpal and is inserted into the ulnar side of the extensor hood of the index. The third and fourth palmar interossei arise from the radial sides of the bodies of the 4th and 5th metacarpals respectively and insert into the radial sides of the extensor hoods of the ring and little fingers.
The pudendal canal is a fascial canal located on the lateral wall of the ischioanal fossa. In this location, it lies on the inferior border of which of the following muscles?
Coccygeus
Obturator internus
Pubococcygeus
Iliococcygeus
Piriformis
The coccygeus, pubococcygeus and iliococcygeus form part of the pelvic diaphragm and are not related to it. The piriformis exits the pelvis via the greater sciatic foramen and is not associated with the canal in the ischiorectal fossa.
Pudendal canal
The pudendal canal is located along the lateral wall of the ischioanal fossa at the inferior margin of the obturator internus muscle. It extends from the lesser sciatic foramen to the posterior margin of the urogenital diaphragm. It conveys the internal pudendal vessels and nerve.
Which of the muscles below does not cause lateral rotation of the hip?
Obturator internus
Quadratus femoris
Gemellus inferior
Piriformis
Pectineus
Mnemonic lateral hip rotators: P-GO-GO-Q (top to bottom)
Piriformis
Gemellus superior
Obturator internus
Gemellus inferior
Obturator externus
Quadratus femoris
Pectineus adducts and medially rotates the femur.
Hip joint
Head of femur articulates with acetabulum of the pelvis
Both covered by articular hyaline cartilage
The acetabulum forms at the union of the ilium, pubis, and ischium
The triradiate cartilage (Y-shaped growth plate) separates the pelvic bones
The acetabulum holds the femoral head by the acetabular labrum
Normal angle between femoral head and femoral shaft is 130o
Ligaments
Transverse ligament: joints anterior and posterior ends of the articular cartilage
Head of femur ligament (ligamentum teres): acetabular notch to the fovea. Contains arterial supply to head of femur in children.
Extracapsular ligaments
Iliofemoral ligament: inverted Y shape. Anterior iliac spine to the trochanteric line
Pubofemoral ligament: acetabulum to lesser trochanter
Ischiofemoral ligament: posterior support. Ischium to greater trochanter.
Blood supply
Medial circumflex femoral and lateral circumflex femoral arteries (Branches of profunda femoris). Also from the inferior gluteal artery. These form an anastomosis and travel to up the femoral neck to supply the head.
A 58 year old woman is admitted with an attack of severe acute pancreatitis. She is managed on the intensive care unit and is making progress. She then deteriorates and a CT scan shows extensive pancreatic necrosis (>40%). There are concerns that this may have become infected. What is the correct course of action?
Undertake a fine needle aspiration of the area
Perform a pancreatic necrosectomy
Perform a Whipples procedure
Arrange an ERCP
Perform a distal pancreatectomy
When there are concerns that pancreatic necrosis may have become infected the usual approach is to perform an image guided FNA for culture. There is always the risk of seeding infection with such a strategy so it must be performed with care. Pancreatic necrosectomy is not usually undertaken until the presence of infection is proven.
Management of Pancreatitis
Management of Acute Pancreatitis in the UK
Diagnosis
Traditionally hyperamylasaemia has been utilised with amylase being elevated three times the normal range.
However, amylase may give both false positive and negative results.
Serum lipase is both more sensitive and specific than serum amylase. It also has a longer half life.
Serum amylase levels do not correlate with disease severity.
Differential causes of hyperamylasaemia
Acute pancreatitis
Pancreatic pseudocyst
Mesenteric infarct
Perforated viscus
Acute cholecystitis
Diabetic ketoacidosis
Assessment of severity
Glasgow, Ranson scoring systems and APACHE II
Biochemical scoring e.g. using CRP
Features that may predict a severe attack within 48 hours of admission to hospital
Initial assessment
Clinical impression of severity
Body mass index >30
Pleural effusion
APACHE score >8
24 hours after admission
Clinical impression of severity
APACHE II >8
Glasgow score of 3 or more
Persisting multiple organ failure
CRP>150
48 hours after admission
Glasgow Score of >3
CRP >150
Persisting or progressive organ failure
Table adapted from UK guidelines for management of acute pancreatitis. GUT 2005, 54 suppl III
Management
Nutrition
There is reasonable evidence to suggest that the use of enteral nutrition does not worsen the outcome in pancreatitis
Most trials to date were underpowered to demonstrate a conclusive benefit.
The rationale behind feeding is that it helps to prevent bacterial translocation from the gut, thereby contributing to the development of infected pancreatic necrosis.
Use of antibiotic therapy
Many UK surgeons administer antibiotics to patients with acute pancreatitis. However, there is very little evidence to support this practice.
A recent Cochrane review highlights the potential benefits of administering Imipenem to patients with established pancreatic necrosis in the hope of averting the progression to infection.
There are concerns that the administration of antibiotics in mild attacks of pancreatitis will not affect outcome and may contribute to antibiotic resistance and increase the risks of antibiotic associated diarrhoea.
Surgery
Patients with acute pancreatitis due to gallstones should undergo early cholecystectomy.
Patients with obstructed biliary system due to stones should undergo early ERCP.
Patients with extensive necrosis where infection is suspected should usually undergo FNA for culture.
Patients with infected necrosis should undergo either radiological drainage or surgical necrosectomy. The choice of procedure depends upon local expertise.
References
www.bsg.org.uk/pdfworddocs/pancreatic.pdf
Antibiotic therapy for prophylaxis against infection of pancreatic necrosis in acute pancreatitis. Villatoro et al. Cochrane Library DOI: 10.1002/14651858.CD002941.pub3. 2010 version.
Which of the nerves below innervates the tensor tympani muscle?
Vestibulocochlear
Facial
Vagus
Trigeminal
Hypoglossal
The tensor tympani muscle (in conjunction with stapedius) helps to mitigate the effects of loud sounds. The tensor tympani is innervated by the trigeminal nerve (the stapedius by the facial). In some people with hyperacousia, the tensor tympani muscle does not function normally.
Trigeminal nerve
The trigeminal nerve is the main sensory nerve of the head. In addition to its major sensory role, it also innervates the muscles of mastication.
Distribution of the trigeminal nerve
Sensory
Scalp
Face
Oral cavity (and teeth)
Nose and sinuses
Dura mater
Motor
Muscles of mastication
Mylohyoid
Anterior belly of digastric
Tensor tympani
Tensor palati
Autonomic connections (ganglia)
Ciliary
Sphenopalatine
Otic
Submandibular
Path
Originates at the pons
Sensory root forms the large, crescentic trigeminal ganglion within Meckel’s cave, and contains the cell bodies of incoming sensory nerve fibres. Here the 3 branches exit.
The motor root cell bodies are in the pons and the motor fibres are distributed via the mandibular nerve. The motor root is not part of the trigeminal ganglion.
Branches of the trigeminal nerve
Ophthalmic nerve Sensory only
Maxillary nerve Sensory only
Mandibular nerve Sensory and motor
Sensory
Ophthalmic Exits skull via the superior orbital fissure
Sensation of: scalp and forehead, the upper eyelid, the conjunctiva and cornea of the eye, the nose (including the tip of the nose, except alae nasi), the nasal mucosa, the frontal sinuses, and parts of the meninges (the dura and blood vessels).
Maxillary nerve Exit skull via the foramen rotundum
Sensation: lower eyelid and cheek, the nares and upper lip, the upper teeth and gums, the nasal mucosa, the palate and roof of the pharynx, the maxillary, ethmoid and sphenoid sinuses, and parts of the meninges.
Mandibular nerve Exit skull via the foramen ovale
Sensation: lower lip, the lower teeth and gums, the chin and jaw (except the angle of the jaw), parts of the external ear, and parts of the meninges.
Motor
Distributed via the mandibular nerve.
The following muscles of mastication are innervated:
Masseter
Temporalis
Medial pterygoid
Lateral pterygoid
Other muscles innervated include:
Tensor veli palatini
Mylohyoid
Anterior belly of digastric
Tensor tympani
During a radical neck dissection, division of which of the following fascial layers will expose the ansa cervicalis?
Pretracheal fascia
Carotid sheath
Prevertebral fascia
Investing layer of fascia
Sibsons fascia
The ansa cervicalis lies anterior to the carotid artery and on the carotid sheath. It may be exposed by division of the pretracheal fascia at the posterolateral aspect of the thyroid gland. The pre vertebral fascia lies more posteriorly and division of the investing layer of fascia will not expose this nerve.
Ansa cervicalis
Superior root Branch of C1 anterolateral to carotid sheath
Inferior root Derived from C2 and C3 roots, passes posterolateral to the internal jugular vein (may lie either deep or superficial to it)
Innervation Sternohyoid
Sternothyroid
Omohyoid
The ansa cervicalis lies anterior to the carotid sheath. The nerve supply to the inferior strap muscles enters at their inferior aspect. Therefore when dividing these muscles to expose a large goitre, the muscles should be divided in their upper half.
A 49 year old man undergoes a distal gastrectomy to treat peptic ulcer disease. Whilst he is recovering from surgery on the ward, which of the effects listed below is most likely to be encountered?
Decreased cardiac output
Bradycardia
Relaxation of the smooth muscle of the GI tract
Vasodilatation
Reduction in angiotensin II levels
The stress response to surgery is multimodal and encompasses a number of physiological responses. However, cardiac output is usually increased (sympathetic nervous system induces hypertension and tachycardia). Its also results in GI tract smooth muscle relaxation. The renin angiotensin system is activated resulting in vasoconstriction and increased angiotensin II levels.
Stress response: Endocrine and metabolic changes
- Surgery precipitates hormonal and metabolic changes causing the stress response.
Stress response is associated with: substrate mobilization, muscle protein loss, sodium and water retention, suppression of anabolic hormone secretion, activation of the sympathetic nervous system, immunological and haematological changes.
The hypothalamic-pituitary axis and the sympathetic nervous systems are activated and there is a failure of the normal feedback mechanisms of control of hormone secretion.
A summary of the hormonal changes associated with the stress response:
Increased Decreased No Change
Growth hormone Insulin Thyroid stimulating hormone
Cortisol Testosterone Luteinizing hormone
Renin Oestrogen Follicle stimulating hormone
Adrenocorticotrophic hormone (ACTH)
Aldosterone
Prolactin
Antidiuretic hormone
Glucagon
Sympathetic nervous system
Stimulates catecholamine release
Causes tachycardia and hypertension
Pituitary gland
ACTH and growth hormone (GH) is stimulated by hypothalamic releasing factors, corticotrophin releasing factor (CRF) and somatotrophin (or growth hormone releasing factor)
Perioperative increased prolactin secretion occurs by release of inhibitory control
Secretion of thyroid stimulating hormone (TSH), luteinizing hormone (LH) and follicle stimulating hormone (FSH) does not change significantly
ACTH stimulates cortisol production within a few minutes of the start of surgery. More ACTH is produced than needed to produce a maximum adrenocortical response.
Cortisol
Significant increases within 4-6 hours of surgery (>1000 nmol litre-1).
The usual negative feedback mechanism fails and concentrations of ACTH and cortisol remain persistently increased.
The magnitude and duration of the increase correlate with the severity of stress and the response is not abolished by the administration of corticosteroids.
The metabolic effects of cortisol are enhanced:
Skeletal muscle protein breakdown to provide gluconeogenic precursors and amino acids for protein synthesis in the liver
Stimulation of lipolysis
‘Anti-insulin effect’
Mineralocorticoid effects
Anti-inflammatory effects
Growth hormone
Increased secretion after surgery has a minor role
Most important for preventing muscle protein breakdown and promote tissue repair by insulin growth factors
Alpha Endorphin
Increased
Antidiuretic hormone
An important vasopressor and enhances haemostasis
Renin is released causing the conversion of angiotensinogen to angiotensin I
Angiotensin II formed by ACE on angiotensin 1, which causes the secretion of aldosterone from the adrenal cortex. This increases sodium reabsorption at the distal convoluted tubule
Insulin
Release inhibited by stress
Occurs via the inhibition of the beta cells in the pancreas by the α2-adrenergic inhibitory effects of catecholamines
Insulin resistance by target cells occurs later
The perioperative period is characterized by a state of functional insulin deficiency
Thyroxine (T4) and tri-iodothyronine (T3)
Circulating concentrations are inversely correlated with sympathetic activity and after surgery there is a reduction in thyroid hormone production, which normalises over a few days.
Metabolic effect of endocrine response
Carbohydrate metabolism
Hyperglycaemia is a main feature of the metabolic response to surgery
Due to increase in glucose production and a reduction in glucose utilization
Catecholamines and cortisol promote glycogenolysis and gluconeogenesis
Initial failure of insulin secretion followed by insulin resistance affects the normal responses
The proportion of the hyperglycaemic response reflects the severity of surgery
Hyperglycaemia impairs wound healing and increase infection rates
Protein metabolism
Initially there is inhibition of protein anabolism, followed later, if the stress response is severe, by enhanced catabolism
The amount of protein degradation is influenced by the type of surgery and also by the nutritional status of the patient
Mainly skeletal muscle protein is affected
The amino acids released form acute phase proteins (fibrinogen, C reactive protein, complement proteins, a2-macroglobulin, amyloid A and ceruloplasmin) and are used for gluconeogenesis
Nutritional support has little effect on preventing catabolism
Lipid metabolism
Increased catecholamine, cortisol and glucagon secretion, and insulin deficiency, promotes lipolysis and ketone body production.
Salt and water metabolism
ADH causes water retention, concentrated urine, and potassium loss and may continue for 3 to 5 days after surgery
Renin causes sodium and water retention
Cytokines
Glycoproteins
Interleukins (IL) 1 to 17, interferons, and tumour necrosis factor
Synthesized by activated macrophages, fibroblasts, endothelial and glial cells in response to tissue injury from surgery or trauma
IL-6 main cytokine associated with surgery. Peak 12 to 24 h after surgery and increase by the degree of tissue damage Other effects of cytokines include fever, granulocytosis, haemostasis, tissue damage limitation and promotion of healing.
Modifying the response
Opioids suppress hypothalamic and pituitary hormone secretion
At high doses the hormonal response to pelvic and abdominal surgery is abolished. However, such doses prolong recovery and increase the need for postoperative ventilatory support
Spinal anaesthesia can reduce the glucose, ACTH, cortisol, GH and epinephrine changes, although cytokine responses are unaltered
Cytokine release is reduced in less invasive surgery
Nutrition prevents the adverse effects of the stress response. Enteral feeding improves recovery
Growth hormone and anabolic steroids may improve outcome
Normothermia decreases the metabolic response
References
Vasiliki Manou-Stathopoulou et al. Redefining the perioperative stress response: a narrative review. Br J Anaesth. 2019 Nov;123(5):570-583.
Deborah Burton, Grainne Nicholson, and George Hall
Endocrine and metabolic response to surgery .
Contin Educ Anaesth Crit Care Pain (2004) 4(5): 144-147 doi:10.1093/bjaceaccp/mkh040
Which of the following are not generally supplied by the right coronary artery?
The sino atrial node
The circumflex artery
The atrioventricular node
Most of the right ventricle
The right atrium
The circumflex artery is generally a branch of the left coronary artery.
Heart anatomy
The walls of each cardiac chamber comprise:
Epicardium
Myocardium
Endocardium
Cardiac muscle is attached to the cardiac fibrous skeleton.
Relations
The heart and roots of the great vessels within the pericardial sac are related to the posterior aspect of the sternum, medial ends of the 3rd to 5th ribs on the left and their associated costal cartilages. The heart and pericardial sac are situated obliquely two thirds to the left and one third to the right of the median plane.
The pulmonary valve lies at the level of the left third costal cartilage.
The mitral valve lies at the level of the fourth costal cartilage.
Coronary sinus
This lies in the posterior part of the coronary groove and receives blood from the cardiac veins. The great cardiac vein lies at its left and the middle and small cardiac veins lie on its right. The smallest cardiac vein (anterior cardiac vein) drains into the right atrium directly.
Aortic sinus
Right coronary artery arises from the right aortic sinus, the left is derived from the left aortic sinus, which lies posteriorly.
Features of the left ventricle as opposed to the right
Structure Left Ventricle
A-V Valve Mitral (double leaflet)
Walls Twice as thick as right
Trabeculae carnae Much thicker and more numerous
Right coronary artery
The RCA supplies:
Right atrium
Diaphragmatic part of the right ventricle
Usually the posterior third of the interventricular septum
The sino atrial node (60% cases)
The atrio ventricular node (80% cases)
Left coronary artery
The LCA supplies:
Left atrium
Most of left ventricle
Part of the right ventricle
Anterior two thirds of the inter ventricular septum
The sino atrial node (remaining 40% cases)
Innervation of the heart
Autonomic nerve fibres from the superficial and deep cardiac plexus. These lie anterior to the bifurcation of the trachea, posterior to the ascending aorta and superior to the bifurcation of the pulmonary trunk. The parasympathetic supply to the heart is from presynaptic fibres of the vagus nerves.
Valves of the heart
Mitral valve Aortic valve Pulmonary valve Tricuspid valve
2 cusps 3 cusps 3 cusps 3 cusps
First heart sound Second heart sound Second heart sound First heart sound
1 anterior cusp 2 anterior cusps 2 anterior cusps 2 anterior cusps
Attached to chordae tendinae No chordae No chordae Attached to chordae tendinae
What is the nerve root value of the external urethral sphincter?
S4
S1, S2, S3
S2, S3, S4
L3, L4, L5
L5, S1, S2
The external urethral sphincter is innervated by branches of the pudendal nerve, therefore the root values are S2, S3, S4.
Urethral anatomy
Female urethra
The female urethra is shorter and more acutely angulated than the male urethra. It is an extra-peritoneal structure and embedded in the endopelvic fascia. The neck of the bladder is subjected to transmitted intra-abdominal pressure and therefore deficiency in this area may result in stress urinary incontinence. Between the layers of the urogenital diaphragm the female urethra is surrounded by the external urethral sphincter, this is innervated by the pudendal nerve. It ultimately lies anterior to the vaginal orifice.
Male urethra
In males the urethra is much longer and is divided into four parts.
Pre-prostatic urethra Extremely short and lies between the bladder and prostate gland.It has a stellate lumen and is between 1 and 1.5cm long.Innervated by sympathetic noradrenergic fibres, as this region is composed of striated muscles bundles they may contract and prevent retrograde ejaculation.
Prostatic urethra This segment is wider than the membranous urethra and contains several openings for the transmission of semen (at the midpoint of the urethral crest).
Membranous urethra Narrowest part of the urethra and surrounded by external sphincter. It traverses the perineal membrane 2.5cm postero-inferior to the symphysis pubis.
Penile urethra Travels through the corpus spongiosum on the underside of the penis. It is the longest urethral segment.It is dilated at its origin as the infrabulbar fossa and again in the glans penis as the navicular fossa. The bulbo-urethral glands open into the spongiose section of the urethra 2.5cm below the perineal membrane.
The urothelium is transitional in nature near to the bladder and becomes squamous more distally.
A 18 year old man presents with an indirect inguinal hernia and undergoes surgery. The deep inguinal ring is exposed and held with a retractor at its medial aspect. Which structure is most likely to lie under the retractor?
Ureter
Inferior epigastric artery
Internal iliac vein
Femoral artery
Lateral border of rectus abdominis
Boundaries of the deep inguinal ring:
Superolaterally - transversalis fascia
Inferomedially - inferior epigastric artery
The deep inguinal ring is closely related to the inferior epigastric artery. The inferior epigastric artery forms part of the structure referred to as Hesselbach’s triangle.
Inguinal canal
Location
Above the inguinal ligament
The inguinal canal is 4cm long
The superficial ring is located anterior to the pubic tubercle
The deep ring is located approximately 1.5-2cm above the half way point between the anterior superior iliac spine and the pubic tubercle
Boundaries of the inguinal canal
Floor
External oblique aponeurosis
Inguinal ligament
Lacunar ligament
Roof
Internal oblique
Transversus abdominis
Anterior wall External oblique aponeurosis
Posterior wall
Transversalis fascia
Conjoint tendon
Laterally
Internal ring
Transversalis fascia
Fibres of internal oblique
Medially
External ring
Conjoint tendon
Contents
Males Spermatic cord and ilioinguinal nerve As it passes through the canal the spermatic cord has 3 coverings:
External spermatic fascia from external oblique aponeurosis
Cremasteric fascia
Internal spermatic fascia
Females Round ligament of uterus and ilioinguinal nerve
A 55-year-old man with a history of type 2 diabetes mellitus, bipolar disorder and chronic obstructive pulmonary disease has bloods taken during a pre operative assessment of an inguinal hernia repair:
Na+ 125 mmol/l
K+ 3.8 mmol/l
Bicarbonate 24 mmol/l
Urea 3.7 mmol/l
Creatinine 92 µmol/l
Due to his smoking history a chest x-ray is ordered which is reported as normal. The Consultant asks you what is the most likely cause for the hyponatraemia?
Metformin
Lithium
Carbamazepine
Sodium valproate
Pioglitazone
SIADH - drug causes: carbamazepine, sulfonylureas, SSRIs, tricyclics
Lithium can cause diabetes insipidus but this is generally associated with a high sodium. Lithium only tends to cause raised antidiuretic hormone levels following a severe overdosage.
syndrome of inappropriate antidiuretic hormone (SIADH): causes
Malignancy
especially small cell lung cancer
also: pancreas, prostate
Neurological
stroke
subarachnoid haemorrhage
subdural haemorrhage
meningitis/encephalitis/abscess
Infections
tuberculosis
pneumonia
Drugs
sulfonylureas
SSRIs, tricyclics
carbamazepine
vincristine
cyclophosphamide
Other causes
positive end-expiratory pressure (PEEP)
porphyrias
You are assisting in an open right adrenalectomy for a large adrenal adenoma. The consultant is distracted and you helpfully pull the adrenal into the wound to improve the view. Unfortunately this is followed by brisk bleeding. The vessel responsible for this is most likely to be:
Portal vein
Phrenic vein
Right renal vein
Superior mesenteric vein
Inferior vena cava
It drains directly via a very short vessel. If the sutures are not carefully tied then it may be avulsed off the IVC. An injury best managed using a Satinsky clamp and a 6/0 prolene suture.
Adrenal gland anatomy
Anatomy
Location Superomedially to the upper pole of each kidney
Relationships of the right adrenal Diaphragm-Posteriorly, Kidney-Inferiorly, Vena Cava-Medially, Hepato-renal pouch and bare area of the liver-Anteriorly
Relationships of the left adrenal Crus of the diaphragm-Postero- medially, Pancreas and splenic vessels-Inferiorly, Lesser sac and stomach-Anteriorly
Arterial supply Superior adrenal arteries- from inferior phrenic artery, Middle adrenal arteries - from aorta, Inferior adrenal arteries -from renal arteries
Venous drainage of the right adrenal Via one central vein directly into the IVC
Venous drainage of the left adrenal Via one central vein into the left renal vein
Which of the following structures separates the posterior cruciate ligament from the popliteal artery?
Oblique popliteal ligament
Transverse ligament
Popliteus tendon
Biceps femoris
Semitendinosus
The posterior cruciate ligament is separated from the popliteal vessels at its origin by the oblique popliteal ligament.It is attached above to the upper margin of the intercondyloid fossa and posterior surface of the femur close to the articular margins of the condyles, and below to the posterior margin of the head of the tibia. The transverse ligament is located anteriorly.
Knee joint
The knee joint is a synovial joint, the largest and most complicated. It consists of two condylar joints between the femur and tibia and a sellar joint between the patella and the femur. The tibiofemoral articular surfaces are incongruent, however, this is improved by the presence of the menisci. The degree of congruence is related to the anatomical position of the knee joint and is greatest in full extension.
Knee joint compartments
Tibiofemoral
Comprised of the patella/femur joint, lateral and medial compartments (between femur condyles and tibia)
Synovial membrane and cruciate ligaments partially separate the medial and lateral compartments
Patellofemoral
Ligamentum patellae
Actions: provides joint stability in full extension
Fibrous capsule
The capsule of the knee joint is a complex, composite structure with contributions from adjacent tendons.
Anterior fibres The capsule does not pass proximal to the patella. It blends with the tendinous expansions of vastus medialis and lateralis
Posterior fibres These fibres are vertical and run from the posterior surface of the femoral condyles to the posterior aspect of the tibial condyle
Medial fibres Attach to the femoral and tibial condyles beyond their articular margins, blending with the tibial collateral ligament
Lateral fibres Attach to the femur superior to popliteus, pass over its tendon to head of fibula and tibial condyle
Bursae
Anterior
Subcutaneous prepatellar bursa; between patella and skin
Deep infrapatellar bursa; between tibia and patellar ligament
Subcutaneous infrapatellar bursa; between distal tibial tuberosity and skin
Laterally
Bursa between lateral head of gastrocnemius and joint capsule
Bursa between fibular collateral ligament and tendon of biceps femoris
Bursa between fibular collateral ligament and tendon of popliteus
Medially
Bursa between medial head of gastrocnemius and the fibrous capsule
Bursa between tibial collateral ligament and tendons of sartorius, gracilis and semitendinosus
Bursa between the tendon of semimembranosus and medial tibial condyle and medial head of gastrocnemius
Posterior Highly variable and inconsistent
Ligaments
Medial collateral ligament Medial epicondyle femur to medial tibial condyle: valgus stability
Lateral collateral ligament Lateral epicondyle femur to fibula head: varus stability
Anterior cruciate ligament Anterior tibia to lateral intercondylar notch femur: prevents tibia sliding anteriorly
Posterior cruciate ligament Posterior tibia to medial intercondylar notch femur: prevents tibia sliding posteriorly
Patellar ligament Central band of the tendon of quadriceps femoris, extends from patella to tibial tuberosity
Menisci
Medial and lateral menisci compensate for the incongruence of the femoral and tibial condyles.
Composed of fibrous tissue.
Medial meniscus is attached to the tibial collateral ligament.
Lateral meniscus is attached to the loose fibres at the lateral edge of the joint and is separate from the fibular collateral ligament. The lateral meniscus is crossed by the popliteus tendon.
Nerve supply
The knee joint is supplied by the femoral, tibial and common peroneal divisions of the sciatic and by a branch from the obturator nerve. Hip pathology pain may be referred to the knee.
Blood supply
Genicular branches of the femoral artery, popliteal and anterior tibial arteries all supply the knee joint.
Which of the following does not cause parotid enlargement?
Liver cirrhosis
Myxoedema
Amiodarone
Tuberculosis
Sjogrens syndrome
Amiodarone
Drugs commonly implicated in parotid gland enlargement include:
Thiouracil, isoprenaline, phenylbutazone, high oestrogen dose contraceptive pills.
Parotid gland clinical
Benign neoplasms
Up to 80% of all salivary gland tumours occur in the parotid gland and up to 80% of these are benign. There is no consistent correlation between the rate of growth and the malignant potential of the lesion. However, benign tumours should not invade structures such as the facial nerve.
With the exception of Warthins tumours, they are commoner in women than men. The median age of developing a lesion is in the 5th decade of life.
Benign tumour types
Tumour type Features
Benign pleomorphic adenoma or benign mixed tumor Most common parotid neoplasm (80%)
Proliferation of epithelial and myoepithelial cells of the ducts and an increase in stromal components
Slow growing, lobular, and not well encapsulated
Recurrence rate of 1-5% with appropriate excision (parotidectomy)
Recurrence possibly secondary to capsular disruption during surgery
Malignant degeneration occurring in 2-10% of adenomas observed for long periods, with carcinoma ex-pleomorphic adenoma occurring most frequently as adenocarcinoma
Warthin tumor (papillary cystadenoma lymphoma or adenolymphoma) Second most common benign parotid tumor (5%)
Most common bilateral benign neoplasm of the parotid
Marked male as compared to female predominance
Occurs later in life (sixth and seventh decades)
Presents as a lymphocytic infiltrate and cystic epithelial proliferation
May represent heterotopic salivary gland epithelial tissue trapped within intraparotid lymph nodes
Incidence of bilaterality and multicentricity of 10%
Malignant transformation rare (almost unheard of)
Monomorphic adenoma Account for less than 5% of tumours
Slow growing
Consist of only one morphological cell type (hence term mono)
Include; basal cell adenoma, canalicular adenoma, oncocytoma, myoepitheliomas
Haemangioma Should be considered in the differential of a parotid mass in a child
Accounts for 90% of parotid tumours in children less than 1 year of age
Hypervascular on imaging
Spontaneous regression may occur and malignant transformation is almost unheard of
Malignant salivary gland tumours
Types of malignancy
Mucoepidermoid carcinoma 30% of all parotid malignancies
Usually low potential for local invasiveness and metastasis (depends mainly on grade)
Adenoid cystic carcinoma Unpredictable growth pattern
Tendency for perineural spread
Nerve growth may display skip lesions resulting in incomplete excision
Distant metastasis more common (visceral rather than nodal spread)
5 year survival 35%
Mixed tumours Often a malignancy occurring in a previously benign parotid lesion
Acinic cell carcinoma Intermediate grade malignancy
May show perineural invasion
Low potential for distant metastasis
5 year survival 80%
Adenocarcinoma Develops from secretory portion of gland
Risk of regional nodal and distant metastasis
5 year survival depends upon stage at presentation, may be up to 75% with small lesions with no nodal involvement
Lymphoma Large rubbery lesion, may occur in association with Warthins tumours
Diagnosis should be based on regional nodal biopsy rather than parotid resection
Treatment is with chemotherapy (and radiotherapy)
Diagnostic evaluation
Plain x-rays may be used to exclude calculi
Sialography may be used to delineate ductal anatomy
FNAC is used in most cases
Superficial parotidectomy may be either diagnostic or therapeutic depending upon the nature of the lesion
Where malignancy is suspected the primary approach should be definitive resection rather than excisional biopsy
CT/ MRI may be used in cases of malignancy for staging primary disease
Treatment
For nearly all lesions this consists of surgical resection, for benign disease this will usually consist of a superficial parotidectomy. For malignant disease a radical or extended radical parotidectomy is performed. The facial nerve is included in the resection if involved. The need for neck dissection is determined by the potential for nodal involvement.
Other parotid disorders
HIV infection
Lymphoepithelial cysts associated with HIV occur almost exclusively in the parotid
Typically presents as bilateral, multicystic, symmetrical swelling
Risk of malignant transformation is low and management usually conservative
Sjogren syndrome
Autoimmune disorder characterised by parotid enlargement, xerostomia and keratoconjunctivitis sicca
90% of cases occur in females
Second most common connective tissue disorder
Bilateral, non tender enlargement of the gland is usual
Histologically, the usual findings are of a lymphocytic infiltrate in acinar units and epimyoepithelial islands surrounded by lymphoid stroma
Treatment is supportive
There is an increased risk of subsequent lymphoma
Sarcoid
Parotid involvement occurs in 6% of patients with sarcoid
Bilateral in most cases
Gland is not tender
Xerostomia may occur
Management of isolated parotid disease is usually conservative
Which of the following is commonest cause of acute abdominal pain in acute unselected surgical ‘take’?
Non specific abdominal pain
Biliary colic
Acute appendicitis
Ureteric colic
Pancreatitis
Non specific abdominal pain is a commonly recorded diagnosis for patients presenting with acute abdominal pain. Following careful diagnostic work up, a proportion of patients may be identified with disorders such as coeliac disease and the diagnosis of non specific abdominal pain should not be used lightly.
Acute abdominal pain
Acute abdominal pain is a common cause of admission to hospital. The relative proportions of conditions presenting with abdominal pain is given below:
Non specific abdominal pain (35%)
Appendicitis (17%)
Intestinal obstruction (15%)
Urological disease (6%)
Gallstone disease (5%)
Colonic diverticular disease (4%)
Abdominal trauma (3%)
Perforated peptic ulcer (3%)
Pancreatitis (2%)
(Data derived from Irvin T. Br. J. Surg 1989 76:1121-1125)
Non specific abdominal pain should really be a diagnosis of exclusion and if care is taken in excluding organic disease the proportion of cases labeled such should decline. It should also be appreciated that a proportion of patients may have an underlying medical cause for their symptoms such as pneumonia or diabetic ketoacidosis.
Key points in management
Early administration of adequate analgesia (including opiates).
Abdominal ultrasound is safe, non invasive and cheap and yields significantly more information than plain radiology. However, plain radiology is still the main test for suspected perforated viscus, especially out of hours.
In up to 50% cases with perforated peptic ulcer, the plain x-rays may show no evidence of free air. If clinical signs suggest otherwise, then a CT scan may be a more accurate investigation, if plain films are normal.
Plain film radiology usually cannot detect <1mm free air, and is 33% sensitive for detection of 1-13mm pockets of free air (Stoker et al. Radiology 2009 253: 31-46).
Think of strangulated intestine when there is fever, raised white cell count, tachycardia and peritonism.
In suspected large bowel obstruction a key investigation is either a water soluble contrast enema or CT scan.
Where need for surgery is difficult to define and imaging is inconclusive the use of laparoscopy as a definitive diagnostic test is both safe and sensible.
The cephalic vein pierces the clavipectoral fascia to terminate in which of the veins listed below?
External jugular
Axillary
Internal jugular
Azygos
Brachial
Path
Dorsal venous arch drains laterally into the cephalic vein
Crosses the anatomical snuffbox and travels laterally up the arm
At the antecubital fossa connected to the basilic vein by the median cubital vein
Pierces deep fascia of deltopectoral groove to join axillary vein
A 52 year man is admitted to the vascular ward for an amputation. The patient complains of unsteadiness. On further examination you detect right facial numbness and right sided nystagmus. There is sensory loss of the left side of the body. What is the most likely cause?
Lateral medullary syndrome
Pontine infarct
Cerebellar infarct
Middle cerebral artery infarct
Posterior cerebral artery infarct
A combination of ipsilateral ataxia, nystagmus, dysphagia, facial numbness, cranial nerve palsy with contralateral hemisensory loss indicates this diagnosis.
Stroke: types
Primary intracerebral haemorrhage (PICH, c. 10%)
Presents with headache, vomiting, loss of consciousness
Total anterior circulation infarcts (TACI, c. 15%)
Involves middle and anterior cerebral arteries
Hemiparesis/hemisensory loss
Homonymous hemianopia
Higher cognitive dysfunction e.g. Dysphasia
Partial anterior circulation infarcts (PACI, c. 25%)
Involves smaller arteries of anterior circulation e.g. upper or lower division of middle cerebral artery
Higher cognitive dysfunction or two of the three TACI features
Lacunar infarcts (LACI, c. 25%)
Involves perforating arteries around the internal capsule, thalamus and basal ganglia
Present with either isolated hemiparesis, hemisensory loss or hemiparesis with limb ataxia
Posterior circulation infarcts (POCI, c. 25%)
Vertebrobasilar arteries
Presents with features of brainstem damage
Ataxia, disorders of gaze and vision, cranial nerve lesions
Lateral medullary syndrome (posterior inferior cerebellar artery)
Wallenberg’s syndrome
Ipsilateral: ataxia, nystagmus, dysphagia, facial numbness, cranial nerve palsy
Contralateral: limb sensory loss
Weber’s syndrome
Ipsilateral III palsy
Contralateral weakness
Anterior cerebral artery
Contralateral hemiparesis and sensory loss, lower extremity > upper
Disconnection syndrome
Middle cerebral artery
Contralateral hemiparesis and sensory loss, upper extremity > lower
Contralateral hemianopia
Aphasia (Wernicke’s)
Gaze abnormalities
Posterior cerebral artery
Contralateral hemianopia with macular sparing
Disconnection syndrome
Lacunar
Present with either isolated hemiparesis, hemisensory loss or hemiparesis with limb ataxia
Lateral medulla (posterior inferior cerebellar artery)
Ipsilateral: ataxia, nystagmus, dysphagia, facial numbness, cranial nerve palsy e.g.
Horner’s
Contralateral: limb sensory loss
Pontine
VI nerve: horizontal gaze palsy
VII nerve
Contralateral hemiparesis
A 43 year old lady is admitted with cholestasis secondary to a stone impacted at the level of the ampulla of vater. Which of the following tests is most likely to be predictive of bleeding diathesis at the time of ERCP in this particular case?
Bleeding time
Prothrombin time
APTT
Platelet count
Factor I levels
PT: Vitamin K dependent factors 2, 7, 9, 10
APTT: Factors 8, 9, 11, 12
Jaundice will impair the production of vitamin K dependent clotting factors. This is most accurately tested by measuring the prothrombin time. APTT can be affected by vitamin K deficiency (due to factor 9 deficiency), however this occurs to a lesser extent and is normally associated with severe liver disease. The bleeding time is a measure of platelet function.
Abnormal coagulation
Cause Factors affected
Heparin Prevents activation factors 2,9,10,11
Warfarin Affects synthesis of factors 2,7,9,10
DIC Factors 1,2,5,8,11
Liver disease Factors 1,2,5,7,9,10,11
Interpretation blood clotting test results
Disorder APTT PT Bleeding time
Haemophilia Increased Normal Normal
von Willebrand’s disease Increased Normal Increased
Vitamin K deficiency Increased Increased Normal
A 28 year old professional footballer is admitted to the emergency department. During a tackle his leg is twisted with his knee flexed. He hears a loud crack and his knee rapidly becomes swollen. Which of the following structures is the main site of injury?
Anterior cruciate ligament
Posterior cruciate ligament
Meniscus
Medial collateral ligament
Lateral collateral ligament
This is common in footballers as the football boot studs stick to the ground and high twisting force is applied to a flexed knee. Rapid joint swelling also supports the diagnosis.
Knee injury
Types of injury
Ruptured anterior cruciate ligament
Sport injury
Mechanism: high twisting force applied to a bent knee
Typically presents with: loud crack, pain and RAPID joint swelling (haemoarthrosis)
Poor healing
Management: intense physiotherapy or surgery
Ruptured posterior cruciate ligament
Mechanism: hyperextension injuries
Tibia lies back on the femur
Paradoxical anterior draw test
Rupture of medial collateral ligament
Mechanism: leg forced into valgus via force outside the leg
Knee unstable when put into valgus position
Menisceal tear
Rotational sporting injuries
Delayed knee swelling
Joint locking (Patient may develop skills to ‘unlock’ the knee
Recurrent episodes of pain and effusions are common, often following minor trauma
Chondromalacia patellae
Teenage girls, following an injury to knee e.g. Dislocation patella
Typical history of pain on going downstairs or at rest
Tenderness, quadriceps wasting
Dislocation of the patella
Most commonly occurs as a traumatic primary event, either through direct trauma or through severe contraction of quadriceps with knee stretched in valgus and external rotation
Genu valgum, tibial torsion and high riding patella are risk factors
Skyline x-ray views of patella are required, although displaced patella may be clinically obvious
An osteochondral fracture is present in 5%
The condition has a 20% recurrence rate
Fractured patella
2 types:
i. Direct blow to patella causing undisplaced fragments
ii. Avulsion fracture
Tibial plateau fracture
Occur in the elderly (or following significant trauma in young)
Mechanism: knee forced into valgus or varus, but the knee fractures before the ligaments rupture
Varus injury affects medial plateau and if valgus injury, lateral plateau depressed fracture occurs
Classified using the Schatzker system (see below)
Schatzker Classification system for tibial plateau fractures
Type Anatomical description Features
1 Vertical split of lateral condyle Fracture through dense bone, usually in the young. It may be virtually undisplaced, or the condylar fragment may be pushed inferiorly and tilted
2 Vertical split of the lateral condyle combined with an adjacent load bearing part of the condyle The wedge fragment (which may be of variable size), is displaced laterally; the joint is widened. Untreated, a valgus deformity may develop
3 Depression of the articular surface with intact condylar rim The split does not extend to the edge of the plateau. Depressed fragments may be firmly embedded in subchondral bone, the joint is stable
4 Fragment of the medial tibial condyle Two injuries are seen in this category; (1) a depressed fracture of osteoporotic bone in the elderly. (2) a high energy fracture resulting in a condylar split that runs from the intercondylar eminence to the medial cortex. Associated ligamentous injury may be severe
5 Fracture of both condyles Both condyles fractured but the column of the metaphysis remains in continuity with the tibial shaft
6 Combined condylar and subcondylar fractures High energy fracture with marked comminution
A 43 year old female has undergone a renal transplant 12 months previously. Over the past few weeks there have been concerns about deteriorating renal function. What is the most useful investigation?
MAG 3 renogram
DMSA scan
Intravenous KUB urogram
Renal CT scan
Micturating cystourethrogram
Because it is excreted by renal tubular cells a MAG 3 renogram provides excellent imaging of renal function and is often used in investigating failing transplants.
Functional renal imaging
DMSA scan
Dimercaptosuccinic acid (DMSA) scintigraphy
DMSA localises to the renal cortex with little accumulation in the renal papilla and medulla. It is useful for the identification of cortical defects and ectopic or aberrant kidneys. It does not provide useful information on the ureter of collecting system.
Diethylene-triamine-penta-acetic acid (DTPA)
This is primarily a glomerular filtration agent. It is most useful for the assessment of renal function. Because it is filtered at the level of the glomerulus it provides useful information about the GFR. Image quality may be degraded in patients with chronic renal impairment and derangement of GFR.
MAG 3 renogram
Mercaptoacetyle triglycine is an is extensively protein bound and is primarily secreted by tubular cells rather than filtered at the glomerulus. This makes it the agent of choice for imaging the kidneys of patients with existing renal impairment (where GFR is impaired).
Micturating cystourethrogram (MCUG scan)
This scan provides information relating to bladder reflux and is obtained by filling the bladder with contrast media (via a catheter) and asking the child to void. Images are taken during this phase and the degree of reflux can be calculated
Intra venous urography
This examination is conducted by the administration of intravenous iodinated contrast media. The agent is filtered by the kidneys and excreted and may provide evidence of renal stones or other structural lesions. A rough approximation of renal function may be obtained using the technique. But it is not primarily a technique to be used for this purpose. With the advent of widespread non contrast CT scan protocols for the detection of urinary tract calculi it is now rarely used.
PET/CT
This may be used to evaluate structurally indeterminate lesions in the staging of malignancy.
References
Davis A et al. Investigating urinary tract infections in children. BMJ 2013 (346):35-37.
When is it mandatory to involve an independent mental capacity advocate ?
When a patient requires a significant intervention, time permits and the patient lacks capacity to decide for themselves
When a patient cannot decide on the best treatment
When parents disagree about treatment of a child
When two doctors disagree about the best treatment option
When a patient has lost trust in their clinician
When a patient requires a significant intervention, time permits and the patient lacks capacity to decide for themselves
The Mental Capacity Act (MCA) requires doctors to involve an independent mental capacity advocate (IMCA) for serious medical treatment decisions when:
a best interests decision is required because the doctor has assessed the person as not having the capacity to make the decision themselves
the person does not have family or friends with whom it is appropriate to consult about the decision.
Serious medical treatment is defined in the Mental Capacity Act 2005 (Independent Mental Capacity Advocates) (General) Regulations 2006 as treatment which involves providing, withdrawing or withholding treatment in circumstances where one or more of the following apply:
in a case where a single treatment is being proposed, there is a fine balance between its benefits to the patient and the burdens and risks it is likely to entail for them
in a case where there is a choice of treatments, a decision as to which one to use is finely balanced
what is proposed would be likely to involve serious consequences for the patient.
The MCA Code of Practice says that serious consequences may include treatment options which:
cause serious and prolonged pain, distress or side effects have potentially major consequences for the patient (for example, major surgery or stopping life-sustaining treatment)
have a serious impact on the patients future life choices.
Consent
There are 3 types of consent:
- Informed
- Expressed
- Implied
Consent forms used in UK NHS
Consent Form 1 For competent adults who are able to consent for themselves where consciousness may be impaired (e.g. GA)
Consent Form 2 For an adult consenting on behalf of a child where consciousness is impaired
Consent Form 3 For an adult or child where consciousness is not impaired
Consent Form 4 For adults who lack capacity to provide informed consent
Capacity
Key points include:
1. Understand and retain information
2. Patient believes the information to be true
3. Patient is able to weigh the information to make a decision
All patients must be assumed to have capacity
Consent in minors
Young children and older children who are not Gillick competent cannot consent for themselves. In British law the patients biological mother can always provide consent. The child’s father can consent if the parents are married (and the father is the biological father), or if the father is named on the birth certificate (irrespective of marital status). If parents are not married and the father is not named on the birth certificate then the father cannot consent.
A 63 year old man undergoes an upper GI endoscopy and adrenaline injection for a large actively bleeding duodenal ulcer. He remains stable for 6 hours and the nurses then call because he has passed 400ml malaena and has become tachycardic (pulse rate 120) and hypotensive (Bp 80/40). What is the best option?
Reassure that blood trapped in the upper portion of the gastrointestinal system will pass and that this episode will resolve with phosphate enema
Perform a repeat upper GI endoscopy
Perform a laparotomy and under-running of the ulcer
Administer tranexamic acid and intravenous proton pump inhibitors
Insert a Minnesota tube
The decision as to how best to manage patients with re-bleeding is difficult. Whilst it is tempting to offer repeat endoscopy, this intervention is best used on those with small ulcers. Large posteriorly sited duodenal ulcers are at high risk for re-bleeding and the timeframe of this event suggests that primary endoscopic haemostasis was inadequate. Surgery thus represents the safest way forward.
Upper gastrointestinal bleeding
Patients may present with the following:
Haematemesis and/ or malaena
Epigastric discomfort
Sudden collapse
The extent to which these will occur will depend upon the source. Mortality is higher in patients presenting with haematemesis than malaena alone.
Oesophageal bleeding
Cause Presenting features
Oesophagitis Small volume of fresh blood, often streaking vomit. Malaena rare. Often ceases spontaneously. Usually history of antecedent GORD type symptoms.
Cancer Usually small volume of blood, except as pre terminal event with erosion of major vessels. Often associated symptoms of dysphagia and constitutional symptoms such as weight loss. May be recurrent until malignancy managed.
Mallory Weiss Tear Typically brisk small to moderate volume of bright red blood following bout of repeated vomiting. Malaena rare. Usually ceases spontaneously.
Varices Usually large volume of fresh blood. Swallowed blood may cause malaena. Often associated with haemodynamic compromise. May stop spontaneously but re-bleeds are common until appropriately managed.
Gastric Bleeding
Cause Presenting features
Gastric cancer May be frank haematemesis or altered blood mixed with vomit. Usually prodromal features of dyspepsia and may have constitutional symptoms. Amount of bleeding variable but erosion of major vessel may produce considerable haemorrhage.
Dieulafoy Lesion Often no prodromal features prior to haematemesis and malaena, but this arteriovenous malformation may produce quite considerable haemorrhage and may be difficult to detect endoscopically.
Diffuse erosive gastritis Usually haematemesis and epigastric discomfort. Usually there is an underlying cause such as recent NSAID usage. Large volume haemorrhage may occur with considerable haemodynamic compromise.
Gastric ulcer Small low volume bleeds more common so would tend to present as iron deficiency anaemia. Erosion into a significant vessel may produce considerable haemorrhage and haematemesis.
Duodenum
Most common cause of major haemorrhage is a posteriorly sited duodenal ulcer. However, ulcers at any site in the duodenum may present with haematemesis, malaena and epigastric discomfort. The pain of duodenal ulcer is slightly different to that of gastric ulcers and often occurs several hours after eating. Peri ampullary tumours may bleed but these are rare. In patients with previous abdominal aortic aneurysm surgery aorto-enteric fistulation remains a rare but important cause of major haemorrhage associated with high mortality.
Management
Admission to hospital careful monitoring, cross match blood, check FBC, LFTs, U+E and Clotting (as a minimum)
Patients with on-going bleeding and haemodynamic instability are likely to require O negative blood pending cross matched blood
Early control of airway is vital (e.g. Drowsy patient with liver failure)
Patients with suspected varices should receive terlipressin prior to endoscopy
Ideally all patients admitted with upper gastrointestinal haemorrhage should undergo Upper GI endoscopy within 24 hours of admission. In those who are unstable this should occur immediately after resuscitation or in tandem with it. The endoscopy department is a potentially dangerous place for unstable patients and it may be safer to perform the endoscopy in theatre with an anaesthetist present.
Varices should be banded or subjected to sclerotherapy. If this is not possible owing to active bleeding then a Sengstaken- Blakemore tube (or Minnesota tube) should be inserted. This should be done with care; gastric balloon should be inflated first and oesophageal balloon second. Remember the balloon will need deflating after 12 hours (ideally sooner) to prevent necrosis. Portal pressure should be lowered by combination of medical therapy +/- TIPSS.
Patients with erosive oesophagitis / gastritis should receive a proton pump inhibitor.
Mallory Weiss tears will typically resolve spontaneously
Identifiable bleeding points should receive combination therapy of injection of adrenaline and either a thermal or mechanical treatment. All who have received intervention should receive a continuous infusion of a proton pump inhibitor (IV omeprazole for 72 hours) to reduce the re-bleeding rate.
Patients with diffuse erosive gastritis who cannot be managed endoscopically and continue to bleed may require gastrectomy
Bleeding ulcers that cannot be controlled endoscopically may require laparotomy and ulcer underruning
Indications for surgery
Patients > 60 years
Continued bleeding despite endoscopic intervention
Recurrent bleeding
Known cardiovascular disease with poor response to hypotension
Surgery
Duodenal ulcer
Laparotomy, duodenotomy and under running of the ulcer. If bleeding is brisk then the ulcer is almost always posteriorly sited and will have invaded the gastroduodenal artery. Large bites using 0 Vicryl are taken above and below the ulcer base to occlude the vessel. The duodenotomy should be longitudinal but closed transversely to avoid stenosis.
For gastric ulcer
Under-running of the bleeding site
Partial gastrectomy-antral ulcer
Partial gastrectomy or under running the ulcer- lesser curve ulcer (involving left gastric artery)
Total gastrectomy if bleeding persists
Summary of Acute Upper GI bleeding recommendations:
The need for admission and timing of endoscopic intervention may be predicted by using the Blatchford score. This considers a patients Hb, serum urea, pulse rate and blood pressure. Those patients with a score of 0 are low risk, all others are considered high risk and require admission and endoscopy.
The requirement for pre endoscopic proton pump inhibition is contentious. In the UK the National Institute of Clinical Excellence guidelines suggest the pre endoscopic PPI therapy is unnecessary. Whilst it is accepted that such treatment has no impact on mortality or morbidity a Cochrane review of this practice in 2007 did suggest that it reduced the stigmata of recent haemorrhage at endoscopy. As a result many will still administer PPI to patients prior to endoscopic intervention.
Following endoscopy it is important to calculate the Rockall score for patients to determine their risk of rebleeding and mortality. A score of 3 or less is associated with a rebleeding rate of 4% and a very low risk of mortality and identifies a group of patients suitable for early discharge.
References
1. http://www.sign.ac.uk/guidelines/fulltext/105/index.html
2. Joint Advisory Group on Endoscopy (JAG) Guidelines - http://www.thejag.org.uk
3. NICE Guideline: Management of acute upper GI bleeding. July 2012.
A 34 year old female presents with a thyroid nodule. She has a family history of thyroid disease and both her sisters have undergone total thyroidectomies. Her past medical history includes hypertension which has been difficult to manage. What is the most likely underlying thyroid lesion?
Papillary thyroid cancer
Follicular adenoma
Medullary thyroid cancer
Anaplastic thyroid cancer
Lymphoma of the thyroid
This is a typical scenario for medullary carcinoma in which a phaeochromocytoma may also be present. It may be inherited in an autosomal dominant fashion and affected family members may be offered prophylactic thyroidectomy.
Thyroid disease
Patients may present with a number of different manifestations of thyroid disease. They can be broadly sub classified according to whether they are euthyroid or have clinical signs of thyroid dysfunction. In addition it needs to be established whether they have a mass or not.
Assessment
History
Examination including USS
If a nodule is identified then it should be sampled ideally via an image guided fine needle aspiration
Radionucleotide scanning is of limited use
Thyroid Tumours
Papillary carcinoma
Follicular carcinoma
Anaplastic carcinoma
Medullary carcinoma
Lymphoma’s
Multinodular goitre
One of the most common reasons for presentation
Provided the patient is euthyroid and asymptomatic and no discrete nodules are seen, they can be reassured.
In those with compressive symptoms surgery is required and the best operation is a total thyroidectomy.
Sub total resections were practised in the past and simply result in recurrent disease that requires a difficult revisional resection.
Endocrine dysfunction
In general these patients are managed by physicians initially.
Surgery may be offered alongside radio iodine for patients with Graves disease that fails with medical management or in patients who would prefer not to be irradiated (e.g. pregnant women).
Patients with hypothyroidism do not generally get offered a thyroidectomy. Sometimes people inadvertently get offered resections during the early phase of Hashimotos thyroiditis, however, with time the toxic phase passes and patients can simply be managed with thyroxine.
Complications following surgery
Anatomical such as recurrent laryngeal nerve damage.
Bleeding. Owing to the confined space haematoma’s may rapidly lead to respiratory compromise owing to laryngeal oedema.
Damage to the parathyroid glands resulting in hypocalcaemia.
Further sources of information
1. http://www.acb.org.uk/docs/TFTguidelinefinal.pdf- Association of Clinical Biochemistry guidelines for thyroid function tests.
- British association of endocrine surgeons website- http://www.baets.org.uk
Which of the conditions listed below is least likely to be associated with hypersplenism?
Coeliac disease
Lymphoma
Leukaemia
Portal hypertension
Feltys syndrome
Many conditions may result in overactivity of the spleen. It is not seen in coeliac disease. Feltys syndrome occurs in association with rheumatoid disease.
Disorders affecting the spleen
One of the commonest conditions to affect the spleen is traumatic rupture and this is addressed elsewhere.
Splenomegaly
Enlargement of the spleen can commonly accompany a variety of haematological disorders. Of these, chronic conditions such as haemolytic anaemia and chronic leukaemia are often associated with splenomegaly. More acute haematological conditions tend not to cause massive splenic enlargement. Infections with parasites (such as malaria) and viruses such as Epstein Barr virus can cause splenomegaly and this one condition that is associated with sudden spontaneous splenic rupture. Portal hypertension can result in splenomegaly and the finding of an enlarged spleen on abdominal examination should prompt a search for the other stigmata of chronic liver disease.
Splenic atrophy
This typically occurs as a result of conditions that impair the blood flow to the spleen. Division of some of the major vessels can cause shrinkage of the spleen and this is commonly seen when splenic artery aneurysms are coiled for example.
Which of the following is not classically seen in coning resulting from raised intra cranial pressure?
Coma
Hypotension
Unreactive mid sized pupils
Cheyne Stokes style respiratory efforts
Bradycardia
Hypotension
Cushings triad
Widening of the pulse pressure
Respiratory changes
Bradycardia
Due to raised ICP systemic hypertension is usually seen. Compression of the respiratory centre will typically result in Cheyne Stokes style respiration.
Coning
The cranial vault is a confined cavity apart from infants with a non fused fontanelle.
Rises in ICP may be accommodated by shifts of CSF.
Once the CSF shifting has reached its capacity ICP will start to rise briskly.
The brain autoregulates its blood supply, as ICP rises the systemic circulation will display changes to try and meet the perfusion needs of the brain. Usually this will involve hypertension.
As ICP rises further, the brain will be compressed, cranial nerve palsies may be seen and compression of essential centres in the brain stem will occur. When the cardiac centre is involved bradycardia will often develop.
A 74 year old woman presents with a breast lump. On examination, it has a soft consistency. The lump is removed and sliced apart. Macroscopically there is a grey, gelatinous surface. Which of the following tumour types is most likely?
Sarcoma
Invasive ductal carcinoma
Mucinous carcinoma
DCIS
Lobular carcinoma
Mucinous carcinomas comprise 2-3% of all breast cancers. They are one of the special type of carcinomas. These have a better prognosis than is associated with tumours of Non Special Type (NST) and axillary nodal disease is rare in this group.
Breast cancer
- Commoner in the older age group
Invasive ductal carcinomas are the most common type. Some may arise as a result of ductal carcinoma in situ (DCIS). There are associated carcinomas of special type e.g. Tubular that may carry better prognosis.
The pathological assessment involves assessment of the tumour and lymph nodes, sentinel lymph node biopsy is often used to minimise the morbidity of an axillary dissection.
Treatment, typically this is either wide local excision or mastectomy. There are many sub types of both of these that fall outside of the MRCS. Some key rules to bear in mind.
Whatever operation is contemplated the final cosmetic outcome does have a bearing. A woman with small breasts and a large tumour will tend to fare better with mastectomy, even if clear pathological and clinical margins can be obtained. Conversely a women with larger breasts may be able to undergo breast conserving surgery even with a relatively large primary lesion (NB tumours >4cm used to attract recommendation for mastectomy). For screen detected and impalpable tumour image guidance will be necessary.
Reconstruction is always an option following any resectional procedure. However, its exact type must be tailored to age and co-morbidities of the patient. The main operations in common use include latissimus dorsi myocutaneous flap and sub pectoral implants. Women wishing to avoid a prosthesis may be offered TRAM or DIEP flaps.
Surgical options
Mastectomy vs Wide local excision
Mastectomy Wide Local Excision
Multifocal tumour Solitary lesion
Central tumour Peripheral tumour
Large lesion in small breast Small lesion in large breast
DCIS >4cm DCIS <4cm
Patient Choice Patient choice
Central lesions may be managed using breast conserving surgery where an acceptable cosmetic result may be obtained, this is rarely the case in small breasts
Whatever surgical option is chosen the aim should be to have a local recurrence rate of 5% or less at 5 years [1].
Nottingham Prognostic Index
The Nottingham Prognostic Index can be used to give an indication of survival. In this system the tumour size is weighted less heavily than other major prognostic parameters.
Calculation of NPI
Tumour Size x 0.2 + Lymph node score(From table below)+Grade score(From table below).
Score Lymph nodes involved Grade
1 0 1
2 1-3 2
3 >3 3
Prognosis
Score Percentage 5 year survival
2.0 to 2.4 93%
2.5 to 3.4 85%
3.5 to 5.4 70%
>5.4 50%
This data was originally published in 1992. It should be emphasised that other factors such as vascular invasion and receptor status also impact on survival and are not included in this data and account for varying prognoses often cited in the literature.
References
For guidance on how breast cancer is managed in the United Kingdom visit the Association of Breast Surgery website (www.https://associationofbreastsurgery.org.uk/).
A 28 year old man presents with a burn injury secondary to a house fire. He has sustained 25% mixed full and partial thickness burns to his torso and limbs. Which of the following resuscitation protocols would be most appropriate in this case?
Administration of 2 ml of Hartmanns x weight in kg x 25 to maintain a urine output of 30ml/ hour
Administration of 4 ml of Hartmanns x weight in kg x 25 to maintain a urine output of 100ml/ hour
Administration of 3 ml of Hartmanns x weight in kg x 25 to maintain a urine output of 70ml/ hour
Administration of 2 ml of 5% dextrose x weight in kg x 25 to maintain a urine output of 50ml/ hour
Administration of 2 ml of Dextrose / saline x weight in kg x 25 to maintain a urine output of 30ml/ hour
A
Lower infusion volumes are now administered to burn patients to minimize the potential harm caused by excessive fluid administration
Fluid resuscitation burns
Indication: >15% total body area burns in adults (>10% children)
The main aim of resuscitation is to prevent the burn deepening
Most fluid is lost 24 hours after injury
First 8-12 hour fluid shifts are from intravascular to interstitial fluid compartments
Therefore circulatory volume can be compromised. However fluid resuscitation causes more fluid into the interstitial compartment especially colloid (therefore avoided in first 8-24 hours)
Protein loss occurs
There are concerns that use of the traditional Parkland formula resulted in the administration of excessive quantities of intravenous fluids in patients with burns. The current consensus guidelines state that fluid resuscitation should begin at 2 ml of lactated Ringers x patients body weight in kg x % TBSA for second- and third-degree burns.
The calculated fluid volume is initiated in the following manner: one-half of the total fluid is provided in the first 8 hours after the burn injury (for example, a 100-kg man with 80% TBSA burns requires 2 × 80 × 100 = 16,000 mL in 24 hours). One-half of that volume (8,000 mL) should be provided in the first 8 hours, so the patient should be started at a rate of 1000 mL/hr. The remaining one-half of the total fluid is administered during the subsequent 16 hours.
The efficacy of fluid replacement is determined in the main by monitoring urine output. For adults this is in the region of 0.5ml/kg/hr and for children less than 30Kg it is 1ml/kg/hr.
Fluid resuscitation according to burn type
Type of burn Age or weight Fluid rate Urine output
Flames or scalding Adults and children over 14 years 2 ml Hartmanns x kg x % TBSA 0.3-0.5ml/kg/hr OR 30-50ml per hour
Flames or scalding Children less than 14 years 3 ml Hartmanns x kg x % TBSA 1ml/kg/hr
Flames or scalds Children less than 30Kg and infants 3 ml Hartmanns x kg x % TBSA. Plus a sugar-containing solution at maintenance rate 1ml/kg/hr
Electrical injury All ages 4 ml Hartmanns x kg x % TBSA until urine clears 1-1.5 ml/kg/hr until urine clears
Fluid resuscitation in electrical burn patients is different because they have far greater soft tissue involvement in muscle compartments and associated muscle death that can result in myoglobinuria.
After 24 hours
Maintenance crystalloid (usually dextrose-saline) is continued at a rate of 1.5 ml x(burn area)x(body weight)
Colloids are rarely used (e.g. albumin)
Antioxidants, such as vitamin C, can be used to minimize oxidant-mediated contributions to the inflammatory cascade in burns
High tension electrical injuries and inhalation injuries require more fluid
Monitor: packed cell volume, plasma sodium, base excess, and lactate
Reference
ATLS Manual 10th Edition.
A 53 year old lady undergoes a mastectomy to remove a breast cancer. Microscopic analysis of the tumour shows a pronounced lymphocytic infiltrate. Which of the tumour types listed below is most often associated with this finding?
Invasive ductal carcinoma
Tubular breast cancer
Medullary breast cancer
Mucinous breast cancer
Adenoid cystic carcinoma of the breast
Medullary breast cancer is a breast cancer of special type. Histologically, it is characterised by a marked lymphocytic infiltrate. Many of the breast cancers of special type (which comprise less than 5% of all breast cancers) have a very good prognosis. These special type cancers include; medullary, tubular, mucinous, adenoid cystic,
Breast cancer
- Commoner in the older age group
Invasive ductal carcinomas are the most common type. Some may arise as a result of ductal carcinoma in situ (DCIS). There are associated carcinomas of special type e.g. Tubular that may carry better prognosis.
The pathological assessment involves assessment of the tumour and lymph nodes, sentinel lymph node biopsy is often used to minimise the morbidity of an axillary dissection.
Treatment, typically this is either wide local excision or mastectomy. There are many sub types of both of these that fall outside of the MRCS. Some key rules to bear in mind.
Whatever operation is contemplated the final cosmetic outcome does have a bearing. A woman with small breasts and a large tumour will tend to fare better with mastectomy, even if clear pathological and clinical margins can be obtained. Conversely a women with larger breasts may be able to undergo breast conserving surgery even with a relatively large primary lesion (NB tumours >4cm used to attract recommendation for mastectomy). For screen detected and impalpable tumour image guidance will be necessary.
Reconstruction is always an option following any resectional procedure. However, its exact type must be tailored to age and co-morbidities of the patient. The main operations in common use include latissimus dorsi myocutaneous flap and sub pectoral implants. Women wishing to avoid a prosthesis may be offered TRAM or DIEP flaps.
Surgical options
Mastectomy vs Wide local excision
Mastectomy Wide Local Excision
Multifocal tumour Solitary lesion
Central tumour Peripheral tumour
Large lesion in small breast Small lesion in large breast
DCIS >4cm DCIS <4cm
Patient Choice Patient choice
Central lesions may be managed using breast conserving surgery where an acceptable cosmetic result may be obtained, this is rarely the case in small breasts
A compelling indication for mastectomy, a larger tumour that would be unsuitable for breast conserving surgery
Whatever surgical option is chosen the aim should be to have a local recurrence rate of 5% or less at 5 years [1].
Nottingham Prognostic Index
The Nottingham Prognostic Index can be used to give an indication of survival. In this system the tumour size is weighted less heavily than other major prognostic parameters.
Calculation of NPI
Tumour Size x 0.2 + Lymph node score(From table below)+Grade score(From table below).
Score Lymph nodes involved Grade
1 0 1
2 1-3 2
3 >3 3
Prognosis
Score Percentage 5 year survival
2.0 to 2.4 93%
2.5 to 3.4 85%
3.5 to 5.4 70%
>5.4 50%
This data was originally published in 1992. It should be emphasised that other factors such as vascular invasion and receptor status also impact on survival and are not included in this data and account for varying prognoses often cited in the literature.
References
For guidance on how breast cancer is managed in the United Kingdom visit the Association of Breast Surgery website (www.https://associationofbreastsurgery.org.uk/).
During a carotid endarterectomy the internal carotid artery is cross clamped. Assuming that no shunt is inserted, which of the following vessels will not have diminished or absent flow as a result?
Anterior cerebral artery
Ophthalmic artery
Middle cerebral artery
Maxillary artery
None of the above
Mnemonic for branches of the cerebral portion of the internal carotid artery ‘Only Press Carotid Arteries Momentarily’
Only = Opthalmic
Press = Posterior communicating
Carotid = Choroidal
Arteries = Anterior cerebral
Momentarily = Middle cerebral
Internal carotid artery
The internal carotid artery is formed from the common carotid opposite the upper border of the thyroid cartilage. It extends superiorly to enter the skull via the carotid canal. From the carotid canal it then passes through the cavernous sinus, above which it divides into the anterior and middle cerebral arteries.
Relations in the neck
Posterior
Longus capitis
Pre-vertebral fascia
Sympathetic chain
Superior laryngeal nerve
Medially
External carotid (near origin)
Wall of pharynx
Ascending pharyngeal artery
Laterally
Internal jugular vein (moves posteriorly at entrance to skull)
Vagus nerve (most posterolaterally)
Anteriorly
Sternocleidomastoid
Lingual and facial veins
Hypoglossal nerve
Relations in the carotid canal
Internal carotid plexus
Cochlea and middle ear cavity
Trigeminal ganglion (superiorly)
Leaves canal lies above the foramen lacerum
Path and relations in the cranial cavity
The artery bends sharply forwards in the cavernous sinus, the aducens nerve lies close to its inferolateral aspect. The oculomotor, trochlear, opthalmic and, usually, the maxillary nerves lie in the lateral wall of the sinus. Near the superior orbital fissure it turns posteriorly and passes postero-medially to pierce the roof of the cavernous sinus inferior to the optic nerve. It then passes between the optic and oculomotor nerves to terminate below the anterior perforated substance by dividing into the anterior and middle cerebral arteries.
Branches
Anterior and middle cerebral artery
Ophthalmic artery
Posterior communicating artery
Anterior choroid artery
Meningeal arteries
Hypophyseal arteries
What is the typical stroke volume in a resting 70 Kg man?
10ml
150ml
125ml
45ml
70ml
Stroke volumes range from 55-100ml. Stroke volume-Cardiac physiology
The stroke volume equates to the volume of blood ejected from the ventricle during each cycle of cardiac contraction. The volumes for both ventricles are typically equal and equate roughly to 70ml for a 70Kg man. It is calculated by subtracting the end systolic volume from the end diastolic volume.
Factors affecting stroke volume
Cardiac size
Contractility
Preload
Afterload
Which of these nerves passes through the greater and lesser sciatic foramina?
Pudendal nerve
Sciatic nerve
Superior gluteal nerve
Inferior gluteal nerve
Posterior cutaneous nerve of the thigh
Structures passing through the lesser and greater sciatic foramina (medial to lateral): PIN
Pudendal nerve
Internal pudendal artery
Nerve to obturator internus
The pudendal nerve originates from the ventral rami of the second, third, and fourth sacral nerves (S2, S3, S4).
It passes between the piriformis and coccygeus muscles and exits the pelvis through the the greater sciatic foramen. It crosses the spine of the ischium and reenters the pelvis through the lesser sciatic foramen. It passes through the pudendal canal.
The pudendal nerve gives off the inferior rectal nerves. It terminates into 2 branches: perineal nerve, and the dorsal nerve of the penis or the dorsal nerve of the clitoris.
A 45 year man presents with hand weakness. He is given a piece of paper to hold between his thumb and index finger. When the paper is pulled, the patient has difficulty maintaining a grip. Grip pressure is maintained by flexing the thumb at the interphalangeal joint. What is the most likely nerve lesion?
Posterior interosseous nerve
Deep branch of ulnar nerve
Anterior interosseous nerve
Superficial branch of the ulnar nerve
Radial nerve
This is a description of Froment’s sign, which tests for ulnar nerve palsy. It mainly tests for the function of adductor pollicis. This is supplied by the deep branch of the ulnar nerve. Remember the anterior interosseous branch (of the median nerve), which innervates the flexor pollicis longus (hence causing flexion of the thumb IP joint), branches off more proximally to the wrist.
Ulnar nerve
Origin
C8, T1
Supplies (no muscles in the upper arm)
Flexor carpi ulnaris
Flexor digitorum profundus
Flexor digiti minimi
Abductor digiti minimi
Opponens digiti minimi
Adductor pollicis
Interossei muscle
Third and fourth lumbricals
Palmaris brevis
Path
Posteromedial aspect of upper arm to flexor compartment of forearm, then along the ulnar. Passes beneath the flexor carpi ulnaris muscle, then superficially over the flexor retinaculum into the palm of the hand.
Branches
Branch Supplies
Muscular branch Flexor carpi ulnaris
Medial half of the flexor digitorum profundus
Palmar cutaneous branch (Arises near the middle of the forearm) Skin on the medial part of the palm
Dorsal cutaneous branch Dorsal surface of the medial part of the hand
Superficial branch Cutaneous fibres to the anterior surfaces of the medial one and one-half digits
Deep branch Hypothenar muscles
All the interosseous muscles
Third and fourth lumbricals
Adductor pollicis
Medial head of the flexor pollicis brevis
Effects of injury
Damage at the wrist
Wasting and paralysis of intrinsic hand muscles (claw hand)
Wasting and paralysis of hypothenar muscles
Loss of sensation medial 1 and half fingers
Damage at the elbow
Radial deviation of the wrist
Clawing less in 4th and 5th digits
A 48 year old lady is admitted with abdominal distension. On examination, she is cachectic and has ascites. Her CA19-9 returns highly elevated. What is the most likely cause?
Metastatic ovarian cancer
Metastatic pancreatic cancer
Metastatic gastric cancer
Metastatic colonic cancer
Pseudomyxoma peritoneii
Although not specific CA 19-9 in the context of this history is highly suggestive of pancreatic cancer over the other scenarios.
Pancreatic cancer
- Adenocarcinoma
Risk factors: Smoking, diabetes, adenoma, familial adenomatous polyposis
Mainly occur in the head of the pancreas (70%)
Spread locally and metastasizes to the liver
Carcinoma of the pancreas should be differentiated from other periampullary tumours with better prognosis
Clinical features
Weight loss
Painless jaundice
Epigastric discomfort (pain usually due to invasion of the coeliac plexus is a late feature)
Pancreatitis
Trousseau’s sign: migratory superficial thrombophlebitis
Investigations
USS: May miss small lesions
CT Scanning (pancreatic protocol). If unresectable on CT then no further staging needed
PET/CT for those with operable disease on CT alone
ERCP/ MRI for bile duct assessment
Staging laparoscopy to exclude peritoneal disease
Management
Head of pancreas: Whipple’s resection (SE dumping and ulcers). Newer techniques include pylorus preservation and SMA/ SMV resection
Carcinoma body and tail: poor prognosis, distal pancreatectomy, if operable
Usually adjuvent chemotherapy for resectable disease
ERCP and stent for jaundice and palliation
Surgical bypass may be needed for duodenal obstruction
A term neonate is undergoing an open inguinal herniotomy. During the operation, the operating surgeon identifies a small round yellow nodule within the inguinal canal. What is this most likely to represent?
Ectopic lymphatic tissue
Adrenal rest
Encysted hydrocele of the cord
Yolk sac tumour
Hamartoma
Adrenal rests are a very common finding in term neonates (50%) and most regress so that by adulthood the population incidence is 1%. The adrenal glands and gonads both develop from the urogenital ridge. In early embryogenesis, the adrenal cortical tissue can migrate with the descending gonads.
Adrenal gland embryology
First detected at 6 weeks’ gestation, the adrenal cortex is derived from the mesoderm of the posterior abdominal wall. Steroid secretion from the fetal cortex begins shortly thereafter. Adult-type zona glomerulosa and fasciculata are detected in fetal life but make up only a small proportion of the gland, and the zona reticularis is not present at all. The fetal cortex predominates throughout fetal life. The adrenal medulla is of ectodermal origin, arising from neural crest cells that migrate to the medial aspect of the developing cortex.
The fetal adrenal gland is relatively large. At 4 months’ gestation, it is 4 times the size of the kidney; however, at birth, it is a third of the size of the kidney. This occurs because of the rapid regression of the fetal cortex at birth. It disappears almost completely by age 1 year; by age 4-5 years, the permanent adult-type adrenal cortex has fully developed.
Anatomic anomalies of the adrenal gland may occur. Because the development of the adrenals is closely associated with that of the kidneys, agenesis of an adrenal gland is usually associated with ipsilateral agenesis of the kidney, and fused adrenal glands (whereby the 2 glands join across the midline posterior to the aorta) are also associated with a fused kidney.
Adrenal hypoplasia occurs in the following 2 forms: (1) hypoplasia or absence of the fetal cortex with a poorly formed medulla and (2) disorganized fetal cortex and medulla with no permanent cortex present. Adrenal heterotopia describes a normal adrenal gland in an abnormal location, such as within the renal or hepatic capsules. Accessory adrenal tissue (adrenal rests), which is usually comprised only of cortex but seen combined with medulla in some cases, is most commonly located in the broad ligament or spermatic cord but can be found anywhere within the abdomen. Even intracranial adrenal rests have been reported.
What type of stoma is most likely to be encountered in a 56 year old man undergoing a low anterior resection for carcinoma of the rectum with a colorectal anastomosis?
Loop colostomy
End colostomy
End ileostomy
Loop ileostomy
Caecostomy
Colonic resections with an anastomosis below the peritoneal reflection may have an anastomotic leak rate (both clinical and radiological) of up to 15%. Therefore most surgeons will defunction such an anastomosis to reduce the clinical severity of an anastomotic leak. A loop ileostomy will achieve this end point and is relatively easy to reverse. Loop colostomy is less popular in this setting as reversal can compromise the blood supply to the anastomosis.
Abdominal stomas
Stomas may be sited during a range of abdominal procedures and involve bringing the lumen or visceral contents onto the skin. In most cases this applies to the bowel. However, other organs or their contents may be diverted in case of need.
With bowel stomas the type method of construction and to a lesser extent the site will be determined by the contents of the bowel. In practice, small bowel stomas should be spouted so that their irritant contents are not in contact with the skin. Colonic stomas do not need to be spouted as their contents are less irritant.
In the ideal situation the site of the stoma should be marked with the patient prior to surgery. Stoma siting is important as it will ultimately influence the ability of the patient to manage their stoma and also reduce the risk of leakage. Leakage of stoma contents and subsequent maceration of the surrounding skin can rapidly progress into a spiraling loss of control of stoma contents.
Types of stomas
Name of stoma Use Common sites
Gastrostomy
Gastric decompression or fixation
Feeding
Epigastrium
Loop jejunostomy
Seldom used as very high output
May be used following emergency laparotomy with planned early closure
Any location according to need
Percutaneous jejunostomy
Usually performed for feeding purposes and site in the proximal bowel
Usually left upper quadrant
Loop ileostomy
Defunctioning of colon e.g. following rectal cancer surgery
Does not decompress colon (if ileocaecal valve competent)
Usually right iliac fossa
End ileostomy
Usually following complete excision of colon or where ileo-colic anastomosis is not planned
May be used to defunction colon, but reversal is more difficult
Usually right iliac fossa
End colostomy Where a colon is diverted or resected and anastomosis is not primarily achievable or desirable Either left or right iliac fossa
Loop colostomy
To defunction a distal segment of colon
Since both lumens are present the distal lumen acts as a vent
May be located in any region of the abdomen, depending upon colonic segment used
Caecostomy Stoma of last resort where loop colostomy is not possible Right iliac fossa
Mucous fistula
To decompress a distal segment of bowel following colonic division or resection
Where closure of a distal resection margin is not safe or achievable
May be located in any region of the abdomen according to clinical need
Which of the following drugs does not interfere with the laboratory analysis of serum cortisol levels?
Dexamethasone
Prednisolone
Hydrocortisone IV
Hydrocortisone PO
Hydrocortisone IM
Prednisolone and it’s metabolites can chemically mimic cortisol in radio-immunoassay techniques of laboratory analysis.
Dexamethasone can be given as glucorticoid replacement during testing for addisons or adrenal insufficiency as it does not interfere with cortisol levels. For example, if you have a patient with polymyalgia rheumatica and they are on long term prednisolone, you can replace the prednisolone with dexamethasone to undertake a short synacthen test.
Cortisol
Glucocorticoid
Released by zona fasiculata of the adrenal gland
90% protein bound; 10% active
Circadian rhythm: High in the mornings
Negative feedback via ACTH
Actions
Glycogenolysis
Gluconeogenesis
Protein catabolism
Lipolysis
Stress response
Anti-inflammatory
Decrease protein in bones
Increase gastric acid
Increases neutrophils/platelets/red blood cells
Inhibits fibroblastic activity
A 67 year old man undergoes a carotid endarterectomy and seems to recover well following surgery. When he is reviewed on the ward post operatively he complains that his voice is hoarse. What is the most likely cause?
Damage to the accessory nerve
Damage to the cervical plexus
Damage to the glossopharyngeal nerve
Damage to the hypoglossal nerve
Damage to the vagus
Many of these nerves are at risk of injury during carotid surgery. However, only damage to the vagus would account for a hoarse voice.
Vagus nerve
The vagus nerve has mixed functions and supplies the structures from the fourth and sixth pharyngeal arches. It also supplies the fore and midgut sections of the embryonic gut tube. It carries afferent fibres from these areas (viz; pharynx, larynx, oesophagus, stomach, lungs, heart and great vessels). The efferent fibres of the vagus are of two main types. The first are preganglionic parasympathetic fibres distributed to the parasympathetic ganglia that innervate smooth muscle of the innervated organs (such as gut). The second type of efferent fibres have direct skeletal muscle innervation, these are largely to the muscles of the larynx and pharynx.
Origin and course
The vagus arises from the lateral surface of the medulla oblongata by a series of rootlets. It is related to the glossopharyngeal nerve cranially and the accessory nerve caudally. It exits through the jugular foramen and is contained within its own dural sheath alongside the accessory nerve. In the neck it descends vertically in the carotid sheath where it is closely related to the internal and common carotid arteries. It leaves the neck and enters the mediastinum. On the right it passes anterior to the first part of the subclavian artery, on the left it lies in the interval between the common carotid and subclavian arteries.
In the mediastinum both nerves pass postero-inferiorly and reach the posterior surface of the corresponding lung root. These then branch into both lungs. At the inferior end of the mediastinum these plexuses reunite to form the formal vagal trunks that pass through the oesophageal hiatus and into the abdomen. The anterior and posterior vagal trunks are formal nerve fibres these then splay out once again sending fibres over the stomach and posteriorly to the coeliac plexus. Branches pass to the liver, spleen and kidney.
Communications and branches
Communication Details
Superior ganglion Located in jugular foramen
Communicates with the superior cervical sympathetic ganglion, accessory nerve
Two branches; meningeal and auricular (the latter may give rise to vagal stimulation following instrumentation of the external auditory meatus)
Inferior ganglion Communicates with the superior cervical sympathetic ganglion, hypoglossal nerve and loop between first and second cervical ventral rami
Two branches; pharyngeal (supplies pharyngeal muscles) and superior laryngeal nerve (inferomedially- deep to both carotid arteries)
Branches in the neck
Branch Detail
Superior and inferior cervical cardiac branches Arise at various points and descend into thorax
On the right these pass posterior to the subclavian artery
On the left the superior branch passes between the arch of the aorta and the trachea to connect with the deep cardiac plexus. The inferior branch descends with the vagus itself.
Right recurrent laryngeal nerve Arises from vagus anterior to the first part of the subclavian artery, hooks under it, and ascends superomedially. It passes close to the common carotid and finally the inferior thyroid artery to insert into the larynx
Branches in the thorax
Branch Details
Left recurrent laryngeal nerve Arises from the vagus on the aortic arch. It hooks around the inferior surface of the arch, posterior to the ligamentum arteriosum and passes upwards through the superior mediastinum and lower part of the neck. It lies in the groove between oesophagus and trachea (supplies both). It passes with the inferior thyroid artery and inserts into the larynx.
Thoracic and cardiac branches There are extensive branches to both the heart and lung roots. These pass throughout both these viscera. The fibres reunite distally prior to passing into the abdomen.
Abdominal branches
After entry into the abdominal cavity the nerves branch extensively. In previous years the extensive network of the distal branches (nerves of Laterjet) over the surface of the distal stomach were important for the operation of highly selective vagotomy. The use of modern PPI’s has reduced the need for such highly selective procedures. Branches pass to the coeliac axis and alongside the vessels to supply the spleen, liver and kidney.
Which of the following is not a content of the posterior triangle of the neck?
Spinal accessory nerve
Phrenic nerve
External jugular vein
Occipital lymph nodes
Internal jugular vein
The IJV does not lie in the posterior triangle. However, the terminal branches of the external jugular vein do.
Posterior triangle of the neck
Boundaries
Apex Sternocleidomastoid and the Trapezius muscles at the Occipital bone
Anterior Posterior border of the Sternocleidomastoid
Posterior Anterior border of the Trapezius
Base Middle third of the clavicle
Contents
Nerves
Accessory nerve
Phrenic nerve
Three trunks of the brachial plexus
Branches of the cervical plexus: Supraclavicular nerve, transverse cervical nerve, great auricular nerve, lesser occipital nerve
Vessels
External jugular vein
Subclavian artery
Muscles
Inferior belly of omohyoid
Scalene
Lymph nodes
Supraclavicular
Occipital
A 40-year-old man presents with pain in his lower back and ‘sciatica’ for the past three days. He describes bending down to pick up a washing machine when he felt ‘something go’. He now has severe pain radiating from his back down the right leg. On examination, there is paraesthesia over the anterior aspect of the right knee and the medial aspect of his lower leg. Power is intact and the right knee reflex is diminished. The femoral stretch test is positive on the right side. Which nerve or nerve root is most likely to be affected?
Common peroneal nerve
Lateral cutaneous nerve of the thigh
L5
L3
L1
L3
Prolapsed disc
A prolapsed lumbar disc usually produces clear dermatomal leg pain associated with neurological deficits.
Features
Leg pain usually worse than back
Pain often worse when sitting
The table below demonstrates the expected features according to the level of compression:
L3 nerve root compression Sensory loss over anterior thigh/knee
Weak quadriceps
Reduced knee reflex
Positive femoral stretch test
L5 nerve root compression Sensory loss dorsum of foot
Weakness in foot and big toe dorsiflexion
Reflexes intact
Positive sciatic nerve stretch test
S1 nerve root compression Sensory loss posterolateral aspect of leg and lateral aspect of foot
Weakness in plantar flexion of foot
Reduced ankle reflex
Positive sciatic nerve stretch test
Management
Similar to that of other musculoskeletal lower back pain: analgesia, physiotherapy, exercises
Persistent symptoms, muscular weakness, bladder or bowel dysfunction are indications for urgent MRI scanning to delineate the disease extent to allow surgical planning
Plain spinal x-rays have no useful role in establishing the extent of disc disease
The oesophagus is constricted at the following levels apart from:
Cricoid cartilage
Arch of the aorta
Lower oesophageal sphincter
Left main stem bronchus
Diaphragmatic hiatus
Constrictions of the oesophagus : ABCD
A- Arch of the Aorta
B- Left main Bronchus
C- Cricoid Cartilage
D- Diaphragmatic Hiatus
The oesophagus is not constricted at the level of the lower oesophageal sphincter.
Oesophagus
- 25cm long
Starts at C6 vertebra, pierces diaphragm at T10 and ends at T11
Squamous epithelium
Constrictions of the oesophagus
Structure Distance from incisors
Cricoid cartilage 15cm
Arch of the Aorta 22.5cm
Left principal bronchus 27cm
Diaphragmatic hiatus 40cm
Relations
Anteriorly
Trachea to T4
Recurrent laryngeal nerve
Left bronchus, Left atrium
Diaphragm
Posteriorly
Thoracic duct to left at T5
Hemiazygos to the left T8
Descending aorta
First 2 intercostal branches of aorta
Left
Thoracic duct
Left subclavian artery
Right
Azygos vein
Arterial, venous and lymphatic drainage of the oesophagus
Artery Vein Lymphatics Muscularis externa
Upper third Inferior thyroid Inferior thyroid Deep cervical Striated muscle
Mid third Aortic branches Azygos branches Mediastinal Smooth & striated muscle
Lower third Left gastric Left gastric Gastric Smooth muscle
Nerve supply
Upper half is supplied by recurrent laryngeal nerve
Lower half by oesophageal plexus (vagus)
Histology
Mucosa :Non-keratinized stratified squamous epithelium
Submucosa: glandular tissue
Muscularis externa (muscularis): composition varies. See table
Adventitia
A 43 year old lady is recovering on the intensive care unit following a Whipples procedure. She has a central venous line in situ. Which of the following will lead to the y descent on the waveform trace ?
Ventricular contraction
Emptying of the right atrium
Emptying of the right ventricle
Opening of the pulmonary valve
Cardiac tamponade
JVP
3 Upward deflections and 2 downward deflections
Upward deflections
a wave = atrial contraction
c wave = ventricular contraction
v wave = atrial venous filling
Downward deflections
x wave = atrium relaxes and tricuspid valve moves down
y wave = ventricular filling
The y descent represents the emptying of the atrium and the filling of the right ventricle.
Cardiac physiology
- The heart has four chambers ejecting blood into both low pressure and high pressure systems.
The pumps generate pressures of between 0-25mmHg on the right side and 0-120 mmHg on the left.
At rest diastole comprises 2/3 of the cardiac cycle.
The product of the frequency of heart rate and stroke volume combine to give the cardiac output which is typically 5-6L per minute.
Detailed descriptions of the various waveforms are often not a feature of MRCS A (although they are on the syllabus). However, they are a very popular topic for surgical physiology in the MRCS B exam.
Electrical properties
Intrinsic myogenic rhythm within cardiac myocytes means that even the denervated heart is capable of contraction.
In the normal situation the cardiac impulse is generated in the sino atrial node in the right atrium and conveyed to the ventricles via the atrioventricular node.
The sino atrial node is also capable of spontaneous discharge and in the absence of background vagal tone will typically discharge around 100x per minute. Hence the higher resting heart rate found in cardiac transplant cases. In the SA and AV nodes the resting membrane potential is lower than in surrounding cardiac cells and will slowly depolarise from -70mV to around -50mV at which point an action potential is generated.
Differences in the depolarisation slopes between SA and AV nodes help to explain why the SA node will depolarise first. The cells have a refractory period during which they cannot be re-stimulated and this period allows for adequate ventricular filling. In pathological tachycardic states this time period is overridden and inadequate ventricular filling may then occur, cardiac output falls and syncope may ensue.
Parasympathetic fibres project to the heart via the vagus and will release acetylcholine. Sympathetic fibres release nor adrenaline and circulating adrenaline comes from the adrenal medulla. Noradrenaline binds to β 1 receptors in the SA node and increases the rate of pacemaker potential depolarisation.
Cardiac cycle
Mid diastole: AV valves open. Ventricles hold 80% of final volume. Outflow valves shut. Aortic pressure is high.
Late diastole: Atria contract. Ventricles receive 20% to complete filling. Typical end diastolic volume 130-160ml.
Early systole: AV valves shut. Ventricular pressure rises. Isovolumetric ventricular contraction. AV Valves bulge into atria (c-wave). Aortic and pulmonary pressure exceeded- blood is ejected. Shortening of ventricles pulls atria downwards and drops intra atrial pressure (x-descent).
Late systole: Ventricular muscles relax and ventricular pressures drop. Although ventricular pressure drops the aortic pressure remains constant owing to peripheral vascular resistance and elastic property of the aorta. Brief period of retrograde flow that occurs in aortic recoil shuts the aortic valve. Ventricles will contain 60ml end systolic volume. The average stroke volume is 70ml (i.e. Volume ejected).
Early diastole: All valves are closed. Isovolumetric ventricular relaxation occurs. Pressure wave associated with closure of the aortic valve increases aortic pressure. The pressure dip before this rise can be seen on arterial waveforms and is called the incisura. During systole the atrial pressure increases such that it is now above zero (v- wave). Eventually atrial pressure exceed ventricular pressure and AV valves open - atria empty passively into ventricles and atrial pressure falls (y -descent )
The negative atrial pressures are of clinical importance as they can allow air embolization to occur if the neck veins are exposed to air. This patient positioning is important in head and neck surgery to avoid this occurrence if veins are inadvertently cut, or during CVP line insertion.
Mechanical properties
Preload = end diastolic volume
Afterload = aortic pressure
It is important to understand the principles of Laplace’s law in surgery.
It states that for hollow organs with a circular cross section, the total circumferential wall tension depends upon the circumference of the wall, multiplied by the thickness of the wall and on the wall tension.
The total luminal pressure depends upon the cross sectional area of the lumen and the transmural pressure. Transmural pressure is the internal pressure minus external pressure and at equilibrium the total pressure must counterbalance each other.
In terms of cardiac physiology the law explains that the rise in ventricular pressure that occurs during the ejection phase is due to physical change in heart size. It also explains why a dilated diseased heart will have impaired systolic function.
Starlings law
Increase in end diastolic volume will produce larger stroke volume.
This occurs up to a point beyond which cardiac fibres are excessively stretched and stroke volume will fall once more. It is important for the regulation of cardiac output in cardiac transplant patients who need to increase their cardiac output.
Baroreceptor reflexes
Baroreceptors located in aortic arch and carotid sinus.
Aortic baroreceptor impulses travel via the vagus and from the carotid via the glossopharyngeal nerve.
They are stimulated by arterial stretch.
Even at normal blood pressures they are tonically active.
Increase in baroreceptor discharge causes:
*Increased parasympathetic discharge to the SA node.
*Decreased sympathetic discharge to ventricular muscle causing decreased contractility and fall in stroke volume.
*Decreased sympathetic discharge to venous system causing increased compliance.
*Decreased peripheral arterial vascular resistance
Atrial stretch receptors
Located in atria at junction between pulmonary veins and vena cava.
Stimulated by atrial stretch and are thus low pressure sensors.
Increased blood volume will cause increased parasympathetic activity.
Very rapid infusion of blood will result in increase in heart rate mediated via atrial receptors: the Bainbridge reflex.
Decreases in receptor stimulation results in increased sympathetic activity this will decrease renal blood flow-decreases GFR-decreases urinary sodium excretion-renin secretion by juxtaglomerular apparatus-Increase in angiotensin II.
Increased atrial stretch will also result in increased release of atrial natriuretic peptide.
A 12 year old male is investigated for swallowing difficulties and is found to have dysphagia lusoria. Which of the diagnostic modalities described below is most likely to demonstrate the cause?
CT angiogram
Upper GI endoscopy
Barium swallow
Capsule endoscopy
Oesophageal manometry
Dysphagia lusoria is caused by compression of the esophagus from any of several congenital vascular abnormalities. The vascular abnormality is usually an aberrant right subclavian artery arising from the left side of the aortic arch, a double aortic arch, or a right aortic arch with left ligamentum arteriosum. The dysphagia may develop in childhood or later in life as a result of arteriosclerotic changes in the aberrant vessel.
Barium swallow shows the extrinsic compression, but arteriography is necessary for absolute diagnosis.
Dysphagia
Causes of dysphagia
Extrinsic
Mediastinal masses
Cervical spondylosis
Oesophageal wall
Achalasia
Diffuse oesophageal spasm
Hypertensive lower oesophageal sphincter
Intrinsic
Tumours
Strictures
Oesophageal web
Schatzki rings
Neurological
CVA
Parkinson’s disease
Multiple Sclerosis
Brainstem pathology
Myasthenia Gravis
Investigation
All patients require an upper GI endoscopy unless there are compelling reasons for this not to be performed. Motility disorders may be best appreciated by undertaking fluoroscopic swallowing studies.
A full blood count should be performed.
Ambulatory oesophageal pH and manometry studies will be required to evaluate conditions such as achalasia and patients with GORD being considered for fundoplication surgery.
A patient is brought to the emergency department following a motor vehicle accident. He is unconscious and has a deep scalp laceration. His heart rate is 120/min, blood pressure is 80/40 mmHg, and respiratory rate is 35/min. Despite rapid administration of 2 litres of Hartmans solution, the patient’s vital signs do not change significantly. The injury likely to explain this patient’s hypotension is:
Epidural haematoma
Sub dural haematoma
Intra parenchymal brain haemorrhage
Base of skull fracture
None of the above
Raised intracranial pressure (Cushing response)
Hypertension
Bradycardia
Respiratory depression
In the patient described, hypotension and tachycardia should not be uncritically attributed to the head injury, since these findings in the setting of blunt trauma are suggestive of serious thoracic, abdominal, or pelvic hemorrhage. When cardiovascular collapse occurs as a result of rising intracranial pressure, it is generally accompanied by hypertension, bradycardia, and respiratory depression.
Which of the structures listed below accompanies the aorta as it traverses the aortic hiatus?
Oesophagus
Thoracic duct
Vagal trunks
Right phrenic nerve
Left phrenic nerve
The aorta is accompanied by the thoracic duct as it traverses the aortic hiatus. The vagal trunks accompany the oesophagus which passes through the muscular part of the diaphragm on the right. The right phrenic nerve accompanies the IVC as it passes through the caval opening. The left phrenic nerve passes through the muscular part of the diaphragm anterior to the central tendon on the left.
Abdominal aorta
Abdominal aortic topography
Origin T12
Termination L4
Posterior relations L1-L4 Vertebral bodies
Anterior relations Lesser omentum
Liver
Left renal vein
Inferior mesenteric vein
Third part of duodenum
Pancreas
Parietal peritoneum
Peritoneal cavity
Right lateral relations Right crus of the diaphragm
Cisterna chyli
IVC (becomes posterior distally)
Left lateral relations 4th part of duodenum
Duodenal-jejunal flexure
Left sympathetic trunk
A 48 year old lady with end stage renal failure receives a cadaveric renal transplant. The organ is ABO group matched only. On completion of the vascular anastomoses the surgeons remove the clamps. Over the course of the next twelve minutes the donated kidney becomes dusky and swollen and appears non viable. Which of the following is the most likely process that has caused this event?
IgG anti HLA Class I antibodies in the recipient
IgM anti HLA Class I antibodies in the recipient
IgG anti HLA Class I antibodies from the donor
IgM anti HLA Class I antibodies from the donor
IgM anti HLA Class II antibodies from the recipient
Episodes of hyperacute rejection are typically due to preformed antibodies. ABO mismatch is the best example. However, IgG anti HLA Class I antibodies are another potential cause. These events are now seen less commonly because the cross matching process generally takes this possibility into account.
Organ Transplant
A number of different organ and tissue transplants are now available. In many cases an allograft is performed, where an organ is transplanted from one individual to another. Allografts will elicit an immune response and this is one of the main reasons for organ rejection.
Graft rejection occurs because allografts have allelic differences at genes that code immunohistocompatability complex genes. The main antigens that give rise to rejection are:
ABO blood group
Human leucocyte antigens (HLA)
Minor histocompatability antigens
ABO Matching
ABO incompatibility will result in early organ rejection (hyperacute) because of pre existing antibodies to other groups. Group O donors can give organs to any type of ABO recipient whereas group AB donor can only donate to AB recipient.
HLA System
The four most important HLA alleles are:
HLA A
HLA B
HLA C
HLA DR
An ideal organ match would be one in which all 8 alleles are matched (remember 2 from each parent, four each = 8 alleles). Modern immunosuppressive regimes help to manage the potential rejection due to HLA mismatching. However, the greater the number of mismatches the worse the long term outcome will be. T lymphocytes will recognise antigens bound to HLA molecules and will then become activated. Clonal expansion then occurs with a response directed against that antigen.
Types of organ rejection
Hyperacute. This occurs immediately through presence of pre formed antibodies (such as ABO incompatibility).
Acute. Occurs during the first 6 months and is usually T cell mediated. Usually tissue infiltrates and vascular lesions.
Chronic. Occurs after the first 6 months. Vascular changes predominate.
Hyperacute
Renal transplants at greatest risk and liver transplants at least risk. Although ABO incompatibility and HLA Class I incompatible transplants will all fare worse in long term.
Acute
All organs may undergo acute rejection. Mononuclear cell infiltrates predominate. All types of transplanted organ are susceptible and it may occur in up to 50% cases.
Chronic
Again all transplants with HLA mismatch may suffer this fate. Previous acute rejections and other immunosensitising events all increase the risk. Vascular changes are most prominent with myointimal proliferation leading to organ ischaemia. Organ specific changes are also seen such as loss of acinar cells in pancreas transplants and rapidly progressive coronary artery disease in cardiac transplants.
Surgical overview-Renal transplantation
A brief overview of the steps involved in renal transplantation is given.
Patients with end stage renal failure who are dialysis dependent or likely to become so in the immediate future are considered for transplant. Exclusion criteria include; active malignancy, old age (due to limited organ availability). Patients are medically optimised.
Donor kidneys, these may be taken from live related donors and close family, members may have less HLA mismatch than members of the general population. Laparoscopic donor nephrectomy further minimises the operative morbidity for the donor. Other organs are typically taken from brain dead or dying patients who have a cardiac arrest and in whom resuscitation is futile. The key event is to minimise the warm ischaemic time in the donor phase.
The kidney once removed is usually prepared on the bench in theatre by the transplant surgeon immediately prior to implantation and factors such as accessory renal arteries and vessel length are assessed and managed.
For first time recipients the operation is performed under general anaesthesia. A Rutherford-Morison incision is made on the preferred side. This provides excellent extraperitoneal access to the iliac vessels. The external iliac artery and vein are dissected out and following systemic heparinisation are cross clamped. The vein and artery are anastamosed to the iliacs and the clamps removed. The ureter is then implanted into the bladder and a stent is usually placed to maintain patency. The wounds are then closed and the patient recovered from surgery.
In the immediate phase a common problem encountered in cadaveric kidneys is acute tubular necrosis and this tends to resolve.
Graft survival times from cadaveric donors are typically of the order of 9 years and monozygotic twin transplant (live donor) may survive as long as 25 years.
A 22 year old woman presents with macroscopic haematuria. She is sexually active. She is known to have renal calculi and had a berry aneurysm clipped. What is the most likely cause?
Interstitial nephritis
Membranous glomerulonephritis
Renal vein thrombosis
Endometriosis
Adult polycystic kidney disease
APKD is associated with liver cysts (70%), berry aneurysms (25%) and pancreatic cysts (10%). Patients may have a renal mass, hypertension, renal calculi and macroscopic haematuria.
Haematuria
Causes of haematuria
Trauma
Injury to renal tract
Renal trauma commonly due to blunt injury (others penetrating injuries)
Ureter trauma rare: iatrogenic
Bladder trauma: due to RTA or pelvic fractures
Infection
Remember TB
Malignancy
Renal cell carcinoma (remember paraneoplastic syndromes): painful or painless
Urothelial malignancies: 90% are transitional cell carcinoma, can occur anywhere along the urinary tract. Painless haematuria.
Squamous cell carcinoma and adenocarcinoma: rare bladder tumours
Prostate cancer
Penile cancers: SCC
Renal disease
Glomerulonephritis
Stones
Microscopic haematuria common
Structural abnormalities
Benign prostatic hyperplasia (BPH) causes haematuria due to hypervascularity of the prostate gland
Cystic renal lesions e.g. polycystic kidney disease
Vascular malformations
Renal vein thrombosis due to renal cell carcinoma
Coagulopathy
Causes bleeding of underlying lesions
Drugs
Cause tubular necrosis or interstitial nephritis: aminoglycosides, chemotherapy
Interstitial nephritis: penicillin, sulphonamides, and NSAIDs
Anticoagulants
Benign
Exercise
Gynaecological
Endometriosis: flank pain, dysuria, and haematuria that is cyclical
Iatrogenic
Catheterisation
Radiotherapy; cystitis, severe haemorrhage, bladder necrosis
Pseudohaematuria For example following consumption of beetroot
References
Http://bestpractice.bmj.com/best-practice/monograph/316/overview/aetiology.html
Which of the following nerves conveys sensory information from the laryngeal mucosa?
Glossopharyngeal
Laryngeal branches of the vagus
Ansa cervicalis
Laryngeal branches of the trigeminal
None of the above
The laryngeal branches of the vagus supply sensory information from the larynx.
Larynx
The larynx lies in the anterior part of the neck at the levels of C3 to C6 vertebral bodies. The laryngeal skeleton consists of a number of cartilagenous segments. Three of these are paired; arytenoid, corniculate and cuneiform. Three are single; thyroid, cricoid and epiglottic. The cricoid cartilage forms a complete ring (the only one to do so).
The laryngeal cavity extends from the laryngeal inlet to the level of the inferior border of the cricoid cartilage.
Divisions of the laryngeal cavity
Laryngeal vestibule Superior to the vestibular folds
Laryngeal ventricle Lies between vestibular folds and superior to the vocal cords
Infraglottic cavity Extends from vocal cords to inferior border of the cricoid cartilage
The vocal folds (true vocal cords) control sound production. The apex of each fold projects medially into the laryngeal cavity. Each vocal fold includes:
Vocal ligament
Vocalis muscle (most medial part of thyroarytenoid muscle)
The glottis is composed of the vocal folds, processes and rima glottidis. The rima glottidis is the narrowest potential site within the larynx, as the vocal cords may be completely opposed, forming a complete barrier.
Muscles of the larynx
Muscle Origin Insertion Innervation Action
Posterior cricoarytenoid Posterior aspect of lamina of cricoid Muscular process of arytenoid Recurrent Laryngeal Abducts vocal fold
Lateral cricoarytenoid Arch of cricoid Muscular process of arytenoid Recurrent laryngeal Adducts vocal fold
Thyroarytenoid Posterior aspect of thyroid cartilage Muscular process of arytenoid Recurrent laryngeal Relaxes vocal fold
Transverse and oblique arytenoids Arytenoid cartilage Contralateral arytenoid Recurrent laryngeal Closure of intercartilagenous part of the rima glottidis
Vocalis Depression between lamina of thyroid cartilage Vocal ligament and vocal process of arytenoid cartilage Recurrent laryngeal Relaxes posterior vocal ligament, tenses anterior part
Cricothyroid Anterolateral part of cricoid Inferior margin and horn of thyroid cartilage External laryngeal Tenses vocal fold
Blood supply
Arterial supply is via the laryngeal arteries, branches of the superior and inferior thyroid arteries. The superior laryngeal artery is closely related to the internal laryngeal nerve. The inferior laryngeal artery is related to the inferior laryngeal nerve. Venous drainage is via superior and inferior laryngeal veins, the former draining into the superior thyroid vein and the latter draining into the middle thyroid vein, or thyroid venous plexus.
Lymphatic drainage
The vocal cords have no lymphatic drainage and this site acts as a lymphatic watershed.
Supraglottic part Upper deep cervical nodes
Subglottic part Prelaryngeal and pretracheal nodes and inferior deep cervical nodes
The aryepiglottic fold and vestibular folds have a dense plexus of lymphatics associated with them and malignancies at these sites have a greater propensity for nodal metastasis.
Which of the metastatic bone tumours described below is at the greatest risk of pathological fracture ?
Proximal humeral lesion from a prostate cancer
Vertebral body lesions from a prostate cancer
Peritrochanteric lesion from a carcinoma of the breast
Proximal humeral lesion from a carcinoma of the breast
Peritrochanteric lesion from a prostate cancer
Peritrochanteric lesions have the greatest risks of fracture (due to loading). The lesions from breast cancer are usually lytic and therefore at higher risk rather than the sclerotic lesions from prostate cancer.
Metastatic bone disease- risk of fracture
Metastatic bone tumours may be described as blastic, lytic or mixed. Osteoblastic metastatic disease has the lowest risk of spontaneous fracture when compared to osteolytic lesions of a similar size.
Lesions affecting the peritrochanteric region are most prone to spontaneous fracture (because of loading forces at that site).
The factors are incorporated into the Mirel Scoring system to stratify the risk of spontaneous fracture for bone metastasis of varying types.
Mirel Scoring system
Score points Site Radiographic appearance Width of bone involved Pain
1 Upper extremity Blastic Less than 1/3 Mild
2 Lower extremity Mixed 1/3 to 2/3 Moderate
3 Peritrochanteric Lytic More than 2/3 Aggravated by function
Depending upon the score the treatment should be as follows:
Score Risk of fracture Treatment
9 or greater Impending (33%) Prophylactic fixation
8 Borderline Consider fixation
7 or less Not impending (4%) Non operative management
Which of the following is not a direct branch of the facial nerve?
Greater petrosal nerve
Nerve to stapedius
Auriculotemporal
Chorda tympani
Buccal branch of facial nerve
The auriculotemporal nerve is a direct branch of the mandibular nerve.
Other branches of the mandibular nerve include:
Lingual
Inferior alveolar
Nerve to the mylohyoid
Mental
Facial nerve
The facial nerve is the main nerve supplying the structures of the second embryonic branchial arch. It is predominantly an efferent nerve to the muscles of facial expression, digastric muscle and also to many glandular structures. It contains a few afferent fibres which originate in the cells of its genicular ganglion and are concerned with taste.
Supply - ‘face, ear, taste, tear’
Face: muscles of facial expression
Ear: nerve to stapedius
Taste: supplies anterior two-thirds of tongue
Tear: parasympathetic fibres to lacrimal glands, also salivary glands
Path
Subarachnoid path
Origin: motor- pons, sensory- nervus intermedius
Pass through the petrous temporal bone into the internal auditory meatus with the vestibulocochlear nerve. Here they combine to become the facial nerve.
Facial canal path
The canal passes superior to the vestibule of the inner ear
At the medial aspect of the middle ear, it becomes wider and contains the geniculate ganglion.
- 3 branches:
1. greater petrosal nerve
2. nerve to stapedius
3. chorda tympani
Stylomastoid foramen
Passes through the stylomastoid foramen (tympanic cavity anterior and mastoid antrum posteriorly)
Posterior auricular nerve and branch to posterior belly of digastric and stylohyoid muscle
Face
Enters parotid gland and divides into 5 branches:
Temporal branch
Zygomatic branch
Buccal branch
Marginal mandibular branch
Cervical branch
Which of the following statements relating to the tympanic membrane is false?
The umbo marks the point of attachment of the handle of the malleus to the tympanic membrane
The lateral aspect of the tympanic membrane is lined by stratified squamous epithelium
The chorda tympani nerve runs medial to the pars tensa
The medial aspect of the tympanic membrane is lined by mucous membrane
The tympanic membrane is approximately 1cm in diameter
The chorda tympani runs medially to the pars flaccida. Ear- anatomy
The ear is composed of three anatomically distinct regions.
External ear
Auricle is composed of elastic cartilage covered by skin. The lobule has no cartilage and contains fat and fibrous tissue.
External auditory meatus is approximately 2.5cm long.
Lateral third of the external auditory meatus is cartilaginous and the medial two thirds is bony.
The region is innervated by the greater auricular nerve. The auriculotemporal branch of the trigeminal nerve supplies most of the external auditory meatus and the lateral surface of the auricle.
Middle ear
Space between the tympanic membrane and cochlea. The aditus leads to the mastoid air cells is the route through which middle ear infections may cause mastoiditis. Anteriorly the eustacian tube connects the middle ear to the naso pharynx.
The tympanic membrane consists of:
Outer layer of stratified squamous epithelium.
Middle layer of fibrous tissue.
Inner layer of mucous membrane continuous with the middle ear.
The tympanic membrane is approximately 1cm in diameter.
The chorda tympani nerve passes on the medial side of the pars flaccida.
The middle ear is innervated by the glossopharyngeal nerve and pain may radiate to the middle ear following tonsillectomy.
Ossicles
Malleus attaches to the tympanic membrane (the Umbo).
Malleus articulates with the incus (synovial joint).
Incus attaches to stapes (another synovial joint).
Internal ear
Cochlea, semi circular canals and vestibule
Organ of corti is the sense organ of hearing and is located on the inside of the cochlear duct on the basilar membrane.
Vestibule accommodates the utricule and the saccule. These structures contain endolymph and are surrounded by perilymph within the vestibule.
The semicircular canals lie at various angles to the petrous temporal bone. All share a common opening into the vestibule.
A 22 year old lady has a long history of severe perianal Crohns disease with multiple fistulae. She is keen to avoid a stoma. However, she has progressive disease and multiple episodes of rectal bleeding. A colonoscopy shows rectal disease only and a small bowel study shows no involvement with Crohns. What is the best operative strategy?
Abdomino perineal excision of the colon and rectum
Proctectomy and end stoma
Pan proctocolectomy and ileoanal pouch
Loop colostomy alone
Sub total colectomy
Crohns disease is a contra indication to having an ileo-anal pouch as its associated with very poor pouch function and significant complications.
Whilst the patient wishes to avoid a stoma, that’s inevitable here.
Surgery for inflammatory bowel disease
Patients with inflammatory bowel disease (UC and Crohns) frequently present in surgical practice. Ulcerative colitis may be cured by surgical resection (Proctocolectomy), this is not the case in Crohns disease which may recur and affect other areas of the gastrointestinal tract.
Ulcerative colitis
Elective indications for surgery include disease that is requiring maximal therapy, or prolonged courses of steroids.
Longstanding UC is associated with a risk of malignant transformation. Dysplastic transformation of the colonic epithelium with associated mass lesions is an absolute indication for a proctocolectomy.
Emergency presentations of poorly controlled colitis that fails to respond to medical therapy should usually be managed with a sub total colectomy. Excision of the rectum is a procedure with a higher morbidity and is not generally performed in the emergency setting. An end ileostomy is usually created and the rectum either stapled off and left in situ, or, if the bowel is very oedematous, may be brought to the surface as a mucous fistula.
Patients with IBD have a high incidence of DVT and appropriate thromboprophylaxis is mandatory.
Restorative options in UC include an ileoanal pouch. This procedure can only be performed whilst the rectum is in situ and cannot usually be undertaken as a delayed procedure following proctectomy.
Ileoanal pouch complications include, anastomotic dehiscence, pouchitis and poor physiological function with seepage and soiling.
Crohns disease
Surgical resection of Crohns disease does not equate with cure, but may produce substantial symptomatic improvement.
Indications for surgery include complications such as fistulae, abscess formation and strictures.
Extensive small bowel resections may result in short bowel syndrome and localised stricturoplasty may allow preservation of intestinal length.
Staging of Crohns will usually involve colonoscopy and a small bowel study (e.g. MRI enteroclysis).
Complex perianal fistulae are best managed with long term draining seton sutures, complex attempts at fistula closure e.g. advancement flaps, may be complicated by non healing and fistula recurrence.
Severe perianal and / or rectal Crohns may require proctectomy. Ileoanal pouch reconstruction in Crohns carries a high risk of fistula formation and pouch failure and is not recommended.
Terminal ileal Crohns remains the commonest disease site and these patients may be treated with limited ileocaecal resections.
Terminal ileal Crohns may affect enterohepatic bile salt recycling and increase the risk of gallstones.
A 58 year old man undergoes a difficult colonoscopy for assessment of a caecal cancer. 48 hours after the procedure he is admitted with septicaemia. His abdomen is soft and non tender. Blood cultures grow gram positive cocci. What is the most likely underlying organism?
Staphylococcus epidermidis
Streptococcus pyogenes
Streptococcus bovis
Clostridium difficle
Bacteroides fragilis
Streptococcus bovis septicaemia is associated with carcinoma of the colon. It can also cause endocarditis. Streptococcus bovis is a gram positive organism. In most cases, its portal of entry is via the GI tract and for that reason its typically associated with large bowel malignancy.
Surgical Microbiology
An extensive topic so an overview is given here. Organisms causing common surgical infections are reasonable topics in the examination. However, microbiology is less rigorously tested than anatomy, for example.
Common organisms
Staphylococcus aureus
Facultative anaerobe
Gram positive coccus
Haemolysis on blood agar plates
Catalase positive
20% population are long term carriers
Exo and entero toxin may result in toxic shock syndrome and gastroenteritis respectively
Ideally treated with penicillin although many strains now resistant through beta Lactamase production. In the UK less than 5% of isolates are sensitive to penicillin.
Resistance to methicillin (and other antibiotics) is mediated by the mec operon , essentially penicillin binding protein is altered and resistance to this class of antibiotics ensues
Common cause of cutaneous infections and abscesses
Streptococcus pyogenes
Gram positive, forms chain like colonies, Lancefield Group A Streptococcus
Produces beta haemolysis on blood agar plates
Rarely part of normal skin microflora
Catalase negative
Releases a number of proteins/ virulence factors into host including hyaluronidase, streptokinase which allow rapid tissue destruction
Releases superantigens such as pyogenic exotoxin A which results in scarlet fever
Remains sensitive to penicillin, macrolides may be used as an alternative.
Escherichia coli
Gram negative rod
Facultative anaerobe, non sporing
Wide range of subtypes and some are normal gut commensals
Some subtypes such as 0157 may produce lethal toxins resulting in haemolytic-uraemic syndrome
Enterotoxigenic E-Coli produces an enterotoxin (ST enterotoxin) that results in large volume fluid secretion into the gut lumen (Via cGMP activation)
Enteropathogenic E-Coli binds to intestinal cells and cause structural damage, this coupled with a moderate (or in case of enteroinvasive E-Coli significant) invasive component produces enteritis and large volume diarrhoea together with fever.
They are resistant to many antibiotics used to treat gram positive infections and acquire resistance rapidly and are recognised as producing beta lactamases
Campylobacter jejuni
Curved, gram negative, non sporulating bacteria
One of the commonest causes of diarrhoea worldwide
Produces enteritis which is often diffuse and blood may be passed
Remains a differential for right iliac fossa pain with diarrhoea
Self limiting infection so antibiotics are not usually advised. However, the quinolones are often rapidly effective.
Helicobacter pylori
Gram negative, helix shaped rod, microaerophillic
Produces hydrogenase that can derive energy from hydrogen released by intestinal bacteria
Flagellated and mobile
Those carrying the cag A gene may cause ulcers
It secretes urease that breaks down gastric urea> Carbon dioxide and ammonia> ammonium>bicarbonate (simplified!) The bicarbonate can neutralise the gastric acid.
Usually colonises the gastric antrum and irritates resulting in increased gastrin release and higher levels of gastric acid. These patients will develop duodenal ulcers. In those with more diffuse H-Pylori infection gastric acid levels are lower and ulcers develop by local tissue damage from H-Pylori- these patients get gastric ulcers.
Diagnosis may be made by serology (approx. 75% sensitive). Biopsy urease test during endoscopy probably the most sensitive.
In patients who are colonised 10-20% risk of peptic ulcer, 1-2% risk gastric cancer, <1% risk MALT lymphoma.
Which of these substances is not released from the islets of Langerhans?
Pancreatic polypeptide
Glucagon
Secretin
Somatostatin
Insulin
Secretin is released from mucosal cells in the duodenum and jejunum.
Pancreas endocrine physiology
Hormones released from the islets of Langerhans
Beta cells Insulin (70% of total secretions)
Alpha cells Glucagon
Delta cells Somatostatin
F cells Pancreatic polypeptide